DISCLAIMER
The material in this PDF is provided solely for educational and revision purposes. While every effort has been made to
reproduce past examination questions accurately or to outline their core concepts and answers, this document is not an official
publication of the Egyptian Board of Diagnostic Radiology (EBDR), nor is it endorsed, reviewed, or approved by the EBDR or
any affiliated authority.
1. Accuracy & Evolving Medical Standards
The compilers have taken reasonable care to ensure the content is correct; however, no warranty, express or implied, is
given regarding the accuracy, completeness, or currency of any question, hint, answer, or explanation contained herein. Medical
imaging guidelines, protocols, and diagnostic criteria evolve continuously. Users must verify all information with:
• Current official EBDR materials and recent examination guidelines
• Latest editions of authoritative radiology textbooks
• Updated professional society guidelines (ACR, ESR, RSNA, etc.)
• Most recent TNM staging manuals (8th edition or newer)
• Peer-reviewed medical literature and current clinical practice standards
2. Specific Areas Requiring Vigilance
Users should be particularly cautious regarding:
• Breast cancer screening recommendations and BI-RADS classifications
• TNM staging criteria (some questions may reference older editions)
• Interventional radiology indications and contraindications
• Radiation dose considerations and imaging protocol selections
• Emerging imaging techniques and evolving diagnostic criteria
3. AI-Generated & Reconstructed Items
A significant portion of the practice questions were generated by artificial intelligence or recreated from partial
recollections of past exams shared by colleagues. These AI-generated items may contain factual errors, outdated
information, or incorrect medical concepts despite efforts to mirror the format and cognitive level typical of professional
radiological examinations. AI-generated content should never be considered as authoritative medical information and must
be independently verified. Users should be particularly cautious with AI-generated explanations and always cross-reference with
established medical literature.
4. Educational Use Only
This material is intended purely for educational practice and should not be used as a substitute for:
• Official EBDR study materials or guidelines
• Peer-reviewed medical literature
• Formal medical education or training
• Clinical decision-making or patient care
• Professional medical advice or consultation
5. Potential for Misinformation
Users are specifically warned that:
• Some content may be outdated or superseded by current medical practice
• AI-generated questions may contain subtle but significant medical inaccuracies
• Reconstructed questions may not accurately reflect official examination standards
• Explanations may contain oversimplifications or errors
• This material should never be used as a reference for clinical practice
6. Copyright & Fair Use
Portions of past examination questions may be reproduced under fair-use provisions for academic study. All copyrights and
trademarks remain the property of their respective owners. Redistribution, commercial use, or reproduction of this PDF
without permission from the original rights-holders is strictly prohibited.
7. No Professional or Examination Guarantee
This document does not guarantee examination success, confer any professional qualification, or substitute for accredited
training, official preparation courses, or comprehensive study programs. Success in professional examinations requires
systematic study of official materials, clinical experience, and comprehensive understanding of radiological principles. Reliance
on this material alone is at the user's own risk.
8. Recommendation for Official Sources
Users are strongly advised to:
• Prioritize official EBDR materials and guidelines
• Consult peer-reviewed radiological literature
• Seek guidance from qualified radiological professionals
• Participate in accredited continuing medical education programs
• Use this material only as supplementary practice, not primary study resource
9. Liability Limitation
To the fullest extent permitted by law, the authors, editors, contributors, and distributors disclaim all liability for any loss,
damage, educational setback, examination failure, or any other consequence arising from use, misuse, or reliance on the
content, including but not limited to errors, omissions, AI-generated inaccuracies, or outdated information. Users assume full
responsibility for verifying all content independently.
10. Feedback & Corrections
If you identify any inaccuracies, errors in AI-generated content, outdated information, or have suggestions for improvement,
please contact the compiler via: https://linktr.ee/Kareem_Alnakeeb. However, users should not wait for corrections and must
independently verify all information.
11. Acknowledgment
We gratefully acknowledge the many colleagues and fellow radiologists who generously contributed past
exam materials and insights used in compiling this document.
Part 2 Examination — Structure & Weighting
1. Key Facts
• Two written papers taken on consecutive days (Day 1 & Day 2)
• Duration: 2 hours per paper
• Marks: 100 marks per paper (200 marks total)
• Coverage: 8 clinical modules, 4 assessed in each paper
2. Day 1 — Paper I (2 hours, 100 marks)
Module Marks
Cardiothoracic & Vascular
• Including Chest imaging (25 marks)
45
Neuroimaging
• Central nervous system
• Head & Neck
• Spine
25
Pediatric 20
Interventional 10
3. Day 2 — Paper II (2 hours, 100 marks)
Module Marks
Musculoskeletal (MSK) & Trauma 33–35
Gastro-intestinal Tract (GIT) 30
Genito-urinary Tract (GUT):
• Urinary system
• Adrenal glands
• Male
• Female (Obstetrics, Gynecology & Breast)
30
Nuclear Medicine 5
Table Of Contents
June 2025 .............................................................................................................................. 1
October 2024....................................................................................................................... 62
July 2024 ............................................................................................................................. 92
Paper 1..................................................................................................................................92
Paper 2................................................................................................................................116
May 2023............................................................................................................................144
September 2022 .................................................................................................................197
May 2022............................................................................................................................261
October 2021......................................................................................................................290
Paper 1................................................................................................................................290
Paper 2................................................................................................................................329
May 2021............................................................................................................................370
Paper 1................................................................................................................................370
Paper 2................................................................................................................................394
September 2020 .................................................................................................................475
Paper 1................................................................................................................................475
Paper 2................................................................................................................................508
September 2019 .................................................................................................................544
Paper 2................................................................................................................................606
April 2019...........................................................................................................................633
Paper 1................................................................................................................................633
Paper 2................................................................................................................................655
September 2017 .................................................................................................................696
April 2017...........................................................................................................................717
Paper 1................................................................................................................................717
Paper 2................................................................................................................................802
October 2015......................................................................................................................846
Paper 2................................................................................................................................846
EBDR Exam MCQs & Concepts June 2025
Dr. Kareem Alnakeeb 1
June 2025
1. A prominent alanine peak on MR spectroscopy of an intracranial mass, suggests that the mass is most
likely
A. Cerebral metastasis
B. Glioblastoma
C. Hemangiopericytoma
D. Meningioma
E. Oligodendroglioma
Source: Radiopaedia
2. A 69-year-old woman presented to her GP with one month’s history of headache and progressive swelling
of both hands and face and an 8 months’ history of progressive weight loss. Chest radiograph showed
widening of the mediastinum and a large mass in the right upper lobe with a midline trachea. What do
you expect the CT chest to show?
A. Cavitating lung primary in RUL without mediastinal nodes
B. Right Pancoast tumour with rib destruction
C. RUL large and multiple small nodules of Wegener’s granulomatosis
D. RUL lung primary with retrosternal goitre
E. RUL lung primary and SVC obstruction
Source: Chin, Teck Yew, et al. Get Through Final FRCR 2A: SBAs. 1st ed., CRC Press, 2017.
Explanation:
• Superior Vena Cava (SVC) Obstruction
• Pathophysiology
o Obstruction of the superior vena cava results in impaired venous drainage of the head and
neck and upper extremities.
• Clinical Manifestations
o Clinical manifestations include facial and neck swelling, distended neck veins, headache
due to cerebral oedema, dyspnoea, stridor and altered mental status.
• Etiology
o Cancer is the most common underlying cause of superior vena cava obstruction and this
includes lung cancer, mediastinal tumours, lymphoma/lymphadenopathy and
mesothelioma, either directly or through malignant mediastinal lymphadenopathy.
o Other causes include catheter-induced iatrogenic SVC obstruction, fibrosing mediastinitis
and Behcet’s disease.
3. Which of the following statements is true of Moyamoya vasculopathy?
A. It cannot present with parenchymal haemorrhage in the adult population
B. Moyamoya disease can be seen in patients with sickle cell disease, prior radiotherapy and
neurofibromatosis.
C. Moyamoya syndrome is commonly considered idiopathic but has a well-recognised familial pattern.
D. The most common presentation is a major territory infarction
Source: Radiopaedia
Explanation:
• Moyamoya disease is considered idiopathic and can have an underlying genetic predisposition.
• Moyamoya syndrome is associated with other underlying conditions such as sickle cell disease,
neurofibromatosis or collagen disorders.
• The posterior circulation is commonly involved although it is true that the terminal internal carotid
arteries are more commonly affected.
EBDR Exam MCQs & Concepts June 2025
Dr. Kareem Alnakeeb 2
• The most common presentation is that of watershed infarction either in the anterior, posterior or
parasagittal distributions.
• Conventional arterial territory infarcts are relatively uncommon due to the diffuse vasculopathy.
4. A 74-year-old man with increased urinary frequency and hesitancy is found to have an enlarged prostate
on digital rectal examination. He is referred for a TRUS and biopsy. Which one of the following statements
best describes the TRUS findings of benign prostatic hypertrophy (BPH)?
A. Dense echogenic foci are seen at the margin of the peripheral and transitional zones.
B. The central zone is enlarged.
C. The peripheral zone is enlarged and appears homogeneously hypoechoic.
D. The peripheral zone is enlarged and is of mixed echogenicity.
E. The transitional zone is enlarged.
Source: McQueen, Andrew S., et al. Grainger & Allison’s Diagnostic Radiology: Single Best Answer
MCQs. 5th ed., Churchill Livingstone, 2009.
5. The central zone atrophies with age while the transitional zone increases in size as it develops BPH.
Peripheral zone enlargement is not a feature of BPH. Eight days after lung transplantation for alpha-1
antitrypsin deficiency, a 45 year old man develops pyrexia, breathlessness and desaturation. HRCT
reveals perihilar heterogenous opacities and ground glass changes with new pleural effusion and septal
thickening. Which of the following is the most likely cause?
A. Reperfusion oedema
B. Acute rejection
C. Anastomotic dehiscence
D. Post-transplantation PCP infection
E. Hyperacute rejection
Source: Currie, Stuart, et al. SBAs for the FRCR 2A. 1st ed., Cambridge University Press, 2010.
Explanation:
• Hyperacute rejection presents within hours of the transplantation.
• Reperfusion oedema usually presents within 24 hours of the transplantation, peaking by about day
four.
• Posttransplant infections can be broadly divided into those occurring within the first month (gram-
negative bacteria, fungi (candida, aspergillosis)) and those occurring after the first month (CMV,
PCP).
• Anastomotic dehiscence is usually an early feature, but the presentation and features are not those
described.
6. A 35-year-old patient received a cadaveric renal transplant 5 days ago and now presents with worsening
renal function and decreasing urine output. Which one of the following findings on a Tc-99m DTPA
radionuclide scan would favor a diagnosis of acute tubular necrosis (ATN) over acute rejection?
A. Delayed renal excretion
B. Elevated resistive index greater than 0.7
C. Increased renal perfusion after administration of an ACEI (eg Captopril)
D. Poor/impaired graft perfusion
E. Preserved renal transplant perfusion
Source: McQueen, Andrew S., et al. Grainger & Allison’s Diagnostic Radiology: Single Best Answer
MCQs. 5th ed., Churchill Livingstone, 2009.
Explanation:
• ATN is an early complication in cadaveric allografts and frequently resolves spontaneously in 1—3
weeks. The radionuclide imaging findings of ATN are of preserved perfusion but poor renal function
and urine excretion.
EBDR Exam MCQs & Concepts June 2025
Dr. Kareem Alnakeeb 3
• In acute rejection however, there is both impaired renal function and reduced perfusion on
radionuclide imaging.
7. On the unenhanced CT, right adrenal mass is detected and appears homogeneous and has an average
density of 7 HU. What is the MOST likely diagnosis?
A. Adrenal adenoma
B. Adrenal hyperplasia
C. Adrenal metastasis
D. Focal adrenal hemorrhage
Source: McQueen, Andrew S., et al. Grainger & Allison’s Diagnostic Radiology: Single Best Answer MCQs.
5th ed., Churchill Livingstone, 2009.
Explanation:
• The 10-HU threshold is now the standard by which radiologists differentiate lipid-rich adenomas
from most other adrenal lesions on unenhanced CT.
• The presence of substantial amounts of intracellular fat is critical in malting the specific diagnosis of
adenoma. Up to 30% of adenomas, however, do not have abundant intracellular fat and, thus, show
attenuation values greater than 10 HU on unenhanced CT.
• Lesions above 10 HU on an unenhanced CT are considered indeterminate and other investigations
may be required.
8. A 29-year-old man has an IVU performed following an episode of haematuria. This demonstrates
complete right-sided ureteric duplication. Which one of the following statements is true?
A. If present, an ectopic ureterocoele is usually related to the lower moiety
B. The lower moiety ureter usually obstructs at the vesicoureteric junction.
C. The upper moiety calyces are prone to vesicoureteric reflux.
D. The upper moiety ureter is prone to ureteric obstruction.
E. The upper moiety ureter usually inserts into the bladder superior to the lower moiety ureter.
Source: McQueen, Andrew S., et al. Grainger & Allison’s Diagnostic Radiology: Single Best Answer
MCQs. 5th ed., Churchill Livingstone, 2009.
9. A 24-year-old motorcyclist is involved in a high-speed accident and is brought to the Emergency
Department. He has abdominal guarding and is hemodynamically unstable. An ultrasound abdomen
performed in the Emergency Department demonstrates free peritoneal fluid and a laparotomy is
performed. In addition to liver and splenic lacerations, the surgeon finds a left retroperitoneal hematoma.
Postoperatively, the on-call urologist requests a CT abdomen to assess the left renal injury. Which one of
the following findings would indicate a Grade 4 renal laceration?
A. Extravasation of contrast from the pelvicalyceal system on delayed phase (5 min) images
B. Large (2-cm) subcapsular hematoma
C. Perinephric hematoma that extends into the pararenal spaces
D. Ill-defined low attenuation change in the lower pole renal cortex
E. Segmental renal infarction
Source: McQueen, Andrew S., et al. Grainger & Allison’s Diagnostic Radiology: Single Best Answer
MCQs. 5th ed., Churchill Livingstone, 2009.
Explanation:
• A deep renal laceration that extends into the collecting system is indicative of a grade 4 injury.
EBDR Exam MCQs & Concepts June 2025
Dr. Kareem Alnakeeb 4
10. A 60-year-old nulliparous woman presents with postmenopausal bleeding. On transvaginal ultrasound,
her endometrium is 8 mm thick and the endomyometrial junction appeared indistinct. The radiologist
suspects invasive endometrial cancer and refers her for an MRI examination. What are the likely findings
on MRI?
A. On unenhanced Tlw images the endometrial cancer appears of high signal intensity compared to the
surrounding myometrium.
B. On contrast-enhanced Tlw images, endometrial cancer shows avid enhancement compared with
surrounding myometrium.
C. On T2w images the normally high signal junctional zone is disrupted.
D. Tlw fat-saturated sequences are best used to assess the junctional zone.
E. The endometrial cancer demonstrates delayed/little enhancement compared to the normal
surrounding myometrium on postcontrast Tlw images.
Source: McQueen, Andrew S., et al. Grainger & Allison’s Diagnostic Radiology: Single Best Answer
MCQs. 5th ed., Churchill Livingstone, 2009.
11. Regarding eosinophilic granuloma: (True or False)
A. Lesions in proximal long bones are usually diaphyseal
B. The commonest site is the skull
C. On MRI, it appears as a well defined lesion of low signal intensity on T1
D. Lesions rarely elicit a periosteal reaction
E. It is a recognised cause of ‘floating teeth’ appearance
Source: Hussain, Shahid, et al. Radiology MCQs for the New FRCR Part 2A. 1st ed., TFM Publishing,
2006.
Explanation:
a) False - 0.01% of all trauma patients are affected
b) True - patchy osteopaenia is seen in 50% as early as 2-3 weeks after symptom onset
c) True
d) True
e) False - this is an end-stage feature
12. Which feature would be most helpful to distinguish chordoma from chondrosarcoma at the clivus?
A. bone destruction
B. enhancement like a pumice stone
C. Midline location
D. T2 hyperintense signal
Source: Radiopaedia
Explanation:
• Chordoma is a midline tumor because it arises from the notochordal remnant.
• Chondrosarcoma arises at the petro-occipital synchondrosis and therefore is centered off the
midline.
• Both chordoma and chondrosarcoma cause bone destruction and are T2 hyperintense.
• Chondrosarcoma enhances like a pumice stone, whereas chordoma often has a more honeycomb
enhancement pattern.
EBDR Exam MCQs & Concepts June 2025
Dr. Kareem Alnakeeb 5
13. A 35-year-old woman with loin pain, dysuria and fever is diagnosed with acute uncomplicated ascending
bacterial pyelonephritis. What appearance is likely to have been seen on CT?
A. Cortical thinning
B. Perinephric fat stranding
C. Alternating bands of hypo- and hyperattenuation of the renal parenchyma
D. Round peripheral hypoattenuating renal lesions
E. Geographic low-attenuation lesion with peripheral enhancement
Source: Proctor, Robin. Final FRCR Part A Modules 4-6 Single Best Answer MCQs: The SRT Collection
of 600 Questions with Explanatory Answers (MasterPass). 1st ed., CRC Press, 2009.
Explanation:
• This appearance is analogous to the striated nephrogram seen on excretory urography and reflects
the underlying pathology of tubular obstruction, interstitial oedema and vasospasm.
• On delayed (3-6 hours) imaging, the hypoattenuating regions show delayed and persistent
enhancement due to prolonged accumulation and transit of contrast through the collecting system.
• In haematogenous seeding of pyelonephritis (from staphylococcal or streptococcal infections),
round peripheral hypoattenuating lesions are seen. They can mimic neoplasia if pyelonephritis is not
suspected clinically.
14. A 30-year-old woman presents acutely with seizures, fever and headache, followed by rapid deterioration
to coma. Emergency MRI shows asymmetrical swelling of the anterior temporal lobes on T1W images.
T2W images reveal concordant asymmetrical but bilateral areas of high signal in the anterior temporal
lobes, insular cortices and hippocampi. There is no enhancement following administration of intravenous
gadolinium. What is the most likely condition?
A. lymphoma
B. HIV encephalitis
C. cytomegalovirus encephalitis
D. herpes simplex encephalitis
E. toxoplasmosis
Source: Bydder, Megan, et al. Get Through Final FRCR Part A: SBAs for the Modular Examination. 1st
ed., Hodder Education Publishers, 2009.
Explanation:
Epidemiology and Etiology
• Herpes simplex virus is the most common cause of fatal endemic encephalitis, often leaving
survivors with severe memory and personality problems.
• Both oral (type 1) and genital (type 2) strains may produce encephalitis with a multimodal
distribution, affecting neonates (due to cross-infection with type 2 from the mother during birth),
children and adults.
• Clinical Presentation
• Childhood and adult infection is caused by the type 1 virus and results in fulminant necrotizing
encephalitis presenting with acute confusion and deteriorating rapidly to coma.
• Focal neurological deficits are seen in only 30% of cases.
Imaging Characteristics
• The virus asymmetrically affects the temporal lobes, insula, orbitofrontal region and cingulate gyrus,
causing oedema.
• This is seen as high signal on T2W/FLAIR images, with DWI appearances variable depending on the
presence of infarction.
• The putamen is characteristically spared, and the areas of encephalitis typically do not show
enhancement on CT or MRI.
EBDR Exam MCQs & Concepts June 2025
Dr. Kareem Alnakeeb 6
15. A 63 year old male patient is seen in the vascular clinic with a pulsatile mass posterior to his right knee.
Ultrasound confirms a 2-cm right popliteal artery aneurysm. The vascular team request a CT angiogram
to help plan management. Which imaging protocol is most appropriate?
A. Arterial phase CT from aortic bifurcation to toes
B. Arterial phase CT from neck to toes
C. Arterial phase CT from aortic bifurcation to knees
D. Arterial phase CT from diaphragm to toes
E. Arterial phase CT from pubic symphysis to toes
Source: Rabone, Amanda, et al. The Final FRCR Self-Assessment (MasterPass). 1st ed., CRC Press,
2020.
Explanation:
Abdominal Aortic Assessment
• Approximately 30-50% of popliteal artery aneurysms are associated with an abdominal aortic
aneurysm; therefore the abdominal aorta needs to be included on the study.
Contralateral Lower Limb Assessment
• In lower limb studies, the contralateral lower limb will automatically be included on the study and
assessment of the contralateral popliteal artery is also vital, as they can be bilateral.
Vascular Considerations for Interventional Planning
• Other important factors to consider when planning interventional radiological management is the
tortuosity of the common and external iliac arteries and mural calcification, particularly at the
femoral vessels where vascular access will be required for endovascular repair.
Peripheral Vessel Evaluation and CT Extent
• The CT should extend to the toes to assess the vessel quality peripheral to the aneurysm, as popliteal
aneurysms are associated with peripheral thromboembolic complications.
16. A series of neonatal radiographs reveal a narrow thorax with short ribs, square iliac wings with horizontal
acetabular roofs, short sacrosciatic notches, progressive narrowing of the interpedicular distance and
posterior scalloping of the vertebral bodies. What is the most likely diagnosis?
A. Achondroplasia
B. Campomelic dysplasia
C. Cleidocranial dysplasia
D. Ellis-van Creveld syndrome
E. Morquio's syndrome
Source: McQueen, Andrew S., et al. Grainger & Allison’s Diagnostic Radiology: Single Best Answer
MCQs. 5th ed., Churchill Livingstone, 2009.
Explanation:
• The iliac wings in Morquio's syndrome are characteristically flaredTather than
17. A low flat renogram curve indicates:
A. Advanced nephropathy
B. Complete obstruction to urine outflow
C. Vesico-ureteric reflux
D. Partial obstruction to urine outflow
Explanation: (by Perplexity AI, generated with the OpenAI O3 model)
• A very low, almost horizontal time–activity curve reflects markedly reduced tracer extraction by
damaged renal parenchyma, typical of end-stage or advanced medical renal disease.
• Complete or partial outflow obstruction instead produces an uptake phase followed by a rising or
plateau phase; vesico-ureteric reflux alters the post-void segment rather than the primary
renographic curve
EBDR Exam MCQs & Concepts June 2025
Dr. Kareem Alnakeeb 7
18. Chron's disease of small intestine:
A. Terminal ileum is affected in 80% of case.
B. Colon is affected in 10% of case.
C. Treated surgically.
D. Presenting with deep penetrating ulcers.
Explanation: (by Perplexity AI, generated with the OpenAI O3 model)
• The terminal ileum is the most frequent site; up to 80% of patients show involvement here
• Pure colonic (isolated) Crohn’s disease occurs in ≈20% of patients, while another ≈50% have both
ileal and colonic disease.
• Surgery helps manage complications (obstruction, fistula, perforation) but it is not first-line therapy
and cannot cure the disease; most patients start with medical treatment and only 50–70% require an
operation during their lifetime
• Transmural inflammation produces deep linear/penetrating ulcers and fissures, giving the classic
“cobblestone” mucosal pattern
19. Central dot sign is seen in:
A. Caroli disease
B. Primary sclerosing cholangitis
C. Polycystic liver disease
D. Liver hamartoma
Explanation: (by Perplexity AI, generated with the OpenAI O3 model)
• The central dot sign is an enhancing portal venous radicle and fibrovascular bundle seen within
fluid-filled, ectatic intrahepatic bile ducts; it is virtually pathognomonic for Caroli’s disease because
the ducts are massively dilated yet remain in continuity with portal triads.
• Primary sclerosing cholangitis produces multifocal strictures and beading without a central
enhancing dot.
• Simple hepatic cysts are avascular, with thin walls and no intraluminal structures.
• Hepatic mesenchymal hamartoma appears as a multicystic mass lacking patent biliary
connections.
• Cavernous hemangiomas show peripheral nodular enhancement progressing centrally, not a single
central dot.
20. Most Common cause of Urethral Stricture:
A. Infection
B. Trauma
C. Iatrogenic
D. Congenital
Source: Radiopaedia
EBDR Exam MCQs & Concepts June 2025
Dr. Kareem Alnakeeb 8
21. Regarding porcelain gallbladder: (True or False)
A. It is often symptomless
B. It is rarely associated with gallstones
C. Oral cholecystogram shows a non-functioning gallbladder
D. 60-70% develop carcinoma of the gallbladder
E. Acute pancreatitis is a recognised cause
Source: Hussain, Shahid, et al. Radiology MCQs for the New FRCR Part 2A. 1st ed., TFM Publishing,
2006.
Explanation:
• A. True
• B. False - 90%
• C. True
• D. False - 10-20%
• E. False
22. Which of the following statements regarding splenic lymphoma is CORRECT?
A. The spleen is involved at presentation in 30-40% of patients with non Hodgkin's lymphoma
B. Focal splenic deposits are usually well defined, round lesions of increased brightness on ultrasound
C. When there is lymphomatous involvement of the spleen, splenomegaly is seen in 70-80%
D. Splenic lymphoma deposits commonly calcify
E. Lymph nodes are seen in the splenic hilum in 50% of patients with Hodgkin's lymphoma
Source: Hussain, Shahid, et al. Radiology MCQs for the New FRCR Part 2A. 1st ed., TFM Publishing,
2006.
Explanation:
• a) True- slightly higher for Hodgkin's lymphoma
• b) False - reduced echogenicity
• c) False - 50%
• d) True
• e) False – uncommon
22. The following statements concerning esophageal carcinoma are true:
A. 90% of cases are squamous cell carcinomas
B. Most commonly located in the upper third of the esophagus
C. Plummer-Vinson syndrome is a recognised predisposing factor
D. Commonest appearance on double contrast barium swallow is of a large ulcer within a bulging mass
E. It is associated with ulcerative colitis
Source: Hussain, Shahid, et al. Radiology MCQs for the New FRCR Part 2A. 1st ed., TFM Publishing,
2006.
Explanation:
A. True
B. False - 20% in the upper third, 30-40% middle third and 30-40% in lower third
C. True
D. False - polypoid/fungating form is commonest
E. False - predisposing factors include Barrett’s oesophagus, alcohol abuse, smoking, coeliac disease,
achalasia, tylosis
EBDR Exam MCQs & Concepts June 2025
Dr. Kareem Alnakeeb 9
23. A 2-year-old child presents with fever, erythema of the oral mucosa with chest and abdominal pain.
Echocardiography reveals the presence of a coronary arterial aneurysms. An underlying vasculitis is
suspected. Which of the following statements is least accurate in this clinical setting?
A. Aneurysms are typically seen in the proximal segments of the coronary arteries.
B. Aneurysms less than 5 mm in diameter are considered small.
C. Smaller aneurysms have a higher likelihood of thrombosis.
D. Multiple coronary artery aneurysms are more common than isolated aneurysms.
E. The most common site for a coronary aneurysm is in the left anterior descending artery.
Source: Chin, Teck Yew, et al. Get Through Final FRCR 2A: SBAs. 1st ed., CRC Press, 2017.
Explanation:
• Overview
o Kawasaki’s disease is a common paediatric vasculitis of medium-sized vessels, with
coronary vasculitis being the hallmark manifestation.
o It is the leading cause of acquired heart disease in children in developed countries.
• Coronary Arterial Aneurysms
o Coronary arterial aneurysms typically occur within the subacute phase of the disease and
may be associated with sudden cardiac death.
o The aneurysms typically develop in the proximal segments of major coronary arteries and
affect the left anterior descending artery followed by the proximal right coronary arteries in
frequency of location.
o Smaller aneurysms, (<5 mm in diameter) are more likely to regress than larger aneurysms
(>8 mm in diameter), which have a higher likelihood of thrombosis and infarction.
24. A specialty trainee from the medical ward shows you a CXR of a breathless patient. You observe splaying
of the carina and a ’double right heart border’. What is the most likely underlying diagnosis?
A. Mitral stenosis.
B. Aortic stenosis.
C. Tricuspid incompetence.
D. Left ventricular aneurysm.
E. Coarctation of the aorta.
Source: Lindsay, Richard, et al. SBAs for the FRCR Part 2A. 1st ed., Oxford University Press, 2012.
Explanation:
• The findings describe left atrial enlargement, which is caused by mitral valve disease (stenosis or
incompetence), ventricular septal defect (VSD), patent ductus arteriosus (PDA), atrial septal defect
(ASD) with shunt reversal, and left atrial myxoma.
• Aortic stenosis produces left ventricular hypertrophy and eventually dilatation, the latter producing
a prominent left heart border with inferior displacement of the cardiac apex.
• A left ventricular aneurysm produces a prominent bulge of the left heart border.
• Tricuspid incompetence produces an enlarged right atrium and thus a prominent right heart border
on plain film.
• Coarctation produces left ventricular enlargement and inferior rib notching of the fourth to eighth
ribs bilaterally if conventional and a ‘reverse figure 3’ sign: a prominent ascending aorta/arch and a
small descending aorta, with an intervening notch.
EBDR Exam MCQs & Concepts June 2025
Dr. Kareem Alnakeeb 10
25. Regarding hepatocellular carcinoma: (True or False)
A. It is the commonest primary visceral malignancy in the world
B. Haemochromatosis is a recognized cause
C. Elevated alpha-fetoprotein is found in 50-60% of cases
D. Has a higher incidence in macronodular than micronodular cirrhosis
E. On MR, hepatoma has a well defined, hypointense capsule on T1 weighted images
Source: Hussain, Shahid, et al. Radiology MCQs for the New FRCR Part 2A. 1st ed., TFM Publishing,
2006.
Explanation:
• True
• True - other causes are cirrhosis, hepatitis, alpha-1 antitrypsin deficiency, Wilson’s disease, aflatoxin,
and thorotrast.
• c) False - 90%
• d) True
• e) False - increased signal intensity on a T2 weighted image. Peripheral gadolinium enhancement is
seen in about 20%
26. A 42-year-old man presents to the Emergency Department with a 7-day history of severe bloody
diarrhoea and abdominal pain. He has previously been fit and well with no significant medical history. On
examination, the patient is dehydrated with generalized abdominal tenderness but no clinical evidence of
peritonism. An abdominal radiograph is performed. Which radiographic finding would be most suggestive
of a toxic megacolon?
A. Caecum measuring 4.5 cm in diameter
B. Multiple mucosal islands in a dilated transverse colon
C. Pseudodiverticulae in the descending colon
D. Thickened haustrae throughout the entire colon
E. ‘Thumbprinting’ of the transverse and descending colon
Source: McQueen, Andrew S., et al. Grainger & Allison’s Diagnostic Radiology: Single Best Answer
MCQs. 5th ed., Churchill Livingstone, 2009.
Explanation:
• The presence of severe ulceration leading to mucosal islands is a major sign of toxic megacolon (the
other key finding is colonic dilatation > 5 cm).
27. Regarding superior mesenteric artery (SMA) syndrome, which ONE of the following statements is correct?
A. It most commonly develops after significant weight gain that increases retroperitoneal fat.
B. It occurs when the superior mesenteric artery is compressed between the third part of the
duodenum and the abdominal aorta.
C. It can be diagnosed on cross-sectional imaging when the aorto-mesenteric angle is < 25° and
the aorto-mesenteric distance is < 8 mm.
D. Posterior Nutcracker syndrome is a recognized gastrointestinal presentation of this condition.
Explanation: (by Perplexity AI, generated with the OpenAI O3 model)
• Pathophysiology: SMA syndrome results from compression of the third part of the
duodenum between the SMA and aorta, usually after rapid weight loss that reduces the fat pad1.
• Imaging criteria: An aorto-mesenteric angle < 22–25° and distance < 8 mm on CT or ultrasound
strongly suggest the diagnosis.
• Nutcracker syndrome involves left renal vein compression and is not a clinical presentation of SMA
syndrome.
EBDR Exam MCQs & Concepts June 2025
Dr. Kareem Alnakeeb 11
28. A 75-year-old diabetic man underwent a left below knee amputation 3 months ago for osteomyelitis of
the distal tibia. Since then, he has experienced recurrent episodes of fever and malaise. MRI is
contraindicated due to a metallic aortic valve. Which is the best investigation to exclude an occult focus
of osteomyelitis?
A. CT
B. US
C. Scintigraphy using gallium
D. Scintigraphy using indium-labelled white cells
E. Scintigraphy using technetium (Tc-99m) monodiphosphonate
Source: McQueen, Andrew S., et al. Grainger & Allison’s Diagnostic Radiology: Single Best Answer
MCQs. 5th ed., Churchill Livingstone, 2009.
Explanation:
• Although an indium-labelled white cell study is more specific, a bone scintigram using Tc-99m
monodiphosphonate is a more sensitive test to exclude osteomyelitis.
29. An 18-year-old woman with Poland syndrome is being assessed by plastic surgery for reconstruction. As
part of her pre-operative work-up a CT chest is requested. What is the classic fi nding in this disorder?
A. Absence of the sternal head of pectoralis major.
B. Hypoplastic clavicles.
C. Anterior protrusion of the ribs.
D. Bilateral breast aplasia.
E. Anterior protrusion of the sternum.
Source: Lindsay, Richard, et al. SBAs for the FRCR Part 2A. 1st ed., Oxford University Press, 2012.
Explanation:
• Poland syndrome is an uncommon congenital unilateral chest wall deformity characterized by
partial or total absence of the greater pectoral muscle and ipsilateral syndactyly.
• Associated anomalies include ipsilateral breast aplasia and atrophy of the second to fifth ribs.
• Hypoplastic clavicles are a feature of cleidocranial dysostosis.
• Anterior protrusion of the ribs gives rise to pectus excavatum, whereas anterior protrusion of the
sternum is seen in pectus carinatum.
30. Which of the following is NOT one of the 4 regions considered when assessing MRI scans for the
dissemination in space component of McDonald criteria for multiple sclerosis (2017 version)?
A. cortical and/or juxtacortical
B. infratentorial
C. periventricular
D. spinal cord
E. subcortical
Source: Radiopaedia
Explanation:
• Dissemination in space assesses for the presence of lesions in the following 4 regions:
o periventricular (≥1 lesion, unless the patient is over the age of 50 in which case it is advised
to seek a higher number of lesions)
o cortical or juxtacortical (≥1 lesion)
o infratentorial (≥1 lesion)
o spinal cord (≥1 lesion)
• Notably, T2-hyperintense lesions of the optic nerve, such as those in a patient presenting with optic
neuritis, cannot be used in fulfilling the 2017 revised McDonald criteria.
• Subcortical lesions (those neither abutting the ventricles nor the cortex) are also not counted.
EBDR Exam MCQs & Concepts June 2025
Dr. Kareem Alnakeeb 12
31. A 53-year-old male smoker is under evaluation for a thoracic aortic aneurysm. Whilst reviewing pre- and
post-contrast CT images of the chest, a 2.4 cm lesion is seen in the left lower lobe, with a mural nodule.
Which of the following features most favors a diagnosis of cavitating lung cancer rather than an
intracavitatory apergilloma?
A. Size of lesion
B. Contrast enhancement > 10HU
C. Wall thickness
D. Adjacent bronchiectasis
E. Volume loss in involved lobe
Source: Leen, Edward. Succeeding in the New FRCR Part 2A Exam: Single Best Answer (SBA)
Revision Questions for Modules 1-6. 1st ed., UNKNO, 2011.
Explanation:
• Degree of contrast enhancement is much higher in cavitating lung tumors.
• Adjacent bronchiectasis is more often seen in aspergillomas.
32. Causes of ‘tree in bud’ appearance include all, except
A. Tuberculosis
B. Allergic bronchopulmonary aspergillosis
C. Cystic fibrosis
D. Tumor emboli
E. Chronic pulmonary embolism
Source: Chin, Teck Yew, et al. Get Through Final FRCR 2A: SBAs. 1st ed., CRC Press, 2017.
Explanation:
• Radiologic Appearance
o The tree-in-bud pattern on HRCT is characterized by small centrilobular nodules of soft-
tissue attenuation connected to multiple branching linear structures of similar calibre
originating from a single stalk.
• Causes and Associated Disorders
o Initially described in cases of endobronchial Mycobacterium tuberculosis, it has
subsequently been reported in peripheral airways diseases such as infection (bacterial,
fungal, viral or parasitic), congenital disorders (like cystic fibrosis and Kartagener’s
syndrome), idiopathic disorders (obliterative bronchiolitis, panbronchiolitis), aspiration,
inhalation, immunologic disorders (like ABPA), connective tissue disorders and peripheral
pulmonary vascular diseases such as neoplastic pulmonary emboli.
33. A patient with known tuberous sclerosis had a routine follow-up CT. A 3 x 2-cm partly calcified
heterogeneously enhancing lesion was seen at the level of the foramen of Monro. What is the most likely
pathology?
A. Colloid cyst
B. Subependymal giant cell astrocytoma
C. Intraventricular
D. Meningioma
E. Germinoma
Source: Proctor, Robin. Final FRCR Part A Modules 4-6 Single Best Answer MCQs: The SRT Collection
of 600 Questions with Explanatory Answers (MasterPass). 1st ed., CRC Press, 2009.
Explanation:
• 15% of patients with tuberous sclerosis develop subependymal astrocytomas. They typically occur
at the foramen of Monro and are usually a well-defined rounded mass with some calcification. They
usually enhance uniformly with contrast and can degrade to a high-grade astrocytoma.
EBDR Exam MCQs & Concepts June 2025
Dr. Kareem Alnakeeb 13
• 95% of tuberous sclerosis patients have subependymal hamartomas. These occur in the
periventricular region, are isointense to white matter on Ti and calcified on CT.
• 55% of patients have cortical tubers, which are high signal on T2-weighted imaging.
34. A 29-year-old woman with fever, malaise, fatigue, intermittent pain and numbness in both hands and
feet, and normal chest radiograph is referred for MRI thorax. MRI shows wall thickening of the origin of the
right subclavian artery and both carotid arteries. What is the diagnosis?
A. Moyamoya disease
B. Takayasu arteritis
C. Churg–Strauss disease
D. PAN
E. Wegener’s granulomatosis
Source: Chin, Teck Yew, et al. Get Through Final FRCR 2A: SBAs. 1st ed., CRC Press, 2017.
Explanation:
Takayasu arteritis
• Takayasu arteritis is a form of granulomatous vasculitis affecting large and medium-sized arteries,
characterised by ocular disturbances and weak pulses in the upper extremities (pulseless disease).
• It is associated with fibrous thickening of the aortic arch with narrowing of the origins of the great
vessels at the arch.
• Takayasu arteritis can be limited to the descending thoracic and abdominal aorta.
• It is seen in young and middle-aged patients, especially Asian and women.
• The diagnosis is confirmed by a characteristic arteriographic pattern of irregular vessel walls,
stenosis, post-stenotic dilatation, aneurysm formation, occlusion and evidence of increased
collateral circulation.
Polyarteritis nodosa
• Polyarteritis nodosa is a fibrinoid necrotising vasculitis that mainly involves small and medium-sized
arteries of the muscles.
• Multiple aneurysm formation is a characteristic finding.
• The kidney is most commonly involved, followed by the GI tract, liver, spleen and pancreas.
• Positive ANCA titres (usually pANCA type) are found in variable percentages of patients.
Wegener’s granulomatosis
• Wegener’s granulomatosis is a distinct clinicopathologic entity characterised by granulomatous
vasculitis of the upper and lower respiratory tract together with glomerulonephritis.
Churg–Strauss syndrome
• Churg–Strauss syndrome is characterised by granulomatous vasculitis of multiple organ systems,
particularly the lung, and involves both arteries and veins as well as pulmonary and systemic
vessels.
Moyamoya disease
• Moyamoya disease is a progressive vasculopathy leading to stenosis of the main intracranial arteries.
• Characteristic angiographic features of the disease include stenosis or occlusion of the arteries of
the circle of Willis, as well as the development of collateral vasculature that produces a typical
angiographic image called ‘clouds of smoke’ or ‘puff of cigarette smoke’.
EBDR Exam MCQs & Concepts June 2025
Dr. Kareem Alnakeeb 14
35. The unrestrained passenger of a vehicle involved in a high-energy road traffic accident is admitted with a
‘hangman's fracture’. What is the most likely appearance on plain film?
A. Fractures through the neural arch of Cl
B. Fractures through the neural arch of C2
C. Fracture of the spinous process of C7
D. Transverse fracture through the base of the dens
E. Wedge compression fracture of an upper cervical vertebra
Source: McQueen, Andrew S., et al. Grainger & Allison’s Diagnostic Radiology: Single Best Answer
MCQs. 5th ed., Churchill Livingstone, 2009.
36. A 46-year-old female presented to the Emergency Department in heart failure with a history of chest pain.
A chest radiograph demonstrated an enlarged heart and bilateral pleural effusions. What imaging finding
would suggest a diagnosis of constrictive pericarditis rather than restrictive cardiomyopathy?
A. A pericardial thickness on CT of 4mm
B. Dilated right atrium
C. Ascites
D. Absence of ventricular hypertrophy
E. Global subendocardial late gadolinium enhancement on MRI
Source: Proctor, Robin. Final FRCR Part A Modules 4-6 Single Best Answer MCQs: The SRT Collection
of 600 Questions with Explanatory Answers (MasterPass). 1st ed., CRC Press, 2009.
Explanation:
• Although pericardial thickening does not confirm a diagnosis of constrictive pericarditis, when the
differential is between that and a restrictive cardiomyopathy it favours the former.
• B, C and D are non-discriminatory. E is occasionally found in restrictive cardiomyopathy.
37. A 32-year-old male presents with increasing shortness of breath following a road traffic accident, in
which he sustained multiple long bone fractures. At 48 hours post-injury, his chest radiograph is normal.
The next day a V /Q scan shows patchy, mottled, peripheral perfusion defects. The following day a chest
radiograph shows patchy, bilateral, alveolar infiltrates. What is the most likely diagnosis?
A. fat embolism
B. thrombotic embolism
C. atypical infection
D. pulmonary contusions
E. pulmonary oedema
Source: Bydder, Megan, et al. Get Through Final FRCR Part A: SBAs for the Modular Examination. 1st
ed., Hodder Education Publishers, 2009.
Explanation:
• In a patient who has sustained multiple fractures, fat embolism should always be considered when a
patient is short of breath in the presence of a normal chest radiograph. This manifests on a V /Q scan
as mottled peripheral perfusion defects.
• Chest radiograph remains normal for up to 72 hours, when discoid atelectasis, diffuse alveolar
infiltrates and consolidation may develop.
• Fat embolism may precede the development of acute respiratory distress syndrome.
• Pulmonary contusions usually manifest earlier, within the first 24 hours.
EBDR Exam MCQs & Concepts June 2025
Dr. Kareem Alnakeeb 15
38. A 6-year-old with spina bifida has a chest X-ray performed for possible lower respiratory tract infection.
The lungs are clear but there is a well-defined, round paraspinal mass with an air–fluid level. What is the
most likely diagnosis?
A. Bronchogenic cyst
B. Morgagni hernia
C. Oesophageal duplication cyst
D. Cystic teratoma
E. Oesophageal tumour
Source: Chin, Teck Yew, et al. Get Through Final FRCR 2A: SBAs. 1st ed., CRC Press, 2017.
Explanation:
Esophageal duplication cysts
• Esophageal duplication cysts are rare congenital anomalies.
• They are associated with vertebral anomalies (spina bifida, hemivertebrae, fusion defects).
• There is also an association with esophageal atresia and small bowel duplication.
• Most cysts develop in the right posteroinferior mediastinum.
• CT demonstrates a well-marginated round, oval or tubular-shaped fluid-filled cystic structure that
has a well-defined, thin wall.
• The cyst is of water attenuation with no enhancement of contents and no infiltration of surrounding
structures.
• Malignant degeneration is rare.
Bronchogenic cyst
• Bronchogenic cyst is the most common cystic mediastinal mass that typically lies in the middle
mediastinum, not in a paraspinal location; in addition, you would not expect an air–fluid level.
Other mediastinal lesions
• Cystic teratoma is an anterior mediastinal mass.
• Morgagni hernia would be unlikely to cause a solitary round lesion; multiple structures would be
expected.
39. What is the typical pattern of late gadolinium enhancement seen in hypertrophic cardiomyopathy?
A. Diffuse subendocardial enhancement
B. Patchy mid-wall enhancement, often at the RV insertion points or in the most hypertrophied
segments
C. Transmural enhancement in a coronary distribution
D. No enhancement is typically seen
Explanation:
• Cardiac-MRI studies of hypertrophic cardiomyopathy consistently show focal, non-ischaemic LGE
that is
o mid-myocardial rather than subendocardial or transmural,
o often multifocal and “patchy”, and
o frequently concentrated where the hypertrophied septum meets the RV free wall (the RV
insertion sites) or in areas of maximal wall thickness.
EBDR Exam MCQs & Concepts June 2025
Dr. Kareem Alnakeeb 16
40. A 72-year-old man presents to the vascular surgeon with abdominal pain 4 months after endovascular
repair of an abdominal aortic aneurysm. An emergency dual phase contrast-enhanced CT is performed.
The unenhanced images reveal high-density material interposed between the stent and the wall of the
aorta. There is further enhancement of this high-density area on arterial phase images. The graft and the
attachments look intact. What is the most likely diagnosis?
A. Type I endoleak
B. Type II endoleak
C. Type III endoleak
D. Type IV endoleak
E. Type V endoleak
Source: Chin, Teck Yew, et al. Get Through Final FRCR 2A: SBAs. 1st ed., CRC Press, 2017.
Explanation:
Type I Endoleak
• In a Type I endoleak, there is poor apposition between one of the attachment sites of a stent graft and
the native aortic or iliac artery wall, and blood leaks through this defect into the aneurysm sac.
• A Type I endoleak can be seen immediately after stent-graft deployment.
• On CT, dense contrast collection is usually seen centrally within the sac and is often continuous with
one of the attachment sites.
Type II Endoleak
• Type II endoleaks are the most common.
• They occur when there is retrograde flow of blood into the aneurysm sac via an excluded aortic
branch, most commonly IMA or a lumbar artery.
• Many Type II endoleaks close spontaneously over time.
• CT shows peripheral or central location of acute haemorrhage.
Type III Endoleak
• Leakage of blood through the body of a stent graft results in a Type III endoleak.
• Type III endoleaks manifest as collections of haemorrhage or contrast material centrally within the
aneurysm sac, usually distant from the attachment sites or native vessels.
Type IV Endoleak
• Opacification of the aneurysm sac immediately after placement of a stent graft without a discernible
source of leakage is designated a Type IV endoleak.
Type V Endoleak (Endotension)
• A Type V endoleak, or endotension, is characterised by continued growth of an excluded aneurysm
sac without direct radiologic evidence of a leak.
41. A 60-year-old female presents with a history of facial pain and diplopia. Clinical examination reveals palsies
of the III, IV, and VI cranial nerves, Horner’s syndrome, and facial sensory loss in the distribution of the
ophthalmic and maxillary divisions of the trigeminal (V) cranial nerve. Where is the causative abnormality
located?
A. Dorello’s canal.
B. Cavernous sinus.
C. Superior orbital fissure.
D. Inferior orbital fissure.
E. Meckel’s cave.
Source: Lindsay, Richard, et al. SBAs for the FRCR Part 2A. 1st ed., Oxford University Press, 2012.
Explanation:
Anatomy of the Cavernous Sinus
EBDR Exam MCQs & Concepts June 2025
Dr. Kareem Alnakeeb 17
• Cranial nerves III, IV, and VI, and ophthalmic (V1) and maxillary (V2) divisions of the V cranial nerve
course through the cavernous sinus along with the internal carotid artery.
• The V2 division of the trigeminal nerve passes through the inferior portion of the cavernous sinus and
exits via the foramen rotundum.
• The remainder of the cranial nerves mentioned above enter the orbit via the superior orbital fissure.
Clinical Implications
• The cavernous sinus location accounts for these features.
• Palsies of cranial nerves III, IV, and VI result in ophthalmoplegia.
• Involvement of V1 and V2 divisions of the trigeminal nerve produces facial pain and sensory loss;
involvement of sympathetic nerves around the internal carotid artery results in Horner’s syndrome.
• This cluster of findings is found in Tolosa Hunt syndrome, an idiopathic inflammatory process
involving the cavernous sinus.
42. Features of diaphyseal aclasia (hereditary multiple exostosis) include: (true or false)
A. Autosomal recessive inheritance
B. Exostoses have a cap of hyaline cartilage, often with a bursa formation over the cap
C. Exostoses arise from the metaphysis and point towards the joint
D. Exostoses stop growing when the nearest epiphyseal centre fuses
E. Malignant transformation to chondrosarcoma occurs in 35-40%
Source: Hussain, Shahid, et al. Radiology MCQs for the New FRCR Part 2A. 1st ed., TFM Publishing,
2006.
Explanation:
• a) False - AD, presents at 2-10 years of age
• b) True
• c) False - arise from metaphysis of long bones near epiphyses and point away from the joint
• d) True
• e) False - <5%
43. On imaging, which of the followings causes 'Bone within Bone' appearance? (true or false)
A. Marfan's syndrome
B. Sickle cell disease
C. Rickets.
D. Fibrous dysplasia
Source: Hussain, Shahid, et al. Radiology MCQs for the New FRCR Part 2A. 1st ed., TFM Publishing,
2006.
Explanation:
The causes include:
• Congenital syphilis
• Infantile cortical hyperostosis
• Sickle cell disease
• Oxalosis
• Paget's disease
• Acromegaly and radiation
EBDR Exam MCQs & Concepts June 2025
Dr. Kareem Alnakeeb 18
44. Concerning dislocations: (True Or False)
A. Anterior dislocation of the hip accounts for 10-20% of all hip dislocations
B. Posterior dislocations of both radius and ulna account for 80-90% of elbow dislocations
C. Anterior dislocation of the shoulder accounts for more then 90% of glenohumeral dislocations
D. A Bankhart lesion is a fracture of the anterior aspect of the superior rim of the glenoid
E. Dislocation of the patella is usually medial
Source: Hussain, Shahid, et al. Radiology MCQs for the New FRCR Part 2A. 1st ed., TFM Publishing,
2006.
Explanation:
• a) True - lies medial and inferior to acetabulum on pelvis X-ray
• b) True - isolated dislocation of the radial head is rare
• c) True - 97% are anterior dislocations. Associated with a Hill-Sachs lesion which is a defect in the
posterolateral aspect of the humeral head
• d) False - inferior rim
• e) False – lateral
45. Which of the following statements is CORRECT? (true or false)
A. Paget’s disease has a prevalence of 10% in people over the age of 80 years of age
B. Ankylosing spondylitis is found more commonly in Black than Caucasian populations
C. Developmental dysplasia of the hip is more common in males
D. Diffuse idiopathic skeletal hyperostosis commonly presents in children
E. The highest incidence of fibrous dysplasia is between 30-50 years of age
Source: Hussain, Shahid, et al. Radiology MCQs for the New FRCR Part 2A. 1st ed., TFM Publishing,
2006.
• A) True- unusual in under 40-year-olds
• b) False - ratio of 3:1
• c) False - much commoner in girls
• d) False - usually presents in over 50-year-olds
• e) False - the highest incidence of fibrous dysplasia is at 3-15 years of age. 75% are seen below 30
years of age
46. A 54-year-old man presents with a swelling in his right popliteal fossa. A Baker’s cyst is suspected
clinically and an ultrasound scan is arranged. This confirms a complex cystic structure with debris. To
help confirm this is a Baker’s cyst, you look for a communication of this cyst with fluid at the posterior
aspect of the knee joint between which two tendons?
A. Semitendinosis and lateral head of gastrocnemius.
B. Semitendinosis and medial head of gastrocnemius.
C. Semitendinosis and semimembranosis.
D. Medial and lateral heads of gastrocnemius.
E. Lateral head of gastrocnemius and semimembranosis.
F. Medial head of gastrocnemius and semimembranosis.
Source: Lindsay, Richard, et al. SBAs for the FRCR Part 2A. 1st ed., Oxford University Press, 2012.
Explanation:
• Identification of anechoic cysts communicating with fluid between the semimembranosis and
gastrocnemius tendons confirms the diagnosis of Baker’s cyst.
• It is important to perform further imaging if the mass in the posterior compartment lacks signs of
communication with fluid between the semimembranosis and medial gastrocnemius tendons.
• If this is the case, there are other possibilities for the lesion, including meniscal cyst or even a myxoid
sarcoma.
EBDR Exam MCQs & Concepts June 2025
Dr. Kareem Alnakeeb 19
47. Skeletal features of thalassemia major include:
A. Central nidus
B. Bone sclerosis
C. Narrowing of medullary cavity
D. Premature fusion of epiphysis
Source: Hussain, Shahid, et al. Radiology MCQs for the New FRCR Part 2A. 1st ed., TFM Publishing,
2006.
Explanation:
• Other features include:
o Erlenmeyer flask deformity
o Arthropathy - secondary to haemochromatosis and CPPD
o Osteoporosis
o marrow hyperplasia
48. Regarding hyperparathyroidism (HPT): Which statement is CORRECT? (true or false)
A. Brown tumours occur more frequently in secondary HPT
B. Rugger Jersey spine occurs more frequently in primary HPT
C. Chondrocalcinosis is seen in 15-20%
D. Increased incidence of slipped upper femoral epiphysis is associated with HPT
E. A normal bone scan in about 80%
Source: Hussain, Shahid, et al. Radiology MCQs for the New FRCR Part 2A. 1st ed., TFM Publishing,
2006.
Explanation:
• A. False - primary
• B. False - secondary
• C. True - more frequent in secondary HPT
• D. True
• E. True
49. A 28-year-old tennis player undergoes a MR arthrogram to investigate recurrent right shoulder instability
following a previous glenohumeral dislocation. The MRI reveals a tear of the anterosuperior labrum,
closely related to the insertion of the biceps tendon. How are these appearances best described?
1. Anterior labral tear
2. Bankart lesion
3. Hill-Sachs lesion
4. Reverse Hill-Sachs lesion
5. Superior labrum from anterior to posterior (SLAP) lesion
Source: McQueen, Andrew S., et al. Grainger & Allison’s Diagnostic Radiology: Single Best Answer MCQs.
5th ed., Churchill Livingstone, 2009.
EBDR Exam MCQs & Concepts June 2025
Dr. Kareem Alnakeeb 20
50. An asymptomatic 65-year-old woman on long-term steroids for rheumatoid disease undergoes dual
energy X-ray absorptiometry (DXA). Her Z score is —2 and her T score is —2.7. What is the WHO definition
of osteoporosis?
1. T score less than —1
2. T score less than —2.5
3. Z score less than —1
4. Z score less than —2.5
5. Mean of T and Z score less than —2
Source: McQueen, Andrew S., et al. Grainger & Allison’s Diagnostic Radiology: Single Best Answer MCQs.
5th ed., Churchill Livingstone, 2009.
Explanation
• Bone density can be measured in relation to an age and sex-matched population (Z score) or in
relation to a population of young adults of the same sex (T score).
• The WHO defines osteoporosis as a T score less than —2.5, therefore relating bone mineral density
to sex-matched peak bone mass.
51. Regarding MRI examination of the shoulder, what are the signal characteristics of the normal
supraspinatus tendon?
A. High signal intensity on all sequences
B. High signal on T1w, low signal on T2w
C. Intermediate signal on all sequences
D. Low signal on all sequences
E. Low signal on T1w, high signal on T2w
Source: McQueen, Andrew S., et al. Grainger & Allison’s Diagnostic Radiology: Single Best Answer
MCQs. 5th ed., Churchill Livingstone, 2009.
52. A 5-year-old boy presents with a 1-month history of pain in right leg. Radiography shows an ill-defined
lucency in the proximal tibial metadiaphysis with periosteal reaction and a wide zone of transition. MRI
shows a intramedullary lesion which returns intermediate signal on T1, high on STIR and has an
extraosseous enhancing mass. What is the most likely diagnosis?
A. Ewing’s sarcoma
B. Osteomyelitis
C. Enchondroma
D. Fibrous dysplasia
E. Giant cell tumour
Source: Gupta, Chaitanya. 300 Single Best Answers for the Final FRCR Part A. 1st ed., Jaypee UK,
2010.
Explanation:
• More than 50% of cases are seen in long bones.
• They are common in the metadiaphyseal region and most common in the 5- to 10-year age group.
• Often presenting similarly to osteomyelitis.
• MRI is extremely useful in determining the extent of the tumour, which is low on T1, high on T2 and
STIR and enhances with contrast.
EBDR Exam MCQs & Concepts June 2025
Dr. Kareem Alnakeeb 21
53. A young man undergoes an MRI of the right knee due to clinical suspicion of an acute rupture of the ACL.
The ACL is indistinct, and cannot be visualised in either the coronal or sagittal plane. Which additional
features would be supportive of a diagnosis of ACL rupture?
1. Bunching up of the PCL
2. Oedema within the medial collateral ligament
3. Posterior translation of the femur on the tibial condyles
4. Straightening of the patellar ligament
5. Tear of the medial meniscus
Source: McQueen, Andrew S., et al. Grainger & Allison’s Diagnostic Radiology: Single Best Answer MCQs.
5th ed., Churchill Livingstone, 2009.
54. A series of neonatal radiographs reveal a narrow thorax with short ribs, square iliac wings with horizontal
acetabular roofs, short sacrosciatic notches, progressive narrowing of the interpedicular distance and
posterior scalloping of the vertebral bodies. What is the most likely diagnosis?
F. Achondroplasia
G. Campomelic dysplasia
H. Cleidocranial dysplasia
I. Ellis-van Creveld syndrome
J. Morquio's syndrome
Source: McQueen, Andrew S., et al. Grainger & Allison’s Diagnostic Radiology: Single Best Answer
MCQs. 5th ed., Churchill Livingstone, 2009.
Explanation:
• The iliac wings in Morquio's syndrome are characteristically flaredTather than
55. MRCP examination for 47-year-old woman with obstructive jaundice shows a smooth stricture in the mid-
common bile duct with associated moderate intrahepatic biliary dilatation. The stricture is caused by
extrinsic compression from a round filling defect within the cystic duct. What is the diagnosis?
A. Acute bacterial cholangitis
B. Gallbladder carcinoma
C. Mirizzi syndrome
D. Primary sclerosing cholangitis (PSC)
Source: McQueen, Andrew S., et al. Grainger & Allison’s Diagnostic Radiology: Single Best Answer MCQs.
5th ed., Churchill Livingstone, 2009.
Explanation
• In Mirizzi syndrome, a gallstone in the cystic duct produces mass effect on the common duct and
can lead to fistula formation.
56. Which finding would indicate a nonresectable pancreatic tumor?
A. The pancreatic duct dilated to 6 mm
B. The presence of a 5-mm coeliac axis lymph node
C. The tumor has invaded the duodenum
D. The tumor in contact with 75% of the superior mesenteric artery
Source: McQueen, Andrew S., et al. Grainger & Allison’s Diagnostic Radiology: Single Best Answer
MCQs. 5th ed., Churchill Livingstone, 2009.
Explanation:
• If the tumor is in contact with more than half of the vessel circumference, it is very unlikely to be
resectable.
EBDR Exam MCQs & Concepts June 2025
Dr. Kareem Alnakeeb 22
57. Which of the following features would be MOST consistent with intestinal polyp?
A. The lesion contains a locule of gas at its base.
B. The lesion has a mean density of — 150 HU.
C. The lesion is of homogeneous attenuation.
D. There are diverticulae seen in the sigmoid colon.
Source: McQueen, Andrew S., et al. Grainger & Allison’s Diagnostic Radiology: Single Best Answer
MCQs. 5th ed., Churchill Livingstone, 2009.
Explanation:
• A polyp will usually demonstrate uniform soft tissue density, similar to the surrounding bowel wall.
58. On ultrasonographic examination, diffuse thickening of gall bladder with hyperechoic shadow at neck
and comet tailing is seen in:
A. Xanthogranulomatous cholecystitis
B. Adenomyomatosis
C. Adenomyomatous polyps
D. Cholesterol crystals
Explanation: (by Perplexity AI, generated with the OpenAI O3 model)
• Adenomyomatosis is a benign hyperplastic condition of the gallbladder characterized by mural
thickening and formation of Rokitansky–Aschoff sinuses. Ultrasonography typically reveals diffuse or
segmental wall thickening, with comet tail or ring-down artefact resulting from cholesterol crystals
trapped within the sinuses, most often seen at the gallbladder neck.
• Xanthogranulomatous cholecystitis produces heterogeneous wall thickening but lacks comet tail
artefacts.
• Adenomyomatous polyps refer to focal instead of diffuse changes and do not exhibit classic comet
tail artefact.
• Cholesterol crystals can contribute to comet tailing, but they most commonly present as echogenic
foci without diffuse wall thickening.
59. On scrotal ultrasound for testicular torsion, which of the following radiological findings would suggest
that the testis is still viable?
A. A diffusely enlarged hypoechoic left testis
B. A normal echogenicity testis on grey-scale imaging
C. A small shrunken left testis with a surrounding hydrocoele and scrotal wall thickening
D. Absent blood flow within the left testis on color flow Doppler but good flow within the tunica vaginalis
Source: McQueen, Andrew S., et al. Grainger & Allison’s Diagnostic Radiology: Single Best Answer MCQs.
5th ed., Churchill Livingstone, 2009.
Explanation:
• The section on testicular torsion within the comprehensive review by Fiitterer et al provides useful
additional information.
60. Which of the following indicates T3 rather than T4 lung cancer?
A. Invasion of the oesophagus
B. Invasion of the trachea
C. Invasion of the pericardium
D. Malignant pleural effusion
E. Invasion of the vertebral body
Source: Leen, Edward. Succeeding in the New FRCR Part 2A Exam: Single Best Answer (SBA)
Revision Questions for Modules 1-6. 1st ed., UNKNO, 2011.
EBDR Exam MCQs & Concepts June 2025
Dr. Kareem Alnakeeb 23
Explanation:
• T3 disease features include a tumor of any size less than 2cm from the carina, invasion of the
parietal pleura, chest wall, diaphragm, mediastinal pleura, pericardium, pleural effusion or satellite
nodule in the same lobe.
• T4 disease is characterized by invasion of the heart, great vessels, trachea, oesophagus, vertebral
body, carina or the presence of a malignant pleural effusion.
• The TNM staging system was updated in 2009 (AJR, 2010).
61. A 78-year-old man has myelodysplastic syndrome and requires frequent blood transfusions. He develops
progressively abnormal liver function tests and a grossly elevated ferritin level. An MRI of the liver is
performed using breath hold half Fourier single shot spin echo T2w images. Which finding would make a
diagnosis of hemosiderosis (iron overload from recurrent blood transfusion) more likely than
haemochromatosis?
A. Increased T2 signal in the liver only
B. Increased T2 signal in the liver and spleen
C. Reduced T2 signal in the liver only
D. Reduced T2 signal in the liver and spleen
E. Reduced T2 signal in the spleen only
Source: McQueen, Andrew S., et al. Grainger & Allison’s Diagnostic Radiology: Single Best Answer
MCQs. 5th ed., Churchill Livingstone, 2009.
Explanation:
• In iron overload due to recurrent transfusions, there is increased iron deposition in the
reticuloendothelial system. This leads to reduced Tl, T2 and T2* signal intensity in the liver and
spleen.
• Haemochromatosis causes diffusely reduced T2 signal in the liver and may lead to cirrhosis, but the
splenic signal intensity should remain normal.
• Diffuse fatty liver will lead to increased T2 signal in the liver with signal loss during out-of-phase
images.
62. An 18-year-old man experiences persistent symptoms following a fracture through the waist of the right
scaphoid. Radiographs of the right scaphoid indicate non-union. An MRI is performed to assess the
vascularity of the proximal pole. Which imaging features are consistent with a diagnosis of avascular
necrosis?
A. Bone marrow enhancement following administration of gadolinium
B. High signal surrounding the fracture on T2w images
C. High signal within the proximal pole on T1w images
D. High signal within the proximal pole on STIR images
E. Low signal within the proximal pole on T1w images
Source: McQueen, Andrew S., et al. Grainger & Allison’s Diagnostic Radiology: Single Best Answer
MCQs. 5th ed., Churchill Livingstone, 2009.
Explanation:
• Low signal on T1 reflects death of the adipocytes. The combination of low signal on T1w images and
low or intermediate signal on T2w images accurately predicts avascular necrosis.
EBDR Exam MCQs & Concepts June 2025
Dr. Kareem Alnakeeb 24
63. A low flat renogram curve indicates:
A. Advanced nephropathy
B. Complete obstruction to urine outflow
C. Vesico-ureteric reflux
D. Partial obstruction to urine outflow
Explanation: (by Perplexity AI, generated with the OpenAI O3 model)
• A very low, almost horizontal time–activity curve reflects markedly reduced tracer extraction by
damaged renal parenchyma, typical of end-stage or advanced medical renal disease.
• Complete or partial outflow obstruction instead produces an uptake phase followed by a rising or
plateau phase; vesico-ureteric reflux alters the post-void segment rather than the primary
renographic curve
64. A 67-year-old male patient presents with an 8-week history of left loin pain. A renal CT is obtained and
this shows a 6-cm enhancing left renal lesion that has a fi brotic central scar. What is the most likely
diagnosis?
A. Renal leiomyoma.
B. Renal oncocytoma.
C. Renal metanephic adenoma.
D. Renal haemangioma (giant).
E. Renal juxta-glomerular cell neoplasm.
Source: Lindsay, Richard, et al. SBAs for the FRCR Part 2A. 1st ed., Oxford University Press, 2012.
Explanation:
Renal oncocytoma
• This is a benign renal cell neoplasm responsible for about 5% of all adult primary renal epithelial
neoplasms.
• It typically occurs in elderly men.
• They usually appear as solitary, well-demarcated, unencapsulated, fairly homogeneous renal
cortical tumours.
• Bilateral, multicentric oncocytomas are seen in hereditary syndromes of renal oncocytosis and Birt–
Hogg–Dubé syndrome.
• A central stellate scar is seen in approximately one-third.
• However, distinguishing them from RCC on imaging is not reliable.
Renal leiomyoma
• Leiomyoma of the kidney is a benign smooth muscle neoplasm.
• It appears as a well-circumscribed, homogeneous, exophytic solid mass that shows uniform
enhancement on contrast-enhanced CT.
• It may occasionally be cystic.
Metanephric adenoma
• Metanephric adenoma is a benign renal neoplasm that is more common in middle-aged to elderly
females.
• It is associated with polycythaemia in 10%.
• It typically appears as a well-defined, unencapsulated, solitary mass that may be hyperattenuating
on unenhanced CT.
• Calcification can be seen in up to 20%.
Renal hemangioma
EBDR Exam MCQs & Concepts June 2025
Dr. Kareem Alnakeeb 25
• Hemangioma of the kidney occurs as an unencapsulated, solitary lesion that frequently arises from
the renal pyramids or the pelvis.
• Contrast-enhanced CT or MRI may show early intense enhancement, with persistent enhancement
on delayed images.
Juxtaglomerular cell neoplasm (Reninoma)
• Juxtaglomerular cell (JGC) neoplasm or reninoma is an extremely rare, benign renal neoplasm of
myoendocrine cell origin, which is associated with a clinical triad of hypertension, hypokalaemia,
and high plasma renin activity.
• It typically appears as a unilateral, well-circumscribed, cortical tumour and often measures less
than 3 cm, but otherwise is indistinguishable from other cortical neoplasms.
65. The radiograph of a 40-year-old man with a painful knee shows multiple calcified loose bodies, each of
similar size, within the joint. The joint space is preserved. What diagnosis is most likely?
A. Calcium pyrophosphate arthropathy
B. Gout
C. Pigmented villonodular synovitis
D. Rheumatoid arthritis
E. Synovial osteochondromatosis
Source: McQueen, Andrew S., et al. Grainger & Allison’s Diagnostic Radiology: Single Best Answer MCQs.
5th ed., Churchill Livingstone, 2009.
66. Aqueduct stenosis is considered as:
A. Communicating non-obstructive
B. Non-communicating non-obstructive
C. Non-communicating obstructive
D. Communicating non-obstructive
67. A 35 year old man attends the accident and emergency department complaining of episodic lower back
pain radiating down the legs. History and clinical examination also suggest pelvic sphincter dysfunction.
MRI shows a spinal cord mass located at the conus medullaris. The mass is isointense on T1 and
hyperintense on T2. It demonstrates contrast enhancement. The most likely diagnosis is:
A. Astrocytoma
B. Intradural lipoma
C. Haemangioblastoma
D. Myxopapillary ependymoma
E. Ganglioglioma
Source: Currie, Stuart, et al. SBAs for the FRCR 2A. 1st ed., Cambridge University Press, 2010.
Explanation:
• This is a variant of ependymoma and is the most common neoplasm of the conus medullaris. It
originates from ependymal glia of the filum terminale. Average age at presentation is 35 years and it is
more common in men.
• T1-weighted imaging shows an isointense or occasionally hyperintense (due to the mucin content)
mass. It is hyperintense on T2 and almost always shows enhancement postcontrast.
• Intradural lipomas are hyperintense on T1-weighted imaging but they should not enhance. They also
tend to occur in younger individuals and usually have an associated, clinically apparent lumbosacral
mass.
• Haemangioblastoma can also demonstrate high signal on T1 but is also highly vascular, can show
signal voids and approximately half of them will have an intratumoural cystic component.
EBDR Exam MCQs & Concepts June 2025
Dr. Kareem Alnakeeb 26
68. A 45-year-old man presents with dysphagia and undergoes a double-contrast barium swallow. This
demonstrates a smooth oblique indentation on the posterior wall of the oesophagus. What is the most
likely cause of these appearances?
A. enlarged left atrium
B. aberrant right subclavian artery
C. aberrant left pulmonary artery
D. right-sided aortic arch
E. coarctation of the aorta
Source: Bydder, Megan, et al. Get Through Final FRCR Part A: SBAs for the Modular Examination. 1st
ed., Hodder Education Publishers, 2009.
Explanation:
Major Vessel Anomalies Causing Esophageal Impressions
• A number of anomalies of the major vessels can cause extrinsic impressions upon the oesophagus.
Right-Sided Aortic Arch
• The commonest aortic anomaly is a right-sided aortic arch, which produces an indentation on the
right lateral oesophageal wall in the absence of the normal left aortic arch impression.
Aberrant Right Subclavian Artery
• An aberrant right subclavian artery originates from the aortic arch just distal to the left subclavian
artery, and passes upwards and to the right, behind the oesophagus, giving rise to an oblique
posterior oesophageal indentation.
Aortic Coarctation
• In aortic coarctation, the preand post-stenotic dilatations of the aorta produce a characteristic
reversed-3 impression upon the left wall of the oesophagus.
Enlarged Left Atrium and Aberrant Left Pulmonary Artery
• An enlarged left atrium and an aberrant left pulmonary artery both cause anterior indentations upon
the oesophagus.
69. Which of the following is an extraconal extraorbital lesion, rather than an extraconal intraorbital lesion?
A. Squamous cell carcinoma of the sinus
B. Teratoma
C. Dermoid cyst
D. Capillary haemangioma
E. Lymphangioma
Source: Leen, Edward. Succeeding in the New FRCR Part 2A Exam: Single Best Answer (SBA)
Revision Questions for Modules 1-6. 1st ed., UNKNO, 2011.
Explanation:
• Other causes are lymphoma, adenocarcinoma, adenoid cystic carcinoma, mucoceles and paranasal
sinusitis.
EBDR Exam MCQs & Concepts June 2025
Dr. Kareem Alnakeeb 27
70. A 15-year-old male presents with a history of recurrent epistaxis and nasal obstruction. MRI
demonstrates a lesion centred at the sphenopalatine foramen, which is hypointense on T1WI and
heterogeneously intermediate signal on T2WI. Intense lesional enhancement and multiple fl ow voids are
noted on post-gadolinium T1WI. What is the diagnosis?
A. Ludwig angina.
B. Nasopharyngeal carcinoma.
C. Inverted papilloma.
D. Juvenile angiofibroma.
E. Glomus jugulare.
Source: Lindsay, Richard, et al. SBAs for the FRCR Part 2A. 1st ed., Oxford University Press, 2012.
Explanation:
Clinical Features
• Juvenile angiofibromas are benign but locally aggressive tumors with high vascularity.
• They typically occur in adolescent boys and present with recurrent epistaxis and nasal obstruction.
Typical Location and Growth Pattern
• They are centered within the sphenopalatine foramen and involve the pterygopalatine fossa,
producing a bowed appearance of the posterior wall of maxillary sinus and widening of
pterygopalatine fossa, inferior orbital, and pterygomaxillary fissures.
• Osseous erosion is commonly seen.
MRI Characteristics
• The specific differentiating feature on MRI is the presence of multiple flow voids on T2WI and
enhanced T1WI.
71. The mother of a three week old child notices a mass in her baby’s lower neck. The child is otherwise well.
There is a history of normal pregnancy and the child was delivered by forceps. Ultrasound scan reveals
homogeneous enlargement of the lower third of the right sternocleidomastoid muscle but no focal lesion
is identified. T2-weighted MRI shows diffuse abnormal high signal intensity over the same area. The most
likely diagnosis is:
A. Hematoma
B. Branchial cleft cyst
C. Fibromatosis colli
D. Neuroblastoma
E. Cystic hygroma
Source: Currie, Stuart, et al. SBAs for the FRCR 2A. 1st ed., Cambridge University Press, 2010.
Explanation:
• This is a rare form of infantile fibromatosis that occurs solely within the sternocleidomastoid
muscle.
o In the vast majority it is associated with birth trauma (e.g. forceps delivery).
o This is thought to lead to compartment syndrome, pressure necrosis and secondary fibrosis
of the muscle.
o It usually locates to the lower third of the muscle, between the sternal and clavicular heads,
and is usually unilateral.
o Ultrasound may reveal a well- or ill-defined mass or may just show homogeneous muscle
enlargement.
o In approximately two-thirds of individuals, the abnormality spontaneously regresses by the
age of two.
• Expected ultrasonographic appearances of a hematoma include a heterogeneous mass of mixed
cystic and solid components.
EBDR Exam MCQs & Concepts June 2025
Dr. Kareem Alnakeeb 28
72. Expected ultrasonographic appearances of a hematoma include a heterogeneous mass of mixed cystic
and solid components. A six year old boy is investigated for refractory complex partial seizures. CT
demonstrates a well-defined, hypodense lesion located in the cortex of the temporal lobe. There is
underlying bone remodeling but no calcification. On MRI the lesion demonstrates high signal on T2 and
predominantly low signal on T1-weighted imaging. There is no surrounding oedema, minimal mass effect
and no contrast enhancement. The most likely diagnosis is:
A. Glioblastoma multiforme
B. Dysembryoblastic neuroepithelial tumour (DNET)
C. Primitive neuroectodermal tumour (PNET)
D. Cavernous haemangiomas
E. Ependymoma
Source: Currie, Stuart, et al. SBAs for the FRCR 2A. 1st ed., Cambridge University Press, 2010.
Explanation:
DNETs
• DNETs are benign tumours of neuroepithelial origin which arise from the cortical/deep grey matter.
• They are preferentially located supratentorially (temporal 62%, frontal 31%).
• CT demonstrates a hypoattenuating mass and there may be thinning and remodelling of the
underlying inner table reflecting the slow growth of the tumour.
• On MRI they are hypointense on T1, hyperintense on T2 and small intratumoural cysts may be
present to cause a characteristic ‘bubbly’ appearance.
• There is minimal mass effect and no associated vasogenic oedema.
• A third of lesions show calcification and most tumors do not enhance.
• If present, the enhancement is faint and patchy.
Gangliogliomas and cavernous haemangiomas
• Gangliogliomas and cavernous haemangiomas are other tumours which may cause epilepsy in
children.
• Cavernous haemangiomas are typically dense on CT and commonly calcify.
PNETs
• PNETs have a tendency for necrosis, cyst formation and calcification.
• They also tend to be hyperdense on CT due to high nuclear to cytoplasmic ratio.
73. A 46-year-old woman from Bangladesh is being treated for pulmonary tuberculosis. Despite anti-
tuberculosis chemotherapy, she develops increasing fevers with abdominal discomfort and distension.
An abdominal and pelvic ultrasound demonstrates a moderate volume of peritoneal free fluid, and a
contrast-enhanced CT of the abdomen and pelvis is performed. What are the likely findings on CT?
A. A mixed solid: cystic ovarian mass with serosal deposits on the liver and spleen
B. Ascites with enlarged mesenteric lymph nodes containing high attenuation
C. Gastric wall thickening extending into the spleen with enlarged coeliac axis lymph nodes and ascites
D. Peritoneal nodularity with high density ascites
E. Portal vein thrombosis with ascites
Source: McQueen, Andrew S., et al. Grainger & Allison’s Diagnostic Radiology: Single Best Answer
MCQs. 5th ed., Churchill Livingstone, 2009.
Explanation:
• In peritoneal TB, the presence of dense ascites, peritoneal nodularity and lymph nodes with low
attenuation centres are characteristic findings.
EBDR Exam MCQs & Concepts June 2025
Dr. Kareem Alnakeeb 29
74. A three year old boy presents with seizures and headaches. CT head shows a hypoattenuating mass lying
superior to the lateral ventricles, within the frontal region. It displays negative Hounsfield units and
peripheral calcification but does not enhance. There is partial agenesis of the corpus callosum. MRI of
the brain demonstrates a pericallosal tumor which is hyperintense on T1 and less hyperintense on T2-
weighted imaging. What is the most likely diagnosis?
A. Dermoid tumour
B. Lipoma
C. Teratoma
D. Neurocytoma
E. Ependymoma
Source: Currie, Stuart, et al. SBAs for the FRCR 2A. 1st ed., Cambridge University Press, 2010.
Explanation:
Definition and Origin
• This is a congenital tumor that results from abnormal differentiation of the meninx primitiva – that
which eventually differentiates into pia, arachnoid and internal dura mater.
Incidence and Congenital Associations
• They account for less than 1% of brain tumors but are associated with congenital abnormalities,
most commonly dysgenesis of the corpus callosum to some degree.
• This is particularly likely when the lipoma is located anteriorly rather than posteriorly.
Imaging Characteristics
• On CT they are well-circumscribed masses with negative Hounsfield units and occasional
calcification, and they do not enhance.
• Characteristically, they are T1 hyperintense and slightly less hyperintense on T2.
Differential Diagnosis
• Dermoids and teratomas can show similar characteristics, with fat and calcium content.
• Teratomas may enhance, although dermoids do not.
• However, the lesion is much more likely to be a lipoma given its position (dermoids tend to be extra-
axial (spinal canal); teratomas are much more commonly found around the pineal region, floor of the
third ventricle, posterior fossa and spine) and given the association with corpus callosum
abnormalities.
75. A neonatal boy has a renal ultrasound performed for the investigation of urinary obstructive symptoms.
The ultrasound shows a distended urinary bladder with bilateral hydronephrosis. Which one of the
following is the most likely underlying pathology?
A. Posterior urethral valve
B. Neurogenic bladder
C. Horseshoe kidney
D. Ectopic ureterocoeles
E. Urethral diverticulum
Source: Currie, Stuart, et al. SBAs for the FRCR 2A. 1st ed., Cambridge University Press, 2010.
Explanation:
• Posterior urethral valve is a congenital disorder characterized by a thick mucosal fold located in the
posterior urethra. It is the most common cause of bilateral urinary tract obstruction in boys.
• It is most commonly discovered in the neonatal period, but very occasionally may present into
adulthood.
• Diagnosis is usually made with ultrasound and surgical treatment is indicated.
EBDR Exam MCQs & Concepts June 2025
Dr. Kareem Alnakeeb 30
76. An abdominal radiograph on a two-year-old child shows a paucity of bowel gas. An abdominal ultrasound
is then performed which shows a 5 cm x 2 cm mass in the right upper quadrant. In the longitudinal plane,
this has a `pseudokidney' appearance with a central echogenic focus and in the transverse plane it has
the appearance of a `bull's-eye'. What is the most likely diagnosis?
A. Ischaemic colitis
B. Intussusception
C. Volvulus
D. Necrotising enterocolitis
E. Appendicitis
Source: Proctor, Robin. Final FRCR Part A Modules 4-6 Single Best Answer MCQs: The SRT Collection
of 600 Questions with Explanatory Answers (MasterPass). 1st ed., CRC Press, 2009.
Explanation:
• Other findings on abdominal radiograph include an abdominal soft-tissue mass in the right upper
quadrant, normal appearances and small bowel obstruction.
77. A 58 year old male with unexplained elevated alkaline phosphatase has an MRCP and the ‘double-duct’
sign is observed. Which one of the following diagnoses is most likely to cause this finding?
A. Acute pancreatitis
B. Annular pancreas
C. Pancreas divisum
D. Periampullary tumour
E. Duodenal perforation
Source: Currie, Stuart, et al. SBAs for the FRCR 2A. 1st ed., Cambridge University Press, 2010.
Explanation:
• The ‘double-duct’ sign is dilatation of the main pancreatic duct and the common bile duct as seen at
ERCP and MRCP, and less commonly with CT and ultrasound.
• It occurs due to an obstructing lesion at the ampulla, most commonly a carcinoma of the head of the
pancreas (in up to 77% of cases) or a carcinoma of the ampulla of Vater (in up to 52% of cases).
• The sign may be absent if there is an accessory pancreatic duct or when the main pancreatic duct
drains into the minor papilla.
78. A 25-year-old with a history of cystic fibrosis presents with massive hemoptysis. Bronchial artery
embolization is requested. Which of the following statements regarding bronchial artery embolization is
false?
A. A descending thoracic aortogram is performed prior to selective bronchial angiography.
B. Bronchial angiography is performed with manual injection of contrast medium.
C. The abnormal bronchial artery is embolized at its origin.
D. Polyvinyl alcohol particles (diameter of 350–500 μm) may be used as the embolic material.
E. Chest pain is the most common complication.
Source: Lindsay, Richard, et al. SBAs for the FRCR Part 2A. 1st ed., Oxford University Press, 2012.
Explanation:
Overview
• Bronchial artery embolization (BAE) is an established procedure in the management of massive
haemoptysis.
• Knowledge of the bronchial artery anatomy and its variations is essential in carrying out the
procedure safely.
Imaging and Angiography
• A preliminary descending thoracic aortogram is performed to identify the number and site of origin of
the bronchial arteries.
EBDR Exam MCQs & Concepts June 2025
Dr. Kareem Alnakeeb 31
• Abnormal bronchial arteries are visualized on the preliminary thoracic aortogram in the majority of
affected patients.
• Selective bronchial angiography is performed with manual injection of contrast.
Catheterization Safety
• Selective bronchial artery catheterization and safe positioning distal to the origin of spinal cord
branches is essential to avoid spinal cord ischemia/infarction.
Embolic Agents
• Polyvinyl alcohol particles (350–500 μm diameter) are the most frequently used embolic agent.
• Smaller particles can freely flow via the intrapulmonary shunts, causing pulmonary or systemic
infarcts.
Complications
• Chest pain is the most common complication (24–91%).
• Other complications include dysphagia (due to embolization of esophageal branches), dissection of
the bronchial artery or aorta (usually self-limited), and spinal cord ischemia.
79. Which imaging modality is most sensitive for detecting ductal carcinoma in situ (DCIS) of the breast?
A. Mammography
B. Magnetic Resonance Imaging (MRI)
C. Ultrasonography
D. Digital Breast Tomosynthesis
Explanation: (by Perplexity AI, generated with the OpenAI O3 model)
• Large prospective and retrospective series show MRI sensitivity for pure DCIS in the 80 – 92% range,
consistently higher than mammography (≈55 – 70%) and far higher than ultrasound (≈35%).
• MRI is especially advantageous for high-grade or non-calcified DCIS that may be occult on
mammography.
• Mammography remains first-line for population screening because it detects micro-calcifications
cost-effectively, but sensitivity drops with dense breasts or non-calcified lesions.
• Ultrasound is chiefly adjunctive; its stand-alone sensitivity for DCIS is low and operator-dependent.
Why the other options are incorrect
• Mammography: Gold standard for screening; however, micro-calcification-based detection misses
a significant proportion of lesions, giving lower overall sensitivity than MRI.
• Ultrasonography: Detects only about one-third of DCIS lesions and primarily masses; limited for
pure micro-calcific or non-mass disease.
80. A 35-year-old female presents with a history of menorrhagia. MRI of pelvis demonstrates a fi broid uterus
for which treatment with high-intensity focused ultrasound (HIFU) is proposed. What is the principle
mechanism of action of HIFU?
A. Coagulation necrosis.
B. Apoptosis.
C. Cavitation.
D. Microstreaming.
E. Radiation forces.
Source: Lindsay, Richard, et al. SBAs for the FRCR Part 2A. 1st ed., Oxford University Press, 2012.
Explanation:
• HIFU is a non-invasive method to treat solid tumors or hemorrhage. As HIFU is essentially
ultrasound, it requires an acoustic window to transmit ultrasound energy and is subject to similar
artefact.
• The principle effect of HIFU is heat generation from absorption of acoustic energy. This causes
coagulation necrosis within seconds.
EBDR Exam MCQs & Concepts June 2025
Dr. Kareem Alnakeeb 32
• Hyperthermia also induces apoptosis, which can be an important delayed effect in tissue exposed to
lower energy HIFU. This mechanism is also a potential limitation of HIFU as adjacent tissue may be
at risk.
• Mechanical effects such as cavitation and microstreaming are also seen with the use of higher
ultrasound intensity
81. A 9-year-oid boy injures his right wrist playing football. The radiograph reveals a fracture extending
through the epiphysis and into the metaphysis. How would this injury be classified in the Salter-Harris
classification?
A. Type I
B. Type II
C. Туре III
D. Type IV
Source: McQueen, Andrew S., et al. Grainger & Allison’s Diagnostic Radiology: Single Best Answer
MCQs. 5th ed., Churchill Livingstone, 2009.
82. A 59-year-old female patient presented with malaise, chest pain and dyspnoea. Her chest radiograph
was normal. An echocardiogram demonstrated a mobile echogenic mass attached to the intra-atrial
septum by a stalk. What is the most likely diagnosis?
A. Pulmonary embolism
B. Papillary fibroelastoma
C. Sarcoma
D. Fibrovillous adenoma
E. Cardiac myxoma
Source: Proctor, Robin. Final FRCR Part A Modules 1–3 Single Best Answer MCQs: The SRT Collection
of 600 Questions with Explanatory Answers (MasterPass). 1st ed., CRC Press, 2009.
Explanation:
• The differential diagnosis of a pedunculated intracardiac lesion includes atrial myxoma and papillary
fibroelastoma.
• Papillary fibroelastomas are rare lesions that are typically asymptomatic.
83. Down syndrome is most commonly associated with:
A. ASD (septum primum type)
B. ASD (septum secundum type)
C. VSD
D. Patent foramen ovale
Explanation: (by Perplexity AI, generated with the OpenAI O3 model)
• Down syndrome is strongly linked to endocardial-cushion maldevelopment, so the characteristic
cardiac lesion is an ostium primum atrial septal defect (the septum primum type of ASD), often
classified within the spectrum of complete or partial atrioventricular septal defects.
84. Percutaneous sclerotherapy is used to treat all of the following except:
A. Arteriovenous malformation (AVM)
B. Capillary telangiectasia
C. Lymphatic malformation
D. Venous malformation
Explanation: (by Perplexity AI, generated with the OpenAI O3 model)
• Percutaneous sclerotherapy is routinely used to treat lymphatic malformations and venous
malformations.
• It is not a primary therapy for capillary telangiectasia, and it is only occasionally employed for limited
slow-flow components of some AVMs.
EBDR Exam MCQs & Concepts June 2025
Dr. Kareem Alnakeeb 33
85. Ultrasound examination of the face and neck is performed to investigate a buccal, soft-tissue mass that
became noticeable during pregnancy. The lesion is heterogeneous and hypoechoic, and has sinusoidal
spaces demonstrating slow flow and circular calcifications. Which of the following is the most likely
diagnosis?
A. benign lymph node
B. malignant lymph node
C. pleomorphic parotid adenoma
D. arteriovenous malformation
E. venous vascular malformation
Source: Bydder, Megan, et al. Get Through Final FRCR Part A: SBAs for the Modular Examination. 1st
ed., Hodder Education Publishers, 2009.
Explanation:
Vascular Malformations
• Phleboliths if present are unique to vascular malformations.
• Arterial malformations are high flow, while venous, capillary or combined malformations are low
flow.
• MRI is required to assess the full extent, particularly intraosseous and intracranial, of head and neck
vascular malformations.
Lymph Nodes
• Benign lymph nodes are smooth, elliptical and hypoechoic with hilar architecture and vascularity.
• Malignant lymph nodes are typically round, are hypoechoic, have no hilum and show peripheral
vascularity. Malignant lymph nodes with necrosis are seen with squamous cell and papillary cell
carcinoma of the thyroid.
• Internal punctate calcification is seen in metastases from papillary or medullary carcinoma of the
thyroid.
86. A severely hypoplastic cerebellar vermis in an enlarged bony posterior fossa, with associated
hydrocephalus and communication of the fourth ventricle with a posterior midline CSF cyst, are features
of which of the following posterior fossa malformations?
A. mega cisterna magna
B. Dandy–Walker malformation
C. Dandy–Walker variant
D. Arnold–Chiari malformation
E. Joubert’s syndrome
Source: Bydder, Megan, et al. Get Through Final FRCR Part A: SBAs for the Modular Examination. 1st
ed., Hodder Education Publishers, 2009.
Explanation:
Dandy–Walker Malformation
• Classically, the Dandy–Walker malformation consists of partial or total absence of the cerebellar
vermis, dilatation of the fourth ventricle into a large cystic mass, an enlarged posterior fossa,
hydrocephalus (in 75% of cases) and torcular–lambdoid inversion (elevation of the torcular Herophili
above the lambdoid suture).
• The proposed etiology is obstruction of CSF outflow at the foramina of Magendie and Luschka.
• The vermis abnormality is the key component in all forms of the Dandy–Walker complex.
• The variant is less severe with a better prognosis.
Chiari Malformations
• Chiari malformations have the fundamental abnormality of an underdeveloped, small posterior
fossa, in contrast to the Dandy–Walker complex where it is normal or enlarged.
EBDR Exam MCQs & Concepts June 2025
Dr. Kareem Alnakeeb 34
87. Thyroid radiofrequency ablation (RFA) is least commonly used in which clinical setting?
A. Hypothyroidism
B. Hyperthyroidism (toxic/autonomous nodule)
C. Malignancy
Explanation: (by Perplexity AI, generated with the OpenAI O3 model)
• RFA has no established role in treating primary hypothyroidism. It is designed to shrink nodules, not
augment deficient hormone production
• In autonomously functioning (toxic) nodules, RFA can normalize thyroid function or reduce
antithyroid-drug requirements, and is endorsed as a second-line option when surgery or radio-iodine
are unsuitable
• RFA is increasingly used for selected thyroid malignancies—especially small papillary micro-
carcinomas in non-surgical candidates and for loco-regional recurrence after
thyroidectomy. Although still “off-label” in many regions, use is growing and supported by guidelines
for carefully selected cases.
88. Which of the following indicates telangiectatic osteosarcoma (TOS) rather than an aneurismal bone cyst
(ABC)?
A. Enhancing septa without nodularity on MR
B. Marked expansile remodelling of bone
C. Cortical thinning
D. Presence of osteoid matrix with septal regions on CT
E. Presence of haemorrhagic spaces
Source: Leen, Edward. Succeeding in the New FRCR Part 2A Exam: Single Best Answer (SBA)
Revision Questions for Modules 1-6. 1st ed., UNKNO, 2011.
Explanation:
• Thick peripheral septa with nodularity, presence of an osteoid matrix within nodular or septal regions,
and aggressive growth features such as cortical destruction indicate TOS rather than ABC.
89. The mother of a three week old child notices a mass in her baby’s lower neck. The child is otherwise well.
There is a history of normal pregnancy and the child was delivered by forceps. Ultrasound scan reveals
homogeneous enlargement of the lower third of the right sternocleidomastoid muscle but no focal lesion
is identified. T2-weighted MRI shows diffuse abnormal high signal intensity over the same area. The most
likely diagnosis is:
A. Hematoma
B. Branchial cleft cyst
C. Fibromatosis colli
D. Neuroblastoma
E. Cystic hygroma
Source: Currie, Stuart, et al. SBAs for the FRCR 2A. 1st ed., Cambridge University Press, 2010.
Explanation:
Definition and Etiology
• Infantile Sternocleidomastoid Fibromatosis is a rare form of infantile fibromatosis that occurs solely
within the sternocleidomastoid muscle.
• In the vast majority it is associated with birth trauma (e.g. forceps delivery).
• This is thought to lead to compartment syndrome, pressure necrosis and secondary fibrosis of the
muscle.
Location and Natural History
• It usually locates to the lower third of the muscle, between the sternal and clavicular heads, and is
usually unilateral.
EBDR Exam MCQs & Concepts June 2025
Dr. Kareem Alnakeeb 35
• In approximately two-thirds of individuals, the abnormality spontaneously regresses by the age of
two.
Imaging Findings
• Ultrasound may reveal a well- or ill-defined mass or may just show homogeneous muscle
enlargement.
• Expected ultrasonographic appearances of a hematoma include a heterogeneous mass of mixed
cystic and solid components.
90. A 70 year old man undergoes surgery for AAA. Two weeks following surgery, he is readmitted to the A&E
department with abdominal pain and fever. Palpation of the abdomen suggests a pulsatile mass. A CT
angiogram is performed, which does not demonstrate contrast extravasation. Which of the features on
CT angiogram would be most worrisome?
A. Presence of a pseudoaneurysm
B. Periaortic soft tissue
C. Thickening of a fluid-filled third part of the duodenum
D. Some ectopic gas in the vicinity
E. Loss of fat plane between the grafted aorta and the adjacent duodenum
Source: Currie, Stuart, et al. SBAs for the FRCR 2A. 1st ed., Cambridge University Press, 2010.
Explanation:
• Two weeks post-procedure, all the other features including the presence of ectopic gas may be
postoperative.
• However, presence of a thickened fluid-filled bowel loop would be extremely worrying for an aorto-
enteric fistula.
• Presence of ectopic gas beyond four weeks is much more likely to be abnormal.
91. A 25-year-old with a history of cystic fibrosis presents with massive hemoptysis. Bronchial artery
embolization is requested. Which of the following statements regarding bronchial artery embolization is
false?
A. A descending thoracic aortogram is performed prior to selective bronchial angiography.
B. Bronchial angiography is performed with manual injection of contrast medium.
C. The abnormal bronchial artery is embolized at its origin.
D. Polyvinyl alcohol particles (diameter of 350–500 μm) may be used as the embolic material.
E. Chest pain is the most common complication.
Source: Lindsay, Richard, et al. SBAs for the FRCR Part 2A. 1st ed., Oxford University Press, 2012.
Explanation:
Overview
• Bronchial artery embolization (BAE) is an established procedure in the management of massive
haemoptysis.
• Knowledge of the bronchial artery anatomy and its variations is essential in carrying out the
procedure safely.
Imaging Evaluation
• A preliminary descending thoracic aortogram is performed to identify the number and site of origin of
the bronchial arteries.
• Abnormal bronchial arteries are visualized on the preliminary thoracic aortogram in the majority of
affected patients.
Angiography Technique
• Selective bronchial angiography is performed with manual injection of contrast.
EBDR Exam MCQs & Concepts June 2025
Dr. Kareem Alnakeeb 36
• Selective bronchial artery catheterization and safe positioning distal to the origin of spinal cord
branches is essential to avoid spinal cord ischemia/infarction.
Embolic Agents
• Polyvinyl alcohol particles (350–500 μm diameter) are the most frequently used embolic agent.
• Smaller particles can freely flow via the intrapulmonary shunts, causing pulmonary or systemic
infarcts.
Complications
• Chest pain is the most common complication (24–91%).
• Other complications include dysphagia (due to embolization of esophageal branches), dissection of
the bronchial artery or aorta (usually self-limited), and spinal cord ischemia.
92. A 20-year-old man presents with gradual onset of neck pain and a painful lump in the upper neck
posteriorly. Plain films show an apparent destructive lesion of the C2 vertebra. MRI shows a large lesion
arising from the posterior elements of C2 and comprising multiple cysts with fluid–fluid levels, with
preservation of the vertebral body. What is the most likely diagnosis?
A. aneurysmal bone cyst
B. giant cell tumor
C. chordoma
D. fibrous dysplasia
E. telangiectatic osteosarcoma
Source: Bydder, Megan, et al. Get Through Final FRCR Part A: SBAs for the Modular Examination. 1st
ed., Hodder Education Publishers, 2009.
Explanation:
Aneurysmal Bone Cysts
• Aneurysmal bone cysts are seen mainly in patients under 20 years of age (75%) and affect the
posterior elements when involving the spine.
• They may arise de novo, or secondary to another lesion such as a giant cell tumor (GCT) or fibrous
dysplasia.
Giant Cell Tumors (GCTs) and MRI Findings
• Both GCTs and telangiectatic osteosarcomas may cause cysts with fluid–fluid levels on MRI, but
GCTs arise from vertebral bodies and usually occur in the sacrum.
Telangiectatic Osteosarcomas
• Telangiectatic osteosarcomas usually affect long bones.
Chordomas
• Chordomas are malignant tumors that usually affect the vertebral body, with destruction and
invasion of the discs and adjacent structures.
93. Which CT finding would be more suggestive of chronic pancreatitis than ductal pancreatic
adenocarcinoma?
A. Common bile duct dilatation
B. Focal enlargement of the pancreatic head
C. Intraductal pancreatic calcification
D. Peripancreatic fat stranding and ascites
Source: McQueen, Andrew S., et al. Grainger & Allison’s Diagnostic Radiology: Single Best Answer
MCQs. 5th ed., Churchill Livingstone, 2009.
EBDR Exam MCQs & Concepts June 2025
Dr. Kareem Alnakeeb 37
94. In gastric carcinoma, which of the following typically makes tumor margins hardest to delineate on
CT/MRI?
A. Pancreatic infiltration
B. Perigastric lymph-node enlargement
C. Ascites
D. Liver metastases
Explanation: (by Perplexity AI, generated with the OpenAI O3 model)
• Direct invasion into the pancreas usually produces a contiguous soft-tissue interface that
actually helps identify loss of the fat plane, aiding recognition of T4 disease rather than obscuring the
primary lesion
• Nodal masses are generally separable from the gastric wall; they may crowd the region but seldom
merge imperceptibly with the tumor, so margins remain discernible.
• Free peritoneal fluid “bathes” the gastric serosa, effaces surrounding fat planes and equalizes soft-
tissue contrast, making it difficult to see the outer border or subtle serosal perforation, especially on
portal-venous-phase CT.
• Hepatic lesions are remote from the primary; they do not interfere with visualizing the gastric wall
itself.
95. Licked Candy Stick sign is seen in which condition?
A. Leprosy
B. Sarcoidosis
C. Hypertension
D. Madura foot (mycetoma)
Explanation: (by Perplexity AI, generated with the OpenAI O3 model)
• Resorption of distal phalanges in leprosy can produce tapered, smooth bone ends likened to a
“licked candy stick”.
• Osseous sarcoid more often shows lytic “lace-like” lesions rather than uniform tapering of bone tips.
• Systemic hypertension has no characteristic skeletal radiographic sign.
• Mycetoma causes multiple punched-out cavities or “dot-in-circle” sign on MRI, not the tapering seen
in the licked candy-stick appearance.
96. Which radiological feature is NOT seen in childhood scurvy?
A. Pelkan spur
B. Frankel’s line
C. Growth-arrest line
D. Zone of demarcation (Trümmerfeld lucent band)
Explanation: (by Perplexity AI, generated with the OpenAI O3 model)
• Metaphyseal spurs (“Pelkan spurs”) caused by healing micro-fractures are classic for scurvy.
• A dense provisional zone of calcification (“white line of Frankel”) is a hallmark sign.
• Transverse growth-arrest (Harris) lines reflect temporary slowing of endochondral growth seen after
severe systemic illness or therapy but are not characteristic of vitamin C deficiency; scurvy instead
shows excessive calcification at the provisional zone, not a halted metaphyseal front.
• The lucent band beneath Frankel’s line—the Trümmerfeld or “scorbutic zone of rarefaction/
demarcation”—is typical of scurvy
EBDR Exam MCQs & Concepts June 2025
Dr. Kareem Alnakeeb 38
97. Which of the following statements about idiopathic scoliosis is true?
A. Infantile scoliosis curves are usually to the right
B. Juvenile scoliosis curves are usually to the left
C. Idiopathic scoliosis is more common in boys
D. Idiopathic scoliosis is more common in girls
Explanation: (by Perplexity AI, generated with the OpenAI O3 model)
• Idiopathic curves presenting before age 3 (infantile) classically have a left-sided thoracic curve; a
right thoracic curve in this age group should raise suspicion of an underlying pathology.
• In children aged 4-10 years (juvenile idiopathic scoliosis) the predominant pattern is a right thoracic
curve, mirroring adolescent disease.
• Although very mild curves have similar incidence, progression-prone idiopathic scoliosis is overall
twice as frequent in girls, with the female:male ratio rising steeply with increasing Cobb angle.
• Female predominance is well documented across early-onset and adolescent groups; girls not only
present more often but have a much higher risk of curve progression that requires treatment
98. A chest radiograph of a 3-year-old child demonstrates marked right lower zone consolidation with a large
pneumatocele. A diagnosis of necrotizing pneumonia is made. What is the most likely causative
organism?
A. Staphylococcus aureus
B. Streptococcus pyogenes
C. Bordatella pertussis
D. Mycobacterium tuberculosis
E. Aspergillus
Source: Chin, Teck Yew, et al. Get Through Final FRCR 2A: SBAs. 1st ed., CRC Press, 2017.
Explanation:
Definition
• Pneumatoceles are thin-walled, air-filled intraparenchymal cysts that develop secondary to localized
bronchiolar and alveolar necrosis, which allow one-way passage of air into the interstitial space.
Common Associations
• They commonly occur in immunocompetent patients and are most commonly associated with S.
aureus, followed by Staphylococcus pneumoniae infections.
Mechanical Ventilation and Complications
• Although there is no clear correlation between the development of pneumatoceles and mechanical
ventilation, patients receiving mechanical ventilation have an increased risk for developing
complications related to pneumatoceles, including an increase in their size.
Genetic Factors
• Other than in hyperimmunoglobulin E syndrome, there is no known genetic or familial tendency for
pneumatoceles.
Prognosis
• The majority of pneumatoceles (more than 85%) resolve spontaneously, partially or completely over
weeks to months without clinical or radiographic sequelae.
EBDR Exam MCQs & Concepts June 2025
Dr. Kareem Alnakeeb 39
99. A 65-year-old man with history of stroke presents with chest pain. The chest radiograph shows a thin
curvilinear area of calcification in the lower part of left heart border. What is the likely site of
calcification?
A. Left atrium
B. Left ventricle
C. Right atrium
D. Left descending coronary artery
E. Mitral valve
Source: Gupta, Chaitanya. 300 Single Best Answers for the Final FRCR Part A. 1st ed., Jaypee UK,
2010.
Explanation:
• This is the typical site for left ventricular calcifications.
• Valvular calcifications are located within the heart.
• Coronary artery calcifications are seen along the upper part of left heart border and have a ‘tram-
track’ appearance.
100. All of the following conditions are recognized causes of bilateral sacroiliitis, except:
A. Whipple’s disease
B. Ankylosing spondylitis
C. Septic arthritis
D. Crohn’s disease
Explanation: (by Perplexity AI, generated with the OpenAI O3 model)
• Arthropathy in Whipple’s disease is typically migratory, intermittent and peripheral; axial involvement
is rare and, when present, more often unilateral rather than bilateral.
• Ankylosing spondylitis classically starts with bilateral, symmetrical sacroiliitis, which is the
radiographic hallmark of the disease.
• Although most infectious sacroiliitis presents unilaterally, about one-third of cases can affect both
sacroiliac joints, especially in hematogenous Staphylococcus or Brucella infections.
• Spondyloarthropathy associated with inflammatory bowel disease frequently manifests as bilateral
sacroiliitis, often independent of bowel activity
101. Fibrosis that is predominantly basal/lower-lobe on HRCT is associated with all of the following systemic
diseases except:
A. Rheumatoid arthritis (RA)
B. Systemic lupus erythematosus (SLE)
C. Systemic sclerosis (scleroderma)
D. Ankylosing spondylitis
Explanation: (by Perplexity AI, generated with the OpenAI O3 model)
• RA-associated interstitial lung disease typically shows lower-lobe UIP/NSIP patterns, with subpleural
reticulation and honeycombing.
• SLE interstitial lung disease is uncommon and, when present, more often shows patchy or upper-
lobe–dominant NSIP/organising pneumonia; basal fibrotic change is not a characteristic distribution.
• Scleroderma is a classic cause of basal-predominant fibrosis (NSIP/UIP) with traction bronchiectasis
and volume loss.
• Pulmonary involvement in ankylosing spondylitis classically produces apical fibrobullous disease;
however, any fibrosis that does develop outside the apices is still reported to spare the bases, so
lower-lobe fibrosis is not typical—but the key “exception” asked is clearly SLE.
EBDR Exam MCQs & Concepts June 2025
Dr. Kareem Alnakeeb 40
102. Which primary lung cancer subtype most frequently presents with cavitation on imaging?
A. Squamous cell carcinoma
B. Small-cell lung tumor
C. Adenocarcinoma
D. Large-cell lung tumor
Explanation: (by Perplexity AI, generated with the OpenAI O3 model)
• Cavitation is seen in up to 20–30% of squamous cell lung cancers, far exceeding other histologies.
Thick-walled, irregular cavities are typical.
• Small-cell carcinoma virtually never cavitates; lesions are usually solid and centrally located.
• Cavitation occurs in only ~2–5% of adenocarcinomas and is therefore much less common than in
squamous tumors.
• Cavitation is reported in ~6% of large-cell carcinomas—higher than adenocarcinoma but still far
below the squamous subtype
103. A 4-month-old boy presents with cyanosis. Examination reveals right ventricular heave and systolic
murmur. A chest radiograph shows a bulging right heart border and widening of the superior
mediastinum, creating a ‘snowman’ appearance. What is the most likely diagnosis?
A. Fallot’s tetralogy
B. total anomalous pulmonary venous return
C. partial anomalous pulmonary venous return
D. transposition of the great vessels
E. coarctation of the aorta
Source: Bydder, Megan, et al. Get Through Final FRCR Part A: SBAs for the Modular Examination. 1st
ed., Hodder Education Publishers, 2009.
Explanation:
Total anomalous pulmonary venous return
General presentation
• Total anomalous pulmonary venous return presents in the first year of life with cyanosis.
• It is due to failure of the pulmonary veins to drain into the left atrium, with drainage instead into
another vascular structure.
Supracardiac type I features
• There are four types – the commonest, type I (supracardiac), has the four pulmonary veins draining
into one common vein, the vertical vein, which drains into the left brachiocephalic vein.
• This dilated vein, along with the dilated left brachiocephalic vein and superior vena cava, causes
widening of the superior mediastinum, which is the ‘head’ of the ‘snowman’.
• The body is formed by enlargement of the right atrium, producing a rounded appearance of the lower
mediastinum.
• There may be increased pulmonary vascularity.
Partial anomalous pulmonary venous return
• Partial anomalous pulmonary venous return presents later in life with symptoms similar to atrial
septal defect.
Transposition of the great vessels
• Transposition of the great vessels presents in the first 2 weeks of life with cyanosis.
• Chest radiograph shows an ‘egg-on-string’ appearance of the mediastinum with increased
pulmonary vascularity.
Infantile-type coarctation
• Infantile-type coarctation presents with symptoms and signs of congestive heart failure in infancy.
• The classic figure-3 sign seen in coarctation is often hidden by the thymus in infants.
EBDR Exam MCQs & Concepts June 2025
Dr. Kareem Alnakeeb 41
104. A 73-year-old woman with previous history of myocardial infarction was referred for a chest radiograph by
her GP to exclude chest infection. No infective focus was identified but a focal bulge was noticed in the
left heart border, with curvilinear calcification along the edge. What is the diagnosis?
A. Myocardial calcification
B. Right atrial calcification
C. Mitral annulus calcification
D. Calcified vegetations
E. Left ventricular aneurysm calcification
Source: Chin, Teck Yew, et al. Get Through Final FRCR 2A: SBAs. 1st ed., CRC Press, 2017.
Explanation:
Cardiac False Aneurysm
• A cardiac false aneurysm is defined as a rupture of the myocardium that is contained by pericardial
adhesion.
• It usually represents a rare complication of myocardial infarction, but it may also occur after cardiac
surgery, chest trauma and endocarditis.
True vs False Left Ventricular Aneurysm
• True left ventricular aneurysms are discrete, dyskinetic areas of the left ventricular wall with a broad
neck.
• Unlike a true aneurysm, which contains some myocardial elements in its wall, the walls of a false
aneurysm are composed of organized hematoma and pericardium only.
• Both demonstrate focal bulge to the cardiac contour and can calcify.
Diagnostic Imaging
• Marked delayed enhancement of the pericardium on dynamic enhanced MRI may help in
differentiating a false aneurysm from a true one.
105. Adamantinoma is a rare low-grade malignant bone tumour; in which bone does it most commonly arise?
A. Femur
B. Fibula
C. Mandible
D. Radius
E. Tibia
Explanation: (by Perplexity AI, generated with the OpenAI O3 model)
• Around 80–90% of adamantinomas originate in the mid-diaphysis of the tibia, classically presenting
as an eccentric lytic lesion with cortical thinning.
• The fibula can be involved but usually secondarily or in multifocal disease, therefore less common
than the tibia.
• Femoral and radial occurrences are rare isolated case reports only.
• The mandible is the favoured site for odontogenic ameloblastoma; true long-bone adamantinoma is
histologically and genetically distinct, so mandibular involvement is not typical.
106. A 55-year-old woman develops right upper-quadrant pain and abnormal liver function tests six weeks
after an uncomplicated laparoscopic cholecystectomy. What is the most appropriate initial imaging
modality to investigate a suspected post-cholecystectomy bile duct problem?
A. Abdominal ultrasound
B. Contrast-enhanced CT abdomen
C. Endoscopic retrograde cholangiopancreatography (ERCP)
D. Magnetic resonance cholangiopancreatography (MRCP)
E. Hepatobiliary (HIDA) scintigraphy
EBDR Exam MCQs & Concepts June 2025
Dr. Kareem Alnakeeb 42
Explanation: (by Perplexity AI, generated with the OpenAI O3 model)
• Ultrasound is quick, widely available, radiation-free and inexpensive, making it the usual first-line
test after cholecystectomy. It can detect biliary dilatation, retained stones, intra-abdominal
collections and vascular complications, guiding whether more advanced studies are needed.
• MRCP (D) provides superior ductal detail but is typically reserved when ultrasound or CT raises
concern or is non-diagnostic.
• ERCP (C) is invasive and kept for therapeutic intervention.
• Contrast CT (B) offers a broad survey but involves radiation and intravenous contrast; it is often
second line.
• HIDA scintigraphy (E) is highly sensitive for active bile leaks but takes longer and is not the routine
starting test.
107. According to the Stanford classification of aortic dissection, which single anatomical feature
distinguishes a Type A dissection from a Type B dissection?
A. Primary intimal tear proximal to the left subclavian artery
B. Dissection confined to the descending thoracic aorta
C. Dissection involves the ascending aorta
D. Dissection extends below the renal arteries
E. Presence of partial thrombosis within the false lumen
Explanation: (by Perplexity AI, generated with the OpenAI O3 model)
• Type A in the Stanford scheme is defined by any involvement of the ascending aorta, irrespective of
where the entry tear lies or how far the dissection extends distally; surgical repair is usually required.
• Type B dissections spare the ascending aorta and are often managed medically or endovascularly.
• Option A is incorrect because the tear may originate distally yet propagate retrograde into the
ascending aorta, still making it Type A.
• Option B describes a pure descending aortic dissection—Type B.
• Option D concerns distal extent, which does not affect Stanford grouping.
• Option E relates to luminal status, not classification.
108. An MRI of the thoracic spine in a 32-year-old man with chronic back pain shows reduced intervertebral
disc height, irregular destruction of the opposing vertebral endplates and a smooth, lobulated
paravertebral soft-tissue mass with minimal surrounding oedema. What is the most likely diagnosis?
A. Degenerative disc disease
B. Metastatic vertebral collapse
C. Pyogenic spondylodiscitis
D. Tuberculous spondylitis
E. Osteoporotic compression fracture
Explanation: (by Perplexity AI, generated with the OpenAI O3 model)
• Tuberculous infection typically involves two contiguous vertebral bodies with intervening disc
narrowing, irregular endplate erosion and a “cold” paravertebral abscess that produces little
inflammatory reaction; these combined features are highly characteristic, making option D correct.
• Degenerative change (A) causes end-plate sclerosis, osteophytes and preserved marrow signal, not
soft-tissue abscess.
• Metastases (B) may collapse a vertebra but rarely cross the disc or form smooth abscesses.
• Acute pyogenic spondylodiscitis (C) also erodes endplates and narrows the disc but usually shows
marked marrow oedema, enhancing inflammatory pannus and systemic sepsis.
• Osteoporotic fracture (E) gives wedge or biconcave collapse without endplate erosion or
paravertebral collection.
EBDR Exam MCQs & Concepts June 2025
Dr. Kareem Alnakeeb 43
109. A non-contrast CT KUB shows a branching staghorn calculus filling the renal pelvis and calyces in a
patient with chronic Proteus urinary tract infection. Which stone composition is most likely?
A. Calcium oxalate monohydrate
B. Uric acid
C. Magnesium ammonium phosphate (struvite)
D. Cystine
E. Xanthine
Explanation: (by Perplexity AI, generated with the OpenAI O3 model)
• Infection with urease-producing organisms such as Proteus raises urinary pH and splits urea into
ammonium, promoting precipitation of magnesium, ammonium and phosphate ions. The resulting
magnesium ammonium phosphate (struvite) stones grow rapidly, often forming staghorn calculi that
appear radiodense on CT.
• Calcium oxalate stones (A) are the commonest overall but form in sterile, acidic urine.
• Uric acid stones (B) are radiolucent and occur in persistently acid urine, not alkaline infection.
• Cystine calculi (D) arise from a hereditary transport defect and have a characteristic hexagonal
crystal appearance.
• Xanthine stones (E) are rare and related to xanthine oxidase deficiency or allopurinol therapy; they
are not infection-related.
110. Enchondroma vs. Chondrosarcoma: classic location for Enchondroma is:
A. Hand and feet (small bones)
B. Pelvis
C. Ribs
D. Scapula
Explanation: (by Perplexity AI, generated with the OpenAI O3 model)
• Enchondroma is most common in the small bones of the hands and feet;
• chondrosarcoma is rare in these locations and more common in the pelvis and long bones.
111. A 45-year-old carpenter presents with a gradually enlarging, mildly tender swelling of the proximal
phalanx of the index finger. Hand radiographs show an expansile intramedullary lytic lesion with
endosteal scalloping that involves more than two-thirds of the cortical thickness but no cortical breach
or soft-tissue mass. Which diagnosis is most likely?
A. Bone infarct
B. Enchondroma
C. Low-grade (grade 1) chondrosarcoma
D. Osteoblastoma
E. Giant cell tumour
Explanation: (by Perplexity AI, generated with the OpenAI O3 model)
• Extensive (>⅔) endosteal scalloping in a cartilaginous lesion of the small bones strongly suggests
low-grade chondrosarcoma rather than a benign enchondroma; benign lesions usually cause little or
no scalloping and lack aggressive remodeling.
• Bone infarct shows serpiginous sclerosis without scalloping.
• Osteoblastoma produces a blastic nidus, not chondroid lucency.
• Giant cell tumour is eccentric, abuts the articular surface, and lacks chondroid matrix.
EBDR Exam MCQs & Concepts June 2025
Dr. Kareem Alnakeeb 44
112. A 55-year-old man undergoes staging CT for an exophytic gastric body mass suspected to be either
adenocarcinoma or a gastrointestinal stromal tumour (GIST). Which CT feature most reliably favours
gastric adenocarcinoma over GIST?
A. Large size (>5 cm) of the primary lesion
B. Central areas of low attenuation within the mass
C. Marked enhancement after intravenous contrast
D. Presence of perigastric lymph-node enlargement
E. Exophytic growth pattern with a claw-shaped gastric wall connection
Explanation: (by Perplexity AI, generated with the OpenAI O3 model)
• Gastric adenocarcinoma spreads early via lymphatics, so enlarged regional lymph nodes are
common and help distinguish it from GIST, which rarely involves lymph nodes (<5%); routine
lymphadenectomy is therefore unnecessary in GIST.
• Central low attenuation (necrosis) and strong enhancement can occur in both tumours, while size
and exophytic growth pattern overlap and are not discriminatory.
113. A 28-year-old primigravida undergoes her routine 20-week anomaly scan that shows mild unilateral
pyelectasis with an anteroposterior renal pelvic diameter of 5 mm. There are no other structural
abnormalities. What is the most appropriate next management step?
A. Immediate referral for fetal MRI
B. Amniocentesis for karyotyping
C. Repeat ultrasonography in 6 weeks
D. Elective delivery at 37 weeks
E. No further follow-up required
Explanation: (by Perplexity AI, generated with the OpenAI O3 model)
• Isolated mild pyelectasis (4–7 mm at 18–20 weeks) is usually a transient finding related to delayed
ureteric maturation; the majority resolve spontaneously.
• Current obstetric imaging guidelines recommend a single follow-up ultrasound 4–6 weeks later to
assess progression or resolution.
• Immediate MRI offers no added value, and invasive karyotyping is reserved for cases with additional
soft markers or risk factors.
• Elective pre-term delivery is unnecessary unless severe, progressive dilatation leads to
complications, and discharging without surveillance risks missing significant hydronephrosis that
may develop.
114. On routine 28-week fetal ultrasound, a male fetus shows bilateral renal pelvis dilatation measuring 11
mm in the anteroposterior plane. Which anteroposterior renal-pelvic diameter (APRPD) threshold at or
after 28 weeks’ gestation usually prompts postnatal urological assessment and possible intervention?
A. ≥5 mm
B. ≥7 mm
C. ≥10 mm
D. ≥15 mm
E. ≥20 mm
Explanation: (by Perplexity AI, generated with the OpenAI O3 model)
• At 28 weeks and beyond, an APRPD of 10 mm or greater is classified as severe
pyelectasis/hydronephrosis.
• This degree of dilatation carries a high risk of underlying pelvi-ureteric junction obstruction or vesico-
ureteric reflux and therefore triggers structured postnatal follow-up, antibiotic prophylaxis and, in
many centres, neonatal imaging with a view to possible surgery.
EBDR Exam MCQs & Concepts June 2025
Dr. Kareem Alnakeeb 45
• Smaller measurements (5 mm or 7 mm) are considered mild to moderate and often resolve
spontaneously; they are usually managed with serial antenatal scans rather than immediate
postnatal intervention.
• Diameters of 15 mm or 20 mm are uncommon and indicate marked obstruction, but the action line in
standard UK guidelines remains 10 mm.
115. An unenhanced CT abdomen shows acute right renal vein thrombosis. Which of the following
sonographic findings is most characteristic of this condition?
A. Small echogenic right kidney with cortical thinning
B. Large hypoechoic right kidney with loss of corticomedullary differentiation
C. Normal-sized kidney with multiple simple cortical cysts
D. Large echogenic kidney with prominent renal sinus fat
E. Small hypoechoic kidney with increased cortical vascularity
Explanation: (by Perplexity AI, generated with the OpenAI O3 model)
• Acute renal vein thrombosis causes venous outflow obstruction, leading to renal congestion and
interstitial oedema.
• On ultrasound this manifests as an enlarged, globally hypoechoic kidney in which the normal
corticomedullary differentiation is effaced, sometimes accompanied by reduced or absent venous
flow on Doppler—option B.
• Chronic thrombotic kidneys are typically small and echogenic (option A).
• Simple cortical cysts (option C) and prominent sinus fat (option D) are unrelated incidental findings.
• Increased cortical vascularity (option E) would be expected in acute pyelonephritis, not venous
thrombosis.
116. Which branch of the mandibular division forms a communicating loop with the facial nerve within the
parotid region, effectively linking cranial nerves V and VII?
A. Inferior alveolar nerve
B. Chorda tympani
C. Auriculotemporal nerve
D. Buccal nerve
E. Lingual nerve
Explanation: (by Perplexity AI, generated with the OpenAI O3 model)
• The auriculotemporal nerve (V3) ascends behind the mandibular neck, then crosses the parotid
gland where it gives sensory fibres to the gland and skin and sends multiple fine branches that
anastomose with the facial nerve, creating a direct connection between CN V and CN VII. It also
carries post-ganglionic secretomotor fibres from the otic ganglion to the parotid.
• Chorda tympani (B) is a branch of the facial nerve that joins, not links, with the lingual nerve to
convey taste and parasympathetic fibres; it is itself CN VII, not a V3 branch.
• Inferior alveolar (A), buccal (D) and lingual (E) nerves are V3 branches without significant facial-nerve
communications.
EBDR Exam MCQs & Concepts June 2025
Dr. Kareem Alnakeeb 46
117. A 7-year-old boy’s jaw radiograph shows loss of alveolar bone giving several incisors a ‘floating tooth’
appearance, and skull radiographs demonstrate multiple lytic calvarial lesions with bevelled edges;
which single diagnosis best explains these findings?
A. Eosinophilic granuloma
B. Osteomyelitis
C. Acute leukaemic infiltration
D. Papillon-Lefèvre syndrome
E. Metastatic neuroblastoma
Explanation: (by Perplexity AI, generated with the OpenAI O3 model)
• Eosinophilic granuloma, the osseous form of Langerhans cell histiocytosis, classically produces
sharply marginated lytic bone defects with asymmetric cortical involvement, giving the calvarial
“bevelled edge” sign and selective alveolar destruction that makes teeth appear to float.
• Osteomyelitis may erode cortex but usually shows periosteal reaction and lacks discrete bevelled
margins.
• Leukaemic infiltration causes diffuse, poorly defined medullary lucencies rather than focal punched-
out skull lesions.
• Papillon-Lefèvre syndrome creates generalized periodontitis without discrete calvarial lesions.
• Metastatic neuroblastoma can give skull “hair-on-end” appearance or sutural widening, but floating
teeth are uncommon and bevelled calvarial edges are not typical.
118. Rigler’s sign indicates:
A. Bowel ischemia
B. Free air on both sides of bowel wall (pneumoperitoneum)
C. Gallstone ileus
D. Pneumatosis intestinalis
119. On a supine abdominal radiograph the bowel wall is outlined on both its luminal and peritoneal surfaces
by gas (double-wall appearance). What approximate minimum volume of intraperitoneal free air is
usually required for this Rigler sign to be visible?
A. 50 mL
B. 100 mL
C. 200 mL
D. 500 mL
E. 1 000 mL
Explanation: (by Perplexity AI, generated with the OpenAI O3 model)
• Rigler sign appears only when a large pneumoperitoneum (>1 000 mL) surrounds gas-filled loops so
that gas lies on both sides of the bowel wall, accentuating it.
• Smaller volumes (50–500 mL) are often sufficient to create sub-diaphragmatic free air on an erect
chest film but do not envelope the bowel circumferentially, so the double-wall sign is absent.
• Therefore options A–D underestimate the amount of free air needed for Rigler sign.
EBDR Exam MCQs & Concepts June 2025
Dr. Kareem Alnakeeb 47
120. Which prenatal ultrasound finding is NOT typically associated with trisomy 21 (Down syndrome) in the
fetus?
A. Nuchal translucency thickening
B. Echogenic intracardiac focus
C. Duodenal atresia (“double-bubble” sign)
D. Choroid plexus cyst
E. Single umbilical artery
Explanation: (by Perplexity AI, generated with the OpenAI O3 model)
• Trisomy 21 commonly shows soft-marker findings such as increased nuchal translucency, echogenic
intracardiac focus, and duodenal atresia; a single umbilical artery can also be seen because
aneuploidies, including Down syndrome, are linked to umbilical-cord vessel anomalies.
• Choroid plexus cysts, however, are typically isolated or linked to trisomy 18, not trisomy 21; when
they appear without other abnormalities they do not raise the risk of Down syndrome, making option
D the exception.
121. Salpingitis isthmica nodosa is most commonly associated with which underlying pelvic condition that
causes tubal inflammation?
A. Endometriosis
B. Polycystic ovarian syndrome
C. Pelvic inflammatory disease
D. Uterine fibroids
E. Cervical carcinoma
Explanation: (by Perplexity AI, generated with the OpenAI O3 model)
• Salpingitis isthmica nodosa (SIN) represents nodular outpouchings of tubal epithelium into the
myosalpinx and is widely regarded as an acquired sequel of chronic tubal infection.
• Large hysterosalpingography and surgical series show that up to 90% of tubes demonstrating SIN
also have histologic or clinical evidence of pelvic inflammatory disease, supporting a strong post-
inflammatory aetiology.
• Chronic salpingitis therefore predisposes to the development of SIN, which in turn increases the
risks of infertility and ectopic pregnancy.
• Endometriosis and uterine fibroids may distort the pelvis but do not produce the characteristic
diverticular scarring of the isthmic tube.
• Polycystic ovarian syndrome and cervical carcinoma have no direct link to SIN.
122. A 68-year-old man with long-standing hypertension presents with sudden severe chest pain; acute aortic
intramural haematoma is suspected. Which imaging test is the modality of choice for initial assessment?
A. Transthoracic echocardiography
B. Transoesophageal echocardiography
C. Magnetic resonance angiography
D. Chest radiography
E. CT angiography
Explanation: (by Perplexity AI, generated with the OpenAI O3 model)
• CT angiography provides rapid, whole-aorta coverage with high spatial resolution, allowing confident
detection of intramural haematoma, dissection flaps and associated complications within minutes.
Multidetector scanners achieve sensitivities and specificities approaching 100%, and the study is
available 24/7 in most emergency departments.
• Transoesophageal echocardiography (B) is sensitive but semi-invasive and less comprehensive for
distal aorta.
• Magnetic resonance angiography (C) lacks speed and availability in unstable patients.
EBDR Exam MCQs & Concepts June 2025
Dr. Kareem Alnakeeb 48
• Transthoracic echocardiography (A) has limited acoustic windows for the thoracic aorta, and chest
radiography (D) is neither sensitive nor specific.
123. In children with β-thalassaemia major, premature fusion of the proximal humeral and other long-bone
epiphyses is most often a consequence of which pathological process?
A. Chronic iron chelation with desferrioxamine
B. Extramedullary haemopoietic marrow expansion within the bone
C. Endocrine iron overload causing hypothyroidism
D. Secondary hyperparathyroidism from chelation-induced zinc loss
E. Vitamin D deficiency osteomalacia
Explanation: (by Perplexity AI, generated with the OpenAI O3 model)
• Hyperactive, ineffective erythropoiesis in β-thalassaemia major markedly enlarges the marrow cavity.
The expanding marrow breaches cortex, extends sub-periosteally and weakens the metaphysis.
• Normal axial loading then compresses the medial physis, producing a Salter-Harris V-type injury that
triggers early physeal fusion and characteristic humeral varus deformity.
• Desferrioxamine can cause metaphyseal dysplasia but does not close the physis.
• Endocrine iron overload, secondary hyperparathyroidism and vitamin D deficiency produce
osteopenia or delayed rather than premature fusion, so options C–E are incorrect.
124. In the radionuclide technetium-99m used for routine gamma-camera imaging, what does the suffix “m”
signify?
A. Monoenergetic gamma emission
B. Metastable nuclear isomer
C. Marker that the isotope is generator-produced
D. Mean photon energy of 140 keV
E. Mass number rounded to the nearest integer
Explanation: (by Perplexity AI, generated with the OpenAI O3 model)
• The “m” in 99mTc denotes a metastable nuclear isomer, meaning the nucleus is in a long-lived
excited state that later decays to its ground state by emitting 140 keV gamma rays. This metastability
(half-life ≈ 6 h) allows sufficient time for imaging while keeping patient dose low.
• Option A is incorrect: many isotopes emit monoenergetic photons without the “m”.
• Option C confuses production method with nuclear state; generator production relates to parent
99Mo, not the “m”.
• Option D cites a characteristic gamma energy but does not define the suffix.
• Option E concerns atomic mass number; the mass is already given by “99”, independent of the “m”.
125. A teenager presents with painless bony swellings of the fingers, and radiographs show multiple expansile
intramedullary lucencies in the phalanges; soft-tissue films demonstrate numerous phlebolith-
containing haemangiomas. What is the most likely diagnosis?
A. Ollier disease
B. Hereditary multiple exostoses
C. Maffucci syndrome
D. McCune–Albright syndrome
E. Klippel–Trenaunay syndrome
EBDR Exam MCQs & Concepts June 2025
Dr. Kareem Alnakeeb 49
Explanation: (by Perplexity AI, generated with the OpenAI O3 model)
• Maffucci syndrome is defined by the combination of multiple enchondromas and soft-tissue
haemangiomas, often with phleboliths; this vascular component distinguishes it from Ollier disease,
which has enchondromas alone.
• Hereditary multiple exostoses produces osteochondromas, not enchondromas.
• McCune–Albright shows polyostotic fibrous dysplasia plus endocrine features and café-au-lait
macules, not haemangiomas.
• Klippel–Trenaunay causes limb over-growth with venous and lymphatic malformations but lacks
enchondromas; bone lesions, when present, are cortical hypertrophy rather than medullary cartilage
tumours.
126. An abdominal radiograph taken for sudden epigastric pain shows a single, markedly dilated loop of
proximal jejunum in the left upper quadrant—the classic “sentinel loop” appearance. Which underlying
condition is this sign most suggestive of?
A. Acute pancreatitis
B. Proximal small-bowel obstruction
C. Acute cholecystitis
D. Perforated peptic ulcer
E. Left ureteric colic
Explanation: (by Perplexity AI, generated with the OpenAI O3 model)
• The sentinel loop represents a localised adynamic ileus adjacent to an inflamed viscus. When it lies
in the left upper quadrant, it overlies the pancreas and therefore points to acute pancreatitis.
• In small-bowel obstruction (B) multiple dilated loops with air–fluid levels are typical rather than a
solitary segment.
• Acute cholecystitis (C) can produce a sentinel loop, but this would be in the right hypochondrium
near the gall-bladder, not the LUQ.
• Perforated peptic ulcer (D) usually shows pneumoperitoneum under the diaphragm.
• Ureteric colic (E) gives no specific bowel gas patterns.
127. According to the modified Hinchey classification of acute diverticulitis, what stage corresponds to
generalised faecal peritonitis?
A. Stage Ia
B. Stage Ib
C. Stage II
D. Stage III
E. Stage IV
Explanation: (by Perplexity AI, generated with the OpenAI O3 model)
• Stage IV represents perforated diverticulitis with faecal contamination of the peritoneal cavity,
necessitating urgent laparotomy and usually resection with stoma formation.
• Stage III is limited to purulent (but not faecal) peritonitis, while Stage II is a distant intra-abdominal or
pelvic abscess that can often be managed with percutaneous drainage.
• Stage Ib is a confined pericolic/mesocolic abscess, and Stage Ia is limited pericolic inflammation
without abscess.
• Recognising the correct stage guides the urgency and type of surgical versus radiological
intervention.
EBDR Exam MCQs & Concepts June 2025
Dr. Kareem Alnakeeb 50
128. MRI brain in a 6-year-old boy with worsening headaches shows marked dilatation of the lateral and third
ventricles, a thin web across the aqueduct of Sylvius and a normal-calibre fourth ventricle. What is the
most likely diagnosis?
A. Communicating hydrocephalus after meningitis
B. Dandy–Walker malformation
C. Aqueductal stenosis causing non-communicating obstructive hydrocephalus
D. Choroid plexus papilloma
E. Normal pressure hydrocephalus
Explanation: (by Perplexity AI, generated with the OpenAI O3 model)
• Triventricular enlargement with an intraluminal membrane at the aqueduct is classic for aqueductal
stenosis, the commonest form of non-communicating (obstructive) hydrocephalus in childhood.
CSF is blocked between the third and fourth ventricles, so the fourth remains normal.
• Communicating hydrocephalus (A) enlarges all four ventricles because the obstruction is distal to
the ventricular system.
• Dandy–Walker malformation (B) features a cystic dilatation of the fourth ventricle and enlarged
posterior fossa.
• A choroid plexus papilloma (D) would present as an enhancing intraventricular mass, often with
global ventricular enlargement.
• Normal pressure hydrocephalus (E) affects elderly patients, shows proportionate ventricular
expansion and an unobstructed aqueduct.
129. On sagittal MRI of the cranio-cervical junction, which imaging finding best distinguishes an Arnold-Chiari
type 1.5 malformation from a classic Chiari I malformation?
A. Inferior displacement of the cerebellar tonsils >5 mm below the foramen magnum
B. Presence of syringomyelia within the cervical spinal cord
C. Caudal descent of the medulla/obex through the foramen magnum in addition to tonsillar
herniation
D. Small posterior fossa with crowding of CSF spaces
E. Basilar invagination with platybasia
Explanation: (by Perplexity AI, generated with the OpenAI O3 model)
• Chiari 1.5 is considered an “advanced” form of Chiari I in which the brain-stem (typically the medulla
or obex) is pulled caudally below the foramen magnum along with the cerebellar tonsils; this brain-
stem descent (option C) is the key imaging discriminator.
• Tonsillar descent alone (option A) defines Chiari I and therefore is not specific.
• Syringomyelia (option B) can accompany either Chiari I or 1.5. A small posterior fossa (option D) is
common to several hind-brain crowding disorders but is not distinctive.
• Basilar invagination (option E) may coexist yet is neither necessary nor sufficient for the Chiari 1.5
diagnosis.
130. In cardiac CT reporting, what minimum percentage luminal narrowing typically defines a “significant”
coronary artery stenosis requiring further clinical correlation?
A. 30%
B. 50%
C. 70%
D. 80%
E. 90%
EBDR Exam MCQs & Concepts June 2025
Dr. Kareem Alnakeeb 51
Explanation: (by Perplexity AI, generated with the OpenAI O3 model)
• A diameter reduction of about 70% is generally accepted as the threshold at which a coronary
stenosis becomes haemodynamically significant and is therefore termed “significant” on CT or
invasive angiography.
• Below this, flow-limiting effects are less predictable: a 50% lesion may still be clinically silent, while
>70% frequently produces ischaemia.
• Higher cut-offs such as 80% or 90% describe critical or near-occlusive disease, not merely
“significant.”
• A 30% narrowing is considered mild and unlikely to impair perfusion.
131. A contrast-enhanced CT abdomen shows diffuse sheet-like soft-tissue infiltration of the greater
omentum producing an “omental cake”. Which primary malignancy most commonly gives this
appearance?
A. Gastric adenocarcinoma
B. Colon adenocarcinoma
C. Ovarian carcinoma
D. Pancreatic adenocarcinoma
E. Non-Hodgkin lymphoma
Explanation: (by Perplexity AI, generated with the OpenAI O3 model)
• Ovarian carcinoma spreads early by trans-coelomic seeding; tumour cells gravitate to the dependent
greater omentum, so diffuse omental thickening (“omental cake”) is classically and most frequently
linked to advanced ovarian cancer.
• Gastric and colonic adenocarcinomas can also seed the peritoneum but do so less often than
ovarian tumours.
• Pancreatic cancer typically causes peritoneal deposits later in the disease and far less frequently
involves the omentum diffusely.
• Lymphoma may infiltrate the omentum but such lymphomatous caking is rare because the omentum
lacks significant lymphoid tissue.
132. Which subtype of colonic polyp has the highest likelihood of progressing to invasive colorectal
carcinoma?
A. Hyperplastic polyp
B. Tubular adenoma
C. Tubulovillous adenoma
D. Villous adenoma
E. Inflammatory pseudopolyp
Explanation: (by Perplexity AI, generated with the OpenAI O3 model)
• Villous adenomas, defined by ≥75% villous architecture, have the greatest malignant potential
among colonic polyps. Reported transformation rates are 15–25% overall and approach 40% in
lesions >4 cm, reflecting high rates of dysplasia and a broad sessile growth pattern that facilitates
invasion.
• Tubular adenomas are far more common but carry only a 1–5% risk. Tubulovillous adenomas sit
between these extremes.
• Hyperplastic and inflammatory polyps are non-neoplastic and rarely, if ever, become cancerous.
• Hence, villous adenoma is the subtype most prone to malignant change while key distractors pose
lower or negligible risk.
EBDR Exam MCQs & Concepts June 2025
Dr. Kareem Alnakeeb 52
133. A 7-year-old girl undergoes abdominal ultrasound for recurrent urinary infections. A single, lobulated
renal mass is seen in the midline pelvis; CT confirms complete fusion of both kidneys without intervening
septum, with two non-crossing ureters draining separately into the bladder. What is the most likely
congenital anomaly?
A. Crossed fused renal ectopia
B. Horseshoe kidney
C. Pancake (shield) kidney
D. Solitary pelvic kidney
E. Duplex collecting system
Explanation: (by Perplexity AI, generated with the OpenAI O3 model)
• Pancake kidney is an extreme fusion anomaly in which the upper and lower poles of both kidneys
fuse completely, forming a flattened mid-pelvic mass. Each half retains its own pelvicalyceal system
and typically short, uncrossed ureters, fitting the CT description.
• Crossed fused ectopia (A) has one ureter crossing the midline to the orthotopic bladder orifice.
• Horseshoe kidney (B) shows fusion only at the lower poles with an isthmus and lies across the
lumbar spine, not entirely in the pelvis.
• A solitary pelvic kidney (D) is unfused and single.
• Duplex systems (E) involve duplicated pelvis/ureter but do not produce a single fused renal mass.
134. A 9-month-old boy presents with intermittent crying episodes, drawing up of the legs, and passage of
“red-currant jelly” stool. Abdominal ultrasound shows a transverse bowel segment with an outer
hypoechoic rim and central echogenic core, producing a“pseudokidney” (target) sign. What is the most
likely diagnosis?
A. Midgut volvulus
B. Meckel diverticulum
C. Intussusception
D. Hypertrophic pyloric stenosis
E. Hirschsprung disease
Explanation: (by Perplexity AI, generated with the OpenAI O3 model)
• The “pseudokidney” or target sign on ultrasound represents concentric bowel-within-bowel with
mesenteric fat and vessels, classic for ileocolic intussusception in infants.
• Midgut volvulus shows a whirlpool sign of twisted mesentery, not a target lesion.
• A Meckel diverticulum may cause bleeding or obstruction but lacks the characteristic ultrasound
appearance.
• Hypertrophic pyloric stenosis produces an elongated, thickened pylorus (“doughnut” sign) in the
epigastrium, not in the right abdomen.
• Hirschsprung disease is a colonic motility disorder diagnosed by contrast enema and rectal biopsy,
without a pseudokidney appearance on ultrasound.
135. Which of the following is NOT a recognised cause of pulsatile tinnitus in clinical practice?
A. Idiopathic intracranial hypertension
B. Glomus jugulare tumour
C. High-riding jugular bulb
D. Dural arteriovenous fistula
E. Vestibular schwannoma
EBDR Exam MCQs & Concepts June 2025
Dr. Kareem Alnakeeb 53
Explanation: (by Perplexity AI, generated with the OpenAI O3 model)
• Pulsatile tinnitus is almost always generated by conditions that alter or amplify vascular flow near
the temporal bone.
• Raised intracranial pressure in idiopathic intracranial hypertension produces turbulent venous flow;
glomus jugulare tumours and dural arteriovenous fistulas are highly vascular lesions that transmit
pulse-synchronous noise; and a high-riding jugular bulb abuts the middle ear, transmitting venous
flow sounds.
• In contrast, vestibular schwannoma is a benign nerve-sheath tumour that typically causes non-
pulsatile tinnitus by compressing the cochleovestibular nerve; it does not create haemodynamically
driven noise, so pulse synchrony is absent.
136. During a single-contrast barium meal, a lenticular pool of contrast with a convex inner margin (the
Carman meniscus sign) is seen within a large ulcer on the gastric body. What is the most likely diagnosis?
A. Benign peptic (non-malignant) gastric ulcer
B. Malignant gastric ulcer
C. Duodenal ulcer disease
D. Gastric diverticulum
E. Hiatal hernia
Explanation: (by Perplexity AI, generated with the OpenAI O3 model)
• The Carman meniscus sign represents barium trapped within an ulcer whose rigid, infiltrative
margins are produced by an underlying carcinoma; the convex inner edge forms a semilunar
“meniscus” that is highly specific for a malignant gastric ulcer.
• Benign ulcers usually have smooth folds reaching the crater and may show Hampton’s line, not a
meniscus.
• Duodenal ulcers (C) occur distal to the pylorus and do not display this sign.
• A gastric diverticulum (D) is an out-pouching without ulcer mound or heaped margins.
• Hiatal hernia (E) alters gastric position but does not create focal ulcerative masses or the
characteristic meniscus configuration.
137. Which coronary artery predominantly supplies the lateral wall of the left ventricle?
A. Left anterior descending artery
B. Right coronary artery
C. Circumflex artery
D. Posterior descending artery
E. Conus branch
Explanation: (by Perplexity AI, generated with the OpenAI O3 model)
• The lateral (obtuse) margin of the left ventricle receives its main blood supply from obtuse marginal
branches that arise from the circumflex artery, a branch of the left main stem.
• The left anterior descending artery mainly perfuses the anterior septum and anterior wall, not the
true lateral wall.
• The right coronary artery and its posterior descending branch chiefly supply the inferior wall and
posterior septum in right-dominant circulation.
• The conus branch is a small vessel from the right coronary artery that supplies the right ventricular
outflow tract, not the left ventricular free wall.
EBDR Exam MCQs & Concepts June 2025
Dr. Kareem Alnakeeb 54
• Therefore, only the circumflex artery consistently provides the principal arterial supply to the left
ventricular lateral wall.
138. Freiberg’s disease is characterized by avascular necrosis involving which part of the foot?
A. Calcaneal apophysis
B. Navicular bone
C. Second metatarsal head
D. Talus
E. Fifth metatarsal base
Explanation: (by Perplexity AI, generated with the OpenAI O3 model)
• Freiberg’s disease (Freiberg infraction) is osteonecrosis of the second metatarsal head, typically
presenting in adolescent females with forefoot pain and stiffness.
• Vascular compromise leads to subchondral collapse and fragmentation of the metatarsal head,
visible on radiographs and MRI.
• Option A describes Sever disease, an overuse apophysitis of the calcaneus, not true AVN.
• Option B is Köhler disease, AVN of the navicular.
• Option D refers to talar AVN, most often post-fracture.
• Option E (Jones fracture site) involves stress or acute fracture of the fifth metatarsal base, not
avascular necrosis.
139. Which imaging modality is most sensitive for detecting active lower gastrointestinal bleeding with flow
rates as low as 0.05–0.1 mL/min?
A. 99mTc-labelled red-blood-cell scintigraphy
B. Digital subtraction angiography
C. Contrast-enhanced multidetector CT angiography
D. Colonoscopy
E. 18F-FDG PET/CT
Explanation: (by Perplexity AI, generated with the OpenAI O3 model)
• 99mTc-labelled RBC scintigraphy can demonstrate active bleeding at very low rates (≈0.05–0.1
mL/min) because the radiolabelled cells pool at the haemorrhage site, allowing prolonged
acquisition and high sensitivity.
• Angiography requires faster bleeding (≈0.5–1 mL/min) to visualise contrast extravasation, while CT
angiography detects rates around 0.3–0.5 mL/min; both are therefore less sensitive than
scintigraphy.
• Colonoscopy often misses intermittent or slow bleeds and relies on direct visualisation rather than
flow rate.
• 18F-FDG PET/CT is not a recognised technique for acute GI haemorrhage localisation.
140. A 27-year-old man with haemoptysis and biopsy-proven anti-GBM (Goodpasture’s) disease undergoes
high-resolution CT of the chest; which imaging pattern is most typical of his pulmonary involvement?
A. Peripheral honeycombing of the lower lobes
B. Diffuse bilateral ground-glass opacification with relative subpleural sparing
C. Multiple centrilobular nodules with tree-in-bud spread
D. Thick-walled cavitary lesion in the upper lobe
E. Patchy mosaic attenuation with expiratory air-trapping
EBDR Exam MCQs & Concepts June 2025
Dr. Kareem Alnakeeb 55
Explanation: (by Perplexity AI, generated with the OpenAI O3 model)
• Goodpasture’s syndrome commonly causes diffuse alveolar haemorrhage.
• Fresh intra-alveolar blood increases parenchymal density, producing widespread ground-glass
opacities or air-space consolidation, usually bilateral and often sparing the extreme lung apices and
costophrenic angles.
• Honeycombing (A) reflects end-stage fibrosis, not haemorrhage.
• Centrilobular tree-in-bud nodules (C) suggest endobronchial infection.
• A cavitary focus (D) is characteristic of necrotising infection or tuberculosis.
• Mosaic attenuation (E) points to small-airway or vascular obstruction rather than hemorrhage.
141. Which imaging feature most strongly suggests a retroperitoneal lymphatic malformation on CT?
A. Well-defined multilocular cystic mass that displaces adjacent organs
B. Homogeneous low-attenuation fluid collection that invades the psoas muscle
C. Multilocular cystic lesion with fluid–fluid levels that insinuates between great vessels without
invasion
D. Solid enhancing mass arising from para-aortic soft tissues
E. Unilocular cyst with peripheral coarse calcification
Explanation: (by Perplexity AI, generated with the OpenAI O3 model)
• Retroperitoneal lymphatic malformations are benign multilocular cystic lesions that typically contain
simple or chylous fluid. They characteristically insinuate between retroperitoneal structures rather
than invading them, and may show fluid–fluid levels when complicated by hemorrhage or high-
protein chyle, helping differentiate them from other cystic or solid retroperitoneal masses. Option C
captures these key features.
• Option A lacks the classic insinuating growth pattern; most lymphatic malformations produce little
true mass effect.
• Option B describes invasive behavior, atypical for a lymphatic malformation.
• Option D depicts a solid enhancing tumor, more in keeping with lymphoma, sarcoma or
paraganglioma.
• Option E (unilocular calcified cyst) better fits hydatid, epidermoid or matured hematoma rather than
a multilocular lymphatic lesion.
142. In a multiphasic renal CT, a solid renal cortical mass measures 35 HU on the non-contrast scan, 120 HU
in the corticomedullary phase, 100 HU in the nephrographic phase and 80 HU on 5-minute excretory
images. Which renal tumour subtype is most likely?
A. Papillary renal cell carcinoma
B. Chromophobe renal cell carcinoma
C. Clear cell renal cell carcinoma
D. Mucinous tubular & spindle cell carcinoma
E. Fat-poor angiomyolipoma
Explanation: (by Perplexity AI, generated with the OpenAI O3 model)
• Clear cell RCC is typically hypervascular, showing brisk enhancement that peaks in the
corticomedullary phase (often >110 HU) and gradually washes out on later phases, exactly as in this
vignette.
• Papillary RCC (A) is hypovascular, seldom exceeding 70 HU even at peak and may be hypo- or iso-
attenuating to cortex throughout.
EBDR Exam MCQs & Concepts June 2025
Dr. Kareem Alnakeeb 56
• Chromophobe RCC (B) and mucinous tubular & spindle cell carcinoma (D) enhance less avidly than
clear cell and show lower peak HU values (~70 HU and ~55 HU respectively).
• Fat-poor angiomyolipoma (E) can mimic clear cell but usually displays lower early enhancement (80–
90 HU) and more rapid wash-out; presence of tumoral fat or homogeneous enhancement pattern
further aids differentiation.
Subtype Non-contrast HU Corticomedullary
(Arterial) phase
Nephrographic
(Venous) phase
Key Features
Clear cell RCC 20–40 HU† Strong, rapid
enhancement (70–
120+ HU),
heterogeneous
Loss of enhancement,
but still hypervascular
Most vascular; rapid,
intense enhancement
Papillary RCC 20–40 HU Mild or minimal,
homogeneous (50–60
HU); remains
hypoattenuating
Remains low, mild
delayed wash-in
Typically
hypovascular; most
homogeneous
Angiomyolipoma Often fat-density (-10
to -20 HU); may vary
May enhance but
usually identifiable by
fat
Variable Macroscopic fat is
diagnostic
† Values are approximate; literature shows attenuation varies, but ccRCC most often has low pre-contrast,
then strong post-contrast enhancement.
Pearl for FRCR 2A
• Clear cell RCC: Greatest, rapid, heterogeneous arterial enhancement (“fastest & brightest”)
• Papillary RCC: Mild, homogeneous enhancement, always < cortex and clear cell
• Angiomyolipoma: Macroscopic fat (very low HU) is diagnostic
143. On an unenhanced CT head following blunt trauma, which intracranial hematoma classically appears
biconvex and is limited by cranial suture lines?
A. Acute subdural hematoma
B. Intracerebral hematoma
C. Extradural (epidural) hematoma
D. Subarachnoid haemorrhage
E. Intraventricular haemorrhage
Explanation: (by Perplexity AI, generated with the OpenAI O3 model)
• An extradural (epidural) haematoma accumulates between the inner table of the skull and the
periosteal dura. Because the dura is firmly attached at the sutures, the collection cannot extend
beyond these lines, giving the characteristic lentiform (biconvex) shape on CT.
• Subdural haematomas lie beneath the dura, so they freely cross sutures but are limited by dural
reflections such as the falx; they therefore appear crescentic.
• Intracerebral, subarachnoid and intraventricular haemorrhages occur within brain parenchyma,
cisterns or ventricles respectively and are not constrained by sutures, so their distribution differs
from an extradural haematoma.
EBDR Exam MCQs & Concepts June 2025
Dr. Kareem Alnakeeb 57
144. A 29-year-old motorcyclist sustains a high-speed head injury. Non-contrast CT brain performed on arrival
shows a focal haemorrhage within the splenium of the corpus callosum with no significant cortical
contusions or skull fracture. Which primary mechanism best explains this haemorrhage distribution?
A. Direct coup–contrecoup contusion
B. Rupture of lenticulostriate perforators from acute hypertension
C. Shearing forces producing diffuse axonal injury
D. Venous bleeding secondary to basilar skull fracture
E. Microvascular fragility from cerebral amyloid angiopathy
Explanation: (by Perplexity AI, generated with the OpenAI O3 model)
• The corpus callosum is tethered between the falx and ventricular walls, so rapid deceleration or
rotational acceleration generates high shear strains across interhemispheric white matter. These
shearing forces tear small medullary vessels and axons, producing characteristic callosal
haemorrhage that often accompanies diffuse axonal injury.
• Direct cortical contusions (A) typically involve the frontal or temporal poles rather than deep midline
structures.
• Acute hypertensive vessel rupture (B) classically causes basal ganglia or thalamic bleeds.
• Basilar skull fractures (D) lead to petrous or posterior fossa venous haemorrhage, not isolated
callosal bleeds.
• Cerebral amyloid angiopathy (E) produces lobar cortical–subcortical haemorrhages in the elderly,
not midline injuries in young trauma patients.
145. On colour Doppler renal ultrasound, which artefact produces a rapidly alternating red–blue “mosaic”
posterior to a 10 mm hyperechoic focus, thereby confirming a renal calculus?
A. Aliasing artefact
B. Mirror-image artefact
C. Twinkle artefact
D. Blooming artefact
E. Reverberation artefact
Explanation: (by Perplexity AI, generated with the OpenAI O3 model)
• Twinkle artefact appears on colour Doppler as a chaotic red–blue signal immediately behind a rough,
highly reflective interface such as a renal stone; it is sensitive for calculi ≥5 mm and reliably
highlights a 10 mm calculus even when acoustic shadowing is subtle.
• Aliasing (A) results from velocities exceeding the Nyquist limit and occurs within true vascular flow,
not behind stationary stones.
• Mirror-image artefact (B) duplicates structures across strong reflectors like the diaphragm, not at the
site of a calculus.
• Blooming (D) is colour “bleed” from over-gain and obscures rather than localises stones.
• Reverberation artefact (E) produces multiple linear echoes from parallel reflectors and lacks the
distinctive colour mosaic.
146. Which renal lesion is most frequently encountered as a palpable abdominal mass in neonates?
A. Congenital mesoblastic nephroma
B. Wilms tumour
C. Multilocular cystic nephroma
D. Rhabdoid tumour of the kidney
E. Renal cell carcinoma
EBDR Exam MCQs & Concepts June 2025
Dr. Kareem Alnakeeb 58
Explanation: (by Perplexity AI, generated with the OpenAI O3 model)
• Congenital mesoblastic nephroma is the commonest renal tumour presenting in the neonatal
period, accounting for the majority of solid renal masses detected before three months of age. Its
benign nature and typical presentation as a unilateral solid mass make it the leading consideration
when a newborn has a renal lump.
• Wilms tumour is far more prevalent in toddlers and preschool children, not neonates.
• Multilocular cystic nephroma is cystic rather than predominantly solid and is much rarer.
• Rhabdoid tumour is aggressive but extremely uncommon, while renal cell carcinoma is exceptionally
rare in infancy and usually occurs in older children or adolescents.
147. According to PI-RADS v2.1, which MRI feature upgrades a prostate lesion from PI-RADS 4 to PI-RADS 5?
A. Low apparent diffusion coefficient <750 mm²/s
B. Marked early enhancement on dynamic contrast imaging
C. ≥1.5 cm maximum lesion diameter
D. Definite extraprostatic extension or invasive behavior
E. Contact with the prostatic capsule exceeding 10 mm
Explanation: (by Perplexity AI, generated with the OpenAI O3 model)
• PI-RADS v2.1 designates category 5 when a lesion already meeting PI-RADS 4 criteria is either ≥15
mm in greatest dimension or shows definite extraprostatic extension (EPE) or invasive features on
MRI.
• EPE/invasion (option D) carries the higher likelihood of clinically significant prostate cancer and
therefore triggers the upgrade.
• While a size ≥1.5 cm (option C) is an alternative route to PI-RADS 5, the question asks for the single
feature that upgrades beyond size alone; definite EPE/invasion is that distinguishing criterion.
• Low ADC (A), early enhancement (B) and capsular contact length (E) contribute to scoring but, on
their own, do not automatically raise a PI-RADS 4 lesion to category 5.
148. In prostate artery embolization performed for benign prostatic hyperplasia, what is the primary
therapeutic aim of the procedure?
A. Prevent malignant transformation of prostatic tissue
B. Shrink the prostate by reducing its arterial blood supply
C. Relieve bladder neck obstruction by stent placement
D. Ablate peri-urethral tissue with focused ultrasound
E. Deliver intra-arterial antibiotics to treat prostatitis
Explanation: (by Perplexity AI, generated with the OpenAI O3 model)
• Prostate artery embolization (PAE) involves catheter-directed delivery of embolic particles into
prostatic arterial branches, producing ischaemia and volume reduction of hyperplastic prostatic
tissue. The consequent prostate shrinkage alleviates lower urinary tract symptoms.
• Option C describes a urological stenting approach, not embolization; Option D refers to high-
intensity focused ultrasound therapy; Option E is irrelevant to BPH management; Option A is
incorrect because PAE targets symptom control, not cancer prevention.
EBDR Exam MCQs & Concepts June 2025
Dr. Kareem Alnakeeb 59
149. During prostatic artery embolisation to treat benign prostatic hyperplasia, the catheter is usually
introduced via which artery?
A. Common femoral artery
B. Radial artery
C. Brachial artery
D. External iliac artery
E. Inferior epigastric artery
Explanation: (by Perplexity AI, generated with the OpenAI O3 model)
• The standard access for prostatic artery embolisation is a percutaneous puncture of the common
femoral artery, allowing a straight course into the internal iliac arteries and their prostatic branches.
• The radial artery (B) is gaining popularity but still accounts for a minority of cases and requires longer
catheters with potentially higher radiation dose.
• The brachial artery (C) is rarely selected because of its small calibre and higher risk of spasm.
• Direct puncture of the external iliac artery (D) is not performed in routine practice; instead it is
reached intraluminally from the femoral puncture.
• The inferior epigastric artery (E) is a branch of the external iliac and is never used as an access site.
150. In uterine fibroid embolization, which vessel is selectively catheterised to deliver the embolic particles?
A. Ovarian artery
B. Internal pudendal artery
C. Inferior mesenteric artery
D. Uterine artery
E. Superior vesical artery
Explanation: (by Perplexity AI, generated with the OpenAI O3 model)
• Uterine fibroid embolization (also called uterine artery embolization) works by occluding the arterial
supply that feeds the fibroid plexus.
• An interventional radiologist advances a catheter into each uterine artery—branches of the anterior
division of the internal iliac—and injects calibrated particles until stasis is achieved, producing
selective fibroid infarction while preserving surrounding myometrium.
• The ovarian artery (A) is an occasional collateral but is not the routine target; internal pudendal (B)
and superior vesical (E) arteries supply pelvic organs other than the uterus, and the inferior
mesenteric artery (C) supplies the hind-gut.
151. On MRI evaluation of a stage IV bladder tumour, how does the muscularis propria typically appear?
A. Intact continuous low-signal band on T2-weighted images
B. Continuous low-signal band with focal early contrast enhancement
C. Complete disruption of the low-signal band with tumour extending into perivesical fat
D. Non-enhancing low-signal band on all sequences
E. Continuous intermediate-signal band with high ADC values on DWI/ADC
Explanation: (by Perplexity AI, generated with the OpenAI O3 model)
• Stage IV (≥T4) disease implies that tumour has breached the muscularis propria and invaded
perivesical fat or adjacent organs.
• Consequently, the normal hypointense muscularis band seen on T2WI is completely lost and
replaced by intermediate-signal tumour that protrudes beyond the bladder wall; dynamic contrast
images show early enhancement through the full thickness, and DWI shows high-signal tumour
outside the bladder.
EBDR Exam MCQs & Concepts June 2025
Dr. Kareem Alnakeeb 60
• Options A, B and D describe preserved or only focally disturbed muscle compatible with ≤T2 disease,
while option E depicts normal muscle signal characteristics rather than invasion.
152. A 3-day-old neonate presents with bilious vomiting and abdominal distension. Abdominal radiography
shows a markedly dilated stomach and proximal duodenum, while Doppler ultrasound demonstrates the
“whirlpool” sign of mesenteric vessels twisting around the superior mesenteric artery. What is the most
likely diagnosis?
A. Annular pancreas
B. Duodenal atresia
C. Hypertrophic pyloric stenosis
D. Jejunal atresia
E. Midgut volvulus
Explanation: (by Perplexity AI, generated with the OpenAI O3 model)
• Midgut volvulus occurs when malrotated mid-intestinal loops twist around the SMA, producing
proximal obstruction (dilated stomach/duodenum) and the pathognomonic whirlpool sign on colour
Doppler, making option E correct.
• Annular pancreas and duodenal atresia both cause bilious vomiting but classically show the
“double-bubble” without a whirlpool sign.
• Hypertrophic pyloric stenosis presents later (2–8 weeks) with non-bilious projectile vomiting and no
dilated duodenum.
• Jejunal atresia gives multiple dilated small-bowel loops (“triple-bubble” or beyond) rather than
isolated proximal dilatation and lacks vascular whirlpool.
153. Neonatal adrenal haemorrhage is more frequently seen on which side?
A. Bilateral equally
B. Left adrenal gland
C. Right adrenal gland
D. Depends on birth weight
E. Depends on gestational age
Explanation: (by Perplexity AI, generated with the OpenAI O3 model)
• The right adrenal gland is more commonly affected because its venous drainage is directly into the
inferior vena cava, making it vulnerable to pressure fluctuations during birth trauma, hypoxia or
sepsis.
• The left gland drains via the longer left adrenal vein into the renal vein, offering a relative pressure
buffer.
• Bilateral haemorrhage is possible but less common, while birth weight and gestational age influence
overall risk but not lateral predilection.
154. Anteroposterior long-leg radiographs of a 3-year-old with progressive bow-legs are reviewed; which one of
the following radiographic findings is NOT typical of Blount disease?
A. Proximal tibial metaphyseal beaking
B. Wedging of the medial tibial epiphysis
C. Tibial shaft valgus
D. Varus angulation of the tibia below the knee
E. Metaphyseal–diaphyseal angle greater than 11°
EBDR Exam MCQs & Concepts June 2025
Dr. Kareem Alnakeeb 61
Explanation: (by Perplexity AI, generated with the OpenAI O3 model)
• Blount disease is a growth disturbance of the medial proximal tibial physis that produces progressive
varus deformity at and just below the knee.
• Classic radiographic hallmarks include medial metaphyseal beaking, depression and wedging of the
medial epiphysis, and an increased metaphyseal–diaphyseal (Drennan) angle above 11°. Together
these lead to overall tibial shaft varus, not valgus.
• Therefore tibial shaft valgus is inconsistent with Blount disease.
• The other options describe well-recognised features seen on plain films in both infantile and
adolescent forms of the condition.
155. Which classification system is most widely used for categorising proximal femoral focal deficiency on
plain radiographs?
A. Amstutz–Wilson classification
B. Hamanishi classification
C. Aitken classification
D. Gillespie–Torode classification
E. Hillman classification
Explanation: (by Perplexity AI, generated with the OpenAI O3 model)
• The four-tier Aitken classification (classes A–D) is the standard radiographic system for proximal
femoral focal deficiency, describing severity by the presence and relationship of the femoral head,
neck and acetabulum; it guides prognosis and surgical planning.
• Later systems such as Amstutz–Wilson and Hamanishi are elaborations, while Gillespie–Torode and
Hillman focus on clinical groupings or specific imaging refinements, so they are less commonly
adopted in everyday practice.
EBDR Exam MCQs & Concepts October 2024
Dr. Kareem Alnakeeb 62
October 2024
1. A 40-year-old mother of two presents with a right lower abdominal lump near a surgical scar and with a
cyclical history of pain. Ultrasound shows a 2 cm solid hypoechoic lesion in the subcutaneous tissue.
Doppler shows internal vascularity. The most likely diagnosis is?
F. Desmoid tumor
G. Endometriosis
H. Metastasis
I. Lymph node
J. Suture granuloma
Source: Gupta, Chaitanya. 300 Single Best Answers for the Final FRCR Part A. 1st ed., Jaypee UK,
2010.
Explanation:
• Endometriosis can be found in surgical scars or needle tracts.
• Most cases of subcutaneous endometriosis occur in Pfannenstiel incisions.
• Abdominal wall endometriosis is thought to occur in up to 1% of cases.
• Clinically it presents as a cyclical painful lump and can arise many years after surgery.
2. A 74-year-old man with increased urinary frequency and hesitancy is found to have an enlarged prostate
on digital rectal examination. He is referred for a TRUS and biopsy. Which one of the following statements
best describes the TRUS findings of benign prostatic hypertrophy (BPH)?
A. Dense echogenic foci are seen at the margin of the peripheral and transitional zones.
B. The central zone is enlarged.
C. The peripheral zone is enlarged and appears homogeneously hypoechoic.
D. The peripheral zone is enlarged and is of mixed echogenicity.
E. The transitional zone is enlarged.
Source: McQueen, Andrew S., et al. Grainger & Allison’s Diagnostic Radiology: Single Best Answer
MCQs. 5th ed., Churchill Livingstone, 2009.
Explanation:
• The central zone atrophies with age while the transitional zone increases in size as it develops BPH.
• Peripheral zone enlargement is not a feature of BPH.
3. HSG shows small diverticular outpouchings in the isthmic portion of the right fallopian tube with distal
tube occlusion. What is the diagnosis?
A. Salpingitis isthmica nodosa (SIN)
B. Tubal polyps
C. Adenomyosis
D. Asherman’s syndrome
E. Ectopic pregnancy
Source: Leen, Edward. Succeeding in the New FRCR Part 2A Exam: Single Best Answer (SBA)
Revision Questions for Modules 1-6. 1st ed., UNKNO, 2011.
Explanation:
• SIN is associated with pelvic inflammatory disease and a higher risk of ectopic pregnancy.
EBDR Exam MCQs & Concepts October 2024
Dr. Kareem Alnakeeb 63
4. You are the radiology SpR consenting a 70-year-old man for a transrectal ultrasound (TRUS) prostate
biopsy. Which one of the following is not a recognized complication of this procedure?
A. Haematuria
B. Haematospermia
C. Perirectal bleeding
D. Pneumoperitoneum
E. Pain/discomfort post-procedure
Source: McQueen, Andrew S., et al. Grainger & Allison’s Diagnostic Radiology: Single Best Answer
MCQs. 5th ed., Churchill Livingstone, 2009.
Explanation:
• Almost all patients complain of pain/discomfort afterwards and up to 80% will experience either
hematuria or haematospermia.
• Perirectal bleeding (up to 37%), infection, vasovagal attack, urinary retention and epididymitis are
other recognized complications.
• Pneumoperitoneum should not occur because the prostate lies well below the peritoneal reflection.
5. Which of the following has the highest specificity for Non-Accidental Iniury (NAI)?
A. Scapula fracture
B. Vetebral fractures
C. Complex skull fracture
D. Digital fracture
E. Epiphyseal separation injuries
Source: Leen, Edward. Succeeding in the New FRCR Part 2A Exam: Single Best Answer (SBA)
Revision Questions for Modules 1-6. 1st ed., UNKNO, 2011.
Explanation:
• B-E are moderately specific. Other high specificity signs are classic metaphyseal fractures, rib
fractures, and sternal fractures.
6. A 37-year-old man with AIDS presents with confusion, lethargy and memory loss. CT of the brain
demonstrates multiple supratentorial enhancing masses. Which imaging feature favors a diagnosis of
toxoplasmosis rather than primary CNS lymphoma?
A. subependymal distribution
B. lesions hyperdense on unenhanced CT
C. lesion size .3 cm
D. hypovascularity on MR perfusion study
E. increased uptake of thallium-201 on SPECT
Source: Bydder, Megan, et al. Get Through Final FRCR Part A: SBAs for the Modular Examination. 1st
ed., Hodder Education Publishers, 2009.
Explanation:
Toxoplasmosis
• Toxoplasmosis is the most common cause of a cerebral mass lesion in patients with AIDS.
• Typical appearances are of multiple, hypoattenuating, ,2 cm lesions with a predilection for the basal
ganglia.
Lymphoma
• Lymphoma is the second commonest mass lesion, with characteristic features including hyperdense
lesions (though less frequently than in non-AIDS lymphoma) in a periventricular location with
subependymal spread.
Common Imaging Features
• Lesions in both conditions may show solid or ring enhancement.
• Hemorrhage is unusual in lymphoma but may be seen in toxoplasmosis, particularly following
treatment.
EBDR Exam MCQs & Concepts October 2024
Dr. Kareem Alnakeeb 64
Thallium Scanning
• Thallium scanning may be useful to distinguish the two if there is diagnostic uncertainty.
• CNS lymphoma is thallium avid whereas toxoplasmosis does not show uptake.
MR Perfusion Studies
• MR perfusion studies may also help to differentiate the two conditions.
• Lymphomas demonstrate increased perfusion relative to surrounding tissue, while toxoplasmosis is
hypovascular.
Clinical Significance
• Differentiation is important, as early radiation therapy confers a significant survival advantage in CNS
lymphoma.
7. Which is the cause of a cystic rather than a hemorrhagic cause of brain metastases?
A. Adenocarcinoma of the lung
B. Malignant melanoma
C. Choriocarcinoma
D. Renal cell carcinoma
E. Thyroid carcinoma
Source: Leen, Edward. Succeeding in the New FRCR Part 2A Exam: Single Best Answer (SBA)
Revision Questions for Modules 1-6. 1st ed., UNKNO, 2011.
Explanation:
• Squamous cell lung cancer and adenocarcinoma of the lung cause cystic metastasis to the brain.
• Answers B-E are causes of hemorrhagic metastases (Mnemonic: MR CT BB)
o M: melanoma
o R: renal cell carcinoma
o C: choriocarcinoma
o T: thyroid carcinoma, teratoma
o B: bronchogenic carcinoma
o B: breast carcinom
8. Which of the following best describes imaging changes in a colloid cyst?
A. Typically hypodense on non-contrast CT
B. Appears high SI on T1
C. Appears low SI on T2
D. Commonly widens septum pellucidi
E. Most commonly causes symmetrical enlargement of lateral ventricles
Source: Leen, Edward. Succeeding in the New FRCR Part 2A Exam: Single Best Answer (SBA)
Revision Questions for Modules 1-6. 1st ed., UNKNO, 2011.
Explanation:
• Protein content/paramagnetic effect of magnesium Mg2+/
calcium Ca2+
/, Iron Fe, in a cyst cause
increased T1 and T2 SI. Colloid cysts appear iso/hyperdense on NCCT.
• They can occasionally widen septum pellucidum and cause asymmetrical enlargement of the lateral
ventricles.
EBDR Exam MCQs & Concepts October 2024
Dr. Kareem Alnakeeb 65
9. Which of the following features favor Rathke’s cleft cyst rather than craniopharyngioma?
A. Absence of calcification
B. Cystic element on MR
C. Involvement of suprasellar and sellar regions
D. Enhancement of the wall
E. High signal intensity on T1
Source: Leen, Edward. Succeeding in the New FRCR Part 2A Exam: Single Best Answer (SBA)
Revision Questions for Modules 1-6. 1st ed., UNKNO, 2011.
Explanation:
• Rathke’s cleft cysts do not calcify. They affect women to men in a 2:1 ratio and adults from 40-60
years of age. They cause variable MR appearances depending on protein content of cyst. They can
rarely show enhancement.
10. Which of the following is an extra-axial posterior fossa tumor in adults?
A. Choroid plexus papilloma
B. Metastasis
C. Haemangioblastoma
D. Lymphoma
E. Glioma
Source: Leen, Edward. Succeeding in the New FRCR Part 2A Exam: Single Best Answer (SBA)
Revision Questions for Modules 1-6. 1st ed., UNKNO, 2011.
Explanation:
• Other extra-axial posterior fossa masses include acoustic neuroma, meningioma, chordoma and
epidermoid.
11. Which is the most common site of metastatic spread in medulloblastoma?
A. Axial skeleton
B. Lymph nodes
C. Lung
D. Subarachnoid space
E. Liver
Source: Leen, Edward. Succeeding in the New FRCR Part 2A Exam: Single Best Answer (SBA)
Revision Questions for Modules 1-6. 1st ed., UNKNO, 2011.
Explanation:
• Subarachnoid space is the most common, with drop metastases occurring in 40%.
12. Following a large postpartum hemorrhage, a 25-year-old woman develops a severe headache and
sudden visual field defect. What is the most likely diagnosis?
A. intracerebral hemorrhage
B. reversible posterior leukoencephalopathy
C. subarachnoid hemorrhage
D. Sheehan’s syndrome
E. vertebral artery dissection
Source: Bydder, Megan, et al. Get Through Final FRCR Part A: SBAs for the Modular Examination. 1st
ed., Hodder Education Publishers, 2009.
Explanation:
Hypertension in Pregnancy
• Many of the acute neurological conditions of pregnancy occur with rising blood pressure.
Sheehan’s Syndrome and Hypotension
• Sheehan’s syndrome results from haemorrhage-induced hypotension causing pituitary infarction.
• Early on, this appears as an enlarged homogeneous pituitary with low T1 signal, high T2 signal and
post-contrast ring enhancement.
EBDR Exam MCQs & Concepts October 2024
Dr. Kareem Alnakeeb 66
• Later, there is an empty sella.
• Clinical manifestations include visual field loss, headache, ophthalmoplegia and pituitary
dysfunction (diabetes insipidus).
• Hemorrhage, sepsis and pulmonary embolism cause hypotension that can cause watershed
infarction as well as Sheehan’s syndrome.
Reversible Posterior Leukoencephalopathy
• Reversible posterior leukoencephalopathy produces cortical blindness, headaches, confusion and
seizures.
• Those affected are often taking immunosuppressant treatment.
• Imaging features can be identical to eclampsia, peripartum cerebral angiopathy and hypertensive
encephalopathy, but with a posterior predominance.
• On CT, there is low attenuation change.
• On MRI, there is high signal on T2W/FLAIR images.
• ADC maps can differentiate between likely reversible vasogenic oedema (high signal on ADC map
showing unrestricted diffusion) and cytotoxic oedema (low signal due to restricted diffusion), which
is more likely to progress to infarct.
Microangiopathic Haemolytic Anaemias
• Microangiopathic haemolytic anaemias, such as thrombotic thrombocytopenic purpura and
haemolytic uraemic syndrome, give widespread ischaemia/infarction and haemorrhagic
transformation.
Vasculitis in Pregnancy
• There is no increased risk in pregnancy of vasculitis such as systemic lupus erythematosus,
Takayasu’s syndrome or Moyamoya syndrome.
Vascular Malformations and Aneurysms
• Arteriovenous malformation is no more likely to bleed in pregnancy, but there is an increased risk
with arterial aneurysms.
13. A 53-year-old man has an MRI of his pelvis as a staging investigation for bladder cancer. The request card
also states that the prostate is mildly enlarged on digital rectal examination and the serum prostate
specific antigen (PSA) level is borderline elevated. The reporting radiologist reviews the prostate in detail.
Which one of the following statements best describes the MRI findings of a normal prostate gland?
A. A On Tlw images, the central zone is of higher signal intensity than the peripheral zone.
B. On Tlw images, the central zone is of lower signal intensity than the peripheral zone.
C. On T2w images, the peripheral zone is of lower signal intensity than the central and transitional zones
D. The peripheral zone is of higher signal intensity than the central zone on T2w images.
E. The seminal vesicles are hypointense on T2w images
Source: McQueen, Andrew S., et al. Grainger & Allison’s Diagnostic Radiology: Single Best Answer
MCQs. 5th ed., Churchill Livingstone, 2009.
EBDR Exam MCQs & Concepts October 2024
Dr. Kareem Alnakeeb 67
14. An elderly male patient presents with signs suggesting acute middle cerebral artery infarction. Around 21
2 hours after symptom onset, an unenhanced CT of the brain is performed. Among other subtle signs, the
basal ganglia are obscured. Reduced perfusion through which of the following vessels best explains this
sign?
A. lenticulostriate arteries
B. anterior choroidal artery
C. callosomarginal artery
D. recurrent artery of Heubner
E. angular artery
Source: Bydder, Megan, et al. Get Through Final FRCR Part A: SBAs for the Modular Examination. 1st
ed., Hodder Education Publishers, 2009.
Explanation:
Lenticulostriate arteries
• The lenticulostriate arteries are vessels arising from the M1 segment of the middle cerebral artery;
there are medial and lateral groups.
• Collectively, they supply the thalamus, caudate and lentiform nuclei.
Anterior cerebral artery branches
• The callosomarginal artery and the recurrent artery of Heubner are anterior cerebral artery branches.
• The latter provides some supply to the anterior limb of the internal capsule, and parts of the caudate
nucleus and globus pallidus.
Middle cerebral artery cortical branch
• The angular artery is a cortical branch of the middle cerebral artery.
Anterior choroidal artery
• The anterior choroidal artery also supplies parts of the internal capsule and basal ganglia but is a
branch of the internal carotid artery.
Basal ganglia anatomy
• The nuclei of the basal ganglia are the amygdala, claustrum, lentiform and caudate nuclei, with the
internal, external and extreme capsules being associated white matter tracts.
15. An 80-year-old man presents acutely with a dense hemiplegia. CT perfusion is performed soon after
admission, which suggests that the entire involved arterial territory is beyond recovery. Which of the
following options represents the most likely combination of cerebral blood flow, mean transit time and
cerebral blood volume, respectively, seen within the affected brain parenchyma, compared with
unaffected parenchyma?
A. increased, increased, increased
B. increased, increased, decreased
C. increased, decreased, decreased
D. decreased, decreased, decreased
E. decreased, increased, decreased
Source: Bydder, Megan, et al. Get Through Final FRCR Part A: SBAs for the Modular Examination. 1st
ed., Hodder Education Publishers, 2009.
Explanation:
Clinical applications
• Cerebral perfusion CT can distinguish viable but ischemic tissue (the penumbra) from tissue that is
beyond recovery.
• Other uses include:
o evaluation of vasospasm after subarachnoid hemorrhage,
o assessment of cerebrovascular reserve with acetazolamide (cerebral arteriole vasodilator)
in cases of vascular stenosis,
EBDR Exam MCQs & Concepts October 2024
Dr. Kareem Alnakeeb 68
o evaluation of collateral flow and cerebrovascular reserve in patients having temporary
balloon occlusion and
o assessment of microvascular permeability of intracranial neoplasms.
Central volume principle
• Cerebral perfusion CT utilizes the central volume principle.
• This states that CBF=BV/MTT, where CBF is cerebral blood flow, CBV is cerebral blood volume and
MTT is mean transit time.
CT perfusion techniques
• In practice, two CT perfusion techniques can be used.
o One is perfused-blood volume mapping, in which a quantity is assigned to cerebral blood
volume by subtracting unenhanced CT data from CT angiographic data.
▪ It has the advantage of imaging the whole brain.
o The second technique is a dynamic, contrast-enhanced technique that acquires data from
a limited number of axial slices, and monitors the first pass of an iodinated contrast agent
bolus through the cerebral circulation.
▪ This requires an unenhanced CT brain, followed by a dynamic CT performed during
injection of 50 ml of iodinated contrast (300 mg I/ml) at 4 ml/s.
▪ The first pass of contrast is observed in the brain.
Perfusion measurement and maps
• Cerebral perfusion is related to the concentration of iodinated contrast, which is directly related to
the attenuation measured.
• Several maps are produced, including the CBV, CBF and MTT.
o MTT is derived from arterial and venous enhancement curves, measured by using regions of
interest placed on an artery (one that is not occluded as part of an acute event) and a venous
sinus.
o CBV is the area under the enhancement curves, and
o CBF is obtained from the central volume equation.
Differentiating infarct and penumbra
• Differentiation of infarcted brain from penumbra is important because, while penumbra can be saved
by timely thrombolysis, infarcted tissue has an increased risk of bleeding from thrombolysis with no
chance of recovery.
• CBF is decreased in both ischemia and infarction,
• MTT is longer (.6 s) in both, while
• CBV is decreased in infarct but increased (or normal) in the penumbra due to cerebral autoregulatory
mechanisms.
Key parameters:
• MTT is the most sensitive for stroke.
• So this or CBF can be used to detect stroke while CBV is used to determine whether there is infarct or
reversible ischemia.
EBDR Exam MCQs & Concepts October 2024
Dr. Kareem Alnakeeb 69
16. Which is most likely to represent an intramedullary mass lesion?
A. Ependymoma
B. Meningioma
C. Neurofibroma
D. Arachnoid cyst
E. Abscess
Source: Leen, Edward. Succeeding in the New FRCR Part 2A Exam: Single Best Answer (SBA)
Revision Questions for Modules 1-6. 1st ed., UNKNO, 2011.
Explanation:
• Other intramedullary masses include astrocytomas are dermoids (lipoma/teratomas), acutely
expanding infarcts and haematoma.
17. A 38-year-old male with Human Immunodeficiency Virus (HIV) stopped taking his retrovirals 6 months
ago and now presents with confusion. CT brain shows non-enhancing hypodensities, with apparent
dilated perivascular spaces, although these were not present on a CT brain from 2 years ago. What is the
most likely cause?
A. Cryptococcus
B. Progressive multifocal leukoencephelopathy
C. Tuberculosis
D. CMV encephalitis
E. Toxoplasmosis
Source: Leen, Edward. Succeeding in the New FRCR Part 2A Exam: Single Best Answer (SBA)
Revision Questions for Modules 1-6. 1st ed., UNKNO, 2011.
Explanation:
• More commonly cryptococcus meningitis but cryptococcus or gelatinous pseudocysts reside in
dilated perivascular spaces.
18. A 30-year-old male with recurrent Transient Ischemic Attacks (TIAs) and a history of migraine with aura
undergoes CT brain. Subcortical infarcts are identified raising suspicion of cerebral autosomal dominant
arteriopathy with subcortical infarcts (CADASIL). Which is the most characteristically involved location
for subcortical infarcts?
A. Anterior temporal pole
B. Frontal lobe
C. Centrum semiovale
D. Deep grey matter structure
E. Pons
Source: Leen, Edward. Succeeding in the New FRCR Part 2A Exam: Single Best Answer (SBA)
Revision Questions for Modules 1-6. 1st ed., UNKNO, 2011.
Explanation:
• A young patient with migraines, auras, TIAs or subcortical strokes should raise suspicion of CADASIL.
• Subcortical infarcts are characteristically in the anterior temporal pole and external capsule but may
involve C, D and E.
EBDR Exam MCQs & Concepts October 2024
Dr. Kareem Alnakeeb 70
19. A 65-year-old woman is referred for a pelvic radiograph to investigate intermittent right hip pain. The
radiograph shows thin lucent lines within both inferior pubic rami. Which radiographic feature would
support a diagnosis of osteoporotic fracture rather than osteomalacia?
A. Callus formation
B. Failure to extend across the entire width of the bone
C. Sclerotic margin to lucencies
D. Similar appearances within the proximal femora
E. Symmetrical appearance
Source: McQueen, Andrew S., et al. Grainger & Allison’s Diagnostic Radiology: Single Best Answer
MCQs. 5th ed., Churchill Livingstone, 2009.
20. Considering Moyamoya disease in adults:
A. Infarct seen in cortical/subcortical areas
B. Multiple small flow voids are characteristic
C. Affects anterior circulation
D. Presentation with ischemia is more common in adults than in children
E. The supraclinoid MCA is spared
Source: Leen, Edward. Succeeding in the New FRCR Part 2A Exam: Single Best Answer (SBA)
Revision Questions for Modules 1-6. 1st ed., UNKNO, 2011.
Explanation:
• Multiple flow voids are due to hypertrophied lenticulostriate arteries.
• Children more often present with ischemia and infarct in cortical/subcortical areas.
• Adults more often present with hemorrhage than children, but when infarcts do occur, they are most
often in the deep white matter.
• The disease can involve the posterior circulation.
• The supraclinoid MCA is the first to be involved.
21. A 36-year-old woman presents with primary infertility. A transabdominal ultrasound shows a normal
anteverted uterus and bilateral adnexal masses. A subsequent MRI shows bilateral high signal ovarian
masses on both T1w and T2w sequences. On fat-suppressed Tlw images, the lesions remain high signal.
What is the most likely diagnosis?
A. Bilateral dermoid cysts
B. Bilateral endometriomas
C. Bilateral ovarian fibromas
D. Bilateral theca lutein cysts
E. Polycystic ovaries
Source: McQueen, Andrew S., et al. Grainger & Allison’s Diagnostic Radiology: Single Best Answer
MCQs. 5th ed., Churchill Livingstone, 2009.
Explanation:
• Theca lutein cysts contain straw-colored fluid which is low signal on T1w and high signal on T2w
images.
EBDR Exam MCQs & Concepts October 2024
Dr. Kareem Alnakeeb 71
22. An 18 year old man undergoes a Tc MDP bone scan to investigate pain in the right hip. A ‘hot’ lesion is
seen in the right proximal femur. No other lesions are seen. Which of the following lesions would appear
as ‘hot’ on a Tc MDP bone scan?
A. Osteopoikilosis
B. Fibrous cortical defect
C. Acute fracture within 12 hours of injury
D. Fibrous dysplasia
E. Haemangioma
Source: Currie, Stuart, et al. SBAs for the FRCR 2A. 1st ed., Cambridge University Press, 2010.
Explanation:
• The most common site of monostotic fibrous dysplasia is the ribs, followed by proximal femur and
craniofacial bones. Three-quarters of cases present before age 30.
• Other benign lesions causing a ‘hot’ on bone scan include Paget’s disease, brown tumours,
aneurysmal bone cysts, osteoid osteoma and chondroblastoma.
• Acute fractures are not usually ‘hot’ until after the first 24–48 hours.
23. A premature baby of a diabetic mother delivered by caesarean section develops tachypnoea soon after
birth. Chest radiographs show hyperinflated lungs with prominent interstitial markings and prominent
horizontal fissure. These changes resolved after 3 days. The most likely diagnosis is?
A. Respiratory distress syndrome
B. Meconium aspiration syndrome
C. Transient tachypnoea of the newborn
D. Left heart failure
E. Normal lung of newborn
Source: Gupta, Chaitanya. 300 Single Best Answers for the Final FRCR Part A. 1st ed., Jaypee UK,
2010.
Explanation:
• If the processes of clearing amniotic fluid from the lungs is impaired in a new born transient
tachypnoea of the newborn develops. This is associated with prematurity, caesarean section and
diabetic mothers. These are typical radiographic features, which resolve in 2–3 days.
24. Which of the following indicates T3 rather than T4 lung cancer?
A. Invasion of the oesophagus
B. Invasion of the trachea
C. Invasion of the pericardium
D. Malignant pleural effusion
E. Invasion of the vertebral body
Source: Leen, Edward. Succeeding in the New FRCR Part 2A Exam: Single Best Answer (SBA)
Revision Questions for Modules 1-6. 1st ed., UNKNO, 2011.
Explanation:
• T3 disease features include a tumor of any size less than 2cm from the carina, invasion of the
parietal pleura, chest wall, diaphragm, mediastinal pleura, pericardium, pleural effusion or satellite
nodule in the same lobe.
• T4 disease is characterized by invasion of the heart, great vessels, trachea, oesophagus, vertebral
body, carina or the presence of a malignant pleural effusion.
• The TNM staging system was updated in 2009 (AJR, 2010).
EBDR Exam MCQs & Concepts October 2024
Dr. Kareem Alnakeeb 72
25. A 70-year-old patient undergoes a staging CT for renal cell carcinoma, which shows ligamentous
ossification extending from the fifth to ninth thoracic vertebrae. There is relative sparing of the left side of
the vertebrae and disc space height is preserved. The apophyseal and sacroiliac joints appear normal.
What is the most likely diagnosis?
A. Ankylosing spondylitis
B. Degenerative disc disease
C. Diffuse idiopathic skeletal hyperostosis
D. Metastatic disease
E. Ossification of the posterior longitudinal ligament
Source: McQueen, Andrew S., et al. Grainger & Allison’s Diagnostic Radiology: Single Best Answer
MCQs. 5th ed., Churchill Livingstone, 2009.
26. A 75-year-old lady undergoes bone mineral density (BMD) measurements at the hip and spine by means
of dual energy radiograph absorptiometry (DXA). What findings would satisfy the World Health
Organisation (WHO) criteria for osteoporosis?
A. BMD below the young adult reference mean
B. BMD between —1 and —2.5 standard deviations below that of the young adult reference mean
C. BMD more than —2.5 standard deviations below the young adult reference mean
D. BMD more than —2.5 standard deviations below the young adult reference mean, with one low-
energy fracture
E. BMD more than —2.5 standard deviations below the young adult reference mean, with two low-
energy fractures
Source: McQueen, Andrew S., et al. Grainger & Allison’s Diagnostic Radiology: Single Best Answer
MCQs. 5th ed., Churchill Livingstone, 2009.
27. A series of neonatal radiographs reveal a narrow thorax with short ribs, square iliac wings with horizontal
acetabular roofs, short sacrosciatic notches, progressive narrowing of the interpedicular distance and
posterior scalloping of the vertebral bodies. What is the most likely diagnosis?
A. Achondroplasia
B. Campomelic dysplasia
C. Cleidocranial dysplasia
D. Ellis-van Creveld syndrome
E. Morquio's syndrome
Source: McQueen, Andrew S., et al. Grainger & Allison’s Diagnostic Radiology: Single Best Answer
MCQs. 5th ed., Churchill Livingstone, 2009.
Explanation:
• The iliac wings in Morquio's syndrome are characteristically flared rather than square.
28. A 33 year old male with no significant past medical history presents with headache, drowsiness and
confusion. CT shows a hypodense lesion with a smooth regular wall centred over the left lentiform
nucleus. There is surrounding oedema and mass effect with effacement of the ipsilateral Sylvian fissure.
On T2-weighted MR imaging, the lesion is hyperintense and is surrounded by a hypointense rim and
hyperintense oedema. There is peripheral enhancement post-contrast injection, and diffusion-weighted
imaging demonstrates restricted diffusion within the lesion. What is the most likely diagnosis?
A. Glioblastoma multiforme
B. Pyogenic abscess
C. Toxoplasmosis
D. Lymphoma
E. Metastasis
Source: Currie, Stuart, et al. SBAs for the FRCR 2A. 1st ed., Cambridge University Press, 2010.
Explanation:
• The differential diagnosis for a solitary ring-enhancing lesion of the brain includes (‘MAGICAL DR’):
Metastasis; Abscess; Glioma/Glioblastoma multiforme; Infarction; Contusion; AIDS
EBDR Exam MCQs & Concepts October 2024
Dr. Kareem Alnakeeb 73
(toxoplasmosis); Lymphoma (often AIDS-related); Demyelinating disease; Resolving
haematoma/Radiation necrosis.
• Classically, abscesses are located at the corticomedullary junction in the frontal and temporal
lobes.
o The most common causative organism is Streptococcus.
o The wall is generally smooth and regular with relative thinning of the medial wall secondary
to a poorer blood supply from white matter (neoplastic lesions usually have a thick, nodular,
irregular rim).
o In this scenario, the enhancing, T2-hypointense rim suggests abscess.
o Restricted diffusion is also highly suggestive of an abscess.
• Lymphoma may be hyperdense on CT due to a high nuclear-to-cytoplasmic ratio and typically shows
solid homogeneous enhancement in immunocompetent patients.
29. A 34-year-old man with chronic back pain is referred by his GP for thoracic and lumbar spine radiographs.
The GP is concerned about the possibility of ankylosing spondylitis. Which radiological feature is atypical
for ankylosing spondylitis, and might suggest an alternative diagnosis?
A. Ankylosis of the apophyseal joints
B. Anterior longitudinal ligament calcification
C. Osteophyte formation
D. Sclerosis of the anterior corners of the vertebrae
E. Vertebral body squaring
Source: McQueen, Andrew S., et al. Grainger & Allison’s Diagnostic Radiology: Single Best Answer
MCQs. 5th ed., Churchill Livingstone, 2009.
Explanation:
• Syndesmophytes, rather than osteophytes, are characteristic features of ankylosing spondylitis.
• They are differentiated from osteophytes by their vertical orientation, as they represent ossification of
the outer border of the annulus fibrosus.
• Progression and maturation of the syndesmophytes result in a ‘bamboo spine’.
30. An incidental finding on plain film is a 2-cm lucency within the diaphysis of the right humerus, which
exhibits chondroid calcification. Which clinical or radiological feature would favour a diagnosis of
chondrosarcoma rather than enchondroma?
A. Age less than 20 years
B. Circular, curvilinear or nodular calcific densities
C. Periosteal reaction
D. Slow growth
E. Well-defined round or elliptical margin
Source: McQueen, Andrew S., et al. Grainger & Allison’s Diagnostic Radiology: Single Best Answer
MCQs. 5th ed., Churchill Livingstone, 2009.
Explanation:
• Outside the hands and feet, chondrosarcoma is five times more common than enchondroma
EBDR Exam MCQs & Concepts October 2024
Dr. Kareem Alnakeeb 74
31. An 80-year-old man undergoes skeletal scintigraphy for multifocal skeletal pain, malaise and weight loss.
The scintigram shows diffusely increased activity throughout die skeleton, with absent renal activity.
What is the most likely diagnosis?
A. Metastatic bladder cancer
B. Metastatic colon cancer
C. Metastatic gastric cancer
D. Metastatic lung cancer
E. Metastatic prostate cancer
Source: McQueen, Andrew S., et al. Grainger & Allison’s Diagnostic Radiology: Single Best Answer
MCQs. 5th ed., Churchill Livingstone, 2009.
32. A previously well 80-year-old woman sustains a subcapital fracture of the right neck of femur following a
fall onto hard ground. The plain film reveals multiple lytic lesions within the pelvic bones and proximal
femora, which are highly suspicious for bone metastases. What is the most likely occult primary lesion?
A. Carcinoma of the bladder
B. Carcinoma of the breast
C. Carcinoma of the bronchus
D. Carcinoma of the colon
E. Carcinoma of the stomach
Source: McQueen, Andrew S., et al. Grainger & Allison’s Diagnostic Radiology: Single Best Answer
MCQs. 5th ed., Churchill Livingstone, 2009.
33. A solitary, lytic lesion with aggressive features is an unexpected incidental finding on radiography of the
left knee. Which radiological feature would favor a diagnosis of metastasis rather than primary bone
tumour?
A. Bone expansion
B. Diaphyseal location
C. Florid periosteal reaction
D. Tumour bone formation
E. Soft tissue mass
34. A 24-year-old man is referred to the gastroenterology outpatient clinic. He describes intermittent bloody
diarrhoea with abdominal pain and has lost 5 kg in weight over the past 6 months. His father and uncle
both have inflammatory bowel disease. Routine laboratory investigations are remarkable for a
moderately elevated CRP. A double contrast barium enema examination is performed. Which of the
following findings would be more consistent with Crohn's disease than ulcerative colitis?
A. Aphthous ulceration interspersed with areas of normal mucosa
B. Fine granular appearance of the descending and sigmoid colon
C. Isolated involvement of the rectum and sigmoid
D. Shortening and narrowing of the entire colon with absence of haustral folds
E. The presence of ‘collar button’ ulceration
Source: McQueen, Andrew S., et al. Grainger & Allison’s Diagnostic Radiology: Single Best Answer
MCQs. 5th ed., Churchill Livingstone, 2009.
Explanation:
• Aphthous ulceration is the earliest sign of Crohn's disease on a double contrast barium enema.
• The other options are all true of ulcerative colitis—from the earliest signs of fine mucosal granularity
to the ‘lead pipe’ appearance of the colon in chronic UC.
• Submucosal ulceration can extend laterally in UC giving the ‘collar button’ appearance.
EBDR Exam MCQs & Concepts October 2024
Dr. Kareem Alnakeeb 75
35. A 70-year-old man complains of a tense painless swelling posterior to his right knee. Ultrasound
demonstrates a large cyst, which communicates with the knee joint between which two structures?
A. Through the interval between semimembranosus and the lateral head of gastrocnemius
B. Through the interval between semimembranosus and the medial head of gastrocnemius
C. Through the interval between semimembranosus and semitendinosus
D. Through the interval between semitendinosus and the lateral head of gastrocnemius
E. Through the interval between semitendinosus and the medial head of gastrocnemius
Source: McQueen, Andrew S., et al. Grainger & Allison’s Diagnostic Radiology: Single Best Answer
MCQs. 5th ed., Churchill Livingstone, 2009.
36. An 80-year-old man has been admitted to hospital with shortness of breath and a productive, purulent
cough. A CXR reveals left lower lobe consolidation. Which additional radiological finding is most likely to
suggest a diagnosis of Klebsiella pneumoniae rather than Legionella pneumophildi
A. Bulging fissures
B. Mediastinal lymphadenopathy
C. Pleural effusion
D. Pneumothorax
E. Septal thickening
Source: McQueen, Andrew S., et al. Grainger & Allison’s Diagnostic Radiology: Single Best Answer
MCQs. 5th ed., Churchill Livingstone, 2009.
Explanation:
• Klebsiella pneumoniae leads to an extensive exudative response leading to cavitating lobar
consolidation and bulging fissures.
• Legionnaire's disease, on the other hand, tends to present with multifocal lobar, homogeneous
opacities with a tendency to appear like masses.
37. Regarding MRI examination of the shoulder, what are the signal characteristics of the normal
supraspinatus tendon?
F. High signal intensity on all sequences
G. High signal on T1w, low signal on T2w
H. Intermediate signal on all sequences
I. Low signal on all sequences
J. Low signal on T1w, high signal on T2w
Source: McQueen, Andrew S., et al. Grainger & Allison’s Diagnostic Radiology: Single Best Answer
MCQs. 5th ed., Churchill Livingstone, 2009.
38. A 66-year-old man undergoes screening for colorectal cancer and is found to have two positive stool
samples for fecal occult blood. The patient is asymptomatic with no significant medical history. He is
referred for CT colonography (CTC). Which one of the following statements is correct regarding CTC?
A. As much as 0.5—1% of examinations result in colonic perforation.
B. A routine examination should involve supine imaging only.
C. Significant extracolonic pathology is identified in 30-40% of symptom-
D. The administration of intravenous contrast (portal venous imaging) is advised for asymptomatic
patients, as it improves the detection of colonic
E. The use of an antispasmodic (eg Buscopan) immediately prior to gas insufflation enables
optimal colonic distension.
Source: McQueen, Andrew S., et al. Grainger & Allison’s Diagnostic Radiology: Single Best Answer
MCQs. 5th ed., Churchill Livingstone, 2009.
Explanation:
• CT colonography (CTC) is accepted as a generally safe technique with a reported perforation rate in
symptomatic patients of 0.03% (compared with 0.13% with optical colonoscopy).
EBDR Exam MCQs & Concepts October 2024
Dr. Kareem Alnakeeb 76
• Supine and prone imaging is widely advocated in CTC to maximize colonic distension and
discriminate between fecal/fluid bowel residue and genuine pathology, while the use of an
antispasmodic is advised to avoid colonic spasm.
• There is no strong evidence that intravenous contrast improves detection of colonic lesions, but it
may be of benefit in symptomatic patients as it enables a more accurate assessment of extracolonic
pathology.
39. A 74-year-old man presents with an 8-week history of altered bowel habit and rectal bleeding. A flexible
sigmoidoscopy demonstrates a malignant stricture in the rectum and biopsies confirm rectal
adenocarcinoma. An MRI is performed and shows an annular neoplasm at 12 cm. The mass invades 4
mm beyond the rectal wall into the perirectal fat and infiltrates the peritoneal reflection anteriorly. There
is a small volume of free peritoneal fluid. What is the radiological T stage?
A. TX
B. T1
C. T2
D. T3
E. T4
Source: McQueen, Andrew S., et al. Grainger & Allison’s Diagnostic Radiology: Single Best Answer
MCQs. 5th ed., Churchill Livingstone, 2009.
Explanation:
• Evidence of peritoneal invasion indicates stage T4 rectal cancer.
40. A male patient is referred to the on-call surgical team with a 3-day history of generalized abdominal pain
and vomiting. The patient has not opened his bowels for 2 days. Examination reveals a distended
abdomen with increased bowel sounds. An abdominal radiograph is performed and demonstrates a large
dilated loop of large bowel with several loops of dilated small bowel centrally. Which other feature would
make a diagnosis of caecal volvulus more likely than that of sigmoid volvulus?
A. Haustrae are visible in the gas-filled viscus.
B. The apex of the viscus lies in the left upper quadrant.
C. The patient is 75 years old.
D. The patient is in long-term institutional care.
E. The viscus rises above the level of the T10 vertebral body.
Source: McQueen, Andrew S., et al. Grainger & Allison’s Diagnostic Radiology: Single Best Answer
MCQs. 5th ed., Churchill Livingstone, 2009.
Explanation:
• The presence of haustrations in a dilated viscus and gas in the appendix are key to the diagnosis of
caecal volvulus.
EBDR Exam MCQs & Concepts October 2024
Dr. Kareem Alnakeeb 77
41. A 23-year-old woman complains of episodes of diarrhoea and rectal bleeding. Her father died of
colorectal cancer aged 39. A double contrast barium enema is performed and demonstrates more than
one hundred small polyps, measuring up to 5 mm in size, throughout the colon. An upper GI endoscopy
demonstrates multiple polypoid lesions in the stomach and duodenum. What is the most likely
diagnosis?
A. Carcinoid syndrome
B. Familial adenomatous polyposis
C. Hereditary non-polyposis colorectal cancer
D. Juvenile polyposis
E. Peutz-Jeghers syndrome
Source: McQueen, Andrew S., et al. Grainger & Allison’s Diagnostic Radiology: Single Best Answer
MCQs. 5th ed., Churchill Livingstone, 2009.
Explanation:
• Autosomal dominant condition with multiple colonic adenomas and 100% risk of colorectal
carcinoma 20 years after diagnosis. Associated with hamartomas in the stomach, gastric and
duodenal adenomas and periampullary carcinoma.
42. A 35-year-old woman is referred to the Radiology Department following the birth of her first child. The
baby was delivered 8 days post-term and was a vaginal delivery following a prolonged labour and
episiotomy. Two months later, the patient continues to experience faecal incontinence and an anal
sphincter tear is suspected. Which investigation would be most useful to demonstrate anal sphincter
damage?
A. Barium evacuation proctogram
B. CT colonography
C. CT with rectal contrast media
D. Endoanal ultrasound
E. MRI of the pelvis with a body coil
Source: McQueen, Andrew S., et al. Grainger & Allison’s Diagnostic Radiology: Single Best Answer
MCQs. 5th ed., Churchill Livingstone, 2009.
Explanation:
• High-frequency endosonography allows an accurate assessment of the four layers of the anal wall;
superficial and deep mucosa, submucosa and muscularis propria.
EBDR Exam MCQs & Concepts October 2024
Dr. Kareem Alnakeeb 78
43. A 64-year-old woman presents with bloating and vague pelvic pain and is referred for a pelvic ultrasound.
On transabdominal ultrasound, she is found to have a large right adnexal mass. Which one of the
following sonographic findings would indicate that this mass is more likely to be malignant than benign?
A. Doppler waveform with a high resistive index (> 0.8)
B. Homogeneously hypoechoic mass with posterior acoustic enhancement
C. Multiple septations that are approximately 1 mm thick
D. Papillary projections
E. Size > 4 cm
Source: McQueen, Andrew S., et al. Grainger & Allison’s Diagnostic Radiology: Single Best Answer
MCQs. 5th ed., Churchill Livingstone, 2009.
44. A 60-year-old nulliparous woman presents with postmenopausal bleeding. On transvaginal ultrasound,
her endometrium is 8 mm thick and the endomyometrial junction appeared indistinct. The radiologist
suspects invasive endometrial cancer and refers her for an MRI examination. What are the likely findings
on MRI?
A. On unenhanced T1w images the endometrial cancer appears of high signal intensity compared to the
surrounding myometrium.
B. On contrast-enhanced T1w images, endometrial cancer shows avid enhancement compared with
surrounding myometrium.
C. On T2w images the normally high signal junctional zone is disrupted.
D. T1w fat-saturated sequences are best used to assess the junctional zone.
E. The endometrial cancer demonstrates delayed/little enhancement compared to the normal
surrounding myometrium on postcontrast T1w images.
Source: McQueen, Andrew S., et al. Grainger & Allison’s Diagnostic Radiology: Single Best Answer
MCQs. 5th ed., Churchill Livingstone, 2009.
45. Dynamic contrast-enhanced CT may be used to characterize adrenal lesions. Which one of the following
statements best describes the imaging characteristics of a primary adrenal carcinoma on portal venous
phase (70 s) and subsequent delayed phase (15 min) contrast-enhanced CT images?
A. Early washout on delayed images
B. No measurable enhancement in either phase
C. Poor early enhancement, with an increase in enhancement on delayed images
D. Washout by greater than 80%, compared with the early postcontrast
E. Washout of less than 40% on delayed images, compared with the portal venous phase images
Source: McQueen, Andrew S., et al. Grainger & Allison’s Diagnostic Radiology: Single Best Answer
MCQs. 5th ed., Churchill Livingstone, 2009.
Explanation:
• Malignant lesions have abnormally high vascular density leading to slower flow and increased
microvascular permeability. This translates to longer transit times for intravenous contrast within
malignant adrenal lesions, compared with simple adenomas.
EBDR Exam MCQs & Concepts October 2024
Dr. Kareem Alnakeeb 79
46. A 19-year-old female student presents with acute abdominal pain, elevated CRP and a low-grade
temperature. On clinical examination, there is tenderness to light palpation in the right iliac fossa and the
patient is febrile. A graded compression ultrasound examination is performed. Which one of the following
statements is true?
A. A transverse appendiceal diameter of 5 mm is diagnostic of acute appendicitis.
B. The finding of a pelvic fluid collection makes a diagnosis of acute appendicitis unlikely.
C. The presence of hyperechoic fat in the right iliac fossa makes a diagnosis of acute appendicitis
unlikely.
D. The sensitivity of graded compression ultrasound in suspected acute appendicitis is 75—90%.
E. The specificity of graded compression ultrasound in suspected acute appendicitis is 35—50%.
Source: McQueen, Andrew S., et al. Grainger & Allison’s Diagnostic Radiology: Single Best Answer
MCQs. 5th ed., Churchill Livingstone, 2009.
Explanation:
• Graded compression ultrasound of the appendix can avoid unnecessary surgery and ionizing
radiation—particularly relevant for children and women of childbearing age.
• The finding of a noncompressible appendix with transverse diameter of 6 mm or greater is highly
suggestive of acute appendicitis (specificity 86-100%).
• Other ultrasound findings include hyperechoic fat in the right iliac fossa, periappendiceal fluid or a
pelvic fluid collection (appendiceal abscess).
47. A 79-year-old woman trips and falls whilst stepping off a bus. She suffers from a fractured right neck of
femur and undergoes a hemiarthroplasty the following day. Her early recovery is complicated by
bronchopneumonia which resolves after 5 days of broad-spectrum antibiotics. On her tenth day in
hospital she develops abdominal pain and diarrhea and pseudomembranous colitis is suspected
clinically. Which one of the following statements is true regarding pseudomembranous colitis?
A. A normal abdominal CT effectively excludes pseudomembranous colitis.
B. Ascites is present in up to 40% of patients.
C. CT carries a low positive predictive value for pseudomembranous colitis.
D. Extensive pericolonic stranding is a typical feature on CT.
E. The rectum is not involved in 40—50% of patients.
Source: McQueen, Andrew S., et al. Grainger & Allison’s Diagnostic Radiology: Single Best Answer
MCQs. 5th ed., Churchill Livingstone, 2009.
Explanation:
• Ascites can occur with other colitides, but is often seen in pseudomembranous colitis.
• CT typically demonstrates mucosal enhancement and marked colonic wall thickening but only mild
pericolonic stranding, in patients with pseudomembranous colitis. These findings have a high
positive predictive value but a normal CT does not exclude pseudomembranous colitis.
• Rectal sparing occurs in around 10% of patients.
EBDR Exam MCQs & Concepts October 2024
Dr. Kareem Alnakeeb 80
48. A 22-year-old pregnant woman (30 weeks' gestation) presents with right flank pain. She has an abdominal
ultrasound which shows dilatation of the right pelvicalyceal system. Which one of the following
additional findings would suggest a diagnosis of mechanical ureteric obstruction rather than pregnancy-
related dilatation?
A. An elevated resistive index (RI)
B. Decreased corticomedullary differentiation
C. Hyperechoic renal parenchyma
D. Renal parenchymal thinning
E. Ureteric and pelvicalyceai dilatation
Source: McQueen, Andrew S., et al. Grainger & Allison’s Diagnostic Radiology: Single Best Answer
MCQs. 5th ed., Churchill Livingstone, 2009.
Explanation:
• Mechanical obstruction is associated with elevation of the RI.
49. A 40-year-old woman has a 15-year history of ulcerative colitis (UC). After the initial diagnosis, she
suffered frequent exacerbations of colitis requiring several hospital admissions. She declined surgical
intervention at that stage and has subsequently been well controlled on medical management. Recently,
she has developed a change in bowel habit and a double contrast barium enema is performed. This
shows a stricture in the descending colon. Which one statement is true regarding strictures in ulcerative
colitis?
A. Abrupt shouldering is typical of a benign stricture in UC.
B. In patients with UC, colorectal carcinomas typically arise from tubular adenomas.
C. The majority of strictures in UC are benign.
D. There is no increased risk of colorectal carcinoma in patients with UC.
E. Widening of the presacral space is pathognomonic of a rectal carcinoma.
Source: McQueen, Andrew S., et al. Grainger & Allison’s Diagnostic Radiology: Single Best Answer
MCQs. 5th ed., Churchill Livingstone, 2009.
Explanation:
• Benign strictures in UC are typically smooth and symmetrical and are due to chronic smooth muscle
hypertrophy. These occur in 10—20% of patients with UC and are most common in the left colon.
• Carcinomas arise from dysplastic changes within the diseased epithelium and not from adenomas
as in the general population.
156. The following statements concerning esophageal carcinoma are true:
A. 90% of cases are squamous cell carcinomas
B. Most commonly located in the upper third of the esophagus
C. Plummer-Vinson syndrome is a recognised predisposing factor
D. Commonest appearance on double contrast barium swallow is of a large ulcer within a bulging mass
E. It is associated with ulcerative colitis
Source: Hussain, Shahid, et al. Radiology MCQs for the New FRCR Part 2A. 1st ed., TFM Publishing,
2006.
Explanation:
A. True
B. False - 20% in the upper third, 30-40% middle third and 30-40% in lower third
C. True
D. False - polypoid/fungating form is commonest
E. False - predisposing factors include Barrett’s oesophagus, alcohol abuse, smoking, coeliac disease,
achalasia, tylosis
EBDR Exam MCQs & Concepts October 2024
Dr. Kareem Alnakeeb 81
50. Regarding diverticular disease, which is False?
A. Colonic diverticulosis affects 70-80% by 80 years of age
B. 10-25% of individuals with colonic diverticular disease develop diverticulitis
C. Rectosigmoid colon is most commonly affected
D. Fistula formation occurs in 40-50% of cases complicating acute diverticulitis
E. Moderate diverticulitis is present when the bowel wall is thickened >3 mm
Source: Hussain, Shahid, et al. Radiology MCQs for the New FRCR Part 2A. 1st ed., TFM Publishing,
2006.
Explanation:
A. True
B. True
C. True
D. False - 14%
E. True
51. Which one of the following statements best describes the CT appearances of a renal oncocytoma
(tubular adenoma)?
A. It appears as a small, ill-defined renal mass in the majority of cases.
B. It is bilateral in 60-80% of cases.
C. It characteristically consists of multiple renal lesions.
D. CT shows punctuate calcification in the majority of patients.
E. Low attenuation (—100 to —50 HU) areas within a large lesion are consistent with an
oncocytoma.
Source: McQueen, Andrew S., et al. Grainger & Allison’s Diagnostic Radiology: Single Best Answer
MCQs. 5th ed., Churchill Livingstone, 2009.
Explanation:
• Large lesions can extend into and engulf the perinephric fat, and can therefore be mistaken for
angiomyolipomas (due to fat content).
52. An 83-year-old man undergoes an emergency left hip hemiarthroplasty following a fractured neck of
femur. Six days after surgery, he develops increasing abdominal distension with nausea and vomiting. An
abdominal radiograph is performed and demonstrates dilatation of the ascending and transverse colon
with the caecum measuring 7.0 cm in diameter. The clinical team believe that the patient may have
colonic pseudo-obstruction and a single contrast (instant) enema is performed using water-soluble
contrast. What are the likely findings in colonic pseudo-obstruction?
A. Extrinsic compression of the sigmoid colon
B. Long, irregular stricture of the sigmoid colon
C. Long, smooth stricture at the splenic flexure
D. No stricture demonstrated
E. Short ‘apple core’ stricture of the descending colon
Source: McQueen, Andrew S., et al. Grainger & Allison’s Diagnostic Radiology: Single Best Answer
MCQs. 5th ed., Churchill Livingstone, 2009.
Explanation:
• An instant enema can exclude mechanical obstruction in patients with colonic pseudo-obstruction.
EBDR Exam MCQs & Concepts October 2024
Dr. Kareem Alnakeeb 82
53. A 54-year-old man has an abdominal ultrasound that shows a 3-cm hyperechoic lesion at the upper pole
of the left kidney. An unenhanced CT abdomen is subsequently performed and demonstrates a left upper
pole heterogeneous renal mass with central areas of low attenuation (5—10 HU). After intravenous
contrast is administered, this mass enhances by more than 30 HU. What is the most likely diagnosis?
A. Angiomyolipoma
B. Oncocytoma
C. Renal abscess
D. Renal cell carcinoma
E. Unilocular renal cyst
Source: McQueen, Andrew S., et al. Grainger & Allison’s Diagnostic Radiology: Single Best Answer
MCQs. 5th ed., Churchill Livingstone, 2009.
Explanation:
• Angiomyolipomas are benign, fat-containing lesions which do not enhance by more than 15 HU and
contain low attenuation (—15 to —20HU) fatty areas.
• Postcontrast enhancement of greater than 20 HU of a solid renal mass is highly suggestive of
malignancy.
54. A 64-year-old man presents with right renal colic and a kidney ureter bladder plain radiograph (CT KUB) is
performed. This demonstrates an incidental 2-cm solid right adrenal mass. On the unenhanced CT, the
mass is homogeneous and has an average density of 7 HU. What is the most likely diagnosis?
A. Adrenal adenoma
B. Adrenal hyperplasia
C. Adrenal metastasis
D. Focal adrenal haemorrhage
E. Primary adrenal malignancy
Source: McQueen, Andrew S., et al. Grainger & Allison’s Diagnostic Radiology: Single Best Answer
MCQs. 5th ed., Churchill Livingstone, 2009.
Explanation:
• The 10-HU threshold is now the standard by which radiologists differentiate lipid-rich adenomas
from most other adrenal lesions on unenhanced CT.
• The presence of substantial amounts of intracellular fat is critical in malting the specific diagnosis of
adenoma.
• Up to 30% of adenomas, however, do not have abundant intracellular fat and, thus, show attenuation
values greater than 10 HU on unenhanced CT.
• Lesions above 10 HU on an unenhanced CT are considered indeterminate and other investigations
may be required.
55. A 34-year-old man is knocked off his bicycle by a car and presents to the Emergency Department with
bruising over the right flank and gross hematuria. The A&E SpR suspects renal injury and requests a CT
abdomen. Which one of the following findings is most likely to be seen in uncomplicated renal contusion
(Grade 1 renal injury)?
A. Ill-defined areas of low attenuation with irregular margins
B. Subcapsular high attenuation collection
C. Wedge-shaped areas of high attenuation, typically involving the renal
D. Well-defined areas of low attenuation within the renal parenchyma
E. Urinoma formation
Source: McQueen, Andrew S., et al. Grainger & Allison’s Diagnostic Radiology: Single Best Answer
MCQs. 5th ed., Churchill Livingstone, 2009.
EBDR Exam MCQs & Concepts October 2024
Dr. Kareem Alnakeeb 83
56. A 24-year-old motorcyclist is involved in a high-speed accident and is brought to the Emergency
Department. He has abdominal guarding and is hemodynamically unstable. An ultrasound abdomen
performed in the Emergency Department demonstrates free peritoneal fluid and a laparotomy is
performed. In addition to liver and splenic lacerations, the surgeon finds a left retroperitoneal hematoma.
Postoperatively, the on-call urologist requests a CT abdomen to assess the left renal injury. Which one of
the following findings would indicate a Grade 4 renal laceration?
F. Extravasation of contrast from the pelvicalyceal system on delayed phase (5 min) images
G. Large (2-cm) subcapsular hematoma
H. Perinephric hematoma that extends into the pararenal spaces
I. Ill-defined low attenuation change in the lower pole renal cortex
J. Segmental renal infarction
Source: McQueen, Andrew S., et al. Grainger & Allison’s Diagnostic Radiology: Single Best Answer
MCQs. 5th ed., Churchill Livingstone, 2009.
Explanation:
• A deep renal laceration that extends into the collecting system is indicative of a grade 4 injury.
57. Which one of the following statements best describes the radiological appearances of a parapelvic renal
cyst?
A. It does not opacify during IVU.
B. If hydronephrosis is present, a parapelvic cyst can be excluded.
C. It shows delayed (10 min) filling on IVU.
D. It may have similar appearances to calyceal diverticula on IVU.
E. The majority arise from the lower renal pole.
Source: McQueen, Andrew S., et al. Grainger & Allison’s Diagnostic Radiology: Single Best Answer
MCQs. 5th ed., Churchill Livingstone, 2009.
Explanation:
• A parapelvic cyst is located near the renal hilum, does not communicate with the renal pelvis (unlike
calyceal diverticula) and therefore does not opacify during IVU.
• It compresses the pelvis and may cause hydronephrosis.
58. A 23-year-old man presents with a 2-day history of vomiting and generalized abdominal pain. Two years
ago, he underwent a small bowel resection for an ileal stricture due to Crohn's disease. Initial blood tests
reveal a raised CRP and white cell count and an abdominal radiograph demonstrates dilated loops of
small bowel. Small bowel obstruction is suspected and a contrast-enhanced CT of the abdomen is
performed. Which one of the following statements is true regarding the role of multidetector CT in small
bowel obstruction?
A. Five to 15% of small bowel obstructions are due to hernias.
B. Twenty to 30% of small bowel obstructions arc due to adhesions.
C. Bowel wall thickening and intramural gas indicate the presence of pneumatosis coli.
D. Closed loop obstruction is less likely to result in bowel ischemia than simple obstruction.
E. In small bowel obstruction due to adhesions, a transition point will not be seen.
Source: McQueen, Andrew S., et al. Grainger & Allison’s Diagnostic Radiology: Single Best Answer
MCQs. 5th ed., Churchill Livingstone, 2009.
Explanation:
• Bowel wall thickening, lack of enhancement, adjacent fluid and pneumatosis intestinalis are all CT
signs of ischemia (strangulation) in small bowel obstruction.
• Fifty to 80% of small bowel obstruction is attributable to adhesions while 10% is due to hernias.
• In adhesions, there will usually be a history of previous abdominal surgery with CT demonstrating
small bowel obstruction.
• The transition point may be identified, but the actual adhesive land is usually not visualized.
EBDR Exam MCQs & Concepts October 2024
Dr. Kareem Alnakeeb 84
59. A 78-year-old man has myelodysplastic syndrome and requires frequent blood transfusions. He develops
progressively abnormal liver function tests and a grossly elevated ferritin level. An MRI of the liver is
performed using breath hold half Fourier single shot spin echo T2w images. Which finding would make a
diagnosis of hemosiderosis (iron overload from recurrent blood transfusion) more likely than
haemochromatosis?
F. Increased T2 signal in the liver only
G. Increased T2 signal in the liver and spleen
H. Reduced T2 signal in the liver only
I. Reduced T2 signal in the liver and spleen
J. Reduced T2 signal in the spleen only
Source: McQueen, Andrew S., et al. Grainger & Allison’s Diagnostic Radiology: Single Best Answer
MCQs. 5th ed., Churchill Livingstone, 2009.
Explanation:
• In iron overload due to recurrent transfusions, there is increased iron deposition in the
reticuloendothelial system. This leads to reduced Tl, T2 and T2* signal intensity in the liver and
spleen.
• Haemochromatosis causes diffusely reduced T2 signal in the liver and may lead to cirrhosis, but the
splenic signal intensity should remain normal.
• Diffuse fatty liver will lead to increased T2 signal in the liver with signal loss during out-of-phase
images.
60. Which of the following is not an angiographic sign of active bleeding?
A. Contrast extravasation
B. Vessel spasm
C. Vessel cut-off
D. Early venous filling
E. Vessel dilatation
Source: Chin, Teck Yew, et al. Get Through Final FRCR 2A: SBAs. 1st ed., CRC Press, 2017.
Explanation:
• The rest are angiographic signs of active bleeding; vessel dilatation is not.
61. A 52-year-old postmenopausal woman presents for her first screening mammogram. Within the right
upper outer quadrant, there is a 2-cm well-defined, oval mass that has dense ‘popcorn’ calcification
within it and is surrounded by a thin radiolucent rim. On ultrasound, the mass is well defined and
hyperechoic with areas of acoustic shadowing due to contained calcification. What is the most likely
diagnosis?
A. Fat necrosis
B. Fibroadenoma
C. Hamartoma
D. Oil cyst
E. Papilloma
Source: McQueen, Andrew S., et al. Grainger & Allison’s Diagnostic Radiology: Single Best Answer
MCQs. 5th ed., Churchill Livingstone, 2009.
Explanation:
• Fibroadenomas may become calcified, particularly after menopause. Classically the calcifications
have a coarse ‘popcorn’ appearance; however, they may also appear small and punctate.
• An oil cyst typically demonstrates eggshell calcification and is the result of fat necrosis.
EBDR Exam MCQs & Concepts October 2024
Dr. Kareem Alnakeeb 85
62. A 15-month-old boy is referred from his GP with a limp. Which radiological finding would be consistent
with DDH?
A. Accelerated epiphyseal ossification
B. Decreased distance between the medial portion of the proximal femoral metaphysis and the pelvis
C. Increased acetabular angle
D. Inferolateral displacement of the femoral head in relation to Perkin's line
E. Preservation of Shenton's line
Source: McQueen, Andrew S., et al. Grainger & Allison’s Diagnostic Radiology: Single Best Answer
MCQs. 5th ed., Churchill Livingstone, 2009.
63. An 18-year-old man experiences persistent symptoms following a fracture through the waist of the right
scaphoid. Radiographs of the right scaphoid indicate non-union. An MRI is performed to assess the
vascularity of the proximal pole. Which imaging features are consistent with a diagnosis of avascular
necrosis?
F. Bone marrow enhancement following administration of gadolinium
G. High signal surrounding the fracture on T2w images
H. High signal within the proximal pole on T1w images
I. High signal within the proximal pole on STIR images
J. Low signal within the proximal pole on T1w images
Source: McQueen, Andrew S., et al. Grainger & Allison’s Diagnostic Radiology: Single Best Answer
MCQs. 5th ed., Churchill Livingstone, 2009.
Explanation:
• Low signal on T1 reflects death of the adipocytes. The combination of low signal on T1w images and
low or intermediate signal on T2w images accurately predicts avascular necrosis.
64. A 68-year-old man presents to his GP with a 1-month history of epigastric pain, vomiting and mild weight
loss. Examination is unremarkable and the patient is referred for an upper gastrointestinal endoscopy.
This demonstrates mild gastritis with biopsies positive for Helicobacter pylori and he is commenced on
eradication therapy. Three months later, the symptoms have persisted and the patient has lost 5 kg in
weight. A double contrast barium meal is performed and reveals a shallow ulcer on the lesser curve of
the stomach. Which additional finding would make the ulcer more likely to be benign than malignant?
A. Hampton's line is present.
B. Nodular mucosal folds stop at the edge of the lesion.
C. The ulcer does not extend beyond the gastric wall.
D. The ulcer has an irregular margin.
E. The ulcer measures 40 mm in size.
Source: McQueen, Andrew S., et al. Grainger & Allison’s Diagnostic Radiology: Single Best Answer
MCQs. 5th ed., Churchill Livingstone, 2009.
Explanation:
• Hampton’s line refers to a lucent line crossing the ulcer base: its presence is highly suggestive of a
benign ulcer.
EBDR Exam MCQs & Concepts October 2024
Dr. Kareem Alnakeeb 86
65. A 53-year-old woman is seen in the general surgical outpatient clinic. She attended her GP with a 1-
month history of upper abdominal pain and was found to have a palpable, firm mass in the epigastrium.
An upper gastrointestinal (GI) endoscopy is normal and the surgical team request a contrast-enhanced
CT of the abdomen. This demonstrates a multicystic mass in the pancreas. Which findings would make a
mucinous cystic tumor more likely than a serous cystadenoma?
A. Central stellate calcification is present within the lesion.
B. The mass contains 12 separate cysts.
C. The smallest cystic component measures 28 mm in diameter.
D. The patient has a known diagnosis of von Hippel-Lindau disease.
E. The tumor is located in the head of the pancreas.
Source: McQueen, Andrew S., et al. Grainger & Allison’s Diagnostic Radiology: Single Best Answer
MCQs. 5th ed., Churchill Livingstone, 2009.
Explanation:
• Mucinous cystic pancreatic tumors (cystadenomas and cystadenocarcinomas) typically contain a
few large cysts, each measuring more than 20 mm diameter.
66. A 41-year-old man has a 3-month history of weight loss and recurrent central abdominal pain. The pain is
intermittent and radiates from the epigastrium through to his back. His past medical history includes
excessive alcohol consumption and two previous admissions to hospital for acute pancreatitis. A
contrast-enhanced CT of the abdomen is performed with pre-contrast, arterial and portal venous phase
images of the upper abdomen. Which CT finding would be more suggestive of chronic pancreatitis than
ductal pancreatic adenocarcinoma?
A. Common bile duct dilatation
B. Focal enlargement of the pancreatic head
C. Intraductal pancreatic calcification
D. Peripancreatic fat stranding and ascites
E. Reduced enhancement of body of pancreas
Source: McQueen, Andrew S., et al. Grainger & Allison’s Diagnostic Radiology: Single Best Answer
MCQs. 5th ed., Churchill Livingstone, 2009.
Explanation:
• Intraductal calcification may be focal or diffuse and is not seen in all patients with chronic
pancreatitis. When it is present, however, it is a highly reliable sign of chronic pancreatitis.
67. A 75-year-old diabetic man underwent a left below knee amputation 3 months ago for osteomyelitis of
the distal tibia. Since then, he has experienced recurrent episodes of fever and malaise. MRI is
contraindicated due to a metallic aortic valve. Which is the best investigation to exclude an occult focus
of osteomyelitis?
A. CT
B. US
C. Scintigraphy using gallium
D. Scintigraphy using indium-labelled white cells
E. Scintigraphy using technetium (Tc-99m) monodiphosphonate
Source: McQueen, Andrew S., et al. Grainger & Allison’s Diagnostic Radiology: Single Best Answer
MCQs. 5th ed., Churchill Livingstone, 2009.
Explanation:
• Although an indium-labelled white cell study is more specific, a bone scintigram using Tc-99m
monodiphosphonate is a more sensitive test to exclude osteomyelitis.
EBDR Exam MCQs & Concepts October 2024
Dr. Kareem Alnakeeb 87
68. A 6-year-old with spina bifida has a chest X-ray performed for possible lower respiratory tract infection.
The lungs are clear but there is a well-defined, round paraspinal mass with an air–fluid level. What is the
most likely diagnosis?
A. Bronchogenic cyst
B. Morgagni hernia
C. Esophageal duplication cyst
D. Cystic teratoma
E. Esophageal tumor
Source: Chin, Teck Yew, et al. Get Through Final FRCR 2A: SBAs. 1st ed., CRC Press, 2017.
Explanation:
Esophageal Duplication Cysts
• Esophageal duplication cysts are rare congenital anomalies.
• They are associated with vertebral anomalies (spina bifida, hemivertebrae, fusion defects).
• There is also an association with esophageal atresia and small bowel duplication.
• Most cysts develop in the right posteroinferior mediastinum.
• CT demonstrates a well-marginated round, oval or tubular-shaped fluid-filled cystic structure that
has a well-defined, thin wall. The cyst is of water attenuation with no enhancement of contents and
no infiltration of surrounding structures.
• Malignant degeneration is rare.
Other Mediastinal Lesions
• Bronchogenic cyst is the most common cystic mediastinal mass that typically lies in the middle
mediastinum, not in a paraspinal location; in addition, you would not expect an air-fluid level.
• Cystic teratoma is an anterior mediastinal mass.
• Morgagni hernia would be unlikely to cause a solitary round lesion; multiple structures would be
expected.
69. A 38-year-old woman undergoes pelvic MRI for chronic pelvic pain. Axial and coronal T2-weighted images
show a tortuous, C-shaped tubular structure in the left adnexa that is uniformly hyperintense on T2 and
hypointense on T1. The wall is thin and smooth; there is no mural nodule or enhancing solid tissue on
post-contrast sequences. According to the ACR O-RADS MRI risk stratification system, which score and
malignancy risk category should be assigned to this lesion?
Option O-RADS MRI score Positive predictive value (PPV) for malignancy
A. 1 <0.1%
B. 2 <0.5%
C. 3 5 – 10%
D. 4 20 – 50%
E. 5 >80%
Answer: B
Source: Perplexity AI
Explanation:
• A dilated fallopian tube containing simple fluid with a thin, smooth wall and no enhancing solid
tissue is classified as O-RADS MRI 2 (“almost certainly benign”), carrying a < 0.5% likelihood of
malignancy. Recognizing this typical appearance of hydrosalpinx prevents over-classification and
unnecessary intervention.
• Key learning point: Hydrosalpinx with simple fluid and no suspicious mural elements is an O-RADS
MRI 2 lesion; only the presence of thickened folds, non-simple fluid, or enhancing solid components
would raise the score to 3 or higher.
EBDR Exam MCQs & Concepts October 2024
Dr. Kareem Alnakeeb 88
70. A 54-year-old woman attends a well woman clinic and is found to have abnormal liver function tests. She
is referred to the hepatology outpatient clinic and an abdominal ultrasound is performed. This
demonstrates diffuse increased reflectivity of the liver parenchyma but no focal parenchymal
abnormality. The hepatology team request an ultrasound-guided percutaneous liver biopsy. Which
statement is true regarding this procedure?
A. Ten to 20% of complications occur in the first 2 hours post procedure.
B. Ascites is an absolute contraindication to percutaneous liver biopsy.
C. Mortality rate is 1 in 500.
D. Over 90% of complications occur in the first 24 hours post procedure.
E. There is no increased risk of complications with malignant liver lesions.
Source: McQueen, Andrew S., et al. Grainger & Allison’s Diagnostic Radiology: Single Best Answer
MCQs. 5th ed., Churchill Livingstone, 2009.
Explanation:
• Following an ultrasound-guided liver biopsy, nearly two-thirds of complications occur in the first 2
hours, with 96% of complications having occurred by 24 hours.
71. A 67-year-old woman undergoes surgical resection of a distal sigmoid adenocarcinoma. The surgeon
performs a primary anastomosis between the descending colon and rectum and leaves a defunctioning
loop colostomy. Nine days later, the patient is experiencing fevers and low abdominal pain. A contrast-
enhanced CT shows a small fluid collection around the anastomosis with no definite abscess identified.
The surgical team are concerned about the integrity of the anastomosis. Which investigation would you
choose to look for an anastomotic leak?
A. Barium enema
B. Barium follow-through
C. MRI pelvis with intravenous gadolinium
D. Water-soluble contrast cystogram
E. Water-soluble contrast enema
Source: McQueen, Andrew S., et al. Grainger & Allison’s Diagnostic Radiology: Single Best Answer
MCQs. 5th ed., Churchill Livingstone, 2009.
Explanation:
• The combination of water-soluble contrast enema and CT is used to look for anastomotic leakage
and abscess formation.
72. A 68-year-old man presents to his GP with weight loss and jaundice. Liver function tests demonstrate
obstructive jaundice and an abdominal ultrasound shows mild intrahepatic biliary dilatation with a
common bile duct measuring 12 mm in diameter. In the pancreatic head, a 3-cm hypoechoic mass is
present. An ERCP is performed with insertion of a plastic stent and brushings confirm a pancreatic ductal
adenocarcinoma. A triple-phase (pre-contrast, arterial and portal venous) multidetector CT of the
pancreas is performed. Which finding would indicate a nonresectable pancreatic tumor?
A. Enhancing pancreatic parenchyma between the tumor and superior
B. The pancreatic duct dilated to 6 mm
C. The presence of a 5-mm coeliac axis lymph node
D. The tumor has invaded the duodenum
E. The tumor in contact with 75% of the superior mesenteric artery
Source: McQueen, Andrew S., et al. Grainger & Allison’s Diagnostic Radiology: Single Best Answer
MCQs. 5th ed., Churchill Livingstone, 2009.
Explanation:
• If the tumour is in contact with more than half of the vessel circumference, it is very unlikely to be
resectable.
EBDR Exam MCQs & Concepts October 2024
Dr. Kareem Alnakeeb 89
73. What level of serum glucose is generally considered acceptable when performing an FDG-PET scan?
A. Less than 50 mg/dL.
B. Less than 100 mg/dL.
C. Less than 200 mg/dL.
D. Less than 300 mg/dL.
Source: Unknown
Explanation: (by Perplexity AI, generated with the OpenAI O3 model)
• Most professional guidelines (EANM, SNMMI, ACR) advise proceeding with an oncologic FDG-PET/CT
if the fasting plasma glucose is below about 11 mmol/L (≈ 200 mg/dL); at higher levels competitive
inhibition by circulating glucose can lower tumor uptake and degrade image quality.
• Values under 100 mg/dL are ideal but not mandatory, whereas levels > 200 mg/dL usually prompt
rescheduling or glucose-lowering measures.
74. A 56-year-old patient presents with classic carcinoid syndrome and is found to have multiple liver
metastases on imaging. What is the most likely site of the primary tumor?
A. Stomach
B. Duodenum
C. Small Bowel
D. Appendix.
E. Rectum
Source: Unknown
Explanation:
• Over 40% of carcinoid tumors originate in the small intestine; the rectum (~27%), appendix (~24%)
and stomach (~8%) are the next most frequent primary sites.
• Duodenal carcinoid tumors are rare.
• Small-intestinal carcinoid tumors frequently symptomatic.
75. A 35-year-old patient received a cadaveric renal transplant 5 days ago and now presents with worsening
renal function and decreasing urine output. Which one of the following findings on a Tc-99m DTPA
radionuclide scan would favor a diagnosis of acute tubular necrosis (ATN) over acute rejection?
F. Delayed renal excretion
G. Elevated resistive index greater than 0.7
H. Increased renal perfusion after administration of an ACEI (eg Captopril)
I. Poor/impaired graft perfusion
J. Preserved renal transplant perfusion
Source: McQueen, Andrew S., et al. Grainger & Allison’s Diagnostic Radiology: Single Best Answer
MCQs. 5th ed., Churchill Livingstone, 2009.
Explanation:
• ATN is an early complication in cadaveric allografts and frequently resolves spontaneously in 1—3
weeks. The radionuclide imaging findings of ATN are of preserved perfusion but poor renal function
and urine excretion.
• In acute rejection however, there is both impaired renal function and reduced perfusion on
radionuclide imaging.
EBDR Exam MCQs & Concepts October 2024
Dr. Kareem Alnakeeb 90
76. A 25-year-old doctor injures her left wrist whilst snowboarding. Initial radiographs are reported as
showing no fracture, but there is clinical suspicion of a scapholunate ligament disruption. Further views
are obtained. Which radiological feature would support the diagnosis?
A. Scapholunate angle less than 30°
B. Scapholunate distance of 2 mm
C. ‘Signet ring’appearance of the scaphoid
D. Rotatory subluxation of the lunate
E. Wedge-shaped appearance of the lunate
Source: McQueen, Andrew S., et al. Grainger & Allison’s Diagnostic Radiology: Single Best Answer
MCQs. 5th ed., Churchill Livingstone, 2009.
Explanation:
• The ‘signet ring’appearance is clue to rotatory subluxation of the scaphoid as a result of disruption
of the scapholunate ligament.
77. An athletic 19-year-old medical student presents to the Emergency Department after sustaining an injury
to his right hip during training. A radiograph reveals a fracture of the anterior superior iliac spine. What is
the most likely diagnosis?
A. Avulsion of the adductor muscles
B. Avulsion of the hamstring muscles
C. Avulsion of iliopsoas
D. Avulsion of rectus femoris
E. Avulsion of sartorius
Source: McQueen, Andrew S., et al. Grainger & Allison’s Diagnostic Radiology: Single Best Answer
MCQs. 5th ed., Churchill Livingstone, 2009.
Explanation:
• Avulsion injuries occur at characteristic sites and are particularly common in children and
adolescents.
78. A 29-year-old man has an IVU performed following an episode of haematuria. This demonstrates
complete right-sided ureteric duplication. Which one of the following statements is true?
F. If present, an ectopic ureterocoele is usually related to the lower moiety
G. The lower moiety ureter usually obstructs at the vesicoureteric junction.
H. The upper moiety calyces are prone to vesicoureteric reflux.
I. The upper moiety ureter is prone to ureteric obstruction.
J. The upper moiety ureter usually inserts into the bladder superior to the lower moiety ureter.
Source: McQueen, Andrew S., et al. Grainger & Allison’s Diagnostic Radiology: Single Best Answer
MCQs. 5th ed., Churchill Livingstone, 2009.
79. A low flat renogram curve indicates:
E. Advanced nephropathy
F. Complete obstruction to urine outflow
G. Vesico-ureteric reflux
H. Partial obstruction to urine outflow
Explanation: (by Perplexity AI, generated with the OpenAI O3 model)
• A very low, almost horizontal time–activity curve reflects markedly reduced tracer extraction by
damaged renal parenchyma, typical of end-stage or advanced medical renal disease.
• Complete or partial outflow obstruction instead produces an uptake phase followed by a rising or
plateau phase; vesico-ureteric reflux alters the post-void segment rather than the primary
renographic curve
EBDR Exam MCQs & Concepts October 2024
Dr. Kareem Alnakeeb 91
80. Regarding gastric emptying scintigraphy
A. T1/2 is the time at which gastric counts falls to its half
B. Tc 99m tin colloid labeled RBCs are injected IV
C. If 20 % of gastric counts remain after 1 hour, delayed gastric emptying is considered.
D. It cannot detect changes in gastric emptying rate in post operative cases.
EBDR Exam MCQs & Concepts July 2024
Dr. Kareem Alnakeeb 92
July 2024
Paper 1
1. A 45-year-old patient presents with progressive visual field defects. MRI demonstrates a well-
circumscribed extra-axial mass with dural attachment showing avid homogeneous enhancement and a
dural tail sign. What are the typical T1 and T2 signal characteristics of this lesion?
A. High T1, high T2 signal
B. Low T1, high T2 signal
C. Isointense T1, isointense to mildly hyperintense T2 signal
D. High T1, low T2 signal
E. Low T1, low T2 signal
Explanation: (by Perplexity AI, generated with the OpenAI O3 model)
• The clinical and imaging features describe a typical meningioma. Meningiomas characteristically
demonstrate isointensity to slight hypointensity relative to grey matter on T1-weighted sequences
and isointensity to mild hyperintensity on T2-weighted imaging.
• The microcystic subtype (1.6% of cases) is an exception, showing low T1 and high T2 signal.
• Options A and D suggesting high T1 signal would be more consistent with melanotic lesions like
uveal melanoma.
• Option B describes typical glioma characteristics, while option E would suggest a calcified or fibrotic
lesion.
2. In bilateral Wilms tumour (stage 5) the primary treatment goal after initial neoadjuvant chemotherapy is
to
A. perform bilateral radical nephrectomies to maximise oncological clearance
B. undertake unilateral radical nephrectomy and contralateral nephron-sparing surgery
C. achieve lung-directed radiotherapy before any renal surgery
D. proceed immediately to renal transplantation once both kidneys are removed
E. delay all surgery until post-chemotherapy radiation has completed
Explanation: (by Perplexity AI, generated with the OpenAI O3 model)
• Neoadjuvant chemotherapy is given first to shrink synchronous tumours in both kidneys. Surgery
then aims to remove the more involved kidney completely while preserving as much functioning
parenchyma as possible in the contralateral kidney, typically via nephron-sparing surgery. This
balances oncological control with long-term renal function.
• Bilateral radical nephrectomies (A) sacrifice all native renal tissue and would leave a child dialysis-
dependent; transplantation (D) is only considered later if renal failure ensues.
• Up-front lung radiotherapy (C) is reserved for metastatic pulmonary disease, not as the initial step in
stage 5.
• Delaying all surgery until after radiotherapy (E) risks local progression and is not standard practice.
3. According to the Children’s Oncology Group (NWTS/COG) staging system, synchronous tumours in both
kidneys at presentation classify Wilms tumour as what stage?
A. Stage I
B. Stage II
C. Stage III
D. Stage IV
E. Stage V
EBDR Exam MCQs & Concepts July 2024
Dr. Kareem Alnakeeb 93
Explanation: (by Perplexity AI, generated with the OpenAI O3 model)
• Bilateral renal involvement is automatically designated stage V in the NWTS/COG scheme,
irrespective of tumour size, resectability or nodal status.
• Stages I–II apply only to unilateral disease that can be completely resected, stage III to residual intra-
abdominal tumour or nodal spread after surgery, and stage IV to distant metastases such as lung or
liver.
• Recognising stage V is crucial because management focuses on nephron-sparing chemotherapy
followed by staged or partial nephrectomy to preserve renal function, rather than immediate radical
nephrectomy used in lower stages.
4. A 3-day-old infant with inspiratory stridor undergoes contrast-enhanced CT, which shows the left
pulmonary artery arising from the right pulmonary artery and passing between the trachea and
oesophagus to reach the left hilum (pulmonary artery sling); which associated abnormality is most
frequently present?
A. Long-segment tracheal stenosis with complete tracheal rings
B. Double aortic arch
C. Patent foramen ovale
D. Aberrant right subclavian artery
E. Pulmonary sequestration
Explanation: (by Perplexity AI, generated with the OpenAI O3 model)
• The aberrant left pulmonary artery (pulmonary sling) frequently compresses and deforms the airway,
and over half of patients have long-segment tracheal stenosis formed by complete cartilaginous
rings, making option A correct.
• Double aortic arch and aberrant right subclavian artery are vascular rings that can coexist but are far
less common associations.
• Patent foramen ovale is a frequent incidental cardiac finding but has no specific link to pulmonary
sling.
• Pulmonary sequestration affects lung parenchyma rather than the central airway and is not
characteristically related to this vascular anomaly.
5. In a patient with post-ductal coarctation of the aorta who also has an aberrant right subclavian artery
arising distal to the coarctation, which chest-radiograph pattern of rib notching is classically expected?
A. Bilateral rib notching of ribs 3–8
B. Unilateral right-sided rib notching of ribs 3–8
C. Unilateral left-sided rib notching of ribs 3–8
D. Rib notching limited to the 1st and 2nd ribs
E. No rib notching is seen at any level
Explanation: (by Perplexity AI, generated with the OpenAI O3 model)
• Inferior rib notching results from dilated intercostal collaterals carrying blood from a subclavian
artery that is proximal to the coarctation down to the descending aorta.
• When the right subclavian artery is aberrant and originates distal to the narrowed segment, only the
left subclavian (which is proximal) can supply collateral flow; therefore, enlarged intercostal arteries
– and rib notching – develop on the left side only, typically affecting ribs 3–8.
• Right-sided notching (option B) occurs when the coarctation lies proximal to the left subclavian so
the right subclavian is the collateral source.
• Bilateral notching (option A) requires both subclavian arteries to originate proximal to the
obstruction.
• The 1st-2nd ribs are spared because their intercostal arteries do not participate in the collateral
circuit, so option D is incorrect.
EBDR Exam MCQs & Concepts July 2024
Dr. Kareem Alnakeeb 94
• Absence of notching (option E) is inconsistent with long-standing adult coarctation.
6. In paediatric non-WNT/SHH medulloblastoma, which post-gadolinium MRI appearance is associated
with the poorest overall and event-free survival?
A. No measurable enhancement
B. Heterogeneous enhancement in 10–75% of tumour volume
C. Extensive enhancement involving >75% of tumour volume
D. Ring enhancement with central necrosis
E. Peripheral nodular enhancement surrounding internal cysts
Explanation: (by Perplexity AI, generated with the OpenAI O3 model)
• Extensive, nearly whole-tumour gadolinium uptake signifies a highly vascular, biologically aggressive
lesion in non-WNT/SHH medulloblastoma.
• A series of 76 patients showed that enhancement of >75% of tumour volume independently
predicted significantly worse overall and event-free survival than either weak/none or heterogeneous
enhancement patterns.
• Minimal enhancement (option A) and heterogeneous partial enhancement (option B) were linked to
better outcomes, while ring (D) and peripheral nodular (E) patterns are described in other posterior-
fossa tumours but are not established prognostic markers in medulloblastoma.
7. On MRI, a 25-year-old patient has a unilocular, T2-hyperintense cystic lesion centred in the sublingual
space that tracks around the posterior margin of the mylohyoid muscle into the submandibular space
without internal enhancement. What is the most likely diagnosis?
A. Plunging ranula
B. Thyroglossal duct cyst
C. First branchial cleft cyst
D. Submandibular abscess
E. Cystic hygroma
Explanation: (by Perplexity AI, generated with the OpenAI O3 model)
• Plunging ranula arises from the sublingual gland; when it dissects through or behind a dehiscence in
the mylohyoid, it presents as a well-defined, thin-walled, unilocular cyst that is high signal on T2 and
shows no enhancement, exactly as described.
• Thyroglossal duct cysts lie in the midline or paramedian track between foramen cecum and thyroid
cartilage and rarely extend lateral to the mylohyoid.
• First branchial cleft cysts occur parotid–external auditory canal region, not the floor of mouth.
• A submandibular abscess would show thick enhancing walls and diffusion restriction with clinical
sepsis.
• Cystic hygroma (lymphatic malformation) is characteristically multiloculated and infiltrative, often in
the posterior triangle, not a solitary unilocular cyst.
8. In the standard cervical lymph-node classification used in head-and-neck imaging, the jugulodigastric
(subdigastric) lymph nodes belong to which nodal level?
A. Level Ib
B. Level IIa
C. Level IIb
D. Level III
E. Level IV
Explanation: (by Perplexity AI, generated with the OpenAI O3 model)
• The jugulodigastric nodes sit just inferior to the posterior belly of the digastric muscle and
immediately anterior (or inseparable from) the internal jugular vein. These anatomical boundaries
place them in the upper internal jugular chain designated Level IIa.
EBDR Exam MCQs & Concepts July 2024
Dr. Kareem Alnakeeb 95
• Level IIb nodes are also in the upper jugular region but lie posterior to a fat plane separating them
from the vein and spinal accessory nerve.
• Level Ib nodes occupy the submandibular triangle, while Levels III and IV track progressively caudal
along the internal jugular vein from the hyoid to the clavicle.
• Recognising the jugulodigastric group as Level IIa is essential because they are often the first
echelon for oropharyngeal and tonsillar metastases and are the largest normal cervical nodes.
9. A 50-year-old intravenous drug user presents with severe mid-back pain, fever and rapidly progressive
paraparesis; which imaging investigation is the most appropriate first-line test to confirm a suspected
spinal epidural abscess?
A. Contrast-enhanced MRI of the whole spine
B. Non-contrast CT of the symptom-level spine
C. Radionuclide white-cell scan
D. Plain thoracic spine radiographs
E. FDG PET/CT
Explanation: (by Perplexity AI, generated with the OpenAI O3 model)
• Spinal epidural abscess is a neurosurgical emergency; diagnosis must be secured quickly and the
entire neuraxis screened for skip lesions. Gadolinium-enhanced MRI provides near-100% sensitivity
and specificity, delineates the epidural collection, shows rim enhancement that distinguishes
abscess from phlegmon, and accurately demonstrates cord or cauda compression—making it the
gold-standard first-line study.
• Non-contrast CT (B) and plain radiographs (D) lack soft-tissue contrast and miss early collections.
• Radionuclide scans (C) and FDG PET/CT (E) detect infection metabolically but are slower, less
specific for epidural compartment involvement and do not guide urgent surgical planning.
10. Regarding the classical CT appearance of pleural empyema, which radiological sign is most
characteristic and helps distinguish it from a peripheral lung abscess?
A. Air-fluid level that is centrally located
B. Cavitary lesion with irregular thick walls
C. Split pleura sign
D. Crescentic pleural fluid layering dependently
E. Tree-in-bud nodularity in the adjacent lung
Explanation: (by Perplexity AI, generated with the OpenAI O3 model)
• On contrast-enhanced CT, pleural empyema typically shows the split pleura sign—smooth,
symmetrically thickened parietal and visceral pleura that enhance and appear separated by pleural
fluid. This finding is highly sensitive and specific for empyema and is rarely seen with peripheral lung
abscesses.
• A lung abscess more often presents as a cavitary parenchymal lesion with irregular thick walls and a
central air-fluid level (distractors A and B).
• Free-flowing, crescentic effusions that layer with gravity suggest uncomplicated pleural effusion, not
empyema (distractor D).
• Tree-in-bud nodularity indicates endobronchial spread of infection rather than pleural disease
(distractor E).
EBDR Exam MCQs & Concepts July 2024
Dr. Kareem Alnakeeb 96
11. A 68-year-old man with heart failure with preserved ejection fraction undergoes cardiac MRI with
gadolinium. The short-axis late-gadolinium images show difficulty nulling the myocardium and
widespread heterogeneous patchy mid-myocardial and subendocardial enhancement of both ventricles.
Which diagnosis best explains this enhancement pattern?
A. Hypertrophic cardiomyopathy
B. Cardiac sarcoidosis
C. Dilated cardiomyopathy with mid-wall fibrosis
D. Cardiac amyloidosis
E. Previous transmural myocardial infarction
Explanation: (by Perplexity AI, generated with the OpenAI O3 model)
• Cardiac amyloidosis typically shows diffuse or patchy subendocardial/mid-myocardial late
gadolinium enhancement with failure to null normal myocardium because amyloid infiltration
shortens T1; this combination is highly suggestive of the disease.
• Hypertrophic cardiomyopathy usually shows focal mid-wall or RV insertion-point enhancement;
sarcoidosis produces focal patchy nodular enhancement often in the basal septum; dilated
cardiomyopathy gives linear mid-septal enhancement; prior infarction shows contiguous
subendocardial or transmural enhancement in a coronary territory—none match the diffuse patchy
pattern with inversion-time “nulling” difficulty.
12. A 13-year-old boy presents with pleuritic chest pain and a tender palpable swelling over the right
posterior rib cage. Chest radiograph shows a permeative rib lesion with a multilaminated “onion-skin”
periosteal reaction and an associated soft-tissue mass. Which diagnosis is most likely?
A. Langerhans cell histiocytosis
B. Chronic osteomyelitis
C. Chondroblastoma
D. Ewing sarcoma
E. Osteoid osteoma
Explanation: (by Perplexity AI, generated with the OpenAI O3 model)
• Rib lesions that combine a permeative pattern, a soft-tissue mass and a lamellated (“onion-skin”)
periosteal reaction are classically seen in Ewing sarcoma, a malignant small-round-blue-cell tumour
that typically affects children and adolescents.
• Langerhans cell histiocytosis can produce laminated periosteal reaction but usually causes well-
defined erosions or punched-out defects without a large soft-tissue mass.
• Osteomyelitis may mimic aggressive periostitis yet commonly shows sequestra, cortical irregularity
and systemic infection markers.
• Chondroblastoma arises in epiphyses of long bones, not ribs, and seldom forms a sizeable soft-
tissue component.
• Osteoid osteoma is a benign cortical lesion producing a small nidus with solid rather than
multilayered periosteal reaction and intense nocturnal pain relieved by NSAIDs.
EBDR Exam MCQs & Concepts July 2024
Dr. Kareem Alnakeeb 97
13. In a patient who has undergone canal-wall-up mastoidectomy, which MRI sequence is most reliable for
detecting residual or recurrent middle-ear cholesteatoma ≥3 mm?
A. Post-contrast T1-weighted spin-echo
B. T2-weighted fast spin-echo
C. Non-echo-planar diffusion-weighted imaging
D. Susceptibility-weighted imaging (SWI)
E. 3D time-of-flight MR angiography
Explanation: (by Perplexity AI, generated with the OpenAI O3 model)
• Cholesteatoma contains tightly packed keratin, giving marked restriction of water diffusion; non-
echo-planar DWI (e.g. PROPELLER or HASTE) exploits this, providing bright high-signal foci against
suppressed background and avoids susceptibility artefact from temporal-bone air interfaces.
• It detects lesions as small as 2–3 mm with >90% sensitivity and obviates many “second-look”
surgeries.
• Post-contrast T1 (A) usually shows no enhancement and may miss small lesions.
• Conventional T2 FSE (B) lacks specificity because fluid and granulation tissue also appear bright.
• SWI (D) is designed for paramagnetic blood products, not keratin, and TOF MRA (E) images flow in
vessels, offering no benefit for cholesteatoma assessment.
14. Which of the following is a recognised advantage of magnetic resonance urography (MRU) compared with
CT urography?
A. Superior detection of small ureteric calculi
B. Ability to combine high soft-tissue contrast imaging with functional assessment in one
examination
C. Shorter acquisition time
D. Lower sensitivity to patient movement
E. Greater availability in emergency settings
Explanation: (by Perplexity AI, generated with the OpenAI O3 model)
• MRU exploits T2-weighted “static-fluid” and post-contrast T1-weighted sequences, allowing
simultaneous high-contrast anatomical imaging of the entire urinary tract and quantitative functional
data such as renal transit times and differential renal function. This one-stop structural-plus-
functional capability is unique to MRU and underpins its use in congenital anomalies,
hydronephrosis and renal impairment.
• CT urography offers faster scans and excellent stone detection but provides little functional
information and uses ionising radiation.
• Acquisition times are actually longer with MRU, small calculi are often missed, motion artefact
remains problematic and MRI scanners are less widely available, so options A, C, D and E are
incorrect.
15. A 38-year-old woman with metastatic, hormone-receptor-negative/HER2-negative breast cancer has
widespread visceral disease 6 months after her initial diagnosis. She is otherwise fit and is being
considered for palliative chemotherapy. Which single-agent intravenous regimen is recommended as
first-line treatment in this setting, according to contemporary guidelines?
A. Capecitabine
B. Paclitaxel
C. Eribulin
D. Vinorelbine
E. Cyclophosphamide + Methotrexate + 5-FU (CMF)
EBDR Exam MCQs & Concepts July 2024
Dr. Kareem Alnakeeb 98
Explanation: (by Perplexity AI, generated with the OpenAI O3 model)
• Modern ESMO and ASCO guidance advise starting fit patients who have triple-negative metastatic
breast cancer with a taxane (paclitaxel or docetaxel) as first-line single-agent chemotherapy because
these drugs offer the best balance of response rate, survival benefit and tolerability.
• Capecitabine, vinorelbine and eribulin are commonly used later-line options after taxane or
anthracycline failure.
• CMF is outdated and reserved only when newer agents are unsuitable.
• Therefore, weekly or 3-weekly paclitaxel is the preferred initial regimen.
16. Ductal carcinoma in situ (DCIS) is suspected on screening mammography; an MRI is performed for
surgical planning and shows a non-mass clumped ductal enhancement extending over 6 cm in the upper
outer quadrant of the left breast with type II (plateau) kinetic curve. According to BI-RADS MRI
descriptors, what is the most appropriate category to assign?
A. BI-RADS 2 (benign)
B. BI-RADS 3 (probably benign)
C. BI-RADS 4A (low-suspicion malignancy)
D. BI-RADS 4C (high-suspicion malignancy)
E. BI-RADS 5 (highly suggestive of malignancy)
Explanation: (by Perplexity AI, generated with the OpenAI O3 model)
• Clumped ductal non-mass enhancement covering >1 segment and showing plateau kinetics is
strongly associated with high-grade DCIS or invasive cancer, carrying a positive predictive value
around 30%, which fits BI-RADS 4C (51–94% likelihood of malignancy).
• BI-RADS 5 requires classic malignant mass features with ≥95% likelihood, which non-mass
enhancement rarely achieves.
• A category 4A would underestimate risk, while BI-RADS 2 or 3 are inappropriate for suspicious
morphology and kinetics.
17. Catheter-directed intra-arterial thrombolysis is most appropriately used for which pattern of lower-limb
ischaemia?
A. Acute limb ischaemia of less than 14 days’duration
B. Chronic critical limb ischaemia with ulceration present for 2 months
C. Stable intermittent claudication due to femoropopliteal stenosis
D. Non-viable limb with fixed paralysis and mottling (Rutherford class III)
E. Thrombosed infra-inguinal bypass graft occluded for 8 weeks
Explanation: (by Perplexity AI, generated with the OpenAI O3 model)
• Thrombolytic agents can dissolve fresh thrombus and are therefore recommended for acute limb
ischaemia (<14 days, Rutherford I–IIa), where clot is still soft and the limb remains viable.
• Chronic critical ischaemia and long-standing graft occlusions contain organised thrombus and
intimal hyperplasia, so chemical lysis is ineffective, often needing surgical or endovascular
reconstruction.
• Intermittent claudication is treated electively; risks of thrombolysis outweigh benefits.
• A non-viable limb (fixed neurological deficit) requires primary amputation or open thrombectomy, as
thrombolysis is too slow and increases bleeding risk
EBDR Exam MCQs & Concepts July 2024
Dr. Kareem Alnakeeb 99
18. On high-resolution CT, which pleural abnormality is considered most characteristic of previous asbestos
exposure?
A. Diffuse parietal pleural thickening involving the costophrenic angles
B. Bilateral calcified plaques on the diaphragmatic pleura
C. Smooth visceral pleural thickening over the lung apices
D. Circumferential mediastinal pleural thickening without calcification
E. A focal pleural mass crossing an interlobar fissure
Explanation: (by Perplexity AI, generated with the OpenAI O3 model)
• Calcified plaques on the diaphragmatic (and parietal) pleura are the classic sentinel sign of asbestos
exposure because this site is seldom affected by other diseases; their bilateral, well-circumscribed,
often calcified appearance reflects prior fibre deposition and pleural reaction.
• Diffuse pleural thickening (A) can follow infection or haemothorax.
• Smooth apical visceral thickening (C) is typical of post-primary TB scarring.
• Non-calcified mediastinal pleural thickening (D) lacks specificity and may occur with any chronic
pleuritis.
• A focal pleural mass crossing a fissure (E) suggests malignant mesothelioma rather than benign
asbestos-related plaque disease.
19. Which of the following conditions classically produces fibrosis that predominantly affects the lung upper
lobes?
A. Ankylosing spondylitis
B. Asbestosis
C. Sarcoidosis
D. Silicosis
E. Tuberculosis
Explanation: (by Perplexity AI, generated with the OpenAI O3 model)
• Asbestosis causes lower-lobe‐predominant interstitial fibrosis because the asbestos fibres settle in
the dependent peripheral lung bases; hence it is the exception in this list.
• Upper-lobe fibrosis is typical of sarcoidosis (perihilar upper-zone scarring), silicosis
(nodular/progressive massive fibrosis in miners), tuberculosis (post-primary fibrocavitary disease in
the apices) and, less commonly, ankylosing spondylitis (apical pleural/subpleural fibrosis).
• These diseases share pathophysiological factors—higher ventilation, oxygen tension and slower
lymphatic clearance—in the upper zones, predisposing them to fibrosis, whereas asbestos exposure
targets the lower lobes.
20. A 12-year-old boy presents with fever and thigh pain; femoral radiograph shows a permeative diaphyseal
lesion with laminated (“onion-skin”) periosteal reaction and a sizable adjacent soft-tissue mass—what is
the most likely diagnosis?
A. Osteoid osteoma
B. Acute osteomyelitis
C. Ewing sarcoma
D. Langerhans cell histiocytosis
E. Osteosarcoma
Explanation: (by Perplexity AI, generated with the OpenAI O3 model)
• The combination of a diaphyseal permeative bone destruction, lamellated periosteal reaction and
bulky soft-tissue component is classic for Ewing sarcoma, a small-round-cell tumour that typically
affects the mid-shaft of long bones in children and adolescents.
• Osteoid osteoma is a small cortical nidus without aggressive periosteal change.
EBDR Exam MCQs & Concepts July 2024
Dr. Kareem Alnakeeb 100
• Acute osteomyelitis can mimic tumour but usually lacks a large organised soft-tissue mass and
shows more ill-defined periosteal lifting.
• Langerhans cell histiocytosis often produces punched-out lesions or beveled edges and minimal
periosteal response.
• Osteosarcoma is more common in the metaphyses and characteristically shows a sunburst or
cloud-like osteoid matrix rather than laminated periosteal layering.
21. A 29-year-old man with primary infertility and a palpable left-sided grade III varicocele undergoes
venography showing reflux in the internal spermatic vein; which interventional radiology technique is
most appropriate to occlude the refluxing vein while preserving arterial flow?
A. Antegrade scrotal sclerotherapy
B. Retrograde coil embolisation via the internal jugular vein
C. Percutaneous balloon angioplasty
D. Laparoscopic Palomo ligation
E. Microsurgical sub-inguinal varicocelectomy
Explanation: (by Perplexity AI, generated with the OpenAI O3 model)
• Retrograde coil embolisation is the standard endovascular treatment for varicocele. Through a
jugular or femoral venous approach, a catheter is advanced into the gonadal vein and coils or glue
are deployed to block reflux, avoiding the testicular artery and lymphatics, with ≥90% technical
success and low recurrence.
• Antegrade sclerotherapy (A) is done through a scrotal incision and is less commonly used.
• Balloon angioplasty (C) treats stenoses, not venous reflux.
• Laparoscopic Palomo (D) and microsurgical varicocelectomy (E) are surgical, not radiological,
options; both require operative ligation rather than intravascular occlusion.
22. A 7-year-old child presents with headache and Parinaud’s syndrome. Non-contrast CT shows a pineal
region mass with scattered “exploded” peripheral calcifications that displace the native pineal
calcification fragments outward. Which tumour most commonly exhibits this calcification pattern?
A. Pineal germinoma
B. Pineoblastoma
C. Pineocytoma
D. Metastasis
E. Pineal meningioma
Explanation: (by Perplexity AI, generated with the OpenAI O3 model)
• Pineal parenchymal tumours (pineoblastoma and pineocytoma) characteristically blast the normal
pineal calcification apart, producing multiple peripheral flecks – the classic “exploded” or
peripherally dispersed pattern.
• Among them, the aggressive, poorly differentiated pineoblastoma is the typical childhood tumour
causing Parinaud’s syndrome and peripheral calcification.
• Germinomas usually engulf the pre-existing calcification centrally, while meningioma and
metastasis seldom arise in the pineal gland and lack this specific pattern.
EBDR Exam MCQs & Concepts July 2024
Dr. Kareem Alnakeeb 101
23. Which type of central venous access device is most commonly inserted via a large vein in the upper arm
for prolonged outpatient chemotherapy delivery?
A. Tunneled Hickman line
B. Implanted port (Port-a-Cath)
C. Peripherally inserted central catheter (PICC)
D. Non-tunnelled subclavian central line
E. Midline peripheral catheter
Explanation: (by Perplexity AI, generated with the OpenAI O3 model)
• A peripherally inserted central catheter (PICC) is a long, flexible tube placed through a basilic,
brachial or cephalic vein above the elbow and advanced so its tip lies in the superior vena cava,
making it ideal for repeated chemotherapy infusions over weeks to months.
• Hickman lines and Port-a-Cath devices are inserted via the chest wall or neck, not the arm, while
non-tunnelled subclavian lines are intended for short-term inpatient use.
• A midline catheter does not reach the central veins, so it cannot safely deliver vesicant
chemotherapy.
24. On cardiac MRI, which imaging plane is obtained by angling perpendicular to the line connecting the
ventricular apex and the centre of the mitral valve annulus, thereby providing stacked cross-sections
through both ventricles?
A. Horizontal long-axis (four-chamber) plane
B. Vertical long-axis (two-chamber) plane
C. Left ventricular outflow tract (three-chamber) plane
D. Short-axis plane
E. Coronal thoracic plane
Explanation: (by Perplexity AI, generated with the OpenAI O3 model)
• The short-axis plane is planned perpendicular to the heart’s long axis (drawn from apex to mitral
valve centre). Sequential slices in this orientation give circular cross-sections of the left and right
ventricles, allowing accurate assessment of ventricular volumes and wall motion.
• The horizontal and vertical long-axis planes (options A and B) run parallel to the long axis, displaying
chambers in longitudinal profile, not cross-section.
• The LV outflow tract or three-chamber view (option C) is oblique, aligned with the aortic and mitral
valves to show the LVOT.
• A standard coronal thoracic plane (option E) is body-orientated, not heart-orientated, and lacks
consistent ventricular short-axis anatomy.
25. A 58-year-old man with Child-Pugh A cirrhosis is found to have a solitary 6 cm hepatocellular carcinoma
in segment VI. Which of the following locoregional therapies is generally NOT considered technically
suitable or oncologically effective for a 6 cm HCC?
A. Conventional trans-arterial chemo-embolisation (TACE)
B. Microwave thermal ablation
C. Radiofrequency thermal ablation
D. Stereotactic body radiotherapy (SBRT)
E. Yttrium-90 trans-arterial radio-embolisation (TARE)
Explanation: (by Perplexity AI, generated with the OpenAI O3 model)
• Radiofrequency ablation is regarded as curative only for tumours ≤3 cm, with substantially higher
incomplete ablation and local recurrence rates once diameter exceeds about 3 – 4 cm; a 6 cm lesion
is therefore unsuitable for RFA.
EBDR Exam MCQs & Concepts July 2024
Dr. Kareem Alnakeeb 102
• In contrast, microwave ablation can treat lesions up to 5–7 cm because of higher intratumoural
temperatures and larger ablation zones; SBRT can deliver ablative doses to tumours up to 6 – 7 cm;
TACE is standard of care for intermediate-size unresectable HCCs; and Y-90 TARE is effective for
larger solitary tumours or multifocal disease.
• Thus, among the options listed, only radiofrequency ablation is generally inappropriate for a 6 cm
HCC.
26. A cardiac MRI shows an avidly contrast-enhancing, broad-based mass infiltrating the right atrial wall and
attached by a thin sessile margin; which diagnosis is most likely?
A. Left-atrial myxoma
B. Organized mural thrombus
C. Metastatic melanoma deposit
D. Primary cardiac angiosarcoma
E. Papillary fibroelastoma
Explanation: (by Perplexity AI, generated with the OpenAI O3 model)
• Malignant cardiac tumours typically present as infiltrative, poorly marginated, vascular masses with
heterogeneous or intense enhancement on post-gadolinium MRI and a broad, sessile attachment;
these features best fit a right-sided primary angiosarcoma.
• Myxomas are usually pedunculated, arise from the inter-atrial septum in the left atrium, and enhance
less avidly.
• Chronic thrombus shows no (or only peripheral) enhancement and often appears non-vascular.
• Cardiac melanoma metastases can be hyper-enhancing but usually occur in patients with known
widespread disease and often involve the left atrium or ventricle.
• Papillary fibroelastomas are small, highly mobile, valvular masses with minimal enhancement, not
infiltrative lesions.
27. A feature that favors a malignant cardiac mass on imaging is:
A. A Lack of first-pass perfusion
B. Thin pedunculated attachment
C. Heterogeneous first-pass contrast enhancement with late gadolinium enhancement
D. Absence of pericardial effusion
Explanation: (by Perplexity AI, generated with the OpenAI O3 model)
• Malignant tumors are vascular, infiltrative and heterogeneously enhance.
• Benign myxomas classically have a thin stalk.
• Thrombi lack enhancement and often have a thin attachment
28. Which primary cardiac tumour is most frequently encountered in adults on routine imaging or at surgery?
A. Cardiac fibroma
B. Papillary fibro-elastoma
C. Cardiac hemangioma
D. Cardiac myxoma
E. Cardiac rhabdomyoma
Explanation: (by Perplexity AI, generated with the OpenAI O3 model)
• Cardiac myxomas account for about half of all primary cardiac tumours in adults and roughly 75%
arise from the left atrial septum, making them the commonest lesion in this age group.
EBDR Exam MCQs & Concepts July 2024
Dr. Kareem Alnakeeb 103
• Fibromas and rhabdomyomas are predominantly paediatric tumours; fibromas are intramural
ventricular masses, while rhabdomyomas are strongly linked to tuberous sclerosis and often regress
spontaneously.
• Papillary fibro-elastomas are the most frequent valvular tumours but overall are less common than
myxomas.
• Cardiac hemangiomas are rare vascular malformations that represent only a small fraction of benign
cardiac masses.
29. In a patient with Marfan syndrome undergoing routine echocardiographic follow-up, which cardiac valve
lesion is encountered most frequently?
A. Mitral stenosis
B. Mitral regurgitation
C. Tricuspid stenosis
D. Tricuspid regurgitation
E. Aortic stenosis
Explanation: (by Perplexity AI, generated with the OpenAI O3 model)
• Myxomatous degeneration of the mitral valve apparatus is common in Marfan syndrome, leading to
mitral valve prolapse and resulting regurgitation—the typical and most prevalent valvular
abnormality in these patients.
• Stenotic lesions of either atrioventricular valve (Options A and C) are rare in Marfan and usually
reflect unrelated rheumatic or congenital pathology.
• Tricuspid regurgitation (Option D) is less frequent and, when present, is often secondary to
pulmonary hypertension rather than primary valvular disease.
• Aortic stenosis (Option E) is not characteristic; Marfan pathology primarily affects the aortic root,
causing dilatation and regurgitation rather than obstruction.
30. On dedicated ocular MRI, which signal intensity combination is most typical for a primary uveal
melanoma?
A. High T1, high T2
B. High T1, low T2
C. High T1, intermediate T2
D. Low T1, low T2
E. Low T1, high T2
Explanation: (by Perplexity AI, generated with the OpenAI O3 model)
• Melanin shortens both T1 and T2 relaxation times. Consequently, pigmented uveal melanomas
characteristically show intrinsic T1 hyper-intensity while appearing hypo-intense on T2-weighted
images. This high-T1/low-T2 pattern is well-described and helps distinguish melanoma from most
other intra-ocular masses, which lack melanin and therefore remain low or iso-intense on T1 and
variably hyper-intense on T2.
• Options A, C and E include a high T2 signal that is uncharacteristic for pigmented melanoma; option
D lacks the typical T1 hyper-intensity produced by melanin.
EBDR Exam MCQs & Concepts July 2024
Dr. Kareem Alnakeeb 104
31. On MRI of a spinal intradural-extramedullary tumour, marked gadolinium enhancement of a tapering
“dural (epidural) tail” is highly specific but not very sensitive for which neoplasm?
A. Astrocytoma
B. Ependymoma
C. Hemangioblastoma
D. Meningioma
E. Schwannoma
Explanation: (by Perplexity AI, generated with the OpenAI O3 model)
• The dural (or epidural) tail sign is produced by reactive vascularised thickening of adjacent dura.
Although it appears in only about 60% of meningiomas (hence limited sensitivity), when present it
points very strongly toward this diagnosis, with reported specificity exceeding 90%.
• Other intradural or extradural tumours such as ependymoma, astrocytoma, hemangioblastoma and
schwannoma can very rarely show a similar tapering enhancement, but this is uncommon, so their
presence does not match the high specificity seen with meningioma.
32. Which posterior cranial fossa tumour is usually intra-axial rather than extra-axial (extramural)?
A. Metastasis
B. Vestibular schwannoma
C. Meningioma
D. Ependymoma
E. Hemangioblastoma
Explanation: (by Perplexity AI, generated with the OpenAI O3 model)
• Ependymomas arise from the ependymal lining of the fourth ventricle and adjacent brain
parenchyma, so they lie within the neural tissue of the posterior fossa (intra-axial); they are therefore
not classed as extra-axial (extramural) lesions.
• Posterior-fossa extra-axial tumours include vestibular schwannoma, meningioma and dural or
leptomeningeal metastases, all of which originate outside the brain parenchyma and displace it.
• Hemangioblastomas are also usually intra-axial but cystic–solid morphology and vascular flow voids
help separate them from ependymoma.
• Key distractors therefore describe classic extra-axial masses, leaving ependymoma as the only intra-
axial option.
33. At what level does the cauda equina begin in the average adult?
A. Above T12
B. Below L1
C. L2/3
D. L3/4
E. L4/5
Explanation: (by Perplexity AI, generated with the OpenAI O3 model)
• The cauda equina begins below the termination of the spinal cord (conus medullaris), which occurs
at the L1-L2 vertebral level in the average adult.
• The spinal cord tapers and ends between the first and second lumbar vertebrae, with the cauda
equina consisting of nerve roots L2-S5 and the coccygeal nerve extending distally from this point.
• While the conus medullaris may vary from T12 to L3 in different individuals, it typically lies around L1
level, making "below L1" the most accurate description of where the cauda equina begins.
• Options C and D represent levels within the cauda equina rather than its commencement, while
option A is too superior and represents spinal cord rather than cauda equina territory.
EBDR Exam MCQs & Concepts July 2024
Dr. Kareem Alnakeeb 105
34. What is the most common congenital anomaly of the pancreas encountered in everyday radiology
practice?
A. Annular pancreas
B. Dorsal pancreatic agenesis
C. Pancreas divisum
D. Heterotopic (ectopic) pancreas
E. True congenital pancreatic cyst
Explanation: (by Perplexity AI, generated with the OpenAI O3 model)
• Pancreas divisum results from failure of fusion of the dorsal and ventral pancreatic ducts during the
7th gestational week.
• Autopsy and MRCP studies show it in 4–14% of the population, making it the commonest congenital
pancreatic anomaly.
• Annular pancreas is far rarer (≈1/20,000) and typically presents with duodenal obstruction in
neonates.
• Dorsal agenesis is very uncommon and often associated with other anomalies.
• Heterotopic pancreas and true congenital pancreatic cysts are both rare incidental findings.
35. A 9-month-old infant presents with progressive jaundice. Ultrasound shows a solid, vascular mass
centred at the porta hepatis that extends into the common bile duct causing marked intra- and
extrahepatic duct dilatation. Serum alpha-fetoprotein is normal. What is the most likely diagnosis?
A. Embryonal rhabdomyosarcoma
B. Infantile haemangioma
C. Hepatoblastoma
D. Choledochal cyst with dysplasia
E. Neuroblastoma
Explanation: (by Perplexity AI, generated with the OpenAI O3 model)
• Embryonal rhabdomyosarcoma is the commonest malignant tumour of the biliary tree in infancy and
typically arises at the porta hepatis, invading the bile ducts to produce obstructive jaundice; it is
usually AFP-negative.
• Hepatoblastoma (C) generally originates within hepatic parenchyma, elevates AFP and only
secondarily compresses ducts rather than growing intraductally.
• Infantile haemangioma (B) is hypervascular but does not infiltrate ducts and seldom causes
obstruction.
• A choledochal cyst with dysplasia (D) presents as cystic dilatation, not a solid vascular mass.
• Neuroblastoma (E) arises from the adrenal or sympathetic chain, may encase vessels but rarely
invades the biliary ducts or presents primarily at the porta hepatis.
EBDR Exam MCQs & Concepts July 2024
Dr. Kareem Alnakeeb 106
36. A 65-year-old man presents with progressive, painless jaundice. MRI with MRCP shows a solid mass at
the porta hepatis extending within the common hepatic duct and proximal common bile duct, producing
an abrupt, band-like stricture (“shouldering”) and marked upstream biliary dilatation. Which single
diagnosis best explains these findings?
A. Cholangiocarcinoma (hilar type)
B. Gallbladder carcinoma invading the bile duct
C. Pancreatic head adenocarcinoma
D. Primary sclerosing cholangitis
E. Metastatic lymph-node compression
Explanation: (by Perplexity AI, generated with the OpenAI O3 model)
• Hilar (Klatskin) cholangiocarcinoma arises at the confluence of the hepatic ducts and commonly
grows intraductally, creating an annular, shouldered stricture with proximal ductal dilatation but little
distal duct involvement, as in this case.
• Gallbladder carcinoma can extend into the bile duct but usually originates from a gallbladder mass,
often with cholelithiasis, which is not described.
• Pancreatic head cancer obstructs the distal common bile duct rather than the porta hepatis.
• Primary sclerosing cholangitis causes multifocal, beaded narrowing, not a solitary mass.
• Metastatic nodes compress ducts extrinsically without intraductal extension or a discrete porta
hepatis mass.
37. Following lumbar discectomy, a patient re-presents with radicular pain. MRI with gadolinium is
performed. Which MRI appearance most reliably indicates recurrent disc herniation rather than
postoperative scar tissue?
A. Extradural lesion that fails to enhance after contrast
B. Extradural lesion that avidly enhances after contrast
C. Intradural lesion that avidly enhances after contrast
D. Intradural lesion that fails to enhance after contrast
E. Uniform enhancement of the surgical bed and surrounding fat planes
Explanation: (by Perplexity AI, generated with the OpenAI O3 model)
• Recurrent nucleus pulposus remains avascular; on contrast-enhanced MRI it appears as an
extradural mass that shows little or no enhancement, often surrounded by a thin enhancing rim of
granulation tissue.
• Postoperative epidural scar, by contrast, is vascularised granulation tissue and therefore
demonstrates diffuse or nodular enhancement after gadolinium (ruling out option B).
• Intradural lesions (options C and D) are anatomically incorrect for disc recurrence and instead
suggest arachnoiditis or tumour.
• Uniform enhancement of the operative site (option E) reflects expected postoperative granulation
and fibrotic change, not disc material.
EBDR Exam MCQs & Concepts July 2024
Dr. Kareem Alnakeeb 107
38. On cardiac MRI, which characteristic imaging feature most strongly suggests cardiac amyloidosis?
A. Patchy mid-myocardial late gadolinium enhancement
B. Focal asymmetric septal hypertrophy
C. Diffuse subendocardial or transmural late gadolinium enhancement with difficulty nulling the
myocardium
D. Apical thinning with hyper-trabeculation
E. Right-sided ventricular outflow tract obstruction
Explanation: (by Perplexity AI, generated with the OpenAI O3 model)
• Cardiac amyloidosis causes widespread interstitial deposition of amyloid fibrils, expanding the
extracellular volume and producing diffuse subendocardial or transmural late gadolinium
enhancement (LGE) on inversion-recovery sequences; the abnormal myocardium nulls at the same
time as blood, making correct TI selection difficult.
• Patchy enhancement (A) can occur but is less specific. Hypertrophic cardiomyopathy typically shows
focal asymmetric septal hypertrophy (B).
• Apical thinning with trabeculation (D) is characteristic of left ventricular non-compaction.
• Right ventricular outflow obstruction (E) suggests conditions such as pulmonary stenosis or
carcinoid heart disease, not amyloid infiltration.
39. In a patient with atrial fibrillation, which imaging technique is regarded as the reference standard for
quantitative calculation of left-ventricular ejection fraction?
A. CT coronary angiography
B. MR coronary angiography
C. Left ventricular cine angiography during cardiac catheterisation
D. Planar radionuclide ventriculography (MUGA scan)
E. PET-CT perfusion imaging
Explanation: (by Perplexity AI, generated with the OpenAI O3 model)
• Multigated planar radionuclide ventriculography (MUGA) remains the benchmark for precise, beat-
to-beat quantification of left-ventricular ejection fraction, even when rhythm is irregular as in atrial
fibrillation.
• Gamma-camera acquisition over hundreds of cardiac cycles averages out cycle-length variability,
giving reproducible EF with <5% inter-study error.
• Cardiac MRI offers excellent spatial resolution but suffers from mis-gating in AF and longer breath-
holds; CT angiography is optimised for coronary anatomy rather than ventricular volumes; invasive
ventriculography provides single-beat EF only and is affected by arrhythmia-related variation; PET-CT
focuses on metabolism or perfusion and does not directly compute EF..
EBDR Exam MCQs & Concepts July 2024
Dr. Kareem Alnakeeb 108
40. In patients with atrial fibrillation, the most accurate non-invasive modality for quantifying LVEF is:
A. Transthoracic or trans-esophageal echocardiography (2-D & 3-D)
B. Cardiac CT angiography (CTA)
C. Cardiac MR (CMR)
D. Invasive left ventriculography
E. Gated SPECT myocardial perfusion (“MUGA” or perfusion-gated)
F. Gated PET-CT
Explanation: (by Perplexity AI, generated with the OpenAI O3 model)
Practical take-aways for patients in AF
• Best overall accuracy: CMR with arrhythmia-rejection remains the reference when precise LVEF is
required for therapy decisions (e.g., device eligibility).
• Best balance of speed and availability: 3-D echocardiography averaged over multiple
representative beats is adequate for most routine assessments; always report the number of beats
averaged.
• CT angiography is reliable when already indicated for coronary imaging, but use dose-modulating
techniques sparingly in fast AF because they discard irregular beats.
• Nuclear techniques are acceptable when simultaneous perfusion data are needed; ensure tight
beat-rejection windows to avoid systematic LVEF depression.
• Single-beat invasive ventriculography is now reserved for catheter-lab situations where no other
modality is available.
EBDR Exam MCQs & Concepts July 2024
Dr. Kareem Alnakeeb 109
41. A child who undergoes MR of the brain for clinically apparent facial abnormalities is shown to have a
defect of midline cleavage of the brain. What structure is abnormal or absent in all forms of
holoprosencephaly, and therefore is the most sensitive indicator of a midline cleavage
abnormality?
A. falx cerebri
B. third ventricle
C. fourth ventricle
D. corpus callosum
E. septum pellucidum
Source: Bydder, Megan, et al. Get Through Final FRCR Part A: SBAs for the Modular Examination. 1st
ed., Hodder Education Publishers, 2009.
Explanation:
Overview
• Holoprosencephaly is failure of the primitive brain to cleave into two hemispheres, and is commonly
associated with midline facial abnormalities (ranging from cyclopia to hypertelorism) and absence of
many intracranial midline structures.
Classification
There are three types;
Alobar Form
• the most severe being the alobar form, which shows no cleavage at all, with absence of the falx
cerebri and third ventricle, fusion of the cerebral hemispheres and thalami, and a single large lateral
ventricle.
Semilobar Form
• The semilobar form has variable cleavage with a partially formed falx, rudimentary third ventricle,
and variable cleavage of the thalami, lateral ventricles and cerebral hemispheres.
Lobar Form
• In the lobar type of holoprosencephaly, brain formation may be nearly normal, but the septum
pellucidum is always absent, as in all forms.
• The falx, corpus callosum and ventricular system may be normal in the lobar type.
42. A plain lumbar spine radiograph of a 45-year-old woman shows marked posterior scalloping of the
vertebral bodies extending over several vertebral lengths. All of the following are diseases associated with
this finding except
K. Marfan
L. Neurofibromatosis
M. Ependymoma
N. Achondroplasia
O. Hypothyroidism
Source: Chin, Teck Yew, et al. Get Through Final FRCR 2A: SBAs. 1st ed., CRC Press, 2017.
Explanation:
Mass-Related Causes
• A common cause of localised posterior vertebral scalloping is increased intraspinal pressure
secondary to an expanding mass.
• Widening of the interpediculate distance and alteration of the configuration of the pedicles are
associated signs.
• Relatively large, slow-growing lesions that originate during a period of active skeletal growth (such as
ependymomas) are most likely to give rise to posterior vertebral scalloping.
Dural Ectasia and Connective-Tissue Disorders
EBDR Exam MCQs & Concepts July 2024
Dr. Kareem Alnakeeb 110
• Dural ectasia is thought to cause posterior vertebral scalloping due to loss of the normal protection
provided to the vertebral body by a strong, intact dura.
• Dural ectasia classically occurs in association with inherited connective-tissue disorders such as
Marfan syndrome (classical) and Ehlers–Danlos syndrome.
Neurofibromatosis and Related Conditions
• Posterior vertebral scalloping is also commonly seen in patients with neurofibromatosis, most likely
due to dural ectasia but also secondary to neurofibromas or a thoracic meningocoele.
• It has also been reported in patients with AS; in these cases, the development of associated
arachnoid cysts may give rise to cauda equina syndrome.
Endocrine Disorders
• Acromegaly has been described as a further cause of diffuse posterior vertebral scalloping, probably
because of a combination of soft-tissue hypertrophy in the spinal canal and increased bone
resorption.
43. A 25 year old man has a routine chest radiograph prior to a work permit application. It demonstrates a
well-defined, rounded mediastinal mass. Which of the following features on CT would make a diagnosis
of bronchogenic cyst less likely?
A. Soft-tissue density
B. Thick wall
C. Precarinal location
D. Communication with tracheal lumen
E. Unilocularity
Source: Currie, Stuart, et al. SBAs for the FRCR 2A. 1st ed., Cambridge University Press, 2010.
Explanation:
• Bronchogenic cyst is the most common intrathoracic foregut duplication cyst.
• It could have all the above features, but in a mediastinal location, the cyst walls are usually thin.
Thick-walled cysts are more likely to be oesophageal.
44. On an axial section of the brain at the level of the third ventricle, which structure lies immediately lateral
to the putamen?
A. internal capsule
B. globus pallidus
C. external capsule
D. thalamus
E. insular cortex
Source: Bydder, Megan, et al. Get Through Final FRCR Part A: SBAs for the Modular Examination. 1st
ed., Hodder Education Publishers, 2009.
Explanation:
• The lentiform nucleus is composed of a larger lateral component (the putamen) and a smaller medial
component (the globus pallidus), separated by a sheet of white matter.
• The lentiform nucleus is bounded medially by the internal capsule. Lateral to the lentiform nucleus
lies the white matter of the external capsule, and then the claustrum, a thin sheet of grey matter.
• The extreme capsule lies lateral to the claustrum, and separates it from the insular cortex.
EBDR Exam MCQs & Concepts July 2024
Dr. Kareem Alnakeeb 111
45. A 50 year old male presents with a history of occasional haemoptysis and exertional shortness of breath
which has been getting progressively worse. Plain chest radiograph demonstrates bibasal reticular
shadowing with volume loss. HRCT demonstrates bibasal fibrosis and traction bronchiectasis. Incidental
note is made of a patulous oesophagus. Which of the following is the most likely cause?
A. Tuberculosis
B. SLE
C. Rheumatoid arthritis
D. Wegener’s granulomatosis
E. Scleroderma
Source: Currie, Stuart, et al. SBAs for the FRCR 2A. 1st ed., Cambridge University Press, 2010.
Explanation:
• Whilst haemoptysis may be a presentation in tuberculosis and Wegener’s and bibasal fibrosis maybe
seen in all of the above except tuberculosis (where apical fibrosis is the more likely feature),
scleroderma is the only condition resulting in a patulous lower oesophageal sphincter, oesophageal
shortening and stricture formation.
46. Which of the following is NOT an indication for percutaneous nephrostomy?
A. Benign obstruction of the ureter
B. Urinary leakage or fistula
C. Pyelonephritis in a non-dilated renal pelvis
D. Malignant obstruction of the ureter
47. A 36 year old female with history of pelvic pain and severe dysmenorrhoea undergoes a pelvic ultrasound
examination which reveals uterine fibroid disease. Which of the following imaging features would be
associated with the best outcome following uterine artery embolisation?
A. Submucosal location
B. Subserosal location
C. Associated adenomyosis
D. Calcification
E. Multiple fibroids
Source: Currie, Stuart, et al. SBAs for the FRCR 2A. 1st ed., Cambridge University Press, 2010.
Explanation:
• Subserosal fibroids, especially pedunculated ones, may often draw their blood supply from adjacent
viscera, which may be a cause of failure of the procedure. They are also associated with a higher
incidence of complications.
• Calcific fibroids are less vascular and may not respond well to embolisation.
• Bulky and multiple fibroids may need multiple interventions or surgery.
• Adenomyosis is a known cause for failure of the procedure.
48. A patient with a known collagen vascular disease has pulmonary fibrosis. HRCT reveals bilateral lower
lobe bronchiectasis. Which collagen vascular disease is most likely?
A. Sjogren syndrome
B. Progressive systemic sclerosis
C. SLE
D. Rheumatoid arthritis
E. Dermatomyositis
Source: Currie, Stuart, et al. SBAs for the FRCR 2A. 1st ed., Cambridge University Press, 2010.
Explanation:
• Whilst pulmonary fibrosis is a feature of all the above conditions, bronchiectasis is most likely seen
in Sjogren syndrome.
EBDR Exam MCQs & Concepts July 2024
Dr. Kareem Alnakeeb 112
49. Eight days after lung transplantation for alpha-1 antitrypsin deficiency, a 45 year old man develops
pyrexia, breathlessness and desaturation. HRCT reveals perihilar heterogenous opacities and ground
glass changes with new pleural effusion and septal thickening. Which of the following is the most likely
cause?
A. Reperfusion oedema
B. Acute rejection
C. Anastomotic dehiscence
D. Post-transplantation PCP infection
E. Hyperacute rejection
Source: Currie, Stuart, et al. SBAs for the FRCR 2A. 1st ed., Cambridge University Press, 2010.
Explanation:
• Hyperacute rejection presents within hours of the transplantation.
• Reperfusion oedema usually presents within 24 hours of the transplantation, peaking by about day
four.
• Posttransplant infections can be broadly divided into those occurring within the first month (gram-
negative bacteria, fungi (candida, aspergillosis)) and those occurring after the first month (CMV,
PCP).
• Anastomotic dehiscence is usually an early feature, but the presentation and features are not those
described.
50. A young man presents with progressive productive cough and halitosis. He had severe pneumonia as a
child. Plain chest radiograph demonstrates bronchial dilatation and bronchial wall thickening with some
volume loss. Which of the following HRCT findings is the most sensitive finding for bronchiectasis?
A. Air trapping
B. Mucous-filled dilated bronchi
C. Bronchial wall thickening
D. Bronchi seen in the subpleural region
E. Lack of bronchial tapering
Source: Currie, Stuart, et al. SBAs for the FRCR 2A. 1st ed., Cambridge University Press, 2010.
Explanation:
• Whilst all the above can be seen in patients with bronchiectasis, a lack of progressive tapering of the
bronchi is the most sensitive (80%).
51. A 26 year old female patient with an optic nerve tumour and café-au-lait spots presents with exertional
breathlessness. Imaging of the chest is most likely to reveal which of the following?
A. Multiple small lower lobe cysts
B. Emphysema
C. Lower zone fibrosis
D. Thick-walled cavities in the upper zone
E. Asymmetrical upper zone fibrosis
Source: Currie, Stuart, et al. SBAs for the FRCR 2A. 1st ed., Cambridge University Press, 2010.
Explanation:
• The case describes neurofibromatosis I, which is associated with lower zone fibrosis and thin-walled
bullae, mainly in the upper zones. Apart from the pulmonary changes, skeletal abnormalities
involving the ribs and spine and mediastinal masses may also be seen.
52. A 22 year old is diagnosed with tuberculosis. Which of the following features will make a diagnosis of
primary tuberculosis more likely?
A. Mediastinal enlargement
B. Septal thickening
C. Upper zone cavitation
D. Miliary nodules
E. Apical consolidation
Source: Currie, Stuart, et al. SBAs for the FRCR 2A. 1st ed., Cambridge University Press, 2010.
EBDR Exam MCQs & Concepts July 2024
Dr. Kareem Alnakeeb 113
Explanation:
• Mediastinal lymph node enlargement is a feature of primary TB. The others are seen with reactivation
or fibrocavitary TB. Miliary TB can be seen in any phase with haematogenous dissemination but
primary presentation is uncommon.
53. A 68 year old miner develops an irregular opacity in the upper zone on plain chest radiograph. Which
imaging feature would be more in favour of malignancy than progressive massive fibrosis (PMF)?
A. Peripheral enhancement on contrast-enhanced MR
B. Peripheral location on axial images
C. Presence of calcification
D. High signal on T2-weighted images
E. Avid lesion on PET-CT
Source: Currie, Stuart, et al. SBAs for the FRCR 2A. 1st ed., Cambridge University Press, 2010.
Explanation:
• PMF has a peripheral location which moves towards the hilum on follow-up imaging. Calcification
and cavitation may also be seen. PMF lesions can be FDG-avid on PET-CT. However, high signal in a
mass on T2-weighted images is strongly suspicious for malignancy.
54. In acute respiratory distress syndrome; what is the first change usually seen on the chest radiograph?
A. confluent consolidation
B. pleural effusions
C. increased heart size with globular shape
D. volume loss with atelectasis
E. patchy ill-defined opacities
Source: Bydder, Megan, et al. Get Through Final FRCR Part A: SBAs for the Modular Examination. 1st
ed., Hodder Education Publishers, 2009.
Explanation:
• Acute respiratory distress syndrome (ARDS) commences with interstitial oedema, progressing to
congestion and extensive alveolar, and interstitial oedema and haemorrhage. The chest radiograph is
often normal for the first 24 hours, before patchy opacities appear in both lungs. These progress to
massive airspace consolidation over the following 24–48 hours. True volume loss, atelectasis,
cardiomegaly and effusions are not seen in ARDS.
55. A middle-aged man presents with easy fatigability. CT shows an anterior mediastinal mass with areas of
calcifications, invading the mediastinal structures. There are multiple small pleural masses. What is the
most likely diagnosis?
A. Thymoma
B. Thymic lipoma
C. Lymphoma
D. Teratoma
E. Asbestosis
Source: Chin, Teck Yew, et al. Get Through Final FRCR 2A: SBAs. 1st ed., CRC Press, 2017.
Explanation:
Classification and Clinical Associations
• Thymomas are classified as encapsulated, infiltrative and metastasizing, with pulmonary and pleural
deposits and thymic carcinoma.
• Half the thymomas are asymptomatic and 30% are associated with myasthenia gravis.
CT Imaging Features
• At CT a benign thymoma appears round, oval or lobulated.
• Focal calcification is seen in 25%, which may be dense, irregular or coarse.
• Benign thymomas show mild homogeneous enhancement; cystic changes are also described.
• Invasive thymoma are heterogeneous in appearance; pericardial and pleural nodules suggest
malignancy.
EBDR Exam MCQs & Concepts July 2024
Dr. Kareem Alnakeeb 114
• Egg-shell calcification is described in invasive thymoma.
• Absent fat planes between thymoma and mediastinum does not necessarily reflect invasion.
56. All of the following are causes of lower zone fibrosis, except
A. Amiodarone
B. Idiopathic pulmonary fibrosis
C. Asbestosis
D. Ankylosing spondylitis
E. Neurofibromatosis I
Source: Chin, Teck Yew, et al. Get Through Final FRCR 2A: SBAs. 1st ed., CRC Press, 2017.
Explanation:
Lower Zone Fibrosis: Causes
• Causes of lower zone fibrosis include asbestosis, aspiration, cryptogenic alveolitis (IPF),
neurofibromatosis I and tuberous sclerosis; connective tissue diseases like RA, scleroderma and
SLE; and drug toxicity to substances like amiodarone and nitrofurantoin.
57. A 58 year old male has a CT staging scan following a diagnosis of adenocarcinoma of the body of the
pancreas. The tumour is 3 cm in size and extends beyond the boundaries of the pancreas but does not
invade any vessels or adjacent organs. Two 1 cm lymph nodes lie adjacent to the tumour. No other
nodes, or metastatic disease in the chest, abdomen or pelvis, are identified. The tumour is best staged as
which one of the following?
A. T1N0M0
B. T1N1M0
C. T2N0M0
D. T3N0M0
E. T3N1M0
Source: Currie, Stuart, et al. SBAs for the FRCR 2A. 1st ed., Cambridge University Press, 2010.
Explanation:
• T1 tumour is disease confined to the pancreas and less than 2 cm in diameter.
• T2 tumour is also confined to the pancreas but greater than 2 cm in diameter.
• As the tumour extends beyond the boundary of the pancreas, it is at least T3.
• Invasion of the coeliac or superior mesenteric arteries would make this a T4 tumour, but as these
features are not present it is T3.
• The presence of regional nodes make it N1 rather than N0 (no nodes involved), and there is no
metastatic disease so it is M0.
• Therefore the correct radiological stage is T3N1M0.
EBDR Exam MCQs & Concepts July 2024
Dr. Kareem Alnakeeb 115
58. A 52 year old male with a metal heart valve has a transrectal ultrasound performed to stage rectal
carcinoma as MRI is contraindicated. A 3 cm hypoechoic mass is identified from three to seven o’clock in
the lower rectum. It extends through an inner hypoechoic layer and into the outer hypoechoic layer, but
the outermost hyperechoic layer is intact and unaffected. What is the correct T staging (TNM system)
based on these observations?
A. T0
B. T1
C. T2
D. T3
E. T4
Source: Currie, Stuart, et al. SBAs for the FRCR 2A. 1st ed., Cambridge University Press, 2010.
Explanation:
• The layers of the rectum are well demonstrated at transrectal ultrasound.
o The innermost hyperechoic layer represents the balloon-mucosa interface,
o the middle hyperechoic layer represents the submucosa and
o the outermost hyperechoic layer represents the serosa.
• The tumour described in the question extends through the submucosa into the muscularis propria
(outer hypoechoic layer) but does not involve the serosa.
o T1 disease is limited to the submucosa,
o T2 is limited to the muscularis propria,
o T3 extends through the serosa and
o T4 represents invasion of adjacent organs.
• The correct staging for the tumour described in the question is therefore T2.
EBDR Exam MCQs & Concepts July 2024
Dr. Kareem Alnakeeb 116
Paper 2
1. Concerning dislocations: Which statement is CORRECT?
A. Posterior dislocation of the hip accounts for 10-20% of all hip dislocations
B. Anterior dislocation of the shoulder accounts for less than 30% of glenohumeral dislocations
C. A Bankart lesion is a fracture of the anterior aspect of the superior rim of the glenoid
D. Dislocation of the patella is usually lateral.
Source: Hussain, Shahid, et al. Radiology MCQs for the New FRCR Part 2A. 1st ed., TFM Publishing,
2006.
Explanation:
• Anterior dislocation of the hip accounts for 10-20% of all hip dislocations - lies medial and inferior to
acetabulum on pelvis X-ray
• Posterior dislocations of both radius and ulna account for 80-90% of elbow dislocations - isolated
dislocation of the radial head is rare
• Anterior dislocation of the shoulder accounts for more then 90% of glenohumeral dislocations - 97%
are anterior dislocations. Associated with a Hill-Sachs lesion which is a defect in the posterolateral
aspect of the humeral head
• A Bankhart lesion is a fracture of the anterior aspect of the inferior rim of the glenoid
• Dislocation of the patella is usually lateral
2. On imaging, which of the followings causes 'Bone within Bone' appearance?
A. Marfan's syndrome
B. Sickle cell disease
C. Rickets.
D. Fibrous dysplasia
Source: Hussain, Shahid, et al. Radiology MCQs for the New FRCR Part 2A. 1st ed., TFM Publishing,
2006.
Explanation:
• The causes include:
o Congenital syphilis
o Infantile cortical hyperostosis
o Sickle cell disease
o Oxalosis
o Paget's disease
o Acromegaly and radiation
EBDR Exam MCQs & Concepts July 2024
Dr. Kareem Alnakeeb 117
3. Which of the followings is a feature of diaphyseal aclasia (hereditary multiple exostosis)?
A. Autosomal recessive inheritance
B. Exostoses arise from the metaphysis and point towards the joint
C. Exostoses stop growing when the nearest epiphyseal center fuses
D. Malignant transformation to chondrosarcoma occurs in 35-40%
Source: Hussain, Shahid, et al. Radiology MCQs for the New FRCR Part 2A. 1st ed., TFM Publishing,
2006.
Explanation:
• AD, presents at 2-10 years of age
• Exostoses have a cap of hyaline cartilage, often with a bursa formation over the cap
• Exostoses arise from metaphysis of long bones near epiphyses and point away from the joint
• Exostoses stop growing when the nearest epiphyseal centre fuses
• Malignant transformation to chondrosarcoma occurs in <5%
4. Malignant fibrous histiocytoma: Which statement is CORRECT?
A. Is the commonest soft tissue sarcoma in females >18 years of age
B. Rarely calcifies
C. Is most commonly found in a retroperitoneal location.
D. Angiomatoid malignant fibrous histiocytoma is frequently seen in <20-year-olds
Source: Hussain, Shahid, et al. Radiology MCQs for the New FRCR Part 2A. 1st ed., TFM Publishing,
2006.
Explanation:
• Is the commonest soft tissue sarcoma in adults >45 years of age
• Presents as a painless soft tissue mass - imaging features of low signal on T1 /high signal on T2 with
variable contrast enhancement
• Rarely calcifies
• 75% are found in the extremities, lower limb > upper limb
• Angiomatoid malignant fibrous histiocytoma is frequently seen in <20-year-olds
5. Regarding myositis ossificans: Which statement is CORRECT?
A. 10-20% of lesions undergo malignant transformation
B. In the acute stages, lesions undergo no contrast enhancement on MRI
C. On a plain radiograph, lesions are seen to be in contact with the periosteum
D. It affects the large muscles of the extremities in 80-90% of cases
Source: Hussain, Shahid, et al. Radiology MCQs for the New FRCR Part 2A. 1st ed., TFM Publishing,
2006.
Explanation:
• Myositis ossificans is a benign, non-neoplastic condition characterized by heterotopic ossification
within muscle, most commonly affecting large muscles of the limbs in 80-90% of cases. It does not
undergo malignant transformation. Commoner in adults. M:F 1:1
• In the acute stage, the lesions typically show contrast enhancement on MRI due to inflammation and
hyperemia.
• Radiographically, lesions are separated from the periosteum by a radiolucent zone.
• Burns are indeed a recognized predisposing factor because they cause muscle injury.
6. Telangiectatic osteosarcoma: Which statement is CORRECT?
A. The commonest type of osteosarcoma
B. Painless
C. High intensity signal on T2 and low signal in T1 weighted MRI
D. Most commonly found in patients 60-80 years of age
Source: Hussain, Shahid, et al. Radiology MCQs for the New FRCR Part 2A. 1st ed., TFM Publishing,
2006.
Explanation:
EBDR Exam MCQs & Concepts July 2024
Dr. Kareem Alnakeeb 118
• Telangiectatic osteosarcoma is a highly aggressive subtype of osteosarcoma, characterized by
blood-filled spaces and rapid local destruction.
• It presents most often with pain and swelling, typically in adolescents and young adults (15–35
years). It is not the most common osteosarcoma variant; the conventional type is more frequent.
• On MRI, telangiectatic osteosarcoma exhibits high T2 signal due to its cystic, fluid-filled nature, not
low T2 signal.
• It rarely occurs in those aged 60–80 years; older patients are much less affected.
• Pain is the commonest presenting symptom rather than absence of pain.
7. Regarding eosinophilic granuloma: Which statement is CORRECT?
A. Lesions in proximal long bones are usually metaphyseal
B. The commonest site is the mandible
C. Lesions rarely elicit a periosteal reaction
D. It is a recognized cause of 'floating teeth' appearance
Source: Hussain, Shahid, et al. Radiology MCQs for the New FRCR Part 2A. 1st ed., TFM Publishing,
2006.
Explanation:
• The commonest site of eosinophilic granuloma is the skull.
• Lesions in proximal long bones are usually diaphyseal, but the skull remains the most common site
overall.
• On MRI, eosinophilic granuloma typically shows increased signal on T1-weighted images, not low
signal, due to xanthomatous histiocytes.
• Lesions can be expansile and lytic, often with periosteal reaction and endosteal scalloping.
• Eosinophilic granuloma is a recognized cause of the 'floating teeth' appearance in the jaw.
8. Which of the following statements is CORRECT?
A. Paget's disease has a prevalence of 10% in people over the age of 80 years of age
B. Developmental dysplasia of the hip is more common in males
C. Diffuse idiopathic skeletal hyperostosis commonly presents in children
D. The highest incidence of fibrous dysplasia is between 30-50 years of age
Source: Hussain, Shahid, et al. Radiology MCQs for the New FRCR Part 2A. 1st ed., TFM Publishing,
2006.
Explanation:
• Paget’s disease of bone becomes increasingly prevalent with age, reaching approximately 10% in
those over 80 years old, and is uncommon in people under 40.
• Ankylosing spondylitis is actually more common in Caucasians than Black populations, with a
reported ratio of around 3:1.
• Developmental dysplasia of the hip is much more common in girls, not boys.
• Diffuse idiopathic skeletal hyperostosis almost always presents in adults over 50, not children.
• Fibrous dysplasia has its highest incidence in childhood and adolescence, especially between ages 3
and 15, with the majority of cases presenting under age 30.
EBDR Exam MCQs & Concepts July 2024
Dr. Kareem Alnakeeb 119
9. Which of the following is a feature of adamantinoma?
A. Most common presentation in patients >50 years of age
B. Over 90% occur in the tibia
C. Osteosclerotic lesion
D. Avascularity
Source: Hussain, Shahid, et al. Radiology MCQs for the New FRCR Part 2A. 1st ed., TFM Publishing,
2006.
Explanation:
• Adamantinoma is a rare, low-grade malignant bone tumor that typically affects younger adults
between 10–50 years, not those over 50.
• It has a striking predilection for the tibial diaphysis, with over 90% of cases arising here and most
commonly affecting the mid-shaft.
• Radiologically, adamantinoma is classically osteolytic rather than sclerotic.
• Although lung metastases may occur, they are seen in approximately 10% of cases—not considered
‘rare’ for this tumor type.
• The lesion is not avascular; on the contrary, it typically displays prominent vascularity, which aids in
distinguishing it from other bone lesions. Thus, the most characteristic feature is its marked tibial
predilection.
10. Which of the following statements regarding seronegative arthritis is CORRECT?
A. Ankylosing spondylitis and inflammatory bowel disease typically cause bilateral symmetrical
sacroiliac joint disease
B. Large joint involvement in psoriatic arthropathy is common
C. The interphalangeal joint of the great toe is rarely affected in Reiter's syndrome
D. Psoriatic arthropathy is associated with bilateral syndesmophytes
Source: Hussain, Shahid, et al. Radiology MCQs for the New FRCR Part 2A. 1st ed., TFM Publishing,
2006.
Explanation:
• Bilateral symmetrical sacroiliitis is the hallmark of ankylosing spondylitis, a key seronegative
spondyloarthropathy.
• While psoriatic arthritis and Reiter’s syndrome often produce an asymmetrical pattern in the
sacroiliac joints, ankylosing spondylitis and the spondyloarthropathy associated with inflammatory
bowel disease typically cause bilateral symmetrical disease.
• Psoriatic arthropathy more commonly involves small joints in a rheumatoid-like distribution and has
asymmetrical, frequently unilateral syndesmophytes (not bilateral and thin).
• Reiter’s syndrome more often affects the feet, especially the interphalangeal joint of the great toe,
and does not show marked DIP erosions as a key feature.
11. Which of the following is a feature of periosteal osteosarcoma?
A. This is the second commonest subtype of osteosarcoma
B. Typically involves the diaphysis of long tubular bones
C. Extension of tumor usually involves the medullary cavity
D. Prognosis of this tumor is better than that of parosteal osteosarcoma
Source: Hussain, Shahid, et al. Radiology MCQs for the New FRCR Part 2A. 1st ed., TFM Publishing,
2006.
Explanation:
• Periosteal osteosarcoma most often affects the diaphyseal regions of long bones, especially the
femur and tibia.
• Though aggressive, it is less common than conventional and parosteal subtypes. It is rare. Seen in
10-20-year-olds.
• It rarely involves the medullary cavity, tending instead to originate and grow from the periosteal
surface, with extension into adjacent soft tissues.
• Its prognosis is less favourable than for parosteal osteosarcoma.
EBDR Exam MCQs & Concepts July 2024
Dr. Kareem Alnakeeb 120
• Unlike osteochondromas, the tumour base is broad, not pedunculated.
12. Which of the followings is a skeletal feature of thalassemia major?
A. Central nidus
B. Bone sclerosis.
C. Narrowing of medullary cavity
D. Premature fusion of epiphysis
Source: Hussain, Shahid, et al. Radiology MCQs for the New FRCR Part 2A. 1st ed., TFM Publishing,
2006.
Explanation:
• Erlenmeyer flask deformity, osteoporosis, and premature fusion of the epiphysis (seen in 10%) are
well-recognised skeletal features in thalassaemia major.
• Arthropathy can also occur, usually as a complication secondary to haemochromatosis or calcium
pyrophosphate deposition disease (CPPD). However, narrowing of the medullary cavity is not a
feature—instead, patients show medullary cavity expansion due to marrow hyperplasia from chronic
anaemia. Recognising this distinction is key, as marrow expansion leads to characteristic bone
changes in thalassaemia.
13. Concerning Brodie's abscess: Which statement is CORRECT?
A. Most common in the elderly
B. Tibial metaphysis is commonest location
C. It is rarely associated with dense marginal sclerosis
D. It has no characteristic features on MRI
Source: Hussain, Shahid, et al. Radiology MCQs for the New FRCR Part 2A. 1st ed., TFM Publishing,
2006.
Explanation:
• Brodie’s abscess is a subacute or chronic form of osteomyelitis, most commonly affecting children
and adolescents.
o The tibial metaphysis is the classic location.
o It appears radiographically as a lytic lesion with well-defined sclerotic margins.
o Staphylococcus aureus is the most common causative organism.
o MRI may show a “double-line” sign due to granulation tissue and surrounding sclerosis.
• Ewing sarcoma and metastatic neuroblastoma usually lack dense marginal sclerosis.
• Osteoid osteoma is typically smaller and presents with intense night pain, while chondrosarcoma is
rare in this age group and site.
14. Regarding hyperparathyroidism (HPT): Which statement is CORRECT?
A. Brown tumors occur more frequently in secondary HPT
B. Rugger Jersey spine occurs more frequently in primary HPT
C. Increased incidence of slipped upper femoral epiphysis is associated with HPT
D. Abnormal bone scan in about 80%
Source: Hussain, Shahid, et al. Radiology MCQs for the New FRCR Part 2A. 1st ed., TFM Publishing,
2006.
Explanation:
• Chondrocalcinosis, due to calcium pyrophosphate deposition, is seen in about 15-20% of patients
with hyperparathyroidism, especially in secondary HPT.
• Brown tumours are more common in primary HPT, not secondary. The "rugger jersey spine" sign is
classically associated with secondary (renal) hyperparathyroidism due to chronic renal failure, not
primary HPT. There is an increased risk of slipped upper femoral epiphysis in HPT. Most cases of HPT
show abnormal bone scan findings.
EBDR Exam MCQs & Concepts July 2024
Dr. Kareem Alnakeeb 121
15. Regarding ultrasound of soft tissue masses: Which statement is CORRECT?
A. Ganglion cysts show sharp posterior acoustic shadowing.
B. Superficial masses are best examined with a 9-13 MHz Frequency Linear Transducer
C. Schwannomas can be differentiated from neurofibromas by ultrasound appearances
D. Lipomas commonly have increased vascularity
Source: Hussain, Shahid, et al. Radiology MCQs for the New FRCR Part 2A. 1st ed., TFM Publishing,
2006.
Explanation:
• Ganglion cysts classically appear as anechoic or hypoechoic lesions with well-defined margins and
often demonstrate posterior acoustic enhancement, which helps distinguish them from solid
masses. They frequently communicate with adjacent tendon sheaths.
• Superficial soft tissue masses are best evaluated with high-frequency (9-13 MHz) linear transducers
rather than low-frequency convex probes.
• Schwannomas and neurofibromas generally have overlapping sonographic features and cannot
reliably be differentiated by ultrasound alone.
• Lipomas typically appear as hyperechoic or isoechoic masses with little or no vascularity on Doppler
imaging.
16. Which of the followings is a feature of Paget's disease?
A. The skull is most commonly affected
B. Increased density of vertebra - 'ivory vertebra'
C. Mostly unilateral
D. Sarcomatous transformation in 10-15%
Source: Hussain, Shahid, et al. Radiology MCQs for the New FRCR Part 2A. 1st ed., TFM Publishing,
2006.
Explanation:
• The pelvis is the most frequently affected site in Paget’s disease, involved in roughly 75% of patients.
• Ivory vertebra (a dense vertebra) can be seen due to increased bone turnover.
• Thickening of the ileopectineal line is a common radiographic finding reflecting pelvic involvement.
• Candle flame lysis refers to a V-shaped lytic lesion in long bones, not sclerosis.
• Sarcomatous transformation is a rare complication, occurring in less than 1% of cases—10-15%
would be uncharacteristically high.
17. The radiograph of an 8-year-old boy with dietary Vitamin D deficiency reveals cupping and fraying of the
distal tibial metaphysis. Which radiological finding is a recognized feature of this condition?
A. Cortical sclerosis involving the margin of the epiphysis
B. Expansion of the costochondral junctions
C. Exuberant periosteal reaction
D. Metaphyseal spurs
Source: McQueen, Andrew S., et al. Grainger & Allison’s Diagnostic Radiology: Single Best Answer MCQs.
5th ed., Churchill Livingstone, 2009.
Explanation:
• Sclerosis of the margins of the epiphysis (Wimberger sign), metaphyseal (Pelcan) spurs, dense
metaphyseal lines (white lines of Frankel) and exuberant periosteal reactions (secondary to recurrent
subperiosteal bleeding) are features of scurvy.
• Expansion of the costochondral junctions results in the characteristic appearance of a rachitic
rosary.
EBDR Exam MCQs & Concepts July 2024
Dr. Kareem Alnakeeb 122
18. A 43-year-old man is investigated for pain related to his left arm. Plain radiography demonstrates a well-
defined, lytic lesion in the proximal humerus, with chondroid matrix mineralization and a wide zone of
transition. What is the MOST likely diagnosis?
A. Chondroblastoma
B. Chondroma
C. Chondrosarcoma
D. Osteochondroma
Source: McQueen, Andrew S., et al. Grainger & Allison’s Diagnostic Radiology: Single Best Answer MCQs.
5th ed., Churchill Livingstone, 2009.
19. What are the MOST likely radiological findings in a 'Monteggia fracture ?
A. A fracture of the distal radius with an associated dislocation of the radial
B. A fracture of the distal radius with an associated disruption of the distal radioulnar joint
C. A fracture of the distal ulna with an associated dislocation of the radial head
D. A fracture of the proximal radius with an associated disruption of the distal radioulnar joint
Source: McQueen, Andrew S., et al. Grainger & Allison’s Diagnostic Radiology: Single Best Answer MCQs.
5th ed., Churchill Livingstone, 2009.
Explanation (from Radiology MCQs for the new FRCR Part 2A):
• Monteggia fracture - proximal ulnar fracture with dislocation of the radial head
• Galleazi fracture - distal radial fracture with dislocation of distal radioulnar joint
• Chauffer’s fracture - triangular fracture of radial styloid process
Mnemonic:
• MUGR (pronounced “MUGGER”): identifies the fractured bone.
o MU: Monteggia Ulna
o GR: Galeazzi Radius
• Use “A to Z” to remember the location of the fracture-dislocation:
o ‘A’is proximal (MonteggiA) - Radial head dislocation and proximal ulna fracture
o ‘Z’ is distal (GaleazZi) - Distal radioulnar joint dislocation and distal radius fracture
• Segond fracture - cortical avulsion fracture of proximal lateral tibia
• Pott’s fracture - fibula fracture above an intact tibiofibular ligament
20. Which of the followings is an indicator of fracture instability of the lumbar spine?
A. Compression fracture greater than 20% of vertebral body height
B. Fracture of the posterior elements
C. Increased inter-pediculate distance.
D. Associated pre-vertebral hematoma.
Explanation: (by Perplexity AI, generated with the OpenAI O3 model)
• Both fracture of the posterior elements and increased inter-pediculate distance (as a sign of
burst fracture) are strong indicators of lumbar spine fracture instability because they both directly or
indirectly imply compromise of the middle and/or posterior columns, which are critical for spinal
stability. However, "Fracture of the posterior elements" directly refers to the disruption of one of the
key anatomical columns responsible for spinal integrity.
21. A 6-week-old boy has a positive family history of developmental dysplasia of the hip (DDH) and is referred
for a hip ultrasound. Which imaging features would be consistent with normal (Graf type 1) hips?
A. Alpha angle greater than 60°
B. Beta angle greater than 77°
C. Compressed cartilage roof
D. Deficient bony roof
Source: McQueen, Andrew S., et al. Grainger & Allison’s Diagnostic Radiology: Single Best Answer MCQs.
5th ed., Churchill Livingstone, 2009.
EBDR Exam MCQs & Concepts July 2024
Dr. Kareem Alnakeeb 123
22. A 28-year-old tennis player undergoes a MR arthrogram to investigate recurrent right shoulder instability
following a previous glenohumeral dislocation. The MRI reveals a tear of the anterosuperior labrum,
closely related to the insertion of the biceps tendon. How are these appearances BEST described?
A. Bankart lesion
B. Hill-Sachs lesion
C. Reverse Hill-Sachs lesion
D. Superior labrum from anterior to posterior (SLAP) lesion
Source: McQueen, Andrew S., et al. Grainger & Allison’s Diagnostic Radiology: Single Best Answer MCQs.
5th ed., Churchill Livingstone, 2009.
23. An asymptomatic 65-year-old woman on long-term steroids for rheumatoid disease undergoes dual
energy X-ray absorptiometry (DXA). Her Z score is —2 and her T score is —2.7. What is the WHO definition
of osteoporosis?
A. T score less than —1
B. T score less than —2.5
C. Z score less than —2.5
D. Mean of T and Z score less than —2
Source: McQueen, Andrew S., et al. Grainger & Allison’s Diagnostic Radiology: Single Best Answer MCQs.
5th ed., Churchill Livingstone, 2009.
Explanation
• Bone density can be measured in relation to an age and sex-matched population (Z score) or in
relation to a population of young adults of the same sex (T score).
• The WHO defines osteoporosis as a T score less than —2.5, therefore relating bone mineral density
to sex-matched peak bone mass.
24. A 13-year-old boy with a short history of pain and swelling around his left elbow. The radiograph reveals a
4-cm area of permeative bone destruction within the distal diaphysis of the left humerus, with a wide
zone of transition. There is an extensive associated soft tissue component and evidence of a 'hair-on-end'
pattern of periosteal reaction. What is MOST likely diagnosis?
A. Chondroblastoma
B. Chondromyxoid fibroma
C. Ewing's sarcoma
D. Malignant fibrous histiocytoma
Source: McQueen, Andrew S., et al. Grainger & Allison’s Diagnostic Radiology: Single Best Answer MCQs.
5th ed., Churchill Livingstone, 2009.
Explanation
• Infection and Langerhans cell histiocytosis (LCH) should be considered in the differential diagnosis
of a permeative bone lesion in a child.
• Askin tumour is a rare primitive neuroectodermal tumour of the chest wall in children.
EBDR Exam MCQs & Concepts July 2024
Dr. Kareem Alnakeeb 124
25. An 80-year-old man undergoes skeletal scintigraphy for multifocal skeletal pain, malaise and weight loss.
The scintigram shows diffusely increased activity throughout die skeleton, with absent renal activity.
What is the MOST likely diagnosis?
A. Metastatic bladder cancer
B. Metastatic colon cancer
C. Metastatic lung cancer
D. Metastatic prostate cancer
Source: McQueen, Andrew S., et al. Grainger & Allison’s Diagnostic Radiology: Single Best Answer MCQs.
5th ed., Churchill Livingstone, 2009.
26. What is the MOST likely appearance of a 'hangman's fracture' on plain film?
A. Fractures through the neural arch of Cl
B. Fractures through the neural arch of C2
C. Fracture of the spinous process of C7
D. Transverse fracture through the base of the dens
Source: McQueen, Andrew S., et al. Grainger & Allison’s Diagnostic Radiology: Single Best Answer MCQs.
5th ed., Churchill Livingstone, 2009.
27. On MRI of the shoulder joint, which pattern of imaging features is compatible with a partial thickness
supraspinatus tendon?
A. A gap between the distal and proximal portions of the tendon, with retraction of the proximal tendon
B. Areas of increased signal on T1 and T2 images
C. Areas of increased signal on Tl and T2 images, extending across the full thickness of the tendon
D. Low signal on all sequences
Source: McQueen, Andrew S., et al. Grainger & Allison’s Diagnostic Radiology: Single Best Answer MCQs.
5th ed., Churchill Livingstone, 2009.
28. MRI of the right knee due to clinical suspicion of an acute rupture of the ACL. The ACL is indistinct, and
cannot be visualized. Which additional features would be supportive of a diagnosis of ACL rupture?
A. Buckling of the PCL
B. Oedema within the medial collateral ligament
C. Posterior translation of the femur on the tibial condyles
D. Tear of the medial meniscus
Source: McQueen, Andrew S., et al. Grainger & Allison’s Diagnostic Radiology: Single Best Answer MCQs.
5th ed., Churchill Livingstone, 2009.
29. Radiographs of the left knee for 30-year-old man shows several small articular erosions and MRI reveals
foci of low T2/T2* signal intensity within the synovium. Which is the MOST likely diagnosis?
A. Calcium pyrophosphate arthropathy
B. Pigmented villonodular synovitis
C. Psoriatic arthropathy
D. Synovial chondromatosis
Source: McQueen, Andrew S., et al. Grainger & Allison’s Diagnostic Radiology: Single Best Answer MCQs.
5th ed., Churchill Livingstone, 2009.
EBDR Exam MCQs & Concepts July 2024
Dr. Kareem Alnakeeb 125
30. Defective osteoclastic function is a predominant feature of which disease?
A. Osteomalacia
B. Osteopetrosis
C. Osteoporosis
D. Rickets
Source: McQueen, Andrew S., et al. Grainger & Allison’s Diagnostic Radiology: Single Best Answer MCQs.
5th ed., Churchill Livingstone, 2009.
31. Which clinical or radiological feature would favor a diagnosis of chondrosarcoma rather than
enchondroma?
A. Age less than 20 years
B. Circular, curvilinear or nodular calcific densities
C. Periosteal reaction
D. Slow growth
Source: McQueen, Andrew S., et al. Grainger & Allison’s Diagnostic Radiology: Single Best Answer MCQs.
5th ed., Churchill Livingstone, 2009.
Explanation
• Outside the hands and feet, chondrosarcoma is five times more common than enchondroma.
32. The radiograph of a 40-year-old man with a painful knee shows multiple calcified loose bodies, each of
similar size, within the joint. The joint space is preserved. What diagnosis is MOST likely?
A. Gout
B. Pigmented villonodular synovitis
C. Rheumatoid arthritis
D. Synovial osteochondromatosis
Source: McQueen, Andrew S., et al. Grainger & Allison’s Diagnostic Radiology: Single Best Answer MCQs.
5th ed., Churchill Livingstone, 2009.
33. Which radiological feature would favor a diagnosis of rheumatoid rather than psoriatic arthritis?
A. Early reduction in bone mineralization
B. Joint ankylosis
C. Pencil-in-cup deformities of the middle phalanges
D. Periosteal reaction
Source: McQueen, Andrew S., et al. Grainger & Allison’s Diagnostic Radiology: Single Best Answer MCQs.
5th ed., Churchill Livingstone, 2009.
Explanation
• Juxta-articular osteopenia is one of the earliest radiographic abnormalities in rheumatoid arthritis,
distinguishing it from psoriatic arthropathy, in which bone mineral density is preserved until late in
the disease process.
34. MRI ankle joint following trauma showed that the Achilles tendon has a convex anterior margin with small
linear area of increased signal within the tendon on T2- and T2*-weighted images. What is the MOST likely
diagnosis?
A. Achilles para-tendonitis
B. Achilles tendinosis
C. Achilles tendinosis with complete tear
D. Achilles tendinosis with partial tear
Source: McQueen, Andrew S., et al. Grainger & Allison’s Diagnostic Radiology: Single Best Answer MCQs.
5th ed., Churchill Livingstone, 2009.
EBDR Exam MCQs & Concepts July 2024
Dr. Kareem Alnakeeb 126
35. The plain radiograph reveals a wrist fracture extending through the epiphysis and into the metaphysis in
9-year-old boy. How would this injury be classified in the Salter-Harris classification?
A. Type I
B. Type II
C. Туре III
D. Type IV
Source: McQueen, Andrew S., et al. Grainger & Allison’s Diagnostic Radiology: Single Best Answer MCQs.
5th ed., Churchill Livingstone, 2009.
36. A series of neonatal radiographs reveal a narrow thorax with short ribs, square iliac wings with horizontal
acetabular roofs, short sacrosciatic notches, progressive narrowing of the interpedicular distance and
posterior scalloping of the vertebral bodies. What is the MOST likely diagnosis?
A. Achondroplasia
B. Campomelic dysplasia
C. Cleidocranial dysplasia
D. Morquio's syndrome
Source: McQueen, Andrew S., et al. Grainger & Allison’s Diagnostic Radiology: Single Best Answer MCQs.
5th ed., Churchill Livingstone, 2009.
Explanation
• The iliac wings in Morquio's syndrome are characteristically flared rather than square.
37. A 19-year-old female student presents with acute abdominal pain, elevated CRP and a low-grade
temperature. On clinical examination, there is tenderness to light palpation in the right iliac fossa and the
patient is febrile. A graded compression ultrasound examination is performed. Which one of the following
statements is TRUE?
A. A transverse appendiceal diameter of 5 mm is diagnostic of acute appendicitis.
B. The finding of a pelvic fluid collection makes a diagnosis of acute appendicitis unlikely.
C. The presence of hyperechoic fat in the right iliac fossa makes diagnosis of acute appendicitis
unlikely.
D. The sensitivity of graded compression ultrasound in suspected acute appendicitis is 75—90%.
Source: McQueen, Andrew S., et al. Grainger & Allison’s Diagnostic Radiology: Single Best Answer MCQs.
5th ed., Churchill Livingstone, 2009.
Explanation
• Graded compression ultrasound of the appendix can avoid unnecessary surgery and ionising
radiation—particularly relevant for children and women of childbearing age.
• The finding of a noncompressible appendix with transverse diameter of 6 mm or greater is highly
suggestive of acute appendicitis (specificity 86-100%).
• Other ultrasound findings include hyperechoic fat in the right iliac fossa, periappendiceal fluid or a
pelvic fluid collection (appendiceal abscess).
38. Which plain radiographic finding would be MOST suggestive of a toxic megacolon?
A. Caecum measuring 4.5 cm in diameter
B. Multiple mucosal islands in a dilated transverse colon
C. Thickened haustra throughout the entire colon
D. 'Thumbprinting' of the transverse and descending colon
Source: McQueen, Andrew S., et al. Grainger & Allison’s Diagnostic Radiology: Single Best Answer MCQs.
5th ed., Churchill Livingstone, 2009.
Explanation
• The presence of severe ulceration leading to mucosal islands is a major sign of toxic megacolon (the
other key finding is colonic dilatation > 5 cm).
EBDR Exam MCQs & Concepts July 2024
Dr. Kareem Alnakeeb 127
39. Abdominal ultrasound for 37-year-old male patient with right upper quadrant pain and the patient is
afebrile, US shows a 5-cm diameter cystic lesion in the right lobe of liver. The mass contains multiple
septations with a large cyst centrally and multiple small cystic spaces peripherally. Echogenic debris is
seen within the cystic lesion and alters in position when the patient lies on his side. What is the MOST
likely diagnosis?
A. Amoebic abscess
B. Hydatid cyst
C. Pyogenic liver abscess
D. Simple liver cyst
Source: McQueen, Andrew S., et al. Grainger & Allison’s Diagnostic Radiology: Single Best Answer MCQs.
5th ed., Churchill Livingstone, 2009.
Explanation
• A multiloculated cystic mass with daughter cysts and echogenic debris (‘hydatid sand’) is
characteristic of a hydatid liver cyst.
40. A 23-year-old woman complains of episodes of diarrhea and rectal bleeding. Her father died of colorectal
cancer aged 39. A double contrast barium enema is performed and demonstrates more than one
hundred small polyps, measuring up to 5 mm in size, throughout the colon. An upper GT endoscopy
demonstrates multiple polypoid lesions in the stomach and duodenum. What is the MOST likely
diagnosis?
A. Carcinoid syndrome
B. Familial adenomatous polyposis
C. Juvenile polyposis
D. Peutz-Jegher's syndrome
Source: McQueen, Andrew S., et al. Grainger & Allison’s Diagnostic Radiology: Single Best Answer MCQs.
5th ed., Churchill Livingstone, 2009.
Explanation
• Autosomal dominant condition with multiple colonic adenomas and 100% risk of colorectal
carcinoma 20 years after diagnosis. Associated with hamartomas in the stomach, gastric and
duodenal adenomas and periampullary carcinoma.
41. Which of the following findings would make a diagnosis of fibrolamellar carcinoma more likely than that
of focal nodular hyperplasia (FNH)?
A. A hyperechoic central scar
B. Delayed enhancement of a central scar
C. Punctuate calcification in the lesion
D. The patient is taking the combined oral contraceptive pill
Source: McQueen, Andrew S., et al. Grainger & Allison’s Diagnostic Radiology: Single Best Answer MCQs.
5th ed., Churchill Livingstone, 2009.
Explanation
• There is considerable overlap in the imaging appearances of these two conditions, but punctate
calcification occurs in over half of patients with fibrolamellar carcinoma and is extremely unusual in
FNH.
42. MRCP examination for 47-year-old woman with obstructive jaundice shows a smooth stricture in the mid-
common bile duct with associated moderate intrahepatic biliary dilatation. The stricture is caused by
extrinsic compression from a round filling defect within the cystic duct. What is the diagnosis?
E. Acute bacterial cholangitis
F. Gallbladder carcinoma
G. Mirizzi syndrome
H. Primary sclerosing cholangitis (PSC)
Source: McQueen, Andrew S., et al. Grainger & Allison’s Diagnostic Radiology: Single Best Answer MCQs.
5th ed., Churchill Livingstone, 2009.
Explanation
EBDR Exam MCQs & Concepts July 2024
Dr. Kareem Alnakeeb 128
• In Mirizzi syndrome, a gallstone in the cystic duct produces mass effect on the common duct and
can lead to fistula formation.
43. Which finding would indicate a nonresectable pancreatic tumor?
A. The pancreatic duct dilated to 6 mm
B. The presence of a 5-mm coeliac axis lymph node
C. The tumor has invaded the duodenum
D. The tumor in contact with 75% of the superior mesenteric artery
Source: McQueen, Andrew S., et al. Grainger & Allison’s Diagnostic Radiology: Single Best Answer MCQs.
5th ed., Churchill Livingstone, 2009.
Explanation
• If the tumour is in contact with more than half of the vessel circumference, it is very unlikely to be
resectable.
44. Which one of the following statements is TRUE regarding pseudomembranous colitis?
A. A normal abdominal CT effectively excludes pseudomembranous colitis.
B. Ascites is present in up to 40% of patients.
C. Extensive pericolonic stranding is a typical feature on CT.
D. The rectum is not involved in 40—50% of patients.
Source: McQueen, Andrew S., et al. Grainger & Allison’s Diagnostic Radiology: Single Best Answer MCQs.
5th ed., Churchill Livingstone, 2009.
Explanation
• Ascites can occur with other colitides, but is often seen in pseudomembranous colitis.
• CT typically demonstrates mucosal enhancement and marked colonic wall thickening but only mild
pericolonic stranding, in patients with pseudomembranous colitis. These findings have a high
positive predictive value but a normal CT does not exclude pseudomembranous colitis.
• Rectal sparing occurs in around 10% of patients.
45. Which of the following features would be MOST consistent with intestinal polyp?
A. The lesion contains a locule of gas at its base.
B. The lesion has a mean density of — 150 HU.
C. The lesion is of homogeneous attenuation.
D. There are diverticulae seen in the sigmoid colon.
Source: McQueen, Andrew S., et al. Grainger & Allison’s Diagnostic Radiology: Single Best Answer MCQs.
5th ed., Churchill Livingstone, 2009.
Explanation
• A polyp will usually demonstrate uniform soft tissue density, similar to the surrounding bowel wall.
46. A 64-year-old woman with past medical history of gallstones. Abdominal ultrasound demonstrates a 6 x 4
cm mixed echogenicity lesion in the gallbladder fossa, with the gallbladder not visualized. On CT, the
gallbladder fossa mass demonstrates central low attenuation with peripheral enhancement and mild
intrahepatic biliary dilatation. Low attenuation lymph nodes are present at the porta hepatis (measuring
up to 1.5 cm short axis). Which diagnosis is MOST likely?
A. Adenomyomatosis
B. Gallbladder carcinoma
C. Hepatocellular carcinoma
D. Xanthogranulomatous cholecystitis
Source: McQueen, Andrew S., et al. Grainger & Allison’s Diagnostic Radiology: Single Best Answer MCQs.
5th ed., Churchill Livingstone, 2009.
Explanation
• A gallbladder fossa mass with little/no visible normal gallbladder and hilar biliary obstruction is
highly suggestive of gallbladder carcinoma.
EBDR Exam MCQs & Concepts July 2024
Dr. Kareem Alnakeeb 129
47. Which CT finding would be more suggestive of chronic pancreatitis than ductal pancreatic
adenocarcinoma?
A. Common bile duct dilatation
B. Focal enlargement of the pancreatic head
C. Intraductal pancreatic calcification
D. Peripancreatic fat stranding and ascites
Source: McQueen, Andrew S., et al. Grainger & Allison’s Diagnostic Radiology: Single Best Answer MCQs.
5th ed., Churchill Livingstone, 2009.
Explanation
• Intraductal calcification may be focal or diffuse and is not seen in all patients with chronic
pancreatitis. When it is present, however, it is a highly reliable sign of chronic pancreatitis.
48. Which statement regarding acute pancreatitis is CORRECT?
A. Mumps is a recognized cause
B. Pancreatic oedema is a late sign
C. Pancreatic necrosis demonstrated on CT is associated with a mortality of 5-10%
D. Right-sided pleural effusion is seen in 5%
Source: Hussain, Shahid, et al. Radiology MCQs for the New FRCR Part 2A. 1st ed., TFM Publishing,
2006.
Explanation:
• Mumps is a recognized although rare cause of acute pancreatitis, alongside more common causes
such as gallstones and alcohol.
• Pancreatic oedema is often an early sign on imaging, not a late one.
• Pancreatic necrosis seen on CT is linked to mortality rates exceeding 20%, not just 5–10%.
• Pleural effusions in pancreatitis are typically left-sided, not right-sided.
• Hemorrhagic pancreatitis is characterized on CT by hyperdense areas measuring 50–70 HU, not
hypodense regions of 5–20 HU.
49. Which of the following statements regarding pancreatic carcinoma is CORRECT?
A. 60-70% of pancreatic carcinomas arise in the tail
B. They are usually hypovascular
C. Calcification is common
D. On ultrasound appears as a hyperechoic pancreatic mass
Source: Hussain, Shahid, et al. Radiology MCQs for the New FRCR Part 2A. 1st ed., TFM Publishing,
2006.
Explanation:
• The majority of pancreatic carcinomas (60-70%) arise in the head of the pancreas, not the tail.
• Pancreatic carcinomas are typically hypovascular on contrast-enhanced imaging.
• Calcification is uncommon in pancreatic adenocarcinoma, occurring in only a small percentage of
cases.
• Contiguous organ invasion is relatively common, especially in locally advanced disease, with
invasion documented in up to 40% of cases.
• On ultrasound, pancreatic carcinomas usually appear hypoechoic rather than hyperechoic.
50. Regarding porcelain gallbladder: (True or False)
F. It is often symptomless
G. It is rarely associated with gallstones
H. Oral cholecystogram shows a non-functioning gallbladder
I. 60-70% develop carcinoma of the gallbladder
J. Acute pancreatitis is a recognised cause
Source: Hussain, Shahid, et al. Radiology MCQs for the New FRCR Part 2A. 1st ed., TFM Publishing,
2006.
Explanation:
• A. True
EBDR Exam MCQs & Concepts July 2024
Dr. Kareem Alnakeeb 130
• B. False - 90%
• C. True
• D. False - 10-20%
• E. False
51. Which of the following statements regarding splenic lymphoma is CORRECT?
A. The spleen is involved at presentation in 30-40% of patients with non Hodgkin's lymphoma
B. Focal splenic deposits are usually well defined, round lesions of increased brightness on ultrasound
C. When there is lymphomatous involvement of the spleen, splenomegaly is seen in 70-80%
D. Lymph nodes are seen in the splenic hilum in 50% of patients with Hodgkin's lymphoma
Source: Hussain, Shahid, et al. Radiology MCQs for the New FRCR Part 2A. 1st ed., TFM Publishing,
2006.
Explanation:
• Slightly higher for Hodgkin's lymphoma.
• Focal splenic lymphoma deposits are most commonly well-defined, rounded, hypoechoic lesions on
ultrasound, reflecting their cellular composition and infiltration pattern
• Splenomegaly is seen in about 50% of cases.
• Splenic lymphoma deposits commonly calcify
• Lymph nodes at the splenic hilum are uncommon in Hodgkin’s lymphoma.
52. Regarding Budd-Chiari syndrome: Which statement is FALSE?
A. It can be caused by obstruction of the supra-hepatic IVC
B. The caudate lobe is markedly atrophic
C. On MRI images 'comma-shaped' intrahepatic collateral vessels are seen
D. A 'spider's web' appearance at hepatic venography is characteristic
Source: Hussain, Shahid, et al. Radiology MCQs for the New FRCR Part 2A. 1st ed., TFM Publishing,
2006.
Explanation:
• Suprehepatic IVC obstruction is Primary cause of Budd-Chiari syndrome; secondary (hepatic vein
thrombosis) more common.
• Early CT: central liver prominent, “Flip-flop” pattern: central enhancement early, peripheral
enhancement late.
• The caudate lobe is typically enlarged—not atrophic—because it drains directly into the IVC.
• Intrahepatic collateral vessels are seen, not absent, and may have a ‘comma’ shape on MRI.
• The ‘spider’s web’ appearance at hepatic venography is characteristic of Budd-Chiari syndrome and
results from multiple small collateral channels forming due to hepatic venous outflow obstruction.
157. The following statements concerning esophageal carcinoma are true:
A. 90% of cases are squamous cell carcinomas
B. Most commonly located in the upper third of the esophagus
C. Plummer-Vinson syndrome is a recognised predisposing factor
D. Commonest appearance on double contrast barium swallow is of a large ulcer within a bulging mass
E. It is associated with ulcerative colitis
Source: Hussain, Shahid, et al. Radiology MCQs for the New FRCR Part 2A. 1st ed., TFM Publishing,
2006.
Explanation:
A. True
B. False - 20% in the upper third, 30-40% middle third and 30-40% in lower third
C. True
D. False - polypoid/fungating form is commonest
E. False - predisposing factors include Barrett’s oesophagus, alcohol abuse, smoking, coeliac disease,
achalasia, tylosis
EBDR Exam MCQs & Concepts July 2024
Dr. Kareem Alnakeeb 131
53. Regarding diverticular disease: Which is FALSE?
A. Colonic diverticulosis affects 70-80% by 80 years of age
B. 10-25% of individuals with colonic diverticular disease develop diverticulitis
C. Rectosigmoid colon is most commonly affected
D. Fistula formation occurs in 40-50% of cases complicating acute diverticulitis
Source: Hussain, Shahid, et al. Radiology MCQs for the New FRCR Part 2A. 1st ed., TFM Publishing,
2006.
Explanation:
• Colonic diverticulosis is very common in the elderly, affecting 70–80% of individuals by 80 years.
• Fistula formation is much less common, seen in approximately 14% of complicated diverticulitis
cases, not 40–50%.
• The rectosigmoid colon—not the ascending colon—is most commonly involved.
• Roughly 10–25% of those with diverticulosis develop diverticulitis.
• Moderate diverticulitis is typically defined by bowel wall thickening over 3mm, not 6mm.
54. The MOST common site of gastro-intestinal stromal tumors (GIST) is:
A. Esophagus.
B. Stomach.
C. Small intestine
D. Colon.
Source: Hussain, Shahid, et al. Radiology MCQs for the New FRCR Part 2A. 1st ed., TFM Publishing,
2006.
Explanation:
• Tumor size is regarded as the most significant criterion for predicting malignant potential in
gastrointestinal stromal tumors (GISTs).
• The most common location for GISTs is the stomach, not the sigmoid colon.
• GISTs can cause gastrointestinal bleeding but are not a common cause of hematemesis.
• Contrast enhancement of GISTs is typically heterogeneous due to hemorrhage, necrosis, or cystic
change.
• There is a recognized association between GISTs and neurofibromatosis Type 1.
55. Chron's disease of small intestine:
A. Terminal ileum is affected in 80% of case.
B. Colon is affected in 10% of case.
C. Treated surgically.
D. Presenting with deep penetrating ulcers.
Explanation: (by Perplexity AI, generated with the OpenAI O3 model)
• Terminal ileum involvement ≈ 70-80% ➜ True.
• Colonic disease only ≈ 20% (±50% ileocolonic) ➜ “10% colon” statement is False.
• Management today is medical first (steroids, immunomodulators, biologics). Surgery only for
strictures, fistulae, perforation, bleeding or refractory disease ➜ “treated surgically” is False.
• Transmural inflammation gives deep, fissuring / penetrating ulcers—cobblestone mucosa ➜ True.
56. Caudate lobe to right lobe ratio that is specific for liver cirrhosis is:
A. More than 0.73
B. Less than 0.55
C. Less than 0.60
D. Less than 0.45
Source: Unknown
Explanation:
• The right lobe and medial segment of left lobe are atrophied. The caudate lobe is not affected.
• There is compensatory hypertrophy of the lateral segments of the left lobe.
EBDR Exam MCQs & Concepts July 2024
Dr. Kareem Alnakeeb 132
• Caudate lobe/right lobe >0.65.
• Quadrate lobe < 3 cm.
• Liver enlarged in early stages and atrophied in lateral stages. The surface is nodular.
• The GB angle is normally more than 40 degrees. It is less than 35 degrees in cirrhosis.
57. Fat within hepatic focal lesion on MRI is seen in:
A. НСС
B. Focal nodular hyperplasia
C. Cholangiocarcinoma
D. Hemangioma
E. Metastasis
Explanation: (by Perplexity AI, generated with the OpenAI O3 model)
• Hepatocellular carcinoma (HCC) is the primary liver lesion most likely to show intralesional fat on
MRI, due to fatty metamorphosis within the tumor. Fat can be detected as a signal drop out on out-
of-phase imaging.
• Focal nodular hyperplasia and hemangiomas do not characteristically contain fat (although FNH
contains a central scar and hemangiomas are blood-filled).
• Cholangiocarcinoma very rarely demonstrates fat, and while certain metastases may mimic this, the
classic primary is HCC, especially in non-cirrhotic livers. Key distractors are common hepatic
masses, but fat within them is distinctly unusual.
58. linitis plastica of the stomach is seen in:
A. Gastric bezoar.
B. Gastric carcinoma.
C. Peutz-Jeghers syndrome.
D. Gastric polyps
E. Gastric lymphoma
Explanation: (by Perplexity AI, generated with the OpenAI O3 model)
• Linitis plastica refers to a diffuse infiltrative process that causes thickening and stiffening of the
stomach wall, resulting in a rigid, nondistensible stomach often referred to as “leather bottle”
stomach. This pattern is most commonly associated with diffuse-type gastric carcinoma, particularly
signet ring cell carcinoma, due to malignant infiltration of the submucosa and muscularis.
• Gastric bezoar and gastric polyps do not cause this diffuse infiltrative change.
• Peutz-Jeghers syndrome is associated with hamartomatous polyps but not linitis plastica.
• Gastric lymphoma can cause thickening but classically presents as large folds or mass-like lesions,
rather than uniform rigidity.
59. The characteristic feature of hepatic focal nodular hyperplasia on MRI is:
A. Central scar retains contrast in delayed phase and poor enhancement in arterial phase
B. Central scar shows wash out of contrast in delayed phase
C. Hyperintense central scar on Tl
D. Cystic changes
E. Hyperintense central scar on T2
Explanation: (by Perplexity AI, generated with the OpenAI O3 model)
• The classic imaging hallmark of hepatic focal nodular hyperplasia (FNH) on MRI is a central scar
that demonstrates delayed contrast retention, meaning it enhances in the delayed phase after
gadolinium administration, while showing poor enhancement in the arterial phase. The scar
typically appears hyperintense on T2-weighted images, but not on T1.
EBDR Exam MCQs & Concepts July 2024
Dr. Kareem Alnakeeb
133
• Cystic changes are not a feature of FNH; these are more characteristic of other hepatic lesions such
as hepatocellular carcinoma or cystic metastases.
• "Wash out" of contrast in the delayed phase is not typical for the FNH central scar; instead, washout
is more suggestive of hepatic adenomas or malignancy.
60. A 60-year-old presents with left groin pain. Ultrasound shows a 2 cm hypoechoic lesion bulging medial to
the epigastric vessels on Valsalva manoeuvre and absent on rest. What is the most likely diagnosis?
A. Direct inguinal hernia
B. Indirect inguinal hernia
C. Obturator hernia
D. Spigelian hernia
E. Femoral hernia
Source: Gupta, Chaitanya. 300 Single Best Answers for the Final FRCR Part A. 1st ed., Jaypee UK,
2010.
Explanation:
• A direct inguinal hernia is seen medial to the inferior epigastric vessels whereas an indirect hernia is
seen lateral to them.
61. Central dot sign is seen in:
A. Caroli disease
B. Primary sclerosing cholangitis
C. Polycystic liver disease
D. Liver hamartoma
Explanation: (by Perplexity AI, generated with the OpenAI O3 model)
• The central dot sign is an enhancing portal venous radicle and fibrovascular bundle seen within
fluid-filled, ectatic intrahepatic bile ducts; it is virtually pathognomonic for Caroli’s disease because
the ducts are massively dilated yet remain in continuity with portal triads.
• Primary sclerosing cholangitis produces multifocal strictures and beading without a central
enhancing dot.
• Simple hepatic cysts are avascular, with thin walls and no intraluminal structures.
• Hepatic mesenchymal hamartoma appears as a multicystic mass lacking patent biliary
connections.
• Cavernous hemangiomas show peripheral nodular enhancement progressing centrally, not a single
central dot.
62. On ultrasonographic examination, diffuse thickening of gall bladder with hyperechoic shadow at neck
and comet tailing is seen in:
A. Xanthogranulomatous cholecystitis
B. Adenomyomatosis
C. Adenomyomatous polyps
D. Cholesterol crystals
Explanation: (by Perplexity AI, generated with the OpenAI O3 model)
• Adenomyomatosis is a benign hyperplastic condition of the gallbladder characterized by mural
thickening and formation of Rokitansky–Aschoff sinuses. Ultrasonography typically reveals diffuse or
segmental wall thickening, with comet tail or ring-down artefact resulting from cholesterol crystals
trapped within the sinuses, most often seen at the gallbladder neck.
• Xanthogranulomatous cholecystitis produces heterogeneous wall thickening but lacks comet tail
artefacts.
EBDR Exam MCQs & Concepts July 2024
Dr. Kareem Alnakeeb
134
• Adenomyomatous polyps refer to focal instead of diffuse changes and do not exhibit classic comet
tail artefact.
• Cholesterol crystals can contribute to comet tailing, but they most commonly present as echogenic
foci without diffuse wall thickening.
63. Retroperitoneal lymphoma:
A. It is the most common retroperitoneal malignancy, accounting for up to one-third of all cases.
B. On CT, it is usually seen as well-defined para-aortic or pelvic non-enhancing masses with areas of
necrosis
C. Vascular invasion is one of its characteristic features
D. It is almost always associated with mediastinal lymphadenopathy
E. It typically calcifies before treatment.
Explanation: (by Perplexity AI, generated with the OpenAI O3 model)
• Retroperitoneal lymphoma is indeed the most common primary retroperitoneal malignancy,
responsible for up to one-third of such cases.
• It usually presents as well-defined, often homogenous soft-tissue masses predominantly in para-
aortic or pelvic locations. However, these masses usually ENCASE vessels rather than invade them
directly, which distinguishes lymphoma from other retroperitoneal malignancies—direct vascular
invasion is uncharacteristic.
• Although mediastinal lymphadenopathy may occur, it is not an almost constant feature.
• Areas of necrosis are uncommon unless the patient has been treated, and calcification typically
occurs only after therapy rather than beforehand.
64. Large-bowel obstruction is diagnosed at CT if:
A. There is colonic wall thickening and luminal dilation more than 4 cm
B. The large bowel is dilated with no dilation of the small bowel
C. The colon is dilated (colon diameter > 5.5 cm, cecum diameter > 10 cm) and filled with feces,
gas, and fluid proximal to an abrupt transition point, after which the colon is collapsed distally
D. Dilated large bowel with associated ileus
Explanation: (by Perplexity AI, generated with the OpenAI O3 model)
• Large-bowel obstruction on CT is characterized by pronounced colonic dilatation (colon >5.5cm,
cecum >10cm), with feces, gas, and fluid accumulating proximal to a clear transition point. Distal to
this point, the bowel appears collapsed. This pattern is crucial for distinguishing true obstruction
from pseudo-obstruction or diffuse ileus, which may not show a transition.
• Colonic wall thickening and a 4cm diameter alone are not specific for obstruction.
• Lack of small bowel dilation does not exclude partial or early obstruction.
• Presence of only gas without a transition point is nonspecific.
65. Early complication of post bariatric surgical technique is:
A. Anastomotic narrowing.
B. Abscess formation.
C. Anastomotic leak.
D. Hemorrhage
Explanation: (by Perplexity AI, generated with the OpenAI O3 model)
• Immediate (days-to-first-weeks) post-operative hazards include anastomotic/staple-line leaks and
early hemorrhage, whereas abscesses and strictures tend to occur later.
EBDR Exam MCQs & Concepts July 2024
Dr. Kareem Alnakeeb
135
66. On scrotal ultrasound for testicular torsion, which of the following radiological findings would suggest
that the testis is still viable?
E. A diffusely enlarged hypoechoic left testis
F. A normal echogenicity testis on grey-scale imaging
G. A small shrunken left testis with a surrounding hydrocoele and scrotal wall thickening
H. Absent blood flow within the left testis on color flow Doppler but good flow within the tunica vaginalis
Source: McQueen, Andrew S., et al. Grainger & Allison’s Diagnostic Radiology: Single Best Answer MCQs.
5th ed., Churchill Livingstone, 2009.
Explanation:
• The section on testicular torsion within the comprehensive review by Fiitterer et al provides useful
additional information.
67. Which one of the following statements BEST describes the expected ultrasound findings in acute,
uncomplicated epididymo-orchitis?
A. A small atrophic right testis
B. Multiple small echogenic foci scattered throughout the testis
C. Patchy areas of increased echogenicity within the testis with reduced flow on color Doppler
D. Well-defined, patchy areas of decreased echogenicity within the testis
Source: McQueen, Andrew S., et al. Grainger & Allison’s Diagnostic Radiology: Single Best Answer MCQs.
5th ed., Churchill Livingstone, 2009.
Explanation:
• In the early phase of acute orchitis, there is oedema of the testis leading to swelling and diffuse low
reflectivity on ultrasound.
• The ultrasound appearances then evolve to increasingly well-defined areas of patchy low reflectivity.
Colour Doppler flow is typically increased within these areas of low reflectivity.
68. Which of the following ultrasound finding would suggest a diagnosis of testicular teratoma rather than
seminoma?
A. A testicular mass that contains areas of calcification
B. A testicular mass that demonstrates increased color Doppler flow
C. A testicular mass that is homogeneously anechoic with posterior acoustic enhancement
D. A testicular mass that has well-defined margins
Source: McQueen, Andrew S., et al. Grainger & Allison’s Diagnostic Radiology: Single Best Answer MCQs.
5th ed., Churchill Livingstone, 2009.
69. An IVU showed complete right-sided ureteric duplication. Which one of the following statements is
TRUE?
A. If present, an ectopic ureterocele is usually related to the lower moiety
B. The upper moiety calyces are prone to vesicoureteric reflux.
C. The upper moiety ureter is prone to ureteric obstruction.
D. The upper moiety ureter usually inserts into the bladder superior to the lower moiety ureter.
Source: McQueen, Andrew S., et al. Grainger & Allison’s Diagnostic Radiology: Single Best Answer
MCQs. 5th ed., Churchill Livingstone, 2009.
EBDR Exam MCQs & Concepts July 2024
Dr. Kareem Alnakeeb
136
70. On the unenhanced CT, right adrenal mass is detected and appears homogeneous and has an average
density of 7 HU. What is the MOST likely diagnosis?
E. Adrenal adenoma
F. Adrenal hyperplasia
G. Adrenal metastasis
H. Focal adrenal hemorrhage
Source: McQueen, Andrew S., et al. Grainger & Allison’s Diagnostic Radiology: Single Best Answer MCQs.
5th ed., Churchill Livingstone, 2009.
Explanation:
• The 10-HU threshold is now the standard by which radiologists differentiate lipid-rich adenomas
from most other adrenal lesions on unenhanced CT.
• The presence of substantial amounts of intracellular fat is critical in malting the specific diagnosis of
adenoma. Up to 30% of adenomas, however, do not have abundant intracellular fat and, thus, show
attenuation values greater than 10 HU on unenhanced CT.
• Lesions above 10 HU on an unenhanced CT are considered indeterminate and other investigations
may be required.
71. A 35-year-old patient received a cadaveric renal transplant 5 days ago and now presents with worsening
renal function and decreasing urine output. Which one of the following findings on a Tc-99m DTPA
radionuclide scan would favor a diagnosis of acute tubular necrosis (ATN) over acute rejection?
A. Delayed renal excretion
B. Elevated resistive index greater than 0.7
C. Increased renal perfusion after administration of an ACEI (eg Captopril)
D. Preserved renal transplant perfusion
Source: McQueen, Andrew S., et al. Grainger & Allison’s Diagnostic Radiology: Single Best Answer MCQs.
5th ed., Churchill Livingstone, 2009.
Explanation:
• ATN is an early complication in cadaveric allografts and frequently resolves spontaneously in 1—3
weeks.
• The radionuclide imaging findings of ATN are of preserved perfusion but poor renal function and urine
excretion.
• In acute rejection however, there is both impaired renal function and reduced perfusion on
radionuclide imaging.
72. Which one of the following radiological findings is a recognized feature of Von Hippel Lindau (VHL)
disease?
A. Bilateral adrenal masses that yield a high signal on T2w sequences
B. Cerebral aneurysms on CT angiography
C. Evidence of calcified subependymal nodules on CT head
D. Polymicrogyria and corpus callosum agenesis on MRI brain
Source: McQueen, Andrew S., et al. Grainger & Allison’s Diagnostic Radiology: Single Best Answer MCQs.
5th ed., Churchill Livingstone, 2009.
Explanation:
• Phaeochromocytomas are hyperintense on T2w sequences and iso- or hypointense to the liver on
Tlw sequences.
73. Regarding urethral injuries, which one of the following statements is CORRECT?
A. Anterior urethral injury is more commonly due to iatrogenic or penetrating trauma than to blunt
trauma
B. Cystography should precede a retrograde urethrogram in a patient with suspected urethral injury.
C. In men, on digital rectal examination the prostate is lower than normal in patients with urethral
trauma.
D. Urethral injuries occur in 50% of major pelvic fractures.
EBDR Exam MCQs & Concepts July 2024
Dr. Kareem Alnakeeb
137
Source: McQueen, Andrew S., et al. Grainger & Allison’s Diagnostic Radiology: Single Best Answer MCQs.
5th ed., Churchill Livingstone, 2009.
74. Which one of the following findings would indicate a Grade 4 renal laceration?
A. Extravasation of contrast from the pelvicalyceal system on delayed phase (5min) images
B. Large (2-cm) subcapsular hematoma
C. Perinephric hematoma that extends into the pararenal spaces
D. Ill-defined low attenuation change in the lower pole renal cortex
Source: McQueen, Andrew S., et al. Grainger & Allison’s Diagnostic Radiology: Single Best Answer MCQs.
5th ed., Churchill Livingstone, 2009.
Explanation:
• A deep renal laceration that extends into the collecting system is indicative of a grade 4 injury.
75. Which one of the following MRI findings would favor a diagnosis of renal angiomyolipoma?
A. High signal on T1w and low signal on T2w sequences
B. High signal on T1w and STIR sequences
C. High signal on T1w and T2w sequences
D. High signal on T2w and low signal on proton density sequences
Explanation: (by Perplexity AI, generated with the OpenAI O3 model)
• Renal angiomyolipomas characteristically contain macroscopic fat. On MRI, fat demonstrates high
signal intensity on T1-weighted images and loses signal on fat-suppressed or STIR sequences.
However, the classic radiological finding is high signal on T1-weighted images and variable (often
low) signal on T2-weighted images due to the heterogeneous tissue components.
• High signal on both T1w and STIR (option B) would not be typical, as fat is suppressed on STIR.
• High signal on both T1w and T2w (option C) better fits fat-containing metastases or certain
hemorrhagic lesions, not classic angiomyolipoma.
• Option D and E do not match the characteristic fat features of angiomyolipoma.
76. Abdominal ultrasound for 22-year-old pregnant woman (30 weeks' gestation) with right loin pain shows
dilatation of the right pelvicalyceal system. Which one of the following additional findings would suggest
a diagnosis of mechanical ureteric obstruction rather than pregnancy-related dilatation?
A. An elevated resistive index (RI)
B. Decreased corticomedullary differentiation
C. Renal parenchymal thinning
D. Ureteric and pelvicalyceal dilatation
Source: McQueen, Andrew S., et al. Grainger & Allison’s Diagnostic Radiology: Single Best Answer MCQs.
5th ed., Churchill Livingstone, 2009.
Explanation:
• Mechanical obstruction is associated with elevation of the RI.
77. The control film of IVU for an immunosuppressed 24-year-old man shows a gas containing, round
lamellated mass within the urinary bladder. Postcontrast, there are multiple filling defects within the
urinary bladder. What is the MOST likely cause of these appearances?
A. Blood clot
B. Bladder calculi
C. Fungal ball
D. Schistosomiasis
Source: McQueen, Andrew S., et al. Grainger & Allison’s Diagnostic Radiology: Single Best Answer MCQs.
5th ed., Churchill Livingstone, 2009.
EBDR Exam MCQs & Concepts July 2024
Dr. Kareem Alnakeeb
138
78. In which part of the prostate gland is a carcinoma MOST likely
A. Central zone
B. Peripheral zone
C. Peri-urethral zone
D. Transitional zone
Source: McQueen, Andrew S., et al. Grainger & Allison’s Diagnostic Radiology: Single Best Answer MCQs.
5th ed., Churchill Livingstone, 2009.
Explanation:
• Roughly 70–75% of prostate adenocarcinomas arise in the peripheral zone, the posterolateral portion
that is accessible on digital-rectal examination.
• About 20% originate in the transitional (central gland) zone and only a small minority in the central or
peri-urethral zones
79. Which one of the following statements BEST describes the MRI findings of a normal prostate gland?
A. On T1 w images, the central zone is of higher signal intensity than the peripheral zone.
B. On T1 w images, the central zone is of lower signal intensity than the peripheral zone.
C. On T2w images, the peripheral zone is of lower signal intensity than the central and transitional zones
D. The peripheral zone is of higher signal intensity than the central zone on T2w images.
Source: McQueen, Andrew S., et al. Grainger & Allison’s Diagnostic Radiology: Single Best Answer MCQs.
5th ed., Churchill Livingstone, 2009.
Explanation:
• Zonal anatomy of the prostate is best demonstrated on T2w sequences.
80. The following stones are NOT visible on CT scan:
A. Urate stones.
B. Matrix stones.
C. Xanthine stones.
D. Cysteine stones
Explanation: (by Perplexity AI, generated with the OpenAI O3 model)
• Non-contrast CT detects nearly all urinary calculi (>99%), including urate, xanthine and cystine
stones, because they remain denser than surrounding soft tissue.
• Matrix stones, composed mainly of proteinaceous material with minimal mineral content, may be
completely radiolucent on CT and therefore escape detection, making them the principal exception.
81. Which one of the following statements BEST describes the characteristic radiological features of
retroperitoneal fibrosis?
A. A plaque-like mass that encases the aorta and displaces it laterally, most commonly to the left
B. A plaque-like mass that displaces the kidneys and ureters laterally at the L1- 2 level
C. A plaque-like mass that displaces the aorta and iliac arteries anteriorly
D. A plaque-like mass that narrows and medially displaces the ureters at the L4-5 level
Source: McQueen, Andrew S., et al. Grainger & Allison’s Diagnostic Radiology: Single Best Answer MCQs.
5th ed., Churchill Livingstone, 2009.
EBDR Exam MCQs & Concepts July 2024
Dr. Kareem Alnakeeb
139
82. Regarding benign and malignant adrenal masses. Which statement is CORRECT?
A. Lesions >4 cm tend to be malignant
B. Approximately one third of benign adenomas have HU of >10 on unenhanced CT
C. Adenomas tend to show delayed enhancement with IV contrast
D. Adenomas tend to show delayed clearance of IV contrast
Source: Hussain, Shahid, et al. Radiology MCQs for the New FRCR Part 2A. 1st ed., TFM Publishing,
2006.
Explanation:
• Lesions larger than 4cm are more likely to be malignant.
• About one third of benign adrenal adenomas have an unenhanced CT attenuation above 10 HU, so a
higher HU does not exclude a benign adenoma.
• Adenomas enhance rapidly, not with delay.
• Adenomas characteristically show rapid washout (clearance) of IV contrast.
• Chemical shift MRI uses T1-weighted sequences, not T2-weighted, to assess intracytoplasmic lipid.
83. Multicystic dysplastic kidney. Which statement is FLASE?
A. Is the second commonest cause of a neonatal abdominal mass
B. Is usually unilateral
C. Is associated with pelvi-ureteric junction (PUJ) obstruction
D. The renal cysts communicate
Source: Hussain, Shahid, et al. Radiology MCQs for the New FRCR Part 2A. 1st ed., TFM Publishing,
2006.
Explanation:
• Multicystic dysplastic kidney is the second most common cause of a neonatal abdominal mass
(after hydronephrosis) and is usually unilateral.
• It is associated with pelviureteric junction (PUJ) obstruction in approximately 10–20% of cases.
• The cysts in multicystic dysplastic kidneys do not communicate with each other, and there is no
intervening normal renal parenchyma—rather, the normal architecture is replaced by multiple non-
communicating cysts with fibrous stroma.
• Bilaterality is rare and usually incompatible with life.
84. Which of the following features on US suggest a malignant rather than benign breast mass?
A. It is taller than it is wide
B. It is markedly hyperechoic
C. It has a thin echogenic capsule
D. It does not cast an acoustic shadow
Explanation: (by Perplexity AI, generated with the OpenAI O3 model)
• A non-parallel or “taller-than-wide” orientation indicates that the lesion is invading tissue planes
perpendicular to the skin, a behavior typical of invasive carcinomas
85. A transvaginal ultrasound is performed on a 36-year-old woman with dysfunctional uterine bleeding. This
demonstrates an enlarged globular uterus with a heterogeneous appearance of the myometrium. The
myometrium contains diffuse echogenic nodules, subendometrial echogenic linear striations and 2- to
6-mm subendometrial cysts. What is the MOST likely diagnosis?
A. Adenomyosis
B. Endometrial polyposis
C. Stage 1A endometrial cancer
D. Uterine fibroid
Source: McQueen, Andrew S., et al. Grainger & Allison’s Diagnostic Radiology: Single Best Answer MCQs.
5th ed., Churchill Livingstone, 2009.
EBDR Exam MCQs & Concepts July 2024
Dr. Kareem Alnakeeb
140
86. Screening mammogram showed a 2-cm well-defined, oval mass that has dense 'popcorn' calcification
within it and is surrounded by a thin radiolucent rim. On ultrasound, the mass is well defined and
hyperechoic with areas of acoustic shadowing due to contained calcification. What is the MOST likely
diagnosis?
A. Fat necrosis
B. Fibroadenoma
C. Hamartoma
D. Papilloma
Source: McQueen, Andrew S., et al. Grainger & Allison’s Diagnostic Radiology: Single Best Answer MCQs.
5th ed., Churchill Livingstone, 2009.
Explanation:
• Fibroadenomas may become calcified, particularly after menopause. Classically the calcifications
have a coarse ‘popcorn’ appearance; however, they may also appear small and punctate.
• An oil cyst typically demonstrates eggshell calcification and is the result of fat necrosis.
87. On transvaginal ultrasound for a 60-year-old nulliparous woman presents with postmenopausal bleeding
presents with postmenopausal bleeding, endometrium is 8 mm thick and the endo-myometrial junction
appeared indistinct. MRI examination is indicated for the possibility of endometrial carcinoma. What are
the likely findings on MRI?
A. On unenhanced T1w images the endometrial cancer appears of high signal intensity compared to the
surrounding myometrium.
B. On contrast-enhanced T1w images, endometrial cancer shows avid enhancement compared with
surrounding myometrium.
C. On T2w images the normally high signal junctional zone is disrupted.
D. The endometrial cancer demonstrates delayed/little enhancement compared to the normal
surrounding myometrium on postcontrast Tl w images.
Source: McQueen, Andrew S., et al. Grainger & Allison’s Diagnostic Radiology: Single Best Answer MCQs.
5th ed., Churchill Livingstone, 2009.
88. Which one of the following sonographic finding would indicate that the adnexal mass is more likely to be
malignant than benign?
A. Homogeneously hypoechoic mass with posterior acoustic enhancement
B. Multiple septations that are approximately 1 mm thick
C. Papillary projections
D. Size > 4 cm
Source: McQueen, Andrew S., et al. Grainger & Allison’s Diagnostic Radiology: Single Best Answer MCQs.
5th ed., Churchill Livingstone, 2009.
89. The HSG shows a single vagina, single cervix but two separate uterine cavities leading to separate uterine
horns. What is the MOST likely diagnosis?
A. Arcuate uterus
B. Bicornuate uterus
C. Didelphys uterus
D. Septate uterus
Source: McQueen, Andrew S., et al. Grainger & Allison’s Diagnostic Radiology: Single Best Answer MCQs.
5th ed., Churchill Livingstone, 2009.
EBDR Exam MCQs & Concepts July 2024
Dr. Kareem Alnakeeb
141
90. Regarding MRI of the breast, which one of the following statements is CORRECT?
A. Breast MRI should be performed during the middle of the menstrual cycle to improve sensitivity.
B. Malignant lesions tend to show poor enhancement following intravenous contrast, compared with.
surrounding breast tissue.
C. MRI has a high sensitivity and specificity for the detection of invasive breast cancer.
D. Post radiotherapy, abnormal enhancement patterns return to normal within 3- 6 months.
Source: McQueen, Andrew S., et al. Grainger & Allison’s Diagnostic Radiology: Single Best Answer MCQs.
5th ed., Churchill Livingstone, 2009.
Explanation:
• Malignant breast lesions enhance postcontrast; however, normal hormonally active breast tissue
can also chance, particularly during the middle of the menstrual cycle (6th—17th days).
• In younger patients it may be helpful to repeat the scan carlier or later in the menstrual cycle to
improve specificity.
91. Regarding ovarian cancer: Which statement is CORRECT?
A. It is the commonest gynecological malignancy
B. CT only has a pre-operative staging accuracy of 50% ,
C. CA-125 is specific for ovarian cancer
D. Doppler ultrasound may help with differentiating benign from malignant disease
Source: Hussain, Shahid, et al. Radiology MCQs for the New FRCR Part 2A. 1st ed., TFM Publishing,
2006.
Explanation:
• Ovarian cancer is associated with colorectal cancer, particularly in the context of Lynch syndrome
(hereditary nonpolyposis colorectal cancer), which increases risk for several cancers including both
ovarian and colorectal malignancies.
• Ovarian cancer is the second most common gynecological malignancy after endometrial carcinoma.
• The accuracy of CT for pre-operative staging of ovarian cancer is higher than 50%, typically ranging
from 70–90%.
• CA-125, while useful as a tumor marker, is not specific for ovarian cancer as it may be elevated in
various benign conditions such as fibroids, endometriosis and pelvic inflammatory disease.
• Doppler ultrasound can aid in distinguishing benign from malignant ovarian masses.
92. Regarding endometriosis: Which statement is CORRECT?
A. 20% of infertile women are affected
B. Endometrioma is rarely anechoic on ultrasound
C. May present with pneumothorax
D. Cystic masses seen are typically hypointense on Tl weighted images
Source: Hussain, Shahid, et al. Radiology MCQs for the New FRCR Part 2A. 1st ed., TFM Publishing,
2006.
Explanation:
• About 20% of infertile women are affected by endometriosis, making it a common association. The
condition most often involves the ovaries (around 80%) and not the fallopian tubes.
• Endometriomas, also known as "chocolate cysts", classically appear as cystic masses with low-level
internal echoes on ultrasound, not anechoic.
• Although rare, thoracic endometriosis may present with recurrent pneumothorax.
• Cystic endometriotic lesions are typically hyperintense on T1-weighted MRI due to blood products.
EBDR Exam MCQs & Concepts July 2024
Dr. Kareem Alnakeeb
142
93. With regards to assessment of tubal patency, which of the following statements is CORRECT?
A. Gadolinium enhanced MRI is the investigation of choice
B. Normal fallopian tubes are visible on pelvic US
C. Hysterosalpingography should be performed in the first half of the menstrual cycle
D. lodine based contrast is used for hysterosalpingo contrast sonography
Source: Unknown
Explanation:
• Laparoscopy and blue dye instillation (with spillage of dye into the peritoneal cavity indicating
patency) is the gold standard, but requires a general anaesthetic.
• MRI and standard pelvic US do not clearly demonstrate the fallopian tubes.
• HSG provides an accurate indication of tubal patency but employs ionizing radiation. It should
therefore be performed in the first half of the menstrual cycle to avoid irradiating a patient with
possible early pregnancy.
• Hysterosalpingo-Contrast-Sonography uses microbubbles to demonstrate the fallopian tubes and
has the advantage of not using ionizing radiation or requiring anaesthesia but is less accurate than
the other methods.
94. Ultrasound in early pregnancy: Which statement is CORRECT?
A. Gestational sac is first seen in 8 weeks.
B. Accurate biparietal diameter can be done at 13 weeks.
C. Yolk sac is seen in 5 weeks.
D. Placenta can be identified at 12 weeks
Source: Unknown
Explanation:
• Accurate biparietal diameter measurement can be performed from 13 weeks.
• The gestation sac is typically first visualized at around 5 weeks and by 10 weeks, it should occupy
more than half the uterine cavity.
• The placenta can usually be identified from 10 weeks onwards.
• The yolk sac is usually visible from about 6 weeks.
95. What level of serum glucose is generally considered acceptable when performing an FDG-PET scan?
A. Less than 50 mg/dL.
B. Less than 100 mg/dL.
C. Less than 200 mg/dL.
D. Less than 300 mg/dL.
Explanation: (by Perplexity AI, generated with the OpenAI O3 model)
• Most professional guidelines (EANM, SNMMI, ACR) advise proceeding with an oncologic FDG-PET/CT
if the fasting plasma glucose is below about 11 mmol/L (≈ 200 mg/dL); at higher levels competitive
inhibition by circulating glucose can lower tumor uptake and degrade image quality.
• Values under 100 mg/dL are ideal but not mandatory, whereas levels > 200 mg/dL usually prompt
rescheduling or glucose-lowering measures.
96. A normal brain perfusion SPECT pattern:
A. Appears as a completely symmetrical tracer uptake distribution in all subjects
B. Is completely identical in all subjects
C. Is influenced by radiopharmaceutical injected
D. Cerebellum is not visualized
Explanation: (by Perplexity AI, generated with the OpenAI O3 model)
• Minor physiological right-to-left differences (often slightly higher left-hemisphere counts) are
frequent in healthy people; perfect symmetry is not obligatory.
• Normal inter-individual variability exists (age, sex, handedness and technical factors all modify
uptake distribution)
EBDR Exam MCQs & Concepts July 2024
Dr. Kareem Alnakeeb
143
• Tracers such as 99
mTc-HMPAO and 99
mTc-ECD have different extraction fractions and wash-out
kinetics, leading to subtle but recognizable differences in regional uptake (for example higher
cerebellar activity with HMPAO)
• Cerebellar cortex normally shows high perfusion and is well seen; it is often chosen as a reference
(“normalizer”) region
97. A low flat renogram curve indicates:
A. Advanced nephropathy
B. Complete obstruction to urine outflow
C. Vesico-ureteric reflux
D. Partial obstruction to urine outflow
Explanation: (by Perplexity AI, generated with the OpenAI O3 model)
• A very low, almost horizontal time–activity curve reflects markedly reduced tracer extraction by
damaged renal parenchyma, typical of end-stage or advanced medical renal disease.
• Complete or partial outflow obstruction instead produces an uptake phase followed by a rising or
plateau phase; vesico-ureteric reflux alters the post-void segment rather than the primary
renographic curve
98. Regarding gastric emptying scintigraphy
A. T1/2 is the time at which gastric counts falls to its half
B. Tc 99m tin colloid labeled RBCs are injected IV
C. If 20 % of gastric counts remain after 1 hour, delayed gastric emptying is considered.
D. It cannot detect changes in gastric emptying rate in post operative cases.
Explanation: (by Perplexity AI, generated with the OpenAI O3 model)
• T1/2, or half-emptying time, refers to the time at which the measured gastric radioactivity drops to
50% of its initial value, a standard parameter in gastric emptying studies.
• Tc-99m sulfur colloid is typically used and is ingested with a solid meal, not injected intravenously as
labeled RBCs.
• Delayed gastric emptying is usually defined as >60% retention at 2 hours or >10% at 4 hours, not at
the 1-hour mark.
• Scintigraphy can reliably assess postoperative gastric emptying changes.
99. Regarding the scintigraphic assessment of a joint prosthesis, which of the following is CORRECT?
A. Increased MDP activity over the greater trochanter is diagnostic of an inflammatory bursitis
B. A cemented total hip replacement (THR) remains 'hot' for only 6-12 months after surgery
C. A hot knee prosthesis 18 months after surgery is abnormal
D. Prosthetic loosening results in a scan that is 'hot' on blood pool and delayed phases
Explanation: (Different AI models give varying answers to this question)
• Normal postoperative uptake
o Cemented hip: fades to baseline within 6–12 months.
o Uncemented hip: may stay active for 3–5 years.
o Knee (cemented or hybrid): activity can persist for ∼3 years.
• Interpretation of multiphase scans
o Hot flow + blood-pool + delayed → think infection or active inflammation.
o Normal flow/blood-pool but hot delayed → think aseptic loosening or stress remodeling
EBDR Exam MCQs & Concepts May 2023
Dr. Kareem Alnakeeb
144
May 2023
1. A 60-year-old female presents with a history of facial pain and diplopia. Clinical examination reveals
palsies of the III, IV, and VI cranial nerves, Horner’s syndrome, and facial sensory loss in the distribution
of the ophthalmic and maxillary divisions of the trigeminal (V) cranial nerve. Where is the causative
abnormality located?
A. Dorello’s canal.
B. Cavernous sinus.
C. Superior orbital fissure.
D. Inferior orbital fissure.
E. Meckel’s cave.
Source: Lindsay, Richard, et al. SBAs for the FRCR Part 2A. 1st ed., Oxford University Press, 2012.
Explanation:
Cavernous Sinus Anatomy
• Cranial nerves III, IV, and VI, and ophthalmic (V1) and maxillary (V2) divisions of the V cranial nerve
course through the cavernous sinus along with the internal carotid artery.
• The V2 division of the trigeminal nerve passes through the inferior portion of the cavernous sinus and
exits via the foramen rotundum.
• The remainder of the cranial nerves mentioned above enter the orbit via the superior orbital fissure.
Clinical Implications
• Palsies of cranial nerves III, IV, and VI result in ophthalmoplegia.
• Involvement of V1 and V2 divisions of the trigeminal nerve produces facial pain and sensory loss;
involvement of sympathetic nerves around the internal carotid artery results in Horner’s syndrome.
• This cluster of findings is found in Tolosa Hunt syndrome, an idiopathic inflammatory process
involving the cavernous sinus.
2. A 40-year-old presents with a painless, slowly growing lateral neck mass located between the angle of
the mandible and the carotid bifurcation. On ultrasound, the lesion is well-circumscribed, cystic, and
displaces nearby structures but does not show internal vascularity. What is the most likely diagnosis?
A. Second branchial cleft cyst
B. Carotid body tumour
C. Cystic metastatic lymph node
D. Cervical thymic cyst
E. Vagal schwannoma
Explanation: (by Perplexity AI, generated with the OpenAI O3 model)
• The most likely diagnosis is a second branchial cleft cyst, which typically presents as a well-
circumscribed, cystic mass situated lateral to the neck, often at the angle of the mandible and just
anterior to the sternocleidomastoid, displacing but not infiltrating vascular structures. Carotid body
tumours are usually solid, hypervascular lesions at the carotid bifurcation, causing splaying of the
carotid vessels.
• Cystic metastatic lymph nodes are considered especially in older adults, but they often exhibit
irregular walls or solid components.
• Although cervical thymic cysts and vagal schwannomas can occur laterally, they are rare and have
distinct imaging or clinical features. The ultrasound finding of a purely cystic, avascular mass
supports a branchial cleft cyst diagnosis.
EBDR Exam MCQs & Concepts May 2023
Dr. Kareem Alnakeeb
145
3. A 60-year-old woman presents with a painless, slowly growing mass in the lateral aspect of the neck. The
patient is referred for imaging with a clinical diagnosis of carotid body paraganglioma. Which of the
following is a distinctive feature of carotid body paraganglioma on imaging?
A. Soft-tissue mass in the carotid space.
B. Intense enhancement after IV contrast administration.
C. High signal on T2WI.
D. Splaying of the internal and external carotid arteries.
E. Low signal on T2WI.
Source: Lindsay, Richard, et al. SBAs for the FRCR Part 2A. 1st ed., Oxford University Press, 2012.
Explanation:
• Carotid body tumour or paraganglioma is the most common paraganglioma of the head and neck. It
arises from the paraganglionic cells located on the medial aspect of the carotid bifurcation.
• On MRI, they are of low to intermediate signal intensity on T1WI and hyperintense on T2WI. They are
hypervascular and demonstrate intense enhancement after contrast administration.
• Splaying of the internal and external carotid arteries and multiple flow voids producing a ‘salt and
pepper’ appearance are distinctive features on imaging.
4. A newborn presents with cyanosis and a chest radiograph showing increased pulmonary vascularity (lung
plethora). Which is the most likely underlying cardiac condition?
A. Patent ductus arteriosus
B. Atrial septal defect
C. Pulmonary atresia
D. Tricuspid regurgitation
E. Tetralogy of Fallot
Explanation: (by Perplexity AI, generated with the OpenAI O3 model)
• Patent ductus arteriosus (PDA) classically leads to increased pulmonary blood flow, resulting in lung
plethora visible on imaging. Cyanosis may occur if right-to-left shunting develops (e.g., with
pulmonary hypertension).
• Atrial septal defect (ASD) usually causes increased flow but less often cyanosis early on.
• Pulmonary atresia typically causes reduced pulmonary flow and oligemic lung fields rather than
plethora.
• Tricuspid regurgitation can result from right heart issues but does not usually lead to marked lung
plethora with cyanosis unless there is a significant shunt or other associated defects.
5. A 45-year-old asthmatic patient presents with worsening cough and expectoration of brownish mucus
plugs. Chest radiograph shows branching tubular opacities in the right upper lobe resembling “finger-in-
glove” appearance. Which is the most likely underlying diagnosis?
A. Bronchiectasis secondary to cystic fibrosis
B. Allergic bronchopulmonary aspergillosis
C. Acute pulmonary embolism
D. Sarcoidosis
E. Primary lung carcinoma
Explanation: (by Perplexity AI, generated with the OpenAI O3 model)
• The “finger-in-glove” sign on chest imaging refers to branching tubular opacities representing
impacted mucus plugs within dilated bronchi.
• In asthma patients, this is classically seen in allergic bronchopulmonary aspergillosis (ABPA), a
hypersensitivity reaction to Aspergillus colonization of the airways.
• Cystic fibrosis can also show similar opacities but is less likely in a middle-aged adult without
suggestive history.
EBDR Exam MCQs & Concepts May 2023
Dr. Kareem Alnakeeb
146
• Acute pulmonary embolism and sarcoidosis do not typically present with branching tubular opacities
on imaging.
• Primary lung carcinoma can cause mass-like consolidations or obstructive changes, not the
characteristic pattern described here.
6. A 26 year old man suffers a blunt injury to his chest in a road traffic accident. The most common
abnormality seen on CT as a result of blunt thoracic injury is:
A. Pneumothorax
B. Pulmonary laceration
C. Haemothorax
D. Tracheo-bronchial injuries
E. Pulmonary contusion
Source: Currie, Stuart, et al. SBAs for the FRCR 2A. 1st ed., Cambridge University Press, 2010.
Explanation:
• Pulmonary contusion is the commonest manifestation of blunt trauma and indicates trauma to
alveoli with alveolar haemorrhage without significant alveolar disruption.
• Whilst plain film changes may not be apparent for up to six hours, CT will demonstrate changes
almost immediately post-trauma and signs of resolution can be seen as early as 48 hours.
• If unresolved, it may progress to ARDS.
7. A 46-year-old motorcyclist was involved in a high-speed RTA. On arrival of the paramedics, the GCS was
recorded as 4/15 and the patient was intubated at the site of injury. An emergency noncontrast CT
showed multiple subtle petechial hemorrhages characteristic of diffuse axonal injury. What is the most
likely site of the petechial hemorrhage?
A. Insular ribbon
B. Watershed areas
C. Periventricular white matter
D. Grey-white matter junction
E. Cerebellum
Source: Proctor, Robin. Final FRCR Part A Modules 4-6 Single Best Answer MCQs: The SRT Collection
of 600 Questions with Explanatory Answers (MasterPass). 1st ed., CRC Press, 2009.
Explanation:
• Diffuse axonal injury (DAI) occurs in severe trauma as a result of shearing stress along the course of
the white matter tracts especially at the grey-white matter junction.
• The injury is usually microscopic and initial CTs are usually normal despite profound clinical
impairment.
• Acute DAI may also be seen as small petechial hemorrhages at the grey-white matter junction (67%),
internal/external capsule, corona radiata, corpus callosum (21%) and brainstem.
• MR features depend on the age of the hemorrhage. Prognosis is poor.
8. Following renal trauma, a renal angiogram is performed to assess which of the following complications?
A. Hematuria
B. Urinary leakage
C. Hypotension
D. Hypertension
E. None of the above
EBDR Exam MCQs & Concepts May 2023
Dr. Kareem Alnakeeb
147
Explanation: (by Perplexity AI, generated with the OpenAI O3 model)
• Renal angiography is primarily indicated after renal trauma to assess ongoing or severe hematuria,
typically due to vascular injury such as pseudoaneurysm or arteriovenous fistula formation. It helps
locate the source of bleeding and can be therapeutic via embolization.
• While urinary leakage is best evaluated by CT urography or retrograde pyelography, and hypotension
is a clinical sign rather than an imaging finding, hypertension is a late or chronic sequela that can
result from vascular injury but is not acutely assessed by angiography in the trauma setting.
9. In classical transposition of the great arteries (TGA), which pattern of atrioventricular (AV) and
ventriculoarterial (VA) connections is typically present?
A. AV concordance, VA concordance
B. AV concordance, VA discordance
C. AV discordance, VA concordance
D. AV discordance, VA discordance
E. AV discordance, normal VA connections
Explanation: (by Perplexity AI, generated with the OpenAI O3 model)
• In classical (D-loop) transposition of the great arteries, the atrioventricular (AV) connections are
concordant (right atrium to right ventricle, left atrium to left ventricle), but the ventriculoarterial (VA)
connections are discordant—the aorta arises from the right ventricle and the pulmonary artery from
the left ventricle. This anatomical configuration causes parallel rather than series circulation,
resulting in severe cyanosis after birth unless there is mixing of blood.
• Options with AV discordance (C, D, E) describe other complex congenital heart diseases, such as
congenitally corrected transposition (L-TGA), not the classical TGA pattern.
10. A 59-year-old patient with Child-Pugh A cirrhosis is found to have a solitary 5cm hepatocellular
carcinoma. Which of the following locoregional therapies is generally considered to be limited by tumour
size below 5cm?
A. Transarterial chemoembolization (TACE)
B. Transarterial radioembolization (TARE)
C. Radiofrequency ablation
D. Microwave ablation
E. Stereotactic body radiotherapy
Explanation: (by Perplexity AI, generated with the OpenAI O3 model)
• Radiofrequency ablation (RFA) is typically reserved for hepatocellular carcinoma (HCC) lesions up to
3–5cm; efficacy significantly declines for tumours larger than 5cm, and complete ablation is less
likely to be achieved.
• Microwave ablation (MWA) also has size limitations but can more reliably treat tumours approaching
5cm, while TACE and TARE are not strictly limited by nodule size in the same way and are used for
larger or multifocal disease.
• Stereotactic body radiotherapy can also be considered for tumours above this size. Therefore, RFA is
the option most limited by a size threshold of <5cm.
EBDR Exam MCQs & Concepts May 2023
Dr. Kareem Alnakeeb
148
11. On musculoskeletal ultrasound, a tendoachilles was assessed but distal assessment was limited due to
a static procedure. The tendon appeared hyperechoic with hypoechoic areas, was hypoechoic compared
to the related nerve, and the distal part could not be evaluated. What is the most likely abnormality?
A. Tendon xanthoma
B. Achilles tendon rupture
C. Normal tendon
D. Paratenonitis
E. Partial thickness tear
Explanation: (by Perplexity AI, generated with the OpenAI O3 model)
• The described ultrasound findings—hyperechoic tendon with interspersed hypoechoic areas and
inability to fully assess the distal tendon—best fit a partial thickness tear of the Achilles tendon.
• Chronic tendinopathy or xanthomas tend to show diffuse thickening, but the classic pattern for tears
is disruption of tendon fibers with mixed echogenicity and potential inability to visualise the distal
end, especially if static imaging is used. A complete rupture usually shows clear tendon discontinuity
with a gap, whereas normal tendon is uniformly echogenic and fibrillar.
• Paratenonitis may show surrounding fluid or inflammation but does not typically cause hypoechoic
defects within the tendon itself.
12. On intravenous pyelogram (IVP), which of the following is the most likely cause of a medially deviated
ureter?
A. Atrophy of psoas muscle
B. Enlarged paraaortic lymph nodes
C. Retrocaval ureter
D. Urinoma
E. Retroperitoneal fibrosis
Explanation: (by Perplexity AI, generated with the OpenAI O3 model)
• Retrocaval ureter results in medial deviation of the ureter on IVP due to the abnormal course of the
ureter passing posterior to the inferior vena cava and then looping medially.
• Atrophy of the psoas muscle may cause lateral displacement of the ureter.
• Enlarged paraaortic lymph nodes usually push the ureter laterally, not medially.
• Urinoma is unlikely to affect ureteral position, and retroperitoneal fibrosis typically causes lateral or
irregular deviation with medial constraint rather than frank medialization.
13. What is the most common benign lesion of the phalanx, which may present with pathological fracture?
A. Chondroblastoma
B. Enchondroma
C. Osteoid osteoma
D. Giant cell tumour
E. Aneurysmal bone cyst
Explanation: (by Perplexity AI, generated with the OpenAI O3 model)
• Enchondroma is the most common benign lesion of the phalanx and frequently affects the small
bones of the hands and feet. It often presents as an incidental finding but may cause thinning of the
cortex and predispose to pathological fractures.
• Chondroblastoma and giant cell tumour are rare in the phalanges and typically occur at different
sites.
• Osteoid osteoma usually involves the diaphysis of long bones and presents with nocturnal pain.
EBDR Exam MCQs & Concepts May 2023
Dr. Kareem Alnakeeb
149
• Aneurysmal bone cysts can also occur in the phalanges but are much less common than
enchondroma.
14. A 25-year-old man presents with persistent knee pain. Radiographs show a well-defined lytic lesion in the
epiphysis of the distal femur with surrounding chondroid-type ring-and-arc calcifications. What is the
most likely diagnosis?
A. Giant cell tumour
B. Chondroblastoma
C. Osteosarcoma
D. Fibrous dysplasia
E. Aneurysmal bone cyst
Explanation: (by Perplexity AI, generated with the OpenAI O3 model)
• Chondroblastoma characteristically arises in the epiphysis of long bones in young patients and
appears as a lytic lesion often containing “ring-and-arc” (chondroid) calcifications due to the
cartilaginous matrix.
• This differentiates it from giant cell tumour (A), which usually lacks matrix calcification and affects
older patients.
• Osteosarcoma (C) often has aggressive features and produces osteoid rather than cartilage; fibrous
dysplasia (D) has a ground-glass matrix; aneurysmal bone cyst (E) is expansile but lacks internal
calcification and affects metaphyses.
15. In a pregnant woman with a history of previous Caesarean section and current placenta previa, what is
the most important assessment during the second pregnancy?
A. Retroplacental Doppler flow
B. Congenital fetal anomaly
C. Oligohydramnios
D. Intra-placental lakes
E. Placenta accreta spectrum
Explanation: (by Perplexity AI, generated with the OpenAI O3 model)
• The most critical assessment in a woman with a previous Caesarean section and a current diagnosis
of placenta previa is for placenta accreta spectrum (PAS). The risk of abnormal placental adherence
(including accreta, increta, and percreta) is markedly increased in this setting.
• Ultrasound with or without MRI is performed to assess for features of PAS, which significantly impact
delivery planning and maternal risk.
• Retroplacental Doppler, fetal anomalies, and oligohydramnios are relevant but not as closely linked
to morbidity or immediate management needs in this context.
• Intra-placental lakes may be seen in PAS but are a nonspecific finding.
16. A patient presents with a tumour located in the upper anorectal canal and regional lymph node
enlargement is seen. Which lymph node group is most likely to be involved?
A. External iliac
B. Internal iliac
C. Common iliac
D. Pudendal
E. Obturator
EBDR Exam MCQs & Concepts May 2023
Dr. Kareem Alnakeeb
150
Explanation: (by Perplexity AI, generated with the OpenAI O3 model)
• The upper anorectal canal drains primarily to the internal iliac lymph nodes due to its embryological
origin above the dentate line. This region follows the lymphatics of the superior rectal and middle
rectal vessels, directing spread to internal iliac nodes first.
• External iliac and common iliac nodes may be involved in more advanced disease but are not
primary nodal groups for this location.
• Pudendal and obturator nodes are less commonly involved from tumours originating in the upper
anorectal canal.
17. A pregnant woman presents with acute abdominal pain and vomiting. The pregnancy was previously
normal. Imaging shows a dilated bowel loop with circumferential thickening, delayed enhancement, and
pneumobilia. What is the most likely diagnosis?
A. Intestinal obstruction
B. Volvulus
C. Internal hernia
D. Mesenteric ischaemia
E. Gallstone ileus
Explanation: (by Perplexity AI, generated with the OpenAI O3 model)
• Gallstone ileus is suggested by the combination of small bowel obstruction, pneumobilia (air in the
biliary tree), and a dilated bowel loop, especially with history of prior normal pregnancy and acute
presentation. The classic radiological triad—small bowel obstruction, ectopic gallstone, and
pneumobilia—is nearly pathognomonic.
• Intestinal obstruction is a descriptive term and not a specific cause; volvulus or internal hernia could
present similarly but would not explain pneumobilia.
• Mesenteric ischaemia may cause bowel wall thickening and enhancement delay, but does not
account for pneumobilia.
• Gallstone ileus results from a fistula between the gallbladder and GI tract allowing a gallstone to
enter and obstruct the bowel.
18. Which of the following is most characteristic of a flash-filling hepatic haemangioma on MRI?
A. T1 hypointensity
B. T2 hyperintensity
C. Delayed washout
D. All of the above
E. None of the above
Explanation: (by Perplexity AI, generated with the OpenAI O3 model)
• Flash-filling hepatic haemangiomas typically appear markedly hyperintense on T2-weighted MRI
sequences, which is a key distinguishing feature.
• While they can be mildly hypointense on T1, this is less specific.
• Delayed washout is not a feature; instead, haemangiomas show persistent enhancement after the
initial rapid fill.
• "All of the above" is incorrect because delayed washout is not observed, making option B the best
answer.
EBDR Exam MCQs & Concepts May 2023
Dr. Kareem Alnakeeb
151
19. Which of the following statements about tumefactive sludge in the gallbladder is correct?
A. Shows vascularity with colour Doppler
B. Looks like a mass
C. Rarely occurs with gallstones
D. All of the above
E. None of the above
Explanation: (by Perplexity AI, generated with the OpenAI O3 model)
• Tumefactive sludge in the gallbladder can mimic a mass due to its echogenic and mass-like
appearance on ultrasound, but it typically does not demonstrate vascularity on colour Doppler,
helping to distinguish it from true neoplastic lesions.
• While tumefactive sludge can occur without gallstones, it is commonly associated with gallstones
and biliary stasis, making option C incorrect. Therefore, the most accurate choice is that tumefactive
sludge looks like a mass but does not usually fulfill the other criteria.
20. On pelvic MRI, which imaging feature best helps differentiate a hemorrhagic ovarian cyst from an
endometrioma?
A. Hemorrhagic cyst shows dark spots in T2
B. Hemorrhagic cyst shows facilitated DW
C. Hemorrhagic cyst shows bright ADC
D. Shading is more in Hemorrhagic cyst
E. Endometrioma shows high T2 signal
Explanation: (by Perplexity AI, generated with the OpenAI O3 model)
• Hemorrhagic cysts typically show facilitated diffusion on DWI, meaning they have high signal on ADC
maps owing to the lack of true diffusion restriction, helping distinguish them from endometriomas.
Endometriomas often show "T2 shading," a progressive loss of T2 signal due to repeated hemorrhage
and high protein content, whereas hemorrhagic cysts do not typically exhibit this pronounced
shading effect.
• True restricted diffusion (low ADC) is more likely in endometriomas than in simple hemorrhagic
cysts.
• Hemorrhagic cysts may have variable T2 signal but do not characteristically show “dark spots” or
more shading than endometriomas.
21. An 18-year-old mountain bike enthusiast is suspected of sustaining a renal injury after attempting a front
wheel touch-up manoeuvre. A laceration to the right kidney is noted on CT, which demonstrates contrast
enhancement during the pyelographic phase of the examination. What is the significance of this finding?
A. Pre-existing angiomyolipoma.
B. Active haemorrhage.
C. Devascularization.
D. Renal infarction.
E. Urine leak.
Source: Lindsay, Richard, et al. SBAs for the FRCR Part 2A. 1st ed., Oxford University Press, 2012.
Explanation:
Lacerations and Contrast Enhancement
• Lacerations generally contain clotted blood and therefore do not enhance on scans obtained with
intravenous contrast.
• Intense enhancement within a laceration during the early phase indicates active haemorrhage.
EBDR Exam MCQs & Concepts May 2023
Dr. Kareem Alnakeeb
152
Indications of Urine Leak and Urinary Extravasation
• Contrast enhancement during the pyelographic phase of the CT examination indicates the presence
of a urine leak.
• A delayed scan of 10–15 minutes may show the extent of the urinary extravasation.
Infarction and Contusions
• Focal areas of infarction do not enhance (unlike contusions).
Cortical Rim Nephrogram and Kidney Devitalization
• The cortical rim nephrogram is a sign of a devascularized kidney, which occurs due to laceration of
the main renal artery.
22. A 55-year-old female patient presents to your hospital with a history of recurrent UTIs and gross
haematuria. Repeated urine cultures are negative, but analysis reveals copious white and red cells in the
urine. The patient fails to improve with antibiotics. A CT scan of renal tracts is carried out, which shows
an atrophic right kidney containing calcification. There is also an area of increased density on the
unenhanced portion of the scan noted in the upper pole of the kidney, with overlying cortical thinning.
There are multiple strictures noted in the ureter, with intervening areas of dilatation, giving a corkscrew
appearance. There is extensive coarse calcification noted in the wall of the bladder. A CXR is carried out
and is normal. Early morning urine collections finally identify mycobacterium tuberculosis in the urine,
confirming the suspicion of renal and urinary tract TB. Which of these features is atypical of renal TB?
A. High-density material in the calyceal system.
B. Bladder calcification.
C. Renal calcification.
D. Normal CXR.
E. Corkscrew appearance to the ureter.
Source: Lindsay, Richard, et al. SBAs for the FRCR Part 2A. 1st ed., Oxford University Press, 2012.
Explanation:
Schistosomiasis and Tuberculosis Associations
• Whilst this can be seen in TB, it is more typically associated with schistosomiasis.
Tuberculosis Effects on the Urinary Tract
• TB usually causes scarring and a reduced capacity bladder.
• Renal calcification is typical.
• The areas of increased attenuation within the calyceal system represent areas of coalescing
caseating granulomas, and may have associated calcification.
• Scarring can also cause stenosis of calyces, causing focal obstruction.
• Occasionally a small calcified kidney is found, evidence of autonephrectomy.
Spread and Imaging in Renal TB
• Passage of the infection via the urine into the ureters causes focal stenoses, which can coalesce to
cause a long stricture, or give a beaded or corkscrew appearance to the ureter.
• Whilst renal TB results from spread from a primary pulmonary infection, the CXR is only abnormal in
25–50% of cases and is therefore not helpful.
EBDR Exam MCQs & Concepts May 2023
Dr. Kareem Alnakeeb
153
23. A 45-year-old man with a history of urinary tract infections undergoes imaging for persistent haematuria.
Which of the following is NOT a typical radiological feature of renal tuberculosis?
A. Atrophic kidney
B. Hydronephrosis
C. Staghorn calculus
D. Parenchymal calcification
E. Bilateral polycystic change
Explanation: (by Perplexity AI, generated with the OpenAI O3 model)
• Renal tuberculosis typically results in an atrophic kidney due to chronic infection, may show
parenchymal or lobar (putty) calcification, and often leads to hydronephrosis due to infundibular or
ureteric stricture.
• A staghorn calculus can also occur secondary to chronic infection.
• Bilateral polycystic change is not characteristic of renal TB, which instead causes destructive, not
cystic, changes to the renal parenchyma.
24. A 40-year-old woman, 4 days post-liver transplant, presents with fever. She is scheduled for a routine
screening mammogram as part of breast cancer screening. What is the most appropriate step regarding
her mammography appointment?
F. Proceed with mammography as scheduled
G. Defer mammography until resolution of fever and reassessment
H. Cancel all future breast cancer screening
I. Immediate breast MRI instead of mammography
J. Begin annual mammography from age 50 only
Explanation: (by Perplexity AI, generated with the OpenAI O3 model)
• In the early post-liver transplant period, patients are immunosuppressed and at increased risk of
infection.
• Fever 4 days after transplantation is a red flag for possible infection or post-operative complication,
and elective procedures, including screening mammography, should be deferred until the patient is
clinically stable and fully assessed.
• Proceeding with mammography risks exposing the patient and staff to potential infection and may
delay essential medical management.
• There is no indication to permanently cancel breast screening, nor to switch to MRI based only on
fever or transplant status.
• Guidelines support age-appropriate screening for breast cancer regardless of transplant history, but
screening should be postponed during acute illness.
25. Osteoporosis circumscripta – well-defined geographic lytic lesions in the skull – represents the early
stages of which condition?
A. Paget’s disease
B. hyperparathyroidism
C. multiple myeloma
D. senile osteoporosis
E. sickle cell disease
EBDR Exam MCQs & Concepts May 2023
Dr. Kareem Alnakeeb
154
Source: Bydder, Megan, et al. Get Through Final FRCR Part A: SBAs for the Modular Examination. 1st
ed., Hodder Education Publishers, 2009.
Explanation:
Paget’s Disease Overview
• Paget’s disease is a common progressive disorder of osteoclasts and osteoblasts resulting in bone
remodelling.
• It is usually polyostotic and asymmetrical, and affects 10% of those aged over 80.
Initial Phase of Paget’s Disease
• Osteoporosis circumscripta is seen in the initial phase of Paget’s disease, which is characterized by
an aggressive, predominantly lytic process with intense osteoclastic activity causing bone
resorption.
• Bone marrow is replaced by fibrous tissue with large vascular channels.
Radiological Features
• Geographic osteoporosis is seen in the skull and long bones, where the characteristic feature is a
flame-shaped radiolucency beginning in a subarticular location and progressing into the diaphysis.
Disease Progression
• The disease then progresses through a mixed phase to a quiescent inactive late stage where bone
turnover is decreased.
Skull Involvement
• The skull is involved in 29–65% of cases, most commonly the anterior calvarium.
26. Which of the following is a recognised cause of a ‘bone within bone’ appearance?
A. Renal osteodystrophy
B. Paget’s disease
C. Hyperparathyroidism
D. Melorheostosis
E. Osteopathia striata
Source: Chin, Teck Yew, et al. Get Through Final FRCR 2A: SBAs. 1st ed., CRC Press, 2017.
Explanation:
• A ‘bone within bone’ appearance describes the radiographic appearance whereby a bone appears to
have another bone within it, which results from endosteal new bone formation.
• Recognised causes include Paget’s disease, sickle cell disease, thalassemia, Gaucher’s disease,
acromegaly, hypervitaminosis D, scurvy and rickets, among many others.
• It can also be a normal finding in infants, particularly in the thoracolumbar spine.
27. In a 26 year old woman with sickle cell disease, which one of the following would not be considered a
typical musculoskeletal manifestation of the disease?
A. Osteopaenia and trabecular thinning
B. ‘Bone within bone’ appearance
C. Avascular necrosis of the femoral head
D. Posterior vertebral scalloping
E. Fish deformity of the vertebrae
Source: Currie, Stuart, et al. SBAs for the FRCR 2A. 1st ed., Cambridge University Press, 2010.
Explanation:
• Posterior vertebral scalloping is not a feature.
EBDR Exam MCQs & Concepts May 2023
Dr. Kareem Alnakeeb
155
• The remainder are all classic features of sickle cell anaemia, along with ‘hair-on-end’ appearance of
the skull due to coarse granular osteoporosis and widening of the diploe.
• Osteomyelitis is a feature and is due to salmonella in over 50% of cases.
28. On pelvic MRI, which signal characteristic underlies the “shading sign” commonly seen in
endometriomas?
A. Bright T1, bright T2
B. Bright T1, dark T2
C. Bright T1, dark T2 due to hemosiderin content
D. Dark T1, dark T2
E. Bright T1, intermediate T2
Explanation: (by Perplexity AI, generated with the OpenAI O3 model)
• The MRI “shading sign” in endometriomas refers to lesions that are hyperintense on T1-weighted
images and show a loss of signal (hypointensity) on T2-weighted images. This results from repeated
hemorrhage and high protein content within the cyst, which causes T2 shortening—darkening the
lesion despite T1 brightness.
• While hemosiderin can further lower T2 signal, classic shading is not exclusively due to it,
distinguishing B from C.
• Options A and E are incorrect as endometriomas characteristically have dark, not bright, T2 signal.
• Option D does not match the hemorrhagic content typical for endometriomas.
29. Which of the following is NOT a functioning pancreatic tumour?
A. Insulinoma
B. Gastrinoma
C. Vipoma
D. Glucagonoma
E. Pancreatic cystadenoma
Explanation: (by Perplexity AI, generated with the OpenAI O3 model)
• Functioning pancreatic tumours are neuroendocrine neoplasms that secrete biologically active
hormones, leading to characteristic clinical syndromes.
• Classic examples include insulinoma (secreting insulin), gastrinoma (gastrin), VIPoma (vasoactive
intestinal peptide), and glucagonoma (glucagon). These tumours present with specific hormonal
syndromes such as hypoglycaemia, Zollinger-Ellison syndrome, Verner-Morrison syndrome, or
necrolytic migratory erythema.
• In contrast, pancreatic cystadenoma is a benign, non-functioning cystic neoplasm of the pancreas
and does not produce hormones or cause endocrine syndromes—making E the correct answer.
• Key distractors, such as glucagonoma, are rare but established functioning neuroendocrine tumours
of the pancreas.
EBDR Exam MCQs & Concepts May 2023
Dr. Kareem Alnakeeb
156
30. A 42-year-old man from an endemic region presents with persistent lower urinary tract symptoms and
haematuria. Imaging reveals calcification of the urinary bladder wall. Which of the following is the most
likely cause?
A. Chronic cystitis
B. Transitional cell carcinoma
C. Schistosomiasis
D. Bladder outlet obstruction
E. Interstitial cystitis
Explanation: (by Perplexity AI, generated with the OpenAI O3 model)
• Schistosomiasis, caused by Schistosoma haematobium, is strongly associated with calcified urinary
bladder walls, particularly in patients from endemic areas. The parasite’s eggs become embedded in
the urothelium, leading to granulomatous inflammation and eventual calcification.
• Tuberculosis (TB) can cause urinary tract calcification but is far less likely to produce a uniformly
calcified bladder wall compared to schistosomiasis.
• Chronic cystitis may cause irregular thickening but rarely true diffuse calcification.
• Transitional cell carcinoma may create focal calcifications but does not involve the entire bladder
wall.
• Bladder outlet obstruction and interstitial cystitis do not cause bladder wall calcification.
31. A 56-year-old woman has a palpable left breast lump. Mammography categorises the lesion as BIRADS 2.
What is the most appropriate next step?
A. MRI
B. Core biopsy
C. Reassurance of the patient
D. Excision biopsy
E. Fine-needle aspiration cytology
Explanation: (by Perplexity AI, generated with the OpenAI O3 model)
• BIRADS 2 lesions are definitively benign by imaging criteria, requiring no further diagnostic
intervention beyond routine screening. Reassurance is appropriate and no biopsy (core or excision)
or MRI is indicated, since there is no suspicion of malignancy.
• Biopsies (options B, D, E) are reserved for indeterminate or suspicious lesions (BIRADS 3-5).
• MRI (option A) is not indicated when the lesion is unequivocally benign on standard imaging.
32. What is the primary imaging modality to differentiate solid from cystic lesions in clinical practice?
A. Mammography
B. Ultrasound
C. CT
D. MRI
E. PET
Explanation: (by Perplexity AI, generated with the OpenAI O3 model)
• Ultrasound is the modality of choice for distinguishing solid from cystic lesions due to its real-time
capability, high sensitivity in detecting fluid content, and ability to assess lesion vascularity with
Doppler imaging.
• Mammography provides limited information on internal lesion composition, primarily showing
density.
EBDR Exam MCQs & Concepts May 2023
Dr. Kareem Alnakeeb
157
• CT and MRI can characterize lesions but are less practical as first-line tools for this purpose due to
cost, accessibility, and radiation (CT).
• PET does not reliably distinguish cystic from solid lesions.
33. On DCE MRI of the breast, which time–signal intensity curve pattern best represents an intermediate
(type II) curve, characterised by an initial signal rise of about 10% followed by a plateau phase?
A. Continuous increase throughout imaging
B. Initial rise and plateau after early enhancement
C. Rapid rise then decrease (washout)
D. No significant enhancement
E. Both A & B
Explanation: (by Perplexity AI, generated with the OpenAI O3 model)
• The type II (intermediate) curve on DCE MRI of the breast is defined by an initial rise in signal
intensity—commonly 10% or more—followed by a plateau phase, with minimal further change. This
pattern is considered suspicious, as it may be seen in both benign and malignant lesions but is more
indeterminate than type I (persistent/continuous increase, seen mostly in benign lesions) or type III
(washout, strongly associated with malignancy).
• Option B most accurately captures this classic type II (plateau) pattern.
• Options A and E refer to the persistent enhancement seen in benign lesions, whereas C describes
the washout (type III) typically linked to malignancy, and D is incorrect as it indicates absence of
enhancement.
34. Which statement about breast cancer screening in low-risk women is most accurate?
A. Screening ultrasound is preferred over mammography for women over 40 years.
B. Contrast-enhanced mammography has entirely replaced standard mammography.
C. MRI is routinely performed annually in all low-risk women.
D. Ultrasound is the first-line screening tool for women under 40 years.
E. All women should begin mammography screening by age 25.
Explanation: (by Perplexity AI, generated with the OpenAI O3 model)
• For low-risk women under 40 years, ultrasound is typically preferred as the first-line imaging modality
due to higher breast density and the increased radiation risk in younger patients.
• Mammography is the mainstay of screening from age 40, but for asymptomatic low-risk women
below this age, routine screening with MRI or contrast-enhanced mammography is not
recommended.
• Annual MRI is reserved for high-risk categories.
• Contrast-enhanced mammography is emerging but does not replace standard mammography for
population screening.
• Mammography before age 40–50 is not standard unless specific risk factors are present.
35. Which of the following is considered the most malignant ultrasonographic feature of lymph node
malignancy?
A. Increased size >10mm
B. Infiltrated hilum
C. Increased cortex
D. Vascularity of the hilum
E. Smooth, thin cortex
EBDR Exam MCQs & Concepts May 2023
Dr. Kareem Alnakeeb
158
Explanation: (by Perplexity AI, generated with the OpenAI O3 model)
• Infiltration or loss of the normal fatty hilum is the most malignant sonographic sign of lymph node
malignancy, as it suggests tumor replacement of lymph node architecture.
• While increased size (>10mm) and cortical thickness may be seen in both benign and malignant
conditions, and vascularity of the hilum is typical of benign nodes, not malignant ones, the presence
of a smooth, thin cortex is also a feature of benignity.
• Key distractors (A, C, D) often occur with benign or reactive nodes, but infiltrated hilum (B) indicates
a higher suspicion for malignancy.
36. A 60-year-old woman presents with a palpable lump in her right breast. Her recent screening
mammogram 6 months previously was negative. Clinical examination reveals a subtle mass in the right
lower quadrant. Which of the following mammographic findings is the most common in invasive lobular
carcinoma (ILC)?
A. Spiculated mass.
B. Architectural distortion.
C. Microcalcifi cation.
D. Nipple retraction.
E. Skin thickening.
Source: Lindsay, Richard, et al. SBAs for the FRCR Part 2A. 1st ed., Oxford University Press, 2012.
Explanation:
• ILC is the second most common form of invasive breast cancer, after ductal carcinoma. It exhibits
the same mammographic features as invasive ductal carcinoma, although architectural distortion is
the most common mammographic finding. Due to the pattern of small cells growing around ducts (
‘Indian files’), mammographic findings are subtle and thus ILC is the most frequently missed
breast cancer. Prognosis is generally poor due to late diagnosis.
37. Which one of the following is the most characteristic mammographic feature of a malignant breast
mass?
A. Spiculated mass and clustered microcalcification
B. Smooth margins
C. Ill-defined infiltrative mass with macrocalcification
D. Asymmetrical distribution and nipple retraction
E. Well-circumscribed mass with uniform density
Explanation: (by Perplexity AI, generated with the OpenAI O3 model)
• Spiculated mass margins and clustered microcalcifications are highly suggestive of malignancy on
mammography, reflecting irregular tumour infiltration and ductal carcinoma in situ components.
• Smooth margins and well-circumscribed masses (options B and E) are more typical of benign lesions
such as fibroadenomas or cysts.
• Ill-defined infiltrative mass with macrocalcification (option C) is less specific for malignancy—
macrocalcifications usually represent benign involution.
• Asymmetrical distribution and nipple retraction (option D) are concerning, but on their own are less
specific than the classical combination in option A.
EBDR Exam MCQs & Concepts May 2023
Dr. Kareem Alnakeeb
159
38. A 54-year-old female patient presents with anaemia and haematuria. A CT of abdomen confi rms renal
cell carcinoma of the right kidney, but there is also enlargement of the right adrenal gland. Which of the
following CT characteristics is most consistent with a benign adrenal adenoma?
A. A pre-contrast attenuation of 50.
B. An immediate post-contrast attenuation of 50.
C. A relative percentage washout (RPW) of 60%.
D. Lesion size of 50 mm.
E. Heterogeneity of the lesion.
Source: Lindsay, Richard, et al. SBAs for the FRCR Part 2A. 1st ed., Oxford University Press, 2012.
Explanation:
Adrenal adenoma diagnostic criteria
• Findings consistent with an adrenal adenoma are: a pre-contrast attenuation of 10 HU or less, an
absolute percentage washout (APW) of 60% or greater, or an RPW of 40% or greater.
Percentage washout calculations
• The percentage washout is calculated by comparing the attenuation value at 15 minutes post
contrast (delayed H), to the value in the portal venous phase (enhanced H), and in the case of APW,
the pre-contrast value.
• RPW = 100 × (enhanced H – delayed H)/enhanced H
• APW = 100 × (enhanced H – delayed H)/(enhanced H – pre-contrast H)
Practical imaging considerations
• In practice, an unenhanced scan is not usually performed and thus only the RPW is calculated.
Adrenal cortical carcinoma indicators
• Adrenal cortical carcinomas usually have an RPW of less than 40% although exceptions have been
reported.
• Their large size (usually greater than 6 cm), heterogeneity pre-contrast (necrosis), and heterogeneous
enhancement are more reliable indicators of the diagnosis.
Other lesions that may mimic adenoma
• Phaeochromocytomas and hypervascular metastases may mimic adenomas, but most metastases
show RPW < 40% and APW < 60%.
39. A 14-year-old girl presents with sudden onset of severe right-sided lower abdominal pain, nausea, and
vomiting. An ultrasound of the pelvis is requested with a suspected diagnosis of ovarian torsion. Which of
the following is the most constant ultrasound finding in ovarian torsion?
A. Enlarged ovary.
B. Absent ovarian blood flow.
C. Pelvic free fluid.
D. Twisted ovarian pedicle.
E. ‘String of pearls’ sign.
Source: Lindsay, Richard, et al. SBAs for the FRCR Part 2A. 1st ed., Oxford University Press, 2012.
Explanation:
Prevalence and Risk Factors
• Ovarian torsion can occur in all age groups, with the highest prevalence in the reproductive age
group.
• Large heavy cysts and cystic neoplasms commonly predispose to ovarian torsion.
• Torsion of normal ovaries is unusual but more common in adolescents.
• Benign cystic teratoma is the most common tumour predisposed to ovarian torsion.
EBDR Exam MCQs & Concepts May 2023
Dr. Kareem Alnakeeb
160
Clinical Presentation
• Clinical symptoms are often nonspecific and therefore imaging is routinely requested.
Ultrasound Findings
• On ultrasound, an enlarged ovary is the most constant finding.
• The enlarged ovary may be heterogeneous due to haemorrhage and oedema.
• Other findings include multiple small cysts aligned in the periphery of the enlarged ovary (string of
pearls sign), coexistent mass, pelvic free fluid, and twisted ovarian pedicle.
Doppler Findings
• Absent arterial flow is described as the classic colour Doppler finding.
• However, the most frequent Doppler finding is decreased or absent venous flow with or without
reduced arterial flow.
• A twisted ovarian pedicle with a ‘whirlpool’ sign is a useful finding on colour Doppler.
40. What is the most common soft tissue mass found in the hand and wrist?
A. Hemangioma
B. Giant cell tumor of tendon sheath
C. Lipoma
D. Epidermoid cyst
E. Ganglion cyst
Explanation: (by Perplexity AI, generated with the OpenAI O3 model)
• Ganglion cysts are the most frequent soft tissue masses in the hand and wrist, encountered in both
adult and paediatric populations. They typically arise near joint capsules or tendon sheaths,
presenting as firm, well-defined swellings—often on the dorsal wrist.
• In contrast, lipomas are rare in this location, hemangiomas are more common in childhood, and
giant cell tumours of tendon sheath—while possible—are less prevalent than ganglion cysts.
• Epidermoid cysts are also uncommon in this anatomical area. Identification relies on clinical history,
examination, and imaging, especially ultrasound or MRI for atypical cases.
41. Which anatomical site is most commonly affected by Freiberg’s avascular necrosis in the foot?
A. Navicular
B. Scaphoid
C. Second metatarsal head
D. Talus
E. Fifth metatarsal base
Explanation: (by Perplexity AI, generated with the OpenAI O3 model)
• Freiberg’s disease refers to avascular necrosis (AVN) of the metatarsal head, most classically
affecting the second metatarsal. This is due to its length and relative vulnerability to altered
biomechanics and blood supply, especially in adolescent females.
• The navicular and scaphoid are both prone to AVN, but in different contexts: the navicular is
classically affected in Kohler’s disease, and the scaphoid (in the wrist) in the context of fracture.
• The talus and fifth metatarsal base are not commonly associated with Freiberg’s AVN.
EBDR Exam MCQs & Concepts May 2023
Dr. Kareem Alnakeeb
161
42. Which of the following is NOT a recognized radiological sign of intestinal tuberculosis?
A. Fleischner sign
B. String sign
C. Stierlin's sign
D. Comb sign
Explanation: (by Perplexity AI, generated with the OpenAI O3 model)
• The comb sign (engorged mesenteric vessels) is typically seen in Crohn’s disease, not intestinal
tuberculosis.
• Fleischner sign (thickened loop due to spasm), string sign (narrow, rigid segment), and Stierlin's
sign (rapid passage through ulcerated segment with minimal mucosal coating) are all described in
intestinal TB and help differentiate it from other pathologies.
• The comb sign is a classic finding in active Crohn’s, reflecting hypervascularity, and is not associated
with tuberculosis.
43. A 36-year-old female presents with progressive lower abdominal distension and menorrhagia. On
examination, there is a large, firm, irregular pelvic mass arising from the pelvis. What is the most likely
diagnosis?
A. Endometriosis
B. Uterine fibroid
C. Dermoid cyst
D. Mesenteric cyst
E. Ovarian carcinoma
Explanation: (by Perplexity AI, generated with the OpenAI O3 model)
• The most likely diagnosis is a uterine fibroid (leiomyoma), which typically presents as a large, firm,
irregular pelvic mass associated with menorrhagia in reproductive-age women.
• Fibroids are hormone-dependent benign smooth muscle tumors of the uterus and classically cause
heavy menstrual bleeding and mass effect symptoms.
• Endometriosis may cause pelvic pain and dysmenorrhoea but rarely presents with a palpable mass
of this size.
• Dermoid cysts (mature teratomas) are ovarian in origin but usually not associated with menorrhagia
or irregular, firm uterine masses.
• Mesenteric cysts are rare, extrapelvic, and non-gynecological, and do not cause uterine-related
symptoms.
• Ovarian carcinoma often presents later in life and with non-specific symptoms or systemic features.
44. What is the most common primary malignant retroperitoneal tumor encountered in adults?
A. Neuroblastoma
B. Liposarcoma
C. Leiomyosarcoma
D. Malignant lymphoma
E. Synovial sarcoma
Explanation: (by Perplexity AI, generated with the OpenAI O3 model)
• Liposarcoma is the most common primary malignant retroperitoneal tumor in adults. It accounts for
around a third of all retroperitoneal sarcomas. The large retroperitoneal space allows these tumors
to grow to a significant size before becoming symptomatic.
• Leiomyosarcoma is the second most common but occurs less frequently than liposarcoma.
• Neuroblastoma is a pediatric tumor seen predominantly in children.
EBDR Exam MCQs & Concepts May 2023
Dr. Kareem Alnakeeb
162
• Malignant lymphoma can involve the retroperitoneum but is not a primary retroperitoneal sarcoma.
• Synovial sarcoma is rare in this location.
45. A 25-year-old woman presents with an incidental abdominal mass on ultrasound. CT reveals a well-
defined, unilocular cystic lesion involving multiple retroperitoneal compartments, with no enhancement
or calcification. Which is the most likely diagnosis?
A. Lymphatic malformation
B. Pancreatic pseudocyst
C. Retroperitoneal sarcoma
D. Paraganglioma
E. Mesenteric cyst
Explanation: (by Perplexity AI, generated with the OpenAI O3 model)
• Lymphatic malformations in the retroperitoneum typically present as well-defined unilocular or
multilocular cystic lesions that may cross anatomical boundaries or compartments. They lack
enhancement and are usually not associated with calcification or solid components.
• Pancreatic pseudocysts generally relate to a history of pancreatitis and usually do not cross
compartments.
• Retroperitoneal sarcomas are more often solid, heterogeneous, and may enhance.
• Paragangliomas are vascular and enhancing masses, not cystic.
• Mesenteric cysts are usually confined to the mesentery and do not typically involve multiple
retroperitoneal compartments.
46. An 18-year-old male fractures his pelvis following a motorcycle accident. He is suspected of sustaining a
bladder injury and undergoes CT cystography. This reveals ill-defi ned contrast medium within the peri-
vesical space with a ‘molar-tooth’ appearance. What is the signifi cance of this fi nding?
A. Interstitial bladder injury.
B. Intraperitoneal rupture.
C. Extraperitoneal rupture.
D. Combined intra- and extraperitoneal rupture.
E. Bladder contusion.
Source: Lindsay, Richard, et al. SBAs for the FRCR Part 2A. 1st ed., Oxford University Press, 2012.
Explanation:
Epidemiology and Imaging
• Over 70% of patients with traumatic bladder injury have a coexisting pelvic fracture.
• CT cystography is considered to be as accurate as conventional cystography.
Extraperitoneal Rupture
• Extraperitoneal rupture accounts for 80% of all cases of traumatic bladder injury.
• It occurs as a result of shearing forces or penetrating injury from bony fragments at the base of the
bladder.
• Contrast can also track down into the scrotum or thigh.
Intraperitoneal Rupture
• Intraperitoneal rupture (15% of cases) follows a direct blow to a distended bladder, with the tear
involving the bladder dome.
• Contrast will be seen to outline small bowel loops.
Other Injury Types
• Combined injuries occur in 5%.
• Interstitial injuries are rare and are detected by contrast dissecting into the bladder wall.
EBDR Exam MCQs & Concepts May 2023
Dr. Kareem Alnakeeb
163
Bladder Contusion
• Imaging is frequently normal in the setting of bladder contusion.
47. A 66-year-old man is diagnosed with a pancreatic body mass involving the celiac axis on CT. According to
widely used resectability criteria, which imaging feature confirms the tumour is unresectable?
A. Less than 180° tumour contact with the celiac axis
B. Greater than 180° tumour contact with the celiac axis without aortic involvement
C. Isolated splenic artery encasement
D. Less than 180° tumour contact with the common hepatic artery
E. Greater than 180° tumour contact with the celiac axis with aortic involvement
Explanation: (by Perplexity AI, generated with the OpenAI O3 model)
• Tumour contact of greater than 180° with the celiac axis involving the aorta (encasement) renders a
pancreatic tumour unresectable according to major guidelines, as this reflects extensive arterial
invasion not amenable to curative surgery.
• Tumours with less extensive encasement (>180° celiac axis contact without aortic involvement) may
still be approached with advanced surgical techniques in select cases after neoadjuvant therapy,
and isolated splenic artery or less common hepatic artery involvement do not preclude resection.
• Only E features definitive local arterial extension prohibiting surgery.
48. A 56-year-old man with a history of chronic gastroesophageal reflux presents with progressive dysphagia.
Endoscopy reveals a granular-appearing mucosa in the distal esophagus and a mid-esophageal stricture.
Which is the most likely underlying histopathological diagnosis?
A. Squamous cell carcinoma
B. Eosinophilic esophagitis
C. Barrett’s esophagus with intestinal metaplasia
D. Infective esophagitis
E. Achalasia
Explanation: (by Perplexity AI, generated with the OpenAI O3 model)
• Barrett’s esophagus is characterized by replacement of the normal squamous epithelium with
metaplastic columnar epithelium, typically in the distal esophagus, usually due to chronic reflux.
Granular mucosa described on endoscopy is a typical finding, and chronic acid injury can rarely lead
to strictures even in the mid-esophagus.
• Squamous cell carcinoma may also cause strictures but would more likely present as an ulcerated or
mass-forming lesion.
• Eosinophilic esophagitis often causes rings and linear furrows, not granular Barrett’s mucosa.
• Infective esophagitis presents with ulceration, and achalasia is a motility disorder rarely producing
these mucosal changes.
EBDR Exam MCQs & Concepts May 2023
Dr. Kareem Alnakeeb
164
49. A 67 year old man is referred for a barium swallow from the surgical outpatient department with a history
of dysphagia to solids. A mid-oesophageal stricture is demonstrated. Which one of the following causes
is unlikely to be in the differential?
A. Barrett’s oesophagus
B. Squamous cell carcinoma of the oesophagus
C. Schatzki ring
D. Caustic substance ingestion
E. Epidermolysis bullosa
Source: Currie, Stuart, et al. SBAs for the FRCR 2A. 1st ed., Cambridge University Press, 2010.
Explanation:
• All are reasonable differentials for a mid-oesophageal stricture, albeit with varying degrees of
frequency, with the exception of a Schatzki ring which is found in the lower oesophagus.
• It occurs near the squamocolumnar junction and is associated with reflux.
• It is nondistensible and best seen in the prone position on barium swallow examinations.
• Schatzki rings are often asymptomatic, but oesophageal dilatation may be required where dysphagia
is severe.
50. Which of the following combinations is required to make the diagnosis of hydrops fetalis?
A. Pleural effusion and mild polyhydramnios
B. Skin edema and pleural effusion
C. Single-organ ascites
D. Increased nuchal translucency alone
E. Pericardial effusion alone
Explanation: (by Perplexity AI, generated with the OpenAI O3 model)
• Hydrops fetalis is defined by the abnormal accumulation of fluid in at least two fetal compartments,
such as skin edema, pleural effusion, pericardial effusion, or fetal ascites.
• Therefore, the presence of both skin edema and pleural effusion (Option B) satisfies this definition.
Single findings alone (e.g., E or C) and isolated non-compartmental fluid (e.g., nuchal translucency
or polyhydramnios) do not meet the diagnostic threshold.
51. A 60-year-old man with a history of prior myocardial infarction undergoes myocardial perfusion
scintigraphy, which shows decreased perfusion alongside reduced contractility in the affected region.
What is the most likely explanation for these findings?
A. Myocardial stunning
B. Hibernating myocardium
C. Acute infarction
D. Hypertrophic cardiomyopathy
E. Restrictive cardiomyopathy
Explanation: (by Perplexity AI, generated with the OpenAI O3 model)
• Hibernating myocardium refers to chronic, viable myocardial tissue that demonstrates both
decreased perfusion and reduced contractility due to prolonged but reversible ischaemia. This state
persists until adequate perfusion is restored, at which point function may improve.
• Myocardial stunning (A) presents with reduced contractility but normal perfusion following transient
ischaemia.
• Acute infarction (C) often results in irreversible loss of contractility and perfusion due to cell death.
• Hypertrophic (D) and restrictive (E) cardiomyopathy commonly cause abnormal contractility but are
not typically associated with focal perfusion deficits related to reversible ischaemia.
EBDR Exam MCQs & Concepts May 2023
Dr. Kareem Alnakeeb
165
52. A 55-year-old woman with long-standing systemic sclerosis presents with worsening dysphagia; a recent
barium swallow (shown) demonstrates a smoothly dilated, aperistaltic distal two-thirds of the esophagus
with rapid emptying into the stomach. Which imaging pattern is most characteristic of this finding?
A. Achalasia (“bird-beak” taper)
B. Scleroderma oesophagus
C. Diffuse oesophageal spasm (“corkscrew” oesophagus)
D. Zenker diverticulum
E. Oesophageal carcinoma causing stricture
Explanation: (by Perplexity AI, generated with the OpenAI O3 model)
• In systemic sclerosis, atrophy and fibrosis of smooth muscle cause loss of peristalsis and lower
oesophageal sphincter incompetence, producing a smooth, tubular dilatation of the distal
oesophagus that empties freely into the stomach—the classic scleroderma pattern. Achalasia
instead shows a narrowed distal “bird-beak” with poor emptying; diffuse spasm gives a corkscrew
appearance; Zenker diverticulum is a pharyngeal pouch, not distal; malignant stricture produces an
irregular, shouldered narrowing, not uniform dilatation.
53. A patient has a dual phase subtraction study for investigation of hyperparathyroidism and a focus of
uptake is seen on the Tc-99m-mibi scan with a corresponding area of increased uptake on the I-123
study in the region of upper pole of the right lobe of the gland. What is the most likely cause for this
finding?
A. Functioning parathyroid adenoma
B. Papillary thyroid tumour
C. Multinodular goitre with prominent nodule
D. Solitary functioning thyroid nodule
E. Submandibular salivary gland
Source: Proctor, Robin. Final FRCR Part A Modules 4-6 Single Best Answer MCQs: The SRT Collection
of 600 Questions with Explanatory Answers (MasterPass). 1st ed., CRC Press, 2009.
Explanation:
• Multinodular goitre can give rise to heterogeneous uptake in both types of scan as a functioning
thyroid nodule will take up both tracers.
• The presence of smooth uniform uptake on the thyroid scan would have been more consistent with a
functioning parathyroid adenoma.
• Salivary gland uptake is seen with Tc-99m pertechnetate but not I131
.
• Thyroid malignancy presents as a cold (nonfunctioning) nodule in 90% of scans.
54. Regarding transient osteoporosis of the femoral head, which of the following statements is INCORRECT?
A. It is more common in females.
B. MRI typically demonstrates bone-marrow oedema.
C. Radiographs show a grade III avascular necrosis appearance of the femoral head.
D. An associated hip joint effusion is often present.
E. The condition is self-limiting and usually resolves within months.
Explanation: (by Perplexity AI, generated with the OpenAI O3 model)
• Transient osteoporosis (TOH) most frequently affects middle-aged men and women in the third-
trimester of pregnancy, so overall prevalence is higher in males, not females; MRI shows diffuse
marrow oedema with low T1 and high T2/STIR signal, and hip effusion is common, while the disease
follows a benign, self-limiting course resolving within 6–12 months.
• In contrast, a grade III avascular necrosis radiographic appearance (crescent sign/collapse) indicates
structural failure of subchondral bone and is not a feature of TOH, thereby making option C incorrect.
EBDR Exam MCQs & Concepts May 2023
Dr. Kareem Alnakeeb
166
• The other options describe recognised characteristics of TOH.
55. A 40-year-old male with a history of haematuria undergoes CT urography. Initial non-contrast scan
demonstrates right-sided medullary nephrocalcinosis. Following intravenous contrast administration, a
striated ‘paintbrush’ appearance of the renal medulla is noted. The left kidney is unremarkable. What is
the diagnosis?
A. Hyperparathyroidism.
B. Renal tubular acidosis.
C. Medullary sponge kidney.
D. Sarcoidosis.
E. Multiple myeloma.
Source: Lindsay, Richard, et al. SBAs for the FRCR Part 2A. 1st ed., Oxford University Press, 2012.
Explanation:
Causes of Medullary Nephrocalcinosis
• Hyperparathyroidism, renal tubular acidosis, and medullary sponge kidney are the three most
common causes of medullary nephrocalcinosis.
• The former two conditions are associated with hypercalciuria that results in uniform medullary
nephrocalcinosis.
• Sarcoidosis and multiple myeloma are associated with hypercalcemia resulting in bilateral
nephrocalcinosis.
Medullary Sponge Kidney Characteristics
• Medullary sponge kidney can affect the kidney segmentally, unilaterally, or bilaterally, therefore
unilateral nephrocalcinosis is suggestive of medullary sponge kidney.
• Medullary sponge kidney is characterized by cystic dilatation of collecting tubules.
• Urine stasis within the dilated tubules predisposes to infection and calculus formation within the
dilated tubules or urinary tracts.
• On excretory urogram, contrast within the dilated tubules produces a striated ‘paintbrush’
appearance of the renal pyramids.
56. An 18-year-old male fractures his pelvis following a motorcycle accident. He is suspected of sustaining a
bladder injury and undergoes CT cystography. This reveals ill-defi ned contrast medium within the peri-
vesical space with a ‘molar-tooth’ appearance. What is the signifi cance of this fi nding?
A. Interstitial bladder injury.
B. Intraperitoneal rupture.
C. Extraperitoneal rupture.
D. Combined intra- and extraperitoneal rupture.
E. Bladder contusion.
Source: Lindsay, Richard, et al. SBAs for the FRCR Part 2A. 1st ed., Oxford University Press, 2012.
Explanation:
Epidemiology and Fracture Association
• Over 70% of patients with traumatic bladder injury have a coexisting pelvic fracture.
Diagnostic Imaging
• CT cystography is considered to be as accurate as conventional cystography.
Extraperitoneal Rupture
• Extraperitoneal rupture accounts for 80% of all cases of traumatic bladder injury.
• It occurs as a result of shearing forces or penetrating injury from bony fragments at the base of the
bladder.
• Contrast can also track down into the scrotum or thigh.
EBDR Exam MCQs & Concepts May 2023
Dr. Kareem Alnakeeb
167
Intraperitoneal Rupture
• Intraperitoneal rupture (15% of cases) follows a direct blow to a distended bladder, with the tear
involving the bladder dome.
• Contrast will be seen to outline small bowel loops.
Combined and Interstitial Injuries
• Combined injuries occur in 5%.
• Interstitial injuries are rare and are detected by contrast dissecting into the bladder wall.
Bladder Contusion
• Imaging is frequently normal in the setting of bladder contusion.
57. A 1.5 cm well-defined hepatic lesion shows markedly hyperintense T2 signal, low T1 signal, and intense,
homogeneous arterial-phase enhancement that persists without wash-out on dynamic MRI. What is the
most likely diagnosis?
A. Hepatocellular carcinoma
B. Flash-filling hepatic haemangioma
C. Focal nodular hyperplasia
D. Hypervascular metastasis (renal cell carcinoma)
E. Cholangiocarcinoma
Explanation: (by Perplexity AI, generated with the OpenAI O3 model)
• Flash-filling (capillary) haemangiomas are typically small (<2 cm) lesions that are T1 hypointense and
T2 markedly hyperintense. They “flash-fill” with brisk, uniform arterial enhancement that matches
blood pool and retain contrast on portal and delayed phases, showing no true wash-out—distinct
from hypervascular malignancies such as HCC or metastases, which usually become hypo- or iso-
intense later.
• Focal nodular hyperplasia shows a central scar and becomes iso-enhancing on delayed images,
while cholangiocarcinoma is usually peripheral, fibrous, and shows progressive enhancement rather
than rapid homogeneous fill-in.
58. On ultrasound, gall-bladder “smudge” artefact is noted as an echogenic focus on the wall with internal
colour Doppler flow. Which is the MOST appropriate interpretation of this finding in routine practice?
A. Gall-bladder wall adenomyomatosis
B. Impacted cholesterol calculus
C. Early gall-bladder carcinoma
D. Hyperplastic cholecystosis (strawberry gall-bladder)
E. Biliary sludge ball
Explanation: (by Perplexity AI, generated with the OpenAI O3 model)
• The “smudge” or “comet-tail” appearance with intralesional Doppler flow represents a Rokitansky-
Aschoff sinus containing cholesterol crystals in segmental or focal adenomyomatosis; colour flow is
due to twinkling artefact, not vascularity.
• Stones and sludge (options B and E) usually cast an acoustic shadow and demonstrate no internal
Doppler signal.
• Hyperplastic cholecystosis (option D) produces diffuse mucosal speckling without a discrete wall-
based focus.
• Gall-bladder carcinoma (option C) manifests as a mural mass or irregular wall thickening and
genuine vascular flow, but true malignancy is rarely linked to the isolated “smudge” sign.
EBDR Exam MCQs & Concepts May 2023
Dr. Kareem Alnakeeb
168
59. Which statement regarding gallbladder carcinoma is CORRECT in relation to patients under 40 years of
age?
A. It is common and almost always associated with gallstones
B. It is common and rarely associated with gallstones
C. It is rare but still almost always associated with gallstones
D. It is rare and gallstones are present in only about half of cases
E. It never occurs because gallstones are absent at young age
Explanation: (by Perplexity AI, generated with the OpenAI O3 model)
• Gallbladder carcinoma is distinctly uncommon in patients younger than 40 years, but when it does
present, cholelithiasis is identified in roughly 50% of cases. Thus, the disease is both rare in the
young and not universally linked to stones.
• Option A is incorrect because although stones are a recognised risk factor, the tumour is not
common in this age group.
• Option B is wrong: stone association is not rare.
• Option C overstates the frequency of stone presence, while option E is false because gallbladder
cancer, though infrequent, can occur despite the lower prevalence of stones in youth.
60. A 58-year-old man presents with cramping abdominal pain and vomiting. CT enterography shows a 4 cm,
well-circumscribed exophytic mass arising from the jejunal wall and projecting both intraluminally and
extraluminally; mottled enhancement and central areas of low attenuation are noted. Which is the most
likely diagnosis?
A. Adenocarcinoma of the small bowel
B. Gastrointestinal stromal tumour (GIST)
C. Carcinoid tumour
D. Small-bowel lymphoma
E. Ectopic pancreatic rest
Explanation: (by Perplexity AI, generated with the OpenAI O3 model)
• GISTs are the second commonest benign small-bowel tumours after lipomas and typically originate
from the muscularis propria, producing mixed transmural–exophytic growth that can both obstruct
the lumen and extend outward. On CT they appear as well-defined, enhancing soft-tissue masses
that may contain necrotic or cystic areas, exactly as in this case.
• Primary small-bowel adenocarcinomas usually arise in the mucosa, are irregular and infiltrative
rather than well-circumscribed.
• Carcinoid tumours tend to be submucosal, small, intensely enhancing nodules often accompanied
by desmoplastic mesenteric stranding.
• Lymphoma often shows diffuse or segmental bowel wall thickening with bulky lymphadenopathy and
rarely causes obstruction due to its pliable nature.
• An ectopic pancreatic rest is a small submucosal nodule with central umbilication and rarely
exceeds 2 cm.
EBDR Exam MCQs & Concepts May 2023
Dr. Kareem Alnakeeb
169
61. Regarding eosinophilic granuloma, which of the following are true? (True or False)
A. Bone lesions are usually solitary
B. Vertebral pedicles are typically affected
C. The cranial vault is rarely involved
D. The mandible is rarely involved
E. A periosteal reaction is not seen on plain film
Source: Bell, J., and N. Davies. MCQs in Clinical Radiology: A Revision Guide for the FRCR. 1st ed.,
Remedica Pub Ltd, 2004.
Explanation:
A. True - About 75% of cases are monostotic.
B. False - Eosinophilic granuloma typically involves the vertebral body, most commonly the thoracic
spine, causing vertebra plana. It rarely involves the posterior elements and the disc spaces are
preserved.
C. False - The skull is the most frequent site of involvement. Typically there is a round or oval lucency
within the skull vault with a bevelled edge.
D. False - It often involves the mandible, causing a 'floating teeth' appearance.
E. False - Although a periosteal reaction is almost never seen in flat bones, a prominent periosteal
reaction is not unusual in the axial skeleton. This may simulate a neoplastic lesion.
62. In cervical spine trauma, which fracture pattern is associated with the highest risk of catastrophic spinal
cord injury and thus considered the most serious?
A. Flexion teardrop fracture
B. Hangman fracture
C. Extension teardrop fracture
D. Burst fracture of C1 (Jefferson fracture)
E. Clay-shoveller fracture
Explanation: (by Perplexity AI, generated with the OpenAI O3 model)
• Flexion teardrop fractures occur after high-energy flexion-compression, fracturing the anteroinferior
vertebral body corner and typically disrupting the posterior ligamentous complex. This often
produces severe kyphotic angulation and retropulsion of a large osseous fragment into the spinal
canal, leading to a high incidence of complete spinal cord injury.
• Hangman fractures (bilateral C2 pars fractures) usually spare the spinal canal, so neurological deficit
is uncommon.
• Extension teardrop fractures are avulsion injuries of the anterior inferior corner and, while unstable,
carry a lower neurological risk than flexion variants.
• Jefferson fractures are burst injuries of C1 with widened ring fragments that rarely cause cord
damage because the canal diameter increases.
• Clay-shoveller fractures are isolated lower cervical/thoracic spinous process avulsions and are
stable with negligible neurological risk.
EBDR Exam MCQs & Concepts May 2023
Dr. Kareem Alnakeeb
170
63. A 34-year-old woman with long-standing Hashimoto thyroiditis presents with biochemical thyrotoxicosis.
Pelvic ultrasound shows a 6 cm complex cystic-solid lesion in the left ovary; colour Doppler is
unremarkable. Which diagnosis best explains both the ovarian mass and her thyroid overactivity?
A. Dysgerminoma
B. Granulosa cell tumour
C. Mature cystic teratoma containing struma ovarii
D. Serous cystadenocarcinoma
E. Theca-lutein cyst
Explanation: (by Perplexity AI, generated with the OpenAI O3 model)
• Struma ovarii is a rare monodermal variant of mature cystic teratoma composed predominantly of
thyroid tissue; functioning lesions can secrete thyroid hormone and cause thyrotoxicosis. Imaging
typically shows a complex cystic-solid ovarian mass with variable echogenic components.
• Dysgerminomas and granulosa cell tumours do not contain thyroid tissue and are not linked to
hyperthyroidism.
• Serous cystadenocarcinoma is usually multilocular with papillary projections but likewise lacks
thyroid tissue and endocrine effects.
• Theca-lutein cysts arise from gonadotropin overstimulation, are usually bilateral and cystic, and do
not produce thyroid hormones.
64. A 24-year-old woman attends A&E with lower abdominal pain and vaginal bleeding. A pregnancy test is
positive. She is hemodynamically stable, and an ultrasound is requested to confirm the presumed
diagnosis of an ectopic pregnancy. Which of the following is the most common location for an ectopic
pregnancy?
A. Cervix
B. Ovary
C. Abdominal cavity
D. Ampullary portion of the fallopian tube
E. Interstitial portion of the fallopian tube
Source: Currie, Stuart, et al. SBAs for the FRCR 2A. 1st ed., Cambridge University Press, 2010.
Explanation:
• The most common site of implantation is the fallopian tube, which accounts for over 90% of ectopic
pregnancies.
• Ovarian and abdominal sites account for only approximately 3% and 1%, respectively.
• Within the fallopian tube the most common site is the ampulla (73%) followed by the fimbrial and
interstitial regions.
65. A 24 year old man is referred for an ultrasound examination following blunt trauma to the scrotum. Which
of the following is not true?
A. The left testis is more susceptible to blunt trauma
B. Intratesticular hematomas need to be followed up until resolution
C. Penetrating injuries are more likely to be bilateral compared to blunt injuries
D. An ultrasound finding of an intact tunica albuginea allows the confident exclusion of a testicular
rupture in the absence of a hematocoele
E. An atrophic testis is more likely to dislocate
EBDR Exam MCQs & Concepts May 2023
Dr. Kareem Alnakeeb
171
Source: Currie, Stuart, et al. SBAs for the FRCR 2A. 1st ed., Cambridge University Press, 2010.
Explanation:
• The testis suffers blunt trauma against the thigh or the symphysis pubis and the right testis, being
higher, is more susceptible.
• Intratesticular haematomas should be followed up to resolution to rule out an underlying neoplasm
and also rule out any ensuing complications such as abscess formation which may necessitate
orchidectomy.
66. A 36 year old man suffers pelvic fracture following a road traffic accident. On examination, blood is noted
at the urethral meatus and the patient has urinary retention. Regarding urothelial injuries:
A. Associated bladder injuries are seen in 50% of patients
B. Anterior urethral injuries are commoner with pelvic fractures
C. They are more commonly associated with pelvic fractures in females rather than males
D. Posterior urethral injuries can be seen in up to 20% of pelvic fractures in males
E. Impotence is a rare complication of male urethral injury
Source: Currie, Stuart, et al. SBAs for the FRCR 2A. 1st ed., Cambridge University Press, 2010.
Explanation:
• Urethral injuries are seen in up to 20% of male patients following pelvic fractures.
• They are much less common in women.
• The posterior urethra is the commonest site; impotence can develop in up to 40% of these patients.
67. A 40 year old female presents with bitemporal hemianopia. CT brain shows a large, slightly hyperdense
suprasellar lesion. The mass contains several lucent foci and there is bone erosion of the sella floor.
There is enhancement post-contrast. T1-weighted MR imaging shows a predominantly isointense mass
causing sella expansion and compression of the optic chiasm. The mass contains foci of low and high
signal intensity. What is the most likely diagnosis?
A. Craniopharyngioma
B. Meningioma
C. Rathke’s cleft cyst
D. Giant internal carotid aneurysm
E. Pituitary adenoma
Source: Currie, Stuart, et al. SBAs for the FRCR 2A. 1st ed., Cambridge University Press, 2010.
Explanation:
Pituitary Adenoma Classification
• Pituitary adenomas are divided into microadenomas (<1 cm) and macroadenomas (>1 cm).
• Macroadenomas may present with endocrine dysfunction but are generally less active than
microadenomas.
Clinical Presentation of Macroadenomas
• Macroadenomas often present with symptoms of mass effect on the optic chiasm.
• If there is lateral extension into the cavernous sinuses patients may present with other local cranial
nerve palsies (III, IV, VI).
Differential Diagnosis of a Suprasellar Mass (‘SATCHMO’)
• The differential diagnosis of a suprasellar mass includes: Suprasellar extension of pituitary
adenoma/sarcoid; Aneurysm/arachnoid cyst; TB/teratoma (other germ-cell tumours);
Craniopharyngioma; Hypothalamic glioma or hamartoma; Meningioma/metastases (especially
breast); and Optic/chiasmatic glioma.
Imaging Findings in This Case
EBDR Exam MCQs & Concepts May 2023
Dr. Kareem Alnakeeb
172
• In this case, the sellar is widened and the floor is eroded suggesting the mass arises from the
pituitary itself.
• Low-density/low-intensity regions on CT/T1 MRI correspond to necrotic areas and high-signal foci on
T1 MRI (found relatively frequently) represent areas of recent haemorrhage.
68. A 49 year old African male presents to the outpatient urology clinic with a five-month history of
macroscopic hematuria. A plain KUB X-ray is requested, which reveals thin arcuate calcification outlining
the bladder and the distal ureters. Which one of the following causes is most likely?
A. Transitional cell carcinoma
B. Squamous cell carcinoma
C. Schistosomiasis
D. E. coli cystitis
E. Proteus cystitis
Source: Currie, Stuart, et al. SBAs for the FRCR 2A. 1st ed., Cambridge University Press, 2010.
Explanation:
• The differential for bladder calcification includes tuberculosis, post-radiotherapy cystitis, urachal
carcinoma, TCC, and squamous cell carcinoma.
• However, schistosomiasis is the commonest cause, especially in the African population, where it is
often endemic.
• The bladder is usually a normal size and shape, with thin curvilinear calcifications.
• Ureteric strictures, inflammatory pseudopolyps and vesicoureteric reflux are seen in addition to
bladder and ureteric calcification.
69. Triple rule out MDCT exam is used for diagnosis of
A. Pulmonary embolism
B. Coronary artery stenosis
C. Aortic dissection
D. Aortic aneurysmal dilatation
E. All of the above
Explanation: (by Perplexity AI, generated with the OpenAI O3 model)
Triple-rule-out (TRO) multidetector CT
• A TRO MDCT scan is designed to evaluate three potentially fatal causes of acute chest pain in one
contrast-enhanced acquisition:
o pulmonary embolism in the pulmonary arteries
o coronary artery stenosis (acute coronary syndrome) in the coronary arteries
o aortic pathology—most importantly acute aortic dissection, but the same acquisition also
shows aortic aneurysmal dilatation
• Because the single scan simultaneously interrogates all three vascular territories, it is intended to
“rule out” all of them at once.
• Aortic aneurysmal dilatation (choice D) is usually evident on the same dataset, but TRO is
conceptually aimed at acute life-threatening causes of chest pain; aneurysmal dilatation alone is
not an acute emergency and is not one of the three traditional targets.
• Therefore the correct option is: E. All of the above – but only because the scan inevitably depicts an
aneurysm if present, even though the classic “triple” refers to embolism, dissection, and coronary
stenosis.
EBDR Exam MCQs & Concepts May 2023
Dr. Kareem Alnakeeb
173
70. MRI Steady State Free Precession sequence (SSFP) is used for:
A. Evaluation of left ventricular function
B. Evaluation of night ventricular function
C. Diagnosis of myocardial wall hypokinesia
D. None of the above
E. All of the above
Explanation: (by Perplexity AI, generated with the OpenAI O3 model)
• Balanced steady-state free precession (SSFP) cine sequences are the routine “work-horse” of
cardiac MRI because they deliver bright-blood images with high temporal resolution. They are
therefore simultaneously used for
o quantitative left ventricular (LV) functional analysis (volumes, mass, ejection fraction) ,
o quantitative right ventricular (RV) functional analysis (volumes, ejection fraction) , and
o visual identification of regional wall-motion abnormalities such as hypokinesia, akinesia or
dyskinesia
71. Anomalous right upper pulmonary vein draining within into SVC Is associated with:
A. Patent foramen oval
B. ASD (septum primum Type)
C. VID
D. ASD (sinus venosus type)
E. None of the above
Explanation: (by Perplexity AI, generated with the OpenAI O3 model)
• Partial anomalous pulmonary venous return in which the right upper (or right superior) pulmonary
vein drains into the superior vena cava is classically linked to a superior (high) sinus venosus atrial
septal defect.
• Case series and reviews report that approximately 80%–90% of these PAPVR cases coexist with a
sinus venosus ASD, whereas associations with patent foramen ovale, septum primum ASD, or
ventricular septal defect are uncommon or incidental.
72. Transposition of great arteries can be defined as
A. AV concordance
B. VA concordance
C. VA discordance
D. AV discordance
E. None of the above
Explanation: (by Perplexity AI, generated with the OpenAI O3 model)
Definition of Transposition of the Great Arteries (TGA)
• Transposition of the great arteries is defined by ventriculo-arterial (VA) discordance—the aorta
arises from the morphologic right ventricle and the pulmonary artery arises from the morphologic left
ventricle, creating parallel rather than serial circulations.
• (The atrioventricular [AV] connection is usually normal, so the hallmark lesion is discordance
between ventricles and great arteries.)
EBDR Exam MCQs & Concepts May 2023
Dr. Kareem Alnakeeb
174
Transposition of the Great Arteries: Segmental Anatomy Cheat-Sheet
Variant Atrioventricular (AV)
Connection
Ventriculo-Arterial
(VA) Connection
Circulatory Physiology
D-TGA
(“complete” or
dextro-TGA)
Concordance (RA→RV,
LA→LV)
Discordance
(RV→Aorta, LV→PA)
Parallel circuits → cyanosis at
birth; mixing needs PFO/ASD,
VSD, or PDA
L-TGA
(“congenitally
corrected” or levo-
TGA)
Discordance (RA→LV,
LA→RV)
Discordance
(LV→PA, RV→Aorta)
“Physiologically corrected” series
flow; systemic work done by RV
Key points to remember
• Transposition is defined by VA discordance—the aorta springs from the morphologic right ventricle
and the pulmonary artery from the morphologic left ventricle.
• D-TGA = VA discordance + AV concordance. A patent ductus arteriosus (PDA) or other shunt is often
life-saving until surgery.
• L-TGA = VA discordance + AV discordance (“double discordance”). Because the blood crosses two
wrong connections, overall flow is in the correct direction, hence “congenitally corrected.”
73. Non-Transmural myocardial infarction involves
A. <25% transmurality
B. 50% transmurality
C. <75% transmurality
D. 90% transmurality
E. None of the above
Explanation: (by Perplexity AI, generated with the OpenAI O3 model)
Non-transmural (sub-endocardial) myocardial infarction
• By definition, a non-transmural MI is confined to the inner layers of the ventricular wall and does not
extend through the full thickness of the myocardium.
• In standard pathology and imaging texts, the cut-off generally used is less than 50% of the wall
thickness.
• Infarcts that involve ≥50% of the wall are classified as transmural.
74. Left ventricle non-compaction is best defined as:
A. Spongy left ventricle
B. Endomyocardial with prominent trabeculation
C. Pseudo-thickening of the lateral wall
D. Unclassified congenital cardiomyopathy
E. All of the above
Explanation: (by Perplexity AI, generated with the OpenAI O3 model)
Left-ventricular non-compaction (LVNC) is typically described with several overlapping phrases:
• Spongy left ventricle – reflects the embryonic, sponge-like myocardial meshwork that fails to
compact.
• Endomyocardium with prominent trabeculation and deep recesses – the hallmark imaging and
pathologic criterion.
EBDR Exam MCQs & Concepts May 2023
Dr. Kareem Alnakeeb
175
• Pseudo-thickening of the lateral wall – on echocardiography or MRI the heavily trabeculated inner
layer can mimic wall hypertrophy, producing apparent thickening.
• Unclassified (or non-classified) congenital cardiomyopathy – listed as such in the European
Society of Cardiology scheme; the American Heart Association places LVNC among genetic
cardiomyopathies.
Because each option represents a commonly used component of the standard definition, the most inclusive
and therefore correct choice is “All of the above.”
75. Black blood Imaging is
A. Used for assessment of cardiac anatomy
B. Used in cardiac CINE imaging
C. Used as a cardiac fat suppression technique
D. None of the above
E. All of the above
Explanation: (by Perplexity AI, generated with the OpenAI O3 model)
Black-blood cardiac MRI: typical uses
Black-blood sequences (most commonly double-inversion recovery fast spin-echo) null the signal from
flowing blood so that the blood pool appears dark while the myocardium remains bright. They are employed
for three principal purposes:
1. Anatomic delineation – because the dark lumen sharply outlines the ventricular walls, valves, great
vessels, and pericardium, black-blood imaging is the work-horse sequence for static assessment of
cardiac anatomy and morphology.
2. Tissue characterization – varying the echo time produces T1- or T2-weighted black-blood images
that highlight edema, fibrosis, or infiltration; fat-suppressed variants such as STIR combine blood
nulling with fat suppression for detecting myocardial edema or masses.
3. Not for cine function – dynamic (cine) evaluation of ventricular motion is instead performed with
bright-blood/SSFP sequences; black-blood techniques are inherently single-shot or segmented still
images and are not used for routine cine acquisition.
They are not a dedicated “cardiac fat-suppression technique” (although fat suppression can be added) and
are unnecessary for standard cine imaging.
76. White blood imaging is
A. Used for assessment of cardiac anatomy
B. Used in cardiac CINE imaging
C. Used as a cardiac fat suppression technique
D. None of the above
E. All of the above
Explanation: (by Perplexity AI, generated with the OpenAI O3 model)
White-blood (bright-blood) cardiac MRI
“White-blood” sequences—principally gradient-echo and steady-state free-precession (SSFP) cine imaging—
render fast-flowing blood bright, allowing clear visualization of intracardiac cavities throughout the cardiac
cycle.
Key applications
• Cine functional imaging: SSFP is the gold-standard for quantifying ventricular volumes, ejection
fraction, wall motion, and valve dynamics because its high temporal resolution captures motion in
real time.
EBDR Exam MCQs & Concepts May 2023
Dr. Kareem Alnakeeb
176
• Anatomic survey: Although cines do depict chamber and valvular anatomy, static high-resolution
anatomic detail is usually obtained with black-blood spin-echo; “white-blood” itself is not the
primary static morphologic sequence.
• Not a fat-suppression technique: Fat suppression can be combined with various sequences, but
“white-blood” refers to bright-blood contrast, not to selective nulling of fat signal.
Answer
B. Used in cardiac CINE imaging
White-blood imaging’s defining role is dynamic (cine) assessment of cardiac function; it is not chiefly a fat-
suppression method, and static anatomic assessment is better served by black-blood imaging.
77. Cardiac sarcoidosis involves
A. Patchy enhancement of LV myocardium
B. Preferential involvement to the base of the heart
C. Bad prognostic value
D. All of the above
E. None of the above
Explanation: (by Perplexity AI, generated with the OpenAI O3 model)
Cardiac sarcoidosis on imaging and prognosis
Cardiac sarcoidosis typically shows three characteristic features:
• Patchy late-gadolinium enhancement (LGE) of the left-ventricular myocardium, often mid-wall or
sub-epicardial rather than sub-endocardial.
• Predominant involvement of the basal segments—especially the basal interventricular septum
and the ventricular free wall—on MRI and autopsy studies.
• Adverse prognosis compared with extracardiac sarcoidosis, with higher risks of arrhythmia, heart
failure and sudden cardiac death; cardiac involvement is therefore regarded as a negative prognostic
marker.
78. Regarding cardiac amyloidosis
A. Patchy enhancement of LV myocardium is typical
B. Global epicardial enhancement is characteristic
C. Subendocardial enhancement is the rule
D. All of the above
E. None of the above
Explanation: (by Perplexity AI, generated with the OpenAI O3 model)
Cardiac amyloidosis: late-gadolinium enhancement (LGE) patterns
Cardiac amyloid deposits expand the interstitium, producing circumferential subendocardial or diffuse
transmural enhancement on cardiac MRI:
• Multiple series identify a global or circumferential subendocardial LGE pattern as the
characteristic and most common appearance, especially in light-chain (AL) amyloidosis.
• More extensive transmural (epicardium-to-endocardium) LGE can develop in advanced disease—
seen more often in transthyretin (ATTR) amyloidosis—but pure epicardial-predominant
enhancement is not typical.
• Patchy mid-myocardial foci are uncommon and, when present, suggest alternative diagnoses such
as sarcoidosis rather than amyloid.
EBDR Exam MCQs & Concepts May 2023
Dr. Kareem Alnakeeb
177
79. A 76-year-old male patient with chronic inflammatory disease and known history of secondary
generalized multisystem amyloidosis showed an abnormal appearance of the heart on
echocardiography. Dynamic enhanced cardiac MR imaging was advised for further characterization. All
of the following are imaging findings seen with cardiac amyloidosis, except
A. Left ventricular wall hypertrophy
B. Subendocardial delayed myocardial hyperenhancement
C. Systolic dysfunction
D. Granular echogenic myocardium
E. Interatrial septal thickening
Source: Chin, Teck Yew, et al. Get Through Final FRCR 2A: SBAs. 1st ed., CRC Press, 2017.
Explanation:
Pathophysiology
• In cardiac amyloidosis, the amyloid protein is deposited in the myocardium, which leads to diastolic
dysfunction that progresses to restrictive cardiomyopathy.
• Because amyloidosis is a systemic process, involvement of all four chambers is common; thus, an
increase in the thickness of the interatrial septum and right atrial free wall by more than 6 mm has
been shown to be a specific finding for cardiac amyloidosis.
Imaging Findings
• Through the use of dynamic enhanced cardiac MRI, a distinct pattern of late enhancement, which
was distributed over the entire subendocardial circumference, has been shown to have high
specificity and sensitivity for cardiac amyloidosis.
• Echocardiogram shows concentric LV hypertrophy, with hyperechoic granular sparkling of the
ventricular wall.
80. Macleod's syndrome:
A. Affects the small bronchi
B. Bronchiectasis is seen
C. Submucosal fibrosis from 4 generation bronchus
D. Pulmonary artery branches are hypoplastic
E. All of the above
Explanation: (by Perplexity AI, generated with the OpenAI O3 model)
MacLeod’s syndrome (Swyer–James–MacLeod)
This acquired post-infectious lung disorder shows a triad of pathologic changes that explain its imaging
features:
1. Bronchiolar injury from bronchiolitis obliterans → fibrosis, narrowing and even obliteration of the
small (membranous and respiratory) bronchioles, beginning about the fourth-generation airway and
beyond.
2. Secondary bronchiectasis—cylindrical or saccular dilatation of affected bronchi is common,
though not universal.
3. Marked reduction in calibre and number of ipsilateral pulmonary-artery branches, sometimes
frank hypoplasia or agenesis, leading to the classic unilateral hyperlucent, hypoperfused lung on
radiographs and CT.
EBDR Exam MCQs & Concepts May 2023
Dr. Kareem Alnakeeb
178
81. In patients with rheumatoid arthritis, what is the commonest pulmonary finding seen on the chest
radiograph?
A. pleural effusion
B. fibrosis
C. pulmonary nodules
D. bronchiectasis
E. heart failure
Source: Bydder, Megan, et al. Get Through Final FRCR Part A: SBAs for the Modular Examination. 1st
ed., Hodder Education Publishers, 2009.
Explanation:
Pulmonary Manifestations of Rheumatoid Arthritis
• Between 2% and 54% of patients with rheumatoid arthritis have pulmonary abnormalities.
• Pleural abnormalities are most frequent, being either an effusion (unilateral in 92% of cases) or
pleural thickening (usually bilateral).
• Fibrosis occurs in 30% of patients with pulmonary involvement.
• Nodules are unusual and seen in advanced disease.
• They are usually peripheral and may cavitate.
• Bronchial abnormalities are seen in 30% of patients with rheumatoid lung, and include
bronchiectasis and bronchiolitis obliterans.
• Other findings include pulmonary arterial hypertension and heart failure secondary to
carditis/pericarditis.
82. A 56 year old female is found to have a small, well-defined anterior mediastinal mass on a chest
radiograph which demonstrates homogeneous soft-tissue density with some peripheral calcification on
CT. On MRI it is isointense to skeletal muscle on T1-weighted images and slightly increased signal on T2-
weighted images. It is most likely to be:
A. Thymic cyst
B. Thymoma
C. Thymolipoma
D. Thymic hyperplasia
E. Thymic carcinoma
Source: Currie, Stuart, et al. SBAs for the FRCR 2A. 1st ed., Cambridge University Press, 2010.
Explanation:
• This case describes the typical features of a thymoma. Thymic hyperplasia and thymic carcinoma
are usually ill-defined abnormalities. The signal from the lesion is not typical for a thymic cyst or
thymolipoma.
EBDR Exam MCQs & Concepts May 2023
Dr. Kareem Alnakeeb
179
83. A 40-year-old schoolteacher presented with a non-productive cough, dyspnea and low-grade pyrexia.
She has never smoked. The CXR demonstrated several bilateral areas of patchy consolidation which were
confirmed on HRCT and shown to be in a mainly sub-pleural distribution. In addition, there are also
patchy ground-glass change and small (<5 mm) centrilobular nodules). What is the most likely diagnosis?
A. Bronchoalveolar cell carcinoma
B. Histoplasmosis
C. Sarcoidosis
D. Cryptogenic organizing pneumonia
E. Multifocal streptococcal pneumonia
Source: Proctor, Robin. Final FRCR Part A Modules 1–3 Single Best Answer MCQs: The SRT Collection
of 600 Questions with Explanatory Answers (MasterPass). 1st ed., CRC Press, 2009.
Explanation:
• These imaging findings are classical of COP. Effusions and adenopathy are also present in up to one
third of patients.
• Bronchoalveolar cell carcinoma is an important differential, but is most commonly solitary and
centrilobular nodules are not a feature, and there is a strong smoking association.
84. A 32 year old male front seat passenger is involved in a road traffic accident and sustains blunt
abdominal trauma. He is admitted via the emergency department and CT reveals a splenic laceration
with subcapsular haematoma. Which one of the following associated injuries is most likely to be found?
A. Diaphragmatic rupture
B. Injury to the liver
C. Injury to the left kidney
D. Ipsilateral rib fractures
E. Injury to the small bowel mesentery
Source: Currie, Stuart, et al. SBAs for the FRCR 2A. 1st ed., Cambridge University Press, 2010.
Explanation:
• All are potential associated injuries and should be actively searched for in the context of blunt
abdominal trauma.
• Rib fractures are found in up to 50% of patients with splenic injuries and as such are the most
common association.
• The left kidney is injured in 10% of patients with splenic injury, and diaphragm rupture is even rarer.
• Diaphragm rupture may be difficult to appreciate on axial slices, and may be more evident on coronal
reformats.
85. An 83-year-old man with a history of bladder cancer and myocardial infarction was referred for
radiofrequency ablation (RFA) because of his co-morbidities. All of the following are true regarding RFA,
except
A. RFA uses high frequency alternating current to generate heat and high temperature to cause cell
death.
B. Cell death occurs by denaturation of proteins (coagulative necrosis).
C. The tip of the electrode is placed in the centre of the lesion.
D. The heat sink effect results in a poor outcome in larger lesions.
E. Cell death starts at 49 degrees.
Source: Chin, Teck Yew, et al. Get Through Final FRCR 2A: SBAs. 1st ed., CRC Press, 2017.
Explanation:
Radiofrequency Ablation Fundamentals
• In RFA, a high-frequency, alternating current with a wavelength of 460–500 kHz is emitted through an
electrode placed within the targeted tissue.
EBDR Exam MCQs & Concepts May 2023
Dr. Kareem Alnakeeb
180
• Grounding pads applied to the patient’s thighs complete the electrical circuit.
• Cell death starts at 49 degrees.
• Temperature above 60 degrees causes immediate cell death, and tissue charring occurs at 105
degrees.
• Cell death is induced by the denaturation of proteins.
Percutaneous Imaging-Guided Technique
• For percutaneous imaging-guided RFA, the energy is delivered into the target tissue by means of
needle-like electrodes.
• Unlike in a typical biopsy, the electrode tip should be advanced to the deep margin of the tumour.
Imaging Follow-Up and Residual Disease
• On follow-up CT, ablated tumours often have internal areas of increased attenuation or increased
signal intensity at CT and MRI.
• Areas of contrast enhancement (>10 HU or >15% with CT and MRI, respectively) are indicative of
residual viable RCC.
• Residual viable tumour can be treated with additional ablation sessions.
Heat-Sink Phenomenon
• Heat-sink phenomena refers to the reduction in tissue temperature due to the conductive effects of
adjacent vessels or airways.
• It is an explanation for distortion of the ablation zone and poor outcome in larger lesions.
• The heat-sink effect can be overcome by pharmacologically reducing blood flow, intra-arterial
embolisation, intravascular balloon occlusion, Pringle manoeuvre or reducing treatment zone.
86. A 45 year old woman with severe portal hypertension and variceal bleeding is referred for a trans-jugular
intrahepatic porto-systemic shunt (TIPSS) procedure following the failure of endoscopic procedures in
controlling the bleeding. Which of the following is the most appropriate regarding TIPSS?
A. The middle hepatic vein is the preferred route of access to the portal vein
B. The right portal vein is usually posterior to the right hepatic vein
C. Flow of contrast towards the porta hepatis usually indicates puncture of the biliary tree
D. The gradient across the shunt should be less than 20mmHg
E. Stenosis tends to occur in the portal vein
Source: Currie, Stuart, et al. SBAs for the FRCR 2A. 1st ed., Cambridge University Press, 2010.
Explanation:
• Usually the right hepatic vein (RHV) is the preferred route of access to the right portal vein, which lies
anterior to the RHV.
• Flow of contrast towards the porta, and especially if it remains there, usually indicates biliary
puncture.
• Puncture of portal vein and hepatic artery usually result in contrast flowing to the periphery.
• The shunt gradient should be less than 12mm of mercury.
• Stenoses usually tend to occur in the hepatic vein or the shunt itself.
EBDR Exam MCQs & Concepts May 2023
Dr. Kareem Alnakeeb
181
87. A 36 year old female with history of pelvic pain and severe dysmenorrhea undergoes a pelvic ultrasound
examination which reveals uterine fibroid disease. Which of the following imaging features would be
associated with the best outcome following uterine artery embolization?
A. Submucosal location
B. Subserosal location
C. Associated adenomyosis
D. Calcification
E. Multiple fibroids
Source: Currie, Stuart, et al. SBAs for the FRCR 2A. 1st ed., Cambridge University Press, 2010.
Explanation:
• Subserosal fibroids, especially pedunculated ones, may often draw their blood supply from adjacent
viscera, which may be a cause of failure of the procedure. They are also associated with a higher
incidence of complications.
• Calcific fibroids are less vascular and may not respond well to embolisation.
• Bulky and multiple fibroids may need multiple interventions or surgery.
• Adenomyosis is a known cause for failure of the procedure.
88. A 60 year old man who recently suffered a hemorrhagic stroke develops pulmonary emboli. He is referred
for an IVC filter insertion and angiography is performed prior to this. The usual reasons for doing so would
be all of the following except:
A. To identify the renal veins
B. To identify the hepatic veins
C. To size the IVC
D. To rule out the presence of a left IVC
E. To evaluate for the presence of an IVC thrombus
Source: Currie, Stuart, et al. SBAs for the FRCR 2A. 1st ed., Cambridge University Press, 2010.
Explanation:
• The hepatic veins do not need to be identified routinely prior to filter insertion.
• Most filters are deployed in an infrarenal position, unless there is IVC thrombus which would
preclude this, in which case the filter is positioned in the suprarenal position.
• A left iliac injection is performed to rule out a left IVC, which could be a cause of filter failure.
89. While reporting plain films in a paediatric radiology setting, you look at the plain abdominal film of an 8-
year-old child admitted with abdominal pain and vomiting. The film demonstrates multiple gas-filled
bowel loops throughout the abdomen, with paucity of gas and a small calcific density in right lower
quadrant. The right psoas outline is not clear. What is the likely diagnosis?
A. Acute appendicitis
B. Acute cholecystitis
C. Renal colic
D. Abdominal teratoma
E. Chronic peritonitis
Source: Chin, Teck Yew, et al. Get Through Final FRCR 2A: SBAs. 1st ed., CRC Press, 2017.
Explanation:
Acute Appendicitis
• The plain film findings and clinical history are suggestive of acute appendicitis, which is the most
common reason for abdominal surgery in children.
• It is one of the most common causes of intestinal obstruction in children.
EBDR Exam MCQs & Concepts May 2023
Dr. Kareem Alnakeeb
182
• Other causes include adhesions, intussusception, incarcerated inguinal hernia, malrotation with
volvulus and Meckel’s diverticulum.
• On plain films, a fecolith or appendicolith with small bowel obstruction (seen in 10% of cases) and
displacement of bowel gas from the right iliac fossa are all typical signs of acute appendicitis.
• However, an abdominal US examination is the preferred imaging modality of choice for investigation
of acute appendicitis in children rather than abdominal radiograph.
Imaging Modalities
• The primary imaging modality in suspected acute appendicitis remains controversial.
• Several authors advocate plain films, US and CT as primary diagnostic sets, with various arguments.
Other Pediatric Abdominal Conditions
• Acute cholecystitis and renal colic are not common diagnoses in children.
• Intussusception most often occurs between 3 months and 1 year of age.
90. A 66-year-old joiner presents to his GP with jaundice and abdominal discomfort. He was subsequently
referred to a gastroenterologist who requests a liver biopsy due to deranged liver function tests. Which of
the following options is not a contraindication for percutaneous liver biopsy?
A. INR above 1.6
B. Platelets less than 60,000/mm3
C. Tense ascites
D. Extra-hepatic biliary obstruction
E. Suspected hemangioma
Source: Chin, Teck Yew, et al. Get Through Final FRCR 2A: SBAs. 1st ed., CRC Press, 2017.
Explanation:
Contraindications for liver biopsy include the following:
A. Uncooperative patient
B. Extrahepatic biliary duct dilatation (except if benefit outweighs the risk)
C. Bacterial cholangitis (relative contraindication due to risk of septic shock)
D. Abnormal coagulation indices (having a normal INR or PT is not a reassurance that the patient
will not bleed; however, there is increased incidence of bleeding with INR above 1.5)
E. Thrombocytopenia (platelet count below 60,000/mm3)
91. A 27-year-old female patient undergoes urgent neuroimaging following loss of consciousness as a result
of an RTA. CT is unremarkable. MRI reveals multiple small areas of increased signal on T2WI in the white
matter near the grey–white matter junction within the frontal and temporal lobes. In the same locations,
DWI reveals areas of increased signal on the B1000 image and reduced signal on the ADC map. What is
the most likely diagnosis?
A. Subarachnoid haemorrhage.
B. Extradural haematoma.
C. Subdural haematoma.
D. Hypoxic brain injury.
E. Diffuse axonal injury.
Source: Lindsay, Richard, et al. SBAs for the FRCR Part 2A. 1st ed., Oxford University Press, 2012.
Explanation:
Computed Tomography (CT)
• CT is initially often normal (up to 80% of cases) in DAI.
EBDR Exam MCQs & Concepts May 2023
Dr. Kareem Alnakeeb
183
• If positive, it may reveal small low attenuation foci (oedema) or high attenuation foci of petechial
haemorrhage.
• The gray/white matter interface of the frontotemporal lobes, corpus callosum (especially the
splenium), and brainstem are the most commonly involved sites in DAI.
Magnetic Resonance Imaging (MRI)
• MRI is much more sensitive and is the investigation of choice.
• The signal on MRI depends on the age of the lesion and whether haemorrhage is present, but
classically hyperintense foci on T2WI sequences are seen acutely.
• In the more chronic phase, the lesions may only be detected as hypointense foci at characteristic
locations on GE sequences: this appearance may remain for years.
• DWI reveals hyperintense foci of restricted diffusion on B1000 images, with corresponding low signal
on the ADC map.
Differential Findings
• The findings on DWI are easily distinguishable from extradural haematoma/subarachnoid
haemorrhage/subdural haematoma/generalized oedema, which are discussed in other questions in
this chapter.
92. A 69-year-old lady was admitted 10 days ago following an acute intracerebral haematoma diagnosed on
CT. What are the most likely radiological findings on the follow-up MRI scan of brain?
A. Haematoma hypointense to grey matter on T1WI, hyperintense on T2WI.
B. Haematoma hyperintense to grey matter on both T1WI and T2WI.
C. Haematoma hyperintense to grey matter on T1WI, hypointense on T2WI.
D. Haematoma hypointense to grey matter on both T1WI and T2WI.
E. Haematoma isointense to grey matter on both T1WI and T2WI.
Source: Lindsay, Richard, et al. SBAs for the FRCR Part 2A. 1st ed., Oxford University Press, 2012.
Explanation:
• The MRI appearances of intracranial haemorrhage are determined primarily by the state of the
haemoglobin (Hb), which evolves with age. This can be staged as hyperacute (first few hours), acute
(1–3 days), early subacute (3–7 days), late subacute (4–7 days to 1 month), or chronic (1 month to
years).
Table 6.1 The sequential signal intensity changes of the evolving intracerebral haematoma
Phase State of Hb Magnetic properties T1 signal intensity T2 signal intensity
Hyperacute Intracellular oxy-Hb Diamagnetic ↔ / ↓ ↑
Acute Intracellular deoxy-Hb Paramagnetic ↔ / ↓ ↓
Early subacute Intracellular met-Hb Paramagnetic ↑↑ ↓↓
Late subacute Extracellular met-Hb Paramagnetic ↑↑ ↑↑
Chronic Haemosiderin Superparamagnetic ↔ / ↓ ↓↓
EBDR Exam MCQs & Concepts May 2023
Dr. Kareem Alnakeeb
184
93. A 52-year-old male presents with dyspnea and cough. A chest radiograph shows an ill-defined opacity in
the right mid-zone, obscuring the heart border. A lateral view shows a thin wedge-shaped opacity with
base in contact with the pleura anteroinferiorly, and pointing posterosuperiorly. What is the most likely
diagnosis?
A. right middle lobe collapse
B. right middle lobe consolidation
C. right lower lobe collapse
D. right lower lobe consolidation
E. encysted pleural fluid
Source: Bydder, Megan, et al. Get Through Final FRCR Part A: SBAs for the Modular Examination. 1st
ed., Hodder Education Publishers, 2009.
Explanation:
Radiographic Findings in Right Middle and Lower Lobe Pathology
• In right middle lobe collapse, the horizontal fissure and lower half of the oblique fissure converge.
• This creates a wedge-shaped opacity on the lateral chest radiograph.
• On the frontal chest radiograph, there is an ill-defined mid-zone opacity.
• With right middle lobe consolidation, there is a mid-zone opacity with a well-defined superior margin,
as the horizontal fissure remains in a normal position and is tangential to the radiograph beam.
• Both obscure the right heart border.
• Lower lobe collapse and consolidation cause basal opacity with loss of clarity of the right
hemidiaphragm.
• The lateral view shows a triangular opacity at the right base posteriorly, larger in consolidation than
collapse.
94. A 20-year-old female with a history of neurofibromatosis presents with reduced visual acuity in the right
eye. She subsequently has CT and MR imaging of the orbits to assess for a tumour relating to the right
optic nerve. Which of the following findings on imaging would be more suggestive of the presence of an
optic nerve glioma, rather than a meningioma arising from the optic nerve sheath?
A. Presence of the ‘tram-track’ sign.
B. Presence of optic canal widening.
C. Presence of marked intense tumour enhancement.
D. Presence of calcification.
E. Presence of bony hyperostosis.
Source: Lindsay, Richard, et al. SBAs for the FRCR Part 2A. 1st ed., Oxford University Press, 2012.
Explanation:
Optic Nerve Canal & Lesion Prevalence
• The presence of a widened optic nerve canal occurs in up to 90% of cases of optic nerve glioma.
• While it can also occur in meningioma, it is more common in glioma and some cases of meningioma
may even have a narrowed canal secondary to bony hyperostosis.
Imaging Characteristics
• The ‘tram-track’ sign is typically associated with meningioma and refers to the more avidly enhancing
meningioma surrounding the non-enhancing optic nerve on axial CT and MR imaging of the orbit.
• Although both meningioma and glioma enhance following intravenous contrast, it is meningioma
that is more typically associated with marked intense enhancement.
Calcification & Bony Changes
• Calcification and bony hyperostosis are features associated with meningioma.
• Calcification is rare in gliomas, unless they have previously undergone radiotherapy.
EBDR Exam MCQs & Concepts May 2023
Dr. Kareem Alnakeeb
185
95. A 45 year old woman undergoes investigation for conductive hearing loss. History reveals several
previous ear infections. Direct visualisation with an otoscope shows a mass behind an intact tympanic
membrane. Coronal CT imaging demonstrates a soft-tissue mass located between the lateral attic wall
and the head of the malleus. There is blunting of the scutum. The mass does not enhance post-contrast.
What is the most likely diagnosis?
A. Chronic otitis media
B. Cholesterol granuloma
C. Cholesteatoma
D. Rhabdomyosarcoma
E. Squamous cell carcinoma
Source: Currie, Stuart, et al. SBAs for the FRCR 2A. 1st ed., Cambridge University Press, 2010.
Explanation:
Definition
• A cholesteatoma consists of a sac lined with stratified squamous epithelium and filled with keratin –
essentially ‘skin growing in the wrong place’.
Types and Origin
• They can be acquired (98%) or congenital (2%).
• Most acquired cholesteatomas arise in the superior portion of the tympanic membrane (pars
flaccida) and extend into Prussak’s space where they can cause medial displacement of the head of
the malleus and erosion of the bony scutum.
Imaging Characteristics
• The characteristic imaging feature of a cholesteatoma is bone erosion associated with a non-
enhancing soft-tissue mass.
Complications
• Intratemporal: ossicular destruction, facial nerve paralysis, labyrinthine fistula, complete hearing
loss, automastoidectomy.
• Intracranial: meningitis, sinus thrombosis, abscess, CSF rhinorrhea.
96. You are reporting a CT scan of neck in a patient with a head and neck cancer. You see an enlarged
necrotic jugulo-digastric lymph node on the right side and wish to describe the appropriate level of this
lymph node in your report. What is the correct level?
A. I.
B. II.
C. III.
D. IV.
E. V.
F. VI.
G. VII.
Source: Lindsay, Richard, et al. SBAs for the FRCR Part 2A. 1st ed., Oxford University Press, 2012.
Explanation:
Overview of Neck Lymph-Node Levels
• Lymph nodes in the neck have been divided into seven levels, generally for the purpose of squamous
cell carcinoma staging.
• This is, however, not all inclusive, as the parotid nodes and retropharyngeal space nodes are not
included in this system.
• Lymph node levels of the neck. Radiopaedia.org.
EBDR Exam MCQs & Concepts May 2023
Dr. Kareem Alnakeeb
186
Level I: Below mylohyoid to hyoid bone anteriorly.
• Level Ia: Submental.
• Level Ib: Submandibular.
Level II: Jugulodigastric (base of skull to hyoid).
Level III: Deep cervical (hyoid to cricoid).
Level IV: Virchow (cricoid to clavicle).
Level V: Posterior triangle groups.
• Level Va: Accessory spinal (posterior triangle), superior half.
• Level Vb: Accessory spinal (posterior triangle), inferior half.
Level VI: Prelaryngeal/pretracheal/Delphian node.
Level VII: Superior mediastinal (between common carotid arteries (CCAs), below top of manubrium).
97. A 12-year-old boy is investigated via MRI brain for headache, nystagmus, and ataxia. Which of the
following radiological fi ndings would suggest a diagnosis of Chiari I malformation as opposed to Chiari
II?
A. Lacunar skull.
B. Myelomeningocoele.
C. Elongation of the fourth ventricle.
D. Caudal displacement of the cerebellar tonsils.
E. Cervicomedullary kinking.
Source: Lindsay, Richard, et al. SBAs for the FRCR Part 2A. 1st ed., Oxford University Press, 2012.
Explanation:
• Chiari II is seen in all patients with open spinal dysraphisms, such myelomeningocoele.
• Lacunar skull (luckenshadel) is also associated with Chiari II.
• Cervicomedullary kinking is common to both, although more so with Chiari II.
• Caudal displacement of the cerebellar tonsils is a feature of Chiari I, whereas in Chiari II the vermis
herniates into the foramen magnum and the tonsils are lateral to the medulla.
98. A 36 year old woman with known polycystic kidney disease presents with a history of sudden onset
headache and has signs of meningism. A CT brain reveals subarachnoid haemorrhage with haematoma
within the septum pallucidum. What is the most likely site for an intracerebral aneurysm?
A. Anterior communicating artery
B. Posterior communicating artery
C. A2 segment of an anterior cerebral artery
D. Tip of the basilar artery
E. Middle cerebral artery
Source: Currie, Stuart, et al. SBAs for the FRCR 2A. 1st ed., Cambridge University Press, 2010.
Explanation:
• A clot in the septum pallucidum is virtually diagnostic of an aneurysm of the anterior communicating
artery.
• Aneurysms of the distal anterior cerebral artery are less common.
EBDR Exam MCQs & Concepts May 2023
Dr. Kareem Alnakeeb
187
99. A 6-year-old girl is brought to your local paediatric outpatients with a history of night sweats, tiredness,
and new onset wheeze not responding to bronchodilators. A CXR is done which shows increased
mediastinal soft tissue noted superiorly. The paravertebral lines are maintained. The aortic knuckle is not
visible. A lateral CXR has been carried out at the request of the paediatrician, which shows increased soft
tissue displacing the trachea posteriorly, causing mild narrowing. What is the most likely diagnosis?
A. Tuberculosis.
B. Lymphangioma.
C. Bronchogenic cyst.
D. Thymic/nodal malignant infiltration.
E. Teratoma.
Source: Lindsay, Richard, et al. SBAs for the FRCR Part 2A. 1st ed., Oxford University Press, 2012.
Explanation:
Lesion Localization
• The first step in this question is to localize the lesion.
• The posterior displacement of the trachea indicates an anterior mediastinal position.
Differential Diagnosis of Anterior Mediastinal Masses in Children
• It is then necessary to consider the common causes of anterior mediastinal masses in children,
which are: normal thymus and thymic/nodal infiltration (leukaemia, lymphoma), with other causes
(lymphangioma and teratoma) being much less common.
• A normal thymus would not be expected to cause significant posterior displacement of the trachea.
• On plain film it would not be possible to differentiate thymic infiltration from anterior mediastinal
nodal infiltration.
• TB in children would be uncommon in the anterior mediastinum, especially when no abnormality is
noted in the hila.
100. A 24-year-old woman who is 28 weeks pregnant is admitted with suspected pulmonary embolism. As the
on-call radiologist, her obstetrician contacts you seeking advice regarding further management. An
admission CXR is normal. What investigation do you advise initially?
A. Venous ultrasound.
B. Low-dose CTPA.
C. Reduced dose lung scintigraphy.
D. MRA.
E. Catheter pulmonary angiography.
Source: Lindsay, Richard, et al. SBAs for the FRCR Part 2A. 1st ed., Oxford University Press, 2012.
Explanation:
Venous Ultrasound
• For pregnant patients, venous ultrasound is recommended before imaging tests with ionizing
radiation are performed.
• Up to 29% of pregnant patients with PE will have a positive venous ultrasound, obviating the need for
further imaging.
V/Q Scanning vs CTPA
• The majority of the PIOPED II investigators currently recommend V/Q scanning over CTPA in the
evaluation of PE in pregnant patients.
• The foetal dose with V/Q is similar to that with CTPA, although the effective dose per breast is much
greater with CTPA.
MRI and Contrast Agents
EBDR Exam MCQs & Concepts May 2023
Dr. Kareem Alnakeeb
188
• MRI requires further evaluation and gadolinium-based contrast agents have not been proven to be
safe in pregnancy.
Catheter Angiography
• The role of catheter angiography is probably limited to those patients requiring mechanical
thrombectomy.
Cumulative Radiation Exposure
• It should be noted that even a combination of CXR, lung scintigraphy, CTPA, and pulmonary
angiography exposes the foetus to approximately 1.5 mGy of radiation, which is well below the
accepted limit of 50 mGy for the induction of deterministic effects in the foetus.
101. A 62 year old man presents with right shoulder pain which radiates down his arm. A plain radiograph
confirms the presence of a right apical mass with destruction of the surrounding ribs. CT-guided biopsy is
performed and is likely to reveal:
A. Large cell lung cancer
B. Squamous cell cancer
C. Small cell lung cancer
D. Adenocarcinoma
E. Carcinoid
Source: Currie, Stuart, et al. SBAs for the FRCR 2A. 1st ed., Cambridge University Press, 2010.
Explanation:
• The case describes a Pancoast tumor for which squamous is the most common cell type.
102. A 3-month-old child presents to the paediatric outpatient clinic with a history of recurrent respiratory
distress. The child had an uneventful delivery, but has had recurrent problems since birth. The child had a
CXR taken prior to discharge home, aged 2 days, which showed a density in the left upper lobe, felt by the
paediatrician to represent the thymus. Whilst the infant has never required admission, the mother is
concerned due to recurrent coughing and dyspnoea. A CXR obtained at the clinic shows a large
hyperlucent area in the left upper lobe. What is the most likely diagnosis?
A. Congenital lobar emphysema.
B. Congenital cystic adenomatoid malformation.
C. Pulmonary sequestration.
D. Persistent PIE.
E. Congenital diaphragmatic hernia.
Source: Lindsay, Richard, et al. SBAs for the FRCR Part 2A. 1st ed., Oxford University Press, 2012.
Explanation:
Lobar Predilection and Presentation
• This has a lobar predilection with around 40% being found in the left upper lobe.
• Most present in the neonatal period with respiratory distress, but they can present later.
Congenital Lobar Emphysema (CLE)
• Congenital lobar emphysema initially presents as an area of soft tissue density due to retained foetal
pulmonary fluid.
• This resolves and is replaced by hyperlucency.
Congenital Cystic Adenomatoid Malformation (CCAM)
• Congenital cystic adenomatoid malformations (CCAM) do not show a lobar predilection, but can be
found anywhere.
• They can be either air or fluid filled and consist of multiple cysts.
EBDR Exam MCQs & Concepts May 2023
Dr. Kareem Alnakeeb
189
• CCAM are graded on the size of the cysts with type 1 lesions containing one or more large cysts, type
2 have numerous small cysts, and type 3 contain microscopic cysts, but appear solid at imaging.
Congenital Diaphragmatic Hernias
• Congenital diaphragmatic hernias are also initially solid on plain radiography and only contain air if
there is bowel present within the hernia and this contains air.
• This would obviously have continuity with the diaphragm and not be contained entirely within the
upper lobe.
Persistent Pulmonary Interstitial Emphysema (PIE)
• Persistent PIE occurs when PIE fails to resolve after 1 week.
• As PIE is almost always seen in infants with surfactant deficiency being ventilated, it would not be in
the differential in this case.
Sequestrations
• Sequestrations are usually solid masses on plain film radiography, unless there has been a history of
infection within the sequestration.
103. A 16-year-old male with a history of epilepsy is investigated via MRI. Axial T2WI demonstrates a cystic
space within the left frontal lobe isointense to CSF. This is causing local mass effect and there is adjacent
enlargement of the left lateral ventricle. What is the most likely diagnosis?
A. Porencephalic cyst.
B. Arachnoid cyst.
C. Schizencephaly.
D. Hydranencephaly.
E. Ependymal cyst.
Source: Lindsay, Richard, et al. SBAs for the FRCR Part 2A. 1st ed., Oxford University Press, 2012.
Explanation:
Porencephaly
• Porencephaly is a congenital/acquired cystic cavity within the brain parenchyma with adjacent
enlargement of the lateral ventricle.
• They develop in utero or early infancy.
Arachnoid Cysts
• Arachnoid cysts are also CSF isointense, but are extra-axial, displacing the brain away from the
adjacent skull.
Ependymal Cysts
• Ependymal cysts are intraventricular and the surrounding brain is usually normal.
Schizencephaly
• Schizencephaly is characterized by an intraparenchymal cleft extending from the ventricular surface
to the brain surface lined by gray matter.
Hydranencephaly
• Hydranencephaly results from an early destructive process of the developing brain.
• The cranial vault is CSF filled with absence of the cortical mantle and ventricles (water-bag brain).
• Death in infancy is typical.
EBDR Exam MCQs & Concepts May 2023
Dr. Kareem Alnakeeb
190
104. Which of the following is not an angiographic sign of active bleeding?
A. Contrast extravasation
B. Vessel spasm
C. Vessel cut-off
D. Early venous filling
E. Vessel dilatation
Source: Chin, Teck Yew, et al. Get Through Final FRCR 2A: SBAs. 1st ed., CRC Press, 2017.
Explanation:
• The rest are angiographic signs of active bleeding; vessel dilatation is not.
105. An 80-year-old man presents with hemoptysis and a mass on chest radiograph. A biopsy shows non-
small-cell lung cancer. CT of chest shows a 4 cm, right middle lobe mass with pleural tethering but no
chest wall invasion. Lymph nodes are seen at the right hilum (17 mm short axis), in the subcarinal space
(20 mm short axis) and in the aortopulmonary space (8 mm short axis). No other abnormalities are seen.
What is the TNM stage?
A. T2 N1 M0
B. T2 N2 M0
C. T2 N3 M0
D. T3 N1 M0
E. T3 N2 M0
Source: Bydder, Megan, et al. Get Through Final FRCR Part A: SBAs for the Modular Examination. 1st
ed., Hodder Education Publishers, 2009.
Explanation:
• The T stage is T2, as the lesion is over 3cm but there is no chest wall or mediastinal invasion or other
associated feature.
• The nodes at the right hilum (N1) and in the subcarinal space (N2) are significantly enlarged, whereas
the node in the aortopulmonary space (N3) is not (,10mm short axis), hence the N stage is N2.
106. A 35-year-old man involved in a major RTA undergoes a lateral view of the cervical spine in the resus on
arrival. All of the following are features associated with atlanto-occipital dislocation, except
A. Soft-tissue swelling anterior to C2 by >10 mm.
B. Basion dens interval >12 mm.
C. Odd’s ratio >1.
D. X-ray can often be normal.
E. Incongruity of articular surface of atlas and occipital condyles.
Source: Chin, Teck Yew, et al. Get Through Final FRCR 2A: SBAs. 1st ed., CRC Press, 2017.
Explanation:
Lateral Radiograph Findings
• Atlanto-occipital dislocation shows the following on lateral radiograph of the cervical spine:
• 10 mm soft-tissue swelling anterior to C2, with pathological convexity (80%),
• basion-dens interval of >12 mm,
• odds ratio (distance between the basion and the posterior arch of the atlas divided by opisthion and
anterior arch of atlas) >1, and
• basion–posterior axial line interval >12 mm anterior/>4 mm posterior to axial line.
Direct Radiographic Signs
• Direct signs include loss of congruity of articular surfaces of atlas and occipital condyle.
Diagnostic Caveat
• Normal X-ray in the presence of atlanto-occipital dislocation is rare.
EBDR Exam MCQs & Concepts May 2023
Dr. Kareem Alnakeeb
191
107. You have been asked to review a chest radiograph by a junior doctor. The image demonstrates subtle hazy
opacification of the upper part of the lower zone of the right lung. The right atrial border is indistinct and
the horizontal fissure runs from the right hilum to the eighth rib in the mid axillary line. What is the most
plausible explanation for these findings?
A. Middle lobe collapse
B. Middle lobe consolidation
C. Pectus excavatum
D. Right lower lobe mediobasal segment consolidation
E. Right lower lobe anteriobasal segment consolidation
Source: Proctor, Robin. Final FRCR Part A Modules 1–3 Single Best Answer MCQs: The SRT Collection
of 600 Questions with Explanatory Answers (MasterPass). 1st ed., CRC Press, 2009.
Explanation:
• The findings are those of middle lobe collapse.
• Signs on the frontal radiograph can be subtle, and it is more easily seen on the lateral radiograph.
• In this case the loss of clarity of the right atrial border indicates the pathology is located in the middle
lobe.
• There is loss of volume (the normal horizontal fissure runs from the hilum to the sixth rib in the mid
axillary line), therefore collapse of the middle lobe, rather than consolidation, is the likely cause for
these appearances.
108. In acute respiratory distress syndrome what is the first change usually seen on the chest radiograph?
A. confluent consolidation
B. pleural effusions
C. increased heart size with globular shape
D. volume loss with atelectasis
E. patchy ill-defined opacities
Source: Bydder, Megan, et al. Get Through Final FRCR Part A: SBAs for the Modular Examination. 1st
ed., Hodder Education Publishers, 2009.
Explanation:
• Acute respiratory distress syndrome (ARDS) commences with interstitial oedema, progressing to
congestion and extensive alveolar, and interstitial oedema and hemorrhage.
• The chest radiograph is often normal for the first 24 hours, before patchy opacities appear in both
lungs.
• These progress to massive airspace consolidation over the following 24–48 hours.
• True volume loss, atelectasis, cardiomegaly and effusions are not seen in ARDS.
109. A 16-year-old male presents with sudden shortness of breath. A chest radiograph shows multiple,
bilateral nodules measuring up to 3 cm, some of which are calcified. There is a moderate left
pneumothorax. The patient has been undergoing treatment for a malignant tumor. What is the most likely
diagnosis?
A. metastases secondary to Wilms’ tumor
B. metastases secondary to osteosarcoma
C. metastases secondary to testicular tumor
D. abscesses secondary to immunosuppression
E. varicella pneumonia secondary to immunosuppression
EBDR Exam MCQs & Concepts May 2023
Dr. Kareem Alnakeeb
192
Source: Bydder, Megan, et al. Get Through Final FRCR Part A: SBAs for the Modular Examination. 1st
ed., Hodder Education Publishers, 2009.
Explanation:
• Osteosarcoma pulmonary metastases are uncommon (seen in 2% of cases) and present as multiple
masses which may calcify. There is a high incidence of associated pneumothorax.
• Wilms’ tumors may also produce multiple pulmonary masses and may be associated with
pneumothorax, but are not known to calcify.
• Testicular tumors may produce calcified lung metastases, but are not associated with
pneumothorax.
• Varicella pneumonia shows patchy consolidation in the acute phase, with multiple, small, calcified
nodules in the chronic phase.
• Abscesses may present as multiple masses but rarely calcify and often cavitate.
110. A 35 year old with asthma presents with malaise, flu-like illness and cough. Previous similar episodes
have occurred. A chest radiograph shows patchy airspace opacification in the mid and upper zones.
Which feature on high-resolution CT would make allergic bronchopulmonary aspergillosis a more likely
diagnosis than extrinsic allergic alveolitis?
A. widespread centrilobular micronodules ,3 mm
B. tubular finger-like opacities
C. bronchiectasis
D. upper-zone fibrosis
E. pleural effusion
Source: Bydder, Megan, et al. Get Through Final FRCR Part A: SBAs for the Modular Examination. 1st
ed., Hodder Education Publishers, 2009.
Explanation:
• Allergic bronchopulmonary aspergillosis (ABPA) is hypersensitivity to aspergillus in people with
asthma. Typical features are of a migratory pneumonitis, predominantly in the upper lobes.
• It may cause bronchiectasis and upper-zone fibrosis, which are features also seen in extrinsic
allergic alveolitis (EAA).
• Tubular opacities, indicating mucus plugging, are seen in ABPA, but not in EAA.
• Centrilobular nodules are seen in EAA, along with mosaic perfusion and patchy ground-glass
change.
• Pleural effusions are rarely seen in EAA and not in ABPA.
111. A 56-year-old male presents with wheezing, cough and recurrent chest infections. A chest radiograph
shows right middle lobe consolidation. CT of the chest shows a 3 cm mass arising within the right middle
lobe bronchus with distal collapse and consolidation. Which feature of the mass would make
hamartoma more likely than carcinoid?
A. central location
B. presence of calcification
C. cavitation
D. presence of fat
E. prominent enhancement
EBDR Exam MCQs & Concepts May 2023
Dr. Kareem Alnakeeb
193
Source: Bydder, Megan, et al. Get Through Final FRCR Part A: SBAs for the Modular Examination. 1st
ed., Hodder Education Publishers, 2009.
Explanation:
• Hamartomas are usually seen in the periphery of the lungs (two-thirds) with 10% being
endobronchial. Calcification is seen in 15%, often popcorn type. Cavitation is rare but fat is seen in
50%.
• Carcinoids are usually located centrally and are endobronchial. Calcification is seen in one-third
and they rarely cavitate. They do not contain fat and show prominent enhancement following
contrast, as they are vascular.
112. A 50-year–old male is admitted under the surgical team having presented with upper abdominal pain and
raised inflammatory markers. Suspecting acute cholecystitis, an ultrasound is requested, but due to
large body habitus there is poor visualization of his gallbladder. To further evaluate hepatobiliary
scintigraphy using 99mTc-labelled iminodiacetic acid is arranged. Which of the following findings are
consistent with acute cholecystitis?
A. Non-visualization of the gallbladder at 1 and 4 hours.
B. Non-visualization of the gallbladder at 1 hour but seen at 4 hours.
C. Visualization of the gallbladder at 1 hour.
D. Visualization of the gallbladder at 30 minutes after morphine administration.
E. Hepatobiliary scintigraphy is not appropriate for investigation of acute cholecystitis.
Source: Lindsay, Richard, et al. SBAs for the FRCR Part 2A. 1st ed., Oxford University Press, 2012.
Explanation:
Indications and Purpose
• Hepatobiliary scintigraphy is most commonly used to evaluate suspected acute cholecystitis.
Patient Preparation
• A minimum of 2 hours fasting is required.
Radiotracer Dynamics
• Following prompt uptake by the liver, the radiotracer is excreted into the biliary system and drains
into the small bowel.
• Activity should be demonstrated within the gallbladder by 1 hour.
• Morphine can be used during the scan to relax the sphincter of Oddi, thus pushing radiolabelled bile
into the gallbladder.
Diagnostic Criteria
• Acute cholecystitis is characterized by non-visualization of the gallbladder at both 1 and 4 hours or at
30 minutes following morphine administration.
• Non-visualization of the gallbladder at 1 hour, but seen at 4 hours, is indicative of chronic
cholecystitis.
• A false-positive diagnosis of acute cholecystitis can occur with previous cholecystectomy,
gallbladder agenesis, and tumor obstructing the cystic duct.
113. A radiologist is reporting a 99mTc bone scan and describes it as a ‘superscan’. He can say this because of
reduced uptake in the:
A. brain
B. skeleton
C. kidneys
D. bowel
E. myocardium.
EBDR Exam MCQs & Concepts May 2023
Dr. Kareem Alnakeeb
194
Source: Lindsay, Richard, et al. SBAs for the FRCR Part 2A. 1st ed., Oxford University Press, 2012.
Explanation:
Definition and Causes
• A superscan refers to a 99mTc-labelled technetium IBS where there is diffuse increased osseous
uptake with apparent reduced renal and soft tissue uptake.
• The appearance is commonly due to widespread osteoblastic bony metastases (e.g. prostate or
breast carcinoma), but is also caused by non-malignant disease (e.g. renal osteodystrophy,
hyperparathyroidism, osteomalacia, myelofibrosis, Paget’s disease).
• In metastatic disease there is usually higher uptake in the axial than the appendicular skeleton.
Normal IBS and FDG-PET Uptake Patterns
• In IBS uptake is normally seen in bone, kidneys, and bladder, soft tissues (low levels), breasts
(particularly in young women), and epiphyses (skeletally immature patients).
• Uptake is seen in the myocardium (high), brain (high), and bowel (moderate) in FDG-PET scanning,
not IBS; however myocardial uptake on IBS can be seen in cases of recent myocardial infarction and
amyloidosis.
Pitfalls and False Results
• Note that poor renal function can often demonstrate reduced or absent renal visualization producing
an appearance similar to a superscan (false positive), whereas urinary tract obstruction in prostatic
carcinoma can increase renal activity and lead to false negative scans.
114. A 5-year-old boy presents with a history of walking difficulty. On examination he is noted to have an
antalgic gait and lower limb length discrepancy, with the right limb being shorter than the left. Plain
radiographs of the right leg show lobular ossific masses arising from the distal femoral epiphysis and the
talus, which resemble osteochondromas. What is the most likely underlying diagnosis?
A. Dysplasia epiphysealis hemimelica (Trevor disease).
B. Multiple epiphyseal dysplasia.
C. Diaphyseal aclasis.
D. Dyschondrosteosis (Leri–Weil disease).
E. Klippel–Trenaunay–Weber syndrome.
Source: Lindsay, Richard, et al. SBAs for the FRCR Part 2A. 1st ed., Oxford University Press, 2012.
Explanation:
• This is an uncommon developmental disorder relating to the formation of an osteochondroma-type
lesion at the epiphyses of usually a single lower extremity.
• The epiphyses most commonly involved are those on either side of the knee or ankle.
• Typically it is only the medial or lateral side of the epiphyses affected (medial:lateral 2:1).
• The disease is usually recognized at a young age because of an antalgic gait, palpable mass, varus or
valgus deformity, or limb length discrepancy.
115. A 5-year-old boy with bilateral wrist pain undergoes a plain fi lm which reveals several pedunculated bony
outgrowths from the metaphysis of both radii, which point away from the adjacent joints. What is the
most likely diagnosis?
A. Ollier disease.
B. Maffucci syndrome.
C. Morquio syndrome.
D. Diaphyseal aclasia.
E. Hunter syndrome.
EBDR Exam MCQs & Concepts May 2023
Dr. Kareem Alnakeeb
195
Source: Lindsay, Richard, et al. SBAs for the FRCR Part 2A. 1st ed., Oxford University Press, 2012.
Explanation:
Description and Etiology
• The description is classic for multiple hereditary osteochondromas/exostoses, also known as
diaphyseal aclasia.
• Osteochondromas are the result of displaced growth plate cartilage, which causes lateral bone
growth from the metaphysis.
• They typically point away from the epiphysis.
• There is continuity of the normal marrow, cortex, and periosteum between the exostosis and the host
bone.
• The cartilage cap, which is the source of growth, may have some chondroid matrix, but the
appearance is otherwise of a deformed but normal bone.
Distribution and Growth
• They are normally found in the extremities, with 36% around the knee.
• Their growth normally ceases at skeletal maturity.
Clinical Manifestations
• Symptoms are related to pressure effects on adjacent neural or vascular structures.
Malignant Transformation
• Less than 1% of solitary osteochondromas undergo malignant transformation to chondrosarcoma.
• Findings that should alert to this are destruction of exostosis bone, destruction of matrix in the
cartilage cap, irregular or thick (>2 cm in adults, >3 cm in children) cap, or growth of the cap after
skeletal maturity.
Multiple Hereditary Osteochondromatosis
• Multiple hereditary osteochondromatosis is an uncommon autosomal dominant condition.
• Patients present with multiple osteochondromas, which cause short stature.
• The elbow and wrist joints are often deformed.
• There is a higher risk of malignant transformation than in solitary osteochondromas, probably 2–5%.
Differential Diagnoses
• Ollier disease is the presence of multiple enchondromas and Mafucci syndrome requires, in
addition, multiple soft-tissue haemangiomas.
• Morquio and Hunter syndromes are mucopolysaccharidoses, with their own musculoskeletal
abnormalities, which often make an appearance in exams.
116. An 80-year-old man undergoes CT brain for confusion. A well-defined intramedullary bony lesion
containing speckled calcification is identified in the left frontal diploë. He has multiple small, rounded,
benign skin tumours since youth. Which diagnosis best explains the imaging and clinical findings?
A. Frontal osteochondroma
B. Frontal osteoma
C. Frontal meningioma
D. Frontal chondrosarcoma
E. Frontal osteoid osteoma
Explanation: (by Perplexity AI, generated with the OpenAI O3 model)
• Intradiploic frontal osteomas are dense, slow-growing, intramedullary bone lesions that show
mature lamellar bone with variable calcification on CT. They are classically associated with multiple
EBDR Exam MCQs & Concepts May 2023
Dr. Kareem Alnakeeb
196
cutaneous sebaceous cysts or epidermoid cysts in Gardner syndrome, matching this patient’s
longstanding skin lesions.
• Osteochondromas (A) arise from the metaphyseal surface of long bones, not the skull.
• Meningiomas (C) may calcify but are extra-axial soft-tissue masses, typically showing hyperostosis
rather than forming a discrete intramedullary bony nidus.
• Chondrosarcomas (D) of the skull base show “rings-and-arcs” chondroid calcification and an
aggressive lytic pattern, unlike the benign sclerotic appearance here.
• Osteoid osteoma (E) produces a small cortical nidus causing intense night pain and is exceedingly
rare in the calvarium.
117. A 40-year-old man of short stature is found to have multiple metaphyseal osteochondromas in the long
bones. What is the approximate risk of malignant transformation of these lesions?
A. Less than 5%
B. Less than 10%
C. Less than 20%
D. Less than 30%
E. Around 50%
Explanation: (by Perplexity AI, generated with the OpenAI O3 model)
• Osteochondromas are benign exostoses that arise from the metaphyses of long bones.
• In solitary lesions, malignant change to secondary peripheral chondrosarcoma is rare (≈1%).
• In patients with multiple hereditary exostoses—classically associated with short stature—the pooled
literature shows a transformation risk of about 5%–10%.
• The other options over- or underestimate this figure: <5% reflects solitary cases; ≥20% is far above
the accepted range for multiple lesions.
118. An adult’s hand X-ray shows a well-defined exophytic ossified mass stuck onto the cortical surface of a
proximal phalanx without corticomedullary continuity. Histology confirms a bizarre osteochondromatous
parosteal proliferation (Nora lesion). Which statement about this entity is CORRECT?
A. It arises within the cortex and medulla of long bones
B. It is confined to the medullary cavity of short tubular bones
C. It is characteristically seen in the hands and feet
D. It always behaves as an overtly malignant tumour
E. It never recurs after marginal excision
Explanation: (by Perplexity AI, generated with the OpenAI O3 model)
• Bizarre parosteal osteochondromatous proliferation (BPOP) is a benign surface‐based lesion that
most frequently affects the small bones of the hands and feet, accounting for more than half of
reported cases. It originates on the periosteal surface and typically lacks medullary continuity.
• Long-bone involvement is unusual, so option A is incorrect.
• Option B is wrong because the lesion is parosteal, not intramedullary.
• Although overwhelmingly benign, rare malignant transformation after multiple recurrences is
documented, so option D overstates its behaviour.
• Recurrence rates approach 30%–50% after simple excision, making option E incorrect.
EBDR Exam MCQs & Concepts September 2022
Dr. Kareem Alnakeeb
197
September 2022
1. Which of the following characteristics is seen in a sequestered disk?
A. Peripheral enhancement
B. Continuity with a disk extrusion
C. Posterior location in the spinal canal
D. Hemorrhage
E. Foraminal location
Source: Riascos-Castaneda, Roy F., et al., editors. RadCases Plus Q&A: Neuro Imaging. 2nd ed.,
Thieme, 2018.
Explanation:
• Sequestered disks show peripheral enhancement in the inflammatory phase.
2. Which of the following conditions presents with lesions that are cigar-shaped on sagittal imaging, wedge-
shaped on axial imaging, are eccentrically located, and are less than or equal to two vertebral bodies in
length?
A. Multiple sclerosis
B. Neuromyelitis optica (NMO)
C. Transverse myelitis
D. Spinal cord infarction
E. Subacute combined degeneration
Source: Riascos-Castaneda, Roy F., et al., editors. RadCases Plus Q&A: Neuro Imaging. 2nd ed.,
Thieme, 2018.
Explanation:
• MS lesions in the spinal cord typically have these features on MR imaging.
3. Regarding Chance fracture, which statement is true?
A. There is no distraction of the posterior elements.
B. There is fracture of the pedicles and variable involvement of the posterior elements.
C. It is more frequently centered in the craniocervical junction.
D. The anterior ligamentous complex is frequently involved.
Source: Riascos-Castaneda, Roy F., et al., editors. RadCases Plus Q&A: Neuro Imaging. 2nd ed.,
Thieme, 2018.
Explanation:
• There is fracture of the pedicles and variable involvement of the posterior elements.
• Chance fractures are more frequently centered at the thoracolumbar (not craniocervical) junction.
• Distraction of the posterior elements and involvement of the posterior (not anterior) ligamentous
complex are common.
4. Which is true regarding Duret hemorrhages?
A. Occur secondary to upward transtentorial herniation
B. Result in tonsillar herniation
C. Are more common in the medulla
D. Are generally centrally located
Source: Riascos-Castaneda, Roy F., et al., editors. RadCases Plus Q&A: Neuro Imaging. 2nd ed.,
Thieme, 2018.
Explanation:
• Duret hemorrhages are typically centrally located.
• They are caused by downward transtentorial herniation, they do not necessarily result in tonsillar
herniation and are more common in the midbrain and pons.
EBDR Exam MCQs & Concepts September 2022
Dr. Kareem Alnakeeb
198
5. Which of the following is a feature required for diagnosis of a Chiari I malformation?
A. Small posterior fossa
B. Cervical cord syrinx
C. Myelomeningocele
D. Cerebellar tonsils below the level of the foramen magnum
Source: Riascos-Castaneda, Roy F., et al., editors. RadCases Plus Q&A: Neuro Imaging. 2nd ed.,
Thieme, 2018.
Explanation:
• Answers a and care characteristics of Chiari II malformations.
• Although cervical cord syrinx is a common finding in Chiari I, it is not necessary to make the
diagnosis.
6. Which of the following posterior fossa cystic abnormalities is associated with a small vermian size?
A. Dandy-Walker malformation
B. Blake pouch cyst
C. Megacistema magna
D. Posterior fossa arachnoid cyst
Source: Riascos-Castaneda, Roy F., et al., editors. RadCases Plus Q&A: Neuro Imaging. 2nd ed.,
Thieme, 2018.
Explanation:
• Blake pouch cyst, megacisterna magna, and posterior fossa arachnoid cysts typically course with a
normal cerebellar vermis.
7. Which imaging modality is considered the gold standard for the evaluation of vasculitis?
A. CT angiography
B. MR angiography
C. Digital subtraction angiography (DSA)
D. Vessel wall MRI
Source: Riascos-Castaneda, Roy F., et al., editors. RadCases Plus Q&A: Neuro Imaging. 2nd ed.,
Thieme, 2018.
Explanation:
• Although occasionally primary angiitis of the central nervous system (PACNS) is not seen on imaging,
DSA is currently considered the gold standard for evaluation of PACNS due to its high spatial
resolution.
8. Which is true regarding advanced imaging in radiation necrosis?
A. Increased N-acetylaspartate (NM) on MR spectroscopy (MRS) images
B. Hypometabolism on fluorodeoxyglucose-positron emission tomography (FDG-PET) scan
C. Increased relative cerebral blood volume (rCBV) on MR perfusion images
D. Myoinositol peak on MRS images
Source: Riascos-Castaneda, Roy F., et al., editors. RadCases Plus Q&A: Neuro Imaging. 2nd ed.,
Thieme, 2018.
Explanation:
• Radiation necrosis is generally hypometabolic on FOG-PET. NM decreases, rCBV is low or normal,
and there is no myoinositol peak on MRS images.
EBDR Exam MCQs & Concepts September 2022
Dr. Kareem Alnakeeb
199
9. Imaging findings of mesial temporal sclerosis (MTS) include which of the following?
A. Small or atrophic hippocampus ipsilateral to the seizure focus with increased T1 signal.
B. Loss of internal architecture and loss of hippocampal head digitations.
C. Dilatation of the contralateral temporal horn.
D. Decreased T2 signal intensity of the ipsilateral amygdala.
E. Atrophy of contralateral mammillary body and fornix.
Source: Riascos-Castaneda, Roy F., et al., editors. RadCases Plus Q&A: Neuro Imaging. 2nd ed.,
Thieme, 2018.
Explanation:
• Findings of MTS include loss of internal architecture and loss of hippocampal head digitations.
• Findings also include small or atrophic hippocampus ipsilateral to the seizure focus with increased
T2 (not Tl) signal, dilatation of the ipsilateral temporal horn, increased (not decreased) T2 signal
intensity of the ipsilateral amygdala, and atrophy of ipsilateral (not contralateral) mammillary body
and fornix.
10. Which of the following is a characteristic of the typical presentation of epidural hematomas?
A. Will not cross the sutures
B. Crescent in shape
C. Content is venous blood
D. Will not cross the dural folds
E. 50% are infratentorial secondary to skull base fractures
Source: Riascos-Castaneda, Roy F., et al., editors. RadCases Plus Q&A: Neuro Imaging. 2nd ed.,
Thieme, 2018.
Explanation:
• Epidural hematomas typically will not cross the sutures and they have a biconvex shape, the content
is arterial blood, they will cross the dural folds, and > 90% are supratentorial in location.
11. Which is a characteristic of pyogenic spondylodiskitis?
A. Sparing of the intervertebral disk
B. Prevertebral soft tissue edema
C. Involvement of the posterior elements
D. Noncontiguous spread of infection
Source: Riascos-Castaneda, Roy F., et al., editors. RadCases Plus Q&A: Neuro Imaging. 2nd ed.,
Thieme, 2018.
Explanation:
• Prevertebral edema is a common feature of pyogenic spondylodiskitis and aids in differentiating it
from degenerative spondylosis. Sparing of the disk, involvement of the posterior elements, and
noncontiguous spread of infection are more common in tuberculous spondylitis.
12. A dark rim is noted on susceptibility images in a cystic lesion of the sella/suprasellar region. Which of the
following is the most likely diagnosis?
A. Chronic pituitary apoplexy
B. Rathke cleft cyst
C. Suprasellar meningioma
D. Papillary craniopharyngioma
E. Cystic degeneration of a pituitary adenoma
EBDR Exam MCQs & Concepts September 2022
Dr. Kareem Alnakeeb
200
Source: Riascos-Castaneda, Roy F., et al., editors. RadCases Plus Q&A: Neuro Imaging. 2nd ed.,
Thieme, 2018.
Explanation:
• Chronic pituitary apoplexy will present as a cystic lesion with a hemosiderin rim better seen on the
susceptibility sensitive images.
• None of the other options would present like this.
• Adamantinomatous craniopharyngiomas are more frequently seen in the pediatric population and
tend to calcify.
13. Which of the following temporal lobe lesions typically present as a cystic mass with an enhancing
nodule?
A. Supratentorial ependymoma
B. Central neurocytoma
C. Ganglioglioma
D. DNET
E. Embryonal tumor with multilayered rosettes
Source: Riascos-Castaneda, Roy F., et al., editors. RadCases Plus Q&A: Neuro Imaging. 2nd ed.,
Thieme, 2018.
Explanation:
• Gangliogliomas present as cystic masses in the temporal lobe with a mural nodule.
• Supratentorial ependymomas tend to be extraventricular and show a heterogenous appearance,
such as embryonal tumors with multilayered rosettes.
14. Apical pseudotumor of the orbit extending to the cavernous sinus associated with painful
ophthalmoplegia is known as which one of the following?
A. Ramsay Hunt syndrome
B. Tolosa-Hunt syndrome
C. Horner syndrome
D. Gradenigo syndrome
E. Bell's palsy
Source: Riascos-Castaneda, Roy F., et al., editors. RadCases Plus Q&A: Neuro Imaging. 2nd ed.,
Thieme, 2018.
Explanation:
• This is the definition of TolosaHunt syndrome.
• Ramsay Hunt syndrome is varicella zoster virus infection affecting the sensory fibers of cranial
nerves VII and VIII.
• Horner syndrome is an interruption of the cervical sympathetic pathway.
• Gradenigo syndrome is petrous apicitis that presents with retro-orbital pain and cranial nerve
palsies.
• Bell's palsy is facial nerve paralysis secondary to herpes simplex infection.
15. Central restricted diffusion is present in which of the following conditions?
A. Metastasis
B. Abscess
C. Cyst
D. Primary brain neoplasm
EBDR Exam MCQs & Concepts September 2022
Dr. Kareem Alnakeeb
201
Source: Riascos-Castaneda, Roy F., et al., editors. RadCases Plus Q&A: Neuro Imaging. 2nd ed.,
Thieme, 2018.
Explanation:
• High diffusion-weighted imaging signal with low apparent diffusion coefficient values are typically
present centrally within cerebral abscesses, representing true restricted diffusion.
16. Which of the following leukodystrophies typically enhances?
A. Canavan disease
B. X-linked adrenoleukodystrophy
C. Alexander disease
D. Pelizaeus-Merzbacher disease
Source: Riascos-Castaneda, Roy F., et al., editors. RadCases Plus Q&A: Neuro Imaging. 2nd ed.,
Thieme, 2018.
Explanation:
• X-linked adrenoleukodystrophy (ALD) is characterized by an intermediate zone that represents
disruption of the blood-brain barrier that typically enhances.
• The other white matter diseases in the question do not enhance.
17. Which of the following mitochondrial disorders classically presents with cortical involvement?
A. Leigh syndrome
B. Myoclonus epilepsy with ragged red fibers (MERRF)
C. Mitochondrial encephalopathy with lactic acidosis and strokelike lesions (MELAS)
D. d) Keams-Sayre syndrome
Source: Riascos-Castaneda, Roy F., et al., editors. RadCases Plus Q&A: Neuro Imaging. 2nd ed.,
Thieme, 2018.
Explanation:
• MELAS is a mitochondrial disorder that presents with cortical and white matter involvement in a
nonvascular distribution, mimicking stroke.
18. Which of the following is typical of Joubert syndrome?
A. Normal decussation of the corticospinal tracts
B. A large fourth ventricular cyst
C. Elongated superior cerebellar peduncles
D. Normal cerebellar vermis
Source: Riascos-Castaneda, Roy F., et al., editors. RadCases Plus Q&A: Neuro Imaging. 2nd ed.,
Thieme, 2018.
Explanation:
• Joubert syndrome features lack of decussation of the both the corticospinal tracts and the superior
cerebellar peduncles.
• Typical features also include elongated superior cerebellar peduncles and a small cerebellar vermis.
• A large fourth ventricular cyst is not a characteristic of Joubert syndrome.
19. Which of the following lesions can show increased T1 signal in the sellar region?
A. Pituitary microadenoma
B. Pituitary macroadenoma
C. Rathke cleft cyst
D. Hamartoma of the tuber cinereum
EBDR Exam MCQs & Concepts September 2022
Dr. Kareem Alnakeeb
202
Source: Riascos-Castaneda, Roy F., et al., editors. RadCases Plus Q&A: Neuro Imaging. 2nd ed.,
Thieme, 2018.
Explanation:
• Rathke deft cyst, pituitary apoplexy, sellar aneurysms, and hemorrhagic sellar metastasis can
typically demonstrate increased T1 signal.
20. Concerning renal lymphoma, which one is TRUE?
A. Multiple or solitary focal nodular masses are the most common form.
B. It demonstrates uniform, hyperintense enhancement after IV gadolinium.
C. Direct extension to and involvement of the psoas muscle is more characteristic of primary renal cell
carcinoma than of renal lymphoma.
D. Tumor thrombus commonly occurs in renal lymphoma.
Source: ACR.
Explanation:
A. Correct. There are 3 basic patterns of renal involvement by lymphoma:
1) direct invasion by adjacent nodal disease,
2) focal masses that may be solitary or multiple (most common), and
3) diffuse infiltration.
B. Incorrect. Renal lymphoma typically enhances minimally to a mildly heterogenous pattern.
C. Incorrect. Renal lymphoma can commonly extend to and involve the adjacent psoas muscle. This
feature is rare in primary renal carcinoma.
D. Incorrect. Renal lymphoma rarely causes tumor thrombus. This is a common feature of renal
carcinoma.
21. Concerning prostate carcinoma, which one of the following is CORRECT?
A. 30% of prostate cancers arise from the peripheral zone of the prostate.
B. T1-weighted images provide the best contrast for detecting most prostate carcinomas.
C. Most prostate cancers demonstrate increased enhancement on immediate post gadolinium fat-
saturated T1 images.
D. Prostate cancer metastasizes early along the gonadal vein/lymphatic pathway to the periaortic and
pericaval region near the level of the kidneys.
Source: ACR.
Explanation:
A. Incorrect. 70% of prostate cancers arise from the peripheral zone, the remainder from the
transitional and central zones
B. Incorrect. Prostate carcinomas in the peripheral zone are generally isointense to surrounding
prostate tissue on T1-weighted images. T2-weighted images demonstrate prostate cancers as low
signal intensity compared to the surrounding normal high signal intensity peripheral zone.
C. Correct. Prostate cancer in the peripheral zone (where the majority of prostate cancers arise)
demonstrates increased enhancement compared to the normal peripheral zone tissue.
D. Incorrect. This metastatic pathway is characteristic of testicular neoplasms, not prostatic. Lymph
node metastases from prostate carcinoma are generally first to the obturator, external and internal
iliac chains.
22. Concerning renal medullary carcinoma, which one is TRUE?
A. Usually peripheral in location
B. Commonly seen in diabetic females
C. Common in patients with sickle trait
D. Often very small at presentation
EBDR Exam MCQs & Concepts September 2022
Dr. Kareem Alnakeeb
203
Source: ACR.
Explanation:
A. Incorrect. They are usually central.
B. Incorrect. Commonly seen in African American patients with sickle trait; more commonly male.
There is no association with diabetes.
C. Correct. Renal medullary carcinoma typically is seen as an infiltrative mass in patients with sickle
trait.
D. Incorrect. They are usually large at presentation.
23. Concerning congenital ureteropelvic junction (UPJ) obstruction, which one of the following is TRUE?
A. It is an uncommon cause of hydronephrosis in children.
B. Urinary tract infection is the most common presentation.
C. Females and males are affected equally.
D. The presence of crossing vessels decreases the success rate of pyeloplasty.
Source: ACR.
Explanation:
A. Incorrect. It is the MOST common cause of hydronephrosis in children.
B. Incorrect.
o UPJ obstruction is being discovered increasingly in the prenatal period due to frequent use of
obstetric ultrasound.
o When detected due to symptoms or signs, congenital ureteropelvic junction obstruction
most often presents in infancy or childhood with an abdominal mass, flank or abdominal
pain, failure to thrive, or nonspecific gastrointestinal complaints.
o Infection, hypertension, hematuria, and stone formation less commonly are the cause for
the child to come to medical attention.
o In a significant number of cases, the disorder is clinically silent into adulthood, when
hematuria, flank pain, fever, or rarely, hypertension, are the presenting clinical symptoms.
o Pain in adults is often episodic and in some cases may only present by high urine flow rates
such as those produced by beer drinking.
C. Incorrect. Males are affected more than females by 2:1.
D. Correct. Crossing vessels are seen in only 15%-20% of cases but significantly reduce the success of
pyeloplasty. Thus, many advocate the use of CT for preoperative planning.
24. What is the Bosniak classification of a renal cyst with complex septations and dense calcification?
A. Bosniak I
B. Bosniak II
C. Bosniak III
D. Bosniak IV
Source: ACR.
Explanation:
A. Incorrect. Bosniak I cysts are simple cysts and have no septations or calcifications. These require no
further evaluation
B. Incorrect. Bosniak II cysts have some atypical features, but are most likely benign. This group of
cysts can have thin septations or calcifications but not complex septations or dense calcifications.
Some lesions in this group are followed (subgroup IIF). Hyperdense, nonenhancing cysts are
included in the Bosniak II category.
EBDR Exam MCQs & Concepts September 2022
Dr. Kareem Alnakeeb
204
C. Correct. Bosniak III cysts can have dense calcifications, complex septations, and multiloculated
cysts. This group cannot be distinguished from malignancy, and often these lesions require surgical
exploration
D. Incorrect. Bosniak IV cystic masses have features which strongly suggest malignancy, such as an
enhancing solid component or thick irregular walls. Lesions in this category are treated as presumed
renal carcinomas.
25. Concerning blunt trauma to the bladder, which one of the following is TRUE?
A. Intraperitoneal rupture accounts for the majority of cases.
B. Less than 20% of extraperitoneal ruptures have pelvic fractures.
C. Intraperitoneal rupture is typically treated with surgical repair.
D. CT with intravenous contrast can exclude major bladder injury.
Source: ACR.
Explanation:
A. Incorrect. Extraperitoneal bladder ruptures account for 80%-90% of major bladder injuries.
Intraperitoneal ruptures account for 10%-20% of major bladder injuries.
B. Incorrect. Extraperitoneal bladder ruptures are almost always associated with pelvic fractures and
many are thought to be due to bladder laceration by the fracture fragments. (Although other causes
of extraperitoneal bladder injury have also been suggested, such as stress applied to the
puboprostatic ligaments causing the bladder wall to tear.)
C. Correct. Intraperitoneal bladder rupture is typically treated with surgical repair of the tear and
diverting vesicostomy.
D. Incorrect. Even delayed images of the bladder with CT and intravenous contrast are not adequate to
exclude major bladder injury. This is because there is inadequate distension of the bladder. At least
300 ml of fluid is required to adequately distend the bladder and evaluate for extravasation.
26. Concerning renal angiomyolipoma’s, which one finding is MOST diagnostic?
A. Fluid/fluid levels
B. Fat
C. Homogeneous soft tissue
D. Large irregular calcification
Source: ACR.
Explanation:
A. Incorrect. These lesions may occasionally hemorrhage but are usually incidental masses with mixed
amounts of soft tissue and macroscopic fat.
B. Correct. While other lesions such as renal cell carcinoma, oncocytoma, Wilm’s and metastasis have
also been reported with areas of fat within these tumors, these cases are rare.
C. Incorrect. Angiomyolipomas have varying amounts of fat and soft tissue. Some have no fat visible by
CT and a solid soft tissue renal mass in such a case is indistinguishable from renal cell carcinoma
and should be treated as such.
D. Incorrect. Calcification in angiomyolipomas is unusual but may occur if there has been prior
hemorrhage.
27. Which of the following is associated with testicular microlithiasis?
A. Testicular torsion
B. Epididymo-orchitis
C. Right-sided varicocele
D. Testicular neoplasm
EBDR Exam MCQs & Concepts September 2022
Dr. Kareem Alnakeeb
205
Source: ACR.
Explanation:
A. Incorrect. Microlithiasis is not typically seen in testicular torsion.
B. Incorrect. While the calcifications may be the result of prior infection, it does not have an increased
association with infection
C. Incorrect. There is no increased incidence of varicocele with testicular microlithiasis.
D. Correct. While testicular microlithiasis is often incidental, there is an increased incidence of
testicular neoplasm, most of which are germ cell tumors.
28. Regarding intravaginal testicular torsion, which one of the following is TRUE?
A. Color Doppler is more sensitive than power Doppler for detecting flow.
B. It is associated with an abnormal mesenteric attachment bilaterally.
C. It accounts for 70 % of cases of acute scrotal pain in adolescents.
D. Symmetric homogeneous echogenicity of the testes excludes the diagnosis.
Source: ACR.
Explanation:
A. Incorrect. Power Doppler is more sensitive than color Doppler for detecting flow, especially in
neonates and young boys. Power Doppler shows superiority in demonstrating intratesticular vessels.
Power Doppler is limited somewhat by being more sensitive to patient motion than color Doppler.
B. Correct. Cases of intravaginal torsion are caused by a bell-clapper deformity of attachment of the
mesentery to the testis. The abnormality is bilateral in nearly all cases.
C. Incorrect. Testicular torsion only accounts for 30% of cases of scrotal pain in boys age 12-18.
Epididymoorchitis or torsion of an appendix testis/epididymis are much more common causes of
scrotal pain.
D. Incorrect. In early torsion (when most critical to detect torsion to permit salvaging the testicle),
testes may have normally preserved gray-scale appearance. Later gray-scale ultrasound may
demonstrate decreased echogenicity of the testis, testicular swelling or reactive hydrocele. Early on,
the sonographic diagnosis of testicular torsion relies on the demonstration of decreased or absent
flow in the torsed testis on color or power Doppler.
29. Concerning adrenal cortical carcinoma, which one is TRUE?
A. It is the most common cause of an adrenal mass.
B. It most often displays areas of macroscopic fat.
C. It usually presents with <10 H.U. on non-contrast CT.
D. It usually presents as a large heterogeneous soft tissue mass.
Source: ACR.
Explanation:
A. Incorrect. Adrenal adenoma and metastatic disease are much more common than primary adrenal
cortical carcinoma.
B. Incorrect. While fat can rarely be seen in these tumors, macroscopic fat in an adrenal lesion is
almost always in a myelolipoma
C. Incorrect. Adrenal adenomas are more likely to present with the above characteristics.
D. Correct. Most adrenal cortical carcinomas are > 6 cm and often have central necrosis. Calcification
is seen in approximately 30% of these lesions.
EBDR Exam MCQs & Concepts September 2022
Dr. Kareem Alnakeeb
206
30. Concerning gonadal vein thrombosis, which one is TRUE?
A. Most common on the right side in post-partum women
B. Best study for diagnosis is excretory urography
C. Usually treated surgically
D. Commonly seen in diabetic males
Source: ACR.
Explanation:
A. Correct. TRUE. Gonadal vein thrombosis is in the differential for cause of fever in postpartum
woman.
B. Incorrect. FALSE. CT or MR are most sensitive in detection of gonadal vein thrombosis. The diagnosis
may also be made with US. IVU would not be expected to be helpful in this diagnosis.
C. Incorrect. FALSE. Patients are usually treated with anticoagulation and antibiotics.
D. Incorrect. FALSE. There is no association with diabetes; gonadal vein thrombosis is most commonly
seen in postpartum women (answer A).
31. What is the MOST common location of metastatic peritoneal implants?
A. Right paracolic gutter
B. Medial border of the cecum
C. Superior border of the sigmoid colon
D. Rectovesical space
Source: ACR.
Explanation:
A. Right paracolic gutter 18%.
B. Medial border of the cecum 41%.
C. Superior border of the sigmoid colon 21%.
D. Rectovesical space in 56%. It is the most dependent portion of the peritoneal cavity in both the
upright and supine positions.
32. Which of the following statements about localized fibrous tumors of the pleura is TRUE?
A. They are associated with asbestos exposure.
B. They are associated with hypertrophic pulmonary osteoarthropathy.
C. They account for the majority of pleural tumors.
D. Most of these tumors arise from parietal pleura.
Source: ACR.
Explanation:
A. Incorrect.
B. Correct. Localized fibrous tumor of the pleura are relatively rare tumors of the pleura. About 80% of
them arise from the visceral pleura. They affect male and female patients equally. They are not
associated with smoking, asbestos exposure or other environmental pollutants. About half of the
patients are asymptomatic when the tumor is discovered incidentally. They occasionally reach very
large size and produce symptoms of cough, dyspnea and chest pain. Paraneoplastic syndromes
such as hypoglycemia and hypertrophic osteoarthropathy are present in 4-5 % of the cases.
C. Incorrect.
D. Incorrect.
EBDR Exam MCQs & Concepts September 2022
Dr. Kareem Alnakeeb
207
33. Concerning Morton’s neuroma, which one of the following is TRUE?
A. It involves the digital branch of the plantar nerves.
B. It is most common between the second and third toes.
C. Histology demonstrates scattered mitosis and hypercellularity.
D. It has diffuse high-signal intensity on T2-weighted spin-echo images.
Source: ACR.
Explanation:
A. Morton’s neuroma is a non-neoplastic condition (compression neuropathy) representing neural
degeneration and perineural fibrosis secondary to entrapment of the digital branch of the medial or
lateral plantar nerves of the foot at the transverse intermetatarsal ligament. There may be associated
inflammation. Patients may experience pain and numbness. Lesions are typically unilateral. There is
a marked female predilection, as high as 18:1.
B. The second web space is the second most common location. It is most common at the third web
space between the third and fourth toes.
C. Because it is caused by impingement, histology shows perineural fibrosis, edema of the
endoneurium, and axonal degeneration and necrosis.
D. On T2W images, it is characterized by isointensity or lower signal intensity relative to fat. This helps to
differentiate a Morton’s neuroma from a true neuroma or fluid at the intermetatarsal bursa, which
has high signal intensity.
34. Concerning tunica albuginea cysts, which one is TRUE
A. They are intratesticular in location.
B. They are not palpable.
C. They range from 2 mm to 5 mm in size.
D. They are located in the posterior and inferior aspect of testis.
Source: ACR.
Explanation:
A. Incorrect. Tunica Albuginea cyst are extra testicular in location, however when large in size may
mimic an intratesticular cyst.
B. Incorrect. These cysts are palpable and patients present with a palpable lump.
C. Correct. These cysts are of mesothelial origin and range from 2-5mm in size.
D. Incorrect. Their characteristic location is at the upper anterior or lateral aspect of the testicle.
35. Which of the following measurements provides the BEST estimate of gestational age in a normal mid-
first-trimester pregnancy?
A. Yolk sac diameter
B. Crown-rump length
C. Mean sac diameter
D. Biparietal diameter
Source: ACR.
Explanation:
A. Yolk sac diameter is not an accepted measure of gestational age
B. An embryo is normally visible beginning at 6 weeks' gestation, and crown-rump length provides the
best estimate of embryonic age
C. The mean sac diameter may be used in the early first trimester but is not as accurate as crown-rump
length in the mid first trimester, once an embryo is visible.
D. Biparietal diameter should be used beginning at 12 weeks' gestation, but cannot be obtained in a 6-7
week embryo.
EBDR Exam MCQs & Concepts September 2022
Dr. Kareem Alnakeeb
208
36. What is the MOST common ultrasound finding in acute pyelonephritis?
A. Perirenal fluid collection
B. Normal-appearing kidney
C. Loss of corticomedullary differentiation
D. Poorly marginated, hypoechoic mass
Source: ACR.
Explanation:
A. Incorrect. A perirenal fluid collection may be seen as a complication of acute pyelonephritis, but in
most cases there are no complications and the ultrasound findings are normal.
B. Correct. A normal appearance is the most common ultrasound finding.
C. Incorrect. Pyelonephritis may alter the echo texture of the renal parenchyma, but in most cases the
ultrasound findings are normal.
D. Incorrect. Pyelonephritis may produce areas of increased as well as decreased echogenicity which
may simulate a mass, but in most cases the ultrasound findings are normal.
37. Concerning serous cystadenoma of the pancreas, which of the following is TRUE?
A. Individual cysts are usually larger than 2 cm.
B. The majority have a calcified central stellate scar.
C. It is common in von Hippel-Lindau disease.
D. It has a propensity to occur in young women.
Source: ACR.
Explanation:
• Although most patients with serous cystadenoma do not have von Hippel-Lindau (VHL) disease,
serous cystadenoma is more prevalent among patients with VHL particularly multiple lesions. In
sporadic cases, serous cystadenoma is usually a solitary lesion.
38. Regarding toxic megacolon, which one of the following is TRUE?
A. CT is usually required for diagnosis.
B. Peritonitis can occur without perforation.
C. It is due to severe and extensive submucosal inflammation.
D. Transverse colonic dilatation with normal haustra is diagnostic.
Source: ACR.
Explanation:
A. Diagnosis is typically made with conventional abdominal radiograph.
B. The inflammatory exudate of the colon seeps through the serosa and may cause peritonitis without
frank perforation.
C. It is due to severe and extensive transmural inflammation.
D. The profound inflammation and extensive ulceration of toxic megacolon always abolish the haustral
pattern, so the presence of haustral folds excludes the diagnosis.
39. Concerning focal hepatic fatty sparing, what is its appearance on imaging studies?
A. Hypodense on CT relative to remaining liver
B. Hyperechoic on US relative to remaining liver
C. Demonstrates no signal loss on fat-suppressed MR images
D. Heterogeneous enhancement
Source: ACR.
Explanation:
A. Incorrect. Areas of fatty sparing are hyperdense.
B. Incorrect. Fatty sparing is hypoechoic.
C. Correct.
EBDR Exam MCQs & Concepts September 2022
Dr. Kareem Alnakeeb
209
D. Incorrect. Enhancement pattern is similar to the entire liver.
40. What is the MOST common malignant primary hepatic tumor?
A. Hepatocellular carcinoma
B. Lymphoma
C. Angiosarcoma
D. Intrahepatic cholangiocarcinoma
Source: ACR.
Explanation:
A. Hepatocellular carcinoma is the most common primary hepatic malignancy.
B. Liver involvement may be an extrahepatic manifestation of hematologic malignancy.
C. Angiosarcoma is rare, and has been associated with exposure to Thorotrast and vinyl chloride.
D. Cholangiocarcinoma is the second most common primary hepatic malignancy.
41. Concerning cervical carcinoma, what stage is a lesion that is confined to the upper two thirds of the
vagina on clinical exam and that shows right hydroureter to the level of a poorly defined cervical soft
tissue mass on CT exam?
A. Stage II A
B. Stage II B
C. Stage III A
D. Stage III B
Source: ACR.
Explanation:
A. Incorrect. At stage II A the tumor has spread beyond the cervix but has no obvious parametrial
involvement, is confined to the upper two thirds of the vagina and no invasion of the ureter or
bladder.
B. Incorrect. Stage II B has obvious parametrial involvement but does not extend to the pelvic side wall.
C. Incorrect. Stage III A extends to the lower third of the vagina but not the pelvic sidewall and does not
obstruct the ureters or invade adjacent organs.
D. Correct. Stage III B tumors extend to pelvic sidewall and/or causes hydronephrosis or non-
functioning kidney.
42. Concerning Mirizzi syndrome, which one is TRUE?
A. Mirizzi syndrome is caused by gallstone impaction in the common hepatic duct.
B. Mirizzi syndrome is facilitated by an anatomic variant.
C. Cholecystocolonic fistula can complicate Mirizzi syndrome.
D. Bile duct injury at surgery is less likely in cases of Mirizzi syndrome.
Source: ACR.
Explanation:
A. Incorrect. Mirizzi syndrome is a complication of longstanding cholelithiasis. It is caused by
impaction of a gallstone in the cystic duct or in the gallbladder neck. Extrinsic mass effect of the
stone in the cystic duct on the common hepatic duct (CHD) or associated inflammatory changes
extending to the CHD causes obstruction of the extrahepatic biliary tree
B. Correct. Mirizzi syndrome is defined as extrinsic obstruction of the common hepatic duct (CHD),
usually caused by mass effect from a stone lodged in the adjacent cystic duct. CHD compression is
more likely in patients with low cystic duct insertions, because the cystic duct runs more parallel and
in closer proximity to the CHD when this variant anatomy is present. Inflammatory changes
extending from the cystic duct or gallbladder neck can also cause narrowing of the bile duct
EBDR Exam MCQs & Concepts September 2022
Dr. Kareem Alnakeeb
210
C. Incorrect. A cholecystocholedochal (not cholecystocolonic) fistula is a complication of Mirizzi
syndrome in 9 – 39% of cases. Fistula repair is required at the time of surgery. This requires either
choledochoplasty or hepaticojejunostomy.
D. Incorrect. Because of adhesions, inflammation and variant bile duct anatomy, the surgical
dissection is more difficult in Mirizzi syndrome. Inadvertent injury to the biliary tree or hepatic artery
is more likely. Although laparoscopic cholecystectomy can be performed successfully in patients
with Mirizzi syndrome, more require open cholecystectomy than in cases of uncomplicated
cholecystitis.
43. Concerning intrahepatic cholangiocarcinoma, which one is TRUE? [not sure]
A. It is usually hyperdense during the arterial phase of contrast enhancement.
B. It demonstrates washout during delayed CT imaging.
C. Post obstructive hepatic atrophy can be a prominent feature.
D. Extrahepatic spread of intrahepatic cholangiocarcinoma is uncommon.
Source: ACR.
Explanation:
A. Incorrect. In the absence of primary sclerosing cholangitis, intrahepatic cholangiocarcinoma usually
presents as a bulky mass. The lesion tends to be hypodense to background liver on non contrast CTs.
Following the dynamic administration of IV contrast, intrahepatic cholangiocarcinoma tends to
remain relatively hypovascular (not hyperdense), particularly centrally.
B. Incorrect. Increased patchy enhancement can be observed during the portal venous phase.
Retention of contrast within the extracellular space of the central stoma at delayed CT imaging (5 –
10 minutes) is relatively characteristic for these lesions.
C. Correct. High-grade obstruction from intrahepatic and hilar cholangiocarcinomas can cause atrophy
of the hepatic parenchyma surrounding the pathologically dilated biliary tree. Capsular retraction
occurs in about 20% of cases. This retraction likely reflects the atrophy of a small volume of hepatic
parenchyma beneath Glisson’s capsule. Surprisingly marked segmental and lobar atrophy can occur
when cholangiocarcinoma affects more central intrahepatic or hilar bile ducts
D. Incorrect. The extrahepatic spread of peripheral, intrahepatic cholangiocarcinoma is not
uncommon. In autopsy series, it is noted in about 50 – 70% of cases. Metastatic disease to regional
celiac and left gastric nodes occurs frequently. The prevalence of microscopic nodal disease with its
tendency to spread back to the liver makes the preoperative diagnosis of cholangiocarcinoma a
contraindication for hepatic transplantation.
44. Concerning pseudocyst of the pancreas, which of the following is TRUE?
A. Most common cystic pancreatic lesion
B. Has an epithelial cell lining
C. Does not communicate with the pancreatic duct
D. Can be distinguished from mucinous cystic neoplasms by imaging
Source: ACR.
Explanation:
• Pseudocyst is the most common pancreatic cystic lesion. Pseudocyst accounts for 85-90% of all
cystic lesions of the pancreas.
• True cysts and cystic neoplasms are not as common, and represent only 10-15% of pancreatic cystic
lesions.
45. Concerning islet cell tumors of the pancreas, which one of the following is TRUE?
A. Gastrinoma is the most common type of functioning islet cell tumor.
B. Zollinger-Ellison Syndrome is caused by gastrinoma.
EBDR Exam MCQs & Concepts September 2022
Dr. Kareem Alnakeeb
211
C. Functioning islet cell tumors usually present as large masses.
D. Functioning islet cell tumors are usually hypovascular.
Source: ACR.
Explanation:
A. Incorrect. Insulinomas are the most common functioning islet cell tumor with gastrinoma being the
second most common.
B. Correct. Zollinger-Ellison syndrome is caused by a gastrinoma. Peptic ulcer disease is seen in 90% of
patients with gastrinoma, usually with more extensive involvement of the duodenum than with
typical peptic ulcer disease. Many patients have diarrhea due to gastric hypersecretion.
C. Incorrect. Most functioning islet cell tumors present as small (1 – 2 cm) hypervascular pancreatic
masses
D. Incorrect. Most functioning islet cell tumors are hypervascular.
46. Concerning gastric lymphoma, which one is TRUE?
A. Accounts for 25% of all gastrointestinal lymphomas.
B. Vast majority are Hodgkin’s lymphoma.
C. Low-grade MALT lymphomas usually present as large bulky masses on upper gastrointestinal barium
studies.
D. Advanced gastric lymphoma is usually large and involves the body and antrum.
Source: ACR.
Explanation:
A. Incorrect. Lymphoma involves the stomach more frequently than any other portion of the
gastrointestinal tract and accounts for 50% of all gastrointestinal lymphomas.
B. Incorrect. Almost all gastric lymphomas are non-Hodgkin’s lymphoma of B-cell origin.
C. Incorrect. Low-grade MALT lymphomas usually present as varied sized, rounded nodules on double
contrast barium studies.
D. Correct. Advanced lymphoma is a large bulky mass and most commonly involves the gastric body
and antrum.
47. In routine adult practice, which underlying condition most frequently produces biliary obstruction?
A. Choledocholithiasis.
B. Cholangiocarcinoma
C. Carcinoma of the pancreatic head
D. Primary sclerosing cholangitis
E. Mirizzi syndrome
Explanation: (by Perplexity AI, generated with the OpenAI O3 model)
• Migrated gallstones obstructing the common bile duct (choledocholithiasis) account for the majority
of biliary obstruction cases encountered in day-to-day UK radiology.
• Malignant strictures—chiefly pancreatic head carcinoma and cholangiocarcinoma—are important
but each presents less often than stones.
• Primary sclerosing cholangitis causes multifocal biliary narrowing rather than a single dominant
blockage and is comparatively uncommon.
• Mirizzi syndrome is a rare variant of gallstone disease involving extrinsic compression of the common
hepatic duct by a cystic-duct stone and is far less frequent than straightforward choledocholithiasis.
EBDR Exam MCQs & Concepts September 2022
Dr. Kareem Alnakeeb
212
48. Concerning jaundice, which of the following is the MOST common etiology?
A. Choledocholithiasis.
B. Pancreatitis.
C. Benign stricture.
D. Pancreatic carcinoma.
Source: ACR.
Explanation:
A. Incorrect. Choledocholithiasis accounts for 20% of cases of biliary obstruction.
B. Incorrect. Pancreatitis accounts for 8%.
C. Correct. Benign stricture form surgery, trauma, or biliary intervention accounts for almost half of the
cases of biliary obstruction.
D. Incorrect. Pancreatic cancer does cause biliary obstruction, but less commonly than benign
stricture.
49. What of the following is associated with primary sclerosing cholangitis (PSC)?
A. Cholangiocarcinoma
B. Choledochal cyst
C. Choledochocele
D. Recurrent pyogenic cholangitis
Source: ACR.
Explanation:
A. Correct. Primary sclerosing cholangitis (PSC) is a significant risk factor for cholangiocarcinoma.
Among patients with PSC, the lifetime risk of cholangiocarcinoma is 10-15%, with an annual risk of
1.0-1.5%.
B. Incorrect. Choledochal cyst is considered to be congenital in etiology. It is postulated to be related
to anomalous development of the junction between the common bile duct and the pancreatic duct.
In patients with pancreaticobiliary maljunction (PBM), gallbladder cancers occur in 15% and bile
duct cancers occur in 5%. However, carcinogenesis in patients with choledochal cyst or PBM is not
related to PSC.
C. Incorrect. Most authors consider a choledochocele to be a Type III choledochal cyst. As such, it is
not related to PSC.
D. Incorrect. Recurrent pyogenic cholangitis (Oriental cholangiohepatitis) is an infectious disease, with
no particular association with PSC.
50. Concerning renal cystic disease, which one is TRUE?
A. Autosomal recessive polycystic disease typically presents as multiple bilateral cysts in adulthood.
B. Autosomal dominant polycystic disease typically presents as enlarged hyperechoic kidneys in the
neonatal period.
C. Acquired cystic renal disease in chronic renal failure patients on dialysis is indistinguishable from
autosomal dominant polycystic disease.
D. Autosomal dominant polycystic disease has a higher incidence of associated hepatic cysts
than does autosomal recessive polycystic disease.
Source: ACR.
Explanation:
A. Incorrect. Autosomal dominant polycystic disease usually presents with multiple bilateral simple
renal cysts between ages 20-39 years. Autosomal recessive polycystic disease has a spectrum of
presentation ages but is typically seen from the neonatal through childhood periods rather than
adulthood.
EBDR Exam MCQs & Concepts September 2022
Dr. Kareem Alnakeeb
213
B. Incorrect. This description is more typical of the appearance of the infantile form of ARPKD.
C. Incorrect. The kidneys are typically small and atrophic with multiple cysts in acquired cystic renal
disease of dialysis as compared to markedly enlarged kidneys in ADPCD.
D. Correct. ADPCD typically has multiple hepatic cysts in over 50% of cases. Autosomal recessive
polycystic disease is associated with hepatic fibrosis particularly in the juvenile onset form.
51. The staging chest CT of a 40 year old man with a known primary malignancy demonstrates cavitating
pulmonary metastases. The least likely type of primary lesion would be:
A. Squamous cell carcinoma
B. Malignant melanoma
C. Renal cell cancer
D. Sarcomas
E. Colonic carcinoma
Source: Currie, Stuart, et al. SBAs for the FRCR 2A. 1st ed., Cambridge University Press, 2010.
52. An 18 year old woman who is 32 weeks pregnant is referred for an obstetric ultrasound for ongoing
abdominal pain. She is shown to have a small placenta relative to gestational age. Which one of the
following would be a possible cause?
A. Molar pregnancy
B. Maternal diabetes
C. Umbilical vein obstruction
D. Pre-eclampsia
E. Maternal anaemia
Source: Currie, Stuart, et al. SBAs for the FRCR 2A. 1st ed., Cambridge University Press, 2010.
Explanation:
• Pre-eclampsia, IUGR, chromosomal abnormality and intrauterine infection can all cause a decrease
in placental size.
• Enlargement of the placenta is defined as a measurement of >5 cm when obtained at right angles to
the long axis of the placenta.
• The causes of placentomegaly include maternal diabetes, chronic intrauterine infection (e.g.
syphilis), maternal anaemia, thalassaemia and twin–twin transfusion syndrome.
• Fetal chromosomal abnormalities may cause either a large or small placenta.
53. On the 20-week fetal anomaly scan, it is noticed that there is less than 1 mm of hypoechoic myometrium
between placenta and echo-bright uterine serosa. An MRI is performed. On T2W images, the placenta is
heterogeneous and bright, and causes junctional zone interruption and marked focal myometrial
thinning. The serosa looks intact. These findings describe which of the following?
A. Placenta accreta.
B. Placenta increta.
C. Placenta percreta.
D. Placenta praevia.
E. Placental abruption.
Source: Bydder, Megan, et al. Get Through Final FRCR Part A: SBAs for the Modular Examination. 1st
ed., Hodder Education Publishers, 2009.
Explanation:
Placenta Accreta Spectrum
• The normal decidua forms a barrier between chorionic villi and uterus, preventing deep invasion of
placental material.
EBDR Exam MCQs & Concepts September 2022
Dr. Kareem Alnakeeb
214
• An underdeveloped or absent decidua permits direct contact of chorionic villi with the myometrium,
known as placenta accreta.
• When the villi invade the myometrium, it becomes placenta increta; if the serosa is penetrated, it is
placenta percreta.
• Diagnosis is difficult on ultrasound scan, but MRI can help.
Risk Factors
• Risk factors are previous caesarean section and myomectomy, multiparity and increasing maternal
age.
Complications and Management
• Complications include maternal haemorrhage, premature delivery, intrauterine growth retardation
and 5% chance of perinatal death.
• To protect the mother, balloon catheters can be placed over the internal iliac arteries prior to
caesarean delivery.
54. You are asked to perform an antenatal ultrasound examination and note that the placenta has an unusual
morphology. You see an additional lobule, which is separate from the main bulk of the placenta. What is
this variant of placental morphology known as?
A. Circumvallate placenta.
B. Bilobed placenta.
C. Placenta membranacea.
D. Succenturiate placenta.
E. Placenta accrete
Source: Lindsay, Richard, et al. SBAs for the FRCR Part 2A. 1st ed., Oxford University Press, 2012.
Explanation:
Accessory Placental Variants
• This is an additional lobule separate from the main bulk of the placenta.
• The significance of this variant is the rupture of vessels connecting the two components or retention
of the accessory lobe with resultant post-partum haemorrhage.
Circumvallate Placenta
• Circumvallate placenta has a chorionic plate smaller than the basal plate, with associated rolled
placental edges.
• There is known to be an increased risk of placental abruption and haemorrhage with this type of
placenta.
Bilobed Placenta
• A bilobed placenta is a placenta with two evenly sized lobes connected by a thin bridge of placental
tissue.
• This has no known increased risk of morbidity.
Placenta Membranacea
• Placenta membranacea is a thin membranous structure circumferentially occupying the entire
periphery of the chorion.
• There is an increased risk of placenta praevia, as a portion of the placenta completely covers the
internal os.
Abnormal Placental Invasion
• Placenta accreta is not a variant of placental morphology.
EBDR Exam MCQs & Concepts September 2022
Dr. Kareem Alnakeeb
215
• It occurs when there is superficial invasion of the chorionic villi of the placenta into the basalis layer
of the uterine wall.
• Deeper invasion of the myometrium is termed ‘placenta increta’.
• Even deeper invasion involving the serosa or adjacent pelvic organs is termed ‘placenta percreta’.
• The risk of this is catastrophic intrapartum haemorrhage at the time of placental delivery.
55. A 23 year old woman undergoes investigation for dyspareunia. Pelvic ultrasound was unremarkable. MRI
demonstrates a 1 cm thin-walled ovoid cystic lesion at the anterolateral aspect of the upper vagina. It is
homogeneously hypointense on T1 and shows marked hyperintensity on T2. What is the most likely
diagnosis?
A. Bartholin cyst
B. Nabothian cyst
C. Cervical fibroid
D. Gartner duct cyst
E. Cervical polyp
Source: Currie, Stuart, et al. SBAs for the FRCR 2A. 1st ed., Cambridge University Press, 2010.
Explanation:
Vaginal and Cervical Lesions
• Gartner’s duct cysts are remnants of mesonephric ducts and have a reported incidence of 1–2%.
• They are ovoid, thin-walled cysts located at the anterolateral aspect of the upper vagina and
generally measure less than 2 cm.
• They may contain proteinaceous material, making them slightly hyperintense on T1.
• They may be associated with Herlyn–Werner–Wunderlich syndrome (ipsilateral renal agenesis and
ipsilateral blind vagina) and ectopic ureter inserting into the cyst.
• Bartholin cysts are located at the lateral introitus adjacent to the labia minora.
• Nabothian cysts are epithelial inclusion cysts which develop in the endocervical canal and are most
commonly found in the perimenopausal period.
• Cervical fibroids and cervical polyps show mainly as solid lesions.
56. On CT perfusion, which of the following is true?
A. The CBV and MTT are decreased and CBF is increased in the infarct.
B. The CBV, CBF, and MTT are decreased in the ischemic area.
C. The CBV and CBF are decreased and MTT is increased in infarct area.
D. CBV and MTT are decreased and CBF is normal or increased in ischemic area.
Explanation: (by Perplexity AI, generated with the OpenAI O3 model)
• In the core of an infarct the cerebral blood volume (CBV) falls because capillary flow has ceased,
cerebral blood flow (CBF) is markedly reduced, and mean transit time (MTT) prolongs due to sluggish
residual flow.
• Penumbral tissue shows different patterns (often preserved CBV with low CBF and prolonged MTT),
so options describing preserved or increased CBV/CBF relate to penumbra, not established
infarction.
• Option B has increased CBF—opposite to infarct physiology; option C wrongly suggests raised CBV;
option D fits luxury reperfusion after ischemia, not the infarct core; option E describes isolated
transit delay without volume or flow loss, typical of benign oligemia rather than infarct.
EBDR Exam MCQs & Concepts September 2022
Dr. Kareem Alnakeeb
216
57. Which anatomic structure of the brain herniates in descending transtentorial (uncal) herniation?
A. Cerebellar tonsils
B. Cerebellar hemispheres
C. Cingulate gyrus
D. Uncus and hippocampus
Explanation: (by Perplexity AI, generated with the OpenAI O3 model)
• In transtentorial herniation, the cingulate gyrus is displaced downward because it sits just above
and slightly lateral to the tentorial edge.
• Descending transtentorial herniation is usually precipitated by a mass-effect lesion in the
supratentorial compartment. The medial temporal lobe—the uncus with the adjacent hippocampal
head—herniates downward over the tentorial edge, compressing the ipsilateral third cranial nerve
and midbrain.
• Cerebellar tonsils and hemispheres (A, B) are involved in tonsillar or upward cerebellar herniation,
not transtentorial descent.
• The cingulate gyrus (C) slides beneath the falx in subfalcine herniation.
58. Regarding fluoroscopically-guided air reduction for the treatment of intussusception in a child. Which of
the following is most likely to predict a successful outcome?
A. 24 hour history of symptoms.
B. Intussusception seen in the sigmoid colon on US.
C. US demonstrates fluid in the lumen around the intussusceptum.
D. US shows blood flow in the intussusceptum.
E. The child is aged >12 months.
Explanation: (by Perplexity AI, generated with the OpenAI O3 model)
• Preserved arterial and venous Doppler signals within the intussusceptum indicate that the bowel
wall remains viable and not strangulated, correlating strongly with a high success rate for pneumatic
(air) reduction.
• Fluid between the intussusceptum and intussuscipiens (option C) is more often associated with
oedema or early ischemia and predicts a lower success rate.
• A longer symptom duration of 24 hours (option A) and age over 12 months (option E) are both linked
to higher failure and perforation risk.
• If the intussusception already reaches the sigmoid colon (option B), a long segment is involved, also
reducing enema success.
59. You perform a transvaginal US and identify a pelvic mass with ground glass internal texture. Which of the
following is the most likely diagnosis?
A. Epithelial ovarian tumor
B. Endometrioma
C. Dermoid cyst
D. Corpus luteum cyst
Explanation: (by Perplexity AI, generated with the OpenAI O3 model)
• Endometriomas contain chronic hemorrhagic debris, producing uniform low-level internal echoes
that give a classic “ground-glass” appearance on ultrasound.
• Malignant epithelial tumors more often show multiloculation, solid components or papillary
excrescences.
• Dermoids display echogenic Rokitansky nodules, acoustic shadowing and fat-fluid levels rather than
homogeneous echoes.
EBDR Exam MCQs & Concepts September 2022
Dr. Kareem Alnakeeb
217
• Functional corpus luteum cysts are usually thin-walled, may have a “ring of fire” peripheral Doppler
signal, and frequently resolve within weeks, lacking the persistent ground-glass texture.
60. Which of the following is the most accurate sonographic measurement used for dating pregnancy in the
first trimester?
A. Biparietal diameter
B. Crown rump length
C. Humerus length
D. Head circumference
E. Abdominal circumference
Explanation: (by Perplexity AI, generated with the OpenAI O3 model)
• Crown-rump length (CRL) is the gold-standard biometric for dating a first-trimester pregnancy
because early embryonic growth is highly linear and shows minimal biological variation, yielding a
±3–5-day error.
• Biparietal diameter and head circumference become reliable only after about 12–13 weeks when
ossification and head shape stabilise, but they are less accurate early on.
• Long-bone (e.g. humerus) and abdominal measurements are used in the second and third
trimesters; in the first trimester limb buds and abdominal contour are too small and variable for
dependable dating.
61. A 24-year-old woman attends A&E with lower abdominal pain and vaginal bleeding. A pregnancy test is
positive. She is hemodynamically stable, and an ultrasound is requested to confirm the presumed
diagnosis of an ectopic pregnancy. Which of the following is the most common location for an ectopic
pregnancy?
A. Cervix
B. Ovary
C. Abdominal cavity
D. Ampullary portion of the Fallopian tube
E. Interstitial portion of the fallopian tube
Source: Currie, Stuart, et al. SBAs for the FRCR 2A. 1st ed., Cambridge University Press, 2010.
Explanation:
• The most common site of implantation is the fallopian tube, which accounts for over 90% of ectopic
pregnancies.
• Ovarian and abdominal sites account for only approximately 3% and 1%, respectively.
• Within the fallopian tube the most common site is the ampulla (73%) followed by the fimbrial and
interstitial regions.
62. A 25-year-old has an ultrasound at 39 weeks gestation of a singleton pregnancy. The amniotic fluid
volume is less than 500 mL. What is the most likely underlying cause for this?
A. Severe growth restriction
B. Maternal diabetes mellitus
C. Trans-oesophageal fistula (TOF)
D. Duodenal atresia
E. Cystic adenomatoid lung
Source: Proctor, Robin. Final FRCR Part A Modules 4-6 Single Best Answer MCQs: The SRT Collection
of 600 Questions with Explanatory Answers (MasterPass). 1st ed., CRC Press, 2009.
Explanation:
• Oligohydramnios is when there is less than 500 mL of amniotic fluid at term.
EBDR Exam MCQs & Concepts September 2022
Dr. Kareem Alnakeeb
218
• It is associated with a 20 times increase in fetal abnormalities and occurs with renal anomalies,
intrauterine growth restriction (IUGR) and most commonly with premature rupture of the
membranes.
• The other options are associated with polyhydramnios (amniotic fluid volume >1500-2000mL at
term).
63. In an acyanotic child with an enlarged heart and an enlarged main pulmonary artery, which is the
diagnosis?
A. Transposition of the great vessels
B. Ventricular Septal Defect (VSD)
C. Truncus arteriosis
D. TAPVR
E. Tricuspid atresia
Source: Leen, Edward. Succeeding in the New FRCR Part 2A Exam: Single Best Answer (SBA)
Revision Questions for Modules 1-6. 1st ed., UNKNO, 2011.
Explanation:
• The other options are causes of an enlarged heart in a cyanotic child with a concave main pulmonary
artery.
64. Concerning splenic trauma, which one is TRUE?
A. The spleen is injured in 35% of penetrating abdominal trauma.
B. The spleen is the second most common solid organ injured in blunt trauma.
C. Grading splenic trauma is a reliable way to predict whether a patient will need splenectomy.
D. Embolization techniques can be used to control splenic hemorrhage.
Source: ACR.
Explanation:
• Splenic artery embolisation is widely accepted for haemorrhage control in haemodynamically stable
patients and reduces splenectomy rates by preserving splenic tissue.
• Penetrating trauma much less commonly involves the spleen than the 35% quoted, making option A
incorrect.
• The spleen is actually the most frequently injured solid organ in blunt abdominal trauma, not the
second, so option B is wrong.
• CT grading correlates imperfectly with clinical course; management decisions depend on
hemodynamics rather than grade alone, so option C is false.
65. Which of the following is the most likely diagnosis of a high-density mass on head CT?
A. Lymphoma
B. Astrocytoma
C. Schwannoma
D. Paraganglioma
Explanation: (by Perplexity AI, generated with the OpenAI O3 model)
• Because of densely packed cells, lymphomas appear hyperdense on CT.
• Primary CNS lymphoma is typically hyperdense but is intraparenchymal and lacks a dural tail.
• High-grade astrocytoma is intra-axial, often heterogeneous and hypodense centrally due to necrosis.
• Vestibular schwannoma arises at the cerebellopontine angle canal and is usually isodense to brain
on non-contrast CT.
• Paraganglioma rarely occurs intracranially and more often shows marked vascularity rather than
uniform hyperdensity.
EBDR Exam MCQs & Concepts September 2022
Dr. Kareem Alnakeeb
219
66. A 26-year-old pregnant woman attends for an obstetric ultrasound at 37 weeks. She is shown to have
polyhydramnios. Which of the following would be a possible cause?
A. Cystic adenoid malformation
B. Ventricular septal defect
C. Infantile polycystic kidney disease
D. Posterior urethral valves
E. Intrauterine growth retardation
Source: Currie, Stuart, et al. SBAs for the FRCR 2A. 1st ed., Cambridge University Press, 2010.
Explanation:
• The remainder of the conditions listed above will cause oligohydramnios.
• Polyhydramnios is defined as amniotic fluid volume >1500–2000 cm3 at term.
• Most cases are due to maternal factors, with diabetes causing the majority of these.
• Oligohydramnios is defined as an amniotic fluid volume of <500 cm3 at term; the most common
causes include demise of the fetus, drugs and renal anomalies.
67. A well-circumscribed, round, 15 mm mass is identified in the breast on first-round screening
mammography. It has no associated calcification. From the following, choose the most appropriate
management:
A. Repeat mammography at the normal screening interval
B. repeat mammography in 6 months
C. MRI of the breast
D. wide local excision of the lesion
E. Ultrasound examination of the mass
Source: Bydder, Megan, et al. Get Through Final FRCR Part A: SBAs for the Modular Examination. 1st
ed., Hodder Education Publishers, 2009.
Explanation:
• Ultrasound scan is useful in determining whether mass lesions seen on the mammogram are cystic
or solid.
68. At a breast cancer multidisciplinary team meeting, the case of a 60-year-old female patient is discussed.
Following clinical examination, she is thought to have multifocal breast cancer, but this is not supported
by the ultrasound and mammography findings. Which of the following is the most appropriate next
investigation?
A. Repeat ultrasound scan
B. Repeat mammography with additional views
C. MRI
D. CT
E. 18
FDG PET
Source: Bydder, Megan, et al. Get Through Final FRCR Part A: SBAs for the Modular Examination. 1st
ed., Hodder Education Publishers, 2009.
Explanation:
Treatment Planning Implications
• Multifocal/multicentric cancer in the breast may alter treatment choice and when clinically
suspected should be investigated with MRI.
• MRI can also be used to assess the extent of residual disease in the breast after breast conservation
surgery in cases where the surgical resection margins are positive.
Recommended Breast MRI Protocol
EBDR Exam MCQs & Concepts September 2022
Dr. Kareem Alnakeeb
220
• An acceptable series of sequences for breast MRI would be: 4 mm slice-thickness, transverse, spin
echo T2W images of both breasts; 4 mm-thick, sagittal, spin echo T2W images of the affected breast;
4 mm-thick, sagittal, dynamic contrast-enhanced T1W gradient echo with fat saturation of the
affected breast; and a delayed post-contrast sequence with the same parameters.
69. A 45-year-old man is admitted after a road traffic accident in which he sustained abdominal injuries.
After fluid resuscitation he undergoes CT of the abdomen and pelvis with intravenous contrast. This
demonstrates a serpiginous area of attenuation value 130 HU at the splenic hilum with surrounding
lower-attenuation material. What is this most likely to represent?
A. active arterial extravasation
B. acute clotted blood
C. acute unclotted blood
D. splenic arterial calcification
E. ascites
Source: Bydder, Megan, et al. Get Through Final FRCR Part A: SBAs for the Modular Examination. 1st
ed., Hodder Education Publishers, 2009.
Explanation:
Attenuation Values in Haemoperitoneum Evaluation
• In the evaluation of haemoperitoneum by CT, attenuation values can help differentiate ascites,
unclotted blood, active bleeding and haematoma.
• Blood usually has a higher measured attenuation than other body fluids, but its appearance depends
on the age, extent and location of haemorrhage.
• Unclotted blood has an attenuation value of 30–45 HU, but this may be lower in patients with a lower
serum haematocrit and if the haemorrhage is more than 48 hours old.
• Clotted blood has an attenuation value of 45–70 HU, and identification of the area of highest
attenuation haematoma (sentinel clot) on CT indicates the site of bleeding.
• Active arterial extravasation is seen as an area of higher attenuation resembling that in the aorta,
ranging from 85 HU to 370 HU.
• It may be surrounded by lower-attenuation haematoma.
• This finding indicates the need for urgent embolization or surgical treatment.
70. A 48-year-old female non-smoker presents to the Accident & Emergency Department with acute
dyspnoea and chest pain. The chest radiograph shows bilateral basal airspace shadowing. Chest CT
shows diffuse basal consolidation and air-bronchograms within a background of ground-glass opacity.
There is septal thickening and bilateral pleural effusions. The most likely diagnosis is?
A. Desquamative interstitial pneumonitis
B. Lymphocytic interstitial pneumonitis
C. Acute interstitial pneumonia
D. Usual interstitial pneumonitis
E. Cryptogenic organising pneumonia
Source: Gupta, Chaitanya. 300 Single Best Answers for the Final FRCR Part A. 1st ed., Jaypee UK,
2010.
Explanation:
• This clinically presents as adult respiratory distress syndrome and has high mortality.
• It has a fulminant course leading to respiratory failure and requiring mechanical ventilation with a
mortality of > 50%.
• CT findings are non-specific but include bilateral, diffuse ground-glass opacity with consolidation
and air bronchograms.
EBDR Exam MCQs & Concepts September 2022
Dr. Kareem Alnakeeb
221
• Honeycombing and traction bronchiectasis may be seen in advanced cases after recovery.
71. A 24-year-old male patient presents following a head injury with GCS of 13. There is bruising over the right
temporal region. A CT scan shows no intracranial haemorrhage but does identify a longitudinal fracture
through the petrous temporal bone. What complication should be considered?
A. sensorineural hearing loss
B. conductive hearing loss
C. vertigo
D. carotid artery injury
E. sigmoid sinus injury
Source: Bydder, Megan, et al. Get Through Final FRCR Part A: SBAs for the Modular Examination. 1st
ed., Hodder Education Publishers, 2009.
Explanation:
Temporal Bone Fracture Types
• Longitudinal fractures of the temporal bone represent 75% of temporal bone fractures and run
parallel to the axis of the petrous pyramid.
• Transverse fractures of the temporal bone are associated with sensorineural hearing loss and vertigo.
Auditory Consequences
• Longitudinal fractures may cause dislocation of the auditory ossicles, usually the incus, causing a
conductive deafness.
• Sensorineural hearing loss is associated with transverse fractures of the temporal bone, as is vertigo.
Facial Nerve Involvement
• Facial nerve palsy is seen in both fracture types, but is less common in longitudinal fractures, where
it frequently recovers spontaneously.
Vascular Considerations
• Carotid artery and major sinus injuries are not directly associated with petrous temporal fractures.
72. In a patient presenting with a Le Fort fracture following facial trauma, which bone is almost invariably
fractured as part of the injury pattern?
A. Zygomatic arch
B. Frontal sinus wall
C. Nasal septum
D. Pterygoid plates
E. Lacrimal bone
Explanation: (by Perplexity AI, generated with the OpenAI O3 model)
• Le Fort fractures are defined by the involvement of the pterygoid plates—fracture lines must traverse
these plates for the injury to qualify as any Le Fort type.
• Their posterior location anchors the mid-face to the skull base; disruption allows displacement of
the maxillary complex.
• Other facial bones may be involved variably: the zygomatic arch (A) and frontal sinus wall (B) are not
required components, nasal septum (C) may be spared, and the thin lacrimal bone (E) is
inconsistently affected.
73. Concerning the LeFort classification of facial injuries, which one is TRUE?
A. All types involve the pterygoid plates and nasal region.
B. All types involve the orbital floors and zygomas.
C. The Type III injury is characterized by a free-floating palate (trans-maxillary fracture).
D. The type II injury is characterized by cranial-facial dissociation.
EBDR Exam MCQs & Concepts September 2022
Dr. Kareem Alnakeeb
222
Source: ACR.
Explanation:
• In the Le Fort system every pattern (I, II and III) must include bilateral fractures of the pterygoid plates
and traverse the nasal region—this unites them as mid-face fractures.
• Option B is incorrect because only Le Fort II and III extend through the orbital floors and involve the
zygomas; Le Fort I is limited below the nasal floor.
• Option C misattributes the “free-floating palate” feature, which actually defines Le Fort I, not III.
• Option D confuses Le Fort II with Le Fort III; cranio-facial (orbital-zygomatic) separation is the
hallmark of Le Fort III.
74. In an infant suspected of hypertrophic pyloric stenosis, which sonographic measurement is diagnostic of
the condition?
A. Pyloric canal length of 19 mm
B. Transverse pyloric diameter of 11 mm
C. Pyloric muscle thickness of 2 mm
D. Pyloric canal length of 7 mm
E. Transverse pyloric diameter of 6 mm
Explanation: (by Perplexity AI, generated with the OpenAI O3 model)
• Hypertrophic pyloric stenosis is confirmed on ultrasound when the pyloric canal measures ≥15–17
mm in length and the muscle wall is ≥3 mm thick; a length of 19 mm therefore fulfils diagnostic
criteria.
• A transverse diameter of 11 mm can be seen in normal infants and is not specific.
• Muscle thickness of 2 mm and canal length of 7 mm both lie within normal limits, while a diameter of
6 mm is clearly normal, making these distractors incorrect.
75. A six week old child has an ultrasound scan of the abdomen performed for non-bilious projectile
vomiting. Which one of the following features would support a diagnosis of infantile pylorospasm over a
diagnosis of hypertrophic pyloric stenosis?
A. Pyloric muscle wall thickness of 2 mm
B. Pyloric canal length of 19 mm
C. Target sign
D. Antral nipple sign
E. Transverse pyloric diameter of 14 mm
Source: Currie, Stuart, et al. SBAs for the FRCR 2A. 1st ed., Cambridge University Press, 2010.
Explanation:
Clinical Presentation
• Hypertrophic pyloric stenosis presents between four and six weeks of life with non-bilious vomiting,
typically in first-born males.
Physical Examination
• A palpable olive-shaped mass is a sign with reported sensitivity of up to 80%, but ultrasound is the
most frequently used imaging modality.
Ultrasound Characteristics
• Typical ultrasound features include the target sign (central hyperechoic mucosa with surrounding
hypoechoic pyloric muscle), the nipple sign (pyloric mucosa indenting the gastric antrum), pyloric
canal length >16 mm, transverse pyloric diameter >13 mm and pyloric muscle wall thickness >3 mm.
Differential Diagnosis
• Pyloric stenosis can be difficult to differentiate radiologically from infantile pylorospasm.
EBDR Exam MCQs & Concepts September 2022
Dr. Kareem Alnakeeb
223
• Typically with pylorospasm the appearances change with time, and so if the pyloric muscle thickness
is measured at less than 3 mm this makes infantile pylorospasm the more likely diagnosis.
76. A 4-week-old male neonate presents with nonbilious vomiting and a hypochloraemic alkalosis.
Hypertrophic pyloric stenosis is suspected and an ultrasound is performed. Which one of the following
ultrasound findings would confirm the diagnosis?
A. A pylorus that does not open
B. Pyloric canal length of greater than 11 mm
C. Pyloric muscle wall thickness of 1 mm
D. Reduced gastric peristalsis
E. Transverse pyloric diameter of greater than 11 mm.
Source: McQueen, Andrew S., et al. Grainger & Allison’s Diagnostic Radiology: Single Best Answer MCQs.
5th ed., Churchill Livingstone, 2009.
Explanation:
• Ultrasound criteria include canal length >16 mm, transverse pyloric diameter >11 mm, muscle wall
thickening > 2.5 mm and increased gastric motility.
• A pylorus that does not open is associated with hypertrophic stenosis; however, it may also be seen
in pylorospasm.
77. A 26-year-old patient with a positive B- HCG undergoes pelvic ultrasound examination. Which finding on
ultrasound is most likely to indicate a nonviable pregnancy?
A. The intradecidual sign
B. Non-visualisation of cardiac activity when crown-rump length (CRL) is 7mm
C. Visualisation of the yolk sac when the gestational sac is 8 mm
D. Gestational sac present at 32 days
E. Asymmetry of the echogenic ring surrounding the gestational sac at five weeks.
Source: Proctor, Robin. Final FRCR Part A Modules 4-6 Single Best Answer MCQs: The SRT Collection
of 600 Questions with Explanatory Answers (MasterPass). 1st ed., CRC Press, 2009.
Explanation:
• At 6.5 weeks the CRL is approximately 5 mm and cardiac movement can be identified.
• The intradecidual sign is seen in intrauterine pregnancy.
• The gestational sac can be seen at the fundus from five weeks and is surrounded by an echogenic
ring which can be asymmetric.
• The yolk sac is seen at approximately five to seven weeks when the gestational sac is 6-9 mm.
78. A 63 year old man with an elevated PSA is diagnosed with prostate cancer following needle biopsy. He
undergoes an MRI examination to help stage his disease. Which of the following sequences would be
most helpful in identifying location and local extent of the tumour?
A. T1-weighted
B. T1-weighted with gadolinium
C. T2-weighted
D. Fat-suppressed T1-weighted
E. Fat-suppressed T2-weighted
Explanation: (by Perplexity AI, generated with the OpenAI O3 model)
• Prostate cancer shows low signal intensity against the normally high-signal peripheral zone on high-
resolution T2-weighted images, allowing clear delineation of tumour focus, assessment of capsular
bulge, neurovascular bundle involvement, and seminal vesicle invasion.
EBDR Exam MCQs & Concepts September 2022
Dr. Kareem Alnakeeb
224
• T1-weighted sequences (with or without gadolinium) lack soft-tissue contrast for zonal anatomy;
gadolinium adds little for local staging. Fat suppression on T1 or T2 degrades prostate-parenchymal
contrast and is mainly used for post-biopsy haemorrhage detection or whole-body marrow imaging,
not for primary lesion mapping.
79. On an erect chest radiograph, what is the approximate minimum volume of intraperitoneal free air that
can usually be detected beneath the diaphragm?
A. 100 ml
B. 5 ml
C. 250 ml
D. 500 ml
E. 1,000 ml
Explanation: (by Perplexity AI, generated with the OpenAI O3 model)
• As little as about 100 ml of intraperitoneal gas will typically outline the underside of the
hemidiaphragm on an erect chest X-ray, creating the classic sub-diaphragmatic crescent.
• Smaller volumes (≈5 ml) are generally below the threshold of plain-film detection and instead require
more sensitive techniques such as computed tomography.
• Larger volumes—250 ml or more—are obviously visible; however, waiting for that amount risks
delayed diagnosis.
• Therefore, 100 ml represents the accepted lower limit on standard erect radiographs, making option
A correct, while options B, C, D and E are respectively too low or unnecessarily high.
80. A runner presents with pain in the third web space of the foot; ultrasound confirms a Morton neuroma.
Which nerve is characteristically involved?
A. Common plantar digital nerve
B. Deep peroneal nerve
C. Lateral plantar nerve
D. Medial plantar nerve
E. Sural nerve
Explanation: (by Perplexity AI, generated with the OpenAI O3 model)
• Morton neuroma is a perineural fibrosis affecting the common plantar digital nerve as it passes
beneath the intermetatarsal ligament, most frequently between the third and fourth metatarsal
heads.
• The deep peroneal nerve (B) supplies the first web space on the dorsum and is not implicated.
• The lateral plantar nerve (C) divides into proper and common plantar branches more laterally and
does not traverse the usual site of a Morton neuroma.
• The medial plantar nerve (D) courses medially and is seldom affected.
• The sural nerve (E) runs along the lateral foot and is unrelated to intermetatarsal neuromas.
81. A 48-year-old man with previous pulmonary tuberculosis presents with life-threatening massive
haemoptysis. Bronchoscopy localises bleeding to the right upper lobe and urgent endovascular
treatment is planned. Which arterial system should be selectively catheterised and assessed first during
the embolisation procedure?
A. Pulmonary arteries
B. Internal mammary arteries
C. Superior thyroid arteries
D. Bronchial arteries
E. Subclavian arteries
EBDR Exam MCQs & Concepts September 2022
Dr. Kareem Alnakeeb
225
Explanation: (by Perplexity AI, generated with the OpenAI O3 model)
• The vast majority (≈90%) of massive haemoptysis arises from hypertrophied bronchial arteries
because post-tuberculous inflammation increases their calibre and pressure; therefore, these
vessels are the primary target for embolisation.
• Pulmonary arterial bleeding is much less common and usually lower pressure, so pulmonary
angiography is reserved for persistent haemorrhage after negative bronchial studies.
• Internal mammary, superior thyroid and subclavian arteries can supply non-bronchial systemic
collaterals but are secondary considerations explored only if bronchial angiography is inconclusive
or bleeding recurs.
82. Where along its course is the Achilles tendon most commonly ruptured in adults presenting with an
acute tear?
A. 0–2 cm proximal to its calcaneal insertion
B. 2–6 cm proximal to its calcaneal insertion
C. At the calcaneal insertion itself
D. In the mid‐substance more than 6 cm from the calcaneus
E. At the musculotendinous junction with gastrocnemius
Explanation: (by Perplexity AI, generated with the OpenAI O3 model)
• The typical site of Achilles tendon rupture lies 2–6 cm above its calcaneal insertion, corresponding to
a relatively hypovascular “watershed” zone that predisposes the tendon to degeneration and tearing.
• Ruptures directly at the insertion (A) are uncommon and usually reflect enthesopathic disease;
complete avulsions from bone (C) remain rare.
• Tears occurring more proximally within the mid‐substance beyond 6 cm (D) or at the
musculotendinous junction (E) are seen mainly in younger athletes after severe trauma but represent
a minority of cases.
83. On a contrast-enhanced CT, a mantle of homogeneous soft-tissue density is seen encasing the
abdominal aorta at the level of L4–L5 and medially displacing both mid-ureters, resulting in bilateral mild
hydronephrosis. Which diagnosis best explains these findings?
A. Abdominal aortic aneurysm with mural thrombus
B. Malignant lymphoma
C. Idiopathic retroperitoneal fibrosis
D. Paraganglioma of the organ of Zuckerkandl
E. Metastatic peri-aortic lymphadenopathy
Explanation: (by Perplexity AI, generated with the OpenAI O3 model)
• Idiopathic retroperitoneal fibrosis classically presents as a plaque-like soft-tissue rind that envelops
the distal aorta and common iliac arteries without anterior displacement of the vessels and pulls the
ureters medially, causing obstructive uropathy.
• Lymphoma and metastatic nodes typically form discrete nodal masses that displace the aorta
anteriorly and do not characteristically medialise the ureters.
• An aortic aneurysm with thrombus demonstrates a dilated aortic lumen surrounded by thrombus
rather than a periaortic mantle.
• Paraganglioma appears as an avidly enhancing focal mass at the aortic bifurcation, not a longitudinal
plaque.
EBDR Exam MCQs & Concepts September 2022
Dr. Kareem Alnakeeb
226
84. A CT scan shows a heterogeneously enhancing liver mass with a tumour thrombus extending from the
hepatic vein into the inferior vena cava and up to the right atrium. Which primary malignancy most
commonly produces this pattern of intravascular spread?
A. Pancreatic ductal adenocarcinoma
B. Hepatocellular carcinoma
C. Intrahepatic cholangiocarcinoma
D. Adrenocortical carcinoma
E. Renal cell carcinoma
Explanation: (by Perplexity AI, generated with the OpenAI O3 model)
• Hepatocellular carcinoma (HCC) has a well-recognised propensity to invade hepatic veins,
progressing into the inferior vena cava and occasionally the right atrium; this occurs in 1.4–4.9% of
HCC cases and is a classic imaging pattern.
• Pancreatic adenocarcinoma and cholangiocarcinoma more often involve the portal or spleno-
mesenteric veins, while direct caval thrombus reaching the atrium is exceptional.
• Renal and adrenocortical carcinomas can extend up the IVC but typically arise below the liver and do
not involve hepatic veins. Therefore, the described cranio-caudal tumour thrombus originating in a
hepatic lesion most strongly indicates HCC.
85. On ultrasound, which description best matches the appearance of the central scar within a focal nodular
hyperplasia (FNH) lesion?
A. Markedly hyperechoic linear focus
B. Homogeneously isoechoic to surrounding liver
C. Irregular anechoic cavity with posterior enhancement
D. Mildly hypoechoic focus compared with adjacent parenchyma
E. Prominent calcified echogenic focus with shadowing
Explanation: (by Perplexity AI, generated with the OpenAI O3 model)
• FNH often contains a central fibrous scar. On grayscale ultrasound the scar is usually mildly
hypoechoic relative to the surrounding liver, reflecting its fibrous composition and lower cellularity,
while the remainder of the lesion may be isoechoic or slightly hyperechoic.
• It is seldom brightly echogenic (ruling out A) and not typically the same echogenicity as liver (B).
• It is solid rather than cystic, so an anechoic cavity with through-transmission (C) is incorrect.
• Calcification and acoustic shadowing are rare in FNH scars, eliminating option E.
86. A 3-year-old boy presents with abdominal distension and raised serum α-fetoprotein. Contrast-
enhanced CT of the liver shows a solitary right-lobe mass with heterogeneous arterial enhancement and
patchy calcification. What is the most likely diagnosis?
A. Focal nodular hyperplasia
B. Hepatoblastoma
C. Hepatocellular carcinoma
D. Mesenchymal hamartoma
E. Metastatic neuroblastoma
Explanation: (by Perplexity AI, generated with the OpenAI O3 model)
• Hepatoblastoma is the commonest primary malignant liver tumour in children under 5 years. It
typically manifests as a solitary right-lobe mass showing heterogeneous arterial enhancement due to
mixed epithelial and mesenchymal elements, often containing calcifications and markedly elevating
α-fetoprotein.
EBDR Exam MCQs & Concepts September 2022
Dr. Kareem Alnakeeb
227
• Hepatocellular carcinoma is rarer at this age and usually arises in cirrhotic livers.
• Focal nodular hyperplasia is exceptional in toddlers and shows a central scar rather than
calcification.
• Mesenchymal hamartoma occurs in infants, has cystic components and does not elevate α-
fetoprotein.
• Metastatic neuroblastoma may calcify but classically presents with multiple subcapsular lesions
and normal α-fetoprotein.
87. A 25-year-old woman on long-term combined oral contraceptive pills presents with acute right-upper-
quadrant pain. CT shows a solitary, well-circumscribed 8 cm hepatic mass containing areas of high
attenuation consistent with intratumoural haemorrhage. Which is the most likely diagnosis?
A. Focal nodular hyperplasia
B. Hepatocellular carcinoma
C. Hepatic adenoma
D. Cavernous haemangioma
E. Hepatic metastasis
Explanation: (by Perplexity AI, generated with the OpenAI O3 model)
• Oral-contraceptive–related hepatic adenomas typically occur in young women and are prone to
haemorrhage, so a painful, hyperattenuating lesion on CT strongly favours this diagnosis.
• Focal nodular hyperplasia usually has a central scar and rarely bleeds.
• Hepatocellular carcinoma is uncommon in a healthy young woman without cirrhosis and often
shows arterial wash-out rather than isolated haemorrhage.
• Cavernous haemangiomas demonstrate peripheral nodular enhancement with centripetal fill-in and
seldom rupture.
• Metastases are unlikely when there is a single lesion with haemorrhage and no known primary.
88. On MRCP, which imaging sign best differentiates a congenital choledochal cyst from a common bile duct
dilatation caused by downstream obstruction?
A. Abrupt irregular cutoff at the distal common bile duct
B. Beaded appearance of the intra-hepatic ducts
C. Smooth, gradual tapering between dilated and normal duct calibre
D. Mural nodularity within the dilated segment
E. Multiple filling defects within the dilated duct
Explanation: (by Perplexity AI, generated with the OpenAI O3 model)
• Choledochal cysts are true congenital dilatations of the extra-hepatic bile duct. They
characteristically merge with the normal calibre duct distally via a smooth, gradual (conical) taper,
reflecting unobstructed bile flow.
• In contrast, malignant or calculous obstruction produces an abrupt irregular cutoff (option A) or
meniscoid/step-off appearance, not a taper.
• Sclerosing cholangitis gives a beaded contour (option B). Mural nodularity (option D) suggests
cholangiocarcinoma.
• Multiple intraductal filling defects (option E) are typical of choledocholithiasis or sludge rather than a
congenital cyst.
EBDR Exam MCQs & Concepts September 2022
Dr. Kareem Alnakeeb
228
89. A newborn boy presents with a markedly distended, wrinkled abdomen, bilateral non-palpable testes and
ultrasound evidence of severe bilateral hydroureteronephrosis with a massively dilated bladder. Which
diagnosis best explains this triad of findings?
A. Abdominal compartment syndrome
B. Necrotising enterocolitis
C. Posterior urethral valves
D. Prune-belly (Eagle-Barrett) syndrome
E. Pseudo-Prune-belly syndrome
Explanation: (by Perplexity AI, generated with the OpenAI O3 model)
• Prune-belly syndrome is defined by the classic triad of deficient anterior abdominal wall musculature
(producing a lax, “prune-like” abdomen), profound urinary tract dilatation/obstruction and bilateral
cryptorchidism. The combination of wrinkled abdominal skin, megacystis with
hydroureteronephrosis and undescended testes therefore points directly to this diagnosis.
• Posterior urethral valves (C) cause bladder outlet obstruction but do not account for absent
abdominal muscles or cryptorchidism.
• Abdominal compartment syndrome (A) produces a tense, not lax, abdomen and lacks urogenital
anomalies.
• Necrotising enterocolitis (B) manifests with pneumatosis and sepsis rather than urinary findings.
• Pseudo-Prune-belly syndrome (E) shows partial or unilateral abdominal wall deficiency and usually
lacks the full triad.
90. A neonate fails a nasogastric tube-passing test; the tube coils in the upper mediastinum on radiograph
and a gas-filled stomach is visible. What is the most likely diagnosis?
A. Pure oesophageal atresia without fistula
B. Oesophageal atresia with distal tracheo-oesophageal fistula
C. Oesophageal atresia with proximal tracheo-oesophageal fistula
D. H-type tracheo-oesophageal fistula without atresia
E. Congenital oesophageal stenosis
Explanation: (by Perplexity AI, generated with the OpenAI O3 model)
• Coiling of the tube in a blind-ending upper pouch indicates oesophageal atresia. The presence of
gastric bubble signifies air passage into the stomach via a distal tracheo-oesophageal fistula, making
the classic type C lesion (atresia with distal fistula) most likely.
• Pure atresia (A) shows no gas in the abdomen.
• A proximal fistula (C) would not aerate the stomach.
• An isolated H-type fistula (D) has no atresia, so the tube reaches the stomach.
• Congenital stenosis (E) allows tube passage, though with resistance.
91. An 11-day-old jaundiced infant has persistent pale stools and conjugated hyperbilirubinaemia.
Ultrasound shows an absent gallbladder and a 4 mm echogenic “triangular cord” anterior to the portal
vein bifurcation. What is the most likely diagnosis?
A. Neonatal hepatitis
B. Choledochal cyst
C. Biliary atresia
D. Alagille syndrome
E. Inspissated bile syndrome
EBDR Exam MCQs & Concepts September 2022
Dr. Kareem Alnakeeb
229
Explanation: (by Perplexity AI, generated with the OpenAI O3 model)
• The echogenic “triangular cord” sign represents fibrotic residual bile duct tissue at the porta hepatis
and, together with a non-visualised or abnormal gallbladder, is highly specific for biliary atresia.
• Early recognition is critical because Kasai porto-enterostomy before 6 weeks markedly improves
outcome.
• Neonatal hepatitis, Alagille syndrome and inspissated bile may produce cholestasis but lack the
triangular cord and typically show a normal-sized gallbladder.
• Choledochal cysts present with cystic biliary dilatation, not an absent gallbladder with fibrotic
remnant.
92. A 6-week-old boy presents with persistent jaundice and hepatosplenomegaly. Ultrasound shows a
normal-appearing gallbladder and a triangular fibrotic echogenic band at the porta hepatis (the
“triangular cord” sign). Which is the most likely underlying diagnosis?
A. Alagille syndrome
B. Biliary atresia
C. Neonatal hepatitis secondary to CMV infection
D. Choledochal cyst (type I)
E. Alpha-1-antitrypsin deficiency
Explanation: (by Perplexity AI, generated with the OpenAI O3 model)
• The echogenic “triangular cord” anterior to the portal vein represents fibrotic remnants of the extra-
hepatic bile duct and is highly specific for biliary atresia.
• Although some infants with biliary atresia can have a small or even normal-looking gallbladder, the
porta hepatis fibrotic band clinches the diagnosis.
• Alagille syndrome may show a small or absent gallbladder but lacks the triangular cord and is usually
accompanied by characteristic facies and cardiac anomalies.
• Neonatal hepatitis (including CMV) and alpha-1-antitrypsin deficiency cause diffuse parenchymal
liver disease without an echogenic hilar band.
• A choledochal cyst would demonstrate a cystic dilatation of the common bile duct rather than a solid
fibrotic band.
93. In an adult patient with a fusiform dilatation of the extra-hepatic common bile duct on MRCP, which
Todani classification type of choledochal cyst is most likely?
A. Type IA
B. Type IB
C. Type IC
D. Type II
E. Type III
Explanation: (by Perplexity AI, generated with the OpenAI O3 model)
• Todani type IC describes a long, smooth, fusiform dilatation that involves the entire extra-hepatic bile
duct, often extending up to the common hepatic duct; this imaging pattern fits the vignette.
o Type IA appears as a saccular (cystic) dilatation of the CBD, not fusiform.
o Type IB is a segmental, focal outpouching of the distal CBD.
o Type II is a true diverticulum of the extra-hepatic duct.
o Type III (choledochocele) is located within the duodenal wall. Hence these alternatives do
not match the described fusiform morphology.
EBDR Exam MCQs & Concepts September 2022
Dr. Kareem Alnakeeb
230
94. On MRI, a cystic dilatation of the segment 2/3 intra-hepatic ducts shows a tiny enhancing “central dot”
with smooth tapering of the extra-hepatic bile duct. Which diagnosis best fits these imaging findings?
A. Primary sclerosing cholangitis
B. Recurrent pyogenic cholangitis
C. Choledochal cyst type I
D. Caroli disease
E. Klatskin (hilar) cholangiocarcinoma
Explanation: (by Perplexity AI, generated with the OpenAI O3 model)
• The “central dot sign” represents an enhancing portal vein radicle within a dilated intra-hepatic duct
and, along with smooth, unobstructed extra-hepatic ducts, is characteristic of Caroli disease—a
congenital ductal-plate malformation causing saccular or beaded ectasia of intra-hepatic bile ducts.
• Primary sclerosing cholangitis (A) and recurrent pyogenic cholangitis (B) produce multifocal
strictures without the central dot.
• Choledochal cyst type I (C) causes fusiform dilatation of the common bile duct, not intra-hepatic
cystic ectasia with a dot.
• A Klatskin tumour (E) gives irregular biliary obstruction and shouldering, not smooth tapering or a
central dot.
95. A 27-year-old woman is incidentally found to have a solitary 3 cm hepatic lesion on ultrasound. Triple-
phase CT shows intense homogeneous arterial enhancement becoming isodense to liver on portal
venous phase with a central scar that demonstrates delayed enhancement. Which is the most likely
diagnosis?
A. Hepatocellular adenoma
B. Fibrolamellar hepatocellular carcinoma
C. Focal nodular hyperplasia
D. Hypervascular metastasis
E. Cavernous haemangioma
Explanation: (by Perplexity AI, generated with the OpenAI O3 model)
• Focal nodular hyperplasia classically presents in young women as a well-circumscribed lesion that
shows brisk, uniform arterial phase enhancement and remains isoattenuating on the portal venous
phase; a central scar with delayed contrast uptake is typical.
• Hepatocellular adenoma (A) enhances heterogeneously and often contains fat or haemorrhage with
no delayed scar enhancement.
• Fibrolamellar HCC (B) may have a scar but usually occurs in adolescents/young adults of either sex,
shows heterogeneous signal, and the scar does not enhance early or may calcify.
• Hypervascular metastases (D) are usually multiple and demonstrate wash-out on portal/delayed
phases.
• Cavernous haemangioma (E) shows peripheral nodular discontinuous enhancement with
progressive centripetal fill-in, not a central scar.
EBDR Exam MCQs & Concepts September 2022
Dr. Kareem Alnakeeb
231
96. On a contrast-enhanced CT performed in the portal-venous phase, which pancreatic tumour classically
appears as a focal low-attenuation mass within the pancreatic head causing upstream duct dilatation?
A. Pancreatic neuroendocrine tumour
B. Solid pseudopapillary neoplasm
C. Pancreatic ductal adenocarcinoma
D. Serous cystadenoma
E. Intraductal papillary mucinous neoplasm
Explanation: (by Perplexity AI, generated with the OpenAI O3 model)
• Pancreatic ductal adenocarcinoma typically presents on portal-venous phase CT as a hypovascular
(low-attenuation) mass relative to the enhancing pancreatic parenchyma, most frequently in the
head, and often results in “double-duct” sign from obstruction of the pancreatic and common
bile ducts, making C correct.
• Neuroendocrine tumours (A) are usually hypervascular and enhance avidly, not low attenuation.
• Solid pseudopapillary neoplasms (B) show mixed cystic-solid components with
variable enhancement, often in young women.
• Serous cystadenomas (D) are microcystic, giving a honeycomb appearance rather than solid low
attenuation.
• Intraductal papillary mucinous neoplasms (E) are cystic lesions arising from or communicating with
the pancreatic ducts and are not solid hypovascular masses.
97. A full-term neonate presents with a midline abdominal wall defect covered by a membrane at the
umbilicus. Which associated abnormality is most frequently seen with this condition?
A. Meckel diverticulum
B. Malrotation
C. Trisomy 18
D. Duodenal atresia
E. Hirschsprung disease
Explanation: (by Perplexity AI, generated with the OpenAI O3 model)
• Omphalocele is a midline, membrane-covered herniation at the umbilical ring.
• Roughly 50% of affected infants have additional anomalies, and chromosomal disorders—especially
trisomies 13 and 18—are the single commonest group.
• Trisomy 18 is classically quoted as the most frequent association, often co-existing with cardiac and
other systemic defects.
• Gastrointestinal malformations such as malrotation, Meckel diverticulum, duodenal atresia, and
Hirschsprung disease do occur but each individually accounts for a much smaller proportion of
cases, making them less likely than a chromosomal trisomy in an unknown neonate with
omphalocele.
98. On CT adrenal wash-out studies, which statement regarding typical lipid-rich adrenal cortical adenomas
is MOST accurate?
A. They are less common than malignant adrenal tumours
B. More than 90% of adenomas still measure >30 HU at 10-min delayed imaging
C. Around 70% demonstrate >60% absolute contrast wash-out by 10 min
D. Fewer than 20% contain intracellular lipid giving ≤10 HU on non-contrast CT
E. Most show progressive enhancement rather than wash-out on delayed scans
EBDR Exam MCQs & Concepts September 2022
Dr. Kareem Alnakeeb
232
Explanation: (by Perplexity AI, generated with the OpenAI O3 model)
• Adrenal cortical adenomas are common incidental findings and classically enhance rapidly then
lose contrast quickly.
• Approximately two-thirds to three-quarters achieve an absolute contrast wash-out ≥60% within 10–
15 min, a highly specific benign feature.
• Malignant lesions usually wash-out much more slowly.
• Adenomas are far more common than primary or secondary malignant adrenal masses, making
option A incorrect.
• Over 80% of adenomas fall to <30 HU by 10 min, not remain above it, so option B is false.
• About 70% are lipid-rich with ≤10 HU on unenhanced CT, not fewer than 20%, invalidating option D.
• Adenomas do not show progressive enhancement; they wash-out, so option E is wrong.
99. On grayscale ultrasound, which description most closely matches the typical appearance of a
hepatocellular carcinoma (HCC) in a cirrhotic liver?
A. Small, well-defined, uniformly hyperechoic nodule
B. Ill-defined, heterogeneous, predominantly hypoechoic mass
C. Round, sharply marginated, anechoic lesion with posterior enhancement
D. Homogeneous, isoechoic lesion containing a central stellate scar
E. Well-circumscribed, heterogeneously echogenic mass with a hypoechoic rim seen only in non-
cirrhotic young women
Explanation: (by Perplexity AI, generated with the OpenAI O3 model)
• HCCs usually arise in a background of cirrhosis and, on standard B-mode ultrasound, classically
appear as ill-defined, heterogeneous, predominantly hypoechoic masses; larger lesions often show
mixed echogenicity because of necrosis or hemorrhage.
• Option A describes a small hyperechoic nodule more typical of a hemangioma.
• Option C fits a simple cyst.
• Option D suggests focal nodular hyperplasia, characterised by a central scar.
• Option E corresponds to hepatic adenoma, typically occurring in young women on the oral
contraceptive pill, appearing as a well-circumscribed heterogeneous lesion that may have a
surrounding hypoechoic halo but generally in a non-cirrhotic liver.
100. In a neonate with bilateral hydronephrosis, ultrasound shows marked tapering of the distal 1–2 cm of
each ureter just before the vesicoureteric junction with proximal ureteric dilatation; what is the most
likely underlying diagnosis?
A. Vesicoureteric reflux
B. Posterior urethral valves
C. Primary obstructive megaureter
D. Ureterocele within the bladder
E. Prune-belly syndrome
Explanation: (by Perplexity AI, generated with the OpenAI O3 model)
• Primary obstructive megaureter is characterised by an intrinsically aperistaltic or stenotic segment in
the distal 1–2 cm of the ureter immediately proximal to the bladder, producing a smooth tapered
lower third and upstream dilatation.
• Vesicoureteric reflux (A) shows refluxing contrast without a true distal taper and often has a non-
dilated ureter when the bladder is empty.
EBDR Exam MCQs & Concepts September 2022
Dr. Kareem Alnakeeb
233
• Posterior urethral valves (B) cause bladder-outlet obstruction with a dilated, trabeculated bladder
and dilated posterior urethra rather than a focal distal ureteric narrowing.
• A ureterocele (D) appears as a cystic filling defect inside the bladder, not as tapering of the distal
ureter.
• Prune-belly syndrome (E) presents with flaccid, massively dilated ureters and bladder but lacks the
short, narrowed intramural segment.
101. A 52-year-old woman presents with progressive exertional breathlessness and intermittent dizziness.
Transthoracic echocardiography demonstrates a mobile, echogenic mass attached by a narrow stalk to
the interatrial septum, prolapsing through the mitral valve during diastole. What is the most likely
diagnosis?
A. Papillary fibroelastoma
B. Cardiac myxoma
C. Thrombus in transit
D. Rhabdomyoma
E. Metastatic melanoma deposit
Explanation: (by Perplexity AI, generated with the OpenAI O3 model)
• Cardiac myxomas are the commonest primary cardiac tumours in adults and typically arise from the
fossa ovalis region of the interatrial septum, attached by a pedicle; their mobility can cause
obstruction of the mitral inflow, explaining positional dyspnoea and syncope.
• Papillary fibroelastomas usually originate from valvular endocardium rather than the septum.
• An intracardiac thrombus lacks a discrete stalk and is less likely to be highly mobile on a narrow
pedicle.
• Rhabdomyomas predominate in infants and involve ventricular myocardium.
• Metastatic melanoma deposits are seldom pedunculated and usually present as multiple masses or
pericardial involvement.
102. A 16-year-old boy with lifelong short stature presents for hand radiographs, which show bilateral
shortening of the fourth and fifth metacarpals. Routine biochemistry reveals hypocalcaemia,
hyperphosphataemia and markedly elevated parathyroid hormone levels. Which single diagnosis best
explains this combination of skeletal and biochemical findings?
A. Idiopathic hypoparathyroidism
B. Pseudohypoparathyroidism (Albright hereditary osteodystrophy)
C. Primary hyperparathyroidism
D. Renal osteodystrophy secondary to chronic kidney disease
E. Vitamin-D–dependent rickets
Explanation: (by Perplexity AI, generated with the OpenAI O3 model)
• Shortened fourth and fifth metacarpals together with short stature and other skeletal stigmata
constitute the Albright hereditary osteodystrophy phenotype.
• When this phenotype co-exists with high PTH, low calcium and high phosphate, the underlying
problem is end-organ resistance to PTH—pseudohypoparathyroidism type 1a.
• Idiopathic hypoparathyroidism (A) shows low or absent PTH, not elevated.
• Primary hyperparathyroidism (C) produces high calcium and low phosphate.
• Renal osteodystrophy (D) causes secondary hyperparathyroidism but typically shows high
phosphate with normal metacarpals and no Albright features.
EBDR Exam MCQs & Concepts September 2022
Dr. Kareem Alnakeeb
234
• Vitamin-D–dependent rickets (E) presents with rachitic bone changes and low–normal phosphate,
not the classic metacarpal shortening.
103. In a middle-aged patient with a vividly enhancing extra-axial mass abutting the falx on contrast MRI (no
image shown), which imaging feature most reliably favours a haemangiopericytoma over a meningioma?
A. Presence of a dural tail
B. Hyperostotic bone reaction of the adjacent calvarium
C. Irregular bone erosion of the inner table
D. Calcification within the lesion on CT
E. Broad-based dural attachment
Explanation: (by Perplexity AI, generated with the OpenAI O3 model)
• Haemangiopericytomas are aggressive, vascular tumours that tend to erode adjacent skull with
scalloped or irregular lytic defects, whereas meningiomas more often incite hyperostosis or leave the
bone intact.
• A dural tail and broad dural base are classic but non-specific signs frequently seen in meningiomas;
they can be absent in haemangiopericytoma.
• Intratumoural calcification is common in meningioma but rare in haemangiopericytoma. Therefore,
irregular bone erosion is the best discriminator in this scenario.
104. Which fracture most frequently leads to avascular necrosis of the femoral head?
A. Intertrochanteric femur fracture
B. Subtrochanteric femur fracture
C. Femoral neck (intracapsular) fracture
D. Shaft of femur fracture
E. Greater trochanter avulsion fracture
Explanation: (by Perplexity AI, generated with the OpenAI O3 model)
• A femoral neck (intracapsular) fracture disrupts the retinacular vessels within the capsule, making
osteonecrosis of the femoral head the commonest vascular complication.
• Intertrochanteric and subtrochanteric fractures lie extracapsular, preserving the main blood supply,
so osteonecrosis is rare.
• Diaphyseal (shaft) and greater-trochanter avulsion fractures are even further from the femoral head
and do not compromise its arterial inflow, hence they seldom cause avascular necrosis.
105. Which of the following conditions is a recognised cause of a high-probability V/Q mismatch on
scintigraphy that proves false-positive for pulmonary embolism at CT pulmonary angiography?
A. Pulmonary veno-occlusive disease
B. Asthma
C. Bacterial pneumonia
D. Hypersensitivity pneumonitis
E. Pulmonary Langerhans cell histiocytosis
Explanation: (by Perplexity AI, generated with the OpenAI O3 model)
• Pulmonary veno-occlusive disease narrows medium- and large-sized pulmonary veins. Downstream
venous obstruction reduces regional blood flow, so technetium-labelled macro-aggregated albumin
fails to lodge in affected segments while ventilation remains normal. The scan therefore shows
multiple mismatched segmental perfusion defects that mimic thromboembolic occlusion, yet CT or
EBDR Exam MCQs & Concepts September 2022
Dr. Kareem Alnakeeb
235
conventional angiography demonstrates patent pulmonary arteries, confirming a false-positive
result.
• Asthma and bacterial pneumonia usually produce matched or reverse-matched defects because
airway or parenchymal disease affects both ventilation and perfusion.
• Hypersensitivity pneumonitis and pulmonary Langerhans cell histiocytosis typically cause diffuse,
mottled, or matched abnormalities rather than discrete segmental mismatches, so they rarely give a
high-probability pattern.
106. A 52-year-old man presents with progressive exertional dyspnoea. Chest radiograph shows a large
homogenous opacity occupying most of the right hemithorax. Contrast-enhanced CT demonstrates a
well-circumscribed, lobulated, enhancing mass arising from the parietal pleura and displacing adjacent
lung. There is no chest wall invasion or effusion. Which is the most likely diagnosis?
A. Malignant mesothelioma
B. Metastatic sarcoma
C. Solitary fibrous tumour
D. Liposarcoma
E. Pleural lymphoma
Explanation: (by Perplexity AI, generated with the OpenAI O3 model)
• Solitary fibrous tumours typically appear as well-defined, lobulated, contrast-enhancing pleural
masses that displace but rarely invade lung or chest wall, and they often present without pleural
effusion, matching the described CT findings.
• Malignant mesothelioma (A) usually shows diffuse, nodular pleural thickening that encases the lung
and is frequently associated with effusion.
• Metastatic sarcoma (B) and pleural lymphoma (E) more often produce multiple or infiltrative pleural
deposits rather than a solitary mass.
• Liposarcoma (D) characteristically contains areas of macroscopic fat on CT, which are not reported
here.
107. A 52-year-old lifelong non-smoker presents with a new 2.8 cm spiculated nodule in the right upper lobe
on CT thorax without significant mediastinal lymphadenopathy. Which histological subtype of primary
lung cancer is most likely?
A. Large-cell carcinoma
B. Squamous cell carcinoma
C. Small-cell carcinoma
D. Adenocarcinoma
E. Typical carcinoid
Explanation: (by Perplexity AI, generated with the OpenAI O3 model)
• Adenocarcinoma is the commonest primary lung cancer in never-smokers and typically arises
peripherally in the lung parenchyma, often appearing as a solitary spiculated nodule on CT.
• Squamous cell and small-cell carcinomas have the strongest association with tobacco exposure and
usually present as central or hilar masses rather than peripheral nodules.
• Large-cell carcinoma is less common overall and still linked to smoking.
• Typical carcinoid tumours are neuroendocrine lesions that more often occur centrally within bronchi
and tend to be smaller, well-defined masses rather than spiculated nodules.
EBDR Exam MCQs & Concepts September 2022
Dr. Kareem Alnakeeb
236
108. Which of the following ultrasound criteria is most appropriate for diagnosing hypertrophic pyloric
stenosis in a premature neonate presenting with non-bilious vomiting?
A. Muscle thickness ≥ 2 mm
B. Muscle thickness ≥ 3 mm
C. Muscle length ≥ 10 mm
D. Muscle length ≥ 16 mm
E. Pyloric canal diameter ≥ 10 mm
Explanation: (by Perplexity AI, generated with the OpenAI O3 model)
• In preterm infants the pyloric muscle is smaller than in term babies, so lower sonographic cut-offs
are used. A muscle thickness of 3 mm or more is widely accepted as the most reliable indicator of
hypertrophic pyloric stenosis in this group, even when muscle length remains below the classic 17
mm seen in term infants.
• Thickness ≥2 mm (A) lacks specificity and overlaps with pylorospasm.
• Muscle length thresholds of 10 mm (C) and 16 mm (D) are too short; length usually enlarges later and
can stay normal in premature babies.
• Canal diameter ≥10 mm (E) is not a standard metric for diagnosis.
109. In a 25-year-old man with chronic liver disease, MRI brain shows bilateral low-signal intensity confined to
the globus pallidus on T2-weighted and susceptibility sequences. Which trace‐metal deposition disorder
most likely explains this finding?
A. Wilson disease
B. Manganese toxicity
C. Chronic lead exposure
D. Pantothenate-kinase-associated neurodegeneration (PKAN)
E. Fahr disease
Explanation: (by Perplexity AI, generated with the OpenAI O3 model)
• The globus pallidus is highly sensitive to paramagnetic metals. Excess manganese—typically from
portal-systemic shunting or prolonged parenteral nutrition—accumulates preferentially in the
pallida, causing marked T2 and T2* hypointensity with corresponding T1 hyperintensity.
• In Wilson disease copper deposition gives mixed or high T2 signal, not uniform low signal.
• Lead and Fahr disease produce calcification leading to CT hyperdensity and signal void on all MRI
sequences rather than selective T2 hypointensity.
• PKAN shows the classic “eye-of-the-tiger” sign: central T2 hyperintensity surrounded by a
hypointense rim, different from the homogeneous low signal seen here.
110. Hallervorden–Spatz disease (now termed neurodegeneration with brain iron accumulation type 1) is
characterised pathologically by which abnormal cerebral deposition?
A. Aluminium
B. Copper
C. Iron
D. Manganese
E. Magnesium
Explanation: (by Perplexity AI, generated with the OpenAI O3 model)
• Hallervorden–Spatz disease shows excessive iron deposition, especially in the globus pallidus and
substantia nigra, producing the characteristic “eye-of-the-tiger” MR appearance.
EBDR Exam MCQs & Concepts September 2022
Dr. Kareem Alnakeeb
237
• Copper accumulation typifies Wilson disease, while manganese deposition is seen in chronic liver
failure or parenteral nutrition.
• Aluminium and magnesium are not associated with this basal-ganglia neurodegeneration pattern.
111. A 60-year-old man presents with history of chronic cough. The chest radiograph shows a 5 cm subpleural
mass in the right lower lobe. There is a curvilinear opacity from the lower pole of the mass and the mass
courses towards the hilum. CT confirms the mass lesion and demonstrates the bronchovascular bundles
converging into the mass in a curvilinear fashion. In addition, there are multiple pleural plaques but no
lymphadenopathy. The most likely diagnosis is?
A. Bronchogenic carcinoma
B. Round atelectasis
C. Large parenchymal metastasis
D. Lymphoma
E. Arteriovenous malformation
Source: Gupta, Chaitanya. 300 Single Best Answers for the Final FRCR Part A. 1st ed., Jaypee UK,
2010.
Explanation:
• Also called ‘folded lung’ or ‘asbestos pseudotumour’. The lesion forms acute angles with the pleura
indicating its parenchymal location.
• Pleural thickening is usually an associated finding.
• It usually affects the lower lobes and there is volume loss.
• The characteristic sign of round atelectasis is the ‘comet tail‘ sign.
• As the lung collapses, the bronchovascular bundle is pulled into the region.
• As they reach the mass they diverge and arch around the surface to merge with the inferior pole of
the mass. This is typically well demonstrated on CT.
112. A 58-year-old man has an incidental, well-circumscribed 2.3 cm solitary pulmonary nodule in the right
mid-zone on chest radiograph. CT confirms a peripheral lesion with internal fat and coarse “popcorn”
calcification, but no contrast enhancement. What is the most likely diagnosis?
A. Granulomatous tuberculoma
B. Peripheral carcinoid tumour
C. Peripheral pulmonary hamartoma
D. Sclerosing haemangioma (pneumocytoma)
E. Metastatic osteogenic sarcoma deposit
Explanation: (by Perplexity AI, generated with the OpenAI O3 model)
• Intralesional fat together with chunky “popcorn” calcification in a small, non-enhancing, well-
defined peripheral nodule is classic for a pulmonary hamartoma, the commonest benign lung
neoplasm.
• Tuberculomas may calcify but do not contain macroscopic fat and usually enhance.
• Carcinoid tumours enhance strongly and rarely show fat or coarse calcification.
• Sclerosing haemangiomas can calcify lightly but typically lack fat and often enhance; 18
F-FDG uptake
tends to be higher than in hamartoma.
• Metastatic osteogenic sarcoma deposits calcify or ossify but internal fat is not a feature and they
often enhance.
EBDR Exam MCQs & Concepts September 2022
Dr. Kareem Alnakeeb
238
113. A 10-year-old girl presents with intermittent limb pain and low-grade fevers; radiographs show bilateral
symmetrical metaphyseal sclerosis in the femora and tibiae without periosteal reaction, and laboratory
cultures are repeatedly negative. Which condition best explains these findings?
A. Chronic recurrent multifocal osteomyelitis
B. Lead deposition in growing bone
C. Acute bacterial osteomyelitis
D. Ewing sarcoma
E. Rickets
Explanation: (by Perplexity AI, generated with the OpenAI O3 model)
• Chronic recurrent multifocal osteomyelitis (CRMO) is a sterile autoinflammatory bone disorder of
childhood causing episodic limb pain, low-grade fevers and multifocal symmetrical metaphyseal
sclerosis on radiographs; cultures remain negative and there is no periosteal reaction, matching the
vignette.
• Lead deposition produces dense sclerotic metaphyseal lines (“lead lines”) but lacks pain and fevers.
• Acute bacterial osteomyelitis is usually unilateral, lytic in early stages and culture-positive.
• Ewing sarcoma presents with an aggressive permeative pattern and layered periosteal reaction.
• Rickets causes metaphyseal cupping and fraying, not sclerosis.
114. Which imaging investigation most reliably detects a suspected pituitary micro-adenoma in a patient with
Cushing’s disease?
A. High-resolution CT sella with contrast
B. Dynamic gadolinium-enhanced MRI pituitary protocol
C. Standard non-contrast MRI brain
D. Inferior petrosal sinus sampling with venography
E. 18
F-FDG PET/CT of the skull base
Explanation: (by Perplexity AI, generated with the OpenAI O3 model)
• Dynamic contrast-enhanced MRI acquires rapid sequential images during and immediately after
gadolinium administration, highlighting the normal gland’s early intense enhancement while the
adenoma remains relatively hypo-enhancing; this maximises lesion-to-gland contrast and achieves
sensitivities around 90% for micro-adenomas.
• Conventional non-contrast MRI (C) and routine contrasted CT (A) miss many lesions <5 mm.
• Inferior petrosal sinus sampling (D) is invasive and used when MRI is negative to confirm a central
ACTH source, not as the primary imaging test.
• FDG PET/CT (E) lacks spatial resolution for millimetric sellar lesions and is reserved for atypical or
metastatic disease work-up.
115. A 28-year-old motorcyclist presents 3 weeks after a closed head injury with progressive orbital pain,
pulsatile proptosis and a “whooshing” bruit over the right eye. CT angiography shows an enlarged right
superior ophthalmic vein and early cavernous sinus opacification. What is the most likely underlying
lesion?
A. Dural arteriovenous malformation of the transverse sinus
B. Direct carotid–cavernous fistula (Barrow type A)
C. Indirect dural carotid–cavernous fistula (Barrow type D)
D. Cavernous sinus thrombosis
E. Traumatic pseudo-aneurysm of the middle meningeal artery
EBDR Exam MCQs & Concepts September 2022
Dr. Kareem Alnakeeb
239
Explanation: (by Perplexity AI, generated with the OpenAI O3 model)
• Head trauma causing a tear in the cavernous segment of the internal carotid artery produces a high-
flow direct carotid–cavernous fistula (type A). The arterialised cavernous sinus drains anteriorly,
dilating the superior ophthalmic vein and giving the classic triad of pulsatile proptosis, bruit and
ocular pain.
• Indirect dural fistulas (C) are usually low-flow, often spontaneous and present more insidiously.
• Cavernous sinus thrombosis (D) causes chemosis, ophthalmoplegia and fever rather than pulsatile
proptosis.
• A transverse sinus malformation (A) would not enlarge the superior ophthalmic vein.
• A middle meningeal pseudo-aneurysm (E) lies extracavernously and typically manifests with epidural
haematoma or scalp bruit, not superior ophthalmic vein dilatation.
116. A preterm neonate on positive-pressure ventilation develops progressive respiratory distress and a chest
radiograph (shown) demonstrates diffuse bilateral granular opacities with air bronchograms. Which
condition best explains this “wet lung” appearance?
A. Meconium aspiration
B. Transient tachypnoea of the newborn
C. Neonatal pneumonia
D. Hyaline membrane disease
E. Congenital lobar emphysema
Explanation: (by Perplexity AI, generated with the OpenAI O3 model)
• Transient tachypnoea of the newborn (TTN) is caused by delayed clearance of fetal lung fluid, giving a
“wet lung” film with diffuse hazy opacities, fluid in fissures and prominent vascular markings. It
typically affects term or near-term infants after Caesarean delivery and resolves within 48–72 hours.
• Hyaline membrane disease (D) also shows diffuse granular opacities but occurs in markedly preterm
infants and is associated with low lung volumes.
• Meconium aspiration (A) produces patchy coarse infiltrates and hyperinflation, not uniform haziness.
• Neonatal pneumonia (C) can mimic TTN yet usually shows asymmetric consolidation and pleural
effusion.
• Congenital lobar emphysema (E) presents with lobar overinflation rather than diffuse opacification.
117. Progressive massive fibrosis is most characteristic of which occupational lung disease involving upper-
lobe conglomerate masses sometimes surrounded by ground-glass change?
A. Asbestosis
B. Coal workers’pneumoconiosis
C. Chronic hypersensitivity pneumonitis
D. Idiopathic pulmonary fibrosis
E. Pulmonary Langerhans cell histiocytosis
Explanation: (by Perplexity AI, generated with the OpenAI O3 model)
• Progressive massive fibrosis (PMF) refers to coalescent fibrotic masses—classically bilateral and
upper-lobe predominant—arising as the advanced form of coal workers’ pneumoconiosis. CT shows
large irregular opacities with strand-like arms and may demonstrate adjacent ground-glass opacity
from dust-related alveolitis.
• Asbestosis (A) usually produces lower-zone subpleural fibrosis and pleural plaques, not
conglomerate upper-lobe masses.
EBDR Exam MCQs & Concepts September 2022
Dr. Kareem Alnakeeb
240
• Chronic hypersensitivity pneumonitis (C) can mimic fibrotic patterns but lacks the signature mass-
like lesions of PMF.
• Idiopathic pulmonary fibrosis (D) causes a basal, peripheral usual interstitial pneumonia pattern.
• Pulmonary Langerhans cell histiocytosis (E) affects upper lobes but shows nodules and cysts rather
than confluent fibrotic masses.
118. A 35-year-old man with progressive dyspnoea and cough undergoes high-resolution CT of the chest,
which shows bilateral perihilar “crazy-paving”—ground-glass opacification with superimposed thickened
inter- and intralobular septa. What is the most likely underlying diagnosis?
A. Pneumocystis jirovecii pneumonia
B. Sarcoidosis
C. Pulmonary alveolar proteinosis
D. Acute respiratory distress syndrome
E. Lipoid pneumonia
Explanation: (by Perplexity AI, generated with the OpenAI O3 model)
• Pulmonary alveolar proteinosis classically produces the “crazy-paving” pattern because
lipoproteinaceous material fills the alveoli (ground-glass) while adjacent septa become thickened.
• Pneumocystis infection can mimic ground-glass change but septal thickening is usually mild and
diffuse rather than perihilar dominant.
• Sarcoidosis favours upper-lobe nodularity and fibrosis, not widespread ground-glass.
• Early ARDS gives dependent ground-glass opacities without the characteristic septal lines.
• Lipoid pneumonia tends to show low-attenuation consolidation and fat-density foci rather than
uniform crazy-paving.
119. Which statement best describes the usual presentation of a peripheral pulmonary hamartoma?
A. Multiple lobulated nodules arising in childhood
B. Endobronchial mass causing obstructive symptoms in young adults
C. Solitary, well-circumscribed parenchymal nodule detected incidentally in middle-aged patients
D. Rapidly enlarging cavitating lesion with surrounding consolidation
E. Calcified mediastinal mass associated with lymphadenopathy
Explanation: (by Perplexity AI, generated with the OpenAI O3 model)
• Pulmonary hamartoma is the commonest benign lung tumour. More than 90% occur peripherally
within the lung parenchyma and appear as a single, smoothly marginated “coin lesion” that is often
found on imaging performed for unrelated reasons. Peak incidence is in the fifth to sixth decades;
occurrence in children or young adults is rare.
• Endobronchial hamartomas (option B) account for only about 5–10% and usually present with cough
or obstruction, not as the typical peripheral nodule.
• Multiple lesions (option A) and rapid growth with cavitation (option D) are atypical and should raise
alternative diagnoses such as metastases or infection.
• Mediastinal location with lymphadenopathy (option E) is exceedingly uncommon for hamartoma.
120. In interstitial lung disease, which specific histopathological pattern confers the HIGHEST relative risk for
subsequent bronchogenic carcinoma development?
A. Desquamative interstitial pneumonia
B. Respiratory bronchiolitis–associated interstitial lung disease
EBDR Exam MCQs & Concepts September 2022
Dr. Kareem Alnakeeb
241
C. Cryptogenic organising pneumonia
D. Nonspecific interstitial pneumonia
E. Usual interstitial pneumonia
Explanation: (by Perplexity AI, generated with the OpenAI O3 model)
• Usual interstitial pneumonia (UIP), the pathological correlate of idiopathic pulmonary fibrosis,
is strongly linked to an increased incidence of bronchogenic carcinoma because of
repetitive epithelial injury and fibrosis that promote oncogenic change. The risk is several-fold
higher than in the general population.
• Cryptogenic organising pneumonia shows patchy, reversible intra-alveolar granulation tissue with no
proven rise in lung cancer incidence.
• Desquamative interstitial pneumonia and respiratory bronchiolitis–associated ILD are smoking-
related conditions with comparatively little data demonstrating a cancer association.
• Nonspecific interstitial pneumonia, often seen in connective-tissue disease, has lower fibrotic
distortion than UIP and has not shown the same carcinogenic propensity.
121. Which histological subtype is most frequently encountered in cases of primary orbital
rhabdomyosarcoma in children?
A. Alveolar
B. Pleomorphic
C. Embryonal
D. Spindle-cell sclerosing
E. Botryoid variant
Explanation: (by Perplexity AI, generated with the OpenAI O3 model)
• Embryonal rhabdomyosarcoma accounts for roughly 70–80% of orbital RMS in the paediatric age
group and carries the most favourable prognosis.
• Alveolar lesions are less common in the orbit and typically affect older children or adults, while
pleomorphic disease is largely restricted to older adults.
• The spindle-cell sclerosing form is rare and usually arises in paratesticular or head-and-neck sites
outside the orbit.
• The botryoid pattern is a variant of embryonal RMS seen mainly in mucosal sites such as the bladder
and nasopharynx, not the orbit.
122. A 32-year-old woman develops heavy vaginal bleeding 20 minutes after an uncomplicated vaginal
delivery. What is the most common cause of primary postpartum haemorrhage?
A. Retained placental tissue
B. Uterine atony
C. Genital tract laceration
D. Uterine inversion
E. Uterine artery pseudoaneurysm
Explanation: (by Perplexity AI, generated with the OpenAI O3 model)
• Uterine atony—failure of the myometrium to contract effectively after placental delivery—is
responsible for the majority (≈70-80%) of primary postpartum haemorrhages, because the flaccid
uterus cannot compress the spiral arteries.
• Retained placental tissue and genital tract lacerations are significant but less frequent causes.
Uterine inversion is rare and usually dramatic.
EBDR Exam MCQs & Concepts September 2022
Dr. Kareem Alnakeeb
242
• Uterine artery pseudoaneurysm is an uncommon iatrogenic cause that typically presents days to
weeks postpartum rather than immediately.
123. A 22-year-old woman undergoes echocardiography for exertional dyspnoea. The study shows marked
enlargement of the right atrium and right ventricle with a normal-sized left atrium; colour Doppler
demonstrates left-to-right shunting at the atrial level. Which congenital abnormality most commonly
explains this combination of findings?
A. Secundum atrial septal defect without associated venous anomaly
B. Sinus venosus atrial septal defect with partial anomalous drainage of the right upper pulmonary
vein
C. Ostium primum atrial septal defect
D. Isolated partial anomalous pulmonary venous return of a left upper lobe vein to the brachiocephalic
vein
E. Total anomalous pulmonary venous return (supracardiac type)
Explanation: (by Perplexity AI, generated with the OpenAI O3 model)
• Sinus venosus ASD typically occurs high in the inter-atrial septum near the superior vena cava and is
frequently accompanied by partial anomalous pulmonary venous return of the right upper lobe vein
into the SVC or right atrium. The combined defects create a significant left-to-right shunt that dilates
the right atrium and ventricle while leaving the left atrium normal.
• Secundum and ostium primum ASDs (A, C) can enlarge right-sided chambers but are not usually
linked to anomalous pulmonary veins.
• Isolated PAPVR of a left upper vein (D) causes a smaller shunt and is less likely to produce
pronounced right-heart enlargement in an otherwise healthy young adult.
• Total anomalous pulmonary venous return (E) presents in infancy with cyanosis and involves all
pulmonary veins, not the partial pattern described.
124. Which intracranial ring-enhancing lesion classically shows marked central restricted diffusion on
diffusion-weighted MRI?
A. High-grade glioma
B. Metastatic adenocarcinoma
C. Pyogenic brain abscess
D. Cerebral toxoplasmosis
E. Radiation necrosis
Explanation: (by Perplexity AI, generated with the OpenAI O3 model)
• Pyogenic brain abscesses contain viscous, cellular pus that markedly impedes Brownian water
motion, producing a bright core on DWI with low ADC—an imaging sign often used to differentiate
them from other ring-enhancing masses.
• Necrotic tumours such as high-grade glioma and metastatic adenocarcinoma usually demonstrate
facilitated diffusion because their fluid centres are less cellular.
• Cerebral toxoplasmosis typically lacks restricted diffusion owing to necrotic rather than purulent
contents.
• Radiation necrosis consists of liquefaction and necrotic debris, so diffusion is not restricted. Thus,
only a pyogenic abscess reliably shows central restricted diffusion.
EBDR Exam MCQs & Concepts September 2022
Dr. Kareem Alnakeeb
243
125. A 25-year-old patient presents with headaches and obstructive hydrocephalus. MRI demonstrates a well-
defined intraventricular mass at the foramen of Monro showing multiple tiny “soap-bubble” cysts,
punctate calcification and mild heterogeneous enhancement. Which of the following tumours is the
most likely diagnosis?
A. Ependymoma
B. Central neurocytoma
C. Choroid plexus papilloma
D. Subependymal giant cell astrocytoma
E. Medulloblastoma
Explanation: (by Perplexity AI, generated with the OpenAI O3 model)
• The combination of location (lateral ventricle/foramen of Monro in a young adult), well-circumscribed
margins and the characteristic “soap-bubble” appearance caused by numerous tiny intratumoral
cysts strongly points to central neurocytoma. These tumours typically calcify and enhance mildly,
matching the vignette.
o Ependymomas may be intraventricular but are more common in children and often show
extension through the ventricular walls with less distinct borders.
o Choroid plexus papillomas usually occur in children, are frond-like, intensely enhance and
often arise in the trigone.
o Subependymal giant cell astrocytoma occurs almost exclusively in tuberous sclerosis and
shows a solid mass at the foramen of Monro that is hyperenhancing and often calcified but
lacks the bubbly cystic pattern.
o Medulloblastoma is a posterior fossa tumour, not intraventricular in the lateral ventricles in
young adults.
126. Which intracranial tumour is classically associated with neurofibromatosis type 2 but not with
neurofibromatosis type 1?
A. Optic pathway glioma
B. Vestibular schwannoma
C. Pilocytic astrocytoma
D. Intraventricular meningioma
E. High-grade glioblastoma
Explanation: (by Perplexity AI, generated with the OpenAI O3 model)
• Neurofibromatosis type 2 (NF2) predisposes to multiple extra-axial tumours, notably bilateral
vestibular schwannomas and meningiomas. Intraventricular meningiomas arise from arachnoid cap
cells within the lateral ventricles and occur in up to half of NF2 patients, whereas they are not a
recognised feature of neurofibromatosis type 1.
• Optic pathway gliomas (A) and pilocytic astrocytomas (C) are typical of NF1, not NF2.
• Vestibular schwannoma (B) is indeed common in NF2 but also occurs sporadically, so it is not as
discriminating.
• High-grade glioblastoma (E) shows no specific association with either phakomatosis.
EBDR Exam MCQs & Concepts September 2022
Dr. Kareem Alnakeeb
244
127. A patient’s cardiac MRI shows late gadolinium enhancement involving 50% of the mid-septal left
ventricular myocardium. Which coronary artery territory is most likely affected and still
contains viable myocardium?
A. Right coronary artery, non-viable
B. Right coronary artery, viable
C. Left circumflex artery, viable
D. Left anterior descending artery, viable
E. Left circumflex artery, non-viable
Explanation: (by Perplexity AI, generated with the OpenAI O3 model)
• Late gadolinium enhancement (LGE) of ≤50% wall thickness indicates that at least half of the
myocytes are intact and capable of functional recovery after revascularisation. The interventricular
septum—particularly its anterior two-thirds—is classically supplied by septal perforators from the
left anterior descending (LAD) artery. Therefore a 50% mid-septal infarct points to the LAD territory
with residual viability.
o Option B: the right coronary artery mainly supplies the inferior septum; an RCA lesion would
less commonly give mid-septal LGE, and viability is possible but the anatomy makes this
choice less likely.
o Option C/E: the left circumflex artery rarely supplies the interventricular septum except in
some left-dominant hearts.
o Option A: “non-viable” contradicts the ≤50% LGE threshold, which denotes viability.
128. A 55-year-old man with exertional angina undergoes CT coronary calcium scoring that reports an
Agatston score of 750. Which statement best reflects the likelihood of significant coronary artery
stenosis in this patient?
A. A calcium score above 400 virtually excludes ≥50% stenosis
B. A score ≥250 has high specificity for ≥50% stenosis on CT angiography
C. Any non-zero score mandates invasive coronary angiography irrespective of symptoms
D. A calcium score of 0 reliably rules out ≥70% stenosis in all age groups
E. Calcium scoring is unsuitable for estimating stenosis risk in symptomatic patients
Explanation: (by Perplexity AI, generated with the OpenAI O3 model)
• A calcium score threshold of about 250 demonstrates 100% specificity and positive predictive value
for ≥50% luminal narrowing on coronary CT angiography in symptomatic cohorts, so a score of 750
makes significant stenosis very likely.
• Option A is incorrect because scores >400 increase—rather than exclude—the probability of
obstructive disease.
• Option C overstates management; many patients are first imaged with CT angiography or functional
tests.
• Option D is wrong: younger patients can have obstructive, non-calcified plaque despite a zero score.
• Option E is incorrect because extensive data show calcium scoring helps stratify symptomatic
patients, with higher scores correlating with increasing stenosis severity.
EBDR Exam MCQs & Concepts September 2022
Dr. Kareem Alnakeeb
245
129. In congenitally corrected transposition of the great arteries, which combination of cardiac connections is
present?
A. Atrioventricular concordance with ventriculoarterial discordance
B. Atrioventricular discordance with ventriculoarterial concordance
C. Atrioventricular and ventriculoarterial concordance
D. Double-inlet left ventricle with malposed great arteries
E. Atrioventricular and ventriculoarterial discordance
Explanation: (by Perplexity AI, generated with the OpenAI O3 model)
• Congenitally corrected transposition (also called “double discordance”) features atrioventricular
(AV) discordance—right atrium draining to the morphologic left ventricle and left atrium to the
morphologic right ventricle—and simultaneous ventriculoarterial (VA) discordance—the
morphologic left ventricle ejects into the pulmonary artery while the morphologic right ventricle
ejects into the aorta. The two discordant connections “cancel out” physiologically, so systemic
venous blood still reaches the lungs and oxygenated blood reaches the body. Option E therefore
describes the hallmark anatomical arrangement.
• Key distractors:
o Option A lacks AV discordance, so blood flow would not be physiologically corrected.
o Option B describes the rare entity of isolated AV discordance with normal great-artery
relations, not congenitally corrected TGA.
o Option C depicts the normal heart.
o Option D refers to a univentricular connection, an entirely different malformation.
130. Following chemoradiotherapy for a glioblastoma, an MRI including multivoxel proton MR-spectroscopy is
performed to distinguish true tumour progression from pseudoprogression. Which spectroscopic change
is most sensitive for active tumour progression?
A. Fall in lipid peak
B. New alanine resonance
C. Decrease in choline/N-acetylaspartate (NAA) ratio
D. Rise in choline/NAA ratio
E. Isolated increase in creatine
Explanation: (by Perplexity AI, generated with the OpenAI O3 model)
• Active glioma progression shows heightened cell-membrane turnover and cellularity, producing a
marked rise in choline with concurrent loss of neuronal NAA, so the choline/NAA ratio increases
(often >1.4), the most sensitive MR-spectroscopic marker of viable tumour. Pseudoprogression or
radiation change typically demonstrates low choline and often elevated lipid-lactate from necrosis,
not an increased choline/NAA ratio.
o Option A is incorrect: lipid peaks usually rise in treatment-related necrosis rather than active
tumour.
o Option B (alanine) characterises meningioma, not glioma assessment.
o Option C describes the opposite of tumour behaviour; a falling ratio suggests necrosis.
o Option E: creatine changes are nonspecific and less sensitive than choline/NAA alterations.
EBDR Exam MCQs & Concepts September 2022
Dr. Kareem Alnakeeb
246
131. A 32-year-old heavy smoker presents with progressive dry cough and exertional dyspnoea. High-
resolution CT shows numerous 2–5 mm centrilobular nodules, some cavitating, and multiple small thin-
walled cysts predominantly in the upper and mid-zone lungs with relative costophrenic sparing. Which
diagnosis best explains this pattern?
A. Respiratory bronchiolitis–interstitial lung disease
B. Pulmonary Langerhans cell histiocytosis
C. Lymphangioleiomyomatosis
D. Sarcoidosis
E. Hypersensitivity pneumonitis
Explanation: (by Perplexity AI, generated with the OpenAI O3 model)
• Pulmonary Langerhans cell histiocytosis (PLCH) almost exclusively affects young adult smokers and
classically shows a mixture of centrilobular nodules that may cavitate and evolve into irregular, thin-
walled cysts concentrated in the upper and middle lung zones with sparing of lung bases.
• Respiratory bronchiolitis–ILD (A) produces faint ground-glass haziness rather than discrete
nodules/cysts.
• Lymphangioleiomyomatosis (C) occurs in women, features uniform round cysts of equal size, and
lacks nodules.
• Sarcoidosis (D) shows perilymphatic nodules and upper-zone fibrosis without cystic change.
• Hypersensitivity pneumonitis (E) gives diffuse ground-glass opacities and mosaic attenuation, not
cavitating nodules evolving into cysts.
132. A 65-year-old woman presents six months after pelvic radiotherapy for cervical cancer with irritative
lower-urinary-tract symptoms. Cystoscopy is unremarkable. MRI pelvis shows diffuse concentric
thickening of the urinary bladder wall with low T2 signal intensity and homogeneous post-contrast
enhancement. Which is the most likely diagnosis?
A. Recurrent cervical tumour invading the bladder
B. Radiation-induced cystitis
C. Schistosomiasis of the bladder
D. Transitional-cell carcinoma of the bladder
E. Tuberculous cystitis
Explanation: (by Perplexity AI, generated with the OpenAI O3 model)
• Radiation-induced cystitis typically appears within months to a few years after pelvic irradiation and
manifests as diffuse, symmetric bladder wall thickening that is low on T2-weighted MRI and
enhances homogeneously—reflecting fibrosis and submucosal vascular changes. There is no
discrete mass or focal mural defect.
• Recurrent cervical tumour (A) would more often produce an eccentric soft-tissue mass extending
from the cervix into the bladder rather than smooth concentric thickening.
• Schistosomiasis (C) gives irregular calcified or nodular thickening, often with mural calcification.
• Transitional-cell carcinoma (D) usually presents as a focal polypoid mass or eccentric asymmetric
thickening, not diffuse symmetric changes.
• Tuberculous cystitis (E) classically shows a contracted, small-capacity bladder with irregular wall
thickening and sometimes calcification, usually years after infection rather than shortly after
radiotherapy.
EBDR Exam MCQs & Concepts September 2022
Dr. Kareem Alnakeeb
247
133. In patients with pulmonary sarcoidosis, up to what percentage will progress to irreversible pulmonary
fibrosis despite treatment?
A. 5%
B. 10%
C. 20%
D. 35%
E. 50%
Explanation: (by Perplexity AI, generated with the OpenAI O3 model)
• Most individuals with sarcoidosis experience either remission or a stable chronic course. However, a
minority develop sarcoidosis-associated pulmonary fibrosis, which carries a markedly worse
prognosis.
• Contemporary epidemiological and pathological reviews consistently show that approximately one-
fifth—around 20%—of patients progress to established fibrotic lung disease even when managed
with corticosteroids or steroid-sparing agents.
• Lower figures such as 5% or 10% underestimate this well-documented risk, whereas 35% and 50%
overstate it and are not supported by current data.
134. Regarding transitional cell carcinoma (TCC) of the urinary tract, which statement is MOST accurate?
A. Primary TCC occurs more frequently in the upper ureter than in the urinary bladder.
B. Squamous metaplasia is a recognised premalignant change preceding TCC in the bladder.
C. Multifocality is common because the urothelium is exposed to carcinogens along its entire
length.
D. Hydronephrosis is uncommon with upper-tract TCC owing to its infiltrative growth pattern.
E. Smoking is not a significant risk factor for upper-tract TCC.
Explanation: (by Perplexity AI, generated with the OpenAI O3 model)
• Transitional cell carcinoma arises from urothelium anywhere from renal pelvis to urethra. Exposure
of the whole lining to urinary carcinogens (“field change”effect) results in synchronous or
metachronous tumours, so multifocality is typical—making option C correct.
• Upper-tract (renal pelvis and ureter) tumours are much less common than bladder TCC, not more
common, invalidating option A.
• Keratinising squamous metaplasia predisposes to squamous, not transitional, cell cancer, so option
B is wrong.
• Obstructive hydronephrosis is actually frequent with ureteric or pelvic TCC because the mass
narrows the lumen, refuting option D.
• Cigarette smoking is a major risk factor for both bladder and upper-tract TCC, so option E is false.
135. A 35-year-old man presents with progressive dyspnoea and palpitations. Cardiac MRI shows a globally
dilated left ventricle with systolic dysfunction (LVEF 30%), diffuse hypokinesia and patchy sub-epicardial
late gadolinium enhancement in the lateral wall. Which is the most likely diagnosis?
A. Acute myocardial infarction
B. Dilated cardiomyopathy secondary to chronic alcohol abuse
C. Acute viral myocarditis
D. Hypertrophic cardiomyopathy
E. Cardiac sarcoidosis
EBDR Exam MCQs & Concepts September 2022
Dr. Kareem Alnakeeb
248
Explanation: (by Perplexity AI, generated with the OpenAI O3 model)
• Sub-epicardial or mid-wall late gadolinium enhancement sparing the endocardium, particularly in
the lateral wall, is characteristic of acute viral myocarditis and reflects inflammation and necrosis of
the outer myocardial layers. Dilated cardiomyopathy from alcohol typically shows mid-wall, not sub-
epicardial, enhancement and requires a history of heavy intake.
• Acute infarction produces sub-endocardial or transmural enhancement in a coronary artery territory.
• Hypertrophic cardiomyopathy shows hypertrophied, non-dilated ventricles with patchy mid-wall
enhancement in hypertrophied segments.
• Cardiac sarcoidosis favours multifocal mid-wall or transmural enhancement, often with right-
ventricular involvement and associated lymphadenopathy.
136. In a patient with acute myocarditis, transthoracic echocardiography shows a markedly reduced left-
ventricular ejection fraction (LVEF 28%) with global hypokinesis. Which term best describes this finding?
A. Diastolic dysfunction
B. Systolic dysfunction
C. Concentric hypertrophy
D. Restrictive physiology
E. Cardiac tamponade
Explanation: (by Perplexity AI, generated with the OpenAI O3 model)
• The hallmark of systolic dysfunction is an abnormally low LVEF reflecting impaired ventricular
contractility; acute myocarditis can cause diffuse myocardial inflammation and global hypokinesis,
reducing the ejection fraction to <40%.
• Diastolic dysfunction (A) involves impaired relaxation with preserved or mildly reduced LVEF.
• Concentric hypertrophy (C) refers to increased wall thickness with normal cavity size, not a low
ejection fraction.
• Restrictive physiology (D) denotes normal systolic function but severely impaired ventricular filling.
• Cardiac tamponade (E) produces chamber compression and pulsus paradoxus rather than a primary
fall in LVEF.
137. Which inherited cardiomyopathy that predisposes to ventricular tachyarrhythmia is characterised
pathologically by progressive fibrofatty replacement of the right ventricular free wall?
A. Dilated cardiomyopathy
B. Hypertrophic cardiomyopathy
C. Arrhythmogenic right ventricular cardiomyopathy
D. Restrictive cardiomyopathy
E. Left ventricular non-compaction
Explanation: (by Perplexity AI, generated with the OpenAI O3 model)
• Arrhythmogenic right ventricular cardiomyopathy (ARVC) is an autosomal-dominant desmosomal
disorder in which normal myocytes are gradually replaced by fibrofatty tissue, especially within the
“triangle of dysplasia” of the right ventricle. This provides the substrate for re-entrant ventricular
tachycardia and sudden death.
• Dilated cardiomyopathy shows chamber dilation and fibrosis rather than fatty infiltration;
hypertrophic cardiomyopathy features myocyte hypertrophy and disarray; restrictive cardiomyopathy
is defined by diastolic dysfunction with normal ventricular thickness; left ventricular non-
EBDR Exam MCQs & Concepts September 2022
Dr. Kareem Alnakeeb
249
compaction results from arrested myocardial development with deep trabeculations—none display
the hallmark fibrofatty replacement seen in ARVC.
138. A 45-year-old woman underwent total thyroidectomy for papillary thyroid carcinoma. Six months later her
stimulated serum thyroglobulin is 48 ng/mL, but a diagnostic ^123I whole-body scan shows no abnormal
uptake. Which ONE of the following is the most appropriate next management step?
A. Empiric high-dose 131
I ablation therapy
B. 18
F-FDG PET/CT to localise iodine-negative metastases
C. Neck ultrasound with fine-needle aspiration of any suspicious nodes
D. Repeat diagnostic 123
I scan in 6 months while on TSH suppression
E. Begin tyrosine-kinase inhibitor therapy (e.g. sorafenib)
Explanation: (by Perplexity AI, generated with the OpenAI O3 model)
• A markedly elevated stimulated thyroglobulin after thyroidectomy indicates residual or metastatic
differentiated thyroid cancer even when radio-iodine imaging is negative. Current guidelines
recommend targeted structural imaging—starting with high-resolution neck ultrasound—because
cervical nodal disease is the commonest source and is potentially curable surgically. Empiric high-
dose 131
I (Option A) may be considered only after structural evaluation, as many iodine-negative
lesions will not respond and the radiation burden is significant.
• 18
F-FDG PET/CT (Option B) is useful if ultrasound is negative or non-diagnostic but is not first-line.
• Simply repeating the 123
I scan (Option D) delays definitive evaluation and rarely converts to positive.
• Tyrosine-kinase inhibitors (Option E) are reserved for progressive, inoperable, radio-iodine-refractory
disease, not as initial investigation.
139. A 7-year-old boy presents with seizures, global developmental delay and multiple hypomelanotic
macules on skin examination. Which inherited condition best explains the constellation of neurological
and cutaneous findings?
A. Neurofibromatosis type 1
B. Tuberous sclerosis complex
C. Sturge–Weber syndrome
D. Ataxia-telangiectasia
E. Phenylketonuria
Explanation: (by Perplexity AI, generated with the OpenAI O3 model)
• Tuberous sclerosis complex is an autosomal-dominant phakomatosis characterised by cortical
tubers causing epilepsy and learning difficulties, alongside cutaneous manifestations such as
hypomelanotic “ash-leaf” macules and shagreen patches.
• Neurofibromatosis 1 produces café-au-lait spots and neurofibromas but not cortical tubers or ash-
leaf lesions.
• Sturge–Weber gives facial port-wine stains and leptomeningeal angiomatosis rather than
hypomelanotic macules.
• Ataxia-telangiectasia features progressive cerebellar ataxia and oculocutaneous telangiectasias,
not seizures from cortical tubers.
• Phenylketonuria leads to intellectual impairment if untreated but lacks the specific skin lesions and
brain hamartomas seen here.
EBDR Exam MCQs & Concepts September 2022
Dr. Kareem Alnakeeb
250
140. A 55-year-old man presents with digital clubbing, joint pains and plain radiographs showing smooth,
bilateral periosteal new bone along the diaphyses of the tibiae and radii with no underlying destructive
lesion. Which underlying condition most commonly produces this skeletal reaction?
A. Benign fibroma of bone
B. Deep (desmoid-type) fibromatosis of the abdominal wall
C. Primary lung carcinoma
D. Crohn’s disease–related enteric fistulae
E. Cyanotic congenital heart disease
Explanation: (by Perplexity AI, generated with the OpenAI O3 model)
• Hypertrophic osteoarthropathy (HOA) is characterised by digital clubbing and symmetric, smooth
periosteal reaction affecting long bones. The classic and commonest secondary cause is a primary
intrathoracic malignancy—especially non-small-cell lung carcinoma—because tumour-derived
vascular and humoral factors drive periosteal new bone.
• Cyanotic heart disease and inflammatory bowel disease can cause HOA but far less frequently.
• Benign bony fibromas and desmoid-type fibromatosis do not generate the diffuse periosteal
response seen in HOA; they produce focal cortical remodeling or soft-tissue masses without
systemic skeletal changes.
141. A 42-year-old marathon runner presents with progressive knee discomfort and intermittent swelling.
Radiographs show multiple smooth, well-defined, round intra-articular calcified bodies; MRI confirms
numerous ossified nodules within the synovium but an otherwise normal joint. Which diagnosis best
explains these imaging findings?
A. Pigmented villonodular synovitis
B. Synovial chondromatosis
C. Intra-articular osteoid osteoma
D. Popliteal artery arteriovenous malformation
E. Hydroxyapatite deposition disease
Explanation: (by Perplexity AI, generated with the OpenAI O3 model)
• Synovial chondromatosis causes metaplasia of synovial membrane into cartilaginous nodules that
calcify, producing multiple smooth “loose bodies” evident on radiographs and MRI—exactly as seen
here. Pigmented villonodular synovitis gives blooming low-signal hemosiderin but rarely calcifies.
• Osteoid osteoma forms a single cortical nidus, not numerous intra-articular bodies.
• Arteriovenous malformations show serpiginous flow voids or enhancing vascular channels, not
ossified nodules.
• Hydroxyapatite deposition disease typically affects tendons (e.g. rotator cuff) creating amorphous
peri-tendinous calcification, not discrete intra-articular ossicles.
142. In a young patient with sickle-cell trait, CT shows an infiltrative, hypo-enhancing solid mass centred in the
renal medulla causing caliectasis and retroperitoneal nodes. Which diagnosis best fits these findings?
A. Papillary renal cell carcinoma
B. Collecting (Bellini) duct carcinoma
C. Renal medullary carcinoma
D. Transitional cell carcinoma of the renal pelvis
E. Oncocytoma
EBDR Exam MCQs & Concepts September 2022
Dr. Kareem Alnakeeb
251
Explanation: (by Perplexity AI, generated with the OpenAI O3 model)
• Renal medullary carcinoma is a rare, highly aggressive tumour almost exclusive to patients with
sickle-cell trait. Imaging typically demonstrates a central medullary mass that is infiltrative, poorly
marginated and markedly hypo-enhancing compared with renal cortex, often accompanied by
caliectasis and regional lymphadenopathy.
• Collecting duct carcinoma can mimic the pattern but lacks the strong sickle-cell association.
• Papillary RCCs are usually well circumscribed and hypovascular cortical tumours.
• Transitional cell carcinoma arises from the urothelium and appears as a filling defect or discrete
pelvic mass rather than a parenchymal infiltrative lesion.
• Oncocytomas are benign, usually cortical and frequently show a characteristic central scar, not
medullary infiltration.
143. In a patient with congenitally corrected transposition of the great arteries (ccTGA), which segmental
arrangement is most characteristic of the condition, showing atrioventricular discordance with
ventriculoarterial discordance but normal atrial situs?
A. {S, D, D}
B. {I, D, L}
C. {S, L, L}
D. {I, L, D}
E. {S, D, L}
Explanation: (by Perplexity AI, generated with the OpenAI O3 model)
• ccTGA shows normal atrial situs solitus (S), left-looped ventricles (L) producing atrioventricular
discordance, and left-looped great arteries (L) causing ventriculoarterial discordance, giving the
classic {S, L, L} segmental set.
• {S, D, D} (option A) is the normal heart without discordance.
• {I, D, L} (option B) begins with atrial situs inversus, not typical for ccTGA.
• {I, L, D} (option D) has both situs inversus and a normally related great-artery arrangement.
• {S, D, L} (option E) would have atrioventricular concordance with only ventriculoarterial discordance
(d-TGA pattern), not the double discordance of ccTGA.
144. A 56-year-old man presents with progressive dyspnoea and dry cough. High-resolution CT shows bibasal
ground-glass opacities with superimposed inter- and intralobular septal thickening (“crazy-paving”) and
small bilateral pleural effusions. Which is the most likely diagnosis?
A. Pneumocystis jirovecii pneumonia
B. Cryptogenic organising pneumonia
C. Acute interstitial pneumonia
D. Lymphocytic interstitial pneumonia
E. Pulmonary alveolar proteinosis
Explanation: (by Perplexity AI, generated with the OpenAI O3 model)
• Pulmonary alveolar proteinosis classically produces diffuse or bibasal ground-glass opacities with a
crazy-paving pattern due to intra-alveolar surfactant accumulation, and minor pleural effusions may
be present.
• Lymphocytic interstitial pneumonia (option D) usually shows thin-walled cysts mixed with ground-
glass change rather than uniform crazy-paving.
EBDR Exam MCQs & Concepts September 2022
Dr. Kareem Alnakeeb
252
• Pneumocystis pneumonia (option A) can cause ground-glass opacities but pleural effusions are
uncommon and septal thickening less pronounced.
• Cryptogenic organising pneumonia (option B) favours peripheral consolidations and bronchocentric
nodules, not crazy-paving.
• Acute interstitial pneumonia (option C) presents with diffuse alveolar damage leading to widespread
consolidation, lacking the characteristic crazy-paving appearance seen here.
145. A 25-year-old woman with facial angiofibromas and epilepsy undergoes abdominal CT, which shows
multiple bilateral renal masses containing macroscopic fat consistent with angiomyolipomas. The renal
lesions and skin findings are most characteristic of which phakomatosis?
A. Neurofibromatosis type 1
B. Sturge–Weber syndrome
C. Tuberous sclerosis complex
D. Von Hippel–Lindau disease
E. Ataxia-telangiectasia
Explanation: (by Perplexity AI, generated with the OpenAI O3 model)
• Renal angiomyolipomas that are multiple and bilateral are a classic extracranial manifestation of
tuberous sclerosis complex (TSC); up to 80% of TSC patients develop them, often alongside facial
angiofibromas and epilepsy. Sporadic angiomyolipoma is usually solitary, while the other
phakomatoses listed rarely produce fat-containing renal tumours.
• Neurofibromatosis type 1 can give dermal neurofibromas but not angiomyolipomas.
• Sturge–Weber causes leptomeningeal angiomata and port-wine stains, not renal fat tumours.
• Von Hippel–Lindau is linked to renal cell carcinoma and cysts, not angiomyolipoma.
• Ataxia-telangiectasia has immunodeficiency and telangiectasias without renal involvement.
146. In acute cervical spine trauma, which ligament’s disruption is most confidently detected on sagittal fat-
saturated (STIR) MRI sequences?
A. Anterior longitudinal ligament
B. Posterior longitudinal ligament
C. Ligamentum flavum
D. Interspinous ligament
E. Supraspinous ligament
Explanation: (by Perplexity AI, generated with the OpenAI O3 model)
• Fat-saturated sagittal T2/STIR images accentuate fluid and oedema, making the normally low-signal
ligamentum flavum stand out when torn or swollen; its elastic fibres show bright high signal
against suppressed fatty marrow, so disruption is readily recognised.
• The anterior and posterior longitudinal ligaments (A, B) are thin ventral structures best assessed on
routine T2 without fat sat, while the interspinous and supraspinous ligaments (D, E) lie posteriorly
in fatty tissue where STIR can help but partial-volume effects and obliquity often obscure focal injury,
reducing confidence compared with the ligamentum flavum.
EBDR Exam MCQs & Concepts September 2022
Dr. Kareem Alnakeeb
253
147. A 75-year-old woman presents with acute left hip pain after a fall; pelvic radiograph shows the fracture
line running just below the femoral head and above the femoral neck’s midpoint. What type of proximal
femoral fracture is this?
A. Basicervical fracture
B. Intertrochanteric fracture
C. Subcapital fracture
D. Subtrochanteric fracture
E. Transcervical fracture
Explanation: (by Perplexity AI, generated with the OpenAI O3 model)
• The fracture described lies immediately distal to the femoral head within the femoral neck’s most
proximal zone, classically termed a subcapital fracture. Subcapital fractures disrupt the retinacular
vessels and carry the highest risk of avascular necrosis, guiding urgent surgical fixation.
• A basicervical fracture (A) occurs at the base of the femoral neck near the intertrochanteric line, not
just below the head.
• Intertrochanteric fractures (B) extend between the greater and lesser trochanters, sparing the
femoral neck.
• Subtrochanteric fractures (D) lie 5 cm or less distal to the lesser trochanter.
• A transcervical fracture (E) passes through the mid-portion of the femoral neck, lying more distal than
the subcapital location.
148. On a coronal T2-weighted MRI of the femoral head, which imaging feature is classically described as the
“double-line sign” of osteonecrosis?
A. An inner bright serpiginous line with an outer dark serpiginous line at the necrosis–viable bone
interface
B. A single low-signal subchondral crescent parallel to the articular surface
C. Diffuse marrow oedema with loss of normal fatty signal in the femoral neck
D. Patchy high T1 signal within the metaphysis due to fatty infiltration
E. Focal cartilage thinning with underlying subchondral cysts
Explanation: (by Perplexity AI, generated with the OpenAI O3 model)
• The double-line sign is highly specific for osteonecrosis. It appears on fluid-sensitive sequences as
two adjacent serpiginous rims: an inner high T2 signal rim reflecting hyperaemic granulation tissue
and an outer low T2 signal rim representing reactive sclerosis at the border between necrotic and
viable bone, thus fulfilling option A.
• Option B describes the crescent sign of subchondral collapse, not the double-line sign.
• Option C represents non-specific marrow oedema seen in many pathologies including transient
osteoporosis.
• Option D refers to fatty marrow replacement rather than osteonecrosis.
• Option E depicts degenerative osteoarthritis, unrelated to the classic MRI sign of osteonecrosis.
EBDR Exam MCQs & Concepts September 2022
Dr. Kareem Alnakeeb
254
149. A 68-year-old ventilated patient has a right internal jugular venous line inserted for haemodynamic
monitoring. A chest radiograph taken immediately after the procedure is shown. Which anatomical
structure should the tip of a correctly placed Swan–Ganz catheter project over on a frontal chest
radiograph?
A. Superior vena cava
B. Right atrium
C. Main pulmonary artery
D. Inferior vena cava
E. Right ventricular outflow tract
Explanation: (by Perplexity AI, generated with the OpenAI O3 model)
• The thermodilution (Swan–Ganz) catheter is designed to measure pulmonary artery pressures and
cardiac output; correct positioning requires the balloon-tipped tip to advance through the right heart
and lie within the main pulmonary artery, usually just beyond its bifurcation. On the frontal chest
radiograph the tip should project over the hilum at the level of the left or right main pulmonary artery
shadow.
• If the tip remains in the superior vena cava or right atrium (options A and B) it cannot measure
pulmonary artery pressures and risks arrhythmia; in the inferior vena cava (option D) it has not
entered the right heart; placement in the right ventricular outflow tract (option E) may produce
ventricular ectopics and gives falsely elevated systolic readings.
150. A 3-month-old infant presents with fever, irritability and firm swelling over the right mandible; radiographs
show symmetrical, dense subperiosteal new bone along the mandibular body sparing the condyles.
Which diagnosis best explains the imaging findings and clinical picture?
A. Acute osteomyelitis
B. Caffey’s disease (infantile cortical hyperostosis)
C. Langerhans cell histiocytosis
D. Hypervitaminosis A
E. Non-accidental injury
Explanation: (by Perplexity AI, generated with the OpenAI O3 model)
• Caffey’s disease produces a triad of fever, soft-tissue swelling and characteristic lamellated
periosteal new bone confined to diaphyseal cortices, most frequently involving the mandible (≈75%
of cases) with metaphyseal sparing.
• Osteomyelitis (A) usually shows focal cortical destruction and subperiosteal abscess rather than
smooth, symmetric cortical thickening.
• Langerhans cell histiocytosis (C) manifests as lytic “punched-out” lesions, not diffuse hyperostosis.
• Hypervitaminosis A (D) may cause periostitis but typically affects long bones in older children and
lacks mandibular predilection.
• Non-accidental injury (E) demonstrates mixed-age fractures and metaphyseal “corner” injuries, not
uniform subperiosteal new bone.
EBDR Exam MCQs & Concepts September 2022
Dr. Kareem Alnakeeb
255
151. Infarction of the medial frontal and superior medial parietal cortices most commonly indicates occlusion
of which intracranial artery territory?
A. Anterior cerebral artery
B. Middle cerebral artery
C. Posterior cerebral artery
D. Anterior choroidal artery
E. Superior cerebellar artery
Explanation: (by Perplexity AI, generated with the OpenAI O3 model)
• The anterior cerebral artery (ACA) supplies the medial aspects of the frontal lobe—including the
paracentral lobule—and the superior medial parietal lobe. Infarcts in this vascular bed therefore
present with weakness and sensory loss mainly in the contralateral lower limb, reflecting
involvement of the paracentral cortex.
• The middle cerebral artery perfuses the lateral frontal, parietal and temporal lobes, not the medial
surfaces, so MCA infarction is a distractor.
• Posterior cerebral artery strokes involve the occipital and inferior temporal regions.
• The anterior choroidal artery supplies deep structures such as the posterior limb of the internal
capsule and hippocampus, not cortical medial surfaces.
• The superior cerebellar artery only supplies the superior cerebellar hemispheres and midbrain, not
supratentorial cortex.
152. A 35-year-old man presents with recurrent lower respiratory tract infections since childhood. Chest CT
shows a well-defined mass of non-functioning lung tissue in the medial basal segment of the left lower
lobe, supplied by an anomalous artery arising from the descending thoracic aorta and draining via the
pulmonary veins. What is the most likely diagnosis?
A. Bronchogenic cyst
B. Intralobar pulmonary sequestration
C. Extralobar pulmonary sequestration
D. Congenital pulmonary airway malformation (CPAM) type II
E. Pulmonary arteriovenous malformation
Explanation: (by Perplexity AI, generated with the OpenAI O3 model)
• An intralobar sequestration consists of aberrant lung tissue within the visceral pleura of an otherwise
normal lobe, classically fed by a systemic arterial branch (usually from the thoracic or abdominal
aorta) and draining into pulmonary veins—features shown on the CT.
• Extralobar sequestrations are separate from normal lung and have their own pleural covering,
typically presenting in infancy and often located near the posterior costophrenic angle.
• Bronchogenic cysts are fluid-filled, not vascular-supplied masses.
• CPAM type II is a multicystic lesion communicating with airways and supplied by the pulmonary
circulation.
• Pulmonary arteriovenous malformations feature a direct artery-to-vein communication without non-
functioning lung tissue.
EBDR Exam MCQs & Concepts September 2022
Dr. Kareem Alnakeeb
256
153. A 45-year-old man with exertional dyspnoea undergoes cardiac MRI. Cine four-chamber and short-axis
steady-state free-precession images (not shown) demonstrate focal systolic hypokinesia and marked
late gadolinium enhancement confined to the mid-anteroseptal left-ventricular wall; maximal wall
thickness elsewhere is 14 mm. Which diagnosis best explains these findings?
A. Acute myocarditis
B. Dilated cardiomyopathy
C. Hypertrophic cardiomyopathy with myocardial scar
D. Ischaemic infarction of left-anterior-descending territory
E. Restrictive cardiomyopathy
Explanation: (by Perplexity AI, generated with the OpenAI O3 model)
• Hypertrophic cardiomyopathy (HCM) can present with focal regions of hypertrophy or scar, often in
the anteroseptum, producing focal contractile impairment on cine imaging and dense mid-
myocardial late enhancement on post-contrast sequences; overall wall thickness may be only mildly
increased, as here.
• Acute myocarditis (A) typically shows subepicardial or patchy mid-wall LGE in a non-coronary
distribution and is rarely limited to a single septal segment.
• Dilated cardiomyopathy (B) produces global systolic dysfunction with diffuse rather than focal mid-
wall fibrosis.
• Ischaemic infarction (D) gives subendocardial or transmural LGE in a coronary vascular territory and
is accompanied by wall thinning, not hypertrophy.
• Restrictive cardiomyopathy (E) manifests with bi-atrial enlargement and diffuse fibrosis, lacking the
discrete septal scar pattern seen here.
154. Which imaging modality is most sensitive for detecting early skeletal metastases during whole-body
oncologic staging?
A. Plain radiography
B. Whole-body MRI
C. 18
F-FDG PET/CT
D. 99
mTc-MDP planar bone scintigraphy
E. CT skeletal survey
Explanation: (by Perplexity AI, generated with the OpenAI O3 model)
• Whole-body MRI directly visualises tumour marrow infiltration, giving the highest lesion-level
sensitivity (≈94%) for bone metastases and outperforming both 99mTc-MDP bone scans and FDG
PET/CT, particularly for early marrow-based or predominantly lytic deposits.
• Bone scintigraphy (option D) is highly sensitive for osteoblastic activity but misses many early or
purely lytic lesions and has lower overall sensitivity (≈62-78%).
• FDG PET/CT (option C) is excellent for metabolically active disease but is less sensitive for
osteoblastic metastases and may be inferior to MRI on a per-patient basis in several cancers.
• CT surveys (option E) and plain radiographs (option A) require 30-50% bone mineral loss before
lesions are visible, making them least sensitive for early metastatic spread.
EBDR Exam MCQs & Concepts September 2022
Dr. Kareem Alnakeeb
257
155. A term neonate presents with mild tachypnoea; chest radiograph shows marked over-inflation of the left
upper lobe without mediastinal shift. What is the most appropriate initial management of suspected
congenital lobar emphysema in this scenario?
A. Emergency lobectomy
B. High-flow nasal oxygen and urgent CT scan
C. Observation with serial clinical and radiographic follow-up
D. Insertion of an intercostal chest drain
E. Continuous positive airway pressure ventilation
Explanation: (by Perplexity AI, generated with the OpenAI O3 model)
• Mild or asymptomatic congenital lobar emphysema (CLE) often stabilises or improves as the infant
grows; therefore, conservative management with close observation and follow-up imaging is
recommended. Surgery (lobectomy) is reserved for progressive respiratory distress or significant
mediastinal shift, making option A inappropriate here.
• High-flow oxygen and CT (option B) may expose the infant to unnecessary radiation and are not first-
line in a stable patient.
• Chest drains (option D) are contraindicated because CLE is not a pneumothorax and drainage risks
worsening the condition.
• CPAP (option E) can further over-distend the affected lobe and is avoided.
156. Which histologically benign pleural tumour is well‐known for local recurrence and late distant
metastases despite apparently complete surgical excision?
A. Desmoplastic mesothelioma
B. Solitary fibrous tumour of the pleura
C. Pleural lipoma
D. Pleural calcifying fibrous tumour
E. Pleural schwannoma
Explanation: (by Perplexity AI, generated with the OpenAI O3 model)
• Solitary fibrous tumour of the pleura is usually benign on histology yet behaves unpredictably, with
up to one-third of cases showing local recurrence and a smaller proportion developing
haematogenous metastases years after resection.
• Desmoplastic mesothelioma (A) is frankly malignant from the outset.
• Pleural lipoma (C), calcifying fibrous tumour (D) and pleural schwannoma (E) are all benign lesions
that very rarely, if ever, recur or metastasise following complete excision.
157. Which imaging feature is most typically seen when a granular cell tumour of the breast mimics an
aggressive primary carcinoma?
A. Clustered micro-calcification
B. Spiculated margin with posterior acoustic shadowing
C. Circumscribed oval mass with posterior enhancement
D. Complex cystic-solid mass with internal septations
E. Hypervascular mass on colour Doppler
Explanation: (by Perplexity AI, generated with the OpenAI O3 model)
• Granular cell tumours are benign neural-derived lesions but on mammography and ultrasound they
frequently appear malignant. The common aggressive appearance is a small, irregular or spiculated
mass that produces marked posterior acoustic shadowing, closely resembling an invasive ductal
carcinoma.
EBDR Exam MCQs & Concepts September 2022
Dr. Kareem Alnakeeb
258
• Calcifications (A) are uncommon; most lesions are non-calcified.
• Well-circumscribed masses with enhancement (C) represent the less typical benign pattern.
• Complex cystic-solid architecture (D) and prominent Doppler vascularity (E) are not characteristic
and would prompt alternative differentials such as papillary or phyllodes tumours.
158. A 29-year-old woman presents with chest discomfort. Chest radiograph shows a well-circumscribed
anterior mediastinal mass containing both fat density and an air–fluid level. Which is the most likely
diagnosis?
A. Bronchogenic cyst
B. Mature teratoma
C. Thymic cyst
D. Pericardial cyst
E. Lymphoma
Explanation: (by Perplexity AI, generated with the OpenAI O3 model)
• Mature (benign) teratomas classically arise in the anterior mediastinum and often contain a mix of
fat, fluid and occasionally teeth or bone. When internal fat interfaces with secreted fluid, an air–fluid
or fat–fluid level may be seen, a highly characteristic feature pointing toward a teratoma.
• Bronchogenic and thymic cysts are purely fluid-attenuation without macroscopic fat.
• Pericardial cysts sit at the cardiophrenic angle and also lack fat.
• Lymphoma appears soft-tissue attenuating and rarely demonstrates cystic change or fat, making it
the least likely diagnosis here.
159. Which congenital coronary artery anomaly is considered“malignant” because its inter-arterial course
predisposes to sudden cardiac death in young adults?
A. Retro-aortic left circumflex artery
B. Prepulmonic left anterior descending artery
C. Inter-arterial right coronary artery from the left sinus
D. Conus branch arising separately from right sinus
E. High take-off coronary ostia
Explanation: (by Perplexity AI, generated with the OpenAI O3 model)
• A right coronary artery that originates from the left coronary sinus and runs between the aorta and
pulmonary trunk has an inter-arterial (malignant) course. Exercise-related expansion of the great
vessels can compress the slit-like proximal segment, causing ischaemia and sudden cardiac death.
• Retro-aortic and prepulmonic courses (options A and B) lie away from the great vessels and are
regarded as benign.
• A separate conus branch (option D) and high ostial take-off (option E) are normal variants that do not
confer the same risk profile.
160. Following blunt head injury, CT brain shows effacement of the basal cisterns without midline shift. Which
intracranial lesion most commonly produces this appearance?
A. Acute epidural haematoma
B. Acute subdural haematoma
C. Cerebral contusion
D. Intracerebral laceration
E. Diffuse cerebral oedema
EBDR Exam MCQs & Concepts September 2022
Dr. Kareem Alnakeeb
259
Explanation: (by Perplexity AI, generated with the OpenAI O3 model)
• Effacement of the suprasellar and perimesencephalic cisterns in the absence of midline shift
indicates raised intracranial pressure from globally swollen brain rather than from a space-occupying
focal haematoma.
• Diffuse cerebral oedema causes symmetrical brain swelling, narrowing of ventricles and cisterns,
and can progress to brain-stem compression.
• Epidural and subdural haematomas usually create mass effect with midline shift; contusions and
lacerations are focal parenchymal injuries that rarely obliterate basal cisterns unless extensive, and
when large enough they also shift midline structures.
161. A 48-year-old man with longstanding bronchiectasis presents with worsening wrist pain; radiographs
show bilateral, symmetrical periosteal new bone along the distal radius and ulna shafts. What is the
most likely underlying mechanism for this skeletal finding?
A. Autoimmune synovial inflammation
B. Deposition of urate crystals in periosteum
C. Hypertrophic pulmonary osteoarthropathy secondary to intrapulmonary shunting
D. Direct metastatic spread from bronchogenic carcinoma
E. Chronic steroid-induced osteoporosis
Explanation: (by Perplexity AI, generated with the OpenAI O3 model)
• Hypertrophic pulmonary osteoarthropathy (HPOA) results from circulating vasodilatory and growth
factors produced in chronic intrapulmonary shunting, leading to periosteal new-bone formation
along long-bone cortices—classically in patients with bronchiectasis or other chronic lung disease.
• Autoimmune synovitis (A) causes joint space narrowing and erosions rather than smooth laminated
periosteal reaction.
• Urate deposition (B) produces punched-out juxta-articular erosions with overhanging edges, not
diffuse periostitis.
• Metastatic spread (D) gives focal lytic or sclerotic lesions, not uniform cortical layering.
• Steroid-induced osteoporosis (E) causes cortical thinning and fractures, not periosteal apposition.
162. A full-term neonate develops respiratory distress shortly after birth; chest radiograph (frontal view) shows
patchy bilateral opacification with areas of lucency and a flattened hemidiaphragm. What is the most
likely underlying diagnosis?
A. Transient tachypnoea of the newborn
B. Meconium aspiration syndrome
C. Respiratory distress syndrome (hyaline membrane disease)
D. Neonatal pneumonia
E. Congenital lobar overinflation
Explanation: (by Perplexity AI, generated with the OpenAI O3 model)
• Patchy coarse opacities intermixed with hyperlucent, over-inflated lung fields and depression of the
diaphragms in a term baby are classic for meconium aspiration syndrome, reflecting airway
obstruction by particulate meconium.
• Transient tachypnoea typically shows prominent vascular markings and fluid in fissures without over-
inflation.
• Hyaline membrane disease occurs in preterm infants and gives uniform reticulogranular opacities
with air bronchograms, not focal hyperinflation.
EBDR Exam MCQs & Concepts September 2022
Dr. Kareem Alnakeeb
260
• Neonatal pneumonia may mimic hyaline membrane disease or show lobar consolidation but lacks
widespread over-inflation.
• Congenital lobar overinflation produces unilateral lobar hyperlucency rather than diffuse bilateral
changes.
163. A 46-year-old man with post-tuberculous lung destruction presents with sudden massive haemoptysis.
Trans-femoral catheter angiography is planned for emergency embolisation. Which vessel should be
catheterised and assessed first?
A. Internal mammary artery
B. Bronchial artery
C. Pulmonary artery
D. Intercostal artery
E. Superior thoracic artery
Explanation: (by Perplexity AI, generated with the OpenAI O3 model)
• In >90% of massive haemoptysis, including that caused by tuberculosis, bleeding arises from the
high-pressure systemic bronchial circulation rather than the low-pressure pulmonary arteries.
• Bronchial artery embolisation is therefore the primary lifesaving procedure, and selective bronchial
arteriography must be performed first to identify hypertrophied or ectopic bronchial branches for
embolisation.
• The pulmonary artery is an uncommon source; internal mammary, intercostal and superior thoracic
arteries act as collateral feeders but are usually interrogated only if bronchial vessels do not account
for the bleeding.
164. A 24-year-old man with cystic fibrosis attends clinic for routine assessment. High-resolution CT of the
thorax shows widespread upper-lobe bronchiectasis but only mild lower-zone disease. Which pulmonary
function parameter would most closely correlate with the extent of bronchiectasis seen on CT?
A. Total lung capacity (TLC)
B. Forced expiratory volume in 1 s (FEV₁)
C. Diffusion capacity for carbon monoxide (DLCO)
D. Peak expiratory flow rate (PEFR)
E. Residual volume (RV)
Explanation: (by Perplexity AI, generated with the OpenAI O3 model)
• In cystic fibrosis, the severity of structural lung changes—especially bronchiectasis—demonstrated
on HRCT correlates best with the degree of airflow obstruction, quantified by FEV₁.
• Studies show a strong inverse relationship between CT bronchiectasis score and percent-predicted
FEV₁ because dilated, inflamed airways cause expiratory airflow limitation.
• TLC and RV reflect hyperinflation rather than airway damage, and DLCO usually remains near normal
until advanced parenchymal disease.
• PEFR is more effort-dependent and less sensitive to small-airway loss, so it correlates poorly with CT
extent.
EBDR Exam MCQs & Concepts May 2022
Dr. Kareem Alnakeeb
261
May 2022
1. A 35 year old with asthma presents with malaise, flu-like illness and cough. Previous similar episodes
have occurred. A chest radiograph shows patchy airspace opacification in the mid and upper zones.
Which feature on high-resolution CT would make allergic bronchopulmonary aspergillosis a more likely
diagnosis than extrinsic allergic alveolitis?
A. widespread centrilobular micronodules ,3 mm
B. tubular finger-like opacities
C. bronchiectasis
D. upper-zone fibrosis
E. pleural effusion
Source: Bydder, Megan, et al. Get Through Final FRCR Part A: SBAs for the Modular Examination. 1st
ed., Hodder Education Publishers, 2009.
Explanation:
• Allergic bronchopulmonary aspergillosis (ABPA) is hypersensitivity to aspergillus in people with
asthma. Typical features are of a migratory pneumonitis, predominantly in the upper lobes.
• It may cause bronchiectasis and upper-zone fibrosis, which are features also seen in extrinsic
allergic alveolitis (EAA).
• Tubular opacities, indicating mucus plugging, are seen in ABPA, but not in EAA.
• Centrilobular nodules are seen in EAA, along with mosaic perfusion and patchy ground-glass
change.
• Pleural effusions are rarely seen in EAA and not in ABPA.
2. A 56-year-old male presents with wheezing, cough and recurrent chest infections. A chest radiograph
shows right middle lobe consolidation. CT of the chest shows a 3 cm mass arising within the right middle
lobe bronchus with distal collapse and consolidation. Which feature of the mass would make
hamartoma more likely than carcinoid?
A. central location
B. presence of calcification
C. cavitation
D. presence of fat
E. prominent enhancement
Source: Bydder, Megan, et al. Get Through Final FRCR Part A: SBAs for the Modular Examination. 1st
ed., Hodder Education Publishers, 2009.
Explanation:
• Hamartomas are usually seen in the periphery of the lungs (two-thirds) with 10% being
endobronchial. Calcification is seen in 15%, often popcorn type. Cavitation is rare but fat is seen in
50%.
• Carcinoids are usually located centrally and are endobronchial. Calcification is seen in one-third
and they rarely cavitate. They do not contain fat and show prominent enhancement following
contrast, as they are vascular.
EBDR Exam MCQs & Concepts May 2022
Dr. Kareem Alnakeeb
262
3. An 80-year-old man presents with hemoptysis and a mass on chest radiograph. A biopsy shows non-
small-cell lung cancer. CT of chest shows a 4 cm, right middle lobe mass with pleural tethering but no
chest wall invasion. Lymph nodes are seen at the right hilum (17 mm short axis), in the subcarinal space
(20 mm short axis) and in the aortopulmonary space (8 mm short axis). No other abnormalities are seen.
What is the TNM stage?
A. T2 N1 M0
B. T2 N2 M0
C. T2 N3 M0
D. T3 N1 M0
E. T3 N2 M0
Source: Bydder, Megan, et al. Get Through Final FRCR Part A: SBAs for the Modular Examination. 1st
ed., Hodder Education Publishers, 2009.
Explanation:
• The T stage is T2, as the lesion is over 3cm but there is no chest wall or mediastinal invasion or other
associated feature.
• The nodes at the right hilum (N1) and in the subcarinal space (N2) are significantly enlarged, whereas
the node in the aortopulmonary space (N3) is not (,10mm short axis), hence the N stage is N2.
4. In acute respiratory distress syndrome what is the first change usually seen on the chest radiograph?
A. confluent consolidation
B. pleural effusions
C. increased heart size with globular shape
D. volume loss with atelectasis
E. patchy ill-defined opacities
Source: Bydder, Megan, et al. Get Through Final FRCR Part A: SBAs for the Modular Examination. 1st
ed., Hodder Education Publishers, 2009.
Explanation:
• Acute respiratory distress syndrome (ARDS) commences with interstitial oedema, progressing to
congestion and extensive alveolar, and interstitial oedema and hemorrhage.
• The chest radiograph is often normal for the first 24 hours, before patchy opacities appear in both
lungs.
• These progress to massive airspace consolidation over the following 24–48 hours.
• True volume loss, atelectasis, cardiomegaly and effusions are not seen in ARDS.
5. A 16-year-old male presents with sudden shortness of breath. A chest radiograph shows multiple,
bilateral nodules measuring up to 3 cm, some of which are calcified. There is a moderate left
pneumothorax. The patient has been undergoing treatment for a malignant tumor. What is the most likely
diagnosis?
A. metastases secondary to Wilms’ tumor
B. metastases secondary to osteosarcoma
C. metastases secondary to testicular tumor
D. abscesses secondary to immunosuppression
E. varicella pneumonia secondary to immunosuppression
EBDR Exam MCQs & Concepts May 2022
Dr. Kareem Alnakeeb
263
Source: Bydder, Megan, et al. Get Through Final FRCR Part A: SBAs for the Modular Examination. 1st
ed., Hodder Education Publishers, 2009.
Explanation:
• Osteosarcoma pulmonary metastases are uncommon (seen in 2% of cases) and present as multiple
masses which may calcify. There is a high incidence of associated pneumothorax.
• Wilms’ tumors may also produce multiple pulmonary masses and may be associated with
pneumothorax, but are not known to calcify.
• Testicular tumors may produce calcified lung metastases, but are not associated with
pneumothorax.
• Varicella pneumonia shows patchy consolidation in the acute phase, with multiple, small, calcified
nodules in the chronic phase.
• Abscesses may present as multiple masses but rarely calcify and often cavitate.
6. A 56-year-old female patient presents with shortness of breath. A chest radiograph is unremarkable. A
high-resolution CT scan is performed which shows mosaic perfusion with no air trapping on expiratory
scan. What is the most likely diagnosis?
A. bronchiolitis obliterans
B. cystic fibrosis
C. hypersensitivity pneumonitis
D. chronic thromboembolic disease
E. asthma
Source: Bydder, Megan, et al. Get Through Final FRCR Part A: SBAs for the Modular Examination. 1st
ed., Hodder Education Publishers, 2009.
Explanation:
• Mosaic perfusion is caused by abnormalities of ventilation, or vascular obstruction.
• Expiratory scans help to distinguish causes by establishing whether there is air trapping.
• With no air trapping present, pulmonary emboli of any cause are most likely.
• Air trapping would suggest airway disease such as bronchiolitis obliterans, or other causes of small
airway obstruction such as bronchiectasis or cystic fibrosis.
7. A known MS patient has presented to the neurologist with clinical features of involvement of the spinal
cord. An MRI of the whole spine has been requested with a view towards assessment of the cord for
possible multiple sclerosis (MS) plaques. MS lesions in the spinal cord occur most commonly in the
A. Cervical segment.
B. Thoracic segment.
C. Lumbar segment.
D. Sacral segment.
E. All segments are equally affected.
Source: Chin, Teck Yew, et al. Get Through Final FRCR 2A: SBAs. 1st ed., CRC Press, 2017.
Explanation:
Multiple Sclerosis Spinal Cord Lesions
• MS can show multiple lesions in the spinal cord.
• Typical spinal cord lesions in MS are relatively small and peripherally located.
• They are most often found in the cervical cord and are usually less than two vertebral segments in
length.
EBDR Exam MCQs & Concepts May 2022
Dr. Kareem Alnakeeb
264
8. A 30-year-old male smoker presents with a history of acute dyspnea. CXR shows bilateral reticulo-
nodular interstitial changes, predominantly in the upper and mid zones, with preservation of lung volume.
There is a right sided apical pneumothorax and a small right pleural effusion. HRCT of chest shows
complex thin- and thick-walled cysts and irregular centrilobular nodules in a similar distribution with
sparing of the bases. The intervening lung appears normal. What is the diagnosis?
A. Lymphangioleiomyomatosis.
B. Bronchiectasis.
C. Metastases.
D. Pulmonary Langerhans cell histiocytosis.
E. Idiopathic pulmonary fibrosis.
Source: Lindsay, Richard, et al. SBAs for the FRCR Part 2A. 1st ed., Oxford University Press, 2012.
Explanation:
Overview and Symptoms
• This is a rare isolated form of Langerhans cell histiocytosis that primarily affects young adult
smokers.
• Most patients are symptomatic and the most frequent symptoms are non-productive cough (50–70%
of cases) and dyspnea (35–87%).
• Less common symptoms include fatigue, weight loss, pleuritic chest pain, and fever.
Chest X-Ray Findings
• The most common finding on CXR is small irregular nodules, usually bilaterally symmetric, with
upper lobe predominance and sparing of the costophrenic angles.
• Coarse reticular and reticulo-nodular pattern is seen in later stages.
Complications and Lung Volumes
• Pneumothorax occurs in up to 25% (may be recurrent).
• Pleural effusion is uncommon, but may occur with pneumothorax.
• Lung volumes are normal or increased in most patients.
High-Resolution CT Findings
• HRCT of chest demonstrates the cysts and nodules in a characteristic distribution with normal
intervening lung.
• Interstitial fibrosis and honeycombing are seen in advanced stages.
Treatment
• Treatment consists of smoking cessation; steroids may be useful in selected patients.
• Chemotherapeutic agents and lung transplantation may be offered in advanced disease.
Prognosis
• The prognosis is variable.
• Stable disease is seen in up to 50%.
• Spontaneous regression is reported in up to 25%.
• A variably progressive, deteriorating course is seen in up to 25%.
EBDR Exam MCQs & Concepts May 2022
Dr. Kareem Alnakeeb
265
9. A 75-year-old man presents with worsening shortness of breath. He was a mine worker. A chest
radiograph shows multiple nodules in the upper zones with a large upper-zone mass on the left. CT
confirms multiple small nodules up to 5 mm with a sausage-shaped mass paralleling the mediastinum.
What is the most likely diagnosis?
A. coal worker’s pneumoconiosis with bronchogenic carcinoma
B. coal worker’s pneumoconiosis with progressive massive fibrosis
C. tuberculosis
D. primary lung carcinoma with metastases
E. chronic extrinsic allergic alveolitis
Source: Bydder, Megan, et al. Get Through Final FRCR Part A: SBAs for the Modular Examination. 1st
ed., Hodder Education Publishers, 2009.
Explanation:
Occupational Lung Diseases
• The history of mining (dust exposure) with small nodules in the upper zones is typical of
pneumoconiosis.
• The sausage-shaped mass is characteristic of progressive massive fibrosis (PMF), although
malignancy cannot be excluded. PMF is often seen to have reduced nodularity surrounding it, as it
incorporates the surrounding nodules and migrates towards the hilum.
Infectious Conditions
• Tuberculosis usually produces a generalized distribution of 2–3 mm nodules in its miliary form, often
with lymphadenopathy.
Neoplastic Processes
• Metastases from primary lung cancer are not uniformly small (though they can be in thyroid cancer
and melanoma).
Hypersensitivity-Related Disorders
• Chronic extrinsic allergic alveolitis produces fibrotic changes in the mid and lower zones.
10. A 67-year-old man has been rushed to the stroke unit with features of acute stroke. All of the following are
true about acute stroke imaging, except
A. CT source images correlate with infarct volume.
B. Matched CBV (Cerebral blood volume) and CBF (Cerebral blood flow) represent salvageable
brain.
C. Diffusion-weighted MR imaging assesses the infarct core.
D. Mismatch between PWI (Perfusion weighted imaging) and DWI (Diffusion weighted imaging) volumes
represents salvageable brain.
E. T2 shine through is seen as bright on DWI.
Source: Chin, Teck Yew, et al. Get Through Final FRCR 2A: SBAs. 1st ed., CRC Press, 2017.
Explanation:
CT Perfusion and Angiography
• An important advance in stroke imaging is the development of CT perfusion imaging.
• CT angiography source images (CTA-SI) represent cerebral blood volume that is reduced in the core
infarct and correlates with infarct volume as seen on DWI (Diffusion Weighted Imaging).
Perfusion Parameters
• CBF (cerebral blood flow), CBV (cerebral blood volume), and MTT (mean transit time) are three
parameters that can distinguish infarcted tissue from potentially salvageable penumbra.
EBDR Exam MCQs & Concepts May 2022
Dr. Kareem Alnakeeb
266
• Ischemic but non-infarcted tissue will have decreased CBF, elevated MTT, and normal or high CBV
(mismatch).
• Once infarcted, there will also be a persistent decrease in CBV (matched defect).
Diffusion-Weighted Imaging (DWI)
• Sensitivity and specificity of DWI for stroke detection is very high.
• DWI bright signals do not necessarily represent irreversibly infarcted tissue but reflect redistribution
of water from the extracellular to the intracellular space in ischemic tissue.
• It is necessary to analyze maps of ADC (apparent diffusion coefficient) to distinguish the effects of
reduced water diffusibility (dark on ADC) from T2 ‘shine-through’ (bright on ADC).
• Both features lead to the DWI bright signals seen in ischemia.
Perfusion–Diffusion Mismatch
• The volumetric mismatch between the PWI and DWI volumes is a marker of potentially salvageable
tissue at risk. Overall DWI provides the best estimate of infarcted core.
11. The absence of which of the following indicates a diagnosis of Dandy-Walker variant rather than Dandy-
Walker malformation?
A. Dysgenesis of the corpus callosum
B. Holoprosencephaly
C. Cerebellar heterotopia
D. Enlargement of the pituitary fossa
E. Cerebellar gyri malformation
Source: Leen, Edward. Succeeding in the New FRCR Part 2A Exam: Single Best Answer (SBA)
Revision Questions for Modules 1-6. 1st ed., UNKNO, 2011.
Explanation:
• The other features are common to both.
12. A CT brain of a 25-year-old male with a head injury but no focal neurology shows no acute abnormality. A
thin cerebrospinal fluid (CSF) density is noted between the frontal horns of the lateral ventricles in the
midline. Which is the diagnosis?
A. Cavum septi pellucidi
B. Cavum vergae
C. Cavum veli interpositii
D. Colloid cyst
E. Arachnoid cyst
Source: Leen, Edward. Succeeding in the New FRCR Part 2A Exam: Single Best Answer (SBA)
Revision Questions for Modules 1-6. 1st ed., UNKNO, 2011.
Explanation:
• Occurs in 80% of term infants and 15% of adults.
• Rarely may dilate and cause obstructive hydrocephalus.
EBDR Exam MCQs & Concepts May 2022
Dr. Kareem Alnakeeb
267
13. MRI brain in a neonate shows multiple abnormalities including an anterior interhemispheric fissure
adjoining a high riding third ventricle, an enlarged foramen of monro and a sunburst gyral pattern. An
interhemispheric cyst is also seen. Which is the diagnosis?
A. Arachnoid cyst
B. Agenesis of the corpus callosum
C. Prominent cavum septum pellucidi and vergae
D. Chiari II malformation
E. Dandy-Walker malformation
Source: Leen, Edward. Succeeding in the New FRCR Part 2A Exam: Single Best Answer (SBA)
Revision Questions for Modules 1-6. 1st ed., UNKNO, 2011.
Explanation:
• Absence of septum pellucidum, corpus callosum and cavum septum pellucidi, and wide separation
of the lateral ventricles, are other features of agenesis of the corpus callosum.
14. A 50-year-old man who presented with progressive dyspnea had a chest radiograph that demonstrated
multiple opacities between 0.5 and 2 mm in size, which were noted to be more dense than soft tissue.
Which of the following diagnoses is most likely?
A. Fungal infection such as histoplasmosis
B. Coal miners' pneumoconiosis
C. Sarcoidosis
D. Acute extrinsic allergic alveolitis
E. Silicosis
Source: Proctor, Robin. Final FRCR Part A Modules 1–3 Single Best Answer MCQs: The SRT Collection
of 600 Questions with Explanatory Answers (MasterPass). 1st ed., CRC Press, 2009.
Explanation:
• All the other conditions would produce nodules of soft tissue density.
• In pure silicosis the nodules are very well defined and very dense.
• There is also relative sparing of the bases and apices with septal lines on HRCT.
15. A 45-year-old female presented with progressive dyspnoea. A chest radiograph demonstrated reticular
densities with preserved lung volumes. An HRCT showed uniform cysts throughout the lung with normal
intervening lung. What is the most likely diagnosis?
A. Tuberous sclerosis
B. Lymphangiomyomatosis
C. Histiocytosis
D. Tuberculosis
E. Cystic fibrosis
Source: Proctor, Robin. Final FRCR Part A Modules 1–3 Single Best Answer MCQs: The SRT Collection
of 600 Questions with Explanatory Answers (MasterPass). 1st ed., CRC Press, 2009.
Explanation:
• LAM is seen exclusively in women, typically those of child-bearing age.
• The typical appearances are of thin-walled cysts with normal intervening lung.
• There may also be small pneumothoraces and chylous effusions.
EBDR Exam MCQs & Concepts May 2022
Dr. Kareem Alnakeeb
268
16. You have been asked to review a chest radiograph by a junior doctor. The image demonstrates subtle hazy
opacification of the upper part of the lower zone of the right lung. The right atrial border is indistinct and
the horizontal fissure runs from the right hilum to the eighth rib in the mid axillary line. What is the most
plausible explanation for these findings?
F. Middle lobe collapse
G. Middle lobe consolidation
H. Pectus excavatum
I. Right lower lobe mediobasal segment consolidation
J. Right lower lobe anteriobasal segment consolidation
Source: Proctor, Robin. Final FRCR Part A Modules 1–3 Single Best Answer MCQs: The SRT Collection
of 600 Questions with Explanatory Answers (MasterPass). 1st ed., CRC Press, 2009.
Explanation:
• The findings are those of middle lobe collapse.
• Signs on the frontal radiograph can be subtle, and it is more easily seen on the lateral radiograph.
• In this case the loss of clarity of the right atrial border indicates the pathology is located in the middle
lobe.
• There is loss of volume (the normal horizontal fissure runs from the hilum to the sixth rib in the mid
axillary line), therefore collapse of the middle lobe, rather than consolidation, is the likely cause for
these appearances.
17. You are asked to review a chest radiograph following a pacemaker insertion. The leads have been placed
via the left subclavian approach and pass down the left mediastinal border before forming a loop with the
tip projected over the right ventricle. What is the most likely explanation?
A. Partial anomalous pulmonary venous return
B. An atrial septal defect
C. A persistent left superior vena cava
D. A ventricular septal defect
E. Normal appearance, no abnormality
Source: Proctor, Robin. Final FRCR Part A Modules 1–3 Single Best Answer MCQs: The SRT Collection
of 600 Questions with Explanatory Answers (MasterPass). 1st ed., CRC Press, 2009.
Explanation:
• A persistent left superior vena cava (SVC) courses along the left mediastinal border and enters the
coronary sinus, which is usually dilated. There is more often than not a right SVC as well.
18. A 64-year-old non-smoker presents to his GP with progressive dyspnea. His chest radiograph
demonstrates a peripheral lung mass. What is the most likely histological type of carcinoma?
A. Squamous cell carcinoma
B. Small cell lung carcinoma
C. Bronchoalveolar cell carcinoma
D. Large cell carcinoma
E. Adenocarcinoma
Source: Proctor, Robin. Final FRCR Part A Modules 1–3 Single Best Answer MCQs: The SRT Collection
of 600 Questions with Explanatory Answers (MasterPass). 1st ed., CRC Press, 2009.
Explanation:
• This is simply a question of incidence. It is the most common type of lung carcinoma and is also the
most common in non-smokers.
EBDR Exam MCQs & Concepts May 2022
Dr. Kareem Alnakeeb
269
19. A 62 year old man presents with right shoulder pain which radiates down his arm. A plain radiograph
confirms the presence of a right apical mass with destruction of the surrounding ribs. CT-guided biopsy is
performed and is likely to reveal:
F. Large cell lung cancer
G. Squamous cell cancer
H. Small cell lung cancer
I. Adenocarcinoma
J. Carcinoid
Source: Currie, Stuart, et al. SBAs for the FRCR 2A. 1st ed., Cambridge University Press, 2010.
Explanation:
• The case describes a Pancoast tumor for which squamous is the most common cell type.
20. A 60 year old female underwent a right pneumonectomy for bronchogenic carcinoma. Which feature on
plain chest radiograph would be a cause of worry seven days after surgery?
A. A sequential increase in the fluid level
B. Shift of the previously central trachea to the right
C. Shift of the previously central trachea to the left
D. Elevation of the right hemi-diaphragm
E. Shift of the cardiac silhouette to the right
Source: Currie, Stuart, et al. SBAs for the FRCR 2A. 1st ed., Cambridge University Press, 2010.
Explanation:
• All the other changes are expected changes at this stage following a pneumonectomy.
• However, contralateral shift of the trachea may be indicative of a post-surgical bronchopleural
fistula.
21. A 52-year-old woman is involved in a RTA. Trauma series radiographs reveal a complex pelvic fracture.
There are no other appreciable injuries. She is hemodynamically unstable, and fluid resuscitation is
commenced. Which of the following steps is the next most important?
A. Trauma protocol CT scan with angiographic sequences
B. Immediate surgery under the orthopedic surgeons
C. Discussion with interventional radiologists with view to catheter angiography and possible
intervention
D. Placement of a pelvic wrap device
E. Blood transfusion
Source: Chin, Teck Yew, et al. Get Through Final FRCR 2A: SBAs. 1st ed., CRC Press, 2017.
Explanation:
Pelvic Wrap Device
• All of these steps may be required, but the most important is to place a pelvic wrap device.
• The purpose of this device is to stabilize the pelvis.
Hemorrhage Management
• If the patient is hypotensive as a result of venous bleeding, a pelvic wrap should stabilize the pelvis
sufficiently to cause significant reduction or cessation in the venous hemorrhage and thus avoid
unnecessary endovascular intervention.
• If this fails to achieve sufficiently prompt hemodynamic stability, there is a significant chance that
there is arterial hemorrhage; hence endovascular intervention is likely to be necessary.
Imaging and Endovascular Planning
EBDR Exam MCQs & Concepts May 2022
Dr. Kareem Alnakeeb
270
• Many centers use angiographic sequences as part of the trauma CT scan to identify such a bleeding
point as part of planning stage of endovascular management.
• CT would be the next step once the pelvic binder is in situ.
22. A 30-year-old teacher presents with thyroid swelling and proptosis. Which of the following is true
regarding Grave’s ophthalmopathy?
A. It commonly involves the medial rectus first.
B. It commonly involves the tendons of the eye muscles.
C. There is associated dilatation of the superior ophthalmic vein.
D. There is decreased density of the orbital fat.
E. It is an autoimmune disease unrelated to thyroid function.
Source: Chin, Teck Yew, et al. Get Through Final FRCR 2A: SBAs. 1st ed., CRC Press, 2017.
Explanation:
Overview
• Graves’ disease of the orbit is also known as thyroid ophthalmopathy or endocrine exophthalmos.
• It is produced by long-acting thyroid-stimulating hormone, probably due to cross-reactivity against
antigens shared by thyroid and orbital tissue.
• Signs and symptoms usually develop within 1 year of hyperparathyroidism.
• It is the most common cause of unilateral/bilateral proptosis in adults.
Extraocular Muscle Involvement
• It commonly involves the inferior rectus first (mnemonic – I’M SLOW).
o Inferior rectus
o Medial rectus
o Superior rectus
o Lateral rectus
o Obliques
Orbital and Vascular Changes
• The superior ophthalmic vein is dilated due to compromised orbital venous drainage at the orbital
apex.
• There is increased density of the orbital fat late in the disease.
23. A 33-year-old asymptomatic patient has been found to have cervical carcinoma and is undergoing a
staging MRI examination. What finding on MRI is most reliable in excluding parametrial invasion?
A. Absence of abnormal pelvic lymphadenopathy
B. No hydronephrosis or hydroureter on either side
C. Tumor volume of less than 90 cu cm
D. Hypointense rim of cervical stroma
E. Fine nodularity of parametrial tissue
Source: Proctor, Robin. Final FRCR Part A Modules 4-6 Single Best Answer MCQs: The SRT Collection
of 600 Questions with Explanatory Answers (MasterPass). 1st ed., CRC Press, 2009.
Explanation:
• A continuous hypointense rim representing the cervical ring measuring >3 mm is most reliable in
excluding parametrial invasion with a quoted specificity of 96-99%.
• Nodularity and thickening of the parametrial tissue are signs of frank invasion.
EBDR Exam MCQs & Concepts May 2022
Dr. Kareem Alnakeeb
271
24. Which structure lies immediately posterior to the standard fenestration site during an endoscopic third
ventriculostomy for aqueductal stenosis?
A. Basilar artery apex
B. Optic chiasm
C. Anterior commissure
D. Fornix columns
E. Pineal gland
Explanation: (by Perplexity AI, generated with the OpenAI O3 model)
• The stoma is fashioned in the floor of the third ventricle just anterior to the paired mammillary
bodies; the interpeduncular cistern—and crucially the basilar artery apex with its perforating
branches—lies directly beneath this thin ventricular floor. Awareness of this vascular landmark
prevents catastrophic hemorrhage.
• The optic chiasm (B) sits anterior-superior to the fenestration site, the anterior commissure (C) is
rostral within the third ventricle roof, the fornix columns (D) run in the anterior roof, and the pineal
gland (E) is posterior to the aqueduct, not beneath the floor fenestration
• Third ventriculostomy:
o type of surgical treatment for obstructive hydrocephalus, especially when obstruction is
located at the level of the aqueduct of Sylvius (e.g. aqueduct stenosis).
o A permanent defect is created in the floor of the third ventricle anterior to the mammillary
bodies, thus connecting the third ventricle with the interpeduncular cistern.
25. The gold standard diagnostic modality to evaluate the coronary arteries grafts is
A. Percutaneous coronary angiography
B. Echocardiography
C. CT coronary angiography
D. MR Coronary angiography
Explanation: (by Perplexity AI, generated with the OpenAI O3 model)
• Percutaneous (invasive) coronary angiography – it remains the gold-standard test for assessing
the patency and anatomy of coronary artery bypass grafts
26. The reason that coronary bypass grafts are superior to native coronary arteries for CТА is:
A. They are less influenced by cardiac motion
B. They have a wider luminal diameter
C. They contain less luminal calcification
D. They follow a straighter course
E. All of the above
Explanation: (by Perplexity AI, generated with the OpenAI O3 model)
• Bypass grafts run in the mediastinum, away from the rapidly moving epicardial surface, so motion
artefact is markedly reduced. Their surgical caliber (≈4–6 mm) is larger than that of native coronary
vessels (≈2–4 mm), making intraluminal contrast opacification more homogeneous and partial-
volume artefact less problematic.
• In addition, graft walls rarely develop the heavy, nodular calcification typical of long-standing
atherosclerotic coronary arteries, improving luminal visualization.
• While each individual feature aids image quality, the combination of reduced motion, larger caliber
and lesser calcification together accounts for the consistently higher diagnostic accuracy of CT graft
studies, making “all of the above” the single best answer.
EBDR Exam MCQs & Concepts May 2022
Dr. Kareem Alnakeeb
272
• Key distractors: options A–C are each true but incomplete explanations; option D is plausible yet less
impactful than the three principal factors above.
27. When performing CT coronary angiography to evaluate stent patency, which technical adjustment is
most effective at reducing blooming artefacts from the metallic stent struts?
A. Increase the tube voltage to 140 kV
B. Reconstruct images with a sharp convolution kernel
C. Acquire the study with retrospective ECG gating
D. Use prospective ECG-triggered high-pitch acquisition
E. Reconstruct slices at 1.5 mm thickness
Explanation: (by Perplexity AI, generated with the OpenAI O3 model)
• Blooming arises mainly from partial-volume averaging and beam-hardening around the highly
attenuating stent struts. Applying a high-spatial-frequency (sharp) reconstruction kernel narrows the
point-spread function, improving edge definition and markedly reducing apparent strut thickness, so
the true lumen is better visualized.
• Raising kV (Option A) increases photon penetration but does little for spatial resolution.
• Retrospective (Option C) and prospective (Option D) ECG strategies influence motion artefact and
radiation dose, not blooming.
• Thicker slices (Option E) exacerbate partial-volume effects and increase blooming, doing the
opposite of what is desired.
28. The specific technical issue in CT imaging of coronary stents is:
A. Blooming artifacts due to beam hardening
B. Respiratory motion artifacts
C. Cardiac motion artifacts,
D. The use of high-density contrast material
Explanation: (by Perplexity AI, generated with the OpenAI O3 model)
• CT assessment of the stent lumen is primarily hampered by blooming artifacts—an apparent
enlargement of the metallic struts produced by partial-volume effects and beam-hardening, which
obscures the true lumen diameter and can mimic or hide in-stent restenosis.
29. A young patient is followed up for a fractured tibia at the outpatient clinic. A repeat radiograph is
acquired, which shows abnormal healing and callus formation at the fracture site. All the following are
possible causes, except
A. Cushing’s syndrome
B. Osteogenesis imperfecta
C. Osteopoikilosis
D. Paralytic state
E. Asthmatic on steroids
Source: Chin, Teck Yew, et al. Get Through Final FRCR 2A: SBAs. 1st ed., CRC Press, 2017.
Explanation:
Causes of Impaired Fracture Healing
• Patients with comorbidities like diabetes, anemia and malnutrition can suffer from impaired bone
fracture healing.
• Drug therapy like corticosteroids and NSAIDs can also produce similar problems.
Osteogenesis Imperfecta
• Osteogenesis imperfecta is a connective-tissue disorder with resultant abnormal bone density and
structure, resulting in poor mineralization and fragile, brittle bones.
EBDR Exam MCQs & Concepts May 2022
Dr. Kareem Alnakeeb
273
Osteopoikilosis
• Osteopoikilosis is a benign condition and usually found incidentally.
• It is a form of sclerosing bone dysplasia with multiple enostoses.
• It is not associated with impaired fracture healing.
30. A male infant is born at 39 + 3 weeks gestation. Prenatal ultrasound demonstrated a partly cystic, partly
echogenic mass in the right upper lobe. Shortly after delivery the infant is in respiratory distress. Initial
chest X-ray demonstrates dense lungs bilaterally with increased volume on the right. On Day 2, a repeat
chest X-ray demonstrates multiple air-filled cystic masses of varying sizes within the right upper lobe with
mediastinal shift to the left. What is the most likely diagnosis?
A. Bronchogenic cyst
B. Morgagni hernia
C. Congenital cystic adenomatoid malformation
D. Congenital lobar emphysema
E. Hyaline membrane disease
Source: Chin, Teck Yew, et al. Get Through Final FRCR 2A: SBAs. 1st ed., CRC Press, 2017.
Explanation:
Definition
• Congenital cystic adenomatoid malformation is a developmental hamartomatous abnormality of
lung with adenomatoid proliferation of cysts resembling bronchioles.
Pathogenesis
• It is thought to be caused by focal arrest in fetal lung development before the seventh week of
gestation.
Incidence
• Congenital cystic adenomatoid malformation represents 25% of all congenital lung lesions.
Postnatal Chest X-ray
• A CXR on Day 1 of life usually demonstrates dense lungs with increased volume on the affected side.
• On Day 2, a CXR usually demonstrates resorption of fluid from affected areas of lung, which are then
replaced with air-containing spaces.
Anatomical Characteristics
• Communication with the tracheobronchial tree is maintained and the vascular supply and drainage
are to the pulmonary circulation.
• There is a slight predilection for the upper lobes.
Clinical Presentation
• Newborns often present with respiratory distress secondary to mass effect and pulmonary
compression or hypoplasia.
• The chest is dull to percussion with decreased air entry.
Prenatal Ultrasound Findings
• Prenatal ultrasound shows a partly cystic, partly echogenic mass.
31. A 74-year-old man presents with neck pain, with right upper-arm pain and radicular symptoms at the
lateral aspect of the forearm and tingling in the thumb. What is the most likely finding on the MRI?
A. Central disc bulge at C3/4 with severe cord compression
B. Right foraminal disc osteophyte at C2/3
C. Right foraminal disc osteophyte at C4/5
D. Right foraminal disc osteophyte at C5/6
E. Right foraminal disc osteophyte at C3/4
EBDR Exam MCQs & Concepts May 2022
Dr. Kareem Alnakeeb
274
Source: Chin, Teck Yew, et al. Get Through Final FRCR 2A: SBAs. 1st ed., CRC Press, 2017.
Explanation:
Dermatomes and Nerve Root Impingement
• Lateral aspect of the forearm and the thumb corresponds to the C6 dermatome.
• Foraminal osteophyte at C5/6 will impinge upon the exiting C6 nerve root.
• (cf. foraminal osteophyte at a thoracic or lumbar level, e.g., T4/5 or L4/5, which will impinge upon the
exiting T4 or L4 nerve roots, subject to the discrepancy between number of cervical vertebrae and
cervical roots. Note that the exiting root at C7/T1 is C8.)
32. A 35-year-old man involved in a major RTA undergoes a lateral view of the cervical spine in the resus on
arrival. All of the following are features associated with atlanto-occipital dislocation, except
F. Soft-tissue swelling anterior to C2 by >10 mm.
G. Basion dens interval >12 mm.
H. Odd’s ratio >1.
I. X-ray can often be normal.
J. Incongruity of articular surface of atlas and occipital condyles.
Source: Chin, Teck Yew, et al. Get Through Final FRCR 2A: SBAs. 1st ed., CRC Press, 2017.
Explanation:
Lateral Radiograph Findings
• Atlanto-occipital dislocation shows the following on lateral radiograph of the cervical spine:
• 10 mm soft-tissue swelling anterior to C2, with pathological convexity (80%),
• basion-dens interval of >12 mm,
• odds ratio (distance between the basion and the posterior arch of the atlas divided by opisthion and
anterior arch of atlas) >1, and
• basion–posterior axial line interval >12 mm anterior/>4 mm posterior to axial line.
Direct Radiographic Signs
• Direct signs include loss of congruity of articular surfaces of atlas and occipital condyle.
EBDR Exam MCQs & Concepts May 2022
Dr. Kareem Alnakeeb
275
Diagnostic Caveat
• Normal X-ray in the presence of atlanto-occipital dislocation is rare.
33. Chest X-ray of a boy shows shift of the heart and mediastinum to the right. There is also a tubular
structure parallel to the right heart border with its maximum width close to the diaphragm. The finding
suggests
A. ASD
B. Scimitar syndrome
C. Total anomalous pulmonary venous return
D. Intralobar sequestration
E. Inhaled foreign body
Source: Chin, Teck Yew, et al. Get Through Final FRCR 2A: SBAs. 1st ed., CRC Press, 2017.
Explanation:
Overview
• Hypogenetic lung syndrome, also known as congenital venolobar syndrome or scimitar syndrome, is
primarily a complex developmental lung abnormality with anomalous venous return.
Key Features
• The most common features are lung hypoplasia, anomalous pulmonary venous return to IVC,
pulmonary artery hypoplasia, bronchial anomalies and systemic arterial supply to hypoplastic lung.
Epidemiology
• It almost always occurs on the right side and is slightly more common in women.
Venous Drainage Patterns
• One constant component of this syndrome is an anomalous pulmonary vein or veins draining at least
a part or the entire affected lung most commonly to the inferior vena cava just above or below the
diaphragm.
• Uncommonly, the anomalous vein may drain into hepatic, portal, azygos veins; the coronary sinus; or
the right atrium.
Imaging Findings
• A scimitar vein is a vertical curvilinear opacity in the right mid-lower lung, running along the right
heart border inferomedially towards the diaphragm to join the IVC. A scimitar vein present on a
frontal chest radiograph is called the scimitar sign.
34. A 67-year-old woman with 5.5 cm atherosclerotic abdominal aortic aneurysm is being worked up for a
potential aortic endograft repair. All of the following are important imaging observations to be determined
prior to treatment, except
A. Tortuosity of the aorta
B. Diameter of aortic aneurysms
C. Non-thrombosed residual lumen of the aneurysm
D. Flow characteristic at the aneurysm neck
E. Length of proximal and distal landing zones
Source: Chin, Teck Yew, et al. Get Through Final FRCR 2A: SBAs. 1st ed., CRC Press, 2017.
Explanation:
Aneurysm Definition and Repair Thresholds
• An aneurysm occurs when a vessel diameter exceeds 1.5 times its normal size. In the abdomen this
corresponds to 3 cm.
• These aneurysms should be repaired when the diameter exceeds 5–5.5 cm or the aneurysm expands
more than 1 cm per year.
EBDR Exam MCQs & Concepts May 2022
Dr. Kareem Alnakeeb
276
• Study results suggest higher complications for aneurysms larger than 5.5–6.5 cm. The shape can be
described as saccular or fusiform. The residual lumen through the aneurysm should measure
approximately 18 mm to allow passage and proper deployment of the device.
Imaging Considerations
• The greatest benefit of 3D volume-rendered imaging is the depiction and precise measurement of
angulation in aneurysms with marked tortuosity.
Proximal Landing Zone (Neck)
• The proximal landing zone consists of the region from the inferior-most renal artery to the beginning
of the aneurysm.
• The maximal acceptable neck diameter is 32 mm. The length of the neck should be at least 15 mm
(although one device allows a 7 mm neck).
• The angle between the superior portion of the aneurysm neck and the suprarenal aorta is preferably
less than 60°.
Distal Landing Zone (Iliac)
• The preferred distal landing zone is the common iliac artery.
• Evaluation is similar to that of the proximal neck with assessment of diameter, length, tortuosity and
degree of calcification and thrombus.
• The common iliac artery diameter should not be larger than 25 mm, and at least 10 mm of length is
required for an adequate seal.
35. A 72-year-old man with a known malignancy undergoes a spinal MRI for characterization of multilevel
vertebral collapse identified on plain radiograph. All of the following are true about malignant vs
osteoporotic causes of vertebral fractures, except
A. The involved vertebra are low on T1W images.
B. Paraspinal mass is useful in differentiating metastatic from benign fracture.
C. DWI can differentiate between malignant and benign compression fracture.
D. Posterior bulging of collapsed vertebral body suggests metastasis.
E. Acute osteoporotic fractures show intense enhancement post-contrast.
Source: Chin, Teck Yew, et al. Get Through Final FRCR 2A: SBAs. 1st ed., CRC Press, 2017.
Explanation:
MR Imaging Features for Differentiating Metastatic and Acute Osteoporotic Compression Fractures
• Distinction between metastatic and acute osteoporotic compression fractures could be made on the
basis of MR imaging findings.
• A convex posterior border of the vertebral body is more frequent in metastatic compression fractures
than acute osteoporotic compression fractures.
• A higher frequency of abnormal signal intensity of the pedicle or posterior element has been
observed in metastatic compression fractures.
• Epidural soft-tissue mass is suggestive of malignant vertebral collapse.
• A paraspinal mass is not helpful in differentiation of the cause of vertebral collapse but is more
commonly encountered in the setting of metastatic compression, where it is typically focal rather
than diffuse.
• Signal intensity abnormalities in the marrow of vertebrae other than the collapsed vertebrae are more
frequently seen in metastatic compression fractures than acute osteoporotic compression fractures.
• Enhancement on post-contrast T1-weighted FS images is not useful in differentiation of acute
osteoporotic fractures from malignant compression fracture, but it may be useful for old or chronic
fractures, which will not show intense enhancement.
EBDR Exam MCQs & Concepts May 2022
Dr. Kareem Alnakeeb
277
• Moreover, on diffusion-weighted imaging (DWI) vertebral metastases with compression fractures can
be safely distinguished from vertebra with benign compression fractures based on significantly
different ADC values.
36. A 4-year-old child presents with short stature and failure to grow. Plain radiographs reveal multiple
abnormalities, including generalized increased density of long bones with thickened cortices, widened
cranial sutures, Wormian bones, a hypoplastic mandible and shortened pointed distal phalanges. Which
of the following is the most likely diagnosis?
A. Pyknodysostosis
B. Osteopetrosis
C. Cleidocranial dysostosis
D. Osteosclerosis
E. Kinky hair syndrome
Source: Chin, Teck Yew, et al. Get Through Final FRCR 2A: SBAs. 1st ed., CRC Press, 2017.
Explanation:
• Overview
o Pyknodysostosis is a congenital abnormality that should be considered in the differential
diagnosis of osteosclerosis.
• Clinical Features
o The patients are typically short, have hypoplastic mandibles, widened cranial sutures,
Wormian bones, brachycephaly, clavicular dysplasia, thick skull base and hypoplasia or
nonpneumatisation of the paranasal sinuses.
• Distinguishing Radiologic Features
o The distinguishing feature is acro-osteolysis with sclerosis.
o The distal phalanges appear as if they have been put in a pencil sharpener – they are pointed
and dense.
37. The causes of medullary nephrocalcinosis include all, except
A. Hyperparathyroidism
B. Renal tubular acidosis
C. Medullary sponge kidney
D. Hypervitaminosis D
E. Alport’s syndrome
Source: Chin, Teck Yew, et al. Get Through Final FRCR 2A: SBAs. 1st ed., CRC Press, 2017.
Explanation:
• Medullary Nephrocalcinosis
o Causes of medullary nephrocalcinosis include hyperparathyroidism, sarcoidosis,
myelomatosis, primary or secondary hyperoxaluria (Crohn’s disease), hyperthyroidism,
osteoporosis, idiopathic hypercalciuria, renal tubular acidosis, medullary sponge kidney
and drug-induced (hypervitaminosis D, milk-alkali syndrome).
• Alport’s Syndrome
o Alport’s syndrome is an autosomal dominant condition also called chronic hereditary
nephritis, associated with ocular abnormalities, deafness, small kidneys, cortical
calcification and progressive renal failure without hypertension.
EBDR Exam MCQs & Concepts May 2022
Dr. Kareem Alnakeeb
278
38. A 66-year-old joiner presents to his GP with jaundice and abdominal discomfort. He was subsequently
referred to a gastroenterologist who requests a liver biopsy due to deranged liver function tests. Which of
the following options is not a contraindication for percutaneous liver biopsy?
A. INR above 1.6
B. Platelets less than 60,000/mm3
C. Tense ascites
D. Extra-hepatic biliary obstruction
E. Suspected haemangioma
Source: Chin, Teck Yew, et al. Get Through Final FRCR 2A: SBAs. 1st ed., CRC Press, 2017.
Explanation:
• Contraindications for liver biopsy include the following:
1. Uncooperative patient
2. Extrahepatic biliary duct dilatation (except if benefit outweighs the risk)
3. Bacterial cholangitis (relative contraindication due to risk of septic shock)
4. Abnormal coagulation indices (having a normal INR or PT is not a reassurance that the patient
will not bleed; however, there is increased incidence of bleeding with INR above 1.5)
5. Presence of ascites
6. Cystic lesion
39. 57. A 40-year-old man who is a known hypothyroid patient, presents with weight loss and dull pain in the
flank and back. He undergoes an abdominal CT. Regarding retroperitoneal fibrosis, all of the following is
seen on imaging, except
A. Medial deviation of the ureters in the middle third, typically bilateral.
B. CT shows soft-tissue mass displacing the aorta anteriorly.
C. T2W MRI shows variable signal.
D. PET CT has high sensitivity.
E. Hydronephrosis is evident on CT urogram.
Source: Chin, Teck Yew, et al. Get Through Final FRCR 2A: SBAs. 1st ed., CRC Press, 2017.
Explanation:
Intravenous Urography
• Intravenous urography usually demonstrates the classic triad of medial deviation of the middle third
of the ureters, tapering of the lumen of one or both ureters in the lower lumbar spine or upper sacral
region, and proximal unilateral or bilateral hydroureteronephrosis with delayed excretion of contrast
material.
CT and MRI Diagnosis
• CT and MRI is the mainstay of non-invasive diagnosis of Retroperitoneal fibrosis (RPF).
• CT Imaging Features
• CT allows comprehensive evaluation of the morphology, location and extent of RPF and involvement
of adjacent organs and vascular structures.
• Moreover, abdominal CT allows detection of diseases often associated with idiopathic RPF (e.g.,
autoimmune pancreatitis) or demonstrating an underlying cause in cases of secondary RPF (e.g.,
malignancy).
• CT shows a well-defined mass, usually anterior and lateral to the aorta, sparing the posterior aspect
and not causing aortic displacement.
MRI Characteristics
• Idiopathic RPF typically has low signal intensity on T1-weighted images.
EBDR Exam MCQs & Concepts May 2022
Dr. Kareem Alnakeeb
279
• The signal intensity on T2-weighted images is variable and reflects the degree of associated active
inflammation (hypercellularity and oedema).
Contrast Enhancement and PET
• After administration of contrast material, early soft-tissue enhancement mirrors the degree of
inflammatory activity observed at T2-weighted imaging.
• The sensitivity of 18F-FDG PET is very high, which allows detection and quantification of the
metabolic activity of retroperitoneal lesions.
• Although sensitivity is high, specificity is low and aortic wall in the elderly can show FDG uptake.
40. A 9-year-old girl was taken to her family doctor with fever and painful knee and wrists. The GP noticed a
skin rash, hepatosplenomegaly and lymphadenopathy. Plain X-ray of the knee and wrist shows expansion
of bones around the knee and advanced carpometacarpal arthritis. What is the likely diagnosis?
A. Still disease
B. Hemophilia
C. Sickle-cell disease
D. Psoriasis
E. Lyme disease
Source: Chin, Teck Yew, et al. Get Through Final FRCR 2A: SBAs. 1st ed., CRC Press, 2017.
Explanation:
• Still Disease (Polyarticular Juvenile Rheumatoid Arthritis)
o Still disease is a clinical manifestation of polyarticular juvenile rheumatoid arthritis
characterized by fever, rash, hepatosplenomegaly and pericarditis.
o There is periosteal reaction of the hand phalanges and broadening of bones with cortical
thickening.
• Radiological and Clinical Compatibility
o The presence of advanced arthropathy in the hands at such a young age along with the other
clinical findings would be compatible with this condition.
• Lyme Disease Comparison
o Lyme disease tends to follow a monoarticular pattern with involvement of the large joints,
usually the knee, with the radiological findings not as profound as that of juvenile
rheumatoid arthritis.
41. A 40-year old with recurrent pulmonary emboli is due to have a hip replacement and it is decided to
deploy a temporary inferior vena cava (IVC) filter. What is the preferred site of deployment of an IVC filter?
A. Suprarenal IVC
B. Infrarenal IVC
C. Proximal to the clot load, no matter the level
D. At the confluence of common iliac vessels
E. At the junction of IVC and right atrium
Source: Chin, Teck Yew, et al. Get Through Final FRCR 2A: SBAs. 1st ed., CRC Press, 2017.
Explanation:
• Placement Considerations
o In the presence of normal anatomy, IVC filters should be placed inferior to the renal veins.
o If the patient has aberrant anatomy such as double IVC, a single suprarenal filter or twin IVC
filters can be placed.
• Rationale
o This is to avoid potential clot propagation and renal vein thrombosis.
EBDR Exam MCQs & Concepts May 2022
Dr. Kareem Alnakeeb
280
• Access and Imaging
o Access is via a right internal jugular puncture or a right femoral vein puncture.
o A cavogram is performed to visualize the renal veins and to look for aberrant anatomy as
described above.
• Indications
o Indications for IVC filter include deep vein thrombosis where anticoagulation is
contraindicated, where the patient is non-compliant with medical treatment or when there is
free-floating thrombus in the IVC.
• Device Management
o Retrievable devices are available and should be removed within 14 days from insertion.
• Complications
o Pulmonary embolism can still occur despite an IVC filter with an incidence of 2.7%–4%.
42. Chest radiograph of a 12-year-old boy shows a cystic lesion with air–fluid level in the right upper lobe. CT
scan confirms the presence of a thin-walled cystic lesion. Rest of the lungs are clear. There is no
lymphadenopathy in the chest. Quantiferon test was negative, and there are no features of infection or
signs of inflammation. What is the diagnosis?
A. TB
B. Intrapulmonary bronchogenic cyst
C. Hydatid cyst
D. Infected bulla
E. Congenital lobar emphysema
Source: Chin, Teck Yew, et al. Get Through Final FRCR 2A: SBAs. 1st ed., CRC Press, 2017.
Explanation:
Location and Prevalence
• Bronchogenic cysts (BCs) are congenital lesions.
• They are usually found in the mediastinum or pulmonary parenchyma and, less commonly, cysts
may be found in the neck, pericardium, pleura, diaphragm or abdominal cavity.
• Intrapulmonary cysts are most common in the lower lobes.
Imaging Characteristics
• Intrapulmonary BCs are usually sharply defined, solitary, non-calcified, round or oval opacities
confined to a single lobe.
• These can present as a homogeneous water density, an air-filled cyst, or with an air–fluid level.
• Signal on MRI depends on the content, and fluid-containing lesions are low on T1-weighted and high
on T2-weighted images; however, proteinaceous content makes them high on T1-weighted imaging.
Differential Diagnosis
• The differential diagnosis of intraparenchymal BCs must include acquired cystic lesions, such as a
lung abscess, a hydatid cyst, infection with Nocardia, an infected bulla, congenital lobar
emphysema, fungal diseases and tuberculosis, especially when the lesions manifest as air-filled or
have an air–fluid level.
EBDR Exam MCQs & Concepts May 2022
Dr. Kareem Alnakeeb
281
43. A 35-year-old non-smoking woman presents with progressive exertional dyspnea, and high-resolution CT
of the chest shows innumerable thin-walled cysts of varying size diffusely involving both lungs with
normal intervening parenchyma; which single diagnosis best explains these findings?
A. Pulmonary Langerhans cell histiocytosis
B. Lymphangioleiomyomatosis
C. Centrilobular emphysema
D. Idiopathic pulmonary fibrosis
E. Pulmonary sarcoidosis
Explanation: (by Perplexity AI, generated with the OpenAI O3 model)
• Lymphangioleiomyomatosis (LAM) classically affects women of child-bearing age and produces
uniform, thin-walled cysts scattered throughout all lung zones while preserving the background lung
architecture, matching the vignette.
• Langerhans cell histiocytosis usually affects young smokers and shows irregular cysts and nodules
predominating in the upper and mid-zones, not a uniform diffuse pattern.
• Emphysema creates areas of low attenuation without a visible wall and is centrilobular or paraseptal
rather than truly cystic.
• Idiopathic pulmonary fibrosis demonstrates basal-predominant reticulation, honeycombing and
traction bronchiectasis, not isolated cysts.
• Sarcoidosis typically has perilymphatic nodules and fibrosis; diffuse thin-walled cysts are
uncommon, making LAM the most appropriate choice.
44. A 29-year-old heavy smoker presents with persistent dry cough. High-resolution CT demonstrates
numerous irregular lung cysts mixed with small centrilobular nodules, predominantly in the upper and
mid-zones with relative sparing of the costophrenic angles. Which single diagnosis best explains this
pattern?
A. Lymphangioleiomyomatosis
B. Pulmonary Langerhans cell histiocytosis
C. Centrilobular emphysema
D. Metastatic adenocarcinoma
E. Idiopathic pulmonary fibrosis
Explanation: (by Perplexity AI, generated with the OpenAI O3 model)
• Pulmonary Langerhans cell histiocytosis (PLCH) characteristically affects young adult smokers
and shows a combination of centrilobular nodules and bizarre-shaped cysts that favor the
upper/mid-lungs while sparing the bases, exactly as in this vignette.
• Lymphangioleiomyomatosis produces uniformly thin-walled round cysts distributed diffusely
without nodules and almost exclusively affects women, so option A is unlikely.
• Centrilobular emphysema causes areas of low attenuation without definable walls and lacks
nodules, ruling out option C.
• Cystic metastases are rare, usually lower-lobe and accompanied by dominant masses; upper-zone
nodular-cystic disease in a smoker favors PLCH over option D.
• Idiopathic pulmonary fibrosis presents with basal-predominant reticulation, honeycombing and
traction bronchiectasis rather than nodules with cysts, excluding option E.
EBDR Exam MCQs & Concepts May 2022
Dr. Kareem Alnakeeb
282
45. Which of the following is NOT recognized as a cause of a “blue ear drum” seen on otoscopy?
A. Cholesterol granuloma
B. Glioma
C. Otitis media with effusion
D. Idiopathic haemotympanum
E. Temporal bone fracture
Explanation: (by Perplexity AI, generated with the OpenAI O3 model)
• A blue-coloured tympanic membrane typically reflects blood or blood-stained fluid in the middle ear.
• Cholesterol granuloma often presents with a blue or “chocolate” eardrum because of altered blood
products behind an intact membrane. Otitis media with effusion (glue ear) can impart a bluish hue
when the effusion is thick or haemorrhagic.
• Idiopathic haemotympanum is defined by painless bluish discoloration due to recurrent bleeding in a
poorly ventilated middle ear.
• Acute haemotympanum after temporal bone fracture likewise stains the drum blue-black.
• In contrast, glioma (a congenital glial rest extending from the intracranial compartment) appears as a
reddish-pink mass in the external auditory canal or middle ear and does not characteristically
produce the uniform blue tympanic membrane seen in haemotympanum.
46. On brain diffusion-weighted MRI, which of the following entities is typically NOT associated with low
apparent diffusion coefficient (ADC) values?
A. Acute cerebral infarction
B. Primary central nervous system lymphoma
C. Intracavitary pus within a cerebral abscess
D. Necrotic tumor core
E. Epidermoid cyst
Explanation: (by Perplexity AI, generated with the OpenAI O3 model)
• Restricted diffusion—manifesting as high signal on DWI and low ADC—occurs when water motion is
impeded by high cellularity or high viscosity.
• Acute infarction causes cytotoxic oedema that traps water within swollen cells, producing low ADC.
• Lymphoma is densely cellular, again restricting diffusion.
• Pus is viscous and cellular, giving low ADC within abscess cavities.
• Epidermoid cysts contain keratin and cholesterol crystals that markedly restrict diffusion.
• By contrast, tumoral necrosis involves cellular breakdown and increased extracellular water,
resulting in facilitated diffusion and therefore relatively high ADC, so it is not associated with low
ADC.
47. On diffusion-weighted MRI of the brain, what is the typical signal pattern seen in the centre of a mature
pyogenic abscess?
A. High signal on DWI with low ADC values
B. Low signal on DWI with high ADC values
C. Isointense on both DWI and ADC maps
D. High signal on DWI with high ADC values
E. Low signal on DWI with low ADC values
Explanation: (by Perplexity AI, generated with the OpenAI O3 model)
• Pus within a pyogenic abscess is highly cellular and viscous, restricting Brownian water motion. This
produces true restricted diffusion: bright signal on DWI and correspondingly low ADC values.
EBDR Exam MCQs & Concepts May 2022
Dr. Kareem Alnakeeb
283
• Options B and C show facilitated or neutral diffusion typical of necrotic tumors or cysts.
• Option D represents “T2 shine-through”, where high DWI signal is accompanied by high ADC and
therefore not true restriction.
• Option E (low signal on both DWI and ADC) is not a recognized pattern for abscesses and would more
likely reflect susceptibility artefact or hemorrhage rather than pus.
48. The commonest primary cardiac tumor in adults is:
A. Myxoma
B. Haemangioma
C. Rhabdomyoma
D. Fibroma
Explanation: (by Perplexity AI, generated with the OpenAI O3 model)
• > 50% of adult primary cardiac neoplasms are left-atrial myxomas.
49. The commonest primary cardiac tumor in children is:
A. Myxoma
B. Haemangioma
C. Rhabdomyoma
D. Fibroma
Explanation: (by Perplexity AI, generated with the OpenAI O3 model)
• Rhabdomyomas constitute more than 60% of all primary cardiac tumors diagnosed in infants and
children, making them by far the most common benign cardiac neoplasm in this age group.
50. Which of the following is not associated with cardiac mass?
A. Carney triad
B. Carney complex
C. Tuberous sclerosis
D. Neurofibromatosis (NF1)
Explanation: (by Perplexity AI, generated with the OpenAI O3 model)
Carney triad lacks cardiac tumors
• Carney triad classically links three extracardiac tumors—gastrointestinal stromal tumor, pulmonary
chondroma, and extra-adrenal paraganglioma—and does not include any cardiac mass in its
diagnostic constellation.
• Therefore, among the options listed, Carney triad is the only condition not associated with cardiac
tumors.
51. Endoscopic third ventriculostomy creates a new communication Between the third ventricle and:
A. fourth ventricle
B. ambient cistern
C. interpeduncular cistern
Explanation: (by Perplexity AI, generated with the OpenAI O3 model)
• Endoscopic third ventriculostomy (ETV) creates a surgical opening in the floor of the third
ventricle—just in front of the mammillary bodies—so cerebrospinal fluid can bypass an obstruction
and flow directly into the interpeduncular (prepontine) cistern in the basal cisterns.
o Therefore, the new communication produced by a third ventriculostomy is:
o Between the third ventricle and the interpeduncular cistern.
• (The other options—fourth ventricle and ambient cistern—are not the structures opened into during
an ETV.)
EBDR Exam MCQs & Concepts May 2022
Dr. Kareem Alnakeeb
284
52. Which of the following is not an angiographic sign of active bleeding?
F. Contrast extravasation
G. Vessel spasm
H. Vessel cut-off
I. Early venous filling
J. Vessel dilatation
Source: Chin, Teck Yew, et al. Get Through Final FRCR 2A: SBAs. 1st ed., CRC Press, 2017.
Explanation:
• The rest are angiographic signs of active bleeding; vessel dilatation is not.
53. On sagittal MRI of an acutely collapsed lumbar vertebral body, which imaging feature is most
characteristic of a malignant compression fracture rather than an osteoporotic fracture?
A. Intravertebral fluid (“cleft”) sign
B. Band-like preservation of normal marrow in the posterior third of the vertebral body
C. Low signal intensity extending into the ipsilateral pedicle
D. Wedge deformity with intact posterior wall and no soft-tissue component
E. Uniform low T1 signal confined to the anterior two-thirds of the body
Explanation: (by Perplexity AI, generated with the OpenAI O3 model)
• Malignant infiltration frequently spreads from the vertebral body into the pedicle, producing
continuous low T1 and T2 signal in the posterior elements; this spread is uncommon in purely
osteoporotic fractures, which usually spare the pedicles and posterior wall.
• Benign osteoporotic fractures often show the intravertebral fluid cleft (A) and a horizontal band of
preserved marrow (B).
• A simple wedge shape with an intact posterior cortex and no paraspinal mass (D) further supports a
benign cause.
• Uniform low T1 signal limited to the body (E) lacks specificity, as both malignant and acute
osteoporotic fractures can appear diffusely hypointense shortly after collapse.
54. Which of the following is not a feature of holoprosencephaly?
A. Single ventricle
B. Fused thalami
C. Absent corpus callosum
D. Tectal beaking
E. Hypoplasia of the optic nerves
Source: Chin, Teck Yew, et al. Get Through Final FRCR 2A: SBAs. 1st ed., CRC Press, 2017.
Explanation:
Holoprosencephaly
• Holoprosencephaly results from a lack of normal cleavage of the forebrain.
• Holoprosencephaly may be divided into alobar, semilobar and lobar forms depending on the degree
of abnormality.
• Single ventricle, fused thalami, absent corpus callosum and hypoplasia of the optic nerves are all
features of the various forms of holoprosencephaly.
Septum Pellucidum
• The septum pellucidum is always absent in this condition.
• In its mildest form, lobar holoprosencephaly, absence of the septum pellucidum may be the only
abnormality.
Associated Disorders and Severity
• The lobar form may be associated with septo-optic dysplasia and the two conditions overlap.
EBDR Exam MCQs & Concepts May 2022
Dr. Kareem Alnakeeb
285
• The degree of facial abnormality and mental retardation mirrors the severity of the intracranial
abnormality.
Chiari II Sign
• Tectal beaking is a feature of Chiari II.
55. Which of the following are true of Holoprosencephaly? (True or False)
A. It results from absence of the supraclinoid internal carotid artery (ICA) system
B. Trisomy 13 is the most common associated chromosomal abnormality
C. The septum pellucidum can be seen in the lobar subtype
D. The falx cerebri may be seen anteriorly in the semilobar type
E. The cerebellum is structurally normal in the alobar type
Source: Bell, J., and N. Davies. MCQs in Clinical Radiology: A Revision Guide for the FRCR. 1st ed.,
Remedica Pub Ltd, 2004.
Explanation:
A. False This abnormality results in Hydranencephaly, in which there is no anterior cerebral mantle or
facial anomaly and the falx cerebri and thalami are normal.
B. True
C. False
D. False
E. True
• b-e)
Overview
• Holoprosencephaly results from partial/complete lack of cleavage of the developing forebrain and is
divided into three types: alobar, semilobar and lobar.
Alobar type
• The alobar type is most severe and is characterised by fused cerebral hemispheres with an anterior
cup-shaped brain, a single monoventricle, fused thalami and absent corpus callosum, fornix, optic
tracts and olfactory bulbs (the midbrain, brain stem and cerebellum are normal).
Semilobar type
• The semilobar type is of intermediate severity and is characterised by partial cleavage into
hemispheres, monoventricles with rudimentary occipital and temporal horns.
• The falx cerebri may be present posteriorly and the thalami are variably fused.
Lobar type
• The lobar type is the least severe and is characterized by lateral ventricles that are almost normal,
but frontal horns point inferiorly and may be 'squared'.
Shared characteristics
• The septum pellucidum is absent in all three subtypes.
Incidence and associations
• The incidence is equal in males and females and is associated with: chromosome abnormalities
(trisomy 13 and 18 are most common), polyhydramnios (60%), and renal and cardiac anomalies.
Facial anomalies
• Facial anomalies are also common, ranging from cyclopia to hypotelorism, and they usually
correlate with the severity of the brain abnormality.
Related condition
EBDR Exam MCQs & Concepts May 2022
Dr. Kareem Alnakeeb
286
• Septo-optic dysplasia (absence of the septum pellucidum and hypoplastic anterior optic pathways)
may be considered a mild form of lobar holoprosencephaly.
56. A 27-year-old woman presents to the Accident & Emergency Department with headaches. A CT scan of
the head shows widely spaced lateral ventricles, dilatation of the trigones and occipital horns of lateral
ventricles with an upward displacement of the dilated 3rd ventricle. The underlying abnormality in the
brain is?
A. Midline arachnoid cyst
B. Agenesis of the corpus callosum
C. Prominent cavum septum pellucidum
D. Hydrocephalus
E. Lobar holoprosencephaly
Source: Gupta, Chaitanya. 300 Single Best Answers for the Final FRCR Part A. 1st ed., Jaypee UK,
2010.
Explanation:
• This is associated with parallel, widely spaced lateral ventricles that may appear crescent shaped.
• There is dilatation of trigones and the occipital horn of lateral ventricles, along with a high riding 3rd
ventricle.
• Callosal agenesis is associated with Dandy–Walker syndrome, Chiari malformations and fetal
alcohol syndrome.
57. A 66-year-old joiner presents to his GP with jaundice and abdominal discomfort. He was subsequently
referred to a gastroenterologist who requests a liver biopsy due to deranged liver function tests. Which of
the following options is not a contraindication for percutaneous liver biopsy?
F. INR above 1.6
G. Platelets less than 60,000/mm3
H. Tense ascites
I. Extra-hepatic biliary obstruction
J. Suspected hemangioma
Source: Chin, Teck Yew, et al. Get Through Final FRCR 2A: SBAs. 1st ed., CRC Press, 2017.
Explanation:
Contraindications for liver biopsy include the following:
• Uncooperative patient
• Extrahepatic biliary duct dilatation (except if benefit outweighs the risk)
• Bacterial cholangitis (relative contraindication due to risk of septic shock)
• Abnormal coagulation indices (having a normal INR or PT is not a reassurance that the patient will not
bleed; however, there is increased incidence of bleeding with INR above 1.5)
• Thrombocytopenia (platelet count below 60,000/mm3)
EBDR Exam MCQs & Concepts May 2022
Dr. Kareem Alnakeeb
287
58. A patient with known tuberous sclerosis had a routine follow-up CT. A 3 x 2-cm partly calcified
heterogeneously enhancing lesion was seen at the level of the foramen of Monro. What is the most likely
pathology?
A. Colloid cyst
B. Subependymal giant cell astrocytoma
C. Intraventricular
D. Meningioma
E. Germinoma
Source: Proctor, Robin. Final FRCR Part A Modules 4-6 Single Best Answer MCQs: The SRT Collection
of 600 Questions with Explanatory Answers (MasterPass). 1st ed., CRC Press, 2009.
Explanation:
• 15% of patients with tuberous sclerosis develop subependymal astrocytomas. They typically occur
at the foramen of Monro and are usually a well-defined rounded mass with some calcification. They
usually enhance uniformly with contrast and can degrade to a high-grade astrocytoma.
• 95% of tuberous sclerosis patients have subependymal hamartomas. These occur in the
periventricular region, are isointense to white matter on Ti and calcified on CT.
• 55% of patients have cortical tubers, which are high signal on T2-weighted imaging.
59. A five year old with seizures and cognitive impairment had an MRI scan. This revealed features highly
suggestive of heterotopia. What are the likely findings on the MRI?
A. CSF lined cleft extending from the ependymal surface to cortical pia
B. Shallow Sylvian fissures and agyric cortex
C. Bilateral nodular subependymal grey matter
D. Squared appearance of the frontal horns and an absent septum pellucidum
E. Poor brain sulcation with intraparenchymal calcification
Source: Proctor, Robin. Final FRCR Part A Modules 4-6 Single Best Answer MCQs: The SRT Collection
of 600 Questions with Explanatory Answers (MasterPass). 1st ed., CRC Press, 2009.
Explanation:
• Heterotopic grey matter occurs secondary to developmental arrest of migrating neuroblasts from the
ventricular walls to the surface of the brain. Nodular and laminar forms are described. Signal is
isointense to grey matter on all sequences.
60. A 62-year-old man with known hepatocellular carcinoma on a background of long-standing liver cirrhosis
is scheduled to have a TACE procedure. Which one of the following is an absolute contraindication to
TACE therapy for hepatocellular carcinoma in a cirrhotic patient?
A. Contrast medium allergy
B. Replacement of 25% of the liver by the tumour
C. Total bilirubin greater than 2 mg/dL
D. Biliary tree obstruction
E. Child–Pugh Class C cirrhosis
Source: Chin, Teck Yew, et al. Get Through Final FRCR 2A: SBAs. 1st ed., CRC Press, 2017.
EBDR Exam MCQs & Concepts May 2022
Dr. Kareem Alnakeeb
288
Explanation:
Absolute and relative contraindications for conventional TACE in patients with HCC are as follows.
Absolute contraindications: Relative contraindications:
1. Decompensated cirrhosis (Childs–Pugh C or
higher)
1. Tumour size >10 cm.
2. Jaundice 2. Co-morbidities involving compromised organ
function such as cardiovascular and lung disease.
3. Clinical encephalopathy 3. Untreated varices present a high risk of bleeding.
4. Refractory ascites 4. Bile duct occlusion or incompetent papilla due to
stent or surgery.
5. Extensive tumour with massive replacement of
both lobes
6. Severely reduced portal vein flow
7. Technical contraindications to hepatic intra-arterial
treatment
8. Renal insufficiency (creatinine clearance <30
mL/min)
61. A 52-year-old woman is involved in a RTA. Trauma series radiographs reveal a complex pelvic fracture.
There are no other appreciable injuries. She is haemodynamically unstable, and fluid resuscitation is
commenced. Which of the following steps is the next most important?
A. Trauma protocol CT scan with angiographic sequences
B. Immediate surgery under the orthopaedic surgeons
C. Discussion with interventional radiologists with view to catheter angiography and possible
intervention
D. Placement of a pelvic wrap device
E. Blood transfusion
Source: Chin, Teck Yew, et al. Get Through Final FRCR 2A: SBAs. 1st ed., CRC Press, 2017.
Explanation:
Application and Purpose of Pelvic Wrap
• All of these steps may be required, but the most important is to place a pelvic wrap device.
• The purpose of this device is to stabilize the pelvis.
• If the patient is hypotensive as a result of venous bleeding, a pelvic wrap should stabilize the pelvis
sufficiently to cause significant reduction or cessation in the venous hemorrhage and thus avoid
unnecessary endovascular intervention.
Indications for Endovascular Intervention
• If this fails to achieve sufficiently prompt hemodynamic stability, there is a significant chance that
there is arterial hemorrhage; hence endovascular intervention is likely to be necessary.
Imaging for Bleeding Source
• Many centers use angiographic sequences as part of the trauma CT scan to identify such a bleeding
point as part of planning stage of endovascular management.
• CT would be the next step once the pelvic binder is in situ.
EBDR Exam MCQs & Concepts May 2022
Dr. Kareem Alnakeeb
289
62. CT brain of a 10 year old girl shows a large cyst in the posterior fossa. All of the following favour pilocystic
astrocytoma over haemangioblastoma, except
A. Size greater than 5 cm
B. Calcifications
C. Smaller nodule
D. Thicker-walled lesion
E. No angiographic contrast blush of the mural nodule
Source: Chin, Teck Yew, et al. Get Through Final FRCR 2A: SBAs. 1st ed., CRC Press, 2017.
Explanation:
Pilocytic Astrocytoma
• Pilocytic astrocytoma is the most common pediatric cerebellar neoplasm and the most common
pediatric glioma.
Differentiation from Hemangioblastoma
• They can be differentiated from hemangioblastoma on the following basis:
• Astrocytomas are more likely to be larger than 5 cm, contain calcification, have a larger mural
nodule, are thick-walled lesions, do not show angiographic contrast blush to the mural nodule and
are not associated with erythrocythemia.
EBDR Exam MCQs & Concepts October 2021
Dr. Kareem Alnakeeb
290
October 2021
Paper 1
1. Which imaging feature is classically associated with an oligodendroglioma on CT or MRI of the brain?
A. Cortical and subcortical location
B. Lack of contrast enhancement
C. Presence of calcification
D. Intraventricular origin
E. Hyperperfusion on arterial spin-labelling
Explanation: (by Perplexity AI, generated with the OpenAI O3 model)
• Oligodendrogliomas most often arise in the cerebral hemispheres at the cortex–subcortical junction
and frequently show coarse or gyriform calcifications on CT; this striking tendency to calcify (seen in
up to 90% of cases) makes calcification the hallmark imaging clue.
• Although some low-grade lesions may enhance little or not at all, contrast enhancement is variable
and therefore not reliably diagnostic.
• They are parenchymal tumors, not intraventricular masses (intraventricular tumors with similar
calcification are more likely central neurocytomas).
• Hyperperfusion is neither typical nor specific.
2. Which imaging combination is most typical of an intracranial epidermoid cyst?
A. Extra-axial lesion showing T2 shine-through on DWI and low attenuation on CT
B. Extra-axial lesion with true restricted diffusion, low attenuation on CT and incomplete
suppression on FLAIR
C. Intra-axial lesion with T2 shine-through on DWI and low attenuation on CT
D. Extra-axial lesion with T2 shine-through on DWI and complete suppression on FLAIR
E. Intra-axial lesion with restricted diffusion and hyperdensity on CT
Explanation: (by Perplexity AI, generated with the OpenAI O3 model)
• Epidermoid cysts are usually extra-axial, appear hypodense (CSF-like) on CT and fail to fully
suppress on FLAIR, giving a “dirty CSF” appearance.
• On diffusion-weighted imaging they are markedly bright because of true restricted diffusion rather
than mere T2 shine-through; the high cellular keratin content limits water motion and differentiates
them from arachnoid cysts.
• Options with intra-axial location, full FLAIR suppression or only shine-through lack this classic triad
and are therefore incorrect.
3. Regarding juvenile nasopharyngeal angiofibroma (shown in the CT image of a vascular nasopharyngeal
mass), which statement is TRUE?
A. It is a malignant tumor
B. It is avascular
C. It originates at the foramen rotundum
D. It may erode adjacent bone
E. It affects females more often than males
Explanation: (by Perplexity AI, generated with the OpenAI O3 model)
• Juvenile nasopharyngeal angiofibroma is a benign yet highly vascular tumor that arises near the
sphenopalatine foramen in adolescent males. Although histologically benign, its aggressive growth
EBDR Exam MCQs & Concepts October 2021
Dr. Kareem Alnakeeb
291
often produces pressure-related bone erosion and remodeling of the pterygoid plates, sphenoid
bone and adjacent sinus walls, a key imaging clue.
• The lesion is not malignant (so option A is wrong) and is anything but avascular (option B).
• Its site of origin is the posterior choanal/sphenopalatine foramen region rather than the foramen
rotundum (option C).
• It shows an overwhelming male predilection, making option E incorrect.
4. On MRI for a suspected middle-ear cholesteatoma, which imaging combination is most characteristic?
A. Central contrast enhancement with restricted diffusion
B. Central contrast enhancement without restricted diffusion
C. No central enhancement and no restricted diffusion
D. No central enhancement but restricted diffusion
E. Rim enhancement with facilitated diffusion
Explanation: (by Perplexity AI, generated with the OpenAI O3 model)
• Acquired and residual cholesteatomas typically appear non-enhancing on post-contrast T1-
weighted images because the keratin matrix is avascular, yet they show marked hyperintensity
on non-echo-planar diffusion-weighted imaging with low ADC values, reflecting true restricted
diffusion of keratin debris.
• Enhancement within the lesion suggests vascular granulation tissue or abscess (options A, B, E).
• Absence of both enhancement and diffusion restriction (option C) favors simple fluid or mucosal
thickening rather than cholesteatoma.
5. Which of the following features favor Rathke’s cleft cyst rather than craniopharyngioma?
A. Absence of calcification
B. Cystic element on MR
C. Involvement of suprasellar and sellar regions
D. Enhancement of the wall
E. High signal intensity on T1
Source: Leen, Edward. Succeeding in the New FRCR Part 2A Exam: Single Best Answer (SBA)
Revision Questions for Modules 1-6. 1st ed., UNKNO, 2011.
Explanation:
• Rathke’s cleft cysts do not calcify. They affect women to men in a 2:1 ratio and adults from 40-60
years of age. They cause variable MR appearances depending on protein content of cyst. They can
rarely show enhancement.
6. Which is the most common imaging finding in neurosarcoidosis?
A. Leptomeningeal contrast enhancement
B. Hyperintense white matter T2 lesion
C. Grey matter lesions enhancing on MR
D. Involvement of the hypothalamus
E. Focal epidural masses
Source: Leen, Edward. Succeeding in the New FRCR Part 2A Exam: Single Best Answer (SBA)
Revision Questions for Modules 1-6. 1st ed., UNKNO, 2011.
Explanation:
• Basilar meninges often involved.
• B-E are recognized imaging findings.
• Spinal disease is less common than brain disease and findings include intramedullary lesions and
intrathecal nodular masses.
EBDR Exam MCQs & Concepts October 2021
Dr. Kareem Alnakeeb
292
7. Which WHO grade 1 meningioma subtype most often appears isointense to cortical grey matter on T2-
weighted MRI?
A. Angiomatous meningioma
B. Fibroblastic meningioma
C. Psammomatous meningioma
D. None of the above
E. Microcystic meningioma
Explanation: (by Perplexity AI, generated with the OpenAI O3 model)
• T2 signal in meningiomas depends chiefly on their collagen and water content. Fibroblastic (fibrous)
meningiomas are rich in dense collagen, giving them low proton mobility and therefore a low-
to-intermediate (commonly isointense) T2 signal relative to cortex.
• Angiomatous and microcystic variants contain abundant vascular channels or microcysts with high
fluid content, so they are typically T2 hyperintense rather than isointense.
• Psammomatous tumours often show coarse calcification; on MRI they are frequently T2 hypointense
or mixed, not predominantly isointense.
• Thus an extra-axial mass that is T2 isointense should raise suspicion for the fibrous subtype.
8. In a patient with a cystic intracranial mass, which MRI sequence most reliably differentiates a pyogenic
cerebral abscess from a pilocytic astrocytoma?
A. FLAIR
B. Diffusion-weighted imaging (DWI)
C. Gradient-echo (T2*)
D. Post-contrast T1-weighted
E. T2 spin-echo
Explanation: (by Perplexity AI, generated with the OpenAI O3 model)
• Pus within a cerebral abscess is highly cellular and viscous, causing marked restriction of water
motion; this appears as very high signal on DWI with corresponding low ADC values, making DWI the
most specific sequence for identifying an abscess.
• Pilocytic astrocytomas, although often cystic, generally show facilitated or only mildly restricted
diffusion, so they are easily distinguished on this sequence.
• FLAIR highlights oedema but cannot separate these entities.
• Gradient-echo is sensitive to hemorrhage or calcification rather than pus.
• Ring enhancement on post-contrast T1 may be seen in both lesions, and conventional T2 signal
characteristics overlap, so neither provides definitive discrimination.
9. On digital subtraction angiography, which single angio-architectural feature best distinguishes a true
arteriovenous malformation (AVM) from an arteriovenous fistula (AVF)?
A. Presence of an angiodense nidus
B. Identifiable feeding artery
C. Overall size of the lesion
D. Pattern of venous drainage
E. Patient age at presentation
Explanation: (by Perplexity AI, generated with the OpenAI O3 model)
• An AVM is defined by a compact tangle of abnormal vessels—the nidus—interposed between
feeding arteries and draining veins. Its angiographic appearance is therefore an “angiodense” nidus,
a key hallmark that is absent in an AVF12.
EBDR Exam MCQs & Concepts October 2021
Dr. Kareem Alnakeeb
293
• AVFs show a single, direct artery-to-vein connection without an intervening nidus; although feeders
and drainers are visible, their mere presence (options B and D) does not differentiate the two
entities3.
• Lesion size (option C) varies widely in both AVMs and AVFs and is not a defining criterion.
• Age at presentation (option E) can overlap and offers no reliable discrimination.
10. Bilateral globus pallidus injury manifest radiologically as high signal on T2W MR sequences is indicative
of poisoning by which of the following substances?
A. lead
B. methanol
C. carbon monoxide
D. carbon dioxide
E. mercury
Source: Bydder, Megan, et al. Get Through Final FRCR Part A: SBAs for the Modular Examination. 1st
ed., Hodder Education Publishers, 2009.
Explanation:
Basal Ganglia Involvement in Carbon Monoxide Poisoning
• Carbon monoxide poisoning results in irreversible formation of carboxyhemoglobin in the blood,
causing anoxic ischemic encephalopathy.
• These changes are usually bilateral and affect the basal ganglia, most commonly the globus pallidus.
• Areas less commonly affected acutely are the putamen (which is characteristically involved in
methanol poisoning) and caudate nucleus.
• Involvement elsewhere can occur but is less common than basal ganglia changes.
MRI Signal Characteristics
• Injury is demonstrated as high signal on T2W and FLAIR images, and shows restricted diffusion on
DWI.
Delayed Post-Anoxic Changes
• Delayed post-anoxic encephalopathy may develop several weeks after carbon monoxide poisoning,
and MRI then shows further high T2 signal changes in the corpus callosum, subcortical U fibres, and
internal and external capsules, with low T2 signal changes in the thalamus and putamen.
11. Regarding carotid–cavernous fistulas (CCFs), which of the following statements is TRUE?
A. They are most commonly produced by rupture of a cavernous carotid aneurysm.
B. Spontaneous CCFs are usually associated with fibromuscular dysplasia.
C. A direct (Barrow type A) fistula is typically treated by a trans-arterial endovascular approach.
D. Fistulas supplied by multiple arterial branches are preferably treated through an arterial route.
E. Indirect (dural) fistulas never close spontaneously.
Explanation: (by Perplexity AI, generated with the OpenAI O3 model)
• Direct CCFs (high-flow Barrow type A), produced by a tear in the cavernous segment of the internal
carotid artery—usually traumatic—are most effectively treated by a trans-arterial route using
detachable balloons, coils, or flow-diverters, aiming to occlude the fistulous site while preserving
carotid patency.
• Most CCFs arise after head trauma; formation from rupture of a cavernous carotid aneurysm is
distinctly uncommon, accounting for <5% of cases, so option A is incorrect.
• Fibromuscular dysplasia is a rare predisposing factor for spontaneous dural CCFs, not a usual
association, making option B wrong.
EBDR Exam MCQs & Concepts October 2021
Dr. Kareem Alnakeeb
294
• When a fistula is fed by multiple small dural branches (indirect types B–D), trans-venous
embolization via the inferior petrosal/superior ophthalmic vein is preferred; hence option D is
incorrect.
• Indirect CCFs can undergo spontaneous thrombosis, so option E is false.
12. Which is the most common site of metastatic spread in medulloblastoma?
A. Axial skeleton
B. Lymph nodes
C. Lung
D. Subarachnoid space
E. Liver
Source: Leen, Edward. Succeeding in the New FRCR Part 2A Exam: Single Best Answer (SBA)
Revision Questions for Modules 1-6. 1st ed., UNKNO, 2011.
Explanation:
• Subarachnoid space is the most common, with drop metastases occurring in 40%.
13. A 30-year-old man presents with a lump in the left cheek. Ultrasound examination shows an 8 mm
hypoechoic and lobulated lesion with a hyperechoic center. The most likely cause of the lesion is?
A. Parotid duct stone
B. Lymph node
C. Warthin’s tumor
D. Pleomorphic adenoma
E. Abscess
Source: Gupta, Chaitanya. 300 Single Best Answers for the Final FRCR Part A. 1st ed., Jaypee UK,
2010.
Explanation:
• Typical appearances of intraglandular lymph nodes are of a hypoechoic periphery with a fatty
hyperechoic center.
14. In adult head-injury patients with suspected diffuse axonal injury, what is the most common initial
appearance on non-contrast CT?
A. Small hemorrhagic foci
B. Normal study
C. Small hemorrhagic foci with marked cerebral oedema
D. Conspicuous lesions involving brainstem and corpus callosum
E. Extensive subarachnoid hemorrhage
Explanation: (by Perplexity AI, generated with the OpenAI O3 model)
• The shearing forces of diffuse axonal injury (DAI) usually produce microscopic axonal disruption
without macroscopic bleeding. Consequently, up to 50–80% of patients show no visible abnormality
on their first CT scan, making a “normal” study the commonest presentation.
• Hemorrhagic petechiae (option A) are classic but occur in a minority; when present with diffuse
oedema (option C) they indicate more severe injury.
• Early CT rarely shows the characteristic corpus callosum or brainstem lesions (option D); these are
better detected on MRI.
• Extensive subarachnoid hemorrhage (option E) is not a typical feature of isolated DAI.
EBDR Exam MCQs & Concepts October 2021
Dr. Kareem Alnakeeb
295
15. Optochiasmatic arachnoiditis most commonly develops as a complication of which intracranial
infection?
A. Toxoplasmosis
B. Neurocysticercosis
C. Optic nerve meningioma
D. Tuberculous meningitis
E. Herpes simplex encephalitis
Explanation: (by Perplexity AI, generated with the OpenAI O3 model)
• Thick basal exudates from tuberculous meningitis tend to accumulate in the suprasellar and
interpeduncular cisterns, encasing the optic nerves and chiasm and provoking optochiasmatic
arachnoiditis with rapid, often bilateral visual loss.
• Other infections such as neurocysticercosis can cause basal arachnoiditis, but chiasmal
involvement is far less typical.
• Optic nerve meningioma produces a fusiform dural-based mass rather than diffuse leptomeningeal
inflammation, while toxoplasmosis and herpes encephalitis characteristically affect parenchyma,
not the leptomeninges, so they do not produce this arachnoiditic pattern.
16. On MRI, how do demyelinating brainstem plaques of multiple sclerosis typically appear?
A. Central and symmetrical
B. Peripheral (abutting CSF spaces)
C. Spare the cerebellum completely
D. Both B and C
E. Diffuse and confluent throughout the pons
Explanation: (by Perplexity AI, generated with the OpenAI O3 model)
• Multiple sclerosis characteristically produces small, well-defined T2/FLAIR hyperintense plaques
that lie at the outer or inner borders of the brainstem, contiguous with the subarachnoid space or
ventricular surface.
• This peripheral predilection reflects perivenular inflammation along penetrating vessels and helps
distinguish MS from vascular small-vessel disease, whose pontine foci are usually central and
symmetric.
• Cerebellar hemispheres are frequently involved in MS, so they are not spared (making options C and
D incorrect).
• Large confluent or diffuse pontine lesions (option E) are uncharacteristic for typical MS plaques.
17. Which statement about cystic lymphangioma is TRUE?
A. It is most often encountered in the pediatric age group
B. It is confined to the neck and never arises in the retroperitoneum
C. It typically shows rim (“ring”) enhancement on contrast-enhanced CT
D. Magnetic resonance imaging is mandatory to make the diagnosis
E. It commonly calcifies on plain radiography
Explanation: (by Perplexity AI, generated with the OpenAI O3 model)
• Cystic lymphangiomas are congenital malformations of lymphatic channels; 80–90% present before
age 2, making childhood their commonest age group of occurrence.
• Although the neck is the classic site, up to 5% arise in the abdomen or retroperitoneum, so neck-only
restriction is incorrect.
• On CT they appear as multiloculated, fluid-attenuation masses that show little or no mural
enhancement; a conspicuous enhancing rim is not typical.
• MRI delineates extent well but is not obligatory; ultrasound or CT often suffice for diagnosis and
treatment planning, so option D is false.
• Calcification is rare, not common, in lymphangiomas; hence option E is wrong.
EBDR Exam MCQs & Concepts October 2021
Dr. Kareem Alnakeeb
296
18. On plain chest radiography, which pulmonary lesion classically appears as a solitary nodule with a
central “umbilication” or notch sign?
A. Hydatid cyst
B. Pulmonary metastasis
C. Both A and B
D. Rounded atelectasis
E. Pulmonary hamartoma
Explanation: (by Perplexity AI, generated with the OpenAI O3 model)
• The “umbilicated” or notched margin of a solitary lung nodule was first described in malignant
lesions, especially metastatic deposits from primaries such as colorectal or renal carcinoma. The
central indentation represents tumor infiltration along pulmonary vessels producing a focal
retraction of the nodule edge, a feature rarely seen in benign masses.
• Hydatid cysts usually present as smooth, spherical cystic opacities; when complicated they show
air–fluid signs (crescent, water-lily) rather than a focal umbilication, so option A is incorrect.
• Rounded atelectasis forms a pleural-based mass that blends with thickened pleura and displays the
comet-tail sign, not an umbilicated border; therefore option D is wrong.
• Pulmonary hamartomas characteristically contain “popcorn” calcification or fat and do not show a
notched margin.
19. Which radiographic features are typically seen in round atelectasis of the lung?
A. Crowded lung markings
B. Pleural thickening
C. Peripheral mass or nodule
D. All of the above
E. None of the above
Explanation: (by Perplexity AI, generated with the OpenAI O3 model)
• Round atelectasis is characterized by a triad of imaging findings on chest radiograph and CT:
1. a peripheral mass or nodule, typically in the lower lobes;
2. adjacent pleural thickening, often related to prior pleural disease such as effusion or asbestos
exposure; and
3. crowding or curving of pulmonary vessels and bronchi toward the lesion, producing the “comet
tail” sign (which reflects crowding of lung markings).
• All individual features listed in options A–C are essential to its recognition, making “all of the above”
the correct choice.
• Isolated crowding or thickening without a mass does not constitute round atelectasis.
20. Round atelectasis: (True or False)
A. has ill-defined edges.
B. is always pleurally based.
C. is always associated with pleural thickening.
D. has a ‘comet tail’ appearance.
E. caused by a previous pleural transudate.
Source: Scoffings, Daniel. Get Through FRCR 2A: Practice Papers for the Modular Examination. 1st
ed., CRC Press, 2004.
Explanation:
A. True
B. True
C. True
D. True
EBDR Exam MCQs & Concepts October 2021
Dr. Kareem Alnakeeb
297
E. False — exudate.
21. Which imaging appearances have been described as showing an intralesional “crescent sign” of
peripheral air outlining internal material?
A. Cyst containing hemorrhage and debris
B. Contained or frank rupture of a cyst with escape of debris
C. Intracavitary pulmonary mycetoma (aspergilloma)
D. All of the above
E. None of the above
Explanation: (by Perplexity AI, generated with the OpenAI O3 model)
• When a pulmonary hydatid or other cyst partially fills with air while still containing blood or debris, a
crescent-shaped rim of gas may separate the cyst wall from the internal content, producing the
classic crescent (meniscus) sign on radiographs or CT.
• If the cyst ruptures into a bronchus, additional air dissects between the pericyst and endocyst, again
generating an air crescent before complete collapse; debris may be visible in the dependent portion.
• A mature aspergilloma forms a mobile fungus ball within a pre-existing cavity; air immediately above
the intracavitary mass creates the identical crescent/meniscus configuration (also called the Monod
sign).
Because the same aerated meniscus can be produced in all three scenarios, “all of the above” is the
single best answer.
22. In HRCT terminology, which term best describes a gas-filled pulmonary space whose wall is thicker
than 3 mm?
A. Emphysematous bulla
B. Pulmonary cyst
C. Tuberculous cavity
D. Pleural bleb
E. Pneumatocele
Explanation: (by Perplexity AI, generated with the OpenAI O3 model)
• A pulmonary cavity is defined by the Fleischner Society as a gas-filled space within lung
parenchyma, consolidation or a nodule whose wall is usually thicker than 4 mm; clinically, post-
primary tuberculosis is the classic cause, so a “TB cavity” fits this description.
• Bullae and blebs have hairline-thin walls (<1 mm) and arise from emphysema or along the pleura,
respectively.
• A pulmonary cyst is also thin-walled (<2 mm) and usually round.
• A pneumatocele is a transient, thin-walled, traumatic or infective air collection.
• Thus, wall thickness >3 mm rules out cyst, bulla, bleb and pneumatocele, leaving a cavity—
commonly tuberculous—as the correct choice.
23. On a routine chest radiograph, which of the following is NOT a recognized sign of lung hyperinflation?
A. Horizontal ribs
B. Low, flattened diaphragm
C. Narrow retrosternal clear space
D. Tubular “ribbon” heart
E. Widened intercostal spaces
EBDR Exam MCQs & Concepts October 2021
Dr. Kareem Alnakeeb
298
Explanation: (by Perplexity AI, generated with the OpenAI O3 model)
• Classic radiographic clues to pulmonary hyperinflation—typically seen in emphysema—include low,
flat hemidiaphragms and more horizontal ribs caused by increased intrathoracic volume.
• The anteroposterior (AP) diameter increases, so the heart appears elongated and tubular (“ribbon
heart”) on the frontal film, and the intercostal spaces widen.
• In contrast, the retrosternal clear space on the lateral view becomes wider, not narrower, because
the over-inflated lungs encroach anteriorly.
• Therefore a “narrow” retrosternal space would argue against hyperinflation, making option C the
exception.
24. In systemic staging of primary lung carcinoma, which organ is the most frequent target of hematogenous
metastasis seen at diagnosis or autopsy studies?
A. Brain
B. Contralateral lung
C. Skeleton
D. Adrenal gland
E. Liver
Explanation: (by Perplexity AI, generated with the OpenAI O3 model)
• Autopsy series and large imaging datasets consistently show that blood-borne spread from lung
cancer most often deposits in the adrenal cortex, with reported involvement in 30–40% of cases.
• Although brain and bone metastases are also common, their incidence is lower.
• Contralateral pulmonary nodules usually represent intrapulmonary spread rather than true distant
hematogenous metastasis, and hepatic deposits rank below adrenal involvement in frequency.
• Recognising the adrenal glands as the prime hematogenous “landing site” is critical because small,
silent adrenal metastases upstage the tumor to stage IV and alter management.
25. On chest radiography showing innumerable 1–3 mm miliary nodules that are densely calcified, which one
of the following causes is LEAST likely?
A. Silicosis
B. Healed histoplasmosis
C. Active miliary tuberculosis
D. Broncholithiasis
E. Metastatic papillary thyroid cancer
Explanation: (by Perplexity AI, generated with the OpenAI O3 model)
• Dense calcification in tiny, randomly distributed miliary nodules classically reflects healed or chronic
processes that produce dystrophic calcium deposition.
• Pneumoconiosis such as silicosis and healed granulomatous infections like histoplasmosis
frequently leave innumerable calcified micronodules.
• Calcified airway‐extruded granulomas (broncholithiasis) can scatter calcified fragments into
adjacent lung, also giving a miliary pattern.
• Certain metastases (not asked originally but included here for option balance) from papillary or
medullary thyroid carcinoma may calcify in a miliary fashion.
• In contrast, active miliary tuberculosis usually presents with non-calcified micronodules;
calcification, if it occurs, is a late sequela after healing. Therefore active miliary TB is the least
compatible with a calcified miliary appearance.
EBDR Exam MCQs & Concepts October 2021
Dr. Kareem Alnakeeb
299
26. On chest imaging, which radiographic feature best helps distinguish a pyogenic lung abscess from a
recently-ruptured pulmonary hydatid cyst?
A. Associated parenchymal consolidation
B. Pleural effusion
C. Air–fluid interface: straight in abscess, wavy or undulating in ruptured hydatid cyst
D. Typical basal location
Explanation: (by Perplexity AI, generated with the OpenAI O3 model)
• A lung abscess usually forms a thick-walled cavity containing pus; its fluid settles evenly, so the air–
fluid margin is flat and horizontal on erect films or CT.
• When a hydatid cyst ruptures into a bronchus, collapsed membranes float on residual fluid
producing an irregular, wavy“water-lily” interface rather than a straight line; this undulating margin
is a classic clue to cyst rupture.
• Consolidation (A) and reactive pleural effusion (B) can accompany either condition and are therefore
non-specific.
• Both lesions may appear anywhere in the lungs, so site preference (D) is unreliable for differentiation.
27. On HRCT, a diffuse “crazy-paving” pattern of ground-glass opacities with superimposed inter- and
intralobular septal thickening is classically associated with which pulmonary disorder?
A. Pulmonary alveolar proteinosis
B. Post-primary tuberculosis
C. Malignant pleural mesothelioma
D. Invasive pulmonary aspergillosis
E. Pneumonic‐type adenocarcinoma
Explanation: (by Perplexity AI, generated with the OpenAI O3 model)
• Pulmonary alveolar proteinosis (PAP) was the condition in which the crazy-paving sign was first
described and it remains the prototypical cause; 80–90% of PAP cases show this striking reticular
network over ground-glass opacity.
• Tuberculosis, aspergillosis and a long list of other infections can occasionally mimic the sign, but
they do so far less frequently and typically with additional focal nodules or cavities that break the
uniform crazy-paving appearance.
• Mesothelioma is a pleural tumor; parenchymal crazy paving is not a recognized feature.
28. In cardiac MRI, cine balanced steady-state free-precession (SSFP) sequences are most commonly used
for which of the following purposes?
A. Quantitative assessment of left-ventricular function
B. Quantitative assessment of right-ventricular function
C. Detection of regional myocardial hypokinesia
D. All of the above
E. Coronary artery stenosis grading
Explanation: (by Perplexity AI, generated with the OpenAI O3 model)
• Cine SSFP yields high blood-to-myocardium contrast and excellent temporal resolution, allowing
accurate measurement of ventricular volumes and ejection fractions for both
the left and right ventricles. The bright-blood depiction and crisp endocardial borders also make wall
motion abnormalities such as hypokinesia or akinesia readily visible on the dynamic images.
• Coronary artery lumen assessment, however, generally requires dedicated MR angiography or CT, not
standard cine SSFP. Thus options A, B and C are all correct, making D the single best answer.
EBDR Exam MCQs & Concepts October 2021
Dr. Kareem Alnakeeb
300
29. On chest CT, which finding is considered the most specific diagnostic clue for a pulmonary aspergilloma
within a pre-existing cavity?
A. Crescent (Monod) sign
B. Demonstrable mobility of the intracavitary fungal ball between different patient positions
C. Pericavitary consolidation with pleural thickening
D. Predilection for the right upper lobe
E. Tree-in-bud nodularity
Explanation: (by Perplexity AI, generated with the OpenAI O3 model)
• A true aspergilloma is a non-invasive “fungus ball” that lies loose inside an old tuberculous or other
cavitary lesion. The hallmark is that the mycetoma is free-moving; when the patient changes from
supine to prone or erect, the soft-tissue mass settles to the new dependent part, proving there is no
attachment—this positional mobility is the single most specific sign.
• The air-crescent (Monod) sign (A) is common but can also appear with cavitating cancer or abscess,
so it is less specific.
• Pericavitary consolidation or pleural thickening (C) merely indicates inflammation and is neither
sensitive nor specific.
• Although most aspergillomas occur in upper-lobe cavities, side predilection (D) is variable and
therefore not diagnostic.
• Tree-in-bud changes (E) suggest endobronchial infection, not an aspergilloma.
30. With respect to pulmonary aspergillosis:
A. an aspergilloma is usually associated with pleural thickening.
B. a fungal ball may be seen to change position within a cavity.
C. allergic bronchopulmonary aspergillosis (ABPA) causes the ‘gloved finger’ sign.
D. ABPA is the commonest cause of pulmonary eosinophilia in the UK.
E. With repeated episodes of ABPA, the changes are more severe peripherally.
Source: Scoffings, Daniel. Get Through FRCR 2A: Practice Papers for the Modular Examination. 1st
ed., CRC Press, 2004.
Explanation:
A. True
B. True
C. True — dilated branching tubular opacities on the PA chest radiograph are due to mucoid impaction
in dilated bronchi.
D. True
E. False — centrally.
31. In ultrasound-guided thoracocentesis, which statement is correct?
A. The needle should be introduced below the rib.
B. Pleural fluid should be withdrawn rapidly.
C. A chest radiograph after tube insertion is mandatory.
D. Both A and C.
Explanation: (by Perplexity AI, generated with the OpenAI O3 model)
• Real-time ultrasound marks a safe window; once the drain is sited a post-procedure chest X-ray is
recommended to confirm position and exclude pneumothorax, especially when a catheter or tube
remains in situ.
EBDR Exam MCQs & Concepts October 2021
Dr. Kareem Alnakeeb
301
• The needle must traverse the upper border of the lower rib to avoid the intercostal vessels and nerve
that run in the costal groove immediately below each rib—placing it “below the rib”greatly
increases bleeding risk, so option A is wrong.
• Therapeutic taps should be slow, intermittent and limited (≤1.5 L at a time) to minimize re-
expansion pulmonary oedema; rapid aspiration is unsafe, making option B incorrect.
32. In aortic dissection, which statement about the Stanford classification is correct?
A. It has completely replaced the DeBakey system.
B. “Stanford type A” refers to dissections limited to the descending thoracic aorta.
C. The classification is unrelated to treatment strategy.
D. None of the above.
Explanation: (by Perplexity AI, generated with the OpenAI O3 model)
• The Stanford system co-exists with (but has not replaced) the older DeBakey scheme, offering a
simpler two-group approach.
• Type A includes any dissection that involves the ascending aorta, whether or not it extends distally;
type B is confined to the descending aorta distal to the left subclavian artery. Therefore option B is
incorrect.
• The key virtue of the Stanford system is its link to management: type A dissections usually require
urgent surgery, whereas most uncomplicated type B dissections are managed medically at
first. Hence option C is wrong.
• With all three statements incorrect, “None of the above”is the only accurate choice.
33. Which plain chest radiograph finding is classically associated with a tension pneumothorax?
A. Ipsilateral transient hemithorax
B. Ipsilateral depressed diaphragm
C. Ipsilateral mediastinal shift
D. Lung collapse
E. Rib “splinting”
Explanation: (by Perplexity AI, generated with the OpenAI O3 model)
• A tension pneumothorax builds positive intrapleural pressure that pushes the diaphragm downward
on the affected side, producing an ipsilateral depressed or flattened hemidiaphragm—one of the
key radiographic clues.
• Lung collapse (D) and contralateral (not ipsilateral) mediastinal shift are typical; option C is therefore
incorrect.
• “Transient” hemithorax (A) is not a recognised term, and rib splinting (E) is clinical, not radiographic.
34. Which of the following conditions is NOT a recognised cause of thin-walled cystic lesions in the lungs?
A. Lymphangioleiomyomatosis
B. Pulmonary Langerhans cell histiocytosis
C. Congestive heart failure
D. Cystic bronchiectasis
E. Birt–Hogg–Dubé syndrome
Explanation: (by Perplexity AI, generated with the OpenAI O3 model)
• True pulmonary cysts are produced by a limited group of diffuse lung diseases such as
lymphangioleiomyomatosis (LAM), pulmonary Langerhans cell histiocytosis (PLCH) and genetic
disorders like Birt–Hogg–Dubé syndrome.
EBDR Exam MCQs & Concepts October 2021
Dr. Kareem Alnakeeb
302
• Cystic bronchiectasis represents markedly dilated bronchi that mimic cysts on CT; it is therefore
included in the differential list of cyst-appearing lung lesions.
• Congestive heart failure causes interstitial oedema, Kerley lines and pleural effusions; it does not
generate parenchymal cysts, making option C the exception.
35. A 60-year-old man with a 30-year heavy-smoking history presents with digital clubbing, hypertrophic
osteoarthropathy, hypercalcemia and cerebellar ataxia; CT shows a centrally located hilar lung mass.
What is the most likely histological subtype?
A. Adenocarcinoma
B. Small-cell carcinoma
C. Large-cell carcinoma
D. Squamous-cell carcinoma
E. Bronchoalveolar carcinoma
Explanation: (by Perplexity AI, generated with the OpenAI O3 model)
• Squamous-cell carcinoma typically arises from a lobar or main bronchus, so it appears as a central
hilar mass on CT. It is the lung cancer most strongly linked to secretion of parathyroid-hormone-
related peptide, giving rise to paraneoplastic hypercalcemia.
• Clubbing and hypertrophic osteoarthropathy are common non-small-cell manifestations and can
accompany squamous tumors. Although paraneoplastic cerebellar degeneration is classically
described with small-cell carcinoma, it may occur with any histology; in this vignette the powerful
combination of central location and hypercalcemia outweighs that pointer.
• Adenocarcinoma and large-cell carcinoma are usually peripheral, while small-cell carcinoma
rarely causes hypercalcemia.
36. A young boy presents with signs of right-sided heart failure and mediastinal shift; chest radiograph shows
a tubular shadow along the right cardiac border that is widest near the diaphragm, giving a “snowman”
(figure-of-eight) appearance. Which congenital lesion most commonly produces this radiographic sign?
A. Atrial septal defect
B. Aortic aneurysm
C. Total anomalous pulmonary venous return (supracardiac type)
D. Pulmonary sequestration
E. Persistent left superior vena cava
Explanation: (by Perplexity AI, generated with the OpenAI O3 model)
• The classic “snowman” or figure-of-eight sign on a frontal chest X-ray is highly characteristic of
supracardiac total anomalous pulmonary venous return (TAPVR).
• The upper “head” of the snowman is formed by dilated vertical and innominate veins on the left and
an enlarged superior vena cava on the right; the lower “body” is the enlarged right atrium and
ventricle.
• The tubular right-paracardiac mass that tapers superiorly and is broadest inferiorly represents the
confluence of these vessels, often causing mediastinal widening and right-sided volume overload.
• An isolated atrial septal defect may enlarge the right heart but does not create the snowman
configuration.
• Aortic pathology, pulmonary sequestration, or a persistent left SVC project differently and lack the
characteristic figure-of-eight outline.
EBDR Exam MCQs & Concepts October 2021
Dr. Kareem Alnakeeb
303
37. Chest X-ray of a boy shows shift of the heart and mediastinum to the right. There is also a tubular
structure parallel to the right heart border with its maximum width close to the diaphragm. The finding
suggests
A. ASD
B. Scimitar syndrome
C. Total anomalous pulmonary venous return
D. Intralobar sequestration
E. Inhaled foreign body
Source: Chin, Teck Yew, et al. Get Through Final FRCR 2A: SBAs. 1st ed., CRC Press, 2017.
Explanation:
Hypogenetic Lung Syndrome Overview
• Hypogenetic lung syndrome, also known as congenital venolobar syndrome or scimitar syndrome, is
primarily a complex developmental lung abnormality with anomalous venous return.
Hallmark Anatomical Features
• One constant component of this syndrome is an anomalous pulmonary vein or veins draining at least
a part or the entire affected lung most commonly to the inferior vena cava just above or below the
diaphragm.
• Uncommonly, the anomalous vein may drain into hepatic, portal, azygos veins; the coronary sinus; or
the right atrium.
Common Clinical and Radiographic Findings
• The most common features are lung hypoplasia, anomalous pulmonary venous return to IVC,
pulmonary artery hypoplasia, bronchial anomalies and systemic arterial supply to hypoplastic lung.
It almost always occurs on the right side and is slightly more common in women.
• A scimitar vein is a vertical curvilinear opacity in the right mid-lower lung, running along the right
heart border inferomedially towards the diaphragm to join the IVC. A scimitar vein present on a
frontal chest radiograph is called the scimitar sign.
38. A 10-year-old boy presents with fever and eosinophilia. MRI of the head shows thickening of the
infundibular stalk and a markedly enhancing mass in the superior aspect of the stalk. There is also
enhancement in the sella extending along the left petrous temporal bone with poorly defined borders.
The features are consistent with
A. Meningioma
B. Petrous apicitis
C. Histiocytosis X
D. Craniopharyngioma
E. Neuroblastoma metastasis
Source: Chin, Teck Yew, et al. Get Through Final FRCR 2A: SBAs. 1st ed., CRC Press, 2017.
Explanation:
Hypothalamic–Neurohypophyseal Axis Involvement
• Space-occupying lesions affect the hypothalamic-neurohypophyseal axis, which is the central
nervous system site most commonly and often earliest involved in Langerhans cell histiocytosis.
• MRI findings have been correlated with symptoms of diabetes insipidus, which is a clinical hallmark
of the condition.
• Typically, the formation of Langerhans cell histiocytosis granulomas leads to a loss in the normally
high signal intensity of the posterior neurohypophysis on T1-weighted images.
EBDR Exam MCQs & Concepts October 2021
Dr. Kareem Alnakeeb
304
• Furthermore, the hypothalamus, the pituitary stalk or both are frequently enlarged and demonstrate
gradually increasing homogeneous enhancement after an intravenous injection of gadolinium,
without subsequent washout.
Differential Diagnosis of Infundibular Lesions
• The differential diagnosis includes other infundibular diseases, such as adenohypophysitis, which
can be differentiated from Langerhans cell histiocytosis by a sharp increase in contrast
enhancement and rapid washout after the administration of the intravenous contrast medium.
• Granulomatous diseases such as sarcoidosis, Wegener disease and leukaemia must also be
considered in the differential.
• Rarer differentials are germ cell tumours (germinoma, teratoma) and haemangioblastoma.
• These produce the same MRI features, with the same pattern of enhancement at dynamic imaging.
Neurodegenerative Intra-Axial Pattern
• The second most frequent pattern of central nervous system involvement in Langerhans cell
histiocytosis is characterised by intra-axial neuro-degenerative changes.
• Bilateral symmetric lesions in the cerebellum, especially the dentate nucleus, basal ganglia, or
brainstem, are most often observed.
Differential Diagnosis of Neurodegenerative Pattern
• The differential diagnosis includes ADEM, acute multiphasic disseminated encephalitis,
disseminated encephalitis, various metabolic and degenerative disorders, leukoencephalopathy
secondary to chemotherapy or radiation therapy, and paraneoplastic encephalitis.
Extra-Axial Granulomas
• Less frequently, Langerhans cell histiocytosis granulomas, which resemble tumours, are observed in
the extra-axial space (in the meninges, pineal gland, choroid plexus and spinal cord).
39. A 4-year-old boy falls off his bike and complains of neck pain. Which of the following features is worrying
for a serious injury on plain cervical X-rays?
A. Atlanto-axial distance <5 mm
B. Displacement of 6 mm of the lateral masses relative to the dens
C. Absence of lordosis
D. Disruption of the spinolamellar line
E. Anterior subluxation of C2 on C3
Source: Chin, Teck Yew, et al. Get Through Final FRCR 2A: SBAs. 1st ed., CRC Press, 2017.
Explanation:
Atlantoaxial Interval
• The atlantoaxial interval is defined as the distance between the anterior aspect of the dens and the
posterior aspect of the anterior ring of the atlas.
• This distance should be 5 mm or less.
Atlas–Axis Radiographic Variants
• Pseudospread of the atlas on the axis (‘pseudo–Jefferson fracture’) can be seen on anterior open-
mouth radiographs.
• Up to 6 mm of displacement of the lateral masses relative to the dens is common in patients up to 4
years old and may be seen in patients up to 7 years old.
• On extension radiographs, overriding of the anterior arch of the atlas onto the odontoid process can
be seen in 20% of healthy children.
Pediatric Cervical Spine Alignment
EBDR Exam MCQs & Concepts October 2021
Dr. Kareem Alnakeeb
305
• In children, the C2–3 space and, to a lesser extent, the C3–4 space have a normal physiologic
displacement.
• The absence of lordosis, although potentially pathologic in an adult, can be seen in children up to 16
years of age when the neck is in a neutral position.
Spinous Process Distances and Ligamentous Integrity
• In children, the flexion manoeuvre can increase the distance between the tips of the C1 and C2
spinous processes.
• Normal posterior intraspinous distance is a good indicator of ligamentous integrity and should not be
more than 1.5 times greater than the intraspinous distance one level either above or below the level
in question.
Vertebral Body Wedging
• Anterior wedging of up to 3 mm of the vertebral bodies should not be confused with compression
fracture.
• Such wedging can be profound at the C3 level.
Prevertebral Soft Tissue Measurements
• A prevertebral space of less than 6 mm at the level of C3 is considered normal in children.
• In paediatric patients, widening of the prevertebral soft tissues can be a normal finding that is related
to expiration.
Injury Indicator
• Disruption of the spinolamellar line is a sign of injury.
40. A 9-month-old girl presents with a palpable pelvic mass; MRI demonstrates a large presacral lesion with
markedly heterogeneous T1 signal containing fat, fluid and soft-tissue components. What is the most
likely diagnosis?
A. Anterior sacral meningocele
B. Embryonal rhabdomyosarcoma
C. Sacrococcygeal teratoma
D. Sacral chordoma
E. Pelvic neuroblastoma
Explanation: (by Perplexity AI, generated with the OpenAI O3 model)
• Sacrococcygeal teratomas commonly present in infants as mixed cystic-solid presacral masses.
Their diverse contents—fat, hemorrhage, calcification, and fluid—produce a strikingly
heterogeneous T1 appearance, often with areas of intrinsic high signal from fat and blood, making
this diagnosis the best fit.
• Anterior sacral meningoceles follow CSF signal on all sequences and appear homogeneously low
on T1, not heterogeneous.
• Embryonal rhabdomyosarcomas are usually iso- to hypointense on T1 and lack macroscopic fat.
• Sacral chordomas are rare in this age group and classically show high T2 signal with low-to-
intermediate T1 without macroscopic fat.
• Pelvic neuroblastomas arise from sympathetic chain tissue, are most often retroperitoneal, and
usually appear homogeneously low-to-intermediate on T1 without fat content.
EBDR Exam MCQs & Concepts October 2021
Dr. Kareem Alnakeeb
306
41. A 13-year-old post-pubescent girl presents to the emergency department with acute abdominal pain
sited predominantly within the right iliac fossa. An ultrasound scan is performed. This reveals an
echogenic mass within the right side of pelvis measuring approximately 4 cm. The sonographer thinks it is
adjacent to and inseparable from the right ovary. What is the most likely diagnosis?
A. Acute appendicitis
B. Ovarian dermoid
C. Ovarian torsion
D. Ectopic pregnancy
E. Hemorrhagic ovarian cyst
Source: Chin, Teck Yew, et al. Get Through Final FRCR 2A: SBAs. 1st ed., CRC Press, 2017.
Explanation:
Causes of Acute Pelvic Pain in Adolescent Girls
• Acute pelvic pain in adolescent girls is a common problem, but ultrasound scanning is very useful in
differentiating the many possible causes.
Hemorrhagic Ovarian Cysts
• Hemorrhagic ovarian cysts are a common cause of pelvic pain in adolescent girls and appear as an
echogenic mass in relation to the ovary.
Acute Appendicitis
• Acute appendicitis is likely to occur as a blind-ending tubular structure.
• This may appear like a ‘target lesion’ in cross section and there may be fluid/collection adjacent to it.
• An acute appendix should be clearly distinct from the right ovary.
Ovarian Dermoids
• Ovarian dermoids are usually predominantly fat filled and therefore echogenic on ultrasound, but
these tend to be a painless, incidental finding.
Ovarian Torsion
• Ovarian torsion can certainly produce an echogenic mass within the right pelvis, but this is less
common than hemorrhagic cysts and would not appear distinct from the ovary.
Ectopic Pregnancy
• Ectopic pregnancy usually appears as a ‘doughnut’-shaped complex mass in relation to one of the
uterine tubes; a fetal heartbeat may be present.
42. Barium enema of a neonate shows an inverted cone shape at the rectosigmoid colon. There is marked
retention of the barium on delayed post-evacuation films after 24 hours. The cause for this is
A. Meconium ileus
B. Meconium plug syndrome
C. Hirschprung’s disease
D. Imperforate anus
E. Hyperplastic polyp of colon
Source: Chin, Teck Yew, et al. Get Through Final FRCR 2A: SBAs. 1st ed., CRC Press, 2017.
Explanation:
Overview
• Hirschsprung’s disease, also called aganglionosis of the colon (absence of parasympathetic ganglia
in muscle and submucosal layers secondary to an arrest of craniocaudal migration of neuroblasts),
results in relaxation failure of the aganglionic segment.
• It affects full-term infants during the first weeks of life, mainly boys.
• It is extremely rare in premature infants.
EBDR Exam MCQs & Concepts October 2021
Dr. Kareem Alnakeeb
307
• It usually affects the rectosigmoid junction and results in short-segment disease (80%).
• Long-segment disease (20%) and total colonic aganglionosis (5%) are less common.
Imaging Findings
• Barium enema shows a ‘transition zone’ (aganglionic segment), which appears normal in size with
dilatation of large and small bowel proximally with marked retention of barium on delayed films after
24 hours.
• Normal children show a rectosigmoid ratio of >1, as the rectum is larger in diameter than the
sigmoid; in the case of Hirschsprung’s disease, the ratio is reversed (rectosigmoid ratio <1).
43. All of the following are recognised indications for transjugular intrahepatic portosystemic shunt (TIPS),
except
A. Right heart failure
B. Budd–Chiari
C. Refractory ascites
D. Acute variceal bleeding
E. Portal hypertensive gastropathy
Source: Chin, Teck Yew, et al. Get Through Final FRCR 2A: SBAs. 1st ed., CRC Press, 2017.
Explanation:
TIPS Procedure Effectiveness
• The TIPS procedure is effective in achieving portal decompression and in managing some of the
major complications of portal hypertension.
Indications for TIPS
• Indications for TIPS include variceal bleeding, secondary prevention, acute bleeding refractory to
medical and endoscopic treatments, refractory ascites, hepatorenal syndrome, Budd–Chiari
syndrome, hepatic veno-occlusive disease, hepatic hydrothorax and portal hypertensive gastropathy.
44. Which of the following statements about modern inferior vena cava (IVC) filters is FALSE?
A. Current filters require surgical cut-down before insertion
B. Filters can be introduced via either the femoral or internal jugular vein
C. The infrarenal IVC just below the iliac confluence is the usual deployment site
D. The majority of commercially available filters cannot be retrieved
Explanation: (by Perplexity AI, generated with the OpenAI O3 model)
• Most contemporary IVC filters are inserted percutaneously through a sheath; surgical venous cut-
down is no longer necessary, making option A false.
• Filters are routinely delivered from either femoral or jugular access depending on anatomy and
operator preference, so option B is correct.
• Standard placement is in the infrarenal IVC (immediately below the lowest renal vein), often a few
centimeters above the iliac confluence, validating option C.
• Although permanent designs still exist, the market has shifted toward retrievable filters, so “most
filters are un-removable” is inaccurate—option D is therefore true (the statement is correct) because
permanent models are now the minority.
EBDR Exam MCQs & Concepts October 2021
Dr. Kareem Alnakeeb
308
45. A 72-year-old man with unresectable pancreatic head carcinoma develops worsening obstructive
jaundice; ERCP cannulation is straightforward and the duodenum is patent. Which biliary drainage option
offers the longest patency with the fewest re-interventions?
A. Self-expanding metallic biliary stent placed endoscopically for malignant obstruction and
prolonged use
B. Plastic biliary stent used temporarily via ERCP or PTC access
C. Percutaneous transhepatic external biliary drain when the common bile duct is completely occluded
by tumor
D. Balloon dilatation of the malignant biliary stricture without stent placement
E. Surgical hepaticojejunostomy bypass
Explanation: (by Perplexity AI, generated with the OpenAI O3 model)
• Self-expanding metallic stents (SEMS) maintain luminal patency for 6–12 months, resist tumour
ingrowth and require fewer exchanges, making them first-line for palliative drainage of malignant
distal biliary obstruction when endoscopic access is feasible.
• Plastic stents (B) clog within weeks, so they suit short-term or pre-operative scenarios.
• External drains (C) relieve jaundice but tether the patient to a bag and are reserved for failed
endoscopy or complete ductal occlusion.
• Balloon dilatation alone (D) has high restenosis rates within days.
• Surgical bypass (E) carries higher morbidity and is generally reserved for fit patients expected to
survive >6 months or when endoscopic options fail.
46. Concerning stent insertion for biliary strictures due to cholangiocarcinoma: (True or False)
A. Metallic stents are preferable if there is marked tumor invasion of the duodenum.
B. Metallic stents cannot be removed.
C. Balloon-expandable stents are preferable to self-expanding stents.
D. Covered stents offer no significant increase in patency rates to non-covered stents.
E. The 12-month patency rate is greater for hilar than non-hilar obstruction.
Source: Scoffings, Daniel. Get Through FRCR 2A: Practice Papers for the Modular Examination. 1st
ed., CRC Press, 2004.
Explanation:
A. False — this is an indication for a plastic stent.
B. True
C. False —self-expanding stents are less rigid, and so easier to deploy around a curve.
D. True
E. False — following metallic stent insertion, the 12-month patency for hilar obstruction is 46%, and for
non-hilar obstruction 89%.
47. In patients whose right upper pulmonary vein anomalously drains into the superior vena cava, which
congenital cardiac defect is classically associated?
A. Patent foramen ovale
B. ASD (septum primum type)
C. Ventricular septal defect
D. ASD (sinus venosus type)
E. None of the above
EBDR Exam MCQs & Concepts October 2021
Dr. Kareem Alnakeeb
309
Explanation: (by Perplexity AI, generated with the OpenAI O3 model)
• A sinus venosus atrial septal defect lies high in the atrial septum near the superior vena cava (SVC)–
right atrium junction.
• Because of this proximity, it frequently co-exists with partial anomalous pulmonary venous
connection in which the right upper or middle lobe pulmonary veins empty directly into the SVC
rather than the left atrium.
• Septum primum ASDs are positioned low and are not typically linked to anomalous pulmonary
venous drainage, while a patent foramen ovale and ventricular septal defects have no anatomic
relationship with the SVC–pulmonary venous confluence, making such association uncommon.
• Therefore, the presence of a right upper pulmonary vein draining into the SVC strongly points to a
sinus venosus ASD.
48. In routine cardiac MRI, so-called “black-blood” double-inversion sequences are primarily used for which
purpose?
A. Defining cardiac anatomy and wall morphology
B. Cine imaging of ventricular function
C. Selective suppression of epicardial fat
D. Assessing myocardial perfusion under stress
E. All of the above
Explanation: (by Perplexity AI, generated with the OpenAI O3 model)
• Black-blood sequences apply a double-inversion recovery pulse to null signal from flowing blood,
leaving the myocardium and adjacent structures conspicuous. This high-resolution, motion-
compensated technique is ideal for depicting cardiac chamber morphology, ventricular wall
thickness, thrombus, masses and great-vessel anatomy.
• It is not a cine method; bright-blood balanced SSFP or GRE sequences are used to assess ventricular
function.
• Fat suppression relies on frequency-selective or Dixon techniques, not the flow-nulling used here.
• Myocardial perfusion requires rapid T1-weighted imaging during contrast bolus, again a different
sequence. Therefore only anatomical assessment is the correct application.
49. A 37-year-old woman involved in a frontal car collision and collapse at the scene of incident was brought
to the A&E department and sent for an emergency whole-body CT. All of the following are correct
regarding blunt cardiac trauma, except
A. Cardiac concussion results in abnormal cardiac enzymes.
B. Traumatic pericardial rupture resulting from blunt chest trauma is rare.
C. Cardiac herniation is a serious complication of pericardial rupture.
D. Traumatic ventricular septal defects affect the muscular portion.
E. Myocardial contusion is associated with cardiac tamponade.
Source: Chin, Teck Yew, et al. Get Through Final FRCR 2A: SBAs. 1st ed., CRC Press, 2017.
Explanation:
Overview of Blunt Cardiac Injury and Related Conditions
• Blunt cardiac injury (BCI) is the most common type of cardiac injury after blunt thoracic trauma.
• In cardiac concussion, the mildest form of cardiac injury, there is no myocardial cell damage or
elevated enzyme levels.
EBDR Exam MCQs & Concepts October 2021
Dr. Kareem Alnakeeb
310
• Cardiac contusion can present as bilateral cardiogenic pulmonary oedema and elevated cardiac
enzymes.
Diagnostic Findings in Cardiac Contusion
• Echocardiography shows increased myocardial echogenicity and focal systolic hypokinesia, and it is
useful in diagnosing other traumatic cardiac injuries that are commonly associated with myocardial
contusion, such as pericardial effusion, tamponade, traumatic ventricular septal defect and valvular
injury.
• Typically, traumatic ventricular septal defects occur in the muscular portion of the interventricular
septum, near the cardiac apex.
Haemopericardium and Tamponade
• Haemopericardium is commonly associated with cardiac rupture.
• Tamponade resulting from ventricular rupture is often fatal; however, bleeding from lower pressure
atria may be survivable.
• Besides cardiac chamber rupture, traumatic haemopericardium may also result from aortic root
injury, myocardial contusion and coronary artery laceration.
Pericardial Rupture and Complications
• Traumatic pericardial rupture resulting from blunt chest trauma is rare.
• Tearing may involve either the pleuropericardium or the diaphragmatic pericardium.
• Cardiac herniation is a serious complication of pericardial rupture.
50. On cardiac MRI delayed-gadolinium enhancement, what percentage of myocardial wall thickness must
show enhancement to classify the infarct as transmural and therefore non-viable?
A. >25%
B. <50%
C. >75%
D. 50–75%
E. None of the above
Explanation: (by Perplexity AI, generated with the OpenAI O3 model)
• A transmural infarct is defined when late gadolinium enhancement involves more than 75% of the
myocardial wall, indicating full-thickness necrosis and negligible likelihood of functional recovery
after revascularization.
• Enhancements affecting 50–75% (option D) or <50% (option B) represent partial-thickness,
potentially viable myocardium, while >25% alone (option A) is too low a threshold.
• Option E is incorrect because a recognized threshold exists.
51. Regarding blunt traumatic injury to the thoracic aorta, which statement is INCORRECT?
A. Aortography is considered the historical gold-standard imaging modality
B. A tear of the ascending aorta is usually rapidly fatal at the scene
C. The majority of patients die before reaching hospital
D. The tear typically involves all three layers of the aortic wall (intima, media and adventitia)
E. Helical CT angiography has largely replaced catheter angiography in initial assessment
Explanation: (by Perplexity AI, generated with the OpenAI O3 model)
• Traumatic aortic rupture classically produces a partial-thickness laceration limited to the intima and
media; the adventitia (and overlying mediastinal pleura) often remain intact, forming a contained
pseudoaneurysm. Therefore option D is incorrect.
EBDR Exam MCQs & Concepts October 2021
Dr. Kareem Alnakeeb
311
• Catheter aortography was long regarded as the definitive test (option A correct), but in modern
practice multidetector CT angiography provides rapid, non-invasive diagnosis, largely superseding it
(option E correct).
• Tears of the ascending aorta occur close to the heart, leading to massive hemopericardium or
exsanguination and are almost universally fatal before imaging (option B correct).
• Epidemiological data show that most victims of blunt aortic injury succumb at the scene, with only a
minority reaching hospital alive (option C correct).
52. Which of the following chest radiograph findings is NOT a typical feature of chronic mitral stenosis in an
adult patient?
A. Small aortic knuckle
B. Double right-heart border from left atrial enlargement
C. Decreased retrosternal clear space
D. Widened carinal angle
E. Straightening of the left heart border
Explanation: (by Perplexity AI, generated with the OpenAI O3 model)
• Mitral stenosis produces long-standing left atrial enlargement, leading to the classic double right-
heart border and straightening of the left heart margin; the enlarged left atrium also elevates the left
main bronchus, widening the carinal angle.
• Reduced forward flow through the valve can make the aortic knuckle appear relatively small.
• A diminished retrosternal clear space reflects right-ventricular enlargement, which is far more
characteristic of pulmonary hypertension or pulmonary valve disease than of isolated mitral
stenosis; therefore option C is the exception.
• The other options all describe well-recognized radiographic signs of mitral stenosis.
53. Regarding coronary artery calcium (CAC) scoring on non-contrast ECG-gated CT, which statement is
FALSE?
A. It can predict future coronary events before clinical disease develops.
B. The calcium score is proportional to overall atherosclerotic burden.
C. Voxels must reach or exceed 130 HU to be counted in the Agatston score.
D. Individual calcification sites on CT almost always correspond to the exact locations of severe
luminal stenoses seen at invasive coronary angiography.
E. CAC scoring is performed on a breath-hold, ECG-synchronized, non-contrast CT acquisition.
Explanation: (by Perplexity AI, generated with the OpenAI O3 model)
• The CAC score reflects the global burden of coronary atherosclerosis; although a high score is
strongly associated with obstructive disease, the spatial match between any single calcified focus
and a critical stenosis at angiography is poor, making option D false.
• CAC measurement is obtained with a non-contrast, ECG-gated CT; voxels ≥130 HU and ≥1 mm² are
counted to generate the Agatston score, which rises with increasing plaque burden (options B, C, E
true).
• Because calcium accrual generally precedes symptomatic coronary artery disease, CAC scoring aids
risk stratification in asymptomatic individuals (option A true).
EBDR Exam MCQs & Concepts October 2021
Dr. Kareem Alnakeeb
312
54. A 56-year-old woman presents with recent-onset exertional dyspnea. Echocardiography shows left-
ventricular systolic dysfunction. Cardiac MRI demonstrates focal hypokinesia, and late gadolinium
enhancement reveals high signal confined to the subendocardium in the affected territory. What is the
most likely diagnosis?
A. Myocarditis
B. Myocardial infarction
C. Hypertrophic cardiomyopathy
D. Cardiac amyloidosis
E. Sarcoidosis
Explanation: (by Perplexity AI, generated with the OpenAI O3 model)
• Subendocardial or transmural late gadolinium enhancement (LGE) that conforms to a coronary
artery distribution is characteristic of ischemic injury, indicating infarction and fibro-scar formation.
• Myocarditis usually shows mid-wall or epicardial LGE, hypertrophic cardiomyopathy classically
exhibits patchy mid-wall or junctional LGE in hypertrophied segments, and amyloidosis produces
diffuse global subendocardial or transmural LGE often with rapid blood-pool wash-out.
• Sarcoidosis more often gives patchy mid-wall or subepicardial LGE not limited to a vascular territory.
• Therefore, myocardial infarction is the best fit given the purely subendocardial LGE pattern.
55. On color Doppler ultrasound, which peak systolic velocity (PSV) threshold is most commonly used to
indicate a hemodynamically significant (>60%) renal artery stenosis?
A. 100 cm/s
B. 150 cm/s
C. 200 cm/s
D. 250 cm/s
E. 300 cm/s
Explanation: (by Perplexity AI, generated with the OpenAI O3 model)
• A PSV of approximately 200 cm/s is widely accepted as the diagnostic cut-off for ≥60% renal artery
stenosis because, at this velocity, turbulence and increased flow reflect a critical luminal narrowing
that produces a measurable pressure gradient.
• Lower thresholds (100 cm/s and 150 cm/s) would yield many false positives, as normal renal arteries
frequently exceed these speeds.
• Higher thresholds (250 cm/s and 300 cm/s) risk missing clinically important lesions because
significant stenoses may not drive PSV to such extreme levels.
• Therefore, 200 cm/s provides the best balance of sensitivity and specificity in routine vascular
laboratories.
56. A 67-year-old man with several episodes of acute variceal bleeding is being investigated prior to TIPS
procedure. Which one of the following is not a contraindication to TIPS?
A. Tricuspid regurgitation
B. Severe congestive cardiac failure
C. Multiple hepatic cysts
D. Severe portal hypertension
E. Unrelieved biliary obstruction
Source: Chin, Teck Yew, et al. Get Through Final FRCR 2A: SBAs. 1st ed., CRC Press, 2017.
EBDR Exam MCQs & Concepts October 2021
Dr. Kareem Alnakeeb
313
Explanation:
Contraindications to placement of a TIPS
Absolute Relative
Primary prevention of variceal bleeding Hepatoma, particularly if central
Severe congestive heart failure Obstruction of all hepatic veins
Tricuspid regurgitation Hepatic encephalopathy
Multiple hepatic cysts Significant portal vein thrombosis
Uncontrolled systemic infection or sepsis Severe uncorrectable coagulopathy (INR >5)
Unrelieved biliary obstruction Thrombocytopenia (<20,000 platelets/mm3)
Severe pulmonary hypertension Moderate pulmonary hypertension
• The main risk factors for developing HE include
A. age >65 years,
B. child score >12,
C. prior HE,
D. placement of a large diameter stent (>10 mm) and
E. low PPG (<5 mm Hg).
57. A 46-year-old motorcyclist was involved in a high-speed RTA. On arrival of the paramedics, the GCS was
recorded as 4/15 and the patient was intubated at the site of injury. An emergency noncontrast CT
showed multiple subtle petechial hemorrhages characteristic of diffuse axonal injury. What is the most
likely site of the petechial hemorrhage?
F. Insular ribbon
G. Watershed areas
H. Periventricular white matter
I. Grey-white matter junction
J. Cerebellum
Source: Proctor, Robin. Final FRCR Part A Modules 4-6 Single Best Answer MCQs: The SRT Collection
of 600 Questions with Explanatory Answers (MasterPass). 1st ed., CRC Press, 2009.
Explanation:
• Diffuse axonal injury (DAI) occurs in severe trauma as a result of shearing stress along the course of
the white matter tracts especially at the grey-white matter junction.
• The injury is usually microscopic and initial CTs are usually normal despite profound clinical
impairment.
• Acute DAI may also be seen as small petechial hemorrhages at the grey-white matter junction (67%),
internal/external capsule, corona radiata, corpus callosum (21%) and brainstem.
• MR features depend on the age of the hemorrhage. Prognosis is poor.
58. A combination of subependymal nodules, giant cell astrocytomas, white matter lesion and retinal
phakomatoses suggests:
A. Tuberous sclerosis
B. NFI
C. NF2
D. Sturge-Weber syndrome
E. Von Hippel-Lindau
EBDR Exam MCQs & Concepts October 2021
Dr. Kareem Alnakeeb
314
Source: Leen, Edward. Succeeding in the New FRCR Part 2A Exam: Single Best Answer (SBA)
Revision Questions for Modules 1-6. 1st ed., UNKNO, 2011.
Explanation:
• These are all features of tuberous sclerosis.
59. A 4-year-old boy is investigated via MRI brain for developmental delay and intractable seizures. Which of
the following findings is in keeping with a diagnosis of schizencephaly?
A. Intracerebral cleft lined by gray matter connecting the lateral ventricle to the subarachnoid
space.
B. Smooth cortical surface with absence of convolutions.
C. Multiple small, irregular cortical convolutions without intervening sulci.
D. Column of grey matter extending from the subependymal to the pial surface.
E. Circumferential, symmetric band of heterotopic grey matter deep to the cortical surface.
Source: Lindsay, Richard, et al. SBAs for the FRCR Part 2A. 1st ed., Oxford University Press, 2012.
Explanation:
• Schizencephaly can be defined as open or closed lip, depending on the presence of separation of the
cleft walls.
• The remaining options describe lissencephaly, polymicrogyria, transmantle heterotopia, and
subcortical band heterotopia, respectively.
• Transmantle heterotopia can potentially be confused with closed lip schizencephaly.
60. On brain MRI, which imaging finding is most characteristic of schizencephaly?
A. Cerebrospinal fluid–filled cleft extending from ventricular ependyma to the cortical surface and
lined by gray matter
B. Cleft extending from ventricle to cortex lined by white matter
C. Cystic cavity that does not communicate with the ventricular system
D. Porencephalic cyst wall producing a cortical dimple only
E. Isolated focal agenesis of the corpus callosum without cortical cleft
Explanation: (by Perplexity AI, generated with the OpenAI O3 model)
• Schizencephaly is a neuronal migration disorder defined by a full-thickness cleft traversing the
cerebral hemisphere, filled with CSF and crucially lined by heterotopic gray matter that is continuous
with cortex at both the ependymal and pial margins.
• This gray-matter lining distinguishes it from porencephalic cysts (choice C, D), which are lined by
gliotic white matter or ependyma.
• A white-matter-lined cleft (choice B) suggests acquired encephaloclastic defects, not
schizencephaly.
• Isolated callosal agenesis (choice E) can coexist but is not a defining feature.
EBDR Exam MCQs & Concepts October 2021
Dr. Kareem Alnakeeb
315
61. A 6-year-old girl presents to her family doctor with fever and pain in the lower left leg. Blood tests reveal
leukocytosis and anemia. Plain X-ray of the leg shows a destructive lesion involving the fibular shaft with
lamellated onion skin periosteal reaction, cortical destruction and large soft-tissue mass. What is the
likely diagnosis?
A. Osteosarcoma
B. Ewing’s sarcoma
C. Chondroblastoma
D. Chondromyxoid fibroma
E. Osteoid osteoma
Source: Chin, Teck Yew, et al. Get Through Final FRCR 2A: SBAs. 1st ed., CRC Press, 2017.
Explanation:
Differential Between Infections and Primary Bone Tumors
• The main differential in such findings would be between infection and a primary bone tumour.
Osteosarcoma vs. Ewing’s Sarcoma
• Because infection has not been given as an option, the choices would be between osteosarcoma
and Ewing’s sarcoma.
Distribution in the Skeleton
• Ewing’s sarcoma tends to occur in both the appendicular and axial skeleton equally, whereas
osteosarcoma mostly occurs in the appendicular skeleton.
Typical Site Within Long Bones
• Ewing’s sarcoma usually begins in the diaphysis of the long bones, whereas osteosarcoma tends to
occur in the metaphysis.
Extra-Osseous Component
• Ewing’s sarcoma may also have a large extra osseous component.
Imaging Overlap
• The two are often differentials of each other, as each can have a large overlap of imaging findings, but
the above traits can help one sway towards the other.
62. On MRI, which lesion most characteristically appears as a well-defined mass showing a central cavity
surrounded by a uniformly thick, smooth enhancing wall?
A. Abscess
B. Hydatid cyst
C. Mycetoma
D. Hemangioma
E. Aneurysmal bone cyst
Explanation: (by Perplexity AI, generated with the OpenAI O3 model)
• A pyogenic abscess typically forms a liquefied necrotic center that is encircled by granulation tissue
and a capsule, producing a smooth ring that is uniformly thick and avidly enhances after contrast
administration.
• Hydatid cysts display daughter cysts and calcification with a thin or laminated wall, not a uniformly
thick ring.
• Mycetoma shows the “dot-in-circle” sign—small T2 dark foci within a high-signal rim—rather than a
solid enhancing wall.
• Hemangiomas contain blood-filled vascular spaces giving a heterogenous, lobulated appearance
with peripheral discontinuous nodular enhancement.
• Aneurysmal bone cysts are expansile, septated lesions with fluid–fluid levels and thin cortical rims,
lacking a concentric enhancing capsule.
EBDR Exam MCQs & Concepts October 2021
Dr. Kareem Alnakeeb
316
63. Which of the following potential findings is NOT typically associated with an untreated abdominal aortic
aneurysm?
A. Intraluminal thrombosis
B. Catastrophic rupture
C. Endoleak
D. Erosion of adjacent vertebral bodies
E. Peripheral embolization
Explanation: (by Perplexity AI, generated with the OpenAI O3 model)
• Endoleak describes persistent blood flow within the aneurysm sac following endovascular
aneurysm repair; it is therefore absent in aneurysms that have never been treated.
• In contrast, intraluminal thrombus frequently lines large aneurysms and can shower emboli distally.
• Progressive sac expansion may compress and erode neighboring structures such as the lumbar
vertebral bodies. The most feared natural-history event is rupture, accounting for high mortality.
• Thus, all options except endoleak represent recognized complications of an untreated aneurysm.
64. What is the most sensitive sign on non-contrast CT for detecting early hydrocephalus?
A. cortical sulcal effacement
B. uncal herniation
C. enlarged third ventricle
D. enlarged fourth ventricle
E. enlarged temporal horns of the lateral ventricles
Source: Bydder, Megan, et al. Get Through Final FRCR Part A: SBAs for the Modular Examination. 1st
ed., Hodder Education Publishers, 2009.
Explanation:
• In many cases of hydrocephalus due to subarachnoid hemorrhage, the temporal horns of the lateral
ventricles become dilated sooner than the frontal horns.
• Dilatation of the temporal horn is often particularly conspicuous, as it is frequently not visualized at
all on CT of normal brains.
• Uncal herniation is herniation of the medial temporal lobe into a subtentorial location, where it may
exert pressure on the brain stem and is a late sign of raised intracranial pressure, often presenting
with oculomotor nerve palsy resulting in a fixed dilated pupil.
65. A preterm neonate born at 24 weeks is referred for a cranial ultrasound study. A focus of increased
echogenicity is present in the right caudothalamic groove extending into the lateral ventricle, without
ventricular dilatation. Which grade germinal matrix hemorrhage would this feature most represent?
A. Grade 1
B. Grade 2
C. Grade 3
D. Grade 4
E. Grade 5
Source: Chin, Teck Yew, et al. Get Through Final FRCR 2A: SBAs. 1st ed., CRC Press, 2017.
Explanation:
Periventricular leukomalacia (PVL)
• The most common location for injury to the premature brain is the periventricular white matter, with
ischemic parenchyma manifesting as periventricular leukomalacia (PVL).
EBDR Exam MCQs & Concepts October 2021
Dr. Kareem Alnakeeb
317
• Initial sonograms show hyperechogenic globular change in the periventricular regions, and MR
images depict areas of T1 hyperintensity within larger areas of T2 hyperintensity.
• Subsequent cavitation and periventricular cyst formation, features that are required for a definitive
diagnosis of PVL, develop 2–6 weeks after injury and are easily seen on sonograms as localized
anechoic or hypoechoic lesions.
• The progressive change ventricular dilatation occurs, described as end-stage PVL.
Germinal matrix hemorrhage
• Subsequent reperfusion to the ischemic tissues in the setting of weakened capillaries and increased
venous pressure result in germinal matrix hemorrhage, ranging in severity from subependymal
hemorrhage (Grade 1) to intraventricular hemorrhage without (Grade 2) and with (Grade 3) ventricular
dilatation, to parenchymal extension and coexisting periventricular venous infarction (Grade 4).
66. In systemic staging of primary lung carcinoma, which organ is the most frequent target of hematogenous
metastasis seen at diagnosis or autopsy studies?
A. Brain
B. Contralateral lung
C. Skeleton
D. Adrenal gland
E. Liver
Explanation: (by Perplexity AI, generated with the OpenAI O3 model)
• Autopsy series and large imaging datasets consistently show that blood-borne spread from lung
cancer most often deposits in the adrenal cortex, with reported involvement in 30–40% of cases.
• Although brain and bone metastases are also common, their incidence is lower.
• Contralateral pulmonary nodules usually represent intrapulmonary spread rather than true distant
hematogenous metastasis, and hepatic deposits rank below adrenal involvement in frequency.
• Recognizing the adrenal glands as the prime hematogenous “landing site” is critical because small,
silent adrenal metastases upstage the tumor to stage IV and alter management.
67. On CT imaging of a hepatic hydatid cyst, which finding corresponds to the “water-lily” sign?
A. Rupture of the pericyst into the biliary tree
B. Separation and collapse of the endocyst membrane within the cyst cavity
C. Presence of multiple daughter cysts producing a honeycomb appearance
D. Peripheral calcification of the cyst wall
E. Infection of the cyst with gas-forming organisms
Explanation: (by Perplexity AI, generated with the OpenAI O3 model)
• The water-lily sign appears when the inner germinal (endocyst) membrane detaches from the outer
pericyst and floats on the hydatid fluid, creating a crinkled membrane reminiscent of a lily pad. This
phenomenon follows partial rupture of the endocyst while the pericyst remains intact, making option
B correct.
• Rupture of the pericyst into the biliary system (option A) produces cyst–biliary communication rather
than floating membranes.
• Multiple daughter cysts (option C) give a honeycomb pattern, not a water-lily appearance.
• Peripheral calcification (option D) indicates an old, inactive cyst.
• Gas from super-infection (option E) leads to air–fluid levels, not the characteristic floating
membrane.
EBDR Exam MCQs & Concepts October 2021
Dr. Kareem Alnakeeb
318
68. In cardiac MRI, “white-blood” (bright-blood) sequences such as balanced SSFP are mainly used for
which of the following purposes?
A. Assessment of cardiac anatomy
B. Functional cardiac CINE imaging
C. Cardiac fat-suppressed imaging
D. All of the above
E. Late gadolinium enhancement of myocardial scar
Explanation: (by Perplexity AI, generated with the OpenAI O3 model)
• White-blood (bright-blood) techniques keep intravascular blood signal high, providing excellent
contrast between blood and myocardium. This makes them the workhorse for anatomical surveys
(multiplanar SSFP stacks) and for dynamic CINE studies that quantify ventricular function and valve
motion.
• Because the sequence is gradient-echo based, fat-saturation pulses can be added, enabling fat-
suppressed bright-blood runs when pericardial or epicardial fat needs definition.
• Late gadolinium enhancement, however, relies on an inversion-recovery “black-blood” style nulling
of normal myocardium rather than bright-blood imaging, so option E is not intrinsically a white-blood
technique. Thus, all the functions listed in options A–C are valid applications, making option D
correct.
69. Large thin-walled air-filled spaces >1 cm in diameter adjacent to the visceral pleura, known as bullae, are
most characteristic of which emphysematous subtype?
A. Paraseptal emphysema
B. Panacinar emphysema
C. Paracicatricial emphysema
D. Centrilobular emphysema
E. Compensatory emphysema
Explanation: (by Perplexity AI, generated with the OpenAI O3 model)
• Bullae arise from coalescence of distal acinar (paraseptal) emphysema, which predominately
affects alveoli bordering interlobular septa and pleural surfaces; as adjacent septa rupture, large
subpleural air spaces form, often in upper lobes.
• Panacinar emphysema diffusely involves the entire acinus and typically produces uniform lower-
lobe hyperlucency without discrete bullae.
• Centrilobular emphysema destroys respiratory bronchioles, giving multiple small central lucencies
rather than single large bullae.
• Paracicatricial (irregular) emphysema results from fibrotic scarring with distorted lung
architecture, not classic bullae formation.
• Compensatory emphysema represents over-inflation of normal lung after resection and lacks true
parenchymal destruction, so bullae are not a feature.
70. Which of the following lesions is NOT typically found in the anterior mediastinum on cross-sectional
imaging?
A. Thymoma
B. Teratoma
C. Lymphoma
D. Pulmonary artery aneurysm
E. Thyroid goitre
EBDR Exam MCQs & Concepts October 2021
Dr. Kareem Alnakeeb
319
Explanation: (by Perplexity AI, generated with the OpenAI O3 model)
• Thymomas, germ-cell tumours such as teratomas, and primary mediastinal (terrible) lymphomas are
classic “4 T” causes of an anterior mediastinal mass.
• A retrosternal extension of multinodular goitre is also a frequent anterior compartment lesion.
• In contrast, a pulmonary artery aneurysm originates within the pulmonary trunk or its branches,
placing it in the middle mediastinum rather than the anterior compartment; therefore, it will not
present as a true anterior mediastinal mass.
71. On thyroid ultrasound, which finding most reliably favors Graves’ disease rather than Hashimoto
thyroiditis in a diffusely enlarged gland?
A. Diffusely increased color Doppler flow giving a “thyroid inferno” appearance
B. Multiple tiny hypoechoic micronodules separated by echogenic fibrous septa (“giraffe-skin” pattern)
C. Heterogeneous hypoechoic parenchyma with overall reduced vascularity
D. Pseudonodular enlargement with coarse echotexture and scattered hyperechoic fibrotic bands
E. Elevated serum thyroid-stimulating hormone (TSH) level
Explanation: (by Perplexity AI, generated with the OpenAI O3 model)
• Graves’ disease causes marked hyperemia from thyroid-stimulating immunoglobulins; Doppler
shows striking, diffuse flow (“thyroid inferno”), a classic clue distinguishing it from Hashimoto.
• Hashimoto typically displays a micronodular, septated “giraffe-skin” pattern with vascularity that is
normal or reduced once fibrosis develops, making option B wrong.
• Option C describes the late fibrotic phase of Hashimoto, not Graves.
• Pseudonodular coarse echotexture (option D) is again characteristic of chronic Hashimoto.
• Graves’ disease suppresses TSH through negative feedback, so an elevated TSH (option E) is
incompatible.
72. On thoracic ultrasound for suspected pleural infection, which sonographic feature combination is most
characteristic of a pleural empyema?
A. Echogenic foci that move with respiration
B. Multiple thin septations within the effusion
C. Both A and B
D. A discrete pleural nodule
E. Anechoic, free-flowing pleural fluid
Explanation: (by Perplexity AI, generated with the OpenAI O3 model)
• Empyemas typically appear as complex, “organized” pleural collections.
• Mobile echogenic particles represent debris such as pus; their movement with breathing
differentiates them from fixed pleural masses.
• Additionally, fibrinous strands form multiple thin septations, giving a loculated “honeycomb”
appearance. The simultaneous presence of moving echogenic debris and numerous septa strongly
favors empyema over other effusions (Option C).
• A solitary pleural nodule (Option D) suggests tumor seeding, not infection, while a purely anechoic,
freely mobile effusion (Option E) is typical of uncomplicated transudates.
• Either feature alone (Options A or B) can occur in complicated parapneumonic effusions but is less
specific than their combination.
EBDR Exam MCQs & Concepts October 2021
Dr. Kareem Alnakeeb
320
73. On chest CT, a “halo sign”—ground-glass opacity surrounding a pulmonary nodule or mass—is most
commonly associated with which of the following categories of disease?
A. Malignant lung lesions
B. Fungal infections
C. Other aggressive/hemorrhagic lesions (e.g., vasculitis, septic emboli)
D. All of the above
Explanation: (by Perplexity AI, generated with the OpenAI O3 model)
The CT halo sign—ground-glass opacity surrounding a pulmonary nodule or mass—is a non-specific but
ominous finding that appears in several settings:
• Invasive pulmonary aspergillosis, mucormycosis, candidiasis —especially in neutropenic or
otherwise immunocompromised patients
• Lepidic-predominant (bronchioloalveolar) adenocarcinoma, hemorrhagic metastases, Kaposi
sarcoma, angiosarcoma, lymphoma
• Granulomatosis with polyangiitis, septic emboli, organizing pneumonia or other rapidly progressive
inflammatory processes that can cause alveolar hemorrhage
Because the halo sign can arise from fungal, malignant, and other aggressive pathologies, the most
inclusive answer is D.
74. In the ACR TI-RADS (2017) ultrasound scoring system for thyroid nodules, which one of the
following features is NOT included in the points-based assessment?
A. Composition
B. Echogenicity
C. Shape (taller-than-wide)
D. Margin
E. Vascularity
Explanation: (by Perplexity AI, generated with the OpenAI O3 model)
• ACR TI-RADS assigns points for five specific sonographic categories—composition, echogenicity,
shape, margin and echogenic foci. These parameters collectively yield a total score that determines
the TI-RADS level and follow-up guidance.
• Doppler vascularity, although often described in reports, is not part of the formal scoring matrix and
therefore carries no points.
• By contrast, composition (solid vs cystic), echogenicity (hypoechoic, isoechoic, etc.), shape (taller-
than-wide), and margin characteristics all directly influence the calculated risk category, so omission
of vascularity makes option E the only incorrect choice.
75. Chest radiograph of a 12-year-old boy shows a cystic lesion with air–fluid level in the right upper lobe. CT
scan confirms the presence of a thin-walled cystic lesion. Rest of the lungs are clear. There is no
lymphadenopathy in the chest. Quantiferon test was negative, and there are no features of infection or
signs of inflammation. What is the diagnosis?
A. TB
B. Intrapulmonary bronchogenic cyst
C. Hydatid cyst
D. Infected bulla
E. Congenital lobar emphysema
EBDR Exam MCQs & Concepts October 2021
Dr. Kareem Alnakeeb
321
Source: Chin, Teck Yew, et al. Get Through Final FRCR 2A: SBAs. 1st ed., CRC Press, 2017.
Explanation:
Definition
• Bronchogenic cysts (BCs) are congenital lesions.
Typical Locations
• They are usually found in the mediastinum or pulmonary parenchyma and, less commonly, cysts
may be found in the neck, pericardium, pleura, diaphragm or abdominal cavity.
Intrapulmonary Prevalence
• Intrapulmonary cysts are most common in the lower lobes.
Radiologic Characteristics
• Intrapulmonary BCs are usually sharply defined, solitary, non-calcified, round or oval opacities
confined to a single lobe.
• These can present as a homogeneous water density, an air-filled cyst, or with an air–fluid level.
• Signal on MRI depends on the content, and fluid-containing lesions are low on T1-weighted and high
on T2-weighted images; however, proteinaceous content makes them high on T1-weighted imaging.
Differential Diagnosis
• The differential diagnosis of intraparenchymal BCs must include acquired cystic lesions, such as a
lung abscess, a hydatid cyst, infection with Nocardia, an infected bulla, congenital lobar
emphysema, fungal diseases and tuberculosis, especially when the lesions manifest as air-filled or
have an air–fluid level.
76. Regarding safe insertion and management of a small-bore pigtail pleural drain, which single statement is
most appropriate?
A. Introduce the catheter just above the upper edge of the rib to avoid the intercostal
neurovascular bundle.
B. A chest radiograph is unnecessary after ultrasound-guided insertion.
C. Both A and B are correct.
D. Large effusions should be drained as rapidly as possible to relieve breathlessness.
E. Prophylactic antibiotics are mandatory before all pigtail drain insertions.
Explanation: (by Perplexity AI, generated with the OpenAI O3 model)
• The intercostal artery, vein and nerve run in the costal groove along the inferior border of each rib;
puncturing the pleura immediately above the rib minimizes the risk of vascular or nerve injury,
making option A correct.
• A post-procedure chest radiograph is usually obtained to confirm drain position and detect
complications, so stating it is “unnecessary” (option B) is inaccurate.
• Option C cannot be right because B is incorrect. Rapid (uncontrolled) evacuation of a large effusion
risks re-expansion pulmonary oedema; current guidance recommends limiting initial drainage to
~1.5 L over the first hour, so option D is wrong.
• Routine prophylactic antibiotics are advised for traumatic tube thoracostomy but are not required for
elective pigtail drains, rendering option E incorrect.
EBDR Exam MCQs & Concepts October 2021
Dr. Kareem Alnakeeb
322
77. A 56-year-old female was referred to the cardiology outpatient clinic with recent onset exertional
dyspnea. An echocardiogram showed left ventricular dysfunction and a cardiac MRI was requested to
identify the cause. Cine images revealed focal hypokinesis in the anteroseptal wall and delayed
enhanced images show increased signal in the subendocardium. What is the most likely diagnosis?
A. Myocarditis
B. Myocardial infarction
C. Hypertrophic cardiomyopathy
D. Amyloidosis
E. Tako-tsubo cardiomyopathy
Source: Proctor, Robin. Final FRCR Part A Modules 1–3 Single Best Answer MCQs: The SRT Collection
of 600 Questions with Explanatory Answers (MasterPass). 1st ed., CRC Press, 2009.
Explanation:
• Purely subendocardial delayed hyperenhancement in a recognized vascular territory is classical of
myocardial infarction. The anteroseptal wall is supplied by the left anterior descending artery.
78. A 67 year old retired decorator presents to his GP following multiple episodes of chest pain during the
past 2 weeks brought on by exertion. Past medical history includes hypertension and gallstones. The GP
refers him to the rapid access chest pain clinic. As part of the investigations the patient has a cardiac MRI
which demonstrates increased T2 weighted signal intensity in the mid anterior and septal walls, with
delayed subendocardial hyperenhancement. What is the most likely diagnosis?
A. Acute myocarditis
B. Hibernating myocardium
C. Myocardial infarct involving the left anterior descending artery
D. Myocardial infarct involving the right coronary artery
E. Myocardial stunning
Source: Rabone, Amanda, et al. The Final FRCR Self-Assessment (MasterPass). 1st ed., CRC Press,
2020.
Explanation:
LAD territory infarction
• The left anterior descending artery and its branches supply the anterolateral and apical walls of the
left ventricle and the interventricular septum.
• The high T2 weighted signal in this region is secondary to oedema suggesting a relatively acute insult,
and the delayed hyperenhancement is typical in infarcted myocardium and tends to be
subendocardial or full thickness.
Myocardial stunning and hibernation
• Myocardial stunning and hibernation often have similar imaging findings.
• Stunning is caused following a transient period of ischemia, whereas hibernation is thought to be
related to more chronic ischemia where the myocardial cells adapt to reduced perfusion by
hibernating and reducing metabolic activity.
• Both these conditions lead to impaired function, manifesting as reduced contractility.
• Stunned myocardium tends to have preserved perfusion whereas it can be reduced in myocardial
hibernation.
Acute myocarditis
• Acute myocarditis may demonstrate increased myocardial T2 weighted signal.
• However, other findings such as a focal area of wall motion abnormality would also be expected.
• Enhancement in myocarditis tends to involve the epicardium and be early rather than the delayed
subendocardial enhancement described in this case.
EBDR Exam MCQs & Concepts October 2021
Dr. Kareem Alnakeeb
323
79. On an erect chest radiograph, which finding is most characteristic of a tension pneumothorax?
A. Visible visceral pleural edge with absent lung markings distal to it
B. Ipsilateral hyper-lucent (transradiant) hemithorax
C. Mediastinal shift toward the opposite hemithorax
D. Widespread subcutaneous emphysema
E. Elevated ipsilateral hemidiaphragm
Explanation: (by Perplexity AI, generated with the OpenAI O3 model)
• Tension pneumothorax occurs when intrapleural pressure rises above atmospheric pressure
throughout the respiratory cycle, forcing air to accumulate and displacing the mediastinum away
from the affected side.
• This mediastinal shift is the key radiographic sign and correlates with impending cardiovascular
collapse, making option C correct.
• Options A and B indicate a pneumothorax but can also be seen in simple (non-tension) cases, so
they lack specificity.
• Subcutaneous emphysema (D) may accompany chest trauma but is neither sensitive nor specific for
tension physiology.
• The diaphragm on the affected side usually flattens or depresses rather than elevates; therefore,
option E is incorrect.
80. Neck US of a previously well 2-year-old girl shows a 3-cm thin-walled cystic structure with multiple
septae of variable thickness in the left posterior triangle with extension into the mediastinum. The
diagnosis is:
A. Third branchial cleft cyst
B. Cervical meningocele
C. Cystic teratoma
D. Lymphangioma
E. Second branchial cleft cyst
Source: Chin, Teck Yew, et al. Get Through Final FRCR 2A: SBAs. 1st ed., CRC Press, 2017.
Explanation:
Cystic Hygroma
• A cystic hygroma is the most common form of lymphangioma and constitutes about 5% of all benign
tumours of infancy and childhood.
• On US scans, most cystic hygromas manifest as a multilocular predominantly cystic mass with septa
of variable thickness.
• The echogenic portions of the lesion correlate with clusters of small, abnormal lymphatic channels.
• Fluid–fluid levels can be observed with a characteristic echogenic, haemorrhagic component
layering in the dependent portion of the lesion.
• Prenatal US may demonstrate a cystic hygroma in the posterior neck soft tissues.
• On CT images, cystic hygromas tend to appear as poorly circumscribed, multiloculated,
hypoattenuated masses.
• They typically have characteristic homogeneous fluid attenuation.
• Usually, the mass is centred in the posterior triangle or in the submandibular space.
Branchial Cleft Cysts
Third Branchial Cleft Cyst
EBDR Exam MCQs & Concepts October 2021
Dr. Kareem Alnakeeb
324
• A third branchial cleft cyst most commonly appears as a unilocular cystic mass centred in the
posterior cervical space on CT and MRI.
Second Branchial Cleft Cyst
• At US, a second branchial cleft cyst is seen as a sharply marginated, round to ovoid, centrally
anechoic mass with a thin peripheral wall that displaces the surrounding soft tissues.
• The ‘classic’ location of these cysts is at the anteromedial border of the sternocleidomastoid muscle.
First Branchial Cleft Cyst
• The first branchial cleft cyst appears as a cystic mass either within, superficial to, or deep to the
parotid gland.
81. Plain radiograph of a 9-month-old baby girl shows a large soft-tissue mass in the pelvis with punctate
calcification. MRI reveals a large, lobulated, sharply demarcated tumour with extremely heterogeneous
signal on T1W images. What is the diagnosis?
A. Anterior sacral meningocoele
B. Sacrococcygeal teratoma
C. Caudal regression syndrome
D. Rhabdomyosarcoma
E. Rectal duplication
Source: Chin, Teck Yew, et al. Get Through Final FRCR 2A: SBAs. 1st ed., CRC Press, 2017.
Explanation:
Sacrococcygeal Teratoma
• Sacrococcygeal teratoma is the most common presacral germ cell tumour in children and the most
common solid tumour in neonates.
Benign Sacrococcygeal Teratoma
• The benign form accounts for 60% of all sacrococcygeal teratomas.
• Benign teratomas are predominantly cystic; have attenuation similar to fluid on CT; and may include
bone, fat and calcification.
• Cystic areas appear low on T1-weighted and high on T2-weighted MRI.
• Fatty tissue demonstrates high signal intensity on T1-weighted images, whereas calcification is
depicted as a signal void.
• The coccyx is always involved, even in benign sacrococcygeal teratoma, and must be resected with
the tumour.
• Approximately 50% of benign teratomas contain calcification, whereas it is seldom seen in malignant
tumours.
Malignant Sacrococcygeal Teratoma
• Malignant teratomas are more solid, and haemorrhage and necrosis are common.
• Malignant teratomas may metastasise.
Anterior Sacral Meningocoele
• Anterior sacral meningocoele is a congenital abnormality that arises from herniation of the CSF-filled
dura mater through a sacral foramen or a defect in the sacral bone.
• Eccentric defect in sacrum results in a scimitar appearance on plain film.
EBDR Exam MCQs & Concepts October 2021
Dr. Kareem Alnakeeb
325
82. A 62-year-old man with known hepatocellular carcinoma on a background of long-standing liver cirrhosis
is scheduled to have a TACE procedure. Which one of the following is an absolute contraindication to
TACE therapy for hepatocellular carcinoma in a cirrhotic patient?
A. Contrast medium allergy
B. Replacement of 25% of the liver by the tumour
C. Total bilirubin greater than 2 mg/dL
D. Biliary tree obstruction
E. Child–Pugh Class C cirrhosis
Source: Chin, Teck Yew, et al. Get Through Final FRCR 2A: SBAs. 1st ed., CRC Press, 2017.
Explanation:
Absolute and relative contraindications for conventional TACE in patients with HCC are as follows.
Absolute contraindications: Relative contraindications:
1. Decompensated cirrhosis (Childs–Pugh C or higher) 1. Tumour size >10 cm.
2. Jaundice 2. Co-morbidities involving compromised organ
function such as cardiovascular and lung disease.
3. Clinical encephalopathy 3. Untreated varices present a high risk of bleeding.
4. Refractory ascites 4. Bile duct occlusion or incompetent papilla due to
stent or surgery.
5. Extensive tumour with massive replacement of both
lobes
6. Severely reduced portal vein flow
7. Technical contraindications to hepatic intra-arterial
treatment
8. Renal insufficiency (creatinine clearance <30 mL/min)
83. Which coronary branch, arising from the left circumflex artery and running along the left ventricular free
wall toward the apex, is most commonly termed the obtuse marginal artery?
A. Diagonal branch of the left anterior descending artery
B. Posterior descending artery
C. Left marginal (lateral) branch of the circumflex artery
D. Acute marginal branch of the right coronary artery
E. Conus branch of the right coronary artery
Explanation: (by Perplexity AI, generated with the OpenAI O3 model)
• The obtuse marginal artery (OM) is the principal lateral branch of the left circumflex (LCx) artery; it
travels along the obtuse (left) margin of the heart toward the apex and supplies the lateral wall of the
left ventricle, making option C correct.
• The diagonal branches (A) originate from the left anterior descending artery, not the LCx.
• The posterior descending artery (B) usually arises from the right coronary or, in left-dominant hearts,
the LCx, but it supplies the inferior interventricular septum, not the obtuse margin.
• The acute marginal branch (D) is a right coronary artery offshoot that courses along the acute (right)
margin.
• The conus branch (E) is an early right coronary branch supplying the right ventricular outflow tract,
unrelated to the left circumflex system.
EBDR Exam MCQs & Concepts October 2021
Dr. Kareem Alnakeeb
326
84. Which of the following are True regarding laryngoceles? (True or False)
A. They typically arise in the vestibule
B. They are bilateral in less than 5% of cast’s.
C. They may appear as hypodense masses on unenhanced CT
D. Carcinoma is a recognized complication
E. They rarely extend beyond the thyrohyoid membrane
Source: Bell, J., and N. Davies. MCQs in Clinical Radiology: A Revision Guide for the FRCR. 1st ed.,
Remedica Pub Ltd, 2004.
Explanation:
A. False - They arise at the saccule of the ventricle, secondary to obstruction of the laryngeal ventricles.
Occasionally this is due to cancer.
B. False - they are bilateral in 25% of cases.
C. True - They may contain air or fluid.
D. True - Carcinoma occurs in less than 1 % of cases. Other complications include infection or
mucocoele formation.
E. False - In relation to the thyrohyoid membrane, 30% are internal, 26% are external and 44% are
mixed.
85. A 1-year-old infant is admitted with acute stridor. A viral cause is suspected. On AP chest radiography no
foreign body is identified, but there is an inverted V appearance of the subglottic trachea. Which of the
following is the most likely diagnosis?
A. Foreign body
B. Acute laryngotracheobronchitis
C. Whooping cough
D. Tracheobronchomalacia
E. Epiglottitis
Source: Chin, Teck Yew, et al. Get Through Final FRCR 2A: SBAs. 1st ed., CRC Press, 2017.
Explanation:
Croup
• Croup (laryngotracheobronchitis) most commonly affects children between 6 months and 3 years
and presents with acute stridor, usually following viral infection.
• A subglottic inverted V sign is seen on plain film, but the epiglottis and aryepiglottic folds are usually
normal.
Epiglottitis
• In contrast, epiglottitis is a life-threatening condition affecting 3–6-year-olds, with a lateral soft-
tissue neck radiograph showing thickening of the epiglottis and aryepiglottic folds described as the
‘thumb sign’.
86. A 20-year-old man presents with swelling around his left eye. A CT scan shows a high attenuation mass
lesion which expands the ethmoid air cells with bony erosion. There are small punctate calcifications
seen within the mass. On MRI, the mass returns low signal on T1 and T2. The most likely diagnosis is?
A. Fungal sinusitis
B. Chronic sinonasal polyposis
C. Nasopharyngeal carcinoma
D. Juvenile angiofibroma
E. Chronic sinusitis
EBDR Exam MCQs & Concepts October 2021
Dr. Kareem Alnakeeb
327
Source: Gupta, Chaitanya. 300 Single Best Answers for the Final FRCR Part A. 1st ed., Jaypee UK,
2010.
Explanation:
• CT findings are typical, showing a hyperdense lesion with calcifications and bony erosion. The
ethmoid sinus is most commonly involved and bony expansion with erosion is characteristic.
• On MRI, the lesion is low signal on T1 and T2 due to high fungal mycelial iron, magnesium and
manganese from amino acid metabolism
87. A right-sided aortic arch with mirror-image branching is most frequently associated with which congenital
cardiac abnormality?
A. pulmonary atresia and ventricular septal defect
B. truncus arteriosus
C. uncomplicated ventricular septal defect
D. Fallot’s tetralogy
E. corrected transposition of the great vessels
Source: Bydder, Megan, et al. Get Through Final FRCR Part A: SBAs for the Modular Examination. 1st
ed., Hodder Education Publishers, 2009.
Explanation:
• There is a 98% incidence of associated congenital heart disease with a right-sided aortic arch with
mirror-image branching. Nearly all of these cases will be tetralogy of Fallot.
• All of the given options are associated with right-sided aortic arch, as well as dextrocardia with situs
inversus and double-outlet right ventricle.
• Right-sided aortic arch with left subclavian artery is associated with only a 12% incidence of
congenital heart disease, again with Fallot’s tetralogy being the most commonly associated
abnormality.
88. A newborn undergoes a chest radiograph which shows a right sided aortic arch. What underlying
condition are they most likely to have?
A. Ebstein anomaly
B. Tetralogy of Fallot
C. Transposition of the great arteries
D. Tricuspid atresia
E. Truncus arteriosus
Source: Rabone, Amanda, et al. The Final FRCR Self-Assessment (MasterPass). 1st ed., CRC Press,
2020.
Explanation:
• A right-sided aortic arch with mirror imaging branching is the most common subtype of a right-sided
aortic arch and is nearly always associated with congenital heart disease. Of these, 900/o are
associated with tetralogy of Fallot. Therefore other radiological findings to look for are a `boot-
shaped' heart and pulmonary oligemia.
• A right-sided aortic arch with an aberrant left subclavian artery is the second most common subtype,
and the persistent ductus ligament can cause tracheal compression. A Kommerell diverticulum is
also a feature, which manifests as dilatation of the aberrant left subclavian artery at the right aortic
arch origin.
EBDR Exam MCQs & Concepts October 2021
Dr. Kareem Alnakeeb
328
89. In persistent left-sided superior vena cava, drainage usually occurs into which structure?
A. left atrium
B. right atrium
C. normal right superior vena cava
D. hemiazygos vein
E. coronary sinus
Source: Bydder, Megan, et al. Get Through Final FRCR Part A: SBAs for the Modular Examination. 1st
ed., Hodder Education Publishers, 2009.
Explanation:
• Persistent left-sided superior vena cava occurs in 0.3% of the general population and in 4.3–11% of
patients with congenital heart disease.
• It is associated with atrial septal defects and azygos continuation of the inferior vena cava.
• It lies lateral to the aortic arch and anterior to the left hilum. It usually drains into the coronary sinus,
but rarely drains into the left atrium, causing a left-to-right shunt.
• The normal right-sided superior vena cava is absent in 10–18% of cases of left-sided superior vena
cava.
EBDR Exam MCQs & Concepts October 2021
Dr. Kareem Alnakeeb
329
Paper 2
1. A lateral thoracolumbar radiograph of a 45-year-old patient on long-term haemodialysis shows broad
bands of sclerosis along the superior and inferior vertebral endplates with a relatively lucent centre,
producing the classic “rugger-jersey spine” appearance. Which underlying disorder most commonly
causes this finding?
A. Hunter syndrome
B. Secondary hyperparathyroidism
C. Ankylosing spondylitis
D. Osteomalacia
E. Paget disease of bone
Explanation: (by Perplexity AI, generated with the OpenAI O3 model)
• The “rugger-jersey” spine sign is virtually pathognomonic for renal osteodystrophy attributable
to secondary hyperparathyroidism.
• Chronic kidney disease leads to phosphate retention and hypocalcaemia, stimulating parathyroid
hormone release; the resulting sub-periosteal resorption and reactive endplate sclerosis generate
the alternating dense-lucent-dense pattern.
• Hunter syndrome produces dysostosis multiplex but not endplate sclerosis.
• Ankylosing spondylitis shows vertebral squaring and marginal syndesmophytes, not alternating
bands.
• Osteomalacia causes generalized osteopenia and Looser zones, while Paget disease gives cortical
thickening and “picture-frame” vertebrae rather than the horizontal sclerotic bands of a rugger-jersey
spine.
2. In infants with suspected developmental dysplasia of the hip (DDH), MRI is useful for all of the following
purposes except:
A. Demonstrate an inverted acetabular labrum
B. Identify a hypertrophic pulvinar
C. Detect early avascular necrosis of the femoral head
D. Assess the integrity of Shenton line
E. Visualize the cartilaginous acetabular roof morphology
Explanation: (by Perplexity AI, generated with the OpenAI O3 model)
• MRI provides high-contrast visualization of non-ossified structures around the infant hip. It clearly
shows an inverted labrum, enlarged pulvinar tissue and early marrow signal changes of avascular
necrosis before they become apparent on radiographs, and it also delineates the cartilaginous
acetabular roof.
• Shenton line, however, is a purely radiographic arc drawn on an anteroposterior pelvic X-ray to judge
femoral head displacement; it cannot be assessed on cross-sectional MRI images, making option D
the exception.
• Options A, B, C and E all exploit MRI’s soft-tissue and cartilage sensitivity, thereby representing
appropriate indications.
EBDR Exam MCQs & Concepts October 2021
Dr. Kareem Alnakeeb
330
3. A 10-year-old schoolboy had a fall and bruised his right knee badly. There was an open wound that was
not treated until the following day. After a further two days he became systemically unwell with a fever.
His knee was extremely tender, swollen, and the movement was restricted. Which of the following is a
feature of septic arthritis?
A. Usually due to Hemophilus
B. Periarticular, soft tissue swelling is rare
C. Blurring of the periarticular fat planes is common
D. The joint space widens after a few weeks
E. A joint effusion is not usually present
Source: Proctor, Robin. Final FRCR Part A Modules 1–3 Single Best Answer MCQs: The SRT Collection
of 600 Questions with Explanatory Answers (MasterPass). 1st ed., CRC Press, 2009.
Explanation:
• Septic arthritis usually occurs in hip, knee, shoulder, elbow and ankle.
• Staphylococcus aureus, followed by group A Streptococcus, are the most common causes.
• Other radiographic features include periarticular soft-tissue swelling, an effusion, periarticular
osteopenia and, later, joint space narrowing.
• Ultrasound may help identify septic arthritis before cartilage lysis occurs.
• The hallmark is joint effusion in a patient with signs of a joint infection.
4. On musculoskeletal ultrasound of a painful, swollen knee suspected of septic arthritis, which
sonographic finding is most frequently seen?
A. Joint effusion
B. Thickened synovial capsule
C. Echogenic intra-articular debris
D. Hyperemia on power Doppler within the synovium
E. Cartilage surface irregularity
Explanation: (by Perplexity AI, generated with the OpenAI O3 model)
• Joint effusion is the earliest and most common ultrasound manifestation of septic arthritis because
bacterial inflammation rapidly drives fluid accumulation within the affected joint.
• Detecting and aspirating this fluid is critical both for diagnosis and for guiding antimicrobial therapy.
• Thickening of the synovial capsule and power Doppler hyperemia are markers of synovitis; they occur
variably and may be absent early in infection.
• Echogenic debris represents pus or fibrin strands but is not present in every effusion and therefore is
less reliable than simply identifying excess fluid.
• Cartilage irregularity is a late feature reflecting destructive change and, when present, suggests
delayed presentation rather than a typical initial finding.
5. Which skeletal dysplasia is typically lethal in the perinatal period?
A. Thanatophoric dysplasia
B. Achondroplasia
C. Cleidocranial dysplasia
D. Fibrous dysplasia
E. Spondyloepiphyseal dysplasia congenita
EBDR Exam MCQs & Concepts October 2021
Dr. Kareem Alnakeeb
331
Explanation: (by Perplexity AI, generated with the OpenAI O3 model)
• Thanatophoric dysplasia results from FGFR3 mutations causing marked micromelia, narrow bell-
shaped thorax and severe pulmonary hypoplasia, leading to stillbirth or death within hours of birth;
hence it is considered uniformly lethal.
• Achondroplasia produces rhizomelic short stature but normal life expectancy, while cleidocranial
dysplasia and fibrous dysplasia are usually compatible with long, near-normal lives.
• Spondyloepiphyseal dysplasia congenita causes short-trunk dwarfism and cervical spine instability,
yet most affected individuals survive into adulthood.
• Only thanatophoric dysplasia consistently leads to perinatal lethality, making it the correct choice.
6. An adult presents after a fall onto the outstretched hand; elbow radiographs show an isolated joint
effusion with no obvious cortical break. Which occult fracture is most commonly responsible for this
finding?
A. Coronoid process fracture
B. Medial humeral epicondyle fracture
C. Lateral humeral condyle fracture
D. Radial head fracture
E. Olecranon fracture
Explanation: (by Perplexity AI, generated with the OpenAI O3 model)
• Intracapsular elbow effusion (positive fat-pad sign) indicates hemarthrosis.
• In adults, axial load through the radius during a fall frequently causes an undisplaced radial head
fracture. These fractures are often hairline and intra-articular, so they may be radiographically occult
while still producing a hemarthrosis detectable as a joint effusion.
• Coronoid fractures are rare and usually accompany elbow dislocations.
• Medial or lateral condylar injuries are classically paediatric.
• Olecranon fractures lie largely extra-articular; an isolated effusion without visible cortical disruption
would therefore be unusual for this injury. Hence an occult radial head fracture is the most likely
diagnosis.
7. Regarding the Ficat-Arlet staging of avascular necrosis of the femoral head, which statement is correct?
A. Stage I disease shows bone marrow oedema on MRI with a normal plain radiograph.
B. Stage II disease demonstrates acetabular cartilage loss on pelvic radiograph.
C. Stage III disease shows a serpiginous double-line sign on T2-weighted MRI with an intact head
contour.
D. Stage III disease is defined by joint-space narrowing and complete collapse of the femoral head.
E. Stage IV disease shows a subchondral crescent sign without secondary osteoarthritis.
Explanation: (by Perplexity AI, generated with the OpenAI O3 model)
• Stage I AVN is radiographically occult; MRI or bone scan reveals early marrow changes such as
oedema and the characteristic double-line sign, hence option A is correct.
• Stage II presents with sclerosis or cysts but no collapse, and acetabular involvement (option B) is
absent until Stage IV.
• Although the double-line sign can appear early, an intact head contour with that sign does not define
Stage III (option C).
• Stage III is marked by a subchondral crescent sign and early collapse, but significant joint-space
narrowing signals Stage IV, making option D incorrect.
EBDR Exam MCQs & Concepts October 2021
Dr. Kareem Alnakeeb
332
• Option E is wrong because secondary osteoarthritis, including joint-space loss, is integral to Stage IV
disease.
8. A 14-year-old boy presents with night pain relieved by NSAIDs. Radiographs show a small, round lucency
with central mineralization completely surrounded by thickened cortex in the mid-tibia cortex. What is
the most likely diagnosis?
A. Osteoblastoma
B. Osteoid osteoma
C. Osteochondroma
D. Non-ossifying fibroma
E. Ewing sarcoma
Explanation: (by Perplexity AI, generated with the OpenAI O3 model)
• Osteoid osteoma classically arises within cortical bone, appearing as a nidus <1.5 cm surrounded
by reactive sclerotic cortex; pain is characteristically relieved by NSAIDs, reflecting prostaglandin
production.
• Osteoblastoma can look similar but is usually >2 cm, less painful at night, and often occurs in the
spine.
• Osteochondroma is a medullary-continuity exostosis projecting from the bone surface, not a purely
cortical intracortical lesion.
• Non-ossifying fibroma is a metaphyseal, cortically based but eccentric lytic defect with a sclerotic
rim, not a tiny nidus with surrounding sclerosis.
• Ewing sarcoma is an aggressive permeative diaphyseal tumor with soft-tissue mass and systemic
symptoms, not a benign cortical nidus.
9. Which underlying condition is classically responsible for the cavernous transformation (portal
cavernoma) seen on Doppler ultrasound or contrast-enhanced CT of the porta hepatis?
A. Acute portal vein thrombosis
B. Chronic portal vein thrombosis
C. Budd–Chiari syndrome
D. Hepatobiliary fistula
E. Hepatocellular carcinoma
Explanation: (by Perplexity AI, generated with the OpenAI O3 model)
• Cavernous transformation represents a network of peri-portal collateral veins that slowly develop to
bypass an obstructed main portal vein. This collateralization takes weeks to months, so it is virtually
pathognomonic for chronic (long-standing) portal vein thrombosis.
• In acute thrombosis (A), the thrombus is still fresh and the portal vein appears dilated rather than
replaced by collaterals.
• Budd–Chiari syndrome (C) affects hepatic veins, not the portal vein, so cavernoma is absent.
• A hepatobiliary fistula (D) is an abnormal communication between bile ducts and adjacent
structures and has no relationship with portal venous flow.
• Hepatocellular carcinoma (E) may invade the portal vein but produces tumour thrombus, not a
cavernous network.
EBDR Exam MCQs & Concepts October 2021
Dr. Kareem Alnakeeb
333
10. A 55-year-old patient undergoes a barium meal that shows a diffusely narrowed, rigid stomach with loss
of peristalsis, producing the classic “leather-bottle” (linitis plastica) appearance. Which underlying lesion
most commonly causes this radiological pattern?
A. Gastrointestinal stromal tumour (GIST)
B. Hyperplastic gastric polyp
C. Peutz-Jeghers hamartomatous polyposis
D. Gastric phytobezoar
E. Diffuse infiltrative (scirrhous) adenocarcinoma
Explanation: (by Perplexity AI, generated with the OpenAI O3 model)
• Linitis plastica results from diffuse submucosal infiltration that stiffens the gastric wall; the
archetypal cause is scirrhous (diffuse) adenocarcinoma, often producing a “leather-bottle”
stomach.
• GISTs grow as focal submucosal masses that typically expand outward rather than circumferentially.
• Hyperplastic and Peutz-Jeghers polyps create discrete intraluminal filling defects without
transmural infiltration.
• Bezoars are intraluminal conglomerates of indigestible material; they cast a mottled filling defect but
do not alter mural pliability. Thus, only diffuse adenocarcinoma explains the uniform rigid narrowing
that defines linitis plastica.
11. Which of the following statements concerning congestive cardiomegaly on an erect chest radiograph is
correct?
A. The portal vein is characteristically dilated.
B. A right-sided pleural effusion is a common associated finding.
C. The condition is unrelated to left-sided heart failure.
D. Kerley B lines are typically absent.
E. All of the above.
Explanation: (by Perplexity AI, generated with the OpenAI O3 model)
• Congestive cardiomegaly reflects elevated left atrial and pulmonary venous pressures; serous fluid
often tracks into the pleural space, classically producing a right-sided or bilateral effusion, so option
B is true.
• Portal vein calibre is usually unaffected—hepatic and inferior vena cava dilation predominate—
rendering option A incorrect.
• Because the process is a direct consequence of left-sided (and frequently global) heart failure,
option C is false.
• Interstitial oedema from pulmonary venous hypertension gives rise to Kerley B lines; their absence
would be atypical, so option D is wrong.
• As only one statement is correct, option E (“all of the above”) is also incorrect.
12. In a supine polytrauma patient undergoing a focused assessment with sonography for trauma (FAST),
which intraperitoneal recess is most commonly the first site where free fluid accumulates?
A. Paracolic gutter
B. Pouch of Douglas
C. Morrison’s pouch (hepatorenal recess)
D. Vesicouterine pouch (anterior to the uterus)
E. Splenorenal recess
EBDR Exam MCQs & Concepts October 2021
Dr. Kareem Alnakeeb
334
Explanation: (by Perplexity AI, generated with the OpenAI O3 model)
• In the supine position, intraperitoneal blood gravitates to the most dependent right upper-quadrant
space, Morrison’s pouch, which lies between the liver and right kidney. This recess is therefore
scanned first in the FAST examination and is the most sensitive early indicator of hemoperitoneum.
• The paracolic gutters fill only after larger volumes accumulate, while the Pouch of Douglas becomes
dependent chiefly when the patient is upright or in Trendelenburg.
• The vesicouterine pouch is shallower and rarely the initial reservoir.
• The splenorenal recess is less frequently the first site because splenic injury often tamponades and
left-sided free fluid disperses preferentially to the pelvis.
13. What is the most common site of involvement in tuberculosis of the gastrointestinal tract?
A. Stomach
B. Duodenum
C. Ileocaecal region
D. Splenic flexure
E. Rectum
Source: Bydder, Megan, et al. Get Through Final FRCR Part A: SBAs for the Modular Examination. 1st
ed., Hodder Education Publishers, 2009.
Explanation:
• Tuberculosis of the gastrointestinal tract may occur through ingestion of infected sputum, or by
hematogenous spread to submucosal lymph nodes from a pulmonary tuberculous focus.
• It most commonly affects the ileocecal region due to its abundance of lymphoid tissue and relative
stasis of gut contents.
• Typical features at this site include circumferential thickening of the terminal ileum and caecum, a
thickened ileocecal valve and ulceration following the orientation of lymphoid follicles (longitudinal
in the terminal ileum and transverse in the colon).
• Marked enlargement of adjacent mesenteric lymph nodes with central areas of low attenuation may
be seen.
14. On contrast-enhanced CT evaluation of a pancreatic head adenocarcinoma, which one of the following
findings does NOT make the tumor unresectable?
A. Encasement of the hepatic artery
B. Encasement of the celiac axis
C. Direct invasion of the stomach wall
D. Tumor extension into the peripancreatic fat planes
E. >180° encasement of the superior mesenteric artery
Explanation: (by Perplexity AI, generated with the OpenAI O3 model)
• Surgical resection is precluded when a pancreatic cancer involves major arteries (celiac axis, hepatic
artery, SMA) or invades adjacent hollow viscera such as the stomach, because clear margins cannot
be achieved and vascular reconstruction is rarely feasible.
• Peripancreatic fat infiltration, however, is common and does not necessarily indicate vascular or
organ invasion; many resectable tumors show stranding in the fat but have intact peri-arterial planes,
so resection can still proceed.
• Hence option D is the only feature that does not, by itself, define unresectability, whereas options A,
B, C and E each denote locally advanced disease beyond surgical cure.
EBDR Exam MCQs & Concepts October 2021
Dr. Kareem Alnakeeb
335
15. Bilateral suprarenal (adrenal) lesions:
A. 10% are adenomas
B. Colonic metastases are the commonest cause
C. They are often associated with adrenal hemorrhage
D. All of the above
Explanation: (by Google NotebookLM AI “based on FRCR books answers”)
Regarding bilateral suprarenal (adrenal) lesions is their association with adrenal hemorrhage:
• Bilateral adrenal lesions are often associated with adrenal hemorrhage. In cases of shock,
hemorrhage is more likely to be bilateral. Non-traumatic hemorrhage of both adrenals may also be
seen, often caused by perinatal stressors. Various conditions, including surgery, sepsis, burns,
hypotension, pregnancy, cardiovascular disease, and steroid use, are associated with non-traumatic
adrenal hemorrhage, which can lead to bilateral involvement. Adrenal hemorrhage is also listed as a
recognized cause of bilateral large adrenals.
• Regarding other options:
o While adrenal adenomas are common (occurring in 1-2% of the population), and some
benign adenomas can have specific CT characteristics, their specific prevalence as bilateral
lesions is not provided as 10%.
o Adrenal metastases can originate from various primary sites, including lung, breast,
melanoma, gastrointestinal tract, and renal cancer.
16. On emergency CT of the perineum, which imaging feature best distinguishes Fournier’s gangrene from
uncomplicated scrotal cellulitis?
A. Scrotal wall oedema
B. Relative sparing of the perineum
C. Air densities within the subcutaneous tissues
D. Heterogeneous testicular echotexture
E. Small reactive hydrocele
Explanation: (by Perplexity AI, generated with the OpenAI O3 model)
• In Fournier’s gangrene, rapidly spreading necrotizing fasciitis allows gas-forming organisms to
produce visible air pockets in the subcutaneous and fascial planes; their presence on CT is a
hallmark and strongly supports the diagnosis over simple cellulitis.
• Scrotal wall oedema (A) and minor perineal sparing (B) occur in both conditions and therefore lack
specificity.
• The testes themselves are usually spared in Fournier’s gangrene because the gonadal artery arises
proximal to the infected fascial planes, so heterogeneous testicular echotexture (D) points instead to
orchitis or infarction.
• A small reactive hydrocele (E) is a non-specific secondary finding and does not differentiate the two
entities.
17. Which hepatic condition classically demonstrates the “central dot sign” on contrast-enhanced CT or
MRI?
A. Caroli’s disease
B. Primary sclerosing cholangitis
C. Simple hepatic cysts
D. Hepatic mesenchymal hamartoma
E. Hepatic cavernous haemangioma
EBDR Exam MCQs & Concepts October 2021
Dr. Kareem Alnakeeb
336
Explanation: (by Perplexity AI, generated with the OpenAI O3 model)
• The central dot sign is an enhancing portal venous radicle and fibrovascular bundle seen within
fluid-filled, ectatic intrahepatic bile ducts; it is virtually pathognomonic for Caroli’s disease because
the ducts are massively dilated yet remain in continuity with portal triads.
• Primary sclerosing cholangitis produces multifocal strictures and beading without a central
enhancing dot.
• Simple hepatic cysts are avascular, with thin walls and no intraluminal structures.
• Hepatic mesenchymal hamartoma appears as a multicystic mass lacking patent biliary
connections.
• Cavernous hemangiomas show peripheral nodular enhancement progressing centrally, not a single
central dot.
18. A 4-year-old boy falls off his bike and complains of neck pain. Which of the following features is worrying
for a serious injury on plain cervical X-rays?
A. Atlanto-axial distance <5 mm
B. Displacement of 6 mm of the lateral masses relative to the dens
C. Absence of lordosis
D. Disruption of the spinolamellar line
E. Anterior subluxation of C2 on C3
Source: Chin, Teck Yew, et al. Get Through Final FRCR 2A: SBAs. 1st ed., CRC Press, 2017.
19. During colonoscopy a 55-year-old man is found to have several colonic polyps. Which polyp subtype
carries the highest risk of malignant transformation?
A. Hyperplastic polyp
B. Inflammatory polyp
C. Tubular adenoma
D. Villous adenoma
E. Peutz-Jeghers hamartomatous polyp
Explanation: (by Perplexity AI, generated with the OpenAI O3 model)
• Villous adenomas have a broad, villous architecture that harbours high-grade dysplasia more
frequently than other polyp types, giving them the greatest propensity to progress to invasive
colorectal carcinoma.
• Tubular adenomas are common but contain less villous tissue, so the malignant risk is lower unless
the lesion is large.
• Hyperplastic and inflammatory (pseudopolyp) types are generally considered non-neoplastic and
rarely undergo malignant change.
• Hamartomatous polyps in Peutz-Jeghers syndrome can be large but malignant conversion of the
polyp itself is uncommon; the syndrome’s cancer risk arises from associated epithelial tumours
elsewhere, not the polyp.
EBDR Exam MCQs & Concepts October 2021
Dr. Kareem Alnakeeb
337
20. A 30-year-old man is admitted with lower abdominal pain following a road traffic accident in which he
was an unrestrained passenger. Blood is seen in his urine and a bladder injury is suspected. Regarding
extraperitoneal bladder rupture, which of the following is incorrect?
A. It is more common than intraperitoneal rupture.
B. A flame-shaped extravasation of contrast can often be seen.
C. It is usually caused by puncture from a pelvic fracture.
D. Contrast most commonly extravasates into the retropubic space of Retzius.
E. The bladder dome is the most common site of injury.
Source: Chin, Teck Yew, et al. Get Through Final FRCR 2A: SBAs. 1st ed., CRC Press, 2017.
Explanation:
• Extraperitoneal bladder rupture accounts for 80% of bladder ruptures.
• A flame-shaped extravasation of contrast is a classic finding and can be seen to extend into the
perivesical fat and into the retropubic space of Retzius, anterior abdominal wall, upper thigh or
scrotum.
• The most common site of injury is close to the anterolateral aspect of the bladder base. It is usually
caused by puncture from pelvic fractures.
21. Regarding traumatic bladder rupture, which statement is INCORRECT?
A. Extraperitoneal rupture is more common than intraperitoneal rupture.
B. Pelvic fractures are more frequently associated with intraperitoneal than extraperitoneal
rupture.
C. Motor vehicle collisions are the commonest mechanism of injury.
D. On CT cystography, intraperitoneal rupture results in contrast outlining the bladder dome.
E. Intraperitoneal rupture usually requires surgical repair.
Explanation: (by Perplexity AI, generated with the OpenAI O3 model)
• Around 60–70% of traumatic bladder ruptures are extraperitoneal, typically occurring with pelvic ring
fractures after high-energy mechanisms such as motor vehicle collisions.
• These fractures tear the anterolateral bladder wall, causing “flame-shaped” contrast in the
perivesical space, and most cases are managed with catheter drainage alone.
• Intraperitoneal rupture occurs at the bladder dome when a full bladder is suddenly compressed; CT
cystography shows contrast freely outlining bowel loops and the bladder dome and mandates
operative repair to prevent peritonitis.
• Therefore, pelvic fractures are much more strongly linked to extraperitoneal, not intraperitoneal,
rupture, making option B incorrect.
• Options A, C, D and E accurately describe epidemiology, imaging and management.
22. Antenatal ultrasound shows an abnormal facial contour with a large cyst without any cortical mantle of
cerebral tissue anteriorly. Septum pellucidum, falx cerebri and optic tracts are not identified with
evidence of a fused midline thalamus. A single large ventricle is identified. Normal brain stem, midbrain
and cerebellum are noted. What is the diagnosis?
A. Alobar holoprosencephaly
B. Lobar holoprosencephaly
C. Hydranencephaly
D. Anencephaly
E. Congenital hydrocephalous
EBDR Exam MCQs & Concepts October 2021
Dr. Kareem Alnakeeb
338
Source: Chin, Teck Yew, et al. Get Through Final FRCR 2A: SBAs. 1st ed., CRC Press, 2017.
Explanation:
Holoprosencephaly Overview
• Holoprosencephaly (HPE) is considered the most common malformation of the brain and face in
humans.
Alobar Holoprosencephaly
• In alobar HPE, prosencephalic cleavage fails, resulting in a single midline forebrain with a primitive
monoventricle often associated with a large dorsal cyst.
• The olfactory bulbs and tracts, the corpus callosum and anterior commissure, the cavum septum
pellucidum and the interhemispheric fissure are absent, whereas the optic nerves may be normal,
fused or absent.
• The basal ganglia, hypothalamic and thalamic nuclei are typically fused in the midline, resulting in
absence of the third ventricle.
Lobar Holoprosencephaly
• In lobar HPE, the interhemispheric fissure is present along nearly the entire midline, and the thalami
are completely or almost completely separated.
• The corpus callosum may be normal or incomplete, but the cavum septum pellucidum is always
absent.
Hydranencephaly
• Hydranencephaly is the result of a vascular insult (anterior circulation) with the cerebral
hemispheres variably replaced by fluid covered with leptomeninges and dura.
• Falx cerebri is present.
• The cerebellum, midbrain, thalami, basal ganglia, choroid plexus and portions of the occipital lobes,
all fed by the posterior circulation, are typically preserved.
• It is differentiated from hydrocephalus by absence of an intact rim of cortex (seen with even the most
severe hydrocephalus).
23. A 37-year-old woman in her third trimester presented to the labour ward with acute onset of pain in the
lower central abdomen. She had had two previous caesarean sections and an appendectomy with
unremarkable recovery. On clinical examination, there was definite tenderness at the site of a previous
caesarean scar. The best imaging modality for evaluation of caesarean section scar dehiscence is
A. CT
B. Transabdominal US
C. MRI
D. Angiography
E. Transvaginal US
Source: Chin, Teck Yew, et al. Get Through Final FRCR 2A: SBAs. 1st ed., CRC Press, 2017.
Explanation:
• Uterine dehiscence is characterized by incomplete rupture of the uterine wall, usually involving the
endometrium and myometrium but with an intact overlying serosal layer.
• MR imaging may be better than CT in evaluating for uterine dehiscence because of its multiplanar
capability and greater soft-tissue contrast and its ability to help identify an intact serosal layer.
EBDR Exam MCQs & Concepts October 2021
Dr. Kareem Alnakeeb
339
24. A 23-year-old woman with history of pelvic inflammatory disease, 2 months amenorrhea, left lower-
abdominal pain, weakness and occasional spotting showed lower-than-expected levels of HCG for
pregnancy on urine assay. As far as US of the pelvis is concerned, which one of the following sonographic
findings definitively distinguishes ectopic pregnancy from corpus luteum?
A. Visualization of a yolk sac
B. ‘Ring of fire’ appearance
C. Low-impedance flow on Doppler to the ring of fire
D. Visualization of a heartbeat
E. Presence of echogenic pelvic fluid
Source: Chin, Teck Yew, et al. Get Through Final FRCR 2A: SBAs. 1st ed., CRC Press, 2017.
Explanation:
Prevalence and Location
• Ninety-five percent of ectopic pregnancies are tubal; they occur mostly in the ampulla (70%).
Adnexal Mass Findings
• An adnexal mass that is separate from the ovary is the most common finding of a tubal pregnancy
and is seen on US images.
• Although not common, an adnexal mass is more specific for an ectopic pregnancy when it contains a
yolk sac or a living embryo.
Tubal Ring Sign
• The tubal ring sign is the second most common sign of a tubal pregnancy.
• The tubal ring sign describes a hyperechoic ring surrounding an extrauterine gestational sac.
Ring of Fire Sign
• A related finding is the ‘ring of fire’ sign, which is recognized by peripheral hypervascularity of the
hyperechoic ring.
• The term previously described the high-velocity, low-impedance flow surrounding an ectopic adnexal
pregnancy.
• Peripheral hypervascularity is a non-specific finding of the ring of fire sign and may also be seen
surrounding a normal maturing follicle or a corpus luteal cyst.
• Location of the ring of fire in an ovary distinguishes a corpus luteal cyst from an ectopic pregnancy
where the ring is extra-ovarian.
Intrauterine Findings
• Intrauterine findings of an ectopic pregnancy include a ‘normal endometrium’, a pseudo–gestational
sac, a trilaminar endometrium and a thin-walled decidual cyst.
Extrauterine Findings
• Extrauterine findings of ectopic pregnancy include pelvic free fluid, hematosalpinx and
hemoperitoneum.
25. A fit and healthy 25-year-old woman presents to the breast clinic with a small mobile non-tender breast
lump that she noticed incidentally. An ultrasound is deemed as the first-line investigation; it reveals an
extremely well-defined homogenous, hypoechoic oval mass with posterior acoustic shadowing. What is
the most likely diagnosis?
A. Cystosarcoma phylloides
B. Fibroadenoma
C. Complex breast cyst
D. Invasive lobular carcinoma
E. Fat necrosis
EBDR Exam MCQs & Concepts October 2021
Dr. Kareem Alnakeeb
340
Source: Chin, Teck Yew, et al. Get Through Final FRCR 2A: SBAs. 1st ed., CRC Press, 2017.
Explanation:
• Fibroadenomas are the most common cause of benign solid mass in the breast.
• On US, they appear round or oval, wider than tall, hypoechoic, well-defined and mostly homogenous
and show a ‘hump and dip’ sign (small focal bulge to the contour with a contagious small sulcus), a
thin echogenic pseudocapsule and rarely either posterior acoustic enhancement (17%–25%) or
posterior acoustic shadow (9%–11%).
26. An 18 year old man undergoes a Tc MDP bone scan to investigate pain in the right hip. A ‘hot’ lesion is
seen in the right proximal femur. No other lesions are seen. Which of the following lesions would appear
as ‘hot’ on a Tc MDP bone scan?
A. Osteopoikilosis
B. Fibrous cortical defect
C. Acute fracture within 12 hours of injury
D. Fibrous dysplasia
E. Hemangioma
Source: Currie, Stuart, et al. SBAs for the FRCR 2A. 1st ed., Cambridge University Press, 2010.
Explanation:
• The most common site of monostotic fibrous dysplasia is the ribs, followed by proximal femur and
craniofacial bones. 3/4 of cases present before age 30.
• Other benign lesions causing a ‘hot’ on bone scan include Paget’s disease, brown tumours,
aneurysmal bone cysts, osteoid osteoma and chondroblastoma.
• Acute fractures are not usually ‘hot’ until after the first 24–48 hours.
27. Regarding a thyroglossal duct (tract) cyst, which of the following statements is/are true?
A. The cyst may contain ectopic thyroid tissue.
B. Any thyroid tissue present can be functionally active and show uptake on radioiodine (I-131
)
scintigraphy.
C. Such functioning tissue can also concentrate technetium-99m pertechnetate on thyroid scans.
D. All of the above.
Explanation: (by Perplexity AI, generated with the OpenAI O3 model)
Key points about thyroglossal duct cysts and the thyroid tissue they may contain:
• The cyst often harbors heterotopic thyroid tissue, with reports indicating it is present in roughly one-
half of cases.
• When present, this ectopic tissue can be functionally active and concentrate radioiodine,
demonstrating uptake on I-131 scintigraphy.
• The same functioning tissue will also take up technetium-99m pertechnetate, the tracer commonly
used in routine thyroid scans.
28. In a 60-year-old patient undergoing Tc-99m HIDA scintigraphy, both hepatic parenchymal uptake and
gallbladder visualization are markedly delayed. Which interpretation is most accurate?
A. A reliable sign of acute cholecystitis
B. Typical for cystic duct obstruction by gallstones
C. Not a reliable sign of acute or chronic cholecystitis
D. Suggestive of congenital gallbladder agenesis
E. Indicative of biliary atresia
EBDR Exam MCQs & Concepts October 2021
Dr. Kareem Alnakeeb
341
Explanation: (by Perplexity AI, generated with the OpenAI O3 model)
• Normal HIDA criteria for acute cholecystitis are non-visualization of the gallbladder despite normal
hepatic uptake and prompt biliary excretion.
• When tracer extraction by hepatocytes is also reduced, delivery to the cystic duct and gallbladder is
proportionally delayed, producing false-positive non-visualization.
• Therefore, concurrent poor hepatic uptake makes the finding unreliable for diagnosing acute or
chronic cholecystitis.
• Gallstone obstruction (B) classically shows absent GB activity but preserved liver uptake; gallbladder
agenesis (D) and biliary atresia (E) likewise retain normal or increased hepatic extraction,
distinguishing them from the combined hepatic-gallbladder delay seen here.
29. Which skeletal dysplasia is classically associated with generalized osteoporosis and fragile bones?
A. Achondroplasia
B. Osteogenesis imperfecta
C. Developmental hip dysplasia
D. Fibrous dysplasia
E. Morquio syndrome
Explanation: (by Perplexity AI, generated with the OpenAI O3 model)
• Osteogenesis imperfecta (OI) is a collagen-I disorder producing thin cortices, low bone mineral
density and recurrent fractures – the textbook picture of generalised osteoporosis.
• Achondroplasia causes short-limb dwarfism with normal or increased bone density, not
osteoporosis.
• Developmental hip dysplasia is a local acetabular malformation without systemic bone loss.
• Fibrous dysplasia replaces focal bone with fibrous tissue; overall bone mass is often normal, and
generalised osteoporosis is not typical.
• Morquio syndrome leads to dysostosis multiplex with thickened bones rather than osteoporotic
ones.
30. CT of the pelvis in a 37-year-old woman who was undergoing ovulation induction showed massive cysts
in the pelvis surrounding a core of central ovarian stroma with relatively higher attenuation. More
cephalad images demonstrated ascites. All of the following are features of ovarian hyperstimulation
syndrome, except
A. Enlarged multicystic ovaries
B. Free intraperitoneal fluid
C. Low serum estradiol levels
D. Pleural or pericardial effusion
E. Risk of deep vein thrombosis
Source: Chin, Teck Yew, et al. Get Through Final FRCR 2A: SBAs. 1st ed., CRC Press, 2017.
Explanation:
Etiology and Occurrence
• Ovarian hyperstimulation syndrome is usually iatrogenic secondary to ovarian stimulant drug therapy
for infertility but may occur as a spontaneous event in pregnancy and is associated with raised serum
estradiol levels.
Clinical Presentation and Complications
• The syndrome consists of ovarian enlargement with extravascular accumulation of exudates leading
to weight gain, ascites, pleural effusions, intravascular volume depletion with hemoconcentration
and oliguria in varying degrees, with increased risk of thrombosis and stroke.
EBDR Exam MCQs & Concepts October 2021
Dr. Kareem Alnakeeb
342
• Pain, abdominal distention, nausea and vomiting are frequently seen.
Imaging Findings
• The imaging findings are similar at US, CT and MR imaging and reflect ovarian enlargement by
distended corpora luteal cysts of varying sizes.
• Because the enlarged follicles are often peripheral in location, a spoked wheel appearance has been
described.
• Ascites, pleural effusion and pericardial effusion are also described.
Differential Diagnosis
• Familiarity with ovarian hyperstimulation syndrome and the appropriate clinical setting should help
avoid the incorrect diagnosis of an ovarian cystic neoplasm.
31. In primary osteoarthritis of the knee, which tissue is the principal site of degenerative change?
A. Joint capsule
B. Tendon
C. Synovium
D. Articular cartilage
E. Subchondral bone
Explanation: (by Perplexity AI, generated with the OpenAI O3 model)
• Osteoarthritis is characterized by progressive loss of the smooth, hyaline articular cartilage that lines
the joint surfaces.
• Mechanical and biochemical stresses lead to cartilage softening, fibrillation and eventual erosion,
producing joint space narrowing on radiographs.
• Secondary changes—synovial inflammation, capsular thickening, tendon irritation and subchondral
bone sclerosis—can occur, but these are consequences rather than the primary pathology.
• The joint capsule (A) and synovium (C) may show reactive changes, while tendons (B) are only
indirectly affected. Subchondral bone (E) develops eburnation and cysts after cartilage loss, not
before it.
32. An ultrasound screening of a 6-week-old infant shows a coronal static view of the right hip with an alpha
angle of 68° and a beta angle of 48°. What is the Graf classification of this hip?
A. Immature hip
B. Normal hip
C. Acetabular deficiency
D. Dislocation
E. Subluxation
Explanation: (by Perplexity AI, generated with the OpenAI O3 model)
• A Graf type I (normal) hip has an alpha angle >60° and a beta angle <55°, indicating a well-developed
bony acetabular roof and a properly covered femoral head.
• The measurements in this case (alpha 68°, beta 48°) meet these criteria, so option B is correct.
• An immature (type IIa) hip (option A) would have an alpha of 50–59°; acetabular deficiency/dysplasia
(option C) presents with alpha 43–49° and higher beta angles.
• A frankly dislocated hip (option D) demonstrates alpha <43° with the femoral head displaced;
subluxation (option E) shows alpha 43–49° with partial lateralization.
• Thus, all distractors have measurement ranges inconsistent with the values provided.
EBDR Exam MCQs & Concepts October 2021
Dr. Kareem Alnakeeb
343
33. A 42-year-old woman was found to have a mass in the right lobe of the liver on US and sent for an MRI
liver for further characterization. All of the following are expected MR features of FNH, except
A. High signal intensity of the central scar on T2W images
B. Uniform enhancement of the mass in the arterial phase
C. Lack of capsular enhancement in the arterial phase
D. Hypointense to surrounding liver on the enhanced portal venous phase
E. Maximum intensity of central scar on enhanced delayed phase images
Source: Chin, Teck Yew, et al. Get Through Final FRCR 2A: SBAs. 1st ed., CRC Press, 2017.
Explanation:
MRI Signal and Enhancement
• Typically, FNH is iso- or hypointense on T1-weighted images, is slightly hyper- or isointense on T2-
weighted images and has a hyperintense central scar on T2-weighted images.
• FNH shows intense homogeneous enhancement in the arterial phase and enhancement of the
central scar in the later phases of gadolinium-enhanced imaging.
Pseudocapsule
• FNH does not have a tumour capsule, although the pseudocapsule surrounding some FNH lesions
may be quite prominent.
• The pseudocapsule of FNH results from compression of the surrounding liver parenchyma by the
FNH, perilesion vessels and inflammatory reaction.
• The pseudocapsule is usually a few millimetres thick and typically shows high signal intensity on T2-
weighted images.
• The pseudocapsule may show enhancement on delayed contrast-enhanced images.
Central Scar
• A central scar is present at imaging in most patients with FNH.
• The amount of scar tissue within FNH and the size of the central scar may vary.
• The central scar is typically high in signal intensity on T2-weighted images and low in signal intensity
on T1-weighted images.
• It shows visible enhancement on delayed contrast-enhanced images.
• High signal intensity of the central scar may be caused by the inflammatory reaction around the
ductular proliferation as well as the vessels within the septa and central scar.
• The central scar is not a specific finding of FNH and can be seen in a variety of other focal liver
lesions, such as giant haemangiomas.
34. A 30-year-old woman undergoes MRI for an incidental 3 cm liver lesion. Which MRI feature is most
characteristic of focal nodular hyperplasia?
A. Central scar demonstrates high signal on T1-weighted imaging
B. Central scar is hypoenhancing in arterial phase and shows delayed contrast retention
C. Lesion contains cystic degeneration
D. Central scar shows avid arterial phase enhancement
E. Lesion demonstrates a thick enhancing capsule on portal venous phase
Explanation: (by Perplexity AI, generated with the OpenAI O3 model)
• Focal nodular hyperplasia (FNH) contains a fibrous central scar that is low signal on T1 and
enhances only on delayed post-gadolinium images because contrast slowly diffuses into the fibrous
tissue.
EBDR Exam MCQs & Concepts October 2021
Dr. Kareem Alnakeeb
344
• This pattern—poor arterial enhancement with delayed retention—is a classic sign, making option B
correct.
• The scar is not bright on T1 (A) and does not show rapid arterial enhancement (D).
• Cystic change (C) and a thick capsule (E) are seen in adenoma or hepatocellular carcinoma, not in
FNH.
35. A 35-year-old man presents with thigh pain; radiographs show a well-defined, expansile lytic lesion with
cortical thinning and focal breach centred in the meta-epiphysis of the distal femur.
A. Simple bone cyst
B. Plasmacytoma
C. Giant cell tumour
D. Fibrous dysplasia
E. Chondroblastoma
Explanation: (by Perplexity AI, generated with the OpenAI O3 model)
• Giant cell tumour (GCT) characteristically arises in skeletally mature patients aged 20–40 years,
abutting the articular surface and spanning the epiphysis into the adjacent metaphysis. It appears as
a well-defined, non-sclerotic lytic lesion that expands and may erode cortex, matching this case.
Simple bone cysts are purely metaphyseal, usually in children, and seldom breach cortex.
• Plasmacytoma occurs in older adults and classically involves the axial skeleton rather than the
distal femur.
• Fibrous dysplasia presents as a ground-glass, intramedullary lesion with intact cortex and does not
typically localise to the epiphysis.
• Chondroblastoma is epiphyseal but usually affects adolescents and shows chondroid matrix
calcification rather than cortical destruction.
36. Regarding colonic diverticulitis, which of the following statements is correct?
A. It predominantly affects young adults.
B. It commonly presents with mechanical large-bowel obstruction.
C. Non-contrast CT is superior to contrast-enhanced CT for detecting complications.
D. None of the above.
Explanation: (by Perplexity AI, generated with the OpenAI O3 model)
• Colonic diverticulitis typically occurs in middle-aged or older patients; while incidence in those
under 50 years is rising, they remain a minority.
• Intestinal obstruction is an uncommon presentation; perforation, abscess or fistula are far more
frequent complications.
• Modern studies show non-contrast CT is overall non-inferior—but not superior—to contrast-
enhanced CT; contrast improves sensitivity for abscess or perforation, so it remains preferred when
feasible.
• Therefore, options A-C are incorrect, making “None of the above” the best choice.
EBDR Exam MCQs & Concepts October 2021
Dr. Kareem Alnakeeb
345
37. On MRI of a patient with chronic back pain, which of the following findings is most characteristic of Pott
disease (spinal tuberculosis) affecting the lumbar spine?
A. Vertebral body bone destruction involving two adjacent levels
B. Iliopsoas abscess tracking along the muscle belly
C. Preservation of intervertebral disc height at the diseased level
D. Advanced degenerative disc desiccation with Modic I change
E. Marginal osteophyte formation with vacuum phenomenon
Explanation: (by Perplexity AI, generated with the OpenAI O3 model)
• Tuberculous spondylitis typically spreads beneath the anterior longitudinal ligament to involve
adjacent vertebral bodies, then tracks into the iliopsoas sheath, producing a cold iliopsoas abscess,
often large and relatively asymptomatic, making it a classic imaging clue.
• Although vertebral destruction (A) and relative disc preservation (C) can be seen, they are not as
specific because similar patterns occur in other low-virulence infections or early pyogenic disease.
• Degenerative disc changes (D) and osteophyte formation with vacuum phenomenon (E) are features
of spondylosis, not infection.
38. A 45-year-old woman with suspected early rheumatoid arthritis undergoes high-frequency ultrasound of
the 2nd metacarpophalangeal joint; which finding is most specific for the diagnosis?
A. Cortical erosion
B. Thickened capsule
C. Synovitis
D. Joint effusion
E. Tenosynovitis
Explanation: (by Perplexity AI, generated with the OpenAI O3 model)
• Cortical erosion represents a breach in the bright cortical line and is highly specific for inflammatory
arthropathies such as rheumatoid arthritis; ultrasound detects these erosions earlier than plain
radiography, making them a key marker of early disease.
• Synovitis, joint effusion and tenosynovitis are sensitive indicators of inflammation but occur in
many arthritides and even in non-inflammatory conditions, limiting specificity.
• A thickened joint capsule is a non-specific feature that may accompany chronic degeneration or
prior injury rather than active rheumatoid pathology. Therefore, demonstration of a definite cortical
erosion best supports an early diagnosis of rheumatoid arthritis.
39. On high-resolution ultrasound of a metacarpophalangeal joint, which finding is regarded as the most
characteristic of rheumatoid arthritis?
A. Cortical bone erosion
B. Simple joint effusion
C. Synovial hypertrophy (active synovitis)
D. Capsular thickening
E. Marginal osteophyte formation
Explanation: (by Perplexity AI, generated with the OpenAI O3 model)
• Cortical bone erosion represents loss of the normally smooth hyperechoic bone cortex caused by
pannus-mediated destruction and is highly specific for rheumatoid arthritis, often appearing before
erosions are visible on radiographs. Ultrasound can depict these sub-millimetre cortical breaks with
great sensitivity, making them a key diagnostic marker.
EBDR Exam MCQs & Concepts October 2021
Dr. Kareem Alnakeeb
346
• Simple effusion (B) and synovial hypertrophy (C) are sensitive but occur in many inflammatory or
degenerative arthritides.
• Capsular thickening (D) is a non-specific chronic change.
• Marginal osteophytes (E) are typical of osteoarthritis, not rheumatoid disease.
40. A nuclear medicine bone scan shows intense radiotracer uptake confined to the left hemithorax on both
anterior and posterior whole-body projections. Which single condition best explains this pattern?
A. Pleural effusion
B. Unilateral hyperinflated lung
C. Prior mastectomy
D. All of the above
E. None of the above
Explanation: (by Perplexity AI, generated with the OpenAI O3 model)
• After a unilateral mastectomy, radiopharmaceutical soft-tissue pooling occurs in the post-surgical
chest wall and axilla, producing conspicuous, hemithoracic tracer activity on both anterior and
posterior views.
• Pleural effusion typically causes peripheral or crescentic uptake only on the anterior image because
fluid layers dependently, making the posterior view less intense.
• Hyperinflated lung contains predominantly air; it shows reduced, not increased, soft-tissue tracer
concentration, so it would appear photopenic rather than “hot.”
• Therefore mastectomy is the sole listed cause giving symmetric anterior-posterior hemithorax
uptake; the other options are incorrect.
41. Which of the following conditions is least likely to produce vertebra plana on spinal imaging?
A. Primary spinal lymphoma
B. Fibrous dysplasia
C. Vertebral metastasis from breast carcinoma
D. Langerhans cell histiocytosis
E. Spinal tuberculosis
Explanation: (by Perplexity AI, generated with the OpenAI O3 model)
• Vertebra plana refers to near-complete collapse of a vertebral body with preserved disc spaces. It is
classically associated with Langerhans cell histiocytosis, but similar flattening can arise from
malignant infiltration (lymphoma, metastasis, myeloma), infectious destruction such as
tuberculosis, or severe trauma. In these entities, bone is weakened by infiltration or lytic destruction,
predisposing to collapse.
• Fibrous dysplasia, however, characteristically causes medullary replacement by fibro-osseous
tissue, leading to bone expansion and cortical thickening rather than flattening; true vertebra plana is
exceptional in this disorder, making it the least likely cause among the options.
42. Which nerve passes directly posterior to the medial epicondyle of the humerus, making it vulnerable to
injury with a “funny-bone” knock?
A. Axillary nerve
B. Median nerve
C. Radial nerve
D. Ulnar nerve
E. Musculocutaneous nerve
EBDR Exam MCQs & Concepts October 2021
Dr. Kareem Alnakeeb
347
Explanation: (by Perplexity AI, generated with the OpenAI O3 model)
• The ulnar nerve courses in the ulnar groove immediately behind the medial epicondyle before
entering the cubital tunnel, so it is the structure struck in a classic “funny-bone” injury.
• The axillary nerve winds around the surgical neck of the humerus, not the epicondyle.
• The median nerve lies anterior to the elbow within the cubital fossa.
• The radial nerve passes anterior to the lateral epicondyle as the posterior interosseous branch.
• The musculocutaneous nerve terminates in the forearm and does not relate to the epicondyle.
43. Which CT feature is NOT typically associated with acute intestinal infarction?
A. Free intraperitoneal air
B. Mural gas
C. Bowel wall thickening
D. Thumb-printing
E. Pneumatosis intestinalis
Explanation: (by Perplexity AI, generated with the OpenAI O3 model)
• Acute intestinal infarction most often shows intramural pneumatosis (pneumatosis intestinalis) and
associated portal or mesenteric venous gas, producing the classic “mural gas” appearance.
Ischemic oedema leads to circumferential bowel wall thickening, while submucosal hemorrhage or
oedema gives rise to the radiological “thumb-printing” sign.
• Free intraperitoneal air indicates bowel perforation rather than ischemia itself and is therefore not a
direct sign of intestinal infarction, making option A the exception.
• Options B, C and D represent well-recognized CT manifestations of ischemic injury.
44. Which anatomical site is least commonly the primary location for a paediatric neuroblastoma?
A. Retroperitoneum
B. Adrenal gland
C. Anterior mediastinum
D. Pelvis
E. Neck
Explanation: (by Perplexity AI, generated with the OpenAI O3 model)
• More than 90% of neuroblastomas arise in the sympathetic chain of the abdomen, most frequently
within the adrenal medulla or adjacent retroperitoneum.
• Thoracic primaries, especially posterior mediastinal masses, account for a smaller but recognised
proportion; anterior mediastinal origin is far rarer but still documented.
• Cervical tumours are uncommon yet well described.
• True pelvic primaries are distinctly rare, representing the least frequent site among the options,
making pelvis the correct choice.
• Distractors: adrenal and retroperitoneal sites are the commonest; anterior mediastinal and neck
primaries, though uncommon, are encountered more often than pelvic lesions.
45. In a pre-term neonate presenting with abdominal distension, bilious vomiting and systemic instability,
which statement regarding necrotizing enterocolitis is CORRECT?
A. It is classified as a “critical case” in the Royal College of Radiologists’ pediatric imaging guidelines
B. Sonographic features usually allow reliable antenatal diagnosis
C. It commonly leads to pneumoperitoneum visible on abdominal radiographs
D. All of the above
EBDR Exam MCQs & Concepts October 2021
Dr. Kareem Alnakeeb
348
Explanation: (by Perplexity AI, generated with the OpenAI O3 model)
• Pneumoperitoneum develops in 15–20% of infants with necrotizing enterocolitis when bowel
perforation occurs, and free intraperitoneal gas is a classic radiographic sign guiding urgent surgical
referral.
• The RCR “Pediatric Digital Radiography” guidance lists NEC imaging as high priority rather than the
highest “critical” category, so option A is inaccurate.
• Antenatal ultrasound may show non-specific dilated bowel loops but does not reliably diagnose
NEC, making option B incorrect. Therefore, only statement C is correct.
46. Q: In an otherwise healthy adult undergoing CT for nonspecific abdominal pain, bilateral adrenal masses
are detected. Which statement regarding their etiology is most accurate?
A. Simple adrenal adenomas account for about 10% of bilateral lesions
B. Bilaterality strongly points to metastatic disease
C. Acute bilateral adrenal hemorrhage is a recognized cause
D. Both A and C
E. None of the above
Explanation: (by Perplexity AI, generated with the OpenAI O3 model)
• Bilateral adrenal enlargement has a broad differential. While solitary adenomas are common,
approximately 10% of adrenal adenomas are bilateral, so adenoma remains possible.
• Acute stress-related or anticoagulation-related hemorrhage can also involve both glands, producing
transient bilateral masses.
• Although metastases (e.g. lung, breast) often seed both adrenals, bilaterality is not specific and
therefore cannot be assumed to “strongly point” to metastatic disease; option B overstates this
association. Therefore both statements A and C are correct.
47. Regarding colonic diverticulitis, which statement is most accurate for clinical practice?
A. Non-contrast CT is preferred over contrast-enhanced CT for diagnosis
B. It predominantly affects patients under 30 years of age
C. It can present with large-bowel obstruction as a complication
D. It never recurs after uncomplicated medical treatment
E. All of the above statements are correct
Explanation: (by Perplexity AI, generated with the OpenAI O3 model)
• Contrast-enhanced CT is the imaging modality of choice because intravenous contrast better
delineates inflamed bowel wall, abscesses and complications; non-contrast studies are less
sensitive. Diverticulitis is classically a disease of older adults; incidence rises sharply after 40 years,
so it is uncommon in the young. Complicated diverticulitis may lead to segmental colonic strictures
causing acute or chronic large-bowel obstruction, making option C the correct choice.
• Recurrence is noted in up to 20–30% of cases even after uncomplicated episodes, so option D is
incorrect. Options A, B and D are therefore false.
EBDR Exam MCQs & Concepts October 2021
Dr. Kareem Alnakeeb
349
48. An intravenous urography (IVU) series in a 55-year-old diabetic man with acute flank pain shows multiple
filling defects producing an irregularly interrupted outline along the papillary tips of both kidneys (“ring-
shadow” sign). What is the most likely diagnosis?
A. Acute papillary necrosis
B. Chronic pyelonephritis
C. Renal transitional cell carcinoma
D. Angiomyolipoma
E. Medullary sponge kidney
Explanation: (by Perplexity AI, generated with the OpenAI O3 model)
• Acute papillary necrosis causes sloughed or infarcted renal papillae that detach and appear as
ring-shaped filling defects on IVU, giving an interrupted or “ring-shadow” appearance of the normal
papillary line.
• Chronic pyelonephritis produces calyceal clubbing and cortical scarring, not discrete ring defects.
• Transitional cell carcinoma typically shows an irregular solitary filling defect or narrowing of the
collecting system, not multiple ring shadows.
• Angiomyolipoma is usually detected as a fat-containing mass on cross-sectional imaging and is not
outlined on IVU.
• Medullary sponge kidney manifests as brush-like medullary striations due to ectatic collecting
ducts, not ring shadows or papillary interruption.
49. A 16-year-old boy presents with persistent knee pain. Radiographs reveal a well-defined lucent lesion
with sclerotic margin centered in the distal femoral epiphysis. What is the most typical anatomical site
for a chondroblastoma?
A. Epiphyseal
B. Diaphyseal
C. Metadiaphyseal
D. Metaphyseal
E. Meta-epiphyseal
Explanation: (by Perplexity AI, generated with the OpenAI O3 model)
• Chondroblastoma is a benign cartilaginous tumor that characteristically arises in the epiphysis (or
apophysis) of long bones in skeletally immature patients, most often around the knee, proximal
humerus or proximal femur. Epiphyseal location is therefore the hallmark and guides diagnosis.
• Lesions purely in the metaphysis (D) or diaphysis (B) suggest other entities such as non-ossifying
fibroma or eosinophilic granuloma, while metadiaphyseal (C) and meta-epiphyseal (E) sites are
atypical and less specific, making these options distractors.
50. A 35-year-old woman with a palpable nodule in the left lobe of the thyroid gland showed a corresponding
area of low activity on nuclear medicine study consistent with a cold nodule. How would you investigate
this patient further?
A. MRI neck
B. CT neck with contrast
C. US neck
D. US neck with FNA
E. Sialogram
Source: Chin, Teck Yew, et al. Get Through Final FRCR 2A: SBAs. 1st ed., CRC Press, 2017.
EBDR Exam MCQs & Concepts October 2021
Dr. Kareem Alnakeeb
350
Explanation:
Thyroid Scanning and Nodule Assessment
• Thyroid scanning using pertechnetate (99MTc) is traditionally used to screen thyroid nodules for
malignancy.
• The finding of a hyperfunctioning or ‘hot’ nodule (uptake of tracer within the nodule with suppression
of uptake in the surrounding normal thyroid tissue) excludes malignancy in almost all patients.
• A non-functioning or ‘cold’ nodule was thought to indicate increased risk of malignancy, with 5%–
15% of these being malignant.
Investigations for Thyroid Nodules
• FNA should be the first-line investigation for assessment of all solitary nodules or a dominant nodule
in a multinodular goitre.
• US is well established as a primary investigation for patients presenting with a lump in the neck;
moreover, the cost-effectiveness and diagnostic accuracy of FNA can be increased by using US
guidance and the presence of an on-site cytopathologist.
51. A 46-year-old woman is found to have a solitary cold nodule in the right thyroid lobe on technetium-99m
pertechnetate scintigraphy. What is the most appropriate next management step?
A. Repeat ultrasound in 12 months
B. Ultrasound-guided fine-needle aspiration
C. Contrast-enhanced CT neck
D. MRI neck with gadolinium
E. Thyroid-stimulating hormone suppression trial
Explanation: (by Perplexity AI, generated with the OpenAI O3 model)
• Cold (non-functioning) thyroid nodules carry a higher malignancy risk than “hot” nodules.
• Current guidelines recommend targeted ultrasound to characterize echogenicity, margins and
vascularity, immediately coupled with ultrasound-guided FNA for cytology when the nodule is ≥1
cm or has suspicious sonographic features.
• Cross-sectional CT or MRI offers no first-line diagnostic advantage and may delay tissue diagnosis;
iodine-containing contrast also interferes with subsequent radioiodine imaging.
• Simple interval ultrasound follow-up alone is inappropriate in a potentially malignant cold nodule,
and TSH suppression therapy is not recommended because it neither diagnoses nor reliably treats
malignancy.
52. In which of the following clinical situations is detailed fetal echocardiography specifically recommended
according to current obstetric-cardiology guidelines?
A. Routine screening at 14 weeks’ gestation
B. All twin pregnancies regardless of other risk factors
C. Sustained fetal heart rhythm abnormality detected on antenatal scan
D. Maternal age above 35 years as the sole indication
E. Normal first-trimester ultrasound with nuchal translucency <2.5 mm
Explanation: (by Perplexity AI, generated with the OpenAI O3 model)
• Sustained or recurrent fetal arrhythmia (e.g. supraventricular tachycardia or complete heart block)
warrants targeted fetal echocardiography to assess for structural heart disease and guide in-utero or
perinatal management.
• Routine screening at 14 weeks (option A) is too early for comprehensive cardiac assessment;
standard anomaly scans with cardiac views are performed at 18-22 weeks.
EBDR Exam MCQs & Concepts October 2021
Dr. Kareem Alnakeeb
351
• Twin pregnancy alone (option B) is not an absolute indication unless additional risk factors exist
such as monochorionicity or abnormal findings.
• Advanced maternal age (option D) increases aneuploidy risk but does not, by itself, mandate fetal
echocardiography.
• A completely normal early ultrasound (option E) does not justify further cardiac imaging.
53. Which hepatic focal lesion typically demonstrates true intratumoral fat on imaging?
A. Hepatocellular carcinoma
B. Cholangiocarcinoma
C. Focal nodular hyperplasia
D. Cavernous hemangioma
Explanation: (by Perplexity AI, generated with the OpenAI O3 model)
Intralesional fat is seen in a subset of hepatic focal lesions, but its frequency varies markedly among the
major tumor types:
• Hepatocellular carcinoma (HCC) – Fat is common enough to be listed as an “ancillary feature
favoring HCC” in LI-RADS. Histologic series show microscopic or macroscopic fat in roughly 15–35%
of small HCCs under 3 cm.
• Focal nodular hyperplasia (FNH) – Fat can be demonstrated histologically in about 20% of resected
FNHs, but only half of those cases show detectable signal loss on chemical-shift MRI, so visible fat
on imaging is distinctly uncommon.
• Cholangiocarcinoma – Intratumoral fat is essentially absent; when apparent, it almost always
represents entrapped steatotic liver or focal fat deposition adjacent to the mass rather than fat inside
the tumor itself.
• Cavernous hemangioma – True fat within the vascular spaces is not a feature; hemangiomas may
sit inside a background of fatty liver or show a rim of “peritumoral fat-sparing,” but the lesion itself
does not contain fat.
Therefore, among the four choices the hepatic focal lesion in which fat is a recognized intrinsic component is:
Hepatocellular carcinoma (HCC)
54. On abdominal ultrasound, which one of the following sonographic features most reliably supports the
diagnosis of congestive hepatomegaly secondary to right-sided heart failure?
A. Dilated, non-pulsatile portal veins
B. Pleural effusion without ascites
C. Normal-calibre hepatic veins with monophasic Doppler flow
D. Simultaneous dilatation of the inferior vena cava and all three hepatic veins with loss of normal
triphasic Doppler waveform
E. Markedly increased liver echogenicity suggestive of fatty infiltration
Explanation: (by Perplexity AI, generated with the OpenAI O3 model)
• Passive hepatic congestion occurs when raised right-atrial pressure is transmitted to the liver. The
most consistent ultrasound findings are a dilated inferior vena cava (>21 mm) together with enlarged
hepatic veins (>8–9 mm) showing dampened or reversed (‘to-and-fro’) flow and loss of the normal
triphasic waveform; these changes directly reflect elevated central venous pressure, making option
D correct.
• Isolated portal-vein dilatation (A) is non-specific and may be seen in portal hypertension.
EBDR Exam MCQs & Concepts October 2021
Dr. Kareem Alnakeeb
352
• Small pleural effusions (B) are common in cardiac failure but are an extra-hepatic sign and not
diagnostic of hepatic congestion.
• Normal-caliber hepatic veins with monophasic flow (C) argue against congestion.
• Increased hepatic echogenicity (E) suggests steatosis rather than venous congestion.
55. In the context of bone tumors and infection, which statement regarding a Codman triangle is FALSE?
A. It forms between the elevated periosteum and underlying cortex
B. It represents an aggressive periosteal reaction
C. It may be demonstrated in a non-ossifying fibroma
D. It is classically associated with osteosarcoma
E. It is usually seen in acute osteomyelitis
Explanation: (by Perplexity AI, generated with the OpenAI O3 model)
• A Codman triangle arises when rapidly growing pathology lifts the periosteum away from the cortex,
producing a triangular edge of new bone (option A true).
• Because such rapid elevation reflects aggressive disease, the reaction is considered aggressive
(option B true).
• Although most typical of high-grade malignancies like osteosarcoma (option D true), any fast-
expanding lesion—including some benign entities such as a large non-ossifying fibroma—can create
the appearance (option C true).
• Acute osteomyelitis often shows a solid or lamellated periosteal response; the Codman triangular
pattern is uncommon, making option E the false statement.
56. A 38-year-old woman presents with chronic menorrhagia. Pelvic MRI shows a 4 cm left adnexal lesion
that is uniformly low signal on T1-weighted images and markedly low signal on T2-weighted images.
Which is the most likely diagnosis?
A. Endometrioma
B. Leiomyoma
C. Hemorrhagic cyst
D. Ovarian fibroma
E. Serous cystadenoma
Explanation: (by Perplexity AI, generated with the OpenAI O3 model)
• Ovarian fibromas are solid, fibrous stromal tumors that appear low signal on both T1 and T2
sequences because of their dense collagen content; they may mimic uterine fibroids but are usually
separate from the uterus.
• Endometriomas and hemorrhagic cysts typically show high T1 signal from blood products, making
options A and C incorrect.
• Leiomyomas (uterine fibroids) can appear low on T1/T2 but would arise from the uterus, not the
adnexa, so B is less likely when a distinct ovarian mass is seen.
• Serous cystadenomas are predominantly cystic with fluid signal (low T1, high T2), unlike the
uniformly low T2 lesion described, excluding option E.
EBDR Exam MCQs & Concepts October 2021
Dr. Kareem Alnakeeb
353
57. In myocardial hibernation, which statement best describes the underlying pathophysiological change?
A. Irreversible myocyte necrosis with normal coronary flow
B. Transient coronary spasm causing acute stunning
C. Persistently reduced myocardial perfusion with preserved viability
D. Normal perfusion with idiopathic cardiomyopathy
E. Patchy myocardial fibrosis following myocarditis
Explanation: (by Perplexity AI, generated with the OpenAI O3 model)
• Myocardial hibernation refers to chronically under-perfused but still viable myocardium that down-
regulates contractile function to match the reduced blood supply; when adequate coronary flow is
restored (e.g. by revascularization) contractility can recover. Therefore, the key feature is a persistent
decrease in perfusion while cellular viability is maintained (Option C).
• Option A is false because hibernation is reversible and does not involve necrosis.
• Option B describes myocardial stunning, a different phenomenon due to acute ischemia with rapid
flow restoration.
• Option D has normal perfusion, so does not fit the definition.
• Option E relates to post-inflammatory scarring, which is non-viable tissue and cannot improve with
revascularization.
58. A 29-year-old woman is referred for pelvic ultrasound because of intermittent lower-abdominal pain.
Transvaginal imaging shows a 3 cm well-defined, thin-walled adnexal cyst that is uniformly hyperechoic
with posterior acoustic shadowing; serum β-hCG is negative. Which is the most likely diagnosis?
A. Hemorrhagic corpus luteum cyst
B. Endometrioma
C. Mature cystic (dermoid) teratoma
D. Serous cystadenoma
E. Para-ovarian cyst
Explanation: (by Perplexity AI, generated with the OpenAI O3 model)
• Mature cystic teratomas typically appear as small, well-circumscribed adnexal masses containing
echogenic sebaceous material and hair that produce marked acoustic shadowing—the classic “tip-
of-the-iceberg” sign—matching the uniformly hyperechoic lesion here.
• Hemorrhagic corpus luteum cysts are usually reticular or crenulated with internal echoes and
evolve over weeks; endometriomas display homogeneous low-level echoes (“ground-glass”) rather
than dense echogenicity.
• Serous cystadenomas and para-ovarian cysts are thin-walled, anechoic and do not cast
shadowing.
59. A 15-year-old girl presents with acute pelvic pain. Beta-human chorionic gonadotrophin (β-hCG) is
normal. US demonstrates a thin-walled 5 cm echogenic adnexal mass with posterior acoustic
enhancement. There is no color internal Doppler signal. Follow-up imaging after 3 months fails to
demonstrate the mass. What is the most likely cause?
A. Appendix abscess
B. Hemorrhagic ovarian cyst
C. Ovarian dermoid
D. Ectopic pregnancy
E. Ovarian torsion
EBDR Exam MCQs & Concepts October 2021
Dr. Kareem Alnakeeb
354
Source: Chin, Teck Yew, et al. Get Through Final FRCR 2A: SBAs. 1st ed., CRC Press, 2017.
Explanation:
Right Hemipelvic Mass Presentations
• All of the above may present as a right hemipelvic mass.
Hemorrhagic Ovarian Cyst
• Hemorrhage into an ovarian follicular cyst is the most common of these, and it usually resolves after
one or two menstrual cycles.
• Several patterns of ultrasound findings have been described, including an echogenic mass, a ground-
glass pattern (diffuse low-level echoes), a whirled pattern of mixed echogenicity and a ‘fishnet
weave’ pattern (fine septations or reticular echoes).
Appendiceal Abscess
• Appendix abscess would be thick-walled.
Ovarian Dermoid
• Ovarian dermoid may present as an echogenic mass, although acoustic shadowing would be more
typical owing to internal calcifications and would not resolve in this fashion.
Ectopic Pregnancy
• Ectopic pregnancy may present as an echogenic ‘tubal mass’, although elevated β-hCG would be a
feature.
Ovarian Torsion
• Ovarian torsion may appear as an enlarged echogenic ovary (owing to oedema) and, like
hemorrhagic cyst, often lacks internal color Doppler flow, although it is less common than
hemorrhagic cyst.
• It would also necessitate urgent surgery rather than follow-up imaging.
60. Q: Which statement regarding Peutz–Jeghers syndrome is correct?
A. It is inherited in an autosomal dominant pattern
B. It presents with mucocutaneous pigmentation and gastrointestinal hamartomatous polyps
C. It follows an autosomal recessive inheritance pattern
D. Both statements B and C are correct
E. There is no increased risk of malignancy compared with the general population
Explanation: (by Perplexity AI, generated with the OpenAI O3 model)
• Peutz–Jeghers syndrome is classically characterized by multiple hamartomatous polyps throughout
the gastrointestinal tract together with distinctive mucocutaneous melanotic macules, especially
around the lips and oral mucosa, making option B correct.
• The condition is inherited in an autosomal dominant fashion due to pathogenic variants in the STK11
(LKB1) gene, so option A is incorrect.
• It is not autosomal recessive, rendering options C and D incorrect.
• Patients have a markedly increased lifetime risk of several malignancies (pancreatic,
gastrointestinal, breast, others), so option E is wrong.
EBDR Exam MCQs & Concepts October 2021
Dr. Kareem Alnakeeb
355
61. Which of the following extra-renal abnormalities is most characteristically associated with autosomal
dominant polycystic kidney disease (ADPKD)?
A. Bladder diverticulum
B. Seminal vesicle cysts
C. Coronary artery aneurysm
D. Portal cavernoma
E. Choledochal cyst
Explanation: (by Perplexity AI, generated with the OpenAI O3 model)
• Up to 40% of adults with ADPKD develop cystic dilatation of the seminal vesicles because polycystin
defects affect epithelial-lined ducts beyond the kidneys.
• Bladder diverticula can occur in raised intravesical pressure but have no specific link to ADPKD.
• Coronary aneurysms are more typical of connective-tissue disorders such as Kawasaki disease.
• Portal cavernoma reflects chronic portal vein thrombosis, unrelated to cystic kidney disease.
• Choledochal cysts are congenital bile duct anomalies without proven association with ADPKD.
62. A 28-year-old woman being investigated for primary infertility undergoes hysterosalpingography, which
shows two endometrial canals that diverge superiorly; the intercornual angle is measured at 120° and the
external uterine fundus demonstrates a 10 mm deep cleft. Which Müllerian duct anomaly is most likely?
A. Arcuate uterus
B. Bicornuate uterus
C. Septate uterus
D. Uterus didelphys
E. Unicornuate uterus
Explanation: (by Perplexity AI, generated with the OpenAI O3 model)
• On HSG, a bicornuate uterus shows incomplete fusion of the Müllerian ducts, producing two
endometrial cavities separated by myometrium.
• Key diagnostic features are a wide intercornual angle (>105°) and an external fundal cleft deeper than
1 cm, both present here.
• A septate uterus (C) has a normal or mildly indented fundal contour and an acute intercornual angle
(<75°).
• Uterus didelphys (D) has two completely separate horns with two cervices and usually a vaginal
septum; the external cleft is much deeper.
• An arcuate uterus (A) shows only a shallow (<1 cm) mid-fundal indentation, while a unicornuate
uterus (E) has a single, elongated uterine cavity.
63. Q: Which anatomical variant most commonly predisposes to extra-articular subacromial shoulder
impingement?
A. High coracoacromial ligament insertion
B. Type II curved acromion
C. Type III hooked acromion
D. Os acromiale
E. Prominent coracoid process
Explanation: (by Perplexity AI, generated with the OpenAI O3 model)
• A type III hooked acromion projects inferiorly, narrowing the subacromial space and repeatedly
abrading the supraspinatus tendon, making it the classic variant associated with mechanical (extra-
articular) impingement.
EBDR Exam MCQs & Concepts October 2021
Dr. Kareem Alnakeeb
356
• A type II curved acromion (B) is common but less strongly linked to symptomatic impingement.
• An os acromiale (D) may alter acromial tilt yet is a rarer cause.
• Coracoid variants such as a high coracoacromial ligament insertion (A) or a prominent coracoid
process (E) relate to subcoracoid, not subacromial, impingement.
64. Regarding the MRI enhancement pattern of focal nodular hyperplasia (FNH) in the liver, which of the
following statements is most accurate?
A. The central scar retains gadolinium contrast on delayed phase images.
B. The lesion demonstrates peripheral wash-out on portal venous phase.
C. FNH shows marked diffusion restriction on high b-value DWI.
D. The lesion is T1 hyperintense relative to spleen on pre-contrast images.
E. The enhancing rim persists on equilibrium phase imaging.
Explanation: (by Perplexity AI, generated with the OpenAI O3 model)
• FNH typically enhances rapidly and homogeneously in the arterial phase with a characteristic central
scar that becomes hyperintense on delayed (hepatobiliary) or equilibrium phases because the
fibrous tissue gradually retains gadolinium, making option A correct.
• Unlike hepatocellular carcinoma, FNH does not exhibit peripheral wash-out on portal venous phase,
so option B is wrong.
• Diffusion restriction is usually absent or mild in FNH; marked restriction suggests malignant lesions,
invalidating option C.
• On non-contrast T1-weighted images, FNH is usually iso- or mildly hypo-intense to liver, not frankly
hyperintense as in option D.
• An enhancing rim is typical of hemangioma or metastasis, not FNH, making option E incorrect.
65. Q: Which imaging finding is classically termed the “double-wall” (Rigler) sign on an abdominal
radiograph?
A. Linear lucency outlining both sides of the bowel wall due to intraluminal and extraluminal gas
B. Air-fluid level within the gallbladder lumen
C. Crescent of subdiaphragmatic free gas beneath the right hemidiaphragm
D. Mottled gas within the biliary tree branching towards the liver periphery
E. Multiple step-ladder air-fluid levels within dilated small-bowel loops
Explanation: (by Perplexity AI, generated with the OpenAI O3 model)
• The Rigler sign appears when air is present on each side of the intestinal wall—inside the bowel
lumen and in the peritoneal cavity—producing two parallel lucent lines that outline both serosal and
mucosal surfaces. It is a reliable indicator of pneumoperitoneum.
• Option B describes emphysematous cholecystitis; option C is the classic subdiaphragmatic free-gas
sign but not the double-wall sign; option D represents pneumobilia; option E depicts small-bowel
obstruction, not pneumoperitoneum.
66. In a patient with a suspected bronchogenic carcinoma, which radiographic sign describes an indentation
on the edge of a peripheral lung mass produced by a supplying pulmonary vessel?
A. Cervicothoracic sign
B. Silhouette sign
C. Air crescent sign
D. Pleural tail sign
E. Regel (notch) sign
EBDR Exam MCQs & Concepts October 2021
Dr. Kareem Alnakeeb
357
Explanation: (by Perplexity AI, generated with the OpenAI O3 model)
• The Regel notch sign refers to a small concave indentation on the margin of a peripheral lung tumour
where a feeding pulmonary vessel enters, helping differentiate a vascular-supplied malignant mass
from a non-vascular lesion.
• The cervicothoracic sign (A) localises mediastinal masses above the clavicles, not peripheral
tumours.
• The silhouette sign (B) is loss of normal cardiomediastinal borders due to adjacent opacity.
• The air crescent sign (C) describes a peripheral rim of air around a fungal ball or resolving invasive
aspergillosis.
• The pleural tail sign (D) is a thin linear extension from a subpleural mass toward the pleura, typical
of pleural-based metastasis or mesothelioma, not a vascular notch.
67. A 38-year-old woman with a history seat belt injury in a road traffic accident 1 year ago, presents with a
right breast lump. Mammography shows a ‘hollow’ spherical abnormality measuring about 4 cm with a
rim of thin curvilinear area of calcification in the right breast. What is the most likely diagnosis?
A. Vascular calcification
B. Fat necrosis
C. Secretory calcifications in ducts
D. Milk of calcium
E. Ductal carcinoma in situ
Source: Gupta, Chaitanya. 300 Single Best Answers for the Final FRCR Part A. 1st ed., Jaypee UK,
2010.
Explanation:
• Egg shell’ calcifications are seen in patients with fat necrosis. This can be secondary to blunt trauma
or it can be post-surgical.
68. On musculoskeletal ultrasound of the hands, which finding is considered most characteristic of active
rheumatoid arthritis?
A. Hyperechoic intra-articular loose bodies
B. Anechoic joint effusion without synovial thickening
C. Power-Doppler signal within hypoechoic synovial hypertrophy
D. Peri-tendinous calcific foci along extensor tendons
E. Subcutaneous tophaceous deposits over extensor surfaces
Explanation: (by Perplexity AI, generated with the OpenAI O3 model)
• Doppler-detectable vascular flow inside a hypoechoic, thickened synovium (power-Doppler
positivity) indicates active synovial inflammation and is highly characteristic of active rheumatoid
arthritis, correlating with disease activity and predicting future erosive damage.
• Simple effusion (option B) may be seen in many arthritides and lacks specificity.
• Hyperechoic loose bodies (option A) are typical of osteoarthritis or synovial chondromatosis, not RA.
• Calcific deposits around tendons (option D) suggest calcific tendinopathy.
• Subcutaneous tophi (option E) are a hallmark of gout, not rheumatoid arthritis.
EBDR Exam MCQs & Concepts October 2021
Dr. Kareem Alnakeeb
358
69. Which of the following is the LEAST likely underlying cause of small-bowel intussusception in adults?
A. Post-infective mucosal oedema following bacterial gastroenteritis
B. Ingested foreign body acting as a lead point
C. Small-bowel lymphoma
D. Intramural lipoma
E. Pneumatosis intestinalis
Explanation: (by Perplexity AI, generated with the OpenAI O3 model)
• Adult intussusception almost always has a structural lead point.
• Neoplasms such as lymphoma or benign tumors like lipomas account for most cases because they
disturb peristalsis and drag the proximal segment into the distal lumen.
• Less commonly, an indigestible foreign body can serve as a lead point.
• Bacterial enteritis may cause transient mucosal oedema and hyper-peristalsis sufficient to
telescope the bowel, particularly in children, but it is still a recognized mechanism in adults.
• Pneumatosis intestinalis, however, consists of intramural gas collections and does not usually create
a discrete mass or traction focus, making it an unlikely precipitant of intussusception.
70. During transrectal ultrasonography of the prostate, which sonographic finding is most suggestive of
prostate carcinoma in the peripheral zone?
A. Uniformly isoechoic but markedly hypervascular nodule
B. Well-defined hyperechoic lesion with peripheral flow only
C. Ill-defined hypoechoic area with increased internal Doppler flow
D. Cystic lesion with posterior acoustic enhancement
E. Diffuse heterogeneous enlargement without focal lesion
Explanation: (by Perplexity AI, generated with the OpenAI O3 model)
• Prostate cancer most often arises in the peripheral zone and typically appears as an ill-defined
hypoechoic focus on B-mode imaging because malignant tissue replaces the normally echogenic
glandular stroma. Malignant neovascularity produces chaotic, increased internal Doppler flow,
supporting the diagnosis.
• Isoechoic or hyperechoic nodules (A, B) are less characteristic of carcinoma; hyperechoic foci
usually represent calcification or fibrosis, while isoechoic lesions are difficult to detect
sonographically.
• Cysts with through-transmission (D) indicate benign cystic change.
• Diffuse heterogeneous enlargement without a discrete lesion (E) is more typical of benign prostatic
hyperplasia than carcinoma.
71. According to CT size criteria, lymphadenopathy is defined when which measurement threshold is
exceeded?
A. Lymph node short-axis diameter >10 mm
B. Lymph node short-axis diameter >8 mm
C. Lymph node short-axis diameter >5 mm
D. Lymph node long-axis diameter >15 mm
E. Lymph node volume >1 cm³
Explanation: (by Perplexity AI, generated with the OpenAI O3 model)
• On contrast-enhanced CT, most radiology guidelines label a lymph node as pathologically enlarged
(lymphadenopathy) when its short-axis diameter surpasses 10 mm. This cutoff balances sensitivity
and specificity across nodal stations.
EBDR Exam MCQs & Concepts October 2021
Dr. Kareem Alnakeeb
359
• Nodes with short axes of 5–8 mm (option B and C) are usually considered within normal limits unless
morphologically suspicious.
• Using the long-axis dimension alone (option D) misses many round but enlarged nodes.
• Routine volumetric thresholds (option E) are not part of standard CT reporting criteria.
72. Chron's disease of small intestine:
A. Terminal ileum is affected in 80% of case.
B. Colon is affected in 10% of case.
C. Treated surgically.
D. Presenting with deep penetrating ulcers.
Explanation: (by Perplexity AI, generated with the OpenAI O3 model)
• Terminal ileum involvement ≈ 70-80% ➜ True.
• Colonic disease only ≈ 20% (±50% ileocolonic) ➜ “10% colon” statement is False.
• Management today is medical first (steroids, immunomodulators, biologics). Surgery only for
strictures, fistulae, perforation, bleeding or refractory disease ➜ “treated surgically” is False.
• Transmural inflammation gives deep, fissuring / penetrating ulcers—cobblestone mucosa ➜ True.
73. A 35-year-old man has distal femur pain. Radiographs demonstrate a well-defined, expansile meta-
epiphyseal lytic lesion breaching the cortex; MRI shows uniformly low signal on both T1- and T2-weighted
images. What is the most likely diagnosis?
A. Simple bone cyst
B. Plasma cell myeloma
C. Langerhans cell histiocytosis
D. Giant cell tumor
E. Chondrosarcoma
Explanation: (by Perplexity AI, generated with the OpenAI O3 model)
• Giant cell tumor typically arises in skeletally mature adults (20–40 years), abuts the articular surface
in the meta-epiphysis, appears expansile and well-defined, and may erode the cortex. It
characteristically shows low to intermediate signal on both T1 and T2 MRI because of abundant solid
cellular stroma and hemosiderin, matching this case.
• Simple bone cysts are metaphyseal, centrally located in children and show high T2 signal.
• Plasma cell myeloma is rare in a solitary form at this age and gives high T2 signal.
• Langerhans cell histiocytosis usually affects children and produces poorly marginated lesions with
impermeant cortices.
• Chondrosarcoma presents later in life and shows chondroid matrix with very high T2 signal.
74. A 13 years old boy presented with pain in his right upper thigh for 6 months, mainly at night and relieved
by analgesics. Plain x-ray revealed a small lucent cortical lesion at the upper femoral shaft with
surrounding sclerosis. What is the most likely diagnosis?
A. Non-ossifying fibroma
B. Osteoid osteoma
C. Bone metastasis
D. Brodie’s abscess
E. Ewing sarcoma
Explanation: (by Perplexity AI, generated with the OpenAI O3 model)
• Osteoid osteoma classically affects adolescents, producing nocturnal pain that responds
dramatically to NSAIDs. Imaging typically demonstrates a tiny cortical lucent nidus (<1.5 cm)
surrounded by dense reactive sclerosis in long-bone diaphyses, especially the femur.
EBDR Exam MCQs & Concepts October 2021
Dr. Kareem Alnakeeb
360
• Non-ossifying fibroma appears as an eccentric metaphyseal lesion with a multilobulated outline
and lacks intense pain relief with analgesia.
• Bone metastases are rare at this age and usually multifocal without a characteristic nidus.
• Brodie’s abscess (subacute osteomyelitis) may mimic the lesion but usually presents with a larger,
metaphyseal, oval lucency with a sclerotic rim and often systemic or inflammatory signs.
• Ewing sarcoma produces a permeative pattern with soft-tissue mass rather than a well-defined
nidus.
75. On ultrasound, which gallbladder condition typically shows diffuse wall thickening with intramural
echogenic foci at the neck producing reverberation “comet-tail” artefacts and distal shadowing?
A. Xanthogranulomatous cholecystitis
B. Adenomyomatosis
C. Gallbladder polyps
D. Impacted gallstone
E. Acute calculous cholecystitis
Explanation: (by Perplexity AI, generated with the OpenAI O3 model)
• Adenomyomatosis is a benign hyperplastic condition where Rokitansky–Aschoff sinuses become
filled with cholesterol crystals. The intramural crystals create multiple closely spaced interfaces,
generating characteristic comet-tail reverberation and short acoustic shadowing on ultrasound,
most pronounced in the gallbladder fundus or neck.
• Xanthogranulomatous cholecystitis (A) produces heterogeneous wall thickening but lacks the crisp
comet-tail artefact.
• Gallbladder polyps (C) appear as non-shadowing intraluminal lesions attached to the wall.
• An impacted gallstone (D) casts a clean acoustic shadow rather than a reverberation tail.
• Acute calculous cholecystitis (E) shows wall oedema and gallstones with posterior shadowing but
not the intramural comet-tail pattern.
76. In ultrasound assessment of chronic liver disease, what caudate-to-right-lobe (C/RL) ratio is considered
specific for cirrhosis?
A. More than 0.73
B. Less than 0.55
C. Less than 0.60
D. Less than 0.35
E. More than 1.00
Explanation: (by Perplexity AI, generated with the OpenAI O3 model)
• The caudate lobe enlarges out of proportion to the right lobe in cirrhosis because its drainage is
preserved while the rest of the liver undergoes fibrosis and atrophy; a C/RL ratio >0.73 strongly
suggests cirrhosis and has high specificity.
• Ratios <0.60 (options B and C) or <0.35 (option D) indicate a relatively small caudate lobe and are not
typical of cirrhosis.
• A ratio >1.00 (option E) is possible but far less commonly used as the diagnostic threshold and lacks
supporting evidence.
EBDR Exam MCQs & Concepts October 2021
Dr. Kareem Alnakeeb
361
77. The MOST common site of gastrointestinal stromal tumours (GISTs) is:
A. Esophagus
B. Stomach
C. Small intestine
D. Colon
E. Rectum
Explanation: (by Perplexity AI, generated with the OpenAI O3 model)
• Around 55–60% of GISTs arise in the stomach owing to the high density of interstitial cells of Cajal in
the gastric wall, which are believed to be their cells of origin.
• The small intestine is the next most frequent site (about 30%), while colonic, rectal and esophageal
tumors are uncommon.
• Hence stomach is the single most common location, making options relating to other bowel
segments incorrect.
78. A 2-year-boy presents with a 2-week history of melena culminating in an acute episode of bright red
blood per rectum. Ultrasound was unremarkable. Upper gastrointestinal endoscopy was negative.
Technetium pertechnetate demonstrates increased uptake in the left upper and right lower quadrants.
What is the most likely diagnosis?
A. Acute appendicitis
B. Intussusception
C. Meckel’s diverticulum
D. Gastrinoma
E. Non-specific inflammatory bowel disease
Source: Chin, Teck Yew, et al. Get Through Final FRCR 2A: SBAs. 1st ed., CRC Press, 2017.
Explanation:
Meckel’s Diverticulum and Technetium Pertechnetate Scan Findings
• The findings on this technetium pertechnetate scan are typical of a Meckel’s diverticulum containing
ectopic gastric mucosa.
• Secretions from ectopic gastric tissue in a Meckel’s diverticulum can cause ulceration of the
diverticulum or adjacent small bowel and can lead to bleeding, which, if profuse, can simulate an
upper gastrointestinal bleed.
• Meckel’s diverticulum may simulate acute appendicitis on clinical examination; however, a
pertechnetate scan is only performed if a Meckel’s diverticulum is suspected clinically.
Interpretation of Tracer Uptake
• The uptake in the left upper quadrant should not mislead the radiologist as it is the result of normal
tracer uptake in the stomach mucosa.
Gastrinoma Location
• A gastrinoma is usually located in the pancreatic islet cells, not the right iliac fossa.
EBDR Exam MCQs & Concepts October 2021
Dr. Kareem Alnakeeb
362
79. Which is the most commonly used imaging modality for the diagnosis of gastrointestinal tract (GIT)
carcinoid tumors?
A. CT scan
B. Indium-111 Octreotide scintigraphy
C. SHIAA
D. HIDA scan
E. PET-CT
Explanation: (by Perplexity AI, generated with the OpenAI O3 model)
• Indium-111 Octreotide scintigraphy (OctreoScan) is the investigation of choice for detecting GIT
carcinoid tumors because these neuroendocrine tumors express somatostatin receptors, which the
radiolabeled octreotide binds to with high sensitivity.
• CT scan is widely used for anatomical localization and staging, but is less sensitive for detecting
small or functionally active tumors.
• SHIAA (5-Hydroxyindoleacetic acid) is a urinary metabolite used for biochemical diagnosis, not
localization.
• HIDA scan is used in biliary pathology and is not useful in carcinoid detection.
• PET-CT may be useful but is not the standard method, especially compared to OctreoScan.
80. Which of the following is NOT a recognised radiological sign of intestinal tuberculosis?
E. Fleischner sign
F. String sign
G. Stierlin's sign
H. Comb sign
Explanation: (by Perplexity AI, generated with the OpenAI O3 model)
• The comb sign (engorged mesenteric vessels) is typically seen in Crohn’s disease, not intestinal
tuberculosis.
• Fleischner sign (thickened loop due to spasm), string sign (narrow, rigid segment), and Stierlin's
sign (rapid passage through ulcerated segment with minimal mucosal coating) are all described in
intestinal TB and help differentiate it from other pathologies.
• The comb sign is a classic finding in active Crohn’s, reflecting hypervascularity, and is not associated
with tuberculosis.
81. A 13-year-old post-pubescent girl presents to the emergency department with acute abdominal pain
sited predominantly within the right iliac fossa. An ultrasound scan is performed. This reveals an
echogenic mass within the right side of pelvis measuring approximately 4 cm. The sonographer thinks it is
adjacent to and inseparable from the right ovary. What is the most likely diagnosis?
A. Acute appendicitis
B. Ovarian dermoid
C. Ovarian torsion
D. Ectopic pregnancy
E. Hemorrhagic ovarian cyst
Source: Chin, Teck Yew, et al. Get Through Final FRCR 2A: SBAs. 1st ed., CRC Press, 2017.
Explanation:
Causes of Acute Pelvic Pain in Adolescent Girls
• Acute pelvic pain in adolescent girls is a common problem, but ultrasound scanning is very useful in
differentiating the many possible causes.
Hemorrhagic Ovarian Cysts
EBDR Exam MCQs & Concepts October 2021
Dr. Kareem Alnakeeb
363
• Hemorrhagic ovarian cysts are a common cause of pelvic pain in adolescent girls and appear as an
echogenic mass in relation to the ovary.
Acute Appendicitis
• Acute appendicitis is likely to occur as a blind-ending tubular structure.
• This may appear like a ‘target lesion’ in cross section and there may be fluid/collection adjacent to it.
• An acute appendix should be clearly distinct from the right ovary.
Ovarian Dermoids
• Ovarian dermoids are usually predominantly fat filled and therefore echogenic on ultrasound, but
these tend to be a painless, incidental finding.
Ovarian Torsion
• Ovarian torsion can certainly produce an echogenic mass within the right pelvis, but this is less
common than hemorrhagic cysts and would not appear distinct from the ovary.
Ectopic Pregnancy
• Ectopic pregnancy usually appears as a ‘doughnut’-shaped complex mass in relation to one of the
uterine tubes; a fetal heartbeat may be present.
82. Which of the following statements about plantar fasciitis is correct?
A. The plantar fascia is not normally attached to the calcaneus
B. Plantar fasciitis may be associated with ankylosing spondylitis
C. The presence of a calcaneal spur is diagnostic on X-ray
D. Plantar fasciitis shows high T1 signal around the fascia
E. Plantar fasciitis never causes morning pain
Explanation: (by Perplexity AI, generated with the OpenAI O3 model)
• Plantar fasciitis can be associated with seronegative spondyloarthropathies such as ankylosing
spondylitis, making option B correct.
• Option A is incorrect because the plantar fascia is normally attached to the calcaneus.
• Option C is wrong—calcaneal spurs are often seen on X-ray, but their presence is not specific or
diagnostic, as they can occur in asymptomatic individuals as well.
• Option D is incorrect because plantar fasciitis typically shows low T1 and high T2 signal around the
fascia on MRI, reflecting oedema.
• Option E is incorrect as plantar fasciitis classically causes more pain in the morning or after periods
of rest.
83. The most common site of gastric ulcer is:
A. Greater curvature
B. Lesser curvature
C. Pyloric antrum
D. Gastric fundus
Explanation: (by Perplexity AI, generated with the OpenAI O3 model)
The most common site of gastric ulcer is the lesser curvature of the stomach. Specifically, gastric ulcers are
frequently found along the lesser curve, often near the incisura angularis, which is an area of relative mucosal
weakness.
To clarify the options:
• Greater curvature: Ulcers are less commonly located here.
• Lesser curvature: Most common site for gastric ulcers4.
EBDR Exam MCQs & Concepts October 2021
Dr. Kareem Alnakeeb
364
• Pyloric antrum: While ulcers can occur here, especially in the pyloric channel, it is not the most
common location.
• Gastric fundus: Rare site for gastric ulcers.
Therefore, the correct answer is Lesser curvature.
84. A 42-year-old woman is referred to the breast clinic and is due an ultrasound scan to evaluate a
suspected lump in the breast. All of the following are ultrasonographic features of a benign breast mass,
except
A. Feeding central vessel on Doppler imaging
B. Well-defined smooth margins
C. Three or fewer lobulations
D. Circumferential blood flow pattern on Doppler imaging
E. Uniform hyperechogenicity
Source: Chin, Teck Yew, et al. Get Through Final FRCR 2A: SBAs. 1st ed., CRC Press, 2017.
Explanation:
Ultrasound Features of Benign Lesions
• US features characteristic of benign lesions have been described.
• These include hyperechogenicity compared to fat, an oval or well-defined, lobulated, gently curving
shape and the presence of a thin echogenic pseudocapsule.
• Doppler examination of benign lesions shows displacement of normal vessels around the edge of the
lesion.
Ultrasound Features of Malignant Lesions
• In contrast, malignant lesions show abnormal vessels that are irregular and centrally penetrating.
85. An 85-year-old male presented with abdominal pain and vomiting for 1 week. CT shows a dilated, C-
shaped loop with circumferential wall thickening, increased attenuation, delayed uneven wall
enhancement, and a few air loculi within the bowel walls. What is the most likely diagnosis?
A. Small bowel obstruction
B. Strangulated small bowel
C. Internal hernia
D. Volvulus
Explanation: (by Perplexity AI, generated with the OpenAI O3 model)
• The described CT features—C-shaped dilated loop, circumferential wall thickening, increased
attenuation, delayed uneven wall enhancement, and intramural gas—are classic for strangulated
small bowel. These findings suggest compromised bowel vascularity and evolving ischaemia, rather
than simple obstruction (which typically lacks evident wall ischaemia).
• Volvulus or internal hernia may cause obstruction but usually require additional specific
anatomical features (e.g., whirl sign, mesenteric vessels converging on a point) and do not by
themselves explain the extent of mural changes and pneumatosis described here.
86. Regarding ovarian torsion, which of the following is NOT true?
A. It can occur during early pregnancy.
B. The left ovary is affected more than the right.
C. It has the highest incidence during ovulation induction.
D. It may be due to enlargement of the ovary.
E. It often presents with acute pelvic pain.
EBDR Exam MCQs & Concepts October 2021
Dr. Kareem Alnakeeb
365
Explanation: (by Perplexity AI, generated with the OpenAI O3 model)
• Ovarian torsion most commonly affects the right ovary rather than the left, likely due to differences
in pelvic anatomy and the presence of the sigmoid colon on the left, which restricts mobility.
• It can occur in early pregnancy, particularly because of corpus luteum cysts.
• Ovulation induction (e.g., in fertility treatments) significantly increases the risk due to ovarian
enlargement and increased mobility.
• Any process causing ovarian enlargement—including cysts or tumors—predisposes to torsion.
• Acute pelvic pain is a classic presenting symptom.
• The left ovary being more commonly affected is incorrect.
87. A 60-year-old presents with left groin pain. Ultrasound shows a 2 cm hypoechoic lesion bulging medial to
the epigastric vessels on Valsalva maneuver and absent on rest. What is the most likely diagnosis?
A. Direct inguinal hernia
B. Indirect inguinal hernia
C. Obturator hernia
D. Spigelian hernia
E. Femoral hernia
Source: Gupta, Chaitanya. 300 Single Best Answers for the Final FRCR Part A. 1st ed., Jaypee UK,
2010.
Explanation:
• A direct inguinal hernia is seen medial to the inferior epigastric vessels whereas an indirect hernia is
seen lateral to them.
88. The following are signs of a normal gestational sac, except
A. Intradecidual sign.
B. Cardiac activity seen with a CRL (crown-rump length) of 6 mm.
C. Double decidual sign.
D. Mean sac diameter increases by 1 mm/day.
E. Embryo seen with a mean sac diameter of 10 mm.
Source: Chin, Teck Yew, et al. Get Through Final FRCR 2A: SBAs. 1st ed., CRC Press, 2017.
Explanation:
• The gestational sac is first identifiable on transvaginal ultrasound at 4.5 weeks. It appears as a round
2–3 mm fluid collection. It is located in the central echogenic part of the endometrium (decidua). In
some cases, it is surrounded by two echogenic rings corresponding to the two layers of decidua,
described as the double decidual sac sign of intrauterine pregnancy. Sometimes the gestational sac
is eccentrically located on one side of a thin white line corresponding to the collapsed uterine cavity,
called the intradecidual sign.
• The yolk sac is the first structure visualized on TVS (trans vaginal scan) within the sac at 5.5 weeks.
Yolk sac is evident when sac diameter is 10 mm. Heartbeat is evident when crown–rump length (CRL)
is 5 mm.
• On TVS, an embryo is seen when the mean sac diameter is 18 mm. Mean sac diameter increases by
approximately 1 mm per day. Lack of fetal pole in a gestational sac with diameter more than 20 mm is
suggestive of an anembryonic or nonviable pregnancy.
Summary of key points:
• Gestational sac first visible by TVS: 4.5–5 weeks
• Sac size at first detection: ~2–3 mm round fluid collection
EBDR Exam MCQs & Concepts October 2021
Dr. Kareem Alnakeeb
366
• Double decidual sac sign: two echogenic rings around sac, supporting intrauterine location
• Yolk sac visible at ~5.5 weeks, when sac diameter ~10 mm
• Embryo seen at sac diameter ~18 mm, heartbeat visible when CRL ~5 mm
• Mean sac diameter increases about 1 mm per day
• Absence of fetal pole when sac diameter >20 mm suggests nonviable pregnancy
89. A woman presents with infertility and undergoes a hysterosalpingogram. This demonstrates a uterus with
two converging horns. A wide angle is seen at the roof of the uterus. Which uterine anomaly does the
patient have?
A. Uterine didelphys
B. Septate uterus
C. Arcuate uterus
D. Bicornuate uterus
E. Unicornuate uterus
Source: Chin, Teck Yew, et al. Get Through Final FRCR 2A: SBAs. 1st ed., CRC Press, 2017.
Explanation:
Imaging Modalities for Uterine Anomalies
• While the presence of a divided rather than triangular uterine cavity at Hysterosalpingogram (HSG)
may suggest the presence of a Mullerian duct anomaly (MDA), it is not possible to differentiate
between subtypes.
• MRI and US provide greater anatomic detail; both of these imaging methods provide information on
the external uterine contour, which is an important diagnostic feature of MDAs.
• Furthermore, both MRI and US may be used to assess for concomitant renal anomalies; renal
anomalies occur at a higher rate among MDA patients.
Unicornuate Uterus
• Unicornuate uterus appears as a small, oblong, off-midline structure on US and MRI.
Uterus Didelphys
• Uterus didelphys results from complete failure of Müllerian duct fusion.
• Each duct develops fully with duplication of the uterine horns, cervix and proximal vagina.
• A fundal cleft greater than 1 cm has been reported to be 100% sensitive and specific in differentiation
of fusion anomalies (didelphys and bicornuate) from reabsorption anomalies (septate and arcuate).
Bicornuate Uterus
• Bicornuate uterus involves duplication of the uterus with possible duplication of the cervix
(bicornuate unicollis or bicornuate bicollis).
• HSG demonstrates opacification of two symmetric fusiform uterine cavities (horns) and fallopian
tubes.
• Historically, an intercornual angle of greater than 105° was used for diagnosis of bicornuate uterus.
Septate Uterus
• Septate uterus is the most common form of MDA, accounting for approximately 55% of cases.
• Historically, an angle of less than 75° between the uterine horns has been reported to be suggestive
of a septate rather than bicornuate uterus.
• However, considerable overlap occurs between septate and bicornuate uteri; as such, the angle
measurement is not a reliable diagnostic feature.
Arcuate Uterus
EBDR Exam MCQs & Concepts October 2021
Dr. Kareem Alnakeeb
367
• Arcuate uterus at HSG shows a single uterine cavity with a broad saddle-shaped indentation at the
uterine fundus.
90. An obese 25-year-old man presents with atypical chest pain. Cardiac MR demonstrates asymmetrical
hypertrophy of the interventricular septum, primarily affecting the anteroinferior portion. What is the
most likely diagnosis?
A. Hypertrophic obstructive cardiomyopathy
B. Restrictive cardiomyopathy
C. Myocardial infarction
D. Dilated cardiomyopathy
E. Constrictive pericarditis
Source: Chin, Teck Yew, et al. Get Through Final FRCR 2A: SBAs. 1st ed., CRC Press, 2017.
Explanation:
Definition
• Hypertrophic cardiomyopathy (HCM) is defined as a diffuse or segmental left-ventricular hypertrophy
with a non-dilated and hyperdynamic chamber, in the absence of another cardiac or systemic
disease explaining the degree of cardiac muscle hypertrophy.
Functional Hallmarks and Symptoms
• Dyspnea on exertion is the most common symptom because the key functional hallmark of
hypertrophic cardiomyopathy is an impaired diastolic function with impaired LV filling in the
presence of preserved systolic function.
• Systolic dysfunction occurs at end-stage disease.
Morphological Variants
• Asymmetric involvement of the interventricular septum is the most common form of the disease,
accounting for an estimated 60%–70% of the cases of HCM.
• Other variants include apical, symmetric, midventricular, mass-like and non-contiguous HCM is
typically associated with hypertrophy of the muscle to 15 mm or thicker and a ratio of thickened
myocardium to normal left-ventricular basal myocardium of 1.3–1.5.
Imaging Characteristics
• With MRI and multidetector computed tomography (CT), apical HCM has a characteristic spade-like
configuration of the LV cavity at end diastole, appreciated on vertical long-axis views.
91. A 45-year-old woman presents with a rapidly enlarging mildly painful breast mass over a period of few
months. An urgent ultrasound is performed. The ultrasound shows that the mass measures 7 cm, filling
up almost the entire breast with fluid-filled clefts in the tumor. What is the diagnosis?
A. Inflammatory carcinoma
B. Cystosarcoma phylloides
C. Complex breast cyst
D. Invasive lobular carcinoma
E. Breast lymphoma
Source: Chin, Teck Yew, et al. Get Through Final FRCR 2A: SBAs. 1st ed., CRC Press, 2017.
Explanation:
Overview of Phylloides Tumour (PT)
• Phylloides tumour (PT) is a rare breast fibroepithelial neoplasm.
• It is now generally accepted that PTs can be classified as benign, borderline or malignant.
Imaging Features and Diagnosis Challenges
EBDR Exam MCQs & Concepts October 2021
Dr. Kareem Alnakeeb
368
• Mammography and ultrasound are notorious for their inability to distinguish the benign or malignant
histologic nature of PTs.
• On US, they can be indistinguishable from fibroadenoma.
• They appear as an inhomogeneous, solid-appearing mass.
• A solid mass containing single or multiple, round or cleft-like cystic spaces and demonstrating
posterior acoustic enhancement strongly suggests a diagnosis of PTs.
• Solid components of the tumour show vascularity on Doppler.
MRI Characteristics of PTs
• On MRI, well-defined margins with a round or lobulated shape and a septate inner structure have
been described as characteristic morphologic signs.
• They are usually low on T1-weighted images and vary from low to very high signal on T2-weighted
images.
• Some have described a slit-like pattern on MRIs of benign PTs; these appear as hyperintense slit-like
fluid-filled spaces on T2-weighted images, with a low signal after enhancement.
• Solid areas of the tumor show enhancement with contrast.
92. A fit and healthy 25-year-old woman presents to the breast clinic with a small mobile non-tender breast
lump that she noticed incidentally. An ultrasound is deemed as the first-line investigation; it reveals an
extremely well-defined homogenous, hypoechoic oval mass with posterior acoustic shadowing. What is
the most likely diagnosis?
A. Cystosarcoma phylloides
B. Fibroadenoma
C. Complex breast cyst
D. Invasive lobular carcinoma
E. Fat necrosis
Source: Chin, Teck Yew, et al. Get Through Final FRCR 2A: SBAs. 1st ed., CRC Press, 2017.
Explanation:
Characteristics of Fibroadenomas
• Fibroadenomas are the most common cause of benign solid mass in the breast.
• On US, they appear round or oval, wider than tall, hypoechoic, well-defined and mostly
homogeneous and show a ‘hump and dip’ sign (small focal bulge to the contour with a contiguous
small sulcus), a thin echogenic pseudocapsule and rarely either posterior acoustic enhancement
(17%–25%) or posterior acoustic shadow (9%–11%).
93. A 30-year-old, nulliparous woman with Stein–Leventhal syndrome is being treated for subfertility with
clomiphene. She develops abdominal pain, distension, nausea and vomiting. Ultrasound examination of
the abdomen reveals both ovaries to be larger than 7 cm in length and packed with large follicles, and
also reveals an ovarian cyst 12 cm in diameter. Ascites and a pleural effusion are also seen. What is the
most likely diagnosis?
A. Endometriosis
B. Ovarian Cyst Torsion
C. Ovarian Hyperstimulation Syndrome
D. Ovarian Serous Cystadenoma
E. Corpus Luteum Of Menstruation
EBDR Exam MCQs & Concepts October 2021
Dr. Kareem Alnakeeb
369
Source: Bydder, Megan, et al. Get Through Final FRCR Part A: SBAs for the Modular Examination. 1st
ed., Hodder Education Publishers, 2009.
Explanation:
• Ovarian hyperstimulation syndrome is more commonly seen with human menopausal
gonadotrophin therapy but can also be seen with clomiphene.
• Severe complications relate to volume depletion, such as hypovolaemia, oliguria, electrolyte
imbalance and thromboembolic events. Intra-abdominal haemorrhage is also reported.
94. All of the following statements are true with regard to stress fractures in young athletic children, except
A. Children with limb misalignment are at greater risk.
B. Stress fracture of the femoral neck involves the superior surface.
C. Shin splints show linear oedema limited to the medial tibia.
D. Distal femoral metaphysis are recognised sites for stress fracture.
E. Pars interarticularis fractures are due to repetitive extension and torsion.
Source: Chin, Teck Yew, et al. Get Through Final FRCR 2A: SBAs. 1st ed., CRC Press, 2017.
Explanation:
Causes and Risk Factors of Stress Fractures in Children
• The most common cause of stress fractures is a chronic and repeated workload.
• Children with extremity malalignment or abnormal weight-bearing also are at increased risk.
Common Locations of Stress Fractures in Children
• Typical locations of stress fractures in children include the tibia, fibula, femur and tarsal and
metatarsal bones.
Diagnostic Modality
• MR imaging is currently the best diagnostic modality for stress fractures.
Characteristics of Femur Stress Fractures
• Stress fractures of the femur tend to occur after skeletal maturity and resemble adult injuries,
affecting the inferior surface of the neck, the shaft and the distal metaphysis.
• This injury is most common in endurance athletes, such as runners, triathletes or soccer players but
also occurs in association with abnormal weight-bearing, such as with a coxa vara deformity.
Tibial Stress Fractures in Adolescent Athletes
• The most common site for stress fractures in the adolescent athlete is the tibia.
• Tibial stress fractures occur with activities requiring sudden stops or changes in direction, such as
football, soccer and tennis.
Stress Response and Shin Splints
• Stress also can result in shin splints, which are probably an early stress response secondary to
periosteal traction.
• In cases of stress fractures, MR imaging shows diffuse and irregular bone marrow oedema, whereas
in shin splints, the area of high signal intensity often is more linear and is limited to the medial aspect
of the tibia.
Spondylolysis as a Stress Injury
• Spondylolysis is a stress injury of the pars interarticularis that is due to repetitive extension and
torsion of the trunk.
• Usually occurring in the lower segments of the lumbar spine, stress injuries of the pars
interarticularis have been observed in young female gymnasts, college football players and wrestlers.
EBDR Exam MCQs & Concepts May 2021
Dr. Kareem Alnakeeb
370
May 2021
Paper 1
1. In mandibular trauma, which fracture site most commonly results in inferior alveolar nerve injury?
A. Angle of the mandible
B. Condylar neck
C. Symphysis menti
D. Coronoid process
E. Body of the mandible
Explanation: (by Perplexity AI, generated with the OpenAI O3 model)
• The inferior alveolar nerve runs within the mandibular canal from the mandibular foramen to the
mental foramen. Fractures at the mandibular angle frequently disrupt this canal, making nerve injury
common.
• Condylar neck and coronoid fractures lie superior to the canal, so neuropathy is rare.
• Symphysis fractures involve the midline and typically spare the canal, while body fractures posterior
to the mental foramen are less common sites of nerve damage compared with angle breaks.
2. Which of the following cerebral veins belongs to the deep venous drainage system of the brain?
A. Superficial Sylvian (middle cerebral) vein
B. Vein of Labbe
C. Vein of Trolard
D. Internal cerebral veins
E. Superior sagittal sinus
Explanation: (by Perplexity AI, generated with the OpenAI O3 model)
• The two internal cerebral veins run posteriorly beneath the splenium of the corpus callosum and
unite to form the vein of Galen; they drain the thalami, basal ganglia, internal capsule, and deep
white matter, making them key components of the deep venous system.
• In contrast, the superficial Sylvian vein, vein of Labbe (inferior anastomotic vein), and vein of Trolard
(superior anastomotic vein) are all part of the superficial cortical drainage and empty into dural
sinuses rather than the deep system.
• The superior sagittal sinus is a dural venous sinus, not a cerebral vein; it receives blood from
superficial cortical veins but is not itself classified within either the superficial or deep cerebral
venous groups
3. In advanced rheumatoid arthritis of the cervical spine, which of the following complications is most
characteristically seen?
A. Abscess
B. Osteophytes
C. Atlanto-axial subluxation
D. Calcification
E. Spinal cord cavernoma
Explanation: (by Perplexity AI, generated with the OpenAI O3 model)
• Chronic pannus formation and ligamentous laxity in longstanding rheumatoid arthritis preferentially
weaken the transverse ligament at the atlanto-axial joint, leading to anterior atlanto-axial subluxation
that may compress the upper cervical cord.
EBDR Exam MCQs & Concepts May 2021
Dr. Kareem Alnakeeb
371
• Suppurative abscesses (A) are not a typical direct consequence of rheumatoid disease.
• Osteophyte formation (B) is more typical of osteoarthritis than rheumatoid arthritis.
• Diffuse ligamentous calcification (D) is unrelated to inflammatory pannus.
• Cavernomas (E) are vascular malformations unlinked to rheumatoid pathology.
4. In subacute combined degeneration of the spinal cord due to vitamin B12 deficiency, which spinal tract
classically undergoes the earliest and most pronounced degenerative change?
A. Lateral corticospinal tract
B. Dorsal (posterior) columns
C. Anterior horn cells
D. Posterior horn cells
E. Spinothalamic tracts
Explanation: (by Perplexity AI, generated with the OpenAI O3 model)
• Subacute combined degeneration characteristically produces symmetric demyelination of the
dorsal columns—especially the gracile and cuneate fasciculi—resulting in loss of proprioception and
vibration sense.
• As the disease progresses, degeneration also affects the lateral corticospinal tracts, leading to
spastic paresis, but dorsal column involvement dominates early and is most severe.
• Anterior and posterior horn cells are typically spared, and spinothalamic tracts remain largely intact,
so options C, D and E are incorrect.
• Option A describes a tract affected later rather than primarily.
5. Evaluating the four statements about the prestyloid parapharyngeal space (PPS)
A. T1-weighted MRI without contrast is best to see fat invasion in the prestyloid PPS.
B. The most common lesion in the prestyloid PPS is a schwannoma.
C. Post-styloid compartment equals the alar (carotid) space
D. The prestyloid compartment contains branches of the vagus nerve
Explanation: (by Perplexity AI, generated with the OpenAI O3 model)
• Normal prestyloid PPS is largely fat; on non-contrast T1 it shows high signal that outlines masses.
Loss or streaking of this bright fat is the earliest sign of tumor or infection, so radiologists routinely
review plain T1 images first for fat obliteration
• Most prestyloid tumors arise from salivary tissue (deep-lobe parotid or ectopic minor salivary
glands). Pleomorphic adenoma is the single most frequent lesion, accounting for 40-50% of all PPS
tumors and 80-90% of salivary tumors in the space. Schwannomas dominate the poststyloid
(carotid) compartment, not the prestyloid.
• The tensor-vascular-styloid fascia behind the styloid process separates the ‘true’ prestyloid PPS from
the posterior carotid (alar/poststyloid) space that contains the internal carotid artery, internal jugular
vein, cranial nerves IX-XII and sympathetic chain. Radiology texts often refer to the poststyloid PPS
simply as the carotid space.
• Vagus nerve and its branches travel inside the carotid sheath of the poststyloid compartment. The
prestyloid space contains mainly fat, salivary tissue, small vessels, and minor branches of the
mandibular division of the trigeminal nerve—not vagal branches
EBDR Exam MCQs & Concepts May 2021
Dr. Kareem Alnakeeb
372
6. During endoscopic sinus surgery, which anatomical dehiscence is most frequently implicated when
severe haemorrhage occurs due to direct arterial injury?
A. Lamina papyracea with injury to the olfactory bulb
B. Sphenoid sinus wall with injury to the intracavernous internal carotid artery
C. Cribriform plate with injury to the optic nerve
D. Retained Haller cells causing obstruction of frontal sinus drainage
E. Posterior maxillary wall with injury to the sphenopalatine artery
Explanation: (by Perplexity AI, generated with the OpenAI O3 model)
• The lateral wall of the sphenoid sinus may be thin or dehiscent, placing the adjacent intracavernous
segment of the internal carotid artery at particular risk during functional endoscopic sinus surgery
(FESS).
• Penetration here can lead to catastrophic haemorrhage, making this the most significant
dehiscence-related complication. Lamina papyracea breaches mainly threaten the orbit and medial
rectus, not major arteries; cribriform plate injury causes CSF leak rather than arterial bleeding;
retained Haller cells obstruct drainage but are not a bleeding source; the sphenopalatine artery runs
in the posterior maxillary wall but is smaller and usually controlled endoscopically.
7. In a postpartum woman with a new sellar mass, MRI shows symmetrical, homogeneously enhancing
enlargement of the pituitary gland and thickened stalk; which statement best characterises lymphocytic
hypophysitis?
A. It is easily distinguished from a non-functioning pituitary adenoma on routine MRI.
B. It is most often triggered by direct cranial trauma.
C. It is commonly induced by standard medications unrelated to immunotherapy.
D. It represents an inflamed gland in which the normal anterior pituitary tissue is present but
compressed.
E. It typically spares the infundibulum.
Explanation: (by Perplexity AI, generated with the OpenAI O3 model)
• Lymphocytic hypophysitis is an autoimmune inflammation of the gland; lymphocytic infiltrates
enlarge the pituitary but do not destroy all parenchyma, so normal tissue is usually present and
merely compressed (hence option D is correct).
• Imaging overlap with non-functioning adenoma is considerable—multiple studies stress that routine
MRI cannot reliably differentiate the two (so A is false).
• Reported aetiologies include pregnancy-related autoimmunity and immune-checkpoint inhibitors;
blunt head trauma is not recognised as a typical cause (B is false), and ordinary medications rarely
precipitate it outside of immunotherapy agents (C is false).
• Stalk thickening is a hallmark finding, meaning it is frequently involved rather than spared (E is false).
8. In patients with an aberrant internal carotid artery (ICA) coursing through the middle ear cavity, which
persistent fetal carotid–basilar anastomosis is most frequently associated?
A. Persistent hypoglossal artery
B. Persistent otic artery
C. Persistent stapedial artery
D. Persistent trigeminal artery
E. Persistent dorsal ophthalmic artery
EBDR Exam MCQs & Concepts May 2021
Dr. Kareem Alnakeeb
373
Explanation: (by Perplexity AI, generated with the OpenAI O3 model)
• An aberrant ICA in the temporal bone represents a collateral pathway that substitutes for an absent
or hypoplastic cervical ICA.
• It is most commonly accompanied by a persistent trigeminal artery, the commonest of the carotid-
basilar embryonic connections, because both lesions reflect arrested regression of early vascular
channels within the petrous apex.
• The persistent hypoglossal and otic arteries are far rarer embryonic channels, and the stapedial
artery, although it traverses the middle ear, typically involutes without relation to an aberrant ICA.
• The dorsal ophthalmic artery is an early orbital branch that does not course near the petrous carotid
canal and is not linked to aberrant ICA formation.
9. For women with adequately treated ductal carcinoma in situ (DCIS), what is the approximate breast-
cancer–specific survival at 20 years?
A. 20%
B. 50%
C. 90%
D. 95%
E. 100%
Explanation: (by Perplexity AI, generated with the OpenAI O3 model)
• Large population-based studies tracking >100,000 women show that, after surgery with or without
radiotherapy, DCIS carries an excellent prognosis: breast-cancer–specific mortality remains about 3–
5% at 20 years, equating to roughly 95% survival.
• Option D reflects this figure, making it correct.
• Option C underestimates survival, while A and B dramatically underestimate it, conflicting with
published long-term data.
• Complete (100%) survival (option E) is unrealistic because a small risk of invasive recurrence or
metastasis persists despite optimal management.
10. A 46-year-old woman’s screening mammogram shows new grouped micro-calcifications in the upper
outer quadrant of the right breast; targeted ultrasound is normal. Six-month mammographic surveillance
demonstrates the calcifications are unchanged in morphology and extent, with no associated mass.
What is the most appropriate next step in management?
A. Routine annual screening mammography
B. Surgical excision of the calcified area
C. Digital breast tomosynthesis only
D. Stereotactic-guided vacuum-assisted biopsy
E. Short-term (6-month) mammographic follow-up
Explanation: (by Perplexity AI, generated with the OpenAI O3 model)
• Persistently grouped micro-calcifications that are new at baseline and remain stable after a short-
interval follow-up still carry a clinically significant risk of ductal carcinoma in situ; BI-RADS
recommends moving from Category 3 (probably benign) surveillance to tissue diagnosis if stability is
confirmed but suspicion persists.
• Stereotactic-guided vacuum-assisted biopsy obtains representative samples with minimal morbidity
and is therefore the investigation of choice. Routine annual screening (A) overlooks the need for
definitive diagnosis.
• Surgical excision (B) is more invasive than necessary when percutaneous biopsy suffices.
EBDR Exam MCQs & Concepts May 2021
Dr. Kareem Alnakeeb
374
• Tomosynthesis alone (C) improves lesion conspicuity but does not provide a tissue diagnosis.
• Continued short-term follow-up (E) delays diagnosis without reducing cancer risk.
11. On screening mammography, which feature is considered the classic hallmark of ductal carcinoma in
situ (DCIS)?
A. Microcalcifications
B. A well-defined solid mass
C. Spontaneous bloody nipple discharge
D. Global breast asymmetry
E. Axillary lymph-node enlargement
Explanation: (by Perplexity AI, generated with the OpenAI O3 model)
• DCIS most often manifests as clusters of fine linear or pleomorphic microcalcifications because
malignant ductal cells produce necrotic debris that calcifies; this pattern alerts radiologists to non-
invasive disease before a palpable mass forms.
• Solid masses (option B) are more typical of invasive carcinomas or benign lesions such as
fibroadenomas.
• Nipple discharge (option C) is a clinical symptom that can arise from multiple intraductal processes
and is not pathognomonic on imaging.
• Global asymmetry (option D) is a nonspecific finding seen with many benign conditions, while
axillary nodal enlargement (option E) is uncommon in pure DCIS since the disease is non-invasive
and lacks metastatic potential.
12. A 37-year-old woman presents with spontaneous unilateral bloody nipple discharge. What is the most
common underlying cause?
A. Ductal carcinoma in situ
B. Intraductal papilloma
C. Atypical ductal hyperplasia
D. Duct ectasia
E. Invasive ductal carcinoma
Explanation: (by Perplexity AI, generated with the OpenAI O3 model)
• Intraductal papilloma is the single most frequent lesion causing spontaneous bloody nipple
discharge in women; the vascular stalk of the papilloma is prone to bleeding within a dilated duct.
• Ductal carcinoma in situ and invasive carcinoma are important differentials but together account for
fewer cases than papilloma in this presentation.
• Duct ectasia typically produces multicoloured or green discharge rather than frank blood, while
atypical ductal hyperplasia is generally an incidental microscopic finding and rarely manifests with
discharge.
13. Regarding breast implant rupture on imaging, which statement is FALSE?
A. Peri-implant fluid can suggest extracapsular silicone spread
B. Subcapsular fluid producing the “subcapsular rim sign” indicates intracapsular rupture
C. The“inverted key-hole sign” is typical of intact implants
D. The “salad-oil sign” on ultrasound represents intracapsular rupture
E. A collapsed envelope with “linguine sign” on MRI confirms intracapsular rupture
EBDR Exam MCQs & Concepts May 2021
Dr. Kareem Alnakeeb
375
Explanation: (by Perplexity AI, generated with the OpenAI O3 model)
• Intracapsular rupture traps silicone between the implant shell and fibrous capsule, giving
appearances such as the subcapsular rim (option B), salad-oil or echo-free fluid–silicone layers on
ultrasound (option D), and the classic collapsed shell or “linguine sign” on MRI (option E).
• Peri-implant fluid outside the capsule (option A) suggests extracapsular leakage but may accompany
rupture.
• The “inverted key-hole sign” describes silicone tracking between shell folds after rupture, not an
intact prosthesis, so option C is false.
14. In a woman diagnosed with breast carcinoma during pregnancy, when is adjuvant radiotherapy ideally
delivered to minimise fetal radiation exposure?
A. During the first trimester
B. After the first trimester
C. After the second trimester
D. Post-delivery
E. At any gestational age with abdominal shielding
Explanation: (by Perplexity AI, generated with the OpenAI O3 model)
• Radiotherapy to the breast imparts scattered dose to the uterus that can exceed teratogenic
thresholds, especially in early gestation.
• Because fetal shielding cannot reduce internal scatter from the maternal thorax to safe levels,
standard guidance is to defer breast or chest-wall irradiation until after childbirth, scheduling surgery
and systemic therapy antenatally instead.
• Administering radiotherapy in the first or second trimester risks organogenesis damage or
neurocognitive deficits, while third-trimester exposure still threatens growth and haematopoiesis.
• Hence options A–C are contraindicated. Option E is incorrect because shielding alone is insufficient
for thoracic fields.
15. Regarding male breast carcinoma, which of the following statements is INCORRECT?
A. Tumours are typically oestrogen-receptor negative
B. BRCA2 mutation markedly increases risk
C. A family history of breast cancer is a recognised risk factor
D. A family history of prostate cancer is a recognised risk factor
E. Klinefelter syndrome confers an increased risk
Explanation: (by Perplexity AI, generated with the OpenAI O3 model)
• Most male breast cancers express oestrogen receptors; ER positivity exceeds 80%, guiding the use of
endocrine therapy, so option A is false.
• BRCA2 germ-line mutations (option B) confer a substantially elevated lifetime risk.
• First-degree relatives with breast cancer (option C) double to triple the risk through shared genetics.
• Familial clustering with prostate cancer (option D) reflects overlapping BRCA2 and other DNA-repair
gene mutations, so it is a recognised association. Klinefelter syndrome (option E) raises risk 10-20-
fold due to oestrogen excess.
EBDR Exam MCQs & Concepts May 2021
Dr. Kareem Alnakeeb
376
16. A 40-year-old woman presents with a retro-areolar, ill-defined 8 cm hypoechoic breast mass containing
irregular tubular tracts, with surrounding skin inflammation and discharging sinuses on clinical
examination. What is the most appropriate next management step?
A. Ultrasound-guided aspiration and core biopsy
B. Dynamic contrast-enhanced breast MRI
C. Commence empirical systemic antibiotics and steroids
D. Wide local surgical excision
E. Immediate mastectomy
Explanation: (by Perplexity AI, generated with the OpenAI O3 model)
• The constellation of a large, poorly defined hypoechoic mass with sinus-tract formation in a young
woman is most suggestive of granulomatous mastitis.
• First-line management is image-guided aspiration and core biopsy to confirm the diagnosis and
exclude malignancy or specific infection such as tuberculosis.
• MRI (B) is useful for extension mapping but should follow histological confirmation.
• Empirical medical therapy (C) without tissue diagnosis risks inappropriate treatment.
• Surgical excision (D) or mastectomy (E) is reserved for refractory or complicated cases after biopsy-
proven diagnosis; performing surgery up-front carries unnecessary morbidity.
17. In which scenario does the incidence of a metachronous contralateral breast cancer NOT significantly
rise after treatment of a first breast tumour?
A. Positive first-degree family history of breast cancer
B. Multicentric index tumour in the treated breast
C. Invasive ductal carcinoma as the index tumour
D. Invasive lobular carcinoma as the index tumour
E. BRCA1/2 germline mutation carrier status
Explanation: (by Perplexity AI, generated with the OpenAI O3 model)
• Risk rises notably when there is a strong family history, a multicentric/multifocal index lesion, lobular
histology or an underlying high-penetrance mutation such as BRCA1/2.
• Multiple studies show that invasive lobular carcinoma confers a higher contralateral risk than ductal
carcinoma because of its bilateral propensity, while multicentricity implies a field change affecting
both breasts.
• Family history and BRCA mutations also double to triple the risk.
• By contrast, invasive ductal carcinoma without the above factors behaves as the reference baseline;
its presence alone does not appreciably increase contralateral cancer incidence, so option C is the
exception.
18. In a patient with invasive ductal carcinoma of the breast, which imaging modality is most sensitive for
detecting an extensive intraductal component before surgery?
A. Mammography
B. Ultrasound
C. Magnetic resonance imaging (MRI)
D. PET-CT
E. Digital breast tomosynthesis
Explanation: (by Perplexity AI, generated with the OpenAI O3 model)
• MRI is the most sensitive test for mapping the true intraductal spread of invasive ductal carcinoma
because gadolinium-enhanced sequences highlight both the enhancing invasive focus and
contiguous non-mass ductal enhancement typical of extensive intraductal components.
EBDR Exam MCQs & Concepts May 2021
Dr. Kareem Alnakeeb
377
• Mammography may miss non-calcified ductal disease, and ultrasound is limited to mass-like areas,
often under-estimating intraductal extension.
• PET-CT lacks spatial resolution for small intraductal foci, and tomosynthesis improves detection of
architectural distortion but is still less sensitive than MRI for non-calcified ductal spread.
19. After neoadjuvant chemotherapy for a breast carcinoma, which imaging modality is most sensitive for
assessing tumour response?
A. Breast ultrasound
B. Mammography
C. Dynamic contrast-enhanced breast MRI
D. 18
F-FDG PET-CT
E. Digital breast tomosynthesis
Explanation: (by Perplexity AI, generated with the OpenAI O3 model)
• Dynamic contrast-enhanced breast MRI detects early vascular and cellular changes, allowing the
most accurate measurement of residual tumour size and extent after neoadjuvant therapy, with
pooled sensitivities exceeding 90%.
• PET-CT identifies metabolic response but has lower spatial resolution and variable sensitivity for
small residual lesions.
• Ultrasound is operator-dependent and less reliable for multifocal disease.
• Mammography and tomosynthesis primarily show structural change (e.g. calcification or density
reduction) and underestimate residual viable tumour, giving lower sensitivities than MRI.
20. For which breast cancer scenario is neoadjuvant chemotherapy most appropriately recommended?
A. Small, clearly operable T1 lesion
B. Locally advanced but non-metastatic, initially inoperable carcinoma
C. Widespread metastatic disease at presentation
D. Solitary mass measuring exactly 2 cm without nodal involvement
E. Screen-detected microcalcifications without palpable mass
Explanation: (by Perplexity AI, generated with the OpenAI O3 model)
• Neoadjuvant chemotherapy is primarily indicated when the tumour is too large or locally advanced to
permit immediate surgery but potentially resectable after downsizing; giving systemic therapy first
can convert an inoperable cancer to operable status and allows early treatment of micrometastases.
• Option B describes this classic setting.
• Option A already meets criteria for upfront surgery; neoadjuvant therapy offers no advantage.
• Option C requires systemic therapy for palliation, not for surgical conversion.
• Option D is resectable at diagnosis (T2 ≤5 cm with no adverse factors), so surgery first is preferred.
• Option E represents ductal carcinoma in situ or early invasive disease detected on screening, also
managed surgically before considering adjuvant therapy.
21. In stereotactic-guided core biopsy for a cluster of suspicious breast microcalcifications, which practice
optimises diagnostic accuracy?
A. Obtaining at least 10 core samples
B. Approaching the lesion craniocaudally in every case
C. Sampling only the radiographic centre of the calcification cluster
D. Relying on a single 14-gauge true-cut core
E. Using vacuum aspiration rather than core needles
EBDR Exam MCQs & Concepts May 2021
Dr. Kareem Alnakeeb
378
Explanation: (by Perplexity AI, generated with the OpenAI O3 model)
• Accuracy rises sharply when ≥10 cores are taken with a 14-gauge needle, giving >95% retrieval of
microcalcifications and reducing underestimation of ductal carcinoma in situ.
• A craniocaudal approach (B) is chosen when the lesion is best accessed that way, but lateral
decubitus or caudocranial paths are equally acceptable depending on location.
• Limiting sampling to the cluster centre (C) risks missing adjacent DCIS.
• A single true-cut core (D) lacks sufficient tissue and has high false-negative rates.
• Vacuum devices (E) improve yield but the question specifies core biopsy; when using standard core
needles, increasing core number is the key modifiable factor.
22. In breast cancer surgery, which of the following is NOT an accepted indication for proceeding to formal
axillary lymph-node dissection?
A. Suspicious lymph nodes on pre-operative axillary ultrasound
B. Negative intra-operative sentinel lymph-node biopsy
C. Clinically palpable, hard axillary lymph nodes on examination
D. Proven metastatic involvement on needle biopsy of an axillary node
E. Positive sentinel lymph-node biopsy showing macrometastasis
Explanation: (by Perplexity AI, generated with the OpenAI O3 model)
• Axillary dissection is generally reserved for patients with proven or highly suspected nodal disease.
• It is routinely performed when axillary nodes are clinically abnormal, imaging suggests malignancy,
or needle biopsy confirms metastasis (options A, C, D) and when the sentinel node contains
macrometastasis (option E).
• Conversely, a negative sentinel lymph-node biopsy indicates absence of nodal spread, so
completion axillary dissection is unnecessary; patients instead continue with breast-conserving
surgery or mastectomy plus radiotherapy as appropriate.
23. A 65-year-old man with a 30-pack-year smoking history undergoes chest CT that shows a 5.5 cm right
upper-lobe mass extending into the visceral pleura with an enlarged ipsilateral hilar lymph node. What is
the TNM stage of this lung cancer?
A. T1 N1 M0
B. T2a N1 M0
C. T2b N1 M0
D. T3 N1 M0
E. T4 N1 M0
Explanation: (by Perplexity AI, generated with the OpenAI O3 model)
• Primary tumour: size >5 cm but ≤7 cm and direct invasion of visceral pleura both qualify as T3 under
the 8th edition TNM classification.
• Nodal status: an ipsilateral hilar node is N1.
• No distant metastases are described, so M0.
Hence the correct stage is T3 N1 M0.
• Key distractors:
o T2a and T2b are limited to tumours ≤5 cm (T2a) or >5–7 cm without pleural invasion;
therefore options B and C are undersized.
o T1 applies only to lesions ≤3 cm (option A).
o T4 (option E) requires involvement of mediastinal organs, vertebral body, or separate tumour
nodule in a different lobe, none of which are present.
EBDR Exam MCQs & Concepts May 2021
Dr. Kareem Alnakeeb
379
24. Which of the following is NOT a recognised acquired cause of pulmonary vein stenosis in adults?
A. Mediastinal neoplastic infiltration
B. Blunt or penetrating thoracic trauma
C. Radio-frequency catheter ablation for atrial fibrillation
D. Fibrosing mediastinitis
E. Sarcoidosis
Explanation: (by Perplexity AI, generated with the OpenAI O3 model)
• Radio-frequency catheter ablation is currently the commonest iatrogenic cause of acquired
pulmonary vein stenosis, and mediastinal conditions such as fibrosing mediastinitis, sarcoidosis and
neoplastic infiltration/compression are well-documented non-iatrogenic causes.
• Direct injury from external chest trauma, however, is not described as a mechanism for progressive
pulmonary vein lumen narrowing; trauma may lacerate a vein but does not lead to chronic stenosis,
making option B the incorrect statement.
25. Which of the following is NOT a typical imaging feature of a cerebral hydatid cyst?
A. A clear cyst with no septations or mural enhancement
B. A cyst wall that is calcified
C. Absence of significant surrounding edema
D. Presence of multiple intracranial cysts
E. Fluid signal identical to cerebrospinal fluid on all MRI sequences
Explanation: (by Perplexity AI, generated with the OpenAI O3 model)
• Cerebral hydatid cysts are classically single, thin-walled, spherical lesions whose contents mirror
cerebrospinal fluid on CT/MRI and show no internal septations, mural nodules, or post-contrast
enhancement.
• They usually produce little or no perilesional edema unless infected or ruptured.
• Calcification of the cyst wall is rare in active lesions and generally indicates an involuted or
previously ruptured/treated cyst, making “calcified wall”the feature that does not belong to the
standard criteria.
• Multiple cysts (secondary hydatidosis) can occur after rupture or hematogenous spread but remain
an accepted presentation of the disease; therefore, option D is not the best choice for “except.”
26. Which of the following best describes the key skeletal abnormality in caudal regression syndrome seen
on radiographs?
A. Partial or complete absence of the sacrum
B. Vertebral agenesis above T12, usually incompatible with life
C. Hemivertebrae throughout the thoracic spine
D. Posterior element fusion of cervical vertebrae
E. Lytic destruction of lumbar pedicles
Explanation: (by Perplexity AI, generated with the OpenAI O3 model)
• Caudal regression syndrome is a spectrum of congenital anomalies characterized primarily by partial
or complete absence of the sacrum and, variably, lower lumbar vertebrae.
• This deficiency underlies the classic clinical features such as lower-limb deformities and neurogenic
bladder.
• Vertebral agenesis above T12 is not typical; involvement that high is exceedingly rare and most
affected fetuses are non-viable, making option B incorrect.
• Hemivertebrae (option C) are more suggestive of congenital scoliosis.
EBDR Exam MCQs & Concepts May 2021
Dr. Kareem Alnakeeb
380
• Posterior element fusion of cervical vertebrae (option D) describes Klippel–Feil syndrome.
• Lytic destruction of lumbar pedicles (option E) is characteristic of metastatic or infectious
processes, not a congenital malformation.
27. A 4-year-old child presents with chronic central cyanosis and marked pulmonary plethora on chest
radiograph; which congenital cardiac lesion best explains this combination?
A. Tetralogy of Fallot
B. Patent ductus arteriosus
C. Total anomalous pulmonary venous return (TAPVR)
D. Pulmonary valve stenosis
E. Transposition of the great arteries
Explanation: (by Perplexity AI, generated with the OpenAI O3 model)
• Total anomalous pulmonary venous return produces right-to-left mixing at atrial level,
causing cyanosis, while the anomalous pulmonary veins drain into the systemic venous circulation,
leading to a large left-to-right shunt and pulmonary over-circulation that gives a plethoric
(hypervascular) lung pattern on imaging.
• Tetralogy of Fallot and pulmonary stenosis both show cyanosis but have reduced pulmonary blood
flow, so lungs appear oligaemic.
• Patent ductus arteriosus causes pulmonary plethora but is not typically cyanotic unless
Eisenmenger reversal develops.
• Transposition of the great arteries is cyanotic yet usually shows normal or slightly decreased
pulmonary vascularity unless a large VSD or PDA is present.
28. A 6-year-old girl presents with headache, neck pain and vomiting. MRI shows a cystic lesion in the
posterior fossa with an enhancing wall and a homogeneously, intensely enhancing mural nodule. Which
is the most likely diagnosis?
A. Arachnoid cyst
B. Ependymoma
C. Hemangioblastoma
D. Medulloblastoma
E. Pilocytic astrocytoma
Explanation: (by Perplexity AI, generated with the OpenAI O3 model)
• Pilocytic astrocytoma is the commonest paediatric posterior fossa tumour after medulloblastoma. It
classically appears as a cyst with a vividly enhancing mural nodule; the cyst wall may also enhance.
• Hemangioblastoma can look similar but is rare in children and usually occurs in adults with von
Hippel–Lindau disease.
• Medulloblastomas are typically solid, midline masses that may show heterogeneous rather than
focal mural enhancement.
• Ependymomas often arise from the floor of the fourth ventricle and show mixed solid-cystic
appearance with calcification.
• Arachnoid cysts follow CSF signal on all sequences and do not enhance, so an enhancing nodule
would not be seen.
EBDR Exam MCQs & Concepts May 2021
Dr. Kareem Alnakeeb
381
29. A 6-year-old child presents with a 9 cm abdominal mass seen on ultrasound. Which of the following
findings would favour Wilms tumour over neuroblastoma?
A. Punctate coarse calcifications within the mass
B. Multiple pulmonary metastases on chest radiograph
C. Osteolytic lesions in the femora
D. The mass displaces, rather than encases, the ipsilateral kidney
E. Involvement of the posterior mediastinum on staging MRI
Explanation: (by Perplexity AI, generated with the OpenAI O3 model)
• Wilms tumours typically arise from renal parenchyma and therefore push and displace adjacent
renal tissue and vessels, often forming a pseudocapsule; they rarely encase structures.
• In contrast, neuroblastomas originate from sympathetic tissue, tend to wrap around and encase the
kidney and major vessels.
• Calcification is common in neuroblastoma but far less frequent in Wilms, so option A disfavors
Wilms.
• Pulmonary metastases (option B) occur in both, though more often sought in Wilms staging, so this
is not a distinguishing feature.
• Bone metastases (option C) and posterior mediastinal spread (option E) are characteristic of
neuroblastoma rather than Wilms.
30. Which of the following statements about long-segment Hirschsprung disease is INCORRECT?
A. Skip (segmental) aganglionosis is a recognised feature
B. It shows a marked male predominance
C. It is occasionally associated with neuroblastoma
D. A contrast enema typically demonstrates a reversed rectosigmoid ratio
E. Aganglionosis always begins in the rectum and extends proximally
Explanation: (by Perplexity AI, generated with the OpenAI O3 model)
• Long-segment Hirschsprung disease still begins in the rectum and extends proximally, giving the
classic reversed rectosigmoid ratio on contrast enema because the rectum is narrower than the
dilated sigmoid (Option D true).
• Like all forms of Hirschsprung disease, it is more common in males (≈4:1) (Option B true) and, as a
neurocristopathy, it can coexist with neuroblastoma or other neural-crest tumours (Option C true).
• Aganglionosis is continuous from the distal rectum; true skip lesions are extremely rare and not a
typical feature, so Option A is the exception.
• Option E is correct, reinforcing that the disease always starts distally and spreads proximally.
31. In which arterial locations is clinically significant atherosclerosis most commonly encountered?
A. Suprarenal abdominal aorta
B. Infrarenal abdominal aorta and iliac arteries
C. Branching points such as coronary artery bifurcations
D. Upper-extremity arteries
E. Intracranial arteries
Explanation: (by Perplexity AI, generated with the OpenAI O3 model)
• Turbulent flow and low shear stress at arterial bifurcations promote endothelial injury and lipid
deposition, making branching points—particularly the coronary, carotid and aorto-iliac
bifurcations—the prime sites for clinically significant atherosclerosis.
• The infrarenal abdominal aorta can be involved but less frequently than branch points.
EBDR Exam MCQs & Concepts May 2021
Dr. Kareem Alnakeeb
382
• The suprarenal aorta and upper-extremity arteries are relatively spared because of higher laminar
flow and distinct wall composition.
• Intracranial arteries are affected predominantly by different vasculopathies (e.g. small-vessel
lipohyalinosis) rather than classic atherosclerosis.
32. According to the classic pathological classification, how many histological types of fibromuscular
dysplasia are recognised?
A. 8
B. 6
C. 3
D. 4
E. 5
Explanation: (by Perplexity AI, generated with the OpenAI O3 model)
• Fibromuscular dysplasia (FMD) is historically divided by the arterial wall layer affected into three
distinct histological sub-types: intimal (focal), medial (multifocal) and adventitial (perimedial)
fibroplasia.
• All share the hallmark non-atherosclerotic, non-inflammatory arterial wall thickening but differ in
location of fibroplasia, imaging appearance and clinical behaviour.
• Options suggesting more than three types are incorrect; although several angiographic patterns (e.g.
“string-of-beads”, focal stenosis) are described, they still map back to only these three underlying
pathological categories.
33. Which statement regarding carotico-cavernous fistulae is correct?
A. They are most commonly caused by rupture of a cavernous internal carotid artery aneurysm
B. Fibromuscular dysplasia is the usual underlying aetiology
C. An indirect fistula is best embolised via a trans-arterial approach
D. High-flow fistulae fed by multiple arterial branches are usually treated through a trans-venous
route
E. All types present with a classic “tram-track” sign on MR imaging
Explanation: (by Perplexity AI, generated with the OpenAI O3 model)
• High-flow direct fistulae with multiple arterial feeders often have tortuous arterial access, so
microcatheters are advanced through the inferior petrosal sinus or superior ophthalmic vein for coil
or liquid embolisation, making the trans-venous route the preferred approach.
• Direct carotico-cavernous fistulae are classically due to traumatic rent in the cavernous ICA, not
rupture of an aneurysm (A wrong).
• Fibromuscular dysplasia relates to renal and carotid stenoses, not cavernous fistula formation (B
wrong).
• Indirect (dural) fistulae are low-flow shunts from meningeal branches and are most successfully
occluded via a trans-venous route rather than arterial (C wrong).
• The “tram-track” sign describes optic nerve sheath meningioma, not carotico-cavernous fistulae (E
wrong).
EBDR Exam MCQs & Concepts May 2021
Dr. Kareem Alnakeeb
383
34. Which imaging feature is NOT typically associated with a cerebral cavernous angioma (“cavernoma”) on
MRI?
A. Prominent mass effect caused by surrounding vasogenic edema
B. Complete low-signal rim produced by hemosiderin deposition
C. “Popcorn” appearance from blood products in various stages
D. Lack of significant contrast enhancement
E. Detection is best on susceptibility-weighted or GRE sequences
Explanation: (by Perplexity AI, generated with the OpenAI O3 model)
• Cavernous angiomas are low-flow vascular malformations whose MRI hallmark is a heterogeneous
“popcorn” core of mixed-stage hemorrhage encircled by a complete hypointense hemosiderin rim;
gradient-echo and susceptibility-weighted imaging accentuate this rim, making these sequences
most sensitive (options B, C and E are true).
• They usually show little or no contrast enhancement, further distinguishing them from neoplasms
(option D is true).
• In the absence of an acute, large hemorrhage, cavernomas exert minimal mass effect or surrounding
vasogenic edema; therefore prominent edema-related mass effect is uncharacteristic, making
option A the false statement.
35. In a patient with a suspected cerebral arteriovenous malformation (AVM), which imaging feature is most
characteristic and therefore most helpful in making the diagnosis?
A. Catheter angiography is superior because it always identifies the nidus
B. A serpiginous “bag-of-worms” appearance is typically demonstrated on contrast-enhanced CT
C. MRI is the most reliable technique for detecting the nidus in all cases
D. Focal arterial stenosis immediately proximal to the nidus is virtually diagnostic
E. CT perfusion invariably shows increased cerebral blood flow in the lesion
Explanation: (by Perplexity AI, generated with the OpenAI O3 model)
• Cerebral AVMs comprise a tangle of abnormal vessels (“nidus”) producing multiple tortuous,
serpiginous channels that enhance intensely with contrast, giving the classic “bag-of-worms”
appearance on CT or MR angiography.
• This striking vascular pattern (Option B) is often the first clue on cross-sectional imaging.
• While digital subtraction angiography (DSA) remains the gold standard for detailed angio-
architecture, it can occasionally miss very small niduses and is invasive, so it is not “always” superior
(Option A wrong).
• High-resolution MRI can depict flow voids but may fail to show the entire nidus in fast-flow lesions
(Option C overstates accuracy).
• Proximal arterial stenosis is not a specific feature and may be absent (Option D wrong).
• CT perfusion may demonstrate high flow but this is neither universal nor specific (Option E wrong).
36. Which of the following conditions is LEAST likely to cause long-segment narrowing of the pulmonary
artery?
A. Takayasu arteritis
B. Behçet disease
C. Idiopathic pulmonary fibrosis
D. Primary pulmonary artery sarcoma
E. Tuberculous mediastinitis
EBDR Exam MCQs & Concepts May 2021
Dr. Kareem Alnakeeb
384
Explanation: (by Perplexity AI, generated with the OpenAI O3 model)
• Takayasu arteritis and Behçet disease are large-vessel vasculitides that can produce concentric,
long-segment stenoses of the main and branch pulmonary arteries.
• Primary pulmonary artery sarcoma typically manifests as an intraluminal mass but may also lead to
extensive segmental narrowing as it infiltrates longitudinally.
• Chronic fibrosing mediastinitis due to tuberculosis can encase and constrict the pulmonary arteries
over several centimetres.
• In contrast, idiopathic pulmonary fibrosis primarily affects lung parenchyma with peripheral, basilar
interstitial fibrosis; it does not directly involve or narrow the pulmonary arterial lumen, so long-
segment pulmonary artery stenosis is not characteristic.
37. On contrast-enhanced MRI of an asymptomatic adult, physiological enhancement of the facial nerve is
most consistently seen in which intratemporal segment?
A. Cisternal (pontine) segment
B. Parotid (extratemporal) segment
C. Tympanic (horizontal) segment
D. Geniculate ganglion
E. Internal auditory canal (canalicular) segment
Explanation: (by Perplexity AI, generated with the OpenAI O3 model)
• The facial nerve possesses a rich circumneural arteriovenous plexus. Normal gadolinium uptake is
therefore common within the facial canal, but the geniculate ganglion enhances in almost every
healthy individual, reported in 96–100% of nerves on modern 3D T1-weighted sequences.
• By contrast, the tympanic and mastoid segments enhance less frequently, while enhancement of the
cisternal, parotid and canalicular segments is usually absent or only mild and inconsistent.
• Hence, symmetrical bright post-contrast signal at the geniculate ganglion is regarded as
physiological; marked enhancement elsewhere should prompt a search for pathology.
38. A 35-year-old woman presents to the breast clinic with unilateral, non-bloody nipple discharge and
normal mammography. What is the most common benign pathology responsible for this presentation?
A. Ductal carcinoma in situ
B. Intraductal papilloma
C. Invasive ductal carcinoma
D. Mammary duct ectasia
E. Fibroadenoma
Explanation: (by Perplexity AI, generated with the OpenAI O3 model)
• The leading cause of spontaneous, unilateral, serous or serosanguinous nipple discharge in women
with otherwise normal imaging is a solitary intraductal papilloma.
• Papillomas arise within a large subareolar duct and frequently cause intermittent discharge when
they bleed or obstruct the duct.
• Ductal carcinoma in situ can also present with discharge but is far less common in this clinical
context; its hallmark on imaging is microcalcification rather than an isolated discharge episode.
• Invasive ductal carcinoma is a rarer cause and would typically present with a mass.
• Mammary duct ectasia usually produces bilateral, multicoloured discharge in perimenopausal
women, not a single-duct, serous pattern.
• Fibroadenoma does not cause nipple discharge.
EBDR Exam MCQs & Concepts May 2021
Dr. Kareem Alnakeeb
385
39. Which of the following best explains the diffuse smooth pachymeningeal enhancement seen on post-
contrast brain MRI in a patient with spontaneous intracranial hypotension?
A. Venous engorgement secondary to low CSF pressure
B. Congenital dural developmental anomaly
C. Post-operative change after lumbar puncture
D. Leptomeningeal carcinomatosis
E. Hypertrophic pachymeningitis
Explanation: (by Perplexity AI, generated with the OpenAI O3 model)
• Intracranial hypotension lowers CSF volume, producing compensatory dilation of dural venous
sinuses and epidural veins.
• The resulting venous engorgement increases dural blood volume, leading to the characteristic
smooth, diffuse pachymeningeal gadolinium enhancement.
• Congenital dural anomalies are unrelated to acquired low-pressure states, postoperative lumbar
puncture changes are typically focal and resolve quickly, leptomeningeal carcinomatosis enhances
the pia–arachnoid rather than the dura, and hypertrophic pachymeningitis causes nodular or
irregular dural thickening, not the smooth enhancement pattern of hypotension.
40. In thyroid eye disease, which extra-ocular muscle is most frequently affected leading to restricted eye
movement and diplopia?
A. Inferior rectus
B. Lateral rectus
C. Medial rectus
D. Superior rectus
E. Superior oblique
Explanation: (by Perplexity AI, generated with the OpenAI O3 model)
• The inferior rectus is most commonly involved in thyroid eye disease because its tight
compartmental space within the orbit and rich glycosaminoglycan-producing fibroblasts make it
particularly susceptible to autoimmune-mediated inflammation and fibrosis.
• This results in limited up-gaze and vertical diplopia. The medial rectus is the second most frequently
affected but less common than the inferior rectus.
• The superior rectus, lateral rectus and superior oblique are involved far less often, so choosing them
would fail to reflect the typical pattern of extra-ocular muscle involvement seen on CT or MRI.
41. A 47-year-old man presents with chronic burning low-back pain and bilateral leg paraesthesia 18 months
after a lumbar discectomy; MRI shows clumped cauda equina roots within an “empty” thecal sac. What
is the single most common precipitating cause of spinal arachnoiditis in current clinical practice?
A. Intrathecal oil-based contrast (myelography)
B. Tuberculous meningitis
C. Lumbar spine surgery
D. Epidural steroid injection
E. Subarachnoid haemorrhage
Explanation: (by Perplexity AI, generated with the OpenAI O3 model)
• The leading trigger for adhesive arachnoiditis today is mechanical irritation from previous lumbar
spine operations; scar tissue and inflammatory exudate develop after surgical manipulation,
producing the characteristic root clumping and chronic neuropathic pain seen on imaging.
EBDR Exam MCQs & Concepts May 2021
Dr. Kareem Alnakeeb
386
• Historical causes such as oil-based myelographic contrast (A) have become rare since water-soluble
agents replaced them.
• Infective meningitis (B) and subarachnoid haemorrhage (E) are recognised aetiologies but
collectively account for fewer cases than postoperative change.
• Chemical irritation from epidural steroids (D) is a documented but less frequent iatrogenic cause.
42. A 40-year-old non-smoker presents with recurrent haemoptysis. CT thorax shows a 2.5 cm vividly
enhancing endobronchial mass in the right main bronchus, situated 2.5 cm distal to the carina,
containing punctate calcification. What is the most likely diagnosis?
A. Inflammatory polyp
B. Endobronchial lipoma
C. Endobronchial sarcoid granuloma
D. Typical bronchial carcinoid
E. Squamous cell carcinoma
Explanation: (by Perplexity AI, generated with the OpenAI O3 model)
• Typical bronchial carcinoids arise centrally, often within a main bronchus, show intense
homogeneous contrast enhancement due to vascularity, and frequently contain speckled or
“popcorn” calcification—features that match this case.
• Inflammatory polyps are usually small, minimally enhancing soft-tissue nodules without marked
vascularity.
• Endobronchial lipomas demonstrate fat attenuation, not vivid enhancement.
• Sarcoid granulomas are rare endobronchial lesions and typically non-enhancing with associated
perilymphatic nodules elsewhere.
• Squamous cell carcinomas may cavitate or invade bronchial wall but enhance less avidly and calcify
only if post-therapeutic or dystrophic.
43. A 55-year-old woman with a 20-year history of insulin-dependent diabetes and hypertension presents
with multiple firm, painless masses in both upper outer breast quadrants. Mammography shows
symmetrically dense glandular tissue without discrete spiculated lesions. Targeted ultrasound
demonstrates bilateral, irregular, markedly hypoechoic masses with pronounced posterior acoustic
shadowing. MRI reveals slow, minimal contrast enhancement of these lesions. What is the most likely
diagnosis?
A. Lobular carcinoma
B. Invasive ductal carcinoma
C. Stromal fibrosis
D. Diabetic mastopathy
E. Fat necrosis
Explanation: (by Perplexity AI, generated with the OpenAI O3 model)
• Diabetic mastopathy classically affects long-standing insulin-dependent diabetics, producing
multiple bilateral, ill-defined, hypoechoic breast masses with a striking posterior shadow and little or
no enhancement on MRI—features that match this case.
• Malignancies such as lobular (A) or invasive ductal carcinoma (B) usually show unilateral spiculated
or irregular masses with early, avid enhancement.
• Stromal fibrosis (C) can mimic malignancy but usually presents as a single lesion and is not strongly
linked to diabetes.
EBDR Exam MCQs & Concepts May 2021
Dr. Kareem Alnakeeb
387
• Fat necrosis (E) typically follows trauma or surgery and often shows oil cysts or calcifications, which
are absent here.
44. In MRI assessment of a suspected middle-ear cholesteatoma, which imaging pattern most reliably
confirms the diagnosis?
A. Central enhancement with restricted diffusion
B. Central enhancement with no restricted diffusion
C. No central enhancement with no restricted diffusion
D. No central enhancement with restricted diffusion
E. Peripheral enhancement with T1 hyperintensity
Explanation: (by Perplexity AI, generated with the OpenAI O3 model)
• Cholesteatoma consists of keratin debris within a sac; on MRI it is typically non-vascular, so it shows
no post-gadolinium enhancement.
• The keratin content causes markedly restricted diffusion, giving high signal on DWI and low ADC
values, making “no central enhancement with restricted diffusion” the hallmark appearance.
• An enhancing lesion (Options A & B) suggests granulation tissue or tumour.
• Lack of both enhancement and diffusion restriction (Option C) fits simple fluid or serous otitis.
• Peripheral enhancement with T1 hyperintensity (Option E) is more consistent with cholesterol
granuloma or haemorrhagic fluid.
45. A 39-year-old man with a 10-year history of heavy smoking presents with ischaemic changes in his left
hand; catheter angiography demonstrates long-segment occlusion of the infra-popliteal tibial arteries on
the right. What is the most likely underlying diagnosis?
A. Diabetes mellitus
B. Buerger disease (thromboangiitis obliterans)
C. Ehlers-Danlos syndrome
D. Systemic sclerosis (scleroderma)
E. Takayasu arteritis
Explanation: (by Perplexity AI, generated with the OpenAI O3 model)
• Buerger disease classically affects male smokers younger than 45 years and causes segmental, non-
atherosclerotic occlusion of medium- and small-calibre arteries in the extremities, leading to distal
limb ischaemia visible on angiography as long, smooth tibial occlusions.
• Diabetes mellitus more often produces tibial occlusive disease but typically in older patients and is
usually accompanied by microvascular changes elsewhere.
• Ehlers-Danlos syndrome causes arterial fragility and aneurysms rather than segmental occlusion.
• Systemic sclerosis results in digital ischemia from vasospasm and intimal proliferation but does not
usually create long tibial artery occlusions.
• Takayasu arteritis targets the aorta and its major branches in young women, not the distal tibial
vessels.
EBDR Exam MCQs & Concepts May 2021
Dr. Kareem Alnakeeb
388
46. A 60-year-old heavy smoker presents with digital clubbing, hypertrophic osteoarthropathy,
hypercalcaemia and cerebellar ataxia; CT shows a central hilar lung mass. What is the most likely
underlying lung cancer?
A. Adenocarcinoma
B. Small cell carcinoma
C. Large cell carcinoma
D. Squamous cell carcinoma
E. Bronchioloalveolar carcinoma
Explanation: (by Perplexity AI, generated with the OpenAI O3 model)
• Squamous cell carcinoma is typically central/hilar, strongly linked to long-term smoking and is the
lung cancer subtype most often associated with paraneoplastic hypercalcaemia (via ectopic PTH-rP)
and skeletal manifestations such as hypertrophic osteoarthropathy and digital clubbing.
• Small cell carcinoma is
Egyptian Board of Diagnostic Radiology – Part 2 QBank - QA + Explanations, 1E (2025)
Egyptian Board of Diagnostic Radiology – Part 2 QBank - QA + Explanations, 1E (2025)
Egyptian Board of Diagnostic Radiology – Part 2 QBank - QA + Explanations, 1E (2025)
Egyptian Board of Diagnostic Radiology – Part 2 QBank - QA + Explanations, 1E (2025)
Egyptian Board of Diagnostic Radiology – Part 2 QBank - QA + Explanations, 1E (2025)
Egyptian Board of Diagnostic Radiology – Part 2 QBank - QA + Explanations, 1E (2025)
Egyptian Board of Diagnostic Radiology – Part 2 QBank - QA + Explanations, 1E (2025)
Egyptian Board of Diagnostic Radiology – Part 2 QBank - QA + Explanations, 1E (2025)
Egyptian Board of Diagnostic Radiology – Part 2 QBank - QA + Explanations, 1E (2025)
Egyptian Board of Diagnostic Radiology – Part 2 QBank - QA + Explanations, 1E (2025)
Egyptian Board of Diagnostic Radiology – Part 2 QBank - QA + Explanations, 1E (2025)
Egyptian Board of Diagnostic Radiology – Part 2 QBank - QA + Explanations, 1E (2025)
Egyptian Board of Diagnostic Radiology – Part 2 QBank - QA + Explanations, 1E (2025)
Egyptian Board of Diagnostic Radiology – Part 2 QBank - QA + Explanations, 1E (2025)
Egyptian Board of Diagnostic Radiology – Part 2 QBank - QA + Explanations, 1E (2025)
Egyptian Board of Diagnostic Radiology – Part 2 QBank - QA + Explanations, 1E (2025)
Egyptian Board of Diagnostic Radiology – Part 2 QBank - QA + Explanations, 1E (2025)
Egyptian Board of Diagnostic Radiology – Part 2 QBank - QA + Explanations, 1E (2025)
Egyptian Board of Diagnostic Radiology – Part 2 QBank - QA + Explanations, 1E (2025)
Egyptian Board of Diagnostic Radiology – Part 2 QBank - QA + Explanations, 1E (2025)
Egyptian Board of Diagnostic Radiology – Part 2 QBank - QA + Explanations, 1E (2025)
Egyptian Board of Diagnostic Radiology – Part 2 QBank - QA + Explanations, 1E (2025)
Egyptian Board of Diagnostic Radiology – Part 2 QBank - QA + Explanations, 1E (2025)
Egyptian Board of Diagnostic Radiology – Part 2 QBank - QA + Explanations, 1E (2025)
Egyptian Board of Diagnostic Radiology – Part 2 QBank - QA + Explanations, 1E (2025)
Egyptian Board of Diagnostic Radiology – Part 2 QBank - QA + Explanations, 1E (2025)
Egyptian Board of Diagnostic Radiology – Part 2 QBank - QA + Explanations, 1E (2025)
Egyptian Board of Diagnostic Radiology – Part 2 QBank - QA + Explanations, 1E (2025)
Egyptian Board of Diagnostic Radiology – Part 2 QBank - QA + Explanations, 1E (2025)
Egyptian Board of Diagnostic Radiology – Part 2 QBank - QA + Explanations, 1E (2025)
Egyptian Board of Diagnostic Radiology – Part 2 QBank - QA + Explanations, 1E (2025)
Egyptian Board of Diagnostic Radiology – Part 2 QBank - QA + Explanations, 1E (2025)
Egyptian Board of Diagnostic Radiology – Part 2 QBank - QA + Explanations, 1E (2025)
Egyptian Board of Diagnostic Radiology – Part 2 QBank - QA + Explanations, 1E (2025)
Egyptian Board of Diagnostic Radiology – Part 2 QBank - QA + Explanations, 1E (2025)
Egyptian Board of Diagnostic Radiology – Part 2 QBank - QA + Explanations, 1E (2025)
Egyptian Board of Diagnostic Radiology – Part 2 QBank - QA + Explanations, 1E (2025)
Egyptian Board of Diagnostic Radiology – Part 2 QBank - QA + Explanations, 1E (2025)
Egyptian Board of Diagnostic Radiology – Part 2 QBank - QA + Explanations, 1E (2025)
Egyptian Board of Diagnostic Radiology – Part 2 QBank - QA + Explanations, 1E (2025)
Egyptian Board of Diagnostic Radiology – Part 2 QBank - QA + Explanations, 1E (2025)
Egyptian Board of Diagnostic Radiology – Part 2 QBank - QA + Explanations, 1E (2025)
Egyptian Board of Diagnostic Radiology – Part 2 QBank - QA + Explanations, 1E (2025)
Egyptian Board of Diagnostic Radiology – Part 2 QBank - QA + Explanations, 1E (2025)
Egyptian Board of Diagnostic Radiology – Part 2 QBank - QA + Explanations, 1E (2025)
Egyptian Board of Diagnostic Radiology – Part 2 QBank - QA + Explanations, 1E (2025)
Egyptian Board of Diagnostic Radiology – Part 2 QBank - QA + Explanations, 1E (2025)
Egyptian Board of Diagnostic Radiology – Part 2 QBank - QA + Explanations, 1E (2025)
Egyptian Board of Diagnostic Radiology – Part 2 QBank - QA + Explanations, 1E (2025)
Egyptian Board of Diagnostic Radiology – Part 2 QBank - QA + Explanations, 1E (2025)
Egyptian Board of Diagnostic Radiology – Part 2 QBank - QA + Explanations, 1E (2025)
Egyptian Board of Diagnostic Radiology – Part 2 QBank - QA + Explanations, 1E (2025)
Egyptian Board of Diagnostic Radiology – Part 2 QBank - QA + Explanations, 1E (2025)
Egyptian Board of Diagnostic Radiology – Part 2 QBank - QA + Explanations, 1E (2025)
Egyptian Board of Diagnostic Radiology – Part 2 QBank - QA + Explanations, 1E (2025)
Egyptian Board of Diagnostic Radiology – Part 2 QBank - QA + Explanations, 1E (2025)
Egyptian Board of Diagnostic Radiology – Part 2 QBank - QA + Explanations, 1E (2025)
Egyptian Board of Diagnostic Radiology – Part 2 QBank - QA + Explanations, 1E (2025)
Egyptian Board of Diagnostic Radiology – Part 2 QBank - QA + Explanations, 1E (2025)
Egyptian Board of Diagnostic Radiology – Part 2 QBank - QA + Explanations, 1E (2025)
Egyptian Board of Diagnostic Radiology – Part 2 QBank - QA + Explanations, 1E (2025)
Egyptian Board of Diagnostic Radiology – Part 2 QBank - QA + Explanations, 1E (2025)
Egyptian Board of Diagnostic Radiology – Part 2 QBank - QA + Explanations, 1E (2025)
Egyptian Board of Diagnostic Radiology – Part 2 QBank - QA + Explanations, 1E (2025)
Egyptian Board of Diagnostic Radiology – Part 2 QBank - QA + Explanations, 1E (2025)
Egyptian Board of Diagnostic Radiology – Part 2 QBank - QA + Explanations, 1E (2025)
Egyptian Board of Diagnostic Radiology – Part 2 QBank - QA + Explanations, 1E (2025)
Egyptian Board of Diagnostic Radiology – Part 2 QBank - QA + Explanations, 1E (2025)
Egyptian Board of Diagnostic Radiology – Part 2 QBank - QA + Explanations, 1E (2025)
Egyptian Board of Diagnostic Radiology – Part 2 QBank - QA + Explanations, 1E (2025)
Egyptian Board of Diagnostic Radiology – Part 2 QBank - QA + Explanations, 1E (2025)
Egyptian Board of Diagnostic Radiology – Part 2 QBank - QA + Explanations, 1E (2025)
Egyptian Board of Diagnostic Radiology – Part 2 QBank - QA + Explanations, 1E (2025)
Egyptian Board of Diagnostic Radiology – Part 2 QBank - QA + Explanations, 1E (2025)
Egyptian Board of Diagnostic Radiology – Part 2 QBank - QA + Explanations, 1E (2025)
Egyptian Board of Diagnostic Radiology – Part 2 QBank - QA + Explanations, 1E (2025)
Egyptian Board of Diagnostic Radiology – Part 2 QBank - QA + Explanations, 1E (2025)
Egyptian Board of Diagnostic Radiology – Part 2 QBank - QA + Explanations, 1E (2025)
Egyptian Board of Diagnostic Radiology – Part 2 QBank - QA + Explanations, 1E (2025)
Egyptian Board of Diagnostic Radiology – Part 2 QBank - QA + Explanations, 1E (2025)
Egyptian Board of Diagnostic Radiology – Part 2 QBank - QA + Explanations, 1E (2025)
Egyptian Board of Diagnostic Radiology – Part 2 QBank - QA + Explanations, 1E (2025)
Egyptian Board of Diagnostic Radiology – Part 2 QBank - QA + Explanations, 1E (2025)
Egyptian Board of Diagnostic Radiology – Part 2 QBank - QA + Explanations, 1E (2025)
Egyptian Board of Diagnostic Radiology – Part 2 QBank - QA + Explanations, 1E (2025)
Egyptian Board of Diagnostic Radiology – Part 2 QBank - QA + Explanations, 1E (2025)
Egyptian Board of Diagnostic Radiology – Part 2 QBank - QA + Explanations, 1E (2025)
Egyptian Board of Diagnostic Radiology – Part 2 QBank - QA + Explanations, 1E (2025)
Egyptian Board of Diagnostic Radiology – Part 2 QBank - QA + Explanations, 1E (2025)
Egyptian Board of Diagnostic Radiology – Part 2 QBank - QA + Explanations, 1E (2025)
Egyptian Board of Diagnostic Radiology – Part 2 QBank - QA + Explanations, 1E (2025)
Egyptian Board of Diagnostic Radiology – Part 2 QBank - QA + Explanations, 1E (2025)
Egyptian Board of Diagnostic Radiology – Part 2 QBank - QA + Explanations, 1E (2025)
Egyptian Board of Diagnostic Radiology – Part 2 QBank - QA + Explanations, 1E (2025)
Egyptian Board of Diagnostic Radiology – Part 2 QBank - QA + Explanations, 1E (2025)
Egyptian Board of Diagnostic Radiology – Part 2 QBank - QA + Explanations, 1E (2025)
Egyptian Board of Diagnostic Radiology – Part 2 QBank - QA + Explanations, 1E (2025)
Egyptian Board of Diagnostic Radiology – Part 2 QBank - QA + Explanations, 1E (2025)
Egyptian Board of Diagnostic Radiology – Part 2 QBank - QA + Explanations, 1E (2025)
Egyptian Board of Diagnostic Radiology – Part 2 QBank - QA + Explanations, 1E (2025)
Egyptian Board of Diagnostic Radiology – Part 2 QBank - QA + Explanations, 1E (2025)
Egyptian Board of Diagnostic Radiology – Part 2 QBank - QA + Explanations, 1E (2025)
Egyptian Board of Diagnostic Radiology – Part 2 QBank - QA + Explanations, 1E (2025)
Egyptian Board of Diagnostic Radiology – Part 2 QBank - QA + Explanations, 1E (2025)
Egyptian Board of Diagnostic Radiology – Part 2 QBank - QA + Explanations, 1E (2025)
Egyptian Board of Diagnostic Radiology – Part 2 QBank - QA + Explanations, 1E (2025)
Egyptian Board of Diagnostic Radiology – Part 2 QBank - QA + Explanations, 1E (2025)
Egyptian Board of Diagnostic Radiology – Part 2 QBank - QA + Explanations, 1E (2025)
Egyptian Board of Diagnostic Radiology – Part 2 QBank - QA + Explanations, 1E (2025)
Egyptian Board of Diagnostic Radiology – Part 2 QBank - QA + Explanations, 1E (2025)
Egyptian Board of Diagnostic Radiology – Part 2 QBank - QA + Explanations, 1E (2025)
Egyptian Board of Diagnostic Radiology – Part 2 QBank - QA + Explanations, 1E (2025)
Egyptian Board of Diagnostic Radiology – Part 2 QBank - QA + Explanations, 1E (2025)
Egyptian Board of Diagnostic Radiology – Part 2 QBank - QA + Explanations, 1E (2025)
Egyptian Board of Diagnostic Radiology – Part 2 QBank - QA + Explanations, 1E (2025)
Egyptian Board of Diagnostic Radiology – Part 2 QBank - QA + Explanations, 1E (2025)
Egyptian Board of Diagnostic Radiology – Part 2 QBank - QA + Explanations, 1E (2025)
Egyptian Board of Diagnostic Radiology – Part 2 QBank - QA + Explanations, 1E (2025)
Egyptian Board of Diagnostic Radiology – Part 2 QBank - QA + Explanations, 1E (2025)
Egyptian Board of Diagnostic Radiology – Part 2 QBank - QA + Explanations, 1E (2025)
Egyptian Board of Diagnostic Radiology – Part 2 QBank - QA + Explanations, 1E (2025)
Egyptian Board of Diagnostic Radiology – Part 2 QBank - QA + Explanations, 1E (2025)
Egyptian Board of Diagnostic Radiology – Part 2 QBank - QA + Explanations, 1E (2025)
Egyptian Board of Diagnostic Radiology – Part 2 QBank - QA + Explanations, 1E (2025)
Egyptian Board of Diagnostic Radiology – Part 2 QBank - QA + Explanations, 1E (2025)
Egyptian Board of Diagnostic Radiology – Part 2 QBank - QA + Explanations, 1E (2025)
Egyptian Board of Diagnostic Radiology – Part 2 QBank - QA + Explanations, 1E (2025)
Egyptian Board of Diagnostic Radiology – Part 2 QBank - QA + Explanations, 1E (2025)
Egyptian Board of Diagnostic Radiology – Part 2 QBank - QA + Explanations, 1E (2025)
Egyptian Board of Diagnostic Radiology – Part 2 QBank - QA + Explanations, 1E (2025)
Egyptian Board of Diagnostic Radiology – Part 2 QBank - QA + Explanations, 1E (2025)
Egyptian Board of Diagnostic Radiology – Part 2 QBank - QA + Explanations, 1E (2025)
Egyptian Board of Diagnostic Radiology – Part 2 QBank - QA + Explanations, 1E (2025)
Egyptian Board of Diagnostic Radiology – Part 2 QBank - QA + Explanations, 1E (2025)
Egyptian Board of Diagnostic Radiology – Part 2 QBank - QA + Explanations, 1E (2025)
Egyptian Board of Diagnostic Radiology – Part 2 QBank - QA + Explanations, 1E (2025)
Egyptian Board of Diagnostic Radiology – Part 2 QBank - QA + Explanations, 1E (2025)
Egyptian Board of Diagnostic Radiology – Part 2 QBank - QA + Explanations, 1E (2025)
Egyptian Board of Diagnostic Radiology – Part 2 QBank - QA + Explanations, 1E (2025)
Egyptian Board of Diagnostic Radiology – Part 2 QBank - QA + Explanations, 1E (2025)
Egyptian Board of Diagnostic Radiology – Part 2 QBank - QA + Explanations, 1E (2025)
Egyptian Board of Diagnostic Radiology – Part 2 QBank - QA + Explanations, 1E (2025)
Egyptian Board of Diagnostic Radiology – Part 2 QBank - QA + Explanations, 1E (2025)
Egyptian Board of Diagnostic Radiology – Part 2 QBank - QA + Explanations, 1E (2025)
Egyptian Board of Diagnostic Radiology – Part 2 QBank - QA + Explanations, 1E (2025)
Egyptian Board of Diagnostic Radiology – Part 2 QBank - QA + Explanations, 1E (2025)
Egyptian Board of Diagnostic Radiology – Part 2 QBank - QA + Explanations, 1E (2025)
Egyptian Board of Diagnostic Radiology – Part 2 QBank - QA + Explanations, 1E (2025)
Egyptian Board of Diagnostic Radiology – Part 2 QBank - QA + Explanations, 1E (2025)
Egyptian Board of Diagnostic Radiology – Part 2 QBank - QA + Explanations, 1E (2025)
Egyptian Board of Diagnostic Radiology – Part 2 QBank - QA + Explanations, 1E (2025)
Egyptian Board of Diagnostic Radiology – Part 2 QBank - QA + Explanations, 1E (2025)
Egyptian Board of Diagnostic Radiology – Part 2 QBank - QA + Explanations, 1E (2025)
Egyptian Board of Diagnostic Radiology – Part 2 QBank - QA + Explanations, 1E (2025)
Egyptian Board of Diagnostic Radiology – Part 2 QBank - QA + Explanations, 1E (2025)
Egyptian Board of Diagnostic Radiology – Part 2 QBank - QA + Explanations, 1E (2025)
Egyptian Board of Diagnostic Radiology – Part 2 QBank - QA + Explanations, 1E (2025)
Egyptian Board of Diagnostic Radiology – Part 2 QBank - QA + Explanations, 1E (2025)
Egyptian Board of Diagnostic Radiology – Part 2 QBank - QA + Explanations, 1E (2025)
Egyptian Board of Diagnostic Radiology – Part 2 QBank - QA + Explanations, 1E (2025)
Egyptian Board of Diagnostic Radiology – Part 2 QBank - QA + Explanations, 1E (2025)
Egyptian Board of Diagnostic Radiology – Part 2 QBank - QA + Explanations, 1E (2025)
Egyptian Board of Diagnostic Radiology – Part 2 QBank - QA + Explanations, 1E (2025)
Egyptian Board of Diagnostic Radiology – Part 2 QBank - QA + Explanations, 1E (2025)
Egyptian Board of Diagnostic Radiology – Part 2 QBank - QA + Explanations, 1E (2025)
Egyptian Board of Diagnostic Radiology – Part 2 QBank - QA + Explanations, 1E (2025)
Egyptian Board of Diagnostic Radiology – Part 2 QBank - QA + Explanations, 1E (2025)
Egyptian Board of Diagnostic Radiology – Part 2 QBank - QA + Explanations, 1E (2025)
Egyptian Board of Diagnostic Radiology – Part 2 QBank - QA + Explanations, 1E (2025)
Egyptian Board of Diagnostic Radiology – Part 2 QBank - QA + Explanations, 1E (2025)
Egyptian Board of Diagnostic Radiology – Part 2 QBank - QA + Explanations, 1E (2025)
Egyptian Board of Diagnostic Radiology – Part 2 QBank - QA + Explanations, 1E (2025)
Egyptian Board of Diagnostic Radiology – Part 2 QBank - QA + Explanations, 1E (2025)
Egyptian Board of Diagnostic Radiology – Part 2 QBank - QA + Explanations, 1E (2025)
Egyptian Board of Diagnostic Radiology – Part 2 QBank - QA + Explanations, 1E (2025)
Egyptian Board of Diagnostic Radiology – Part 2 QBank - QA + Explanations, 1E (2025)
Egyptian Board of Diagnostic Radiology – Part 2 QBank - QA + Explanations, 1E (2025)
Egyptian Board of Diagnostic Radiology – Part 2 QBank - QA + Explanations, 1E (2025)
Egyptian Board of Diagnostic Radiology – Part 2 QBank - QA + Explanations, 1E (2025)
Egyptian Board of Diagnostic Radiology – Part 2 QBank - QA + Explanations, 1E (2025)
Egyptian Board of Diagnostic Radiology – Part 2 QBank - QA + Explanations, 1E (2025)
Egyptian Board of Diagnostic Radiology – Part 2 QBank - QA + Explanations, 1E (2025)
Egyptian Board of Diagnostic Radiology – Part 2 QBank - QA + Explanations, 1E (2025)
Egyptian Board of Diagnostic Radiology – Part 2 QBank - QA + Explanations, 1E (2025)
Egyptian Board of Diagnostic Radiology – Part 2 QBank - QA + Explanations, 1E (2025)
Egyptian Board of Diagnostic Radiology – Part 2 QBank - QA + Explanations, 1E (2025)
Egyptian Board of Diagnostic Radiology – Part 2 QBank - QA + Explanations, 1E (2025)
Egyptian Board of Diagnostic Radiology – Part 2 QBank - QA + Explanations, 1E (2025)
Egyptian Board of Diagnostic Radiology – Part 2 QBank - QA + Explanations, 1E (2025)
Egyptian Board of Diagnostic Radiology – Part 2 QBank - QA + Explanations, 1E (2025)
Egyptian Board of Diagnostic Radiology – Part 2 QBank - QA + Explanations, 1E (2025)
Egyptian Board of Diagnostic Radiology – Part 2 QBank - QA + Explanations, 1E (2025)
Egyptian Board of Diagnostic Radiology – Part 2 QBank - QA + Explanations, 1E (2025)
Egyptian Board of Diagnostic Radiology – Part 2 QBank - QA + Explanations, 1E (2025)
Egyptian Board of Diagnostic Radiology – Part 2 QBank - QA + Explanations, 1E (2025)
Egyptian Board of Diagnostic Radiology – Part 2 QBank - QA + Explanations, 1E (2025)
Egyptian Board of Diagnostic Radiology – Part 2 QBank - QA + Explanations, 1E (2025)
Egyptian Board of Diagnostic Radiology – Part 2 QBank - QA + Explanations, 1E (2025)
Egyptian Board of Diagnostic Radiology – Part 2 QBank - QA + Explanations, 1E (2025)
Egyptian Board of Diagnostic Radiology – Part 2 QBank - QA + Explanations, 1E (2025)
Egyptian Board of Diagnostic Radiology – Part 2 QBank - QA + Explanations, 1E (2025)
Egyptian Board of Diagnostic Radiology – Part 2 QBank - QA + Explanations, 1E (2025)
Egyptian Board of Diagnostic Radiology – Part 2 QBank - QA + Explanations, 1E (2025)
Egyptian Board of Diagnostic Radiology – Part 2 QBank - QA + Explanations, 1E (2025)
Egyptian Board of Diagnostic Radiology – Part 2 QBank - QA + Explanations, 1E (2025)
Egyptian Board of Diagnostic Radiology – Part 2 QBank - QA + Explanations, 1E (2025)
Egyptian Board of Diagnostic Radiology – Part 2 QBank - QA + Explanations, 1E (2025)
Egyptian Board of Diagnostic Radiology – Part 2 QBank - QA + Explanations, 1E (2025)
Egyptian Board of Diagnostic Radiology – Part 2 QBank - QA + Explanations, 1E (2025)
Egyptian Board of Diagnostic Radiology – Part 2 QBank - QA + Explanations, 1E (2025)
Egyptian Board of Diagnostic Radiology – Part 2 QBank - QA + Explanations, 1E (2025)
Egyptian Board of Diagnostic Radiology – Part 2 QBank - QA + Explanations, 1E (2025)
Egyptian Board of Diagnostic Radiology – Part 2 QBank - QA + Explanations, 1E (2025)
Egyptian Board of Diagnostic Radiology – Part 2 QBank - QA + Explanations, 1E (2025)
Egyptian Board of Diagnostic Radiology – Part 2 QBank - QA + Explanations, 1E (2025)
Egyptian Board of Diagnostic Radiology – Part 2 QBank - QA + Explanations, 1E (2025)
Egyptian Board of Diagnostic Radiology – Part 2 QBank - QA + Explanations, 1E (2025)
Egyptian Board of Diagnostic Radiology – Part 2 QBank - QA + Explanations, 1E (2025)
Egyptian Board of Diagnostic Radiology – Part 2 QBank - QA + Explanations, 1E (2025)
Egyptian Board of Diagnostic Radiology – Part 2 QBank - QA + Explanations, 1E (2025)
Egyptian Board of Diagnostic Radiology – Part 2 QBank - QA + Explanations, 1E (2025)
Egyptian Board of Diagnostic Radiology – Part 2 QBank - QA + Explanations, 1E (2025)
Egyptian Board of Diagnostic Radiology – Part 2 QBank - QA + Explanations, 1E (2025)
Egyptian Board of Diagnostic Radiology – Part 2 QBank - QA + Explanations, 1E (2025)
Egyptian Board of Diagnostic Radiology – Part 2 QBank - QA + Explanations, 1E (2025)
Egyptian Board of Diagnostic Radiology – Part 2 QBank - QA + Explanations, 1E (2025)
Egyptian Board of Diagnostic Radiology – Part 2 QBank - QA + Explanations, 1E (2025)
Egyptian Board of Diagnostic Radiology – Part 2 QBank - QA + Explanations, 1E (2025)
Egyptian Board of Diagnostic Radiology – Part 2 QBank - QA + Explanations, 1E (2025)
Egyptian Board of Diagnostic Radiology – Part 2 QBank - QA + Explanations, 1E (2025)
Egyptian Board of Diagnostic Radiology – Part 2 QBank - QA + Explanations, 1E (2025)
Egyptian Board of Diagnostic Radiology – Part 2 QBank - QA + Explanations, 1E (2025)
Egyptian Board of Diagnostic Radiology – Part 2 QBank - QA + Explanations, 1E (2025)
Egyptian Board of Diagnostic Radiology – Part 2 QBank - QA + Explanations, 1E (2025)
Egyptian Board of Diagnostic Radiology – Part 2 QBank - QA + Explanations, 1E (2025)
Egyptian Board of Diagnostic Radiology – Part 2 QBank - QA + Explanations, 1E (2025)
Egyptian Board of Diagnostic Radiology – Part 2 QBank - QA + Explanations, 1E (2025)
Egyptian Board of Diagnostic Radiology – Part 2 QBank - QA + Explanations, 1E (2025)
Egyptian Board of Diagnostic Radiology – Part 2 QBank - QA + Explanations, 1E (2025)
Egyptian Board of Diagnostic Radiology – Part 2 QBank - QA + Explanations, 1E (2025)
Egyptian Board of Diagnostic Radiology – Part 2 QBank - QA + Explanations, 1E (2025)
Egyptian Board of Diagnostic Radiology – Part 2 QBank - QA + Explanations, 1E (2025)
Egyptian Board of Diagnostic Radiology – Part 2 QBank - QA + Explanations, 1E (2025)
Egyptian Board of Diagnostic Radiology – Part 2 QBank - QA + Explanations, 1E (2025)
Egyptian Board of Diagnostic Radiology – Part 2 QBank - QA + Explanations, 1E (2025)
Egyptian Board of Diagnostic Radiology – Part 2 QBank - QA + Explanations, 1E (2025)
Egyptian Board of Diagnostic Radiology – Part 2 QBank - QA + Explanations, 1E (2025)
Egyptian Board of Diagnostic Radiology – Part 2 QBank - QA + Explanations, 1E (2025)
Egyptian Board of Diagnostic Radiology – Part 2 QBank - QA + Explanations, 1E (2025)
Egyptian Board of Diagnostic Radiology – Part 2 QBank - QA + Explanations, 1E (2025)
Egyptian Board of Diagnostic Radiology – Part 2 QBank - QA + Explanations, 1E (2025)
Egyptian Board of Diagnostic Radiology – Part 2 QBank - QA + Explanations, 1E (2025)
Egyptian Board of Diagnostic Radiology – Part 2 QBank - QA + Explanations, 1E (2025)
Egyptian Board of Diagnostic Radiology – Part 2 QBank - QA + Explanations, 1E (2025)
Egyptian Board of Diagnostic Radiology – Part 2 QBank - QA + Explanations, 1E (2025)
Egyptian Board of Diagnostic Radiology – Part 2 QBank - QA + Explanations, 1E (2025)
Egyptian Board of Diagnostic Radiology – Part 2 QBank - QA + Explanations, 1E (2025)
Egyptian Board of Diagnostic Radiology – Part 2 QBank - QA + Explanations, 1E (2025)
Egyptian Board of Diagnostic Radiology – Part 2 QBank - QA + Explanations, 1E (2025)
Egyptian Board of Diagnostic Radiology – Part 2 QBank - QA + Explanations, 1E (2025)
Egyptian Board of Diagnostic Radiology – Part 2 QBank - QA + Explanations, 1E (2025)
Egyptian Board of Diagnostic Radiology – Part 2 QBank - QA + Explanations, 1E (2025)
Egyptian Board of Diagnostic Radiology – Part 2 QBank - QA + Explanations, 1E (2025)
Egyptian Board of Diagnostic Radiology – Part 2 QBank - QA + Explanations, 1E (2025)
Egyptian Board of Diagnostic Radiology – Part 2 QBank - QA + Explanations, 1E (2025)
Egyptian Board of Diagnostic Radiology – Part 2 QBank - QA + Explanations, 1E (2025)
Egyptian Board of Diagnostic Radiology – Part 2 QBank - QA + Explanations, 1E (2025)
Egyptian Board of Diagnostic Radiology – Part 2 QBank - QA + Explanations, 1E (2025)
Egyptian Board of Diagnostic Radiology – Part 2 QBank - QA + Explanations, 1E (2025)
Egyptian Board of Diagnostic Radiology – Part 2 QBank - QA + Explanations, 1E (2025)
Egyptian Board of Diagnostic Radiology – Part 2 QBank - QA + Explanations, 1E (2025)
Egyptian Board of Diagnostic Radiology – Part 2 QBank - QA + Explanations, 1E (2025)
Egyptian Board of Diagnostic Radiology – Part 2 QBank - QA + Explanations, 1E (2025)
Egyptian Board of Diagnostic Radiology – Part 2 QBank - QA + Explanations, 1E (2025)
Egyptian Board of Diagnostic Radiology – Part 2 QBank - QA + Explanations, 1E (2025)
Egyptian Board of Diagnostic Radiology – Part 2 QBank - QA + Explanations, 1E (2025)
Egyptian Board of Diagnostic Radiology – Part 2 QBank - QA + Explanations, 1E (2025)
Egyptian Board of Diagnostic Radiology – Part 2 QBank - QA + Explanations, 1E (2025)
Egyptian Board of Diagnostic Radiology – Part 2 QBank - QA + Explanations, 1E (2025)
Egyptian Board of Diagnostic Radiology – Part 2 QBank - QA + Explanations, 1E (2025)
Egyptian Board of Diagnostic Radiology – Part 2 QBank - QA + Explanations, 1E (2025)
Egyptian Board of Diagnostic Radiology – Part 2 QBank - QA + Explanations, 1E (2025)
Egyptian Board of Diagnostic Radiology – Part 2 QBank - QA + Explanations, 1E (2025)
Egyptian Board of Diagnostic Radiology – Part 2 QBank - QA + Explanations, 1E (2025)
Egyptian Board of Diagnostic Radiology – Part 2 QBank - QA + Explanations, 1E (2025)
Egyptian Board of Diagnostic Radiology – Part 2 QBank - QA + Explanations, 1E (2025)
Egyptian Board of Diagnostic Radiology – Part 2 QBank - QA + Explanations, 1E (2025)
Egyptian Board of Diagnostic Radiology – Part 2 QBank - QA + Explanations, 1E (2025)
Egyptian Board of Diagnostic Radiology – Part 2 QBank - QA + Explanations, 1E (2025)
Egyptian Board of Diagnostic Radiology – Part 2 QBank - QA + Explanations, 1E (2025)
Egyptian Board of Diagnostic Radiology – Part 2 QBank - QA + Explanations, 1E (2025)
Egyptian Board of Diagnostic Radiology – Part 2 QBank - QA + Explanations, 1E (2025)
Egyptian Board of Diagnostic Radiology – Part 2 QBank - QA + Explanations, 1E (2025)
Egyptian Board of Diagnostic Radiology – Part 2 QBank - QA + Explanations, 1E (2025)
Egyptian Board of Diagnostic Radiology – Part 2 QBank - QA + Explanations, 1E (2025)
Egyptian Board of Diagnostic Radiology – Part 2 QBank - QA + Explanations, 1E (2025)
Egyptian Board of Diagnostic Radiology – Part 2 QBank - QA + Explanations, 1E (2025)
Egyptian Board of Diagnostic Radiology – Part 2 QBank - QA + Explanations, 1E (2025)
Egyptian Board of Diagnostic Radiology – Part 2 QBank - QA + Explanations, 1E (2025)
Egyptian Board of Diagnostic Radiology – Part 2 QBank - QA + Explanations, 1E (2025)
Egyptian Board of Diagnostic Radiology – Part 2 QBank - QA + Explanations, 1E (2025)
Egyptian Board of Diagnostic Radiology – Part 2 QBank - QA + Explanations, 1E (2025)
Egyptian Board of Diagnostic Radiology – Part 2 QBank - QA + Explanations, 1E (2025)
Egyptian Board of Diagnostic Radiology – Part 2 QBank - QA + Explanations, 1E (2025)
Egyptian Board of Diagnostic Radiology – Part 2 QBank - QA + Explanations, 1E (2025)
Egyptian Board of Diagnostic Radiology – Part 2 QBank - QA + Explanations, 1E (2025)
Egyptian Board of Diagnostic Radiology – Part 2 QBank - QA + Explanations, 1E (2025)
Egyptian Board of Diagnostic Radiology – Part 2 QBank - QA + Explanations, 1E (2025)
Egyptian Board of Diagnostic Radiology – Part 2 QBank - QA + Explanations, 1E (2025)
Egyptian Board of Diagnostic Radiology – Part 2 QBank - QA + Explanations, 1E (2025)
Egyptian Board of Diagnostic Radiology – Part 2 QBank - QA + Explanations, 1E (2025)
Egyptian Board of Diagnostic Radiology – Part 2 QBank - QA + Explanations, 1E (2025)
Egyptian Board of Diagnostic Radiology – Part 2 QBank - QA + Explanations, 1E (2025)
Egyptian Board of Diagnostic Radiology – Part 2 QBank - QA + Explanations, 1E (2025)
Egyptian Board of Diagnostic Radiology – Part 2 QBank - QA + Explanations, 1E (2025)
Egyptian Board of Diagnostic Radiology – Part 2 QBank - QA + Explanations, 1E (2025)
Egyptian Board of Diagnostic Radiology – Part 2 QBank - QA + Explanations, 1E (2025)
Egyptian Board of Diagnostic Radiology – Part 2 QBank - QA + Explanations, 1E (2025)
Egyptian Board of Diagnostic Radiology – Part 2 QBank - QA + Explanations, 1E (2025)
Egyptian Board of Diagnostic Radiology – Part 2 QBank - QA + Explanations, 1E (2025)
Egyptian Board of Diagnostic Radiology – Part 2 QBank - QA + Explanations, 1E (2025)
Egyptian Board of Diagnostic Radiology – Part 2 QBank - QA + Explanations, 1E (2025)
Egyptian Board of Diagnostic Radiology – Part 2 QBank - QA + Explanations, 1E (2025)
Egyptian Board of Diagnostic Radiology – Part 2 QBank - QA + Explanations, 1E (2025)
Egyptian Board of Diagnostic Radiology – Part 2 QBank - QA + Explanations, 1E (2025)
Egyptian Board of Diagnostic Radiology – Part 2 QBank - QA + Explanations, 1E (2025)
Egyptian Board of Diagnostic Radiology – Part 2 QBank - QA + Explanations, 1E (2025)
Egyptian Board of Diagnostic Radiology – Part 2 QBank - QA + Explanations, 1E (2025)
Egyptian Board of Diagnostic Radiology – Part 2 QBank - QA + Explanations, 1E (2025)
Egyptian Board of Diagnostic Radiology – Part 2 QBank - QA + Explanations, 1E (2025)
Egyptian Board of Diagnostic Radiology – Part 2 QBank - QA + Explanations, 1E (2025)
Egyptian Board of Diagnostic Radiology – Part 2 QBank - QA + Explanations, 1E (2025)
Egyptian Board of Diagnostic Radiology – Part 2 QBank - QA + Explanations, 1E (2025)
Egyptian Board of Diagnostic Radiology – Part 2 QBank - QA + Explanations, 1E (2025)
Egyptian Board of Diagnostic Radiology – Part 2 QBank - QA + Explanations, 1E (2025)
Egyptian Board of Diagnostic Radiology – Part 2 QBank - QA + Explanations, 1E (2025)
Egyptian Board of Diagnostic Radiology – Part 2 QBank - QA + Explanations, 1E (2025)
Egyptian Board of Diagnostic Radiology – Part 2 QBank - QA + Explanations, 1E (2025)
Egyptian Board of Diagnostic Radiology – Part 2 QBank - QA + Explanations, 1E (2025)
Egyptian Board of Diagnostic Radiology – Part 2 QBank - QA + Explanations, 1E (2025)
Egyptian Board of Diagnostic Radiology – Part 2 QBank - QA + Explanations, 1E (2025)
Egyptian Board of Diagnostic Radiology – Part 2 QBank - QA + Explanations, 1E (2025)
Egyptian Board of Diagnostic Radiology – Part 2 QBank - QA + Explanations, 1E (2025)
Egyptian Board of Diagnostic Radiology – Part 2 QBank - QA + Explanations, 1E (2025)
Egyptian Board of Diagnostic Radiology – Part 2 QBank - QA + Explanations, 1E (2025)
Egyptian Board of Diagnostic Radiology – Part 2 QBank - QA + Explanations, 1E (2025)
Egyptian Board of Diagnostic Radiology – Part 2 QBank - QA + Explanations, 1E (2025)
Egyptian Board of Diagnostic Radiology – Part 2 QBank - QA + Explanations, 1E (2025)
Egyptian Board of Diagnostic Radiology – Part 2 QBank - QA + Explanations, 1E (2025)
Egyptian Board of Diagnostic Radiology – Part 2 QBank - QA + Explanations, 1E (2025)
Egyptian Board of Diagnostic Radiology – Part 2 QBank - QA + Explanations, 1E (2025)
Egyptian Board of Diagnostic Radiology – Part 2 QBank - QA + Explanations, 1E (2025)
Egyptian Board of Diagnostic Radiology – Part 2 QBank - QA + Explanations, 1E (2025)
Egyptian Board of Diagnostic Radiology – Part 2 QBank - QA + Explanations, 1E (2025)
Egyptian Board of Diagnostic Radiology – Part 2 QBank - QA + Explanations, 1E (2025)
Egyptian Board of Diagnostic Radiology – Part 2 QBank - QA + Explanations, 1E (2025)
Egyptian Board of Diagnostic Radiology – Part 2 QBank - QA + Explanations, 1E (2025)
Egyptian Board of Diagnostic Radiology – Part 2 QBank - QA + Explanations, 1E (2025)
Egyptian Board of Diagnostic Radiology – Part 2 QBank - QA + Explanations, 1E (2025)
Egyptian Board of Diagnostic Radiology – Part 2 QBank - QA + Explanations, 1E (2025)
Egyptian Board of Diagnostic Radiology – Part 2 QBank - QA + Explanations, 1E (2025)
Egyptian Board of Diagnostic Radiology – Part 2 QBank - QA + Explanations, 1E (2025)
Egyptian Board of Diagnostic Radiology – Part 2 QBank - QA + Explanations, 1E (2025)
Egyptian Board of Diagnostic Radiology – Part 2 QBank - QA + Explanations, 1E (2025)
Egyptian Board of Diagnostic Radiology – Part 2 QBank - QA + Explanations, 1E (2025)
Egyptian Board of Diagnostic Radiology – Part 2 QBank - QA + Explanations, 1E (2025)
Egyptian Board of Diagnostic Radiology – Part 2 QBank - QA + Explanations, 1E (2025)
Egyptian Board of Diagnostic Radiology – Part 2 QBank - QA + Explanations, 1E (2025)
Egyptian Board of Diagnostic Radiology – Part 2 QBank - QA + Explanations, 1E (2025)
Egyptian Board of Diagnostic Radiology – Part 2 QBank - QA + Explanations, 1E (2025)
Egyptian Board of Diagnostic Radiology – Part 2 QBank - QA + Explanations, 1E (2025)
Egyptian Board of Diagnostic Radiology – Part 2 QBank - QA + Explanations, 1E (2025)
Egyptian Board of Diagnostic Radiology – Part 2 QBank - QA + Explanations, 1E (2025)
Egyptian Board of Diagnostic Radiology – Part 2 QBank - QA + Explanations, 1E (2025)
Egyptian Board of Diagnostic Radiology – Part 2 QBank - QA + Explanations, 1E (2025)
Egyptian Board of Diagnostic Radiology – Part 2 QBank - QA + Explanations, 1E (2025)
Egyptian Board of Diagnostic Radiology – Part 2 QBank - QA + Explanations, 1E (2025)
Egyptian Board of Diagnostic Radiology – Part 2 QBank - QA + Explanations, 1E (2025)
Egyptian Board of Diagnostic Radiology – Part 2 QBank - QA + Explanations, 1E (2025)
Egyptian Board of Diagnostic Radiology – Part 2 QBank - QA + Explanations, 1E (2025)
Egyptian Board of Diagnostic Radiology – Part 2 QBank - QA + Explanations, 1E (2025)
Egyptian Board of Diagnostic Radiology – Part 2 QBank - QA + Explanations, 1E (2025)
Egyptian Board of Diagnostic Radiology – Part 2 QBank - QA + Explanations, 1E (2025)
Egyptian Board of Diagnostic Radiology – Part 2 QBank - QA + Explanations, 1E (2025)
Egyptian Board of Diagnostic Radiology – Part 2 QBank - QA + Explanations, 1E (2025)
Egyptian Board of Diagnostic Radiology – Part 2 QBank - QA + Explanations, 1E (2025)
Egyptian Board of Diagnostic Radiology – Part 2 QBank - QA + Explanations, 1E (2025)
Egyptian Board of Diagnostic Radiology – Part 2 QBank - QA + Explanations, 1E (2025)
Egyptian Board of Diagnostic Radiology – Part 2 QBank - QA + Explanations, 1E (2025)
Egyptian Board of Diagnostic Radiology – Part 2 QBank - QA + Explanations, 1E (2025)
Egyptian Board of Diagnostic Radiology – Part 2 QBank - QA + Explanations, 1E (2025)
Egyptian Board of Diagnostic Radiology – Part 2 QBank - QA + Explanations, 1E (2025)
Egyptian Board of Diagnostic Radiology – Part 2 QBank - QA + Explanations, 1E (2025)
Egyptian Board of Diagnostic Radiology – Part 2 QBank - QA + Explanations, 1E (2025)
Egyptian Board of Diagnostic Radiology – Part 2 QBank - QA + Explanations, 1E (2025)
Egyptian Board of Diagnostic Radiology – Part 2 QBank - QA + Explanations, 1E (2025)
Egyptian Board of Diagnostic Radiology – Part 2 QBank - QA + Explanations, 1E (2025)
Egyptian Board of Diagnostic Radiology – Part 2 QBank - QA + Explanations, 1E (2025)
Egyptian Board of Diagnostic Radiology – Part 2 QBank - QA + Explanations, 1E (2025)
Egyptian Board of Diagnostic Radiology – Part 2 QBank - QA + Explanations, 1E (2025)
Egyptian Board of Diagnostic Radiology – Part 2 QBank - QA + Explanations, 1E (2025)
Egyptian Board of Diagnostic Radiology – Part 2 QBank - QA + Explanations, 1E (2025)
Egyptian Board of Diagnostic Radiology – Part 2 QBank - QA + Explanations, 1E (2025)
Egyptian Board of Diagnostic Radiology – Part 2 QBank - QA + Explanations, 1E (2025)
Egyptian Board of Diagnostic Radiology – Part 2 QBank - QA + Explanations, 1E (2025)
Egyptian Board of Diagnostic Radiology – Part 2 QBank - QA + Explanations, 1E (2025)
Egyptian Board of Diagnostic Radiology – Part 2 QBank - QA + Explanations, 1E (2025)
Egyptian Board of Diagnostic Radiology – Part 2 QBank - QA + Explanations, 1E (2025)
Egyptian Board of Diagnostic Radiology – Part 2 QBank - QA + Explanations, 1E (2025)
Egyptian Board of Diagnostic Radiology – Part 2 QBank - QA + Explanations, 1E (2025)
Egyptian Board of Diagnostic Radiology – Part 2 QBank - QA + Explanations, 1E (2025)
Egyptian Board of Diagnostic Radiology – Part 2 QBank - QA + Explanations, 1E (2025)
Egyptian Board of Diagnostic Radiology – Part 2 QBank - QA + Explanations, 1E (2025)
Egyptian Board of Diagnostic Radiology – Part 2 QBank - QA + Explanations, 1E (2025)
Egyptian Board of Diagnostic Radiology – Part 2 QBank - QA + Explanations, 1E (2025)
Egyptian Board of Diagnostic Radiology – Part 2 QBank - QA + Explanations, 1E (2025)
Egyptian Board of Diagnostic Radiology – Part 2 QBank - QA + Explanations, 1E (2025)
Egyptian Board of Diagnostic Radiology – Part 2 QBank - QA + Explanations, 1E (2025)
Egyptian Board of Diagnostic Radiology – Part 2 QBank - QA + Explanations, 1E (2025)
Egyptian Board of Diagnostic Radiology – Part 2 QBank - QA + Explanations, 1E (2025)
Egyptian Board of Diagnostic Radiology – Part 2 QBank - QA + Explanations, 1E (2025)
Egyptian Board of Diagnostic Radiology – Part 2 QBank - QA + Explanations, 1E (2025)
Egyptian Board of Diagnostic Radiology – Part 2 QBank - QA + Explanations, 1E (2025)
Egyptian Board of Diagnostic Radiology – Part 2 QBank - QA + Explanations, 1E (2025)
Egyptian Board of Diagnostic Radiology – Part 2 QBank - QA + Explanations, 1E (2025)
Egyptian Board of Diagnostic Radiology – Part 2 QBank - QA + Explanations, 1E (2025)
Egyptian Board of Diagnostic Radiology – Part 2 QBank - QA + Explanations, 1E (2025)
Egyptian Board of Diagnostic Radiology – Part 2 QBank - QA + Explanations, 1E (2025)
Egyptian Board of Diagnostic Radiology – Part 2 QBank - QA + Explanations, 1E (2025)
Egyptian Board of Diagnostic Radiology – Part 2 QBank - QA + Explanations, 1E (2025)
Egyptian Board of Diagnostic Radiology – Part 2 QBank - QA + Explanations, 1E (2025)
Egyptian Board of Diagnostic Radiology – Part 2 QBank - QA + Explanations, 1E (2025)
Egyptian Board of Diagnostic Radiology – Part 2 QBank - QA + Explanations, 1E (2025)
Egyptian Board of Diagnostic Radiology – Part 2 QBank - QA + Explanations, 1E (2025)
Egyptian Board of Diagnostic Radiology – Part 2 QBank - QA + Explanations, 1E (2025)
Egyptian Board of Diagnostic Radiology – Part 2 QBank - QA + Explanations, 1E (2025)
Egyptian Board of Diagnostic Radiology – Part 2 QBank - QA + Explanations, 1E (2025)
Egyptian Board of Diagnostic Radiology – Part 2 QBank - QA + Explanations, 1E (2025)
Egyptian Board of Diagnostic Radiology – Part 2 QBank - QA + Explanations, 1E (2025)
Egyptian Board of Diagnostic Radiology – Part 2 QBank - QA + Explanations, 1E (2025)
Egyptian Board of Diagnostic Radiology – Part 2 QBank - QA + Explanations, 1E (2025)
Egyptian Board of Diagnostic Radiology – Part 2 QBank - QA + Explanations, 1E (2025)
Egyptian Board of Diagnostic Radiology – Part 2 QBank - QA + Explanations, 1E (2025)
Egyptian Board of Diagnostic Radiology – Part 2 QBank - QA + Explanations, 1E (2025)
Egyptian Board of Diagnostic Radiology – Part 2 QBank - QA + Explanations, 1E (2025)
Egyptian Board of Diagnostic Radiology – Part 2 QBank - QA + Explanations, 1E (2025)
Egyptian Board of Diagnostic Radiology – Part 2 QBank - QA + Explanations, 1E (2025)
Egyptian Board of Diagnostic Radiology – Part 2 QBank - QA + Explanations, 1E (2025)
Egyptian Board of Diagnostic Radiology – Part 2 QBank - QA + Explanations, 1E (2025)
Egyptian Board of Diagnostic Radiology – Part 2 QBank - QA + Explanations, 1E (2025)
Egyptian Board of Diagnostic Radiology – Part 2 QBank - QA + Explanations, 1E (2025)
Egyptian Board of Diagnostic Radiology – Part 2 QBank - QA + Explanations, 1E (2025)
Egyptian Board of Diagnostic Radiology – Part 2 QBank - QA + Explanations, 1E (2025)
Egyptian Board of Diagnostic Radiology – Part 2 QBank - QA + Explanations, 1E (2025)
Egyptian Board of Diagnostic Radiology – Part 2 QBank - QA + Explanations, 1E (2025)
Egyptian Board of Diagnostic Radiology – Part 2 QBank - QA + Explanations, 1E (2025)
Egyptian Board of Diagnostic Radiology – Part 2 QBank - QA + Explanations, 1E (2025)
Egyptian Board of Diagnostic Radiology – Part 2 QBank - QA + Explanations, 1E (2025)
Egyptian Board of Diagnostic Radiology – Part 2 QBank - QA + Explanations, 1E (2025)
Egyptian Board of Diagnostic Radiology – Part 2 QBank - QA + Explanations, 1E (2025)
Egyptian Board of Diagnostic Radiology – Part 2 QBank - QA + Explanations, 1E (2025)
Egyptian Board of Diagnostic Radiology – Part 2 QBank - QA + Explanations, 1E (2025)
Egyptian Board of Diagnostic Radiology – Part 2 QBank - QA + Explanations, 1E (2025)
Egyptian Board of Diagnostic Radiology – Part 2 QBank - QA + Explanations, 1E (2025)
Egyptian Board of Diagnostic Radiology – Part 2 QBank - QA + Explanations, 1E (2025)
Egyptian Board of Diagnostic Radiology – Part 2 QBank - QA + Explanations, 1E (2025)
Egyptian Board of Diagnostic Radiology – Part 2 QBank - QA + Explanations, 1E (2025)
Egyptian Board of Diagnostic Radiology – Part 2 QBank - QA + Explanations, 1E (2025)
Egyptian Board of Diagnostic Radiology – Part 2 QBank - QA + Explanations, 1E (2025)
Egyptian Board of Diagnostic Radiology – Part 2 QBank - QA + Explanations, 1E (2025)
Egyptian Board of Diagnostic Radiology – Part 2 QBank - QA + Explanations, 1E (2025)
Egyptian Board of Diagnostic Radiology – Part 2 QBank - QA + Explanations, 1E (2025)
Egyptian Board of Diagnostic Radiology – Part 2 QBank - QA + Explanations, 1E (2025)
Egyptian Board of Diagnostic Radiology – Part 2 QBank - QA + Explanations, 1E (2025)
Egyptian Board of Diagnostic Radiology – Part 2 QBank - QA + Explanations, 1E (2025)
Egyptian Board of Diagnostic Radiology – Part 2 QBank - QA + Explanations, 1E (2025)
Egyptian Board of Diagnostic Radiology – Part 2 QBank - QA + Explanations, 1E (2025)
Egyptian Board of Diagnostic Radiology – Part 2 QBank - QA + Explanations, 1E (2025)
Egyptian Board of Diagnostic Radiology – Part 2 QBank - QA + Explanations, 1E (2025)
Egyptian Board of Diagnostic Radiology – Part 2 QBank - QA + Explanations, 1E (2025)
Egyptian Board of Diagnostic Radiology – Part 2 QBank - QA + Explanations, 1E (2025)
Egyptian Board of Diagnostic Radiology – Part 2 QBank - QA + Explanations, 1E (2025)
Egyptian Board of Diagnostic Radiology – Part 2 QBank - QA + Explanations, 1E (2025)
Egyptian Board of Diagnostic Radiology – Part 2 QBank - QA + Explanations, 1E (2025)
Egyptian Board of Diagnostic Radiology – Part 2 QBank - QA + Explanations, 1E (2025)
Egyptian Board of Diagnostic Radiology – Part 2 QBank - QA + Explanations, 1E (2025)
Egyptian Board of Diagnostic Radiology – Part 2 QBank - QA + Explanations, 1E (2025)
Egyptian Board of Diagnostic Radiology – Part 2 QBank - QA + Explanations, 1E (2025)
Egyptian Board of Diagnostic Radiology – Part 2 QBank - QA + Explanations, 1E (2025)
Egyptian Board of Diagnostic Radiology – Part 2 QBank - QA + Explanations, 1E (2025)
Egyptian Board of Diagnostic Radiology – Part 2 QBank - QA + Explanations, 1E (2025)

Egyptian Board of Diagnostic Radiology – Part 2 QBank - QA + Explanations, 1E (2025)

  • 2.
    DISCLAIMER The material inthis PDF is provided solely for educational and revision purposes. While every effort has been made to reproduce past examination questions accurately or to outline their core concepts and answers, this document is not an official publication of the Egyptian Board of Diagnostic Radiology (EBDR), nor is it endorsed, reviewed, or approved by the EBDR or any affiliated authority. 1. Accuracy & Evolving Medical Standards The compilers have taken reasonable care to ensure the content is correct; however, no warranty, express or implied, is given regarding the accuracy, completeness, or currency of any question, hint, answer, or explanation contained herein. Medical imaging guidelines, protocols, and diagnostic criteria evolve continuously. Users must verify all information with: • Current official EBDR materials and recent examination guidelines • Latest editions of authoritative radiology textbooks • Updated professional society guidelines (ACR, ESR, RSNA, etc.) • Most recent TNM staging manuals (8th edition or newer) • Peer-reviewed medical literature and current clinical practice standards 2. Specific Areas Requiring Vigilance Users should be particularly cautious regarding: • Breast cancer screening recommendations and BI-RADS classifications • TNM staging criteria (some questions may reference older editions) • Interventional radiology indications and contraindications • Radiation dose considerations and imaging protocol selections • Emerging imaging techniques and evolving diagnostic criteria 3. AI-Generated & Reconstructed Items A significant portion of the practice questions were generated by artificial intelligence or recreated from partial recollections of past exams shared by colleagues. These AI-generated items may contain factual errors, outdated information, or incorrect medical concepts despite efforts to mirror the format and cognitive level typical of professional radiological examinations. AI-generated content should never be considered as authoritative medical information and must be independently verified. Users should be particularly cautious with AI-generated explanations and always cross-reference with established medical literature. 4. Educational Use Only This material is intended purely for educational practice and should not be used as a substitute for: • Official EBDR study materials or guidelines • Peer-reviewed medical literature • Formal medical education or training • Clinical decision-making or patient care • Professional medical advice or consultation
  • 3.
    5. Potential forMisinformation Users are specifically warned that: • Some content may be outdated or superseded by current medical practice • AI-generated questions may contain subtle but significant medical inaccuracies • Reconstructed questions may not accurately reflect official examination standards • Explanations may contain oversimplifications or errors • This material should never be used as a reference for clinical practice 6. Copyright & Fair Use Portions of past examination questions may be reproduced under fair-use provisions for academic study. All copyrights and trademarks remain the property of their respective owners. Redistribution, commercial use, or reproduction of this PDF without permission from the original rights-holders is strictly prohibited. 7. No Professional or Examination Guarantee This document does not guarantee examination success, confer any professional qualification, or substitute for accredited training, official preparation courses, or comprehensive study programs. Success in professional examinations requires systematic study of official materials, clinical experience, and comprehensive understanding of radiological principles. Reliance on this material alone is at the user's own risk. 8. Recommendation for Official Sources Users are strongly advised to: • Prioritize official EBDR materials and guidelines • Consult peer-reviewed radiological literature • Seek guidance from qualified radiological professionals • Participate in accredited continuing medical education programs • Use this material only as supplementary practice, not primary study resource 9. Liability Limitation To the fullest extent permitted by law, the authors, editors, contributors, and distributors disclaim all liability for any loss, damage, educational setback, examination failure, or any other consequence arising from use, misuse, or reliance on the content, including but not limited to errors, omissions, AI-generated inaccuracies, or outdated information. Users assume full responsibility for verifying all content independently. 10. Feedback & Corrections If you identify any inaccuracies, errors in AI-generated content, outdated information, or have suggestions for improvement, please contact the compiler via: https://linktr.ee/Kareem_Alnakeeb. However, users should not wait for corrections and must independently verify all information. 11. Acknowledgment We gratefully acknowledge the many colleagues and fellow radiologists who generously contributed past exam materials and insights used in compiling this document.
  • 4.
    Part 2 Examination— Structure & Weighting 1. Key Facts • Two written papers taken on consecutive days (Day 1 & Day 2) • Duration: 2 hours per paper • Marks: 100 marks per paper (200 marks total) • Coverage: 8 clinical modules, 4 assessed in each paper 2. Day 1 — Paper I (2 hours, 100 marks) Module Marks Cardiothoracic & Vascular • Including Chest imaging (25 marks) 45 Neuroimaging • Central nervous system • Head & Neck • Spine 25 Pediatric 20 Interventional 10 3. Day 2 — Paper II (2 hours, 100 marks) Module Marks Musculoskeletal (MSK) & Trauma 33–35 Gastro-intestinal Tract (GIT) 30 Genito-urinary Tract (GUT): • Urinary system • Adrenal glands • Male • Female (Obstetrics, Gynecology & Breast) 30 Nuclear Medicine 5
  • 5.
    Table Of Contents June2025 .............................................................................................................................. 1 October 2024....................................................................................................................... 62 July 2024 ............................................................................................................................. 92 Paper 1..................................................................................................................................92 Paper 2................................................................................................................................116 May 2023............................................................................................................................144 September 2022 .................................................................................................................197 May 2022............................................................................................................................261 October 2021......................................................................................................................290 Paper 1................................................................................................................................290 Paper 2................................................................................................................................329 May 2021............................................................................................................................370 Paper 1................................................................................................................................370 Paper 2................................................................................................................................394 September 2020 .................................................................................................................475 Paper 1................................................................................................................................475 Paper 2................................................................................................................................508 September 2019 .................................................................................................................544 Paper 2................................................................................................................................606 April 2019...........................................................................................................................633 Paper 1................................................................................................................................633 Paper 2................................................................................................................................655 September 2017 .................................................................................................................696 April 2017...........................................................................................................................717 Paper 1................................................................................................................................717 Paper 2................................................................................................................................802 October 2015......................................................................................................................846 Paper 2................................................................................................................................846
  • 7.
    EBDR Exam MCQs& Concepts June 2025 Dr. Kareem Alnakeeb 1 June 2025 1. A prominent alanine peak on MR spectroscopy of an intracranial mass, suggests that the mass is most likely A. Cerebral metastasis B. Glioblastoma C. Hemangiopericytoma D. Meningioma E. Oligodendroglioma Source: Radiopaedia 2. A 69-year-old woman presented to her GP with one month’s history of headache and progressive swelling of both hands and face and an 8 months’ history of progressive weight loss. Chest radiograph showed widening of the mediastinum and a large mass in the right upper lobe with a midline trachea. What do you expect the CT chest to show? A. Cavitating lung primary in RUL without mediastinal nodes B. Right Pancoast tumour with rib destruction C. RUL large and multiple small nodules of Wegener’s granulomatosis D. RUL lung primary with retrosternal goitre E. RUL lung primary and SVC obstruction Source: Chin, Teck Yew, et al. Get Through Final FRCR 2A: SBAs. 1st ed., CRC Press, 2017. Explanation: • Superior Vena Cava (SVC) Obstruction • Pathophysiology o Obstruction of the superior vena cava results in impaired venous drainage of the head and neck and upper extremities. • Clinical Manifestations o Clinical manifestations include facial and neck swelling, distended neck veins, headache due to cerebral oedema, dyspnoea, stridor and altered mental status. • Etiology o Cancer is the most common underlying cause of superior vena cava obstruction and this includes lung cancer, mediastinal tumours, lymphoma/lymphadenopathy and mesothelioma, either directly or through malignant mediastinal lymphadenopathy. o Other causes include catheter-induced iatrogenic SVC obstruction, fibrosing mediastinitis and Behcet’s disease. 3. Which of the following statements is true of Moyamoya vasculopathy? A. It cannot present with parenchymal haemorrhage in the adult population B. Moyamoya disease can be seen in patients with sickle cell disease, prior radiotherapy and neurofibromatosis. C. Moyamoya syndrome is commonly considered idiopathic but has a well-recognised familial pattern. D. The most common presentation is a major territory infarction Source: Radiopaedia Explanation: • Moyamoya disease is considered idiopathic and can have an underlying genetic predisposition. • Moyamoya syndrome is associated with other underlying conditions such as sickle cell disease, neurofibromatosis or collagen disorders. • The posterior circulation is commonly involved although it is true that the terminal internal carotid arteries are more commonly affected.
  • 8.
    EBDR Exam MCQs& Concepts June 2025 Dr. Kareem Alnakeeb 2 • The most common presentation is that of watershed infarction either in the anterior, posterior or parasagittal distributions. • Conventional arterial territory infarcts are relatively uncommon due to the diffuse vasculopathy. 4. A 74-year-old man with increased urinary frequency and hesitancy is found to have an enlarged prostate on digital rectal examination. He is referred for a TRUS and biopsy. Which one of the following statements best describes the TRUS findings of benign prostatic hypertrophy (BPH)? A. Dense echogenic foci are seen at the margin of the peripheral and transitional zones. B. The central zone is enlarged. C. The peripheral zone is enlarged and appears homogeneously hypoechoic. D. The peripheral zone is enlarged and is of mixed echogenicity. E. The transitional zone is enlarged. Source: McQueen, Andrew S., et al. Grainger & Allison’s Diagnostic Radiology: Single Best Answer MCQs. 5th ed., Churchill Livingstone, 2009. 5. The central zone atrophies with age while the transitional zone increases in size as it develops BPH. Peripheral zone enlargement is not a feature of BPH. Eight days after lung transplantation for alpha-1 antitrypsin deficiency, a 45 year old man develops pyrexia, breathlessness and desaturation. HRCT reveals perihilar heterogenous opacities and ground glass changes with new pleural effusion and septal thickening. Which of the following is the most likely cause? A. Reperfusion oedema B. Acute rejection C. Anastomotic dehiscence D. Post-transplantation PCP infection E. Hyperacute rejection Source: Currie, Stuart, et al. SBAs for the FRCR 2A. 1st ed., Cambridge University Press, 2010. Explanation: • Hyperacute rejection presents within hours of the transplantation. • Reperfusion oedema usually presents within 24 hours of the transplantation, peaking by about day four. • Posttransplant infections can be broadly divided into those occurring within the first month (gram- negative bacteria, fungi (candida, aspergillosis)) and those occurring after the first month (CMV, PCP). • Anastomotic dehiscence is usually an early feature, but the presentation and features are not those described. 6. A 35-year-old patient received a cadaveric renal transplant 5 days ago and now presents with worsening renal function and decreasing urine output. Which one of the following findings on a Tc-99m DTPA radionuclide scan would favor a diagnosis of acute tubular necrosis (ATN) over acute rejection? A. Delayed renal excretion B. Elevated resistive index greater than 0.7 C. Increased renal perfusion after administration of an ACEI (eg Captopril) D. Poor/impaired graft perfusion E. Preserved renal transplant perfusion Source: McQueen, Andrew S., et al. Grainger & Allison’s Diagnostic Radiology: Single Best Answer MCQs. 5th ed., Churchill Livingstone, 2009. Explanation: • ATN is an early complication in cadaveric allografts and frequently resolves spontaneously in 1—3 weeks. The radionuclide imaging findings of ATN are of preserved perfusion but poor renal function and urine excretion.
  • 9.
    EBDR Exam MCQs& Concepts June 2025 Dr. Kareem Alnakeeb 3 • In acute rejection however, there is both impaired renal function and reduced perfusion on radionuclide imaging. 7. On the unenhanced CT, right adrenal mass is detected and appears homogeneous and has an average density of 7 HU. What is the MOST likely diagnosis? A. Adrenal adenoma B. Adrenal hyperplasia C. Adrenal metastasis D. Focal adrenal hemorrhage Source: McQueen, Andrew S., et al. Grainger & Allison’s Diagnostic Radiology: Single Best Answer MCQs. 5th ed., Churchill Livingstone, 2009. Explanation: • The 10-HU threshold is now the standard by which radiologists differentiate lipid-rich adenomas from most other adrenal lesions on unenhanced CT. • The presence of substantial amounts of intracellular fat is critical in malting the specific diagnosis of adenoma. Up to 30% of adenomas, however, do not have abundant intracellular fat and, thus, show attenuation values greater than 10 HU on unenhanced CT. • Lesions above 10 HU on an unenhanced CT are considered indeterminate and other investigations may be required. 8. A 29-year-old man has an IVU performed following an episode of haematuria. This demonstrates complete right-sided ureteric duplication. Which one of the following statements is true? A. If present, an ectopic ureterocoele is usually related to the lower moiety B. The lower moiety ureter usually obstructs at the vesicoureteric junction. C. The upper moiety calyces are prone to vesicoureteric reflux. D. The upper moiety ureter is prone to ureteric obstruction. E. The upper moiety ureter usually inserts into the bladder superior to the lower moiety ureter. Source: McQueen, Andrew S., et al. Grainger & Allison’s Diagnostic Radiology: Single Best Answer MCQs. 5th ed., Churchill Livingstone, 2009. 9. A 24-year-old motorcyclist is involved in a high-speed accident and is brought to the Emergency Department. He has abdominal guarding and is hemodynamically unstable. An ultrasound abdomen performed in the Emergency Department demonstrates free peritoneal fluid and a laparotomy is performed. In addition to liver and splenic lacerations, the surgeon finds a left retroperitoneal hematoma. Postoperatively, the on-call urologist requests a CT abdomen to assess the left renal injury. Which one of the following findings would indicate a Grade 4 renal laceration? A. Extravasation of contrast from the pelvicalyceal system on delayed phase (5 min) images B. Large (2-cm) subcapsular hematoma C. Perinephric hematoma that extends into the pararenal spaces D. Ill-defined low attenuation change in the lower pole renal cortex E. Segmental renal infarction Source: McQueen, Andrew S., et al. Grainger & Allison’s Diagnostic Radiology: Single Best Answer MCQs. 5th ed., Churchill Livingstone, 2009. Explanation: • A deep renal laceration that extends into the collecting system is indicative of a grade 4 injury.
  • 10.
    EBDR Exam MCQs& Concepts June 2025 Dr. Kareem Alnakeeb 4 10. A 60-year-old nulliparous woman presents with postmenopausal bleeding. On transvaginal ultrasound, her endometrium is 8 mm thick and the endomyometrial junction appeared indistinct. The radiologist suspects invasive endometrial cancer and refers her for an MRI examination. What are the likely findings on MRI? A. On unenhanced Tlw images the endometrial cancer appears of high signal intensity compared to the surrounding myometrium. B. On contrast-enhanced Tlw images, endometrial cancer shows avid enhancement compared with surrounding myometrium. C. On T2w images the normally high signal junctional zone is disrupted. D. Tlw fat-saturated sequences are best used to assess the junctional zone. E. The endometrial cancer demonstrates delayed/little enhancement compared to the normal surrounding myometrium on postcontrast Tlw images. Source: McQueen, Andrew S., et al. Grainger & Allison’s Diagnostic Radiology: Single Best Answer MCQs. 5th ed., Churchill Livingstone, 2009. 11. Regarding eosinophilic granuloma: (True or False) A. Lesions in proximal long bones are usually diaphyseal B. The commonest site is the skull C. On MRI, it appears as a well defined lesion of low signal intensity on T1 D. Lesions rarely elicit a periosteal reaction E. It is a recognised cause of ‘floating teeth’ appearance Source: Hussain, Shahid, et al. Radiology MCQs for the New FRCR Part 2A. 1st ed., TFM Publishing, 2006. Explanation: a) False - 0.01% of all trauma patients are affected b) True - patchy osteopaenia is seen in 50% as early as 2-3 weeks after symptom onset c) True d) True e) False - this is an end-stage feature 12. Which feature would be most helpful to distinguish chordoma from chondrosarcoma at the clivus? A. bone destruction B. enhancement like a pumice stone C. Midline location D. T2 hyperintense signal Source: Radiopaedia Explanation: • Chordoma is a midline tumor because it arises from the notochordal remnant. • Chondrosarcoma arises at the petro-occipital synchondrosis and therefore is centered off the midline. • Both chordoma and chondrosarcoma cause bone destruction and are T2 hyperintense. • Chondrosarcoma enhances like a pumice stone, whereas chordoma often has a more honeycomb enhancement pattern.
  • 11.
    EBDR Exam MCQs& Concepts June 2025 Dr. Kareem Alnakeeb 5 13. A 35-year-old woman with loin pain, dysuria and fever is diagnosed with acute uncomplicated ascending bacterial pyelonephritis. What appearance is likely to have been seen on CT? A. Cortical thinning B. Perinephric fat stranding C. Alternating bands of hypo- and hyperattenuation of the renal parenchyma D. Round peripheral hypoattenuating renal lesions E. Geographic low-attenuation lesion with peripheral enhancement Source: Proctor, Robin. Final FRCR Part A Modules 4-6 Single Best Answer MCQs: The SRT Collection of 600 Questions with Explanatory Answers (MasterPass). 1st ed., CRC Press, 2009. Explanation: • This appearance is analogous to the striated nephrogram seen on excretory urography and reflects the underlying pathology of tubular obstruction, interstitial oedema and vasospasm. • On delayed (3-6 hours) imaging, the hypoattenuating regions show delayed and persistent enhancement due to prolonged accumulation and transit of contrast through the collecting system. • In haematogenous seeding of pyelonephritis (from staphylococcal or streptococcal infections), round peripheral hypoattenuating lesions are seen. They can mimic neoplasia if pyelonephritis is not suspected clinically. 14. A 30-year-old woman presents acutely with seizures, fever and headache, followed by rapid deterioration to coma. Emergency MRI shows asymmetrical swelling of the anterior temporal lobes on T1W images. T2W images reveal concordant asymmetrical but bilateral areas of high signal in the anterior temporal lobes, insular cortices and hippocampi. There is no enhancement following administration of intravenous gadolinium. What is the most likely condition? A. lymphoma B. HIV encephalitis C. cytomegalovirus encephalitis D. herpes simplex encephalitis E. toxoplasmosis Source: Bydder, Megan, et al. Get Through Final FRCR Part A: SBAs for the Modular Examination. 1st ed., Hodder Education Publishers, 2009. Explanation: Epidemiology and Etiology • Herpes simplex virus is the most common cause of fatal endemic encephalitis, often leaving survivors with severe memory and personality problems. • Both oral (type 1) and genital (type 2) strains may produce encephalitis with a multimodal distribution, affecting neonates (due to cross-infection with type 2 from the mother during birth), children and adults. • Clinical Presentation • Childhood and adult infection is caused by the type 1 virus and results in fulminant necrotizing encephalitis presenting with acute confusion and deteriorating rapidly to coma. • Focal neurological deficits are seen in only 30% of cases. Imaging Characteristics • The virus asymmetrically affects the temporal lobes, insula, orbitofrontal region and cingulate gyrus, causing oedema. • This is seen as high signal on T2W/FLAIR images, with DWI appearances variable depending on the presence of infarction. • The putamen is characteristically spared, and the areas of encephalitis typically do not show enhancement on CT or MRI.
  • 12.
    EBDR Exam MCQs& Concepts June 2025 Dr. Kareem Alnakeeb 6 15. A 63 year old male patient is seen in the vascular clinic with a pulsatile mass posterior to his right knee. Ultrasound confirms a 2-cm right popliteal artery aneurysm. The vascular team request a CT angiogram to help plan management. Which imaging protocol is most appropriate? A. Arterial phase CT from aortic bifurcation to toes B. Arterial phase CT from neck to toes C. Arterial phase CT from aortic bifurcation to knees D. Arterial phase CT from diaphragm to toes E. Arterial phase CT from pubic symphysis to toes Source: Rabone, Amanda, et al. The Final FRCR Self-Assessment (MasterPass). 1st ed., CRC Press, 2020. Explanation: Abdominal Aortic Assessment • Approximately 30-50% of popliteal artery aneurysms are associated with an abdominal aortic aneurysm; therefore the abdominal aorta needs to be included on the study. Contralateral Lower Limb Assessment • In lower limb studies, the contralateral lower limb will automatically be included on the study and assessment of the contralateral popliteal artery is also vital, as they can be bilateral. Vascular Considerations for Interventional Planning • Other important factors to consider when planning interventional radiological management is the tortuosity of the common and external iliac arteries and mural calcification, particularly at the femoral vessels where vascular access will be required for endovascular repair. Peripheral Vessel Evaluation and CT Extent • The CT should extend to the toes to assess the vessel quality peripheral to the aneurysm, as popliteal aneurysms are associated with peripheral thromboembolic complications. 16. A series of neonatal radiographs reveal a narrow thorax with short ribs, square iliac wings with horizontal acetabular roofs, short sacrosciatic notches, progressive narrowing of the interpedicular distance and posterior scalloping of the vertebral bodies. What is the most likely diagnosis? A. Achondroplasia B. Campomelic dysplasia C. Cleidocranial dysplasia D. Ellis-van Creveld syndrome E. Morquio's syndrome Source: McQueen, Andrew S., et al. Grainger & Allison’s Diagnostic Radiology: Single Best Answer MCQs. 5th ed., Churchill Livingstone, 2009. Explanation: • The iliac wings in Morquio's syndrome are characteristically flaredTather than 17. A low flat renogram curve indicates: A. Advanced nephropathy B. Complete obstruction to urine outflow C. Vesico-ureteric reflux D. Partial obstruction to urine outflow Explanation: (by Perplexity AI, generated with the OpenAI O3 model) • A very low, almost horizontal time–activity curve reflects markedly reduced tracer extraction by damaged renal parenchyma, typical of end-stage or advanced medical renal disease. • Complete or partial outflow obstruction instead produces an uptake phase followed by a rising or plateau phase; vesico-ureteric reflux alters the post-void segment rather than the primary renographic curve
  • 13.
    EBDR Exam MCQs& Concepts June 2025 Dr. Kareem Alnakeeb 7 18. Chron's disease of small intestine: A. Terminal ileum is affected in 80% of case. B. Colon is affected in 10% of case. C. Treated surgically. D. Presenting with deep penetrating ulcers. Explanation: (by Perplexity AI, generated with the OpenAI O3 model) • The terminal ileum is the most frequent site; up to 80% of patients show involvement here • Pure colonic (isolated) Crohn’s disease occurs in ≈20% of patients, while another ≈50% have both ileal and colonic disease. • Surgery helps manage complications (obstruction, fistula, perforation) but it is not first-line therapy and cannot cure the disease; most patients start with medical treatment and only 50–70% require an operation during their lifetime • Transmural inflammation produces deep linear/penetrating ulcers and fissures, giving the classic “cobblestone” mucosal pattern 19. Central dot sign is seen in: A. Caroli disease B. Primary sclerosing cholangitis C. Polycystic liver disease D. Liver hamartoma Explanation: (by Perplexity AI, generated with the OpenAI O3 model) • The central dot sign is an enhancing portal venous radicle and fibrovascular bundle seen within fluid-filled, ectatic intrahepatic bile ducts; it is virtually pathognomonic for Caroli’s disease because the ducts are massively dilated yet remain in continuity with portal triads. • Primary sclerosing cholangitis produces multifocal strictures and beading without a central enhancing dot. • Simple hepatic cysts are avascular, with thin walls and no intraluminal structures. • Hepatic mesenchymal hamartoma appears as a multicystic mass lacking patent biliary connections. • Cavernous hemangiomas show peripheral nodular enhancement progressing centrally, not a single central dot. 20. Most Common cause of Urethral Stricture: A. Infection B. Trauma C. Iatrogenic D. Congenital Source: Radiopaedia
  • 14.
    EBDR Exam MCQs& Concepts June 2025 Dr. Kareem Alnakeeb 8 21. Regarding porcelain gallbladder: (True or False) A. It is often symptomless B. It is rarely associated with gallstones C. Oral cholecystogram shows a non-functioning gallbladder D. 60-70% develop carcinoma of the gallbladder E. Acute pancreatitis is a recognised cause Source: Hussain, Shahid, et al. Radiology MCQs for the New FRCR Part 2A. 1st ed., TFM Publishing, 2006. Explanation: • A. True • B. False - 90% • C. True • D. False - 10-20% • E. False 22. Which of the following statements regarding splenic lymphoma is CORRECT? A. The spleen is involved at presentation in 30-40% of patients with non Hodgkin's lymphoma B. Focal splenic deposits are usually well defined, round lesions of increased brightness on ultrasound C. When there is lymphomatous involvement of the spleen, splenomegaly is seen in 70-80% D. Splenic lymphoma deposits commonly calcify E. Lymph nodes are seen in the splenic hilum in 50% of patients with Hodgkin's lymphoma Source: Hussain, Shahid, et al. Radiology MCQs for the New FRCR Part 2A. 1st ed., TFM Publishing, 2006. Explanation: • a) True- slightly higher for Hodgkin's lymphoma • b) False - reduced echogenicity • c) False - 50% • d) True • e) False – uncommon 22. The following statements concerning esophageal carcinoma are true: A. 90% of cases are squamous cell carcinomas B. Most commonly located in the upper third of the esophagus C. Plummer-Vinson syndrome is a recognised predisposing factor D. Commonest appearance on double contrast barium swallow is of a large ulcer within a bulging mass E. It is associated with ulcerative colitis Source: Hussain, Shahid, et al. Radiology MCQs for the New FRCR Part 2A. 1st ed., TFM Publishing, 2006. Explanation: A. True B. False - 20% in the upper third, 30-40% middle third and 30-40% in lower third C. True D. False - polypoid/fungating form is commonest E. False - predisposing factors include Barrett’s oesophagus, alcohol abuse, smoking, coeliac disease, achalasia, tylosis
  • 15.
    EBDR Exam MCQs& Concepts June 2025 Dr. Kareem Alnakeeb 9 23. A 2-year-old child presents with fever, erythema of the oral mucosa with chest and abdominal pain. Echocardiography reveals the presence of a coronary arterial aneurysms. An underlying vasculitis is suspected. Which of the following statements is least accurate in this clinical setting? A. Aneurysms are typically seen in the proximal segments of the coronary arteries. B. Aneurysms less than 5 mm in diameter are considered small. C. Smaller aneurysms have a higher likelihood of thrombosis. D. Multiple coronary artery aneurysms are more common than isolated aneurysms. E. The most common site for a coronary aneurysm is in the left anterior descending artery. Source: Chin, Teck Yew, et al. Get Through Final FRCR 2A: SBAs. 1st ed., CRC Press, 2017. Explanation: • Overview o Kawasaki’s disease is a common paediatric vasculitis of medium-sized vessels, with coronary vasculitis being the hallmark manifestation. o It is the leading cause of acquired heart disease in children in developed countries. • Coronary Arterial Aneurysms o Coronary arterial aneurysms typically occur within the subacute phase of the disease and may be associated with sudden cardiac death. o The aneurysms typically develop in the proximal segments of major coronary arteries and affect the left anterior descending artery followed by the proximal right coronary arteries in frequency of location. o Smaller aneurysms, (<5 mm in diameter) are more likely to regress than larger aneurysms (>8 mm in diameter), which have a higher likelihood of thrombosis and infarction. 24. A specialty trainee from the medical ward shows you a CXR of a breathless patient. You observe splaying of the carina and a ’double right heart border’. What is the most likely underlying diagnosis? A. Mitral stenosis. B. Aortic stenosis. C. Tricuspid incompetence. D. Left ventricular aneurysm. E. Coarctation of the aorta. Source: Lindsay, Richard, et al. SBAs for the FRCR Part 2A. 1st ed., Oxford University Press, 2012. Explanation: • The findings describe left atrial enlargement, which is caused by mitral valve disease (stenosis or incompetence), ventricular septal defect (VSD), patent ductus arteriosus (PDA), atrial septal defect (ASD) with shunt reversal, and left atrial myxoma. • Aortic stenosis produces left ventricular hypertrophy and eventually dilatation, the latter producing a prominent left heart border with inferior displacement of the cardiac apex. • A left ventricular aneurysm produces a prominent bulge of the left heart border. • Tricuspid incompetence produces an enlarged right atrium and thus a prominent right heart border on plain film. • Coarctation produces left ventricular enlargement and inferior rib notching of the fourth to eighth ribs bilaterally if conventional and a ‘reverse figure 3’ sign: a prominent ascending aorta/arch and a small descending aorta, with an intervening notch.
  • 16.
    EBDR Exam MCQs& Concepts June 2025 Dr. Kareem Alnakeeb 10 25. Regarding hepatocellular carcinoma: (True or False) A. It is the commonest primary visceral malignancy in the world B. Haemochromatosis is a recognized cause C. Elevated alpha-fetoprotein is found in 50-60% of cases D. Has a higher incidence in macronodular than micronodular cirrhosis E. On MR, hepatoma has a well defined, hypointense capsule on T1 weighted images Source: Hussain, Shahid, et al. Radiology MCQs for the New FRCR Part 2A. 1st ed., TFM Publishing, 2006. Explanation: • True • True - other causes are cirrhosis, hepatitis, alpha-1 antitrypsin deficiency, Wilson’s disease, aflatoxin, and thorotrast. • c) False - 90% • d) True • e) False - increased signal intensity on a T2 weighted image. Peripheral gadolinium enhancement is seen in about 20% 26. A 42-year-old man presents to the Emergency Department with a 7-day history of severe bloody diarrhoea and abdominal pain. He has previously been fit and well with no significant medical history. On examination, the patient is dehydrated with generalized abdominal tenderness but no clinical evidence of peritonism. An abdominal radiograph is performed. Which radiographic finding would be most suggestive of a toxic megacolon? A. Caecum measuring 4.5 cm in diameter B. Multiple mucosal islands in a dilated transverse colon C. Pseudodiverticulae in the descending colon D. Thickened haustrae throughout the entire colon E. ‘Thumbprinting’ of the transverse and descending colon Source: McQueen, Andrew S., et al. Grainger & Allison’s Diagnostic Radiology: Single Best Answer MCQs. 5th ed., Churchill Livingstone, 2009. Explanation: • The presence of severe ulceration leading to mucosal islands is a major sign of toxic megacolon (the other key finding is colonic dilatation > 5 cm). 27. Regarding superior mesenteric artery (SMA) syndrome, which ONE of the following statements is correct? A. It most commonly develops after significant weight gain that increases retroperitoneal fat. B. It occurs when the superior mesenteric artery is compressed between the third part of the duodenum and the abdominal aorta. C. It can be diagnosed on cross-sectional imaging when the aorto-mesenteric angle is < 25° and the aorto-mesenteric distance is < 8 mm. D. Posterior Nutcracker syndrome is a recognized gastrointestinal presentation of this condition. Explanation: (by Perplexity AI, generated with the OpenAI O3 model) • Pathophysiology: SMA syndrome results from compression of the third part of the duodenum between the SMA and aorta, usually after rapid weight loss that reduces the fat pad1. • Imaging criteria: An aorto-mesenteric angle < 22–25° and distance < 8 mm on CT or ultrasound strongly suggest the diagnosis. • Nutcracker syndrome involves left renal vein compression and is not a clinical presentation of SMA syndrome.
  • 17.
    EBDR Exam MCQs& Concepts June 2025 Dr. Kareem Alnakeeb 11 28. A 75-year-old diabetic man underwent a left below knee amputation 3 months ago for osteomyelitis of the distal tibia. Since then, he has experienced recurrent episodes of fever and malaise. MRI is contraindicated due to a metallic aortic valve. Which is the best investigation to exclude an occult focus of osteomyelitis? A. CT B. US C. Scintigraphy using gallium D. Scintigraphy using indium-labelled white cells E. Scintigraphy using technetium (Tc-99m) monodiphosphonate Source: McQueen, Andrew S., et al. Grainger & Allison’s Diagnostic Radiology: Single Best Answer MCQs. 5th ed., Churchill Livingstone, 2009. Explanation: • Although an indium-labelled white cell study is more specific, a bone scintigram using Tc-99m monodiphosphonate is a more sensitive test to exclude osteomyelitis. 29. An 18-year-old woman with Poland syndrome is being assessed by plastic surgery for reconstruction. As part of her pre-operative work-up a CT chest is requested. What is the classic fi nding in this disorder? A. Absence of the sternal head of pectoralis major. B. Hypoplastic clavicles. C. Anterior protrusion of the ribs. D. Bilateral breast aplasia. E. Anterior protrusion of the sternum. Source: Lindsay, Richard, et al. SBAs for the FRCR Part 2A. 1st ed., Oxford University Press, 2012. Explanation: • Poland syndrome is an uncommon congenital unilateral chest wall deformity characterized by partial or total absence of the greater pectoral muscle and ipsilateral syndactyly. • Associated anomalies include ipsilateral breast aplasia and atrophy of the second to fifth ribs. • Hypoplastic clavicles are a feature of cleidocranial dysostosis. • Anterior protrusion of the ribs gives rise to pectus excavatum, whereas anterior protrusion of the sternum is seen in pectus carinatum. 30. Which of the following is NOT one of the 4 regions considered when assessing MRI scans for the dissemination in space component of McDonald criteria for multiple sclerosis (2017 version)? A. cortical and/or juxtacortical B. infratentorial C. periventricular D. spinal cord E. subcortical Source: Radiopaedia Explanation: • Dissemination in space assesses for the presence of lesions in the following 4 regions: o periventricular (≥1 lesion, unless the patient is over the age of 50 in which case it is advised to seek a higher number of lesions) o cortical or juxtacortical (≥1 lesion) o infratentorial (≥1 lesion) o spinal cord (≥1 lesion) • Notably, T2-hyperintense lesions of the optic nerve, such as those in a patient presenting with optic neuritis, cannot be used in fulfilling the 2017 revised McDonald criteria. • Subcortical lesions (those neither abutting the ventricles nor the cortex) are also not counted.
  • 18.
    EBDR Exam MCQs& Concepts June 2025 Dr. Kareem Alnakeeb 12 31. A 53-year-old male smoker is under evaluation for a thoracic aortic aneurysm. Whilst reviewing pre- and post-contrast CT images of the chest, a 2.4 cm lesion is seen in the left lower lobe, with a mural nodule. Which of the following features most favors a diagnosis of cavitating lung cancer rather than an intracavitatory apergilloma? A. Size of lesion B. Contrast enhancement > 10HU C. Wall thickness D. Adjacent bronchiectasis E. Volume loss in involved lobe Source: Leen, Edward. Succeeding in the New FRCR Part 2A Exam: Single Best Answer (SBA) Revision Questions for Modules 1-6. 1st ed., UNKNO, 2011. Explanation: • Degree of contrast enhancement is much higher in cavitating lung tumors. • Adjacent bronchiectasis is more often seen in aspergillomas. 32. Causes of ‘tree in bud’ appearance include all, except A. Tuberculosis B. Allergic bronchopulmonary aspergillosis C. Cystic fibrosis D. Tumor emboli E. Chronic pulmonary embolism Source: Chin, Teck Yew, et al. Get Through Final FRCR 2A: SBAs. 1st ed., CRC Press, 2017. Explanation: • Radiologic Appearance o The tree-in-bud pattern on HRCT is characterized by small centrilobular nodules of soft- tissue attenuation connected to multiple branching linear structures of similar calibre originating from a single stalk. • Causes and Associated Disorders o Initially described in cases of endobronchial Mycobacterium tuberculosis, it has subsequently been reported in peripheral airways diseases such as infection (bacterial, fungal, viral or parasitic), congenital disorders (like cystic fibrosis and Kartagener’s syndrome), idiopathic disorders (obliterative bronchiolitis, panbronchiolitis), aspiration, inhalation, immunologic disorders (like ABPA), connective tissue disorders and peripheral pulmonary vascular diseases such as neoplastic pulmonary emboli. 33. A patient with known tuberous sclerosis had a routine follow-up CT. A 3 x 2-cm partly calcified heterogeneously enhancing lesion was seen at the level of the foramen of Monro. What is the most likely pathology? A. Colloid cyst B. Subependymal giant cell astrocytoma C. Intraventricular D. Meningioma E. Germinoma Source: Proctor, Robin. Final FRCR Part A Modules 4-6 Single Best Answer MCQs: The SRT Collection of 600 Questions with Explanatory Answers (MasterPass). 1st ed., CRC Press, 2009. Explanation: • 15% of patients with tuberous sclerosis develop subependymal astrocytomas. They typically occur at the foramen of Monro and are usually a well-defined rounded mass with some calcification. They usually enhance uniformly with contrast and can degrade to a high-grade astrocytoma.
  • 19.
    EBDR Exam MCQs& Concepts June 2025 Dr. Kareem Alnakeeb 13 • 95% of tuberous sclerosis patients have subependymal hamartomas. These occur in the periventricular region, are isointense to white matter on Ti and calcified on CT. • 55% of patients have cortical tubers, which are high signal on T2-weighted imaging. 34. A 29-year-old woman with fever, malaise, fatigue, intermittent pain and numbness in both hands and feet, and normal chest radiograph is referred for MRI thorax. MRI shows wall thickening of the origin of the right subclavian artery and both carotid arteries. What is the diagnosis? A. Moyamoya disease B. Takayasu arteritis C. Churg–Strauss disease D. PAN E. Wegener’s granulomatosis Source: Chin, Teck Yew, et al. Get Through Final FRCR 2A: SBAs. 1st ed., CRC Press, 2017. Explanation: Takayasu arteritis • Takayasu arteritis is a form of granulomatous vasculitis affecting large and medium-sized arteries, characterised by ocular disturbances and weak pulses in the upper extremities (pulseless disease). • It is associated with fibrous thickening of the aortic arch with narrowing of the origins of the great vessels at the arch. • Takayasu arteritis can be limited to the descending thoracic and abdominal aorta. • It is seen in young and middle-aged patients, especially Asian and women. • The diagnosis is confirmed by a characteristic arteriographic pattern of irregular vessel walls, stenosis, post-stenotic dilatation, aneurysm formation, occlusion and evidence of increased collateral circulation. Polyarteritis nodosa • Polyarteritis nodosa is a fibrinoid necrotising vasculitis that mainly involves small and medium-sized arteries of the muscles. • Multiple aneurysm formation is a characteristic finding. • The kidney is most commonly involved, followed by the GI tract, liver, spleen and pancreas. • Positive ANCA titres (usually pANCA type) are found in variable percentages of patients. Wegener’s granulomatosis • Wegener’s granulomatosis is a distinct clinicopathologic entity characterised by granulomatous vasculitis of the upper and lower respiratory tract together with glomerulonephritis. Churg–Strauss syndrome • Churg–Strauss syndrome is characterised by granulomatous vasculitis of multiple organ systems, particularly the lung, and involves both arteries and veins as well as pulmonary and systemic vessels. Moyamoya disease • Moyamoya disease is a progressive vasculopathy leading to stenosis of the main intracranial arteries. • Characteristic angiographic features of the disease include stenosis or occlusion of the arteries of the circle of Willis, as well as the development of collateral vasculature that produces a typical angiographic image called ‘clouds of smoke’ or ‘puff of cigarette smoke’.
  • 20.
    EBDR Exam MCQs& Concepts June 2025 Dr. Kareem Alnakeeb 14 35. The unrestrained passenger of a vehicle involved in a high-energy road traffic accident is admitted with a ‘hangman's fracture’. What is the most likely appearance on plain film? A. Fractures through the neural arch of Cl B. Fractures through the neural arch of C2 C. Fracture of the spinous process of C7 D. Transverse fracture through the base of the dens E. Wedge compression fracture of an upper cervical vertebra Source: McQueen, Andrew S., et al. Grainger & Allison’s Diagnostic Radiology: Single Best Answer MCQs. 5th ed., Churchill Livingstone, 2009. 36. A 46-year-old female presented to the Emergency Department in heart failure with a history of chest pain. A chest radiograph demonstrated an enlarged heart and bilateral pleural effusions. What imaging finding would suggest a diagnosis of constrictive pericarditis rather than restrictive cardiomyopathy? A. A pericardial thickness on CT of 4mm B. Dilated right atrium C. Ascites D. Absence of ventricular hypertrophy E. Global subendocardial late gadolinium enhancement on MRI Source: Proctor, Robin. Final FRCR Part A Modules 4-6 Single Best Answer MCQs: The SRT Collection of 600 Questions with Explanatory Answers (MasterPass). 1st ed., CRC Press, 2009. Explanation: • Although pericardial thickening does not confirm a diagnosis of constrictive pericarditis, when the differential is between that and a restrictive cardiomyopathy it favours the former. • B, C and D are non-discriminatory. E is occasionally found in restrictive cardiomyopathy. 37. A 32-year-old male presents with increasing shortness of breath following a road traffic accident, in which he sustained multiple long bone fractures. At 48 hours post-injury, his chest radiograph is normal. The next day a V /Q scan shows patchy, mottled, peripheral perfusion defects. The following day a chest radiograph shows patchy, bilateral, alveolar infiltrates. What is the most likely diagnosis? A. fat embolism B. thrombotic embolism C. atypical infection D. pulmonary contusions E. pulmonary oedema Source: Bydder, Megan, et al. Get Through Final FRCR Part A: SBAs for the Modular Examination. 1st ed., Hodder Education Publishers, 2009. Explanation: • In a patient who has sustained multiple fractures, fat embolism should always be considered when a patient is short of breath in the presence of a normal chest radiograph. This manifests on a V /Q scan as mottled peripheral perfusion defects. • Chest radiograph remains normal for up to 72 hours, when discoid atelectasis, diffuse alveolar infiltrates and consolidation may develop. • Fat embolism may precede the development of acute respiratory distress syndrome. • Pulmonary contusions usually manifest earlier, within the first 24 hours.
  • 21.
    EBDR Exam MCQs& Concepts June 2025 Dr. Kareem Alnakeeb 15 38. A 6-year-old with spina bifida has a chest X-ray performed for possible lower respiratory tract infection. The lungs are clear but there is a well-defined, round paraspinal mass with an air–fluid level. What is the most likely diagnosis? A. Bronchogenic cyst B. Morgagni hernia C. Oesophageal duplication cyst D. Cystic teratoma E. Oesophageal tumour Source: Chin, Teck Yew, et al. Get Through Final FRCR 2A: SBAs. 1st ed., CRC Press, 2017. Explanation: Esophageal duplication cysts • Esophageal duplication cysts are rare congenital anomalies. • They are associated with vertebral anomalies (spina bifida, hemivertebrae, fusion defects). • There is also an association with esophageal atresia and small bowel duplication. • Most cysts develop in the right posteroinferior mediastinum. • CT demonstrates a well-marginated round, oval or tubular-shaped fluid-filled cystic structure that has a well-defined, thin wall. • The cyst is of water attenuation with no enhancement of contents and no infiltration of surrounding structures. • Malignant degeneration is rare. Bronchogenic cyst • Bronchogenic cyst is the most common cystic mediastinal mass that typically lies in the middle mediastinum, not in a paraspinal location; in addition, you would not expect an air–fluid level. Other mediastinal lesions • Cystic teratoma is an anterior mediastinal mass. • Morgagni hernia would be unlikely to cause a solitary round lesion; multiple structures would be expected. 39. What is the typical pattern of late gadolinium enhancement seen in hypertrophic cardiomyopathy? A. Diffuse subendocardial enhancement B. Patchy mid-wall enhancement, often at the RV insertion points or in the most hypertrophied segments C. Transmural enhancement in a coronary distribution D. No enhancement is typically seen Explanation: • Cardiac-MRI studies of hypertrophic cardiomyopathy consistently show focal, non-ischaemic LGE that is o mid-myocardial rather than subendocardial or transmural, o often multifocal and “patchy”, and o frequently concentrated where the hypertrophied septum meets the RV free wall (the RV insertion sites) or in areas of maximal wall thickness.
  • 22.
    EBDR Exam MCQs& Concepts June 2025 Dr. Kareem Alnakeeb 16 40. A 72-year-old man presents to the vascular surgeon with abdominal pain 4 months after endovascular repair of an abdominal aortic aneurysm. An emergency dual phase contrast-enhanced CT is performed. The unenhanced images reveal high-density material interposed between the stent and the wall of the aorta. There is further enhancement of this high-density area on arterial phase images. The graft and the attachments look intact. What is the most likely diagnosis? A. Type I endoleak B. Type II endoleak C. Type III endoleak D. Type IV endoleak E. Type V endoleak Source: Chin, Teck Yew, et al. Get Through Final FRCR 2A: SBAs. 1st ed., CRC Press, 2017. Explanation: Type I Endoleak • In a Type I endoleak, there is poor apposition between one of the attachment sites of a stent graft and the native aortic or iliac artery wall, and blood leaks through this defect into the aneurysm sac. • A Type I endoleak can be seen immediately after stent-graft deployment. • On CT, dense contrast collection is usually seen centrally within the sac and is often continuous with one of the attachment sites. Type II Endoleak • Type II endoleaks are the most common. • They occur when there is retrograde flow of blood into the aneurysm sac via an excluded aortic branch, most commonly IMA or a lumbar artery. • Many Type II endoleaks close spontaneously over time. • CT shows peripheral or central location of acute haemorrhage. Type III Endoleak • Leakage of blood through the body of a stent graft results in a Type III endoleak. • Type III endoleaks manifest as collections of haemorrhage or contrast material centrally within the aneurysm sac, usually distant from the attachment sites or native vessels. Type IV Endoleak • Opacification of the aneurysm sac immediately after placement of a stent graft without a discernible source of leakage is designated a Type IV endoleak. Type V Endoleak (Endotension) • A Type V endoleak, or endotension, is characterised by continued growth of an excluded aneurysm sac without direct radiologic evidence of a leak. 41. A 60-year-old female presents with a history of facial pain and diplopia. Clinical examination reveals palsies of the III, IV, and VI cranial nerves, Horner’s syndrome, and facial sensory loss in the distribution of the ophthalmic and maxillary divisions of the trigeminal (V) cranial nerve. Where is the causative abnormality located? A. Dorello’s canal. B. Cavernous sinus. C. Superior orbital fissure. D. Inferior orbital fissure. E. Meckel’s cave. Source: Lindsay, Richard, et al. SBAs for the FRCR Part 2A. 1st ed., Oxford University Press, 2012. Explanation: Anatomy of the Cavernous Sinus
  • 23.
    EBDR Exam MCQs& Concepts June 2025 Dr. Kareem Alnakeeb 17 • Cranial nerves III, IV, and VI, and ophthalmic (V1) and maxillary (V2) divisions of the V cranial nerve course through the cavernous sinus along with the internal carotid artery. • The V2 division of the trigeminal nerve passes through the inferior portion of the cavernous sinus and exits via the foramen rotundum. • The remainder of the cranial nerves mentioned above enter the orbit via the superior orbital fissure. Clinical Implications • The cavernous sinus location accounts for these features. • Palsies of cranial nerves III, IV, and VI result in ophthalmoplegia. • Involvement of V1 and V2 divisions of the trigeminal nerve produces facial pain and sensory loss; involvement of sympathetic nerves around the internal carotid artery results in Horner’s syndrome. • This cluster of findings is found in Tolosa Hunt syndrome, an idiopathic inflammatory process involving the cavernous sinus. 42. Features of diaphyseal aclasia (hereditary multiple exostosis) include: (true or false) A. Autosomal recessive inheritance B. Exostoses have a cap of hyaline cartilage, often with a bursa formation over the cap C. Exostoses arise from the metaphysis and point towards the joint D. Exostoses stop growing when the nearest epiphyseal centre fuses E. Malignant transformation to chondrosarcoma occurs in 35-40% Source: Hussain, Shahid, et al. Radiology MCQs for the New FRCR Part 2A. 1st ed., TFM Publishing, 2006. Explanation: • a) False - AD, presents at 2-10 years of age • b) True • c) False - arise from metaphysis of long bones near epiphyses and point away from the joint • d) True • e) False - <5% 43. On imaging, which of the followings causes 'Bone within Bone' appearance? (true or false) A. Marfan's syndrome B. Sickle cell disease C. Rickets. D. Fibrous dysplasia Source: Hussain, Shahid, et al. Radiology MCQs for the New FRCR Part 2A. 1st ed., TFM Publishing, 2006. Explanation: The causes include: • Congenital syphilis • Infantile cortical hyperostosis • Sickle cell disease • Oxalosis • Paget's disease • Acromegaly and radiation
  • 24.
    EBDR Exam MCQs& Concepts June 2025 Dr. Kareem Alnakeeb 18 44. Concerning dislocations: (True Or False) A. Anterior dislocation of the hip accounts for 10-20% of all hip dislocations B. Posterior dislocations of both radius and ulna account for 80-90% of elbow dislocations C. Anterior dislocation of the shoulder accounts for more then 90% of glenohumeral dislocations D. A Bankhart lesion is a fracture of the anterior aspect of the superior rim of the glenoid E. Dislocation of the patella is usually medial Source: Hussain, Shahid, et al. Radiology MCQs for the New FRCR Part 2A. 1st ed., TFM Publishing, 2006. Explanation: • a) True - lies medial and inferior to acetabulum on pelvis X-ray • b) True - isolated dislocation of the radial head is rare • c) True - 97% are anterior dislocations. Associated with a Hill-Sachs lesion which is a defect in the posterolateral aspect of the humeral head • d) False - inferior rim • e) False – lateral 45. Which of the following statements is CORRECT? (true or false) A. Paget’s disease has a prevalence of 10% in people over the age of 80 years of age B. Ankylosing spondylitis is found more commonly in Black than Caucasian populations C. Developmental dysplasia of the hip is more common in males D. Diffuse idiopathic skeletal hyperostosis commonly presents in children E. The highest incidence of fibrous dysplasia is between 30-50 years of age Source: Hussain, Shahid, et al. Radiology MCQs for the New FRCR Part 2A. 1st ed., TFM Publishing, 2006. • A) True- unusual in under 40-year-olds • b) False - ratio of 3:1 • c) False - much commoner in girls • d) False - usually presents in over 50-year-olds • e) False - the highest incidence of fibrous dysplasia is at 3-15 years of age. 75% are seen below 30 years of age 46. A 54-year-old man presents with a swelling in his right popliteal fossa. A Baker’s cyst is suspected clinically and an ultrasound scan is arranged. This confirms a complex cystic structure with debris. To help confirm this is a Baker’s cyst, you look for a communication of this cyst with fluid at the posterior aspect of the knee joint between which two tendons? A. Semitendinosis and lateral head of gastrocnemius. B. Semitendinosis and medial head of gastrocnemius. C. Semitendinosis and semimembranosis. D. Medial and lateral heads of gastrocnemius. E. Lateral head of gastrocnemius and semimembranosis. F. Medial head of gastrocnemius and semimembranosis. Source: Lindsay, Richard, et al. SBAs for the FRCR Part 2A. 1st ed., Oxford University Press, 2012. Explanation: • Identification of anechoic cysts communicating with fluid between the semimembranosis and gastrocnemius tendons confirms the diagnosis of Baker’s cyst. • It is important to perform further imaging if the mass in the posterior compartment lacks signs of communication with fluid between the semimembranosis and medial gastrocnemius tendons. • If this is the case, there are other possibilities for the lesion, including meniscal cyst or even a myxoid sarcoma.
  • 25.
    EBDR Exam MCQs& Concepts June 2025 Dr. Kareem Alnakeeb 19 47. Skeletal features of thalassemia major include: A. Central nidus B. Bone sclerosis C. Narrowing of medullary cavity D. Premature fusion of epiphysis Source: Hussain, Shahid, et al. Radiology MCQs for the New FRCR Part 2A. 1st ed., TFM Publishing, 2006. Explanation: • Other features include: o Erlenmeyer flask deformity o Arthropathy - secondary to haemochromatosis and CPPD o Osteoporosis o marrow hyperplasia 48. Regarding hyperparathyroidism (HPT): Which statement is CORRECT? (true or false) A. Brown tumours occur more frequently in secondary HPT B. Rugger Jersey spine occurs more frequently in primary HPT C. Chondrocalcinosis is seen in 15-20% D. Increased incidence of slipped upper femoral epiphysis is associated with HPT E. A normal bone scan in about 80% Source: Hussain, Shahid, et al. Radiology MCQs for the New FRCR Part 2A. 1st ed., TFM Publishing, 2006. Explanation: • A. False - primary • B. False - secondary • C. True - more frequent in secondary HPT • D. True • E. True 49. A 28-year-old tennis player undergoes a MR arthrogram to investigate recurrent right shoulder instability following a previous glenohumeral dislocation. The MRI reveals a tear of the anterosuperior labrum, closely related to the insertion of the biceps tendon. How are these appearances best described? 1. Anterior labral tear 2. Bankart lesion 3. Hill-Sachs lesion 4. Reverse Hill-Sachs lesion 5. Superior labrum from anterior to posterior (SLAP) lesion Source: McQueen, Andrew S., et al. Grainger & Allison’s Diagnostic Radiology: Single Best Answer MCQs. 5th ed., Churchill Livingstone, 2009.
  • 26.
    EBDR Exam MCQs& Concepts June 2025 Dr. Kareem Alnakeeb 20 50. An asymptomatic 65-year-old woman on long-term steroids for rheumatoid disease undergoes dual energy X-ray absorptiometry (DXA). Her Z score is —2 and her T score is —2.7. What is the WHO definition of osteoporosis? 1. T score less than —1 2. T score less than —2.5 3. Z score less than —1 4. Z score less than —2.5 5. Mean of T and Z score less than —2 Source: McQueen, Andrew S., et al. Grainger & Allison’s Diagnostic Radiology: Single Best Answer MCQs. 5th ed., Churchill Livingstone, 2009. Explanation • Bone density can be measured in relation to an age and sex-matched population (Z score) or in relation to a population of young adults of the same sex (T score). • The WHO defines osteoporosis as a T score less than —2.5, therefore relating bone mineral density to sex-matched peak bone mass. 51. Regarding MRI examination of the shoulder, what are the signal characteristics of the normal supraspinatus tendon? A. High signal intensity on all sequences B. High signal on T1w, low signal on T2w C. Intermediate signal on all sequences D. Low signal on all sequences E. Low signal on T1w, high signal on T2w Source: McQueen, Andrew S., et al. Grainger & Allison’s Diagnostic Radiology: Single Best Answer MCQs. 5th ed., Churchill Livingstone, 2009. 52. A 5-year-old boy presents with a 1-month history of pain in right leg. Radiography shows an ill-defined lucency in the proximal tibial metadiaphysis with periosteal reaction and a wide zone of transition. MRI shows a intramedullary lesion which returns intermediate signal on T1, high on STIR and has an extraosseous enhancing mass. What is the most likely diagnosis? A. Ewing’s sarcoma B. Osteomyelitis C. Enchondroma D. Fibrous dysplasia E. Giant cell tumour Source: Gupta, Chaitanya. 300 Single Best Answers for the Final FRCR Part A. 1st ed., Jaypee UK, 2010. Explanation: • More than 50% of cases are seen in long bones. • They are common in the metadiaphyseal region and most common in the 5- to 10-year age group. • Often presenting similarly to osteomyelitis. • MRI is extremely useful in determining the extent of the tumour, which is low on T1, high on T2 and STIR and enhances with contrast.
  • 27.
    EBDR Exam MCQs& Concepts June 2025 Dr. Kareem Alnakeeb 21 53. A young man undergoes an MRI of the right knee due to clinical suspicion of an acute rupture of the ACL. The ACL is indistinct, and cannot be visualised in either the coronal or sagittal plane. Which additional features would be supportive of a diagnosis of ACL rupture? 1. Bunching up of the PCL 2. Oedema within the medial collateral ligament 3. Posterior translation of the femur on the tibial condyles 4. Straightening of the patellar ligament 5. Tear of the medial meniscus Source: McQueen, Andrew S., et al. Grainger & Allison’s Diagnostic Radiology: Single Best Answer MCQs. 5th ed., Churchill Livingstone, 2009. 54. A series of neonatal radiographs reveal a narrow thorax with short ribs, square iliac wings with horizontal acetabular roofs, short sacrosciatic notches, progressive narrowing of the interpedicular distance and posterior scalloping of the vertebral bodies. What is the most likely diagnosis? F. Achondroplasia G. Campomelic dysplasia H. Cleidocranial dysplasia I. Ellis-van Creveld syndrome J. Morquio's syndrome Source: McQueen, Andrew S., et al. Grainger & Allison’s Diagnostic Radiology: Single Best Answer MCQs. 5th ed., Churchill Livingstone, 2009. Explanation: • The iliac wings in Morquio's syndrome are characteristically flaredTather than 55. MRCP examination for 47-year-old woman with obstructive jaundice shows a smooth stricture in the mid- common bile duct with associated moderate intrahepatic biliary dilatation. The stricture is caused by extrinsic compression from a round filling defect within the cystic duct. What is the diagnosis? A. Acute bacterial cholangitis B. Gallbladder carcinoma C. Mirizzi syndrome D. Primary sclerosing cholangitis (PSC) Source: McQueen, Andrew S., et al. Grainger & Allison’s Diagnostic Radiology: Single Best Answer MCQs. 5th ed., Churchill Livingstone, 2009. Explanation • In Mirizzi syndrome, a gallstone in the cystic duct produces mass effect on the common duct and can lead to fistula formation. 56. Which finding would indicate a nonresectable pancreatic tumor? A. The pancreatic duct dilated to 6 mm B. The presence of a 5-mm coeliac axis lymph node C. The tumor has invaded the duodenum D. The tumor in contact with 75% of the superior mesenteric artery Source: McQueen, Andrew S., et al. Grainger & Allison’s Diagnostic Radiology: Single Best Answer MCQs. 5th ed., Churchill Livingstone, 2009. Explanation: • If the tumor is in contact with more than half of the vessel circumference, it is very unlikely to be resectable.
  • 28.
    EBDR Exam MCQs& Concepts June 2025 Dr. Kareem Alnakeeb 22 57. Which of the following features would be MOST consistent with intestinal polyp? A. The lesion contains a locule of gas at its base. B. The lesion has a mean density of — 150 HU. C. The lesion is of homogeneous attenuation. D. There are diverticulae seen in the sigmoid colon. Source: McQueen, Andrew S., et al. Grainger & Allison’s Diagnostic Radiology: Single Best Answer MCQs. 5th ed., Churchill Livingstone, 2009. Explanation: • A polyp will usually demonstrate uniform soft tissue density, similar to the surrounding bowel wall. 58. On ultrasonographic examination, diffuse thickening of gall bladder with hyperechoic shadow at neck and comet tailing is seen in: A. Xanthogranulomatous cholecystitis B. Adenomyomatosis C. Adenomyomatous polyps D. Cholesterol crystals Explanation: (by Perplexity AI, generated with the OpenAI O3 model) • Adenomyomatosis is a benign hyperplastic condition of the gallbladder characterized by mural thickening and formation of Rokitansky–Aschoff sinuses. Ultrasonography typically reveals diffuse or segmental wall thickening, with comet tail or ring-down artefact resulting from cholesterol crystals trapped within the sinuses, most often seen at the gallbladder neck. • Xanthogranulomatous cholecystitis produces heterogeneous wall thickening but lacks comet tail artefacts. • Adenomyomatous polyps refer to focal instead of diffuse changes and do not exhibit classic comet tail artefact. • Cholesterol crystals can contribute to comet tailing, but they most commonly present as echogenic foci without diffuse wall thickening. 59. On scrotal ultrasound for testicular torsion, which of the following radiological findings would suggest that the testis is still viable? A. A diffusely enlarged hypoechoic left testis B. A normal echogenicity testis on grey-scale imaging C. A small shrunken left testis with a surrounding hydrocoele and scrotal wall thickening D. Absent blood flow within the left testis on color flow Doppler but good flow within the tunica vaginalis Source: McQueen, Andrew S., et al. Grainger & Allison’s Diagnostic Radiology: Single Best Answer MCQs. 5th ed., Churchill Livingstone, 2009. Explanation: • The section on testicular torsion within the comprehensive review by Fiitterer et al provides useful additional information. 60. Which of the following indicates T3 rather than T4 lung cancer? A. Invasion of the oesophagus B. Invasion of the trachea C. Invasion of the pericardium D. Malignant pleural effusion E. Invasion of the vertebral body Source: Leen, Edward. Succeeding in the New FRCR Part 2A Exam: Single Best Answer (SBA) Revision Questions for Modules 1-6. 1st ed., UNKNO, 2011.
  • 29.
    EBDR Exam MCQs& Concepts June 2025 Dr. Kareem Alnakeeb 23 Explanation: • T3 disease features include a tumor of any size less than 2cm from the carina, invasion of the parietal pleura, chest wall, diaphragm, mediastinal pleura, pericardium, pleural effusion or satellite nodule in the same lobe. • T4 disease is characterized by invasion of the heart, great vessels, trachea, oesophagus, vertebral body, carina or the presence of a malignant pleural effusion. • The TNM staging system was updated in 2009 (AJR, 2010). 61. A 78-year-old man has myelodysplastic syndrome and requires frequent blood transfusions. He develops progressively abnormal liver function tests and a grossly elevated ferritin level. An MRI of the liver is performed using breath hold half Fourier single shot spin echo T2w images. Which finding would make a diagnosis of hemosiderosis (iron overload from recurrent blood transfusion) more likely than haemochromatosis? A. Increased T2 signal in the liver only B. Increased T2 signal in the liver and spleen C. Reduced T2 signal in the liver only D. Reduced T2 signal in the liver and spleen E. Reduced T2 signal in the spleen only Source: McQueen, Andrew S., et al. Grainger & Allison’s Diagnostic Radiology: Single Best Answer MCQs. 5th ed., Churchill Livingstone, 2009. Explanation: • In iron overload due to recurrent transfusions, there is increased iron deposition in the reticuloendothelial system. This leads to reduced Tl, T2 and T2* signal intensity in the liver and spleen. • Haemochromatosis causes diffusely reduced T2 signal in the liver and may lead to cirrhosis, but the splenic signal intensity should remain normal. • Diffuse fatty liver will lead to increased T2 signal in the liver with signal loss during out-of-phase images. 62. An 18-year-old man experiences persistent symptoms following a fracture through the waist of the right scaphoid. Radiographs of the right scaphoid indicate non-union. An MRI is performed to assess the vascularity of the proximal pole. Which imaging features are consistent with a diagnosis of avascular necrosis? A. Bone marrow enhancement following administration of gadolinium B. High signal surrounding the fracture on T2w images C. High signal within the proximal pole on T1w images D. High signal within the proximal pole on STIR images E. Low signal within the proximal pole on T1w images Source: McQueen, Andrew S., et al. Grainger & Allison’s Diagnostic Radiology: Single Best Answer MCQs. 5th ed., Churchill Livingstone, 2009. Explanation: • Low signal on T1 reflects death of the adipocytes. The combination of low signal on T1w images and low or intermediate signal on T2w images accurately predicts avascular necrosis.
  • 30.
    EBDR Exam MCQs& Concepts June 2025 Dr. Kareem Alnakeeb 24 63. A low flat renogram curve indicates: A. Advanced nephropathy B. Complete obstruction to urine outflow C. Vesico-ureteric reflux D. Partial obstruction to urine outflow Explanation: (by Perplexity AI, generated with the OpenAI O3 model) • A very low, almost horizontal time–activity curve reflects markedly reduced tracer extraction by damaged renal parenchyma, typical of end-stage or advanced medical renal disease. • Complete or partial outflow obstruction instead produces an uptake phase followed by a rising or plateau phase; vesico-ureteric reflux alters the post-void segment rather than the primary renographic curve 64. A 67-year-old male patient presents with an 8-week history of left loin pain. A renal CT is obtained and this shows a 6-cm enhancing left renal lesion that has a fi brotic central scar. What is the most likely diagnosis? A. Renal leiomyoma. B. Renal oncocytoma. C. Renal metanephic adenoma. D. Renal haemangioma (giant). E. Renal juxta-glomerular cell neoplasm. Source: Lindsay, Richard, et al. SBAs for the FRCR Part 2A. 1st ed., Oxford University Press, 2012. Explanation: Renal oncocytoma • This is a benign renal cell neoplasm responsible for about 5% of all adult primary renal epithelial neoplasms. • It typically occurs in elderly men. • They usually appear as solitary, well-demarcated, unencapsulated, fairly homogeneous renal cortical tumours. • Bilateral, multicentric oncocytomas are seen in hereditary syndromes of renal oncocytosis and Birt– Hogg–Dubé syndrome. • A central stellate scar is seen in approximately one-third. • However, distinguishing them from RCC on imaging is not reliable. Renal leiomyoma • Leiomyoma of the kidney is a benign smooth muscle neoplasm. • It appears as a well-circumscribed, homogeneous, exophytic solid mass that shows uniform enhancement on contrast-enhanced CT. • It may occasionally be cystic. Metanephric adenoma • Metanephric adenoma is a benign renal neoplasm that is more common in middle-aged to elderly females. • It is associated with polycythaemia in 10%. • It typically appears as a well-defined, unencapsulated, solitary mass that may be hyperattenuating on unenhanced CT. • Calcification can be seen in up to 20%. Renal hemangioma
  • 31.
    EBDR Exam MCQs& Concepts June 2025 Dr. Kareem Alnakeeb 25 • Hemangioma of the kidney occurs as an unencapsulated, solitary lesion that frequently arises from the renal pyramids or the pelvis. • Contrast-enhanced CT or MRI may show early intense enhancement, with persistent enhancement on delayed images. Juxtaglomerular cell neoplasm (Reninoma) • Juxtaglomerular cell (JGC) neoplasm or reninoma is an extremely rare, benign renal neoplasm of myoendocrine cell origin, which is associated with a clinical triad of hypertension, hypokalaemia, and high plasma renin activity. • It typically appears as a unilateral, well-circumscribed, cortical tumour and often measures less than 3 cm, but otherwise is indistinguishable from other cortical neoplasms. 65. The radiograph of a 40-year-old man with a painful knee shows multiple calcified loose bodies, each of similar size, within the joint. The joint space is preserved. What diagnosis is most likely? A. Calcium pyrophosphate arthropathy B. Gout C. Pigmented villonodular synovitis D. Rheumatoid arthritis E. Synovial osteochondromatosis Source: McQueen, Andrew S., et al. Grainger & Allison’s Diagnostic Radiology: Single Best Answer MCQs. 5th ed., Churchill Livingstone, 2009. 66. Aqueduct stenosis is considered as: A. Communicating non-obstructive B. Non-communicating non-obstructive C. Non-communicating obstructive D. Communicating non-obstructive 67. A 35 year old man attends the accident and emergency department complaining of episodic lower back pain radiating down the legs. History and clinical examination also suggest pelvic sphincter dysfunction. MRI shows a spinal cord mass located at the conus medullaris. The mass is isointense on T1 and hyperintense on T2. It demonstrates contrast enhancement. The most likely diagnosis is: A. Astrocytoma B. Intradural lipoma C. Haemangioblastoma D. Myxopapillary ependymoma E. Ganglioglioma Source: Currie, Stuart, et al. SBAs for the FRCR 2A. 1st ed., Cambridge University Press, 2010. Explanation: • This is a variant of ependymoma and is the most common neoplasm of the conus medullaris. It originates from ependymal glia of the filum terminale. Average age at presentation is 35 years and it is more common in men. • T1-weighted imaging shows an isointense or occasionally hyperintense (due to the mucin content) mass. It is hyperintense on T2 and almost always shows enhancement postcontrast. • Intradural lipomas are hyperintense on T1-weighted imaging but they should not enhance. They also tend to occur in younger individuals and usually have an associated, clinically apparent lumbosacral mass. • Haemangioblastoma can also demonstrate high signal on T1 but is also highly vascular, can show signal voids and approximately half of them will have an intratumoural cystic component.
  • 32.
    EBDR Exam MCQs& Concepts June 2025 Dr. Kareem Alnakeeb 26 68. A 45-year-old man presents with dysphagia and undergoes a double-contrast barium swallow. This demonstrates a smooth oblique indentation on the posterior wall of the oesophagus. What is the most likely cause of these appearances? A. enlarged left atrium B. aberrant right subclavian artery C. aberrant left pulmonary artery D. right-sided aortic arch E. coarctation of the aorta Source: Bydder, Megan, et al. Get Through Final FRCR Part A: SBAs for the Modular Examination. 1st ed., Hodder Education Publishers, 2009. Explanation: Major Vessel Anomalies Causing Esophageal Impressions • A number of anomalies of the major vessels can cause extrinsic impressions upon the oesophagus. Right-Sided Aortic Arch • The commonest aortic anomaly is a right-sided aortic arch, which produces an indentation on the right lateral oesophageal wall in the absence of the normal left aortic arch impression. Aberrant Right Subclavian Artery • An aberrant right subclavian artery originates from the aortic arch just distal to the left subclavian artery, and passes upwards and to the right, behind the oesophagus, giving rise to an oblique posterior oesophageal indentation. Aortic Coarctation • In aortic coarctation, the preand post-stenotic dilatations of the aorta produce a characteristic reversed-3 impression upon the left wall of the oesophagus. Enlarged Left Atrium and Aberrant Left Pulmonary Artery • An enlarged left atrium and an aberrant left pulmonary artery both cause anterior indentations upon the oesophagus. 69. Which of the following is an extraconal extraorbital lesion, rather than an extraconal intraorbital lesion? A. Squamous cell carcinoma of the sinus B. Teratoma C. Dermoid cyst D. Capillary haemangioma E. Lymphangioma Source: Leen, Edward. Succeeding in the New FRCR Part 2A Exam: Single Best Answer (SBA) Revision Questions for Modules 1-6. 1st ed., UNKNO, 2011. Explanation: • Other causes are lymphoma, adenocarcinoma, adenoid cystic carcinoma, mucoceles and paranasal sinusitis.
  • 33.
    EBDR Exam MCQs& Concepts June 2025 Dr. Kareem Alnakeeb 27 70. A 15-year-old male presents with a history of recurrent epistaxis and nasal obstruction. MRI demonstrates a lesion centred at the sphenopalatine foramen, which is hypointense on T1WI and heterogeneously intermediate signal on T2WI. Intense lesional enhancement and multiple fl ow voids are noted on post-gadolinium T1WI. What is the diagnosis? A. Ludwig angina. B. Nasopharyngeal carcinoma. C. Inverted papilloma. D. Juvenile angiofibroma. E. Glomus jugulare. Source: Lindsay, Richard, et al. SBAs for the FRCR Part 2A. 1st ed., Oxford University Press, 2012. Explanation: Clinical Features • Juvenile angiofibromas are benign but locally aggressive tumors with high vascularity. • They typically occur in adolescent boys and present with recurrent epistaxis and nasal obstruction. Typical Location and Growth Pattern • They are centered within the sphenopalatine foramen and involve the pterygopalatine fossa, producing a bowed appearance of the posterior wall of maxillary sinus and widening of pterygopalatine fossa, inferior orbital, and pterygomaxillary fissures. • Osseous erosion is commonly seen. MRI Characteristics • The specific differentiating feature on MRI is the presence of multiple flow voids on T2WI and enhanced T1WI. 71. The mother of a three week old child notices a mass in her baby’s lower neck. The child is otherwise well. There is a history of normal pregnancy and the child was delivered by forceps. Ultrasound scan reveals homogeneous enlargement of the lower third of the right sternocleidomastoid muscle but no focal lesion is identified. T2-weighted MRI shows diffuse abnormal high signal intensity over the same area. The most likely diagnosis is: A. Hematoma B. Branchial cleft cyst C. Fibromatosis colli D. Neuroblastoma E. Cystic hygroma Source: Currie, Stuart, et al. SBAs for the FRCR 2A. 1st ed., Cambridge University Press, 2010. Explanation: • This is a rare form of infantile fibromatosis that occurs solely within the sternocleidomastoid muscle. o In the vast majority it is associated with birth trauma (e.g. forceps delivery). o This is thought to lead to compartment syndrome, pressure necrosis and secondary fibrosis of the muscle. o It usually locates to the lower third of the muscle, between the sternal and clavicular heads, and is usually unilateral. o Ultrasound may reveal a well- or ill-defined mass or may just show homogeneous muscle enlargement. o In approximately two-thirds of individuals, the abnormality spontaneously regresses by the age of two. • Expected ultrasonographic appearances of a hematoma include a heterogeneous mass of mixed cystic and solid components.
  • 34.
    EBDR Exam MCQs& Concepts June 2025 Dr. Kareem Alnakeeb 28 72. Expected ultrasonographic appearances of a hematoma include a heterogeneous mass of mixed cystic and solid components. A six year old boy is investigated for refractory complex partial seizures. CT demonstrates a well-defined, hypodense lesion located in the cortex of the temporal lobe. There is underlying bone remodeling but no calcification. On MRI the lesion demonstrates high signal on T2 and predominantly low signal on T1-weighted imaging. There is no surrounding oedema, minimal mass effect and no contrast enhancement. The most likely diagnosis is: A. Glioblastoma multiforme B. Dysembryoblastic neuroepithelial tumour (DNET) C. Primitive neuroectodermal tumour (PNET) D. Cavernous haemangiomas E. Ependymoma Source: Currie, Stuart, et al. SBAs for the FRCR 2A. 1st ed., Cambridge University Press, 2010. Explanation: DNETs • DNETs are benign tumours of neuroepithelial origin which arise from the cortical/deep grey matter. • They are preferentially located supratentorially (temporal 62%, frontal 31%). • CT demonstrates a hypoattenuating mass and there may be thinning and remodelling of the underlying inner table reflecting the slow growth of the tumour. • On MRI they are hypointense on T1, hyperintense on T2 and small intratumoural cysts may be present to cause a characteristic ‘bubbly’ appearance. • There is minimal mass effect and no associated vasogenic oedema. • A third of lesions show calcification and most tumors do not enhance. • If present, the enhancement is faint and patchy. Gangliogliomas and cavernous haemangiomas • Gangliogliomas and cavernous haemangiomas are other tumours which may cause epilepsy in children. • Cavernous haemangiomas are typically dense on CT and commonly calcify. PNETs • PNETs have a tendency for necrosis, cyst formation and calcification. • They also tend to be hyperdense on CT due to high nuclear to cytoplasmic ratio. 73. A 46-year-old woman from Bangladesh is being treated for pulmonary tuberculosis. Despite anti- tuberculosis chemotherapy, she develops increasing fevers with abdominal discomfort and distension. An abdominal and pelvic ultrasound demonstrates a moderate volume of peritoneal free fluid, and a contrast-enhanced CT of the abdomen and pelvis is performed. What are the likely findings on CT? A. A mixed solid: cystic ovarian mass with serosal deposits on the liver and spleen B. Ascites with enlarged mesenteric lymph nodes containing high attenuation C. Gastric wall thickening extending into the spleen with enlarged coeliac axis lymph nodes and ascites D. Peritoneal nodularity with high density ascites E. Portal vein thrombosis with ascites Source: McQueen, Andrew S., et al. Grainger & Allison’s Diagnostic Radiology: Single Best Answer MCQs. 5th ed., Churchill Livingstone, 2009. Explanation: • In peritoneal TB, the presence of dense ascites, peritoneal nodularity and lymph nodes with low attenuation centres are characteristic findings.
  • 35.
    EBDR Exam MCQs& Concepts June 2025 Dr. Kareem Alnakeeb 29 74. A three year old boy presents with seizures and headaches. CT head shows a hypoattenuating mass lying superior to the lateral ventricles, within the frontal region. It displays negative Hounsfield units and peripheral calcification but does not enhance. There is partial agenesis of the corpus callosum. MRI of the brain demonstrates a pericallosal tumor which is hyperintense on T1 and less hyperintense on T2- weighted imaging. What is the most likely diagnosis? A. Dermoid tumour B. Lipoma C. Teratoma D. Neurocytoma E. Ependymoma Source: Currie, Stuart, et al. SBAs for the FRCR 2A. 1st ed., Cambridge University Press, 2010. Explanation: Definition and Origin • This is a congenital tumor that results from abnormal differentiation of the meninx primitiva – that which eventually differentiates into pia, arachnoid and internal dura mater. Incidence and Congenital Associations • They account for less than 1% of brain tumors but are associated with congenital abnormalities, most commonly dysgenesis of the corpus callosum to some degree. • This is particularly likely when the lipoma is located anteriorly rather than posteriorly. Imaging Characteristics • On CT they are well-circumscribed masses with negative Hounsfield units and occasional calcification, and they do not enhance. • Characteristically, they are T1 hyperintense and slightly less hyperintense on T2. Differential Diagnosis • Dermoids and teratomas can show similar characteristics, with fat and calcium content. • Teratomas may enhance, although dermoids do not. • However, the lesion is much more likely to be a lipoma given its position (dermoids tend to be extra- axial (spinal canal); teratomas are much more commonly found around the pineal region, floor of the third ventricle, posterior fossa and spine) and given the association with corpus callosum abnormalities. 75. A neonatal boy has a renal ultrasound performed for the investigation of urinary obstructive symptoms. The ultrasound shows a distended urinary bladder with bilateral hydronephrosis. Which one of the following is the most likely underlying pathology? A. Posterior urethral valve B. Neurogenic bladder C. Horseshoe kidney D. Ectopic ureterocoeles E. Urethral diverticulum Source: Currie, Stuart, et al. SBAs for the FRCR 2A. 1st ed., Cambridge University Press, 2010. Explanation: • Posterior urethral valve is a congenital disorder characterized by a thick mucosal fold located in the posterior urethra. It is the most common cause of bilateral urinary tract obstruction in boys. • It is most commonly discovered in the neonatal period, but very occasionally may present into adulthood. • Diagnosis is usually made with ultrasound and surgical treatment is indicated.
  • 36.
    EBDR Exam MCQs& Concepts June 2025 Dr. Kareem Alnakeeb 30 76. An abdominal radiograph on a two-year-old child shows a paucity of bowel gas. An abdominal ultrasound is then performed which shows a 5 cm x 2 cm mass in the right upper quadrant. In the longitudinal plane, this has a `pseudokidney' appearance with a central echogenic focus and in the transverse plane it has the appearance of a `bull's-eye'. What is the most likely diagnosis? A. Ischaemic colitis B. Intussusception C. Volvulus D. Necrotising enterocolitis E. Appendicitis Source: Proctor, Robin. Final FRCR Part A Modules 4-6 Single Best Answer MCQs: The SRT Collection of 600 Questions with Explanatory Answers (MasterPass). 1st ed., CRC Press, 2009. Explanation: • Other findings on abdominal radiograph include an abdominal soft-tissue mass in the right upper quadrant, normal appearances and small bowel obstruction. 77. A 58 year old male with unexplained elevated alkaline phosphatase has an MRCP and the ‘double-duct’ sign is observed. Which one of the following diagnoses is most likely to cause this finding? A. Acute pancreatitis B. Annular pancreas C. Pancreas divisum D. Periampullary tumour E. Duodenal perforation Source: Currie, Stuart, et al. SBAs for the FRCR 2A. 1st ed., Cambridge University Press, 2010. Explanation: • The ‘double-duct’ sign is dilatation of the main pancreatic duct and the common bile duct as seen at ERCP and MRCP, and less commonly with CT and ultrasound. • It occurs due to an obstructing lesion at the ampulla, most commonly a carcinoma of the head of the pancreas (in up to 77% of cases) or a carcinoma of the ampulla of Vater (in up to 52% of cases). • The sign may be absent if there is an accessory pancreatic duct or when the main pancreatic duct drains into the minor papilla. 78. A 25-year-old with a history of cystic fibrosis presents with massive hemoptysis. Bronchial artery embolization is requested. Which of the following statements regarding bronchial artery embolization is false? A. A descending thoracic aortogram is performed prior to selective bronchial angiography. B. Bronchial angiography is performed with manual injection of contrast medium. C. The abnormal bronchial artery is embolized at its origin. D. Polyvinyl alcohol particles (diameter of 350–500 μm) may be used as the embolic material. E. Chest pain is the most common complication. Source: Lindsay, Richard, et al. SBAs for the FRCR Part 2A. 1st ed., Oxford University Press, 2012. Explanation: Overview • Bronchial artery embolization (BAE) is an established procedure in the management of massive haemoptysis. • Knowledge of the bronchial artery anatomy and its variations is essential in carrying out the procedure safely. Imaging and Angiography • A preliminary descending thoracic aortogram is performed to identify the number and site of origin of the bronchial arteries.
  • 37.
    EBDR Exam MCQs& Concepts June 2025 Dr. Kareem Alnakeeb 31 • Abnormal bronchial arteries are visualized on the preliminary thoracic aortogram in the majority of affected patients. • Selective bronchial angiography is performed with manual injection of contrast. Catheterization Safety • Selective bronchial artery catheterization and safe positioning distal to the origin of spinal cord branches is essential to avoid spinal cord ischemia/infarction. Embolic Agents • Polyvinyl alcohol particles (350–500 μm diameter) are the most frequently used embolic agent. • Smaller particles can freely flow via the intrapulmonary shunts, causing pulmonary or systemic infarcts. Complications • Chest pain is the most common complication (24–91%). • Other complications include dysphagia (due to embolization of esophageal branches), dissection of the bronchial artery or aorta (usually self-limited), and spinal cord ischemia. 79. Which imaging modality is most sensitive for detecting ductal carcinoma in situ (DCIS) of the breast? A. Mammography B. Magnetic Resonance Imaging (MRI) C. Ultrasonography D. Digital Breast Tomosynthesis Explanation: (by Perplexity AI, generated with the OpenAI O3 model) • Large prospective and retrospective series show MRI sensitivity for pure DCIS in the 80 – 92% range, consistently higher than mammography (≈55 – 70%) and far higher than ultrasound (≈35%). • MRI is especially advantageous for high-grade or non-calcified DCIS that may be occult on mammography. • Mammography remains first-line for population screening because it detects micro-calcifications cost-effectively, but sensitivity drops with dense breasts or non-calcified lesions. • Ultrasound is chiefly adjunctive; its stand-alone sensitivity for DCIS is low and operator-dependent. Why the other options are incorrect • Mammography: Gold standard for screening; however, micro-calcification-based detection misses a significant proportion of lesions, giving lower overall sensitivity than MRI. • Ultrasonography: Detects only about one-third of DCIS lesions and primarily masses; limited for pure micro-calcific or non-mass disease. 80. A 35-year-old female presents with a history of menorrhagia. MRI of pelvis demonstrates a fi broid uterus for which treatment with high-intensity focused ultrasound (HIFU) is proposed. What is the principle mechanism of action of HIFU? A. Coagulation necrosis. B. Apoptosis. C. Cavitation. D. Microstreaming. E. Radiation forces. Source: Lindsay, Richard, et al. SBAs for the FRCR Part 2A. 1st ed., Oxford University Press, 2012. Explanation: • HIFU is a non-invasive method to treat solid tumors or hemorrhage. As HIFU is essentially ultrasound, it requires an acoustic window to transmit ultrasound energy and is subject to similar artefact. • The principle effect of HIFU is heat generation from absorption of acoustic energy. This causes coagulation necrosis within seconds.
  • 38.
    EBDR Exam MCQs& Concepts June 2025 Dr. Kareem Alnakeeb 32 • Hyperthermia also induces apoptosis, which can be an important delayed effect in tissue exposed to lower energy HIFU. This mechanism is also a potential limitation of HIFU as adjacent tissue may be at risk. • Mechanical effects such as cavitation and microstreaming are also seen with the use of higher ultrasound intensity 81. A 9-year-oid boy injures his right wrist playing football. The radiograph reveals a fracture extending through the epiphysis and into the metaphysis. How would this injury be classified in the Salter-Harris classification? A. Type I B. Type II C. Туре III D. Type IV Source: McQueen, Andrew S., et al. Grainger & Allison’s Diagnostic Radiology: Single Best Answer MCQs. 5th ed., Churchill Livingstone, 2009. 82. A 59-year-old female patient presented with malaise, chest pain and dyspnoea. Her chest radiograph was normal. An echocardiogram demonstrated a mobile echogenic mass attached to the intra-atrial septum by a stalk. What is the most likely diagnosis? A. Pulmonary embolism B. Papillary fibroelastoma C. Sarcoma D. Fibrovillous adenoma E. Cardiac myxoma Source: Proctor, Robin. Final FRCR Part A Modules 1–3 Single Best Answer MCQs: The SRT Collection of 600 Questions with Explanatory Answers (MasterPass). 1st ed., CRC Press, 2009. Explanation: • The differential diagnosis of a pedunculated intracardiac lesion includes atrial myxoma and papillary fibroelastoma. • Papillary fibroelastomas are rare lesions that are typically asymptomatic. 83. Down syndrome is most commonly associated with: A. ASD (septum primum type) B. ASD (septum secundum type) C. VSD D. Patent foramen ovale Explanation: (by Perplexity AI, generated with the OpenAI O3 model) • Down syndrome is strongly linked to endocardial-cushion maldevelopment, so the characteristic cardiac lesion is an ostium primum atrial septal defect (the septum primum type of ASD), often classified within the spectrum of complete or partial atrioventricular septal defects. 84. Percutaneous sclerotherapy is used to treat all of the following except: A. Arteriovenous malformation (AVM) B. Capillary telangiectasia C. Lymphatic malformation D. Venous malformation Explanation: (by Perplexity AI, generated with the OpenAI O3 model) • Percutaneous sclerotherapy is routinely used to treat lymphatic malformations and venous malformations. • It is not a primary therapy for capillary telangiectasia, and it is only occasionally employed for limited slow-flow components of some AVMs.
  • 39.
    EBDR Exam MCQs& Concepts June 2025 Dr. Kareem Alnakeeb 33 85. Ultrasound examination of the face and neck is performed to investigate a buccal, soft-tissue mass that became noticeable during pregnancy. The lesion is heterogeneous and hypoechoic, and has sinusoidal spaces demonstrating slow flow and circular calcifications. Which of the following is the most likely diagnosis? A. benign lymph node B. malignant lymph node C. pleomorphic parotid adenoma D. arteriovenous malformation E. venous vascular malformation Source: Bydder, Megan, et al. Get Through Final FRCR Part A: SBAs for the Modular Examination. 1st ed., Hodder Education Publishers, 2009. Explanation: Vascular Malformations • Phleboliths if present are unique to vascular malformations. • Arterial malformations are high flow, while venous, capillary or combined malformations are low flow. • MRI is required to assess the full extent, particularly intraosseous and intracranial, of head and neck vascular malformations. Lymph Nodes • Benign lymph nodes are smooth, elliptical and hypoechoic with hilar architecture and vascularity. • Malignant lymph nodes are typically round, are hypoechoic, have no hilum and show peripheral vascularity. Malignant lymph nodes with necrosis are seen with squamous cell and papillary cell carcinoma of the thyroid. • Internal punctate calcification is seen in metastases from papillary or medullary carcinoma of the thyroid. 86. A severely hypoplastic cerebellar vermis in an enlarged bony posterior fossa, with associated hydrocephalus and communication of the fourth ventricle with a posterior midline CSF cyst, are features of which of the following posterior fossa malformations? A. mega cisterna magna B. Dandy–Walker malformation C. Dandy–Walker variant D. Arnold–Chiari malformation E. Joubert’s syndrome Source: Bydder, Megan, et al. Get Through Final FRCR Part A: SBAs for the Modular Examination. 1st ed., Hodder Education Publishers, 2009. Explanation: Dandy–Walker Malformation • Classically, the Dandy–Walker malformation consists of partial or total absence of the cerebellar vermis, dilatation of the fourth ventricle into a large cystic mass, an enlarged posterior fossa, hydrocephalus (in 75% of cases) and torcular–lambdoid inversion (elevation of the torcular Herophili above the lambdoid suture). • The proposed etiology is obstruction of CSF outflow at the foramina of Magendie and Luschka. • The vermis abnormality is the key component in all forms of the Dandy–Walker complex. • The variant is less severe with a better prognosis. Chiari Malformations • Chiari malformations have the fundamental abnormality of an underdeveloped, small posterior fossa, in contrast to the Dandy–Walker complex where it is normal or enlarged.
  • 40.
    EBDR Exam MCQs& Concepts June 2025 Dr. Kareem Alnakeeb 34 87. Thyroid radiofrequency ablation (RFA) is least commonly used in which clinical setting? A. Hypothyroidism B. Hyperthyroidism (toxic/autonomous nodule) C. Malignancy Explanation: (by Perplexity AI, generated with the OpenAI O3 model) • RFA has no established role in treating primary hypothyroidism. It is designed to shrink nodules, not augment deficient hormone production • In autonomously functioning (toxic) nodules, RFA can normalize thyroid function or reduce antithyroid-drug requirements, and is endorsed as a second-line option when surgery or radio-iodine are unsuitable • RFA is increasingly used for selected thyroid malignancies—especially small papillary micro- carcinomas in non-surgical candidates and for loco-regional recurrence after thyroidectomy. Although still “off-label” in many regions, use is growing and supported by guidelines for carefully selected cases. 88. Which of the following indicates telangiectatic osteosarcoma (TOS) rather than an aneurismal bone cyst (ABC)? A. Enhancing septa without nodularity on MR B. Marked expansile remodelling of bone C. Cortical thinning D. Presence of osteoid matrix with septal regions on CT E. Presence of haemorrhagic spaces Source: Leen, Edward. Succeeding in the New FRCR Part 2A Exam: Single Best Answer (SBA) Revision Questions for Modules 1-6. 1st ed., UNKNO, 2011. Explanation: • Thick peripheral septa with nodularity, presence of an osteoid matrix within nodular or septal regions, and aggressive growth features such as cortical destruction indicate TOS rather than ABC. 89. The mother of a three week old child notices a mass in her baby’s lower neck. The child is otherwise well. There is a history of normal pregnancy and the child was delivered by forceps. Ultrasound scan reveals homogeneous enlargement of the lower third of the right sternocleidomastoid muscle but no focal lesion is identified. T2-weighted MRI shows diffuse abnormal high signal intensity over the same area. The most likely diagnosis is: A. Hematoma B. Branchial cleft cyst C. Fibromatosis colli D. Neuroblastoma E. Cystic hygroma Source: Currie, Stuart, et al. SBAs for the FRCR 2A. 1st ed., Cambridge University Press, 2010. Explanation: Definition and Etiology • Infantile Sternocleidomastoid Fibromatosis is a rare form of infantile fibromatosis that occurs solely within the sternocleidomastoid muscle. • In the vast majority it is associated with birth trauma (e.g. forceps delivery). • This is thought to lead to compartment syndrome, pressure necrosis and secondary fibrosis of the muscle. Location and Natural History • It usually locates to the lower third of the muscle, between the sternal and clavicular heads, and is usually unilateral.
  • 41.
    EBDR Exam MCQs& Concepts June 2025 Dr. Kareem Alnakeeb 35 • In approximately two-thirds of individuals, the abnormality spontaneously regresses by the age of two. Imaging Findings • Ultrasound may reveal a well- or ill-defined mass or may just show homogeneous muscle enlargement. • Expected ultrasonographic appearances of a hematoma include a heterogeneous mass of mixed cystic and solid components. 90. A 70 year old man undergoes surgery for AAA. Two weeks following surgery, he is readmitted to the A&E department with abdominal pain and fever. Palpation of the abdomen suggests a pulsatile mass. A CT angiogram is performed, which does not demonstrate contrast extravasation. Which of the features on CT angiogram would be most worrisome? A. Presence of a pseudoaneurysm B. Periaortic soft tissue C. Thickening of a fluid-filled third part of the duodenum D. Some ectopic gas in the vicinity E. Loss of fat plane between the grafted aorta and the adjacent duodenum Source: Currie, Stuart, et al. SBAs for the FRCR 2A. 1st ed., Cambridge University Press, 2010. Explanation: • Two weeks post-procedure, all the other features including the presence of ectopic gas may be postoperative. • However, presence of a thickened fluid-filled bowel loop would be extremely worrying for an aorto- enteric fistula. • Presence of ectopic gas beyond four weeks is much more likely to be abnormal. 91. A 25-year-old with a history of cystic fibrosis presents with massive hemoptysis. Bronchial artery embolization is requested. Which of the following statements regarding bronchial artery embolization is false? A. A descending thoracic aortogram is performed prior to selective bronchial angiography. B. Bronchial angiography is performed with manual injection of contrast medium. C. The abnormal bronchial artery is embolized at its origin. D. Polyvinyl alcohol particles (diameter of 350–500 μm) may be used as the embolic material. E. Chest pain is the most common complication. Source: Lindsay, Richard, et al. SBAs for the FRCR Part 2A. 1st ed., Oxford University Press, 2012. Explanation: Overview • Bronchial artery embolization (BAE) is an established procedure in the management of massive haemoptysis. • Knowledge of the bronchial artery anatomy and its variations is essential in carrying out the procedure safely. Imaging Evaluation • A preliminary descending thoracic aortogram is performed to identify the number and site of origin of the bronchial arteries. • Abnormal bronchial arteries are visualized on the preliminary thoracic aortogram in the majority of affected patients. Angiography Technique • Selective bronchial angiography is performed with manual injection of contrast.
  • 42.
    EBDR Exam MCQs& Concepts June 2025 Dr. Kareem Alnakeeb 36 • Selective bronchial artery catheterization and safe positioning distal to the origin of spinal cord branches is essential to avoid spinal cord ischemia/infarction. Embolic Agents • Polyvinyl alcohol particles (350–500 μm diameter) are the most frequently used embolic agent. • Smaller particles can freely flow via the intrapulmonary shunts, causing pulmonary or systemic infarcts. Complications • Chest pain is the most common complication (24–91%). • Other complications include dysphagia (due to embolization of esophageal branches), dissection of the bronchial artery or aorta (usually self-limited), and spinal cord ischemia. 92. A 20-year-old man presents with gradual onset of neck pain and a painful lump in the upper neck posteriorly. Plain films show an apparent destructive lesion of the C2 vertebra. MRI shows a large lesion arising from the posterior elements of C2 and comprising multiple cysts with fluid–fluid levels, with preservation of the vertebral body. What is the most likely diagnosis? A. aneurysmal bone cyst B. giant cell tumor C. chordoma D. fibrous dysplasia E. telangiectatic osteosarcoma Source: Bydder, Megan, et al. Get Through Final FRCR Part A: SBAs for the Modular Examination. 1st ed., Hodder Education Publishers, 2009. Explanation: Aneurysmal Bone Cysts • Aneurysmal bone cysts are seen mainly in patients under 20 years of age (75%) and affect the posterior elements when involving the spine. • They may arise de novo, or secondary to another lesion such as a giant cell tumor (GCT) or fibrous dysplasia. Giant Cell Tumors (GCTs) and MRI Findings • Both GCTs and telangiectatic osteosarcomas may cause cysts with fluid–fluid levels on MRI, but GCTs arise from vertebral bodies and usually occur in the sacrum. Telangiectatic Osteosarcomas • Telangiectatic osteosarcomas usually affect long bones. Chordomas • Chordomas are malignant tumors that usually affect the vertebral body, with destruction and invasion of the discs and adjacent structures. 93. Which CT finding would be more suggestive of chronic pancreatitis than ductal pancreatic adenocarcinoma? A. Common bile duct dilatation B. Focal enlargement of the pancreatic head C. Intraductal pancreatic calcification D. Peripancreatic fat stranding and ascites Source: McQueen, Andrew S., et al. Grainger & Allison’s Diagnostic Radiology: Single Best Answer MCQs. 5th ed., Churchill Livingstone, 2009.
  • 43.
    EBDR Exam MCQs& Concepts June 2025 Dr. Kareem Alnakeeb 37 94. In gastric carcinoma, which of the following typically makes tumor margins hardest to delineate on CT/MRI? A. Pancreatic infiltration B. Perigastric lymph-node enlargement C. Ascites D. Liver metastases Explanation: (by Perplexity AI, generated with the OpenAI O3 model) • Direct invasion into the pancreas usually produces a contiguous soft-tissue interface that actually helps identify loss of the fat plane, aiding recognition of T4 disease rather than obscuring the primary lesion • Nodal masses are generally separable from the gastric wall; they may crowd the region but seldom merge imperceptibly with the tumor, so margins remain discernible. • Free peritoneal fluid “bathes” the gastric serosa, effaces surrounding fat planes and equalizes soft- tissue contrast, making it difficult to see the outer border or subtle serosal perforation, especially on portal-venous-phase CT. • Hepatic lesions are remote from the primary; they do not interfere with visualizing the gastric wall itself. 95. Licked Candy Stick sign is seen in which condition? A. Leprosy B. Sarcoidosis C. Hypertension D. Madura foot (mycetoma) Explanation: (by Perplexity AI, generated with the OpenAI O3 model) • Resorption of distal phalanges in leprosy can produce tapered, smooth bone ends likened to a “licked candy stick”. • Osseous sarcoid more often shows lytic “lace-like” lesions rather than uniform tapering of bone tips. • Systemic hypertension has no characteristic skeletal radiographic sign. • Mycetoma causes multiple punched-out cavities or “dot-in-circle” sign on MRI, not the tapering seen in the licked candy-stick appearance. 96. Which radiological feature is NOT seen in childhood scurvy? A. Pelkan spur B. Frankel’s line C. Growth-arrest line D. Zone of demarcation (Trümmerfeld lucent band) Explanation: (by Perplexity AI, generated with the OpenAI O3 model) • Metaphyseal spurs (“Pelkan spurs”) caused by healing micro-fractures are classic for scurvy. • A dense provisional zone of calcification (“white line of Frankel”) is a hallmark sign. • Transverse growth-arrest (Harris) lines reflect temporary slowing of endochondral growth seen after severe systemic illness or therapy but are not characteristic of vitamin C deficiency; scurvy instead shows excessive calcification at the provisional zone, not a halted metaphyseal front. • The lucent band beneath Frankel’s line—the Trümmerfeld or “scorbutic zone of rarefaction/ demarcation”—is typical of scurvy
  • 44.
    EBDR Exam MCQs& Concepts June 2025 Dr. Kareem Alnakeeb 38 97. Which of the following statements about idiopathic scoliosis is true? A. Infantile scoliosis curves are usually to the right B. Juvenile scoliosis curves are usually to the left C. Idiopathic scoliosis is more common in boys D. Idiopathic scoliosis is more common in girls Explanation: (by Perplexity AI, generated with the OpenAI O3 model) • Idiopathic curves presenting before age 3 (infantile) classically have a left-sided thoracic curve; a right thoracic curve in this age group should raise suspicion of an underlying pathology. • In children aged 4-10 years (juvenile idiopathic scoliosis) the predominant pattern is a right thoracic curve, mirroring adolescent disease. • Although very mild curves have similar incidence, progression-prone idiopathic scoliosis is overall twice as frequent in girls, with the female:male ratio rising steeply with increasing Cobb angle. • Female predominance is well documented across early-onset and adolescent groups; girls not only present more often but have a much higher risk of curve progression that requires treatment 98. A chest radiograph of a 3-year-old child demonstrates marked right lower zone consolidation with a large pneumatocele. A diagnosis of necrotizing pneumonia is made. What is the most likely causative organism? A. Staphylococcus aureus B. Streptococcus pyogenes C. Bordatella pertussis D. Mycobacterium tuberculosis E. Aspergillus Source: Chin, Teck Yew, et al. Get Through Final FRCR 2A: SBAs. 1st ed., CRC Press, 2017. Explanation: Definition • Pneumatoceles are thin-walled, air-filled intraparenchymal cysts that develop secondary to localized bronchiolar and alveolar necrosis, which allow one-way passage of air into the interstitial space. Common Associations • They commonly occur in immunocompetent patients and are most commonly associated with S. aureus, followed by Staphylococcus pneumoniae infections. Mechanical Ventilation and Complications • Although there is no clear correlation between the development of pneumatoceles and mechanical ventilation, patients receiving mechanical ventilation have an increased risk for developing complications related to pneumatoceles, including an increase in their size. Genetic Factors • Other than in hyperimmunoglobulin E syndrome, there is no known genetic or familial tendency for pneumatoceles. Prognosis • The majority of pneumatoceles (more than 85%) resolve spontaneously, partially or completely over weeks to months without clinical or radiographic sequelae.
  • 45.
    EBDR Exam MCQs& Concepts June 2025 Dr. Kareem Alnakeeb 39 99. A 65-year-old man with history of stroke presents with chest pain. The chest radiograph shows a thin curvilinear area of calcification in the lower part of left heart border. What is the likely site of calcification? A. Left atrium B. Left ventricle C. Right atrium D. Left descending coronary artery E. Mitral valve Source: Gupta, Chaitanya. 300 Single Best Answers for the Final FRCR Part A. 1st ed., Jaypee UK, 2010. Explanation: • This is the typical site for left ventricular calcifications. • Valvular calcifications are located within the heart. • Coronary artery calcifications are seen along the upper part of left heart border and have a ‘tram- track’ appearance. 100. All of the following conditions are recognized causes of bilateral sacroiliitis, except: A. Whipple’s disease B. Ankylosing spondylitis C. Septic arthritis D. Crohn’s disease Explanation: (by Perplexity AI, generated with the OpenAI O3 model) • Arthropathy in Whipple’s disease is typically migratory, intermittent and peripheral; axial involvement is rare and, when present, more often unilateral rather than bilateral. • Ankylosing spondylitis classically starts with bilateral, symmetrical sacroiliitis, which is the radiographic hallmark of the disease. • Although most infectious sacroiliitis presents unilaterally, about one-third of cases can affect both sacroiliac joints, especially in hematogenous Staphylococcus or Brucella infections. • Spondyloarthropathy associated with inflammatory bowel disease frequently manifests as bilateral sacroiliitis, often independent of bowel activity 101. Fibrosis that is predominantly basal/lower-lobe on HRCT is associated with all of the following systemic diseases except: A. Rheumatoid arthritis (RA) B. Systemic lupus erythematosus (SLE) C. Systemic sclerosis (scleroderma) D. Ankylosing spondylitis Explanation: (by Perplexity AI, generated with the OpenAI O3 model) • RA-associated interstitial lung disease typically shows lower-lobe UIP/NSIP patterns, with subpleural reticulation and honeycombing. • SLE interstitial lung disease is uncommon and, when present, more often shows patchy or upper- lobe–dominant NSIP/organising pneumonia; basal fibrotic change is not a characteristic distribution. • Scleroderma is a classic cause of basal-predominant fibrosis (NSIP/UIP) with traction bronchiectasis and volume loss. • Pulmonary involvement in ankylosing spondylitis classically produces apical fibrobullous disease; however, any fibrosis that does develop outside the apices is still reported to spare the bases, so lower-lobe fibrosis is not typical—but the key “exception” asked is clearly SLE.
  • 46.
    EBDR Exam MCQs& Concepts June 2025 Dr. Kareem Alnakeeb 40 102. Which primary lung cancer subtype most frequently presents with cavitation on imaging? A. Squamous cell carcinoma B. Small-cell lung tumor C. Adenocarcinoma D. Large-cell lung tumor Explanation: (by Perplexity AI, generated with the OpenAI O3 model) • Cavitation is seen in up to 20–30% of squamous cell lung cancers, far exceeding other histologies. Thick-walled, irregular cavities are typical. • Small-cell carcinoma virtually never cavitates; lesions are usually solid and centrally located. • Cavitation occurs in only ~2–5% of adenocarcinomas and is therefore much less common than in squamous tumors. • Cavitation is reported in ~6% of large-cell carcinomas—higher than adenocarcinoma but still far below the squamous subtype 103. A 4-month-old boy presents with cyanosis. Examination reveals right ventricular heave and systolic murmur. A chest radiograph shows a bulging right heart border and widening of the superior mediastinum, creating a ‘snowman’ appearance. What is the most likely diagnosis? A. Fallot’s tetralogy B. total anomalous pulmonary venous return C. partial anomalous pulmonary venous return D. transposition of the great vessels E. coarctation of the aorta Source: Bydder, Megan, et al. Get Through Final FRCR Part A: SBAs for the Modular Examination. 1st ed., Hodder Education Publishers, 2009. Explanation: Total anomalous pulmonary venous return General presentation • Total anomalous pulmonary venous return presents in the first year of life with cyanosis. • It is due to failure of the pulmonary veins to drain into the left atrium, with drainage instead into another vascular structure. Supracardiac type I features • There are four types – the commonest, type I (supracardiac), has the four pulmonary veins draining into one common vein, the vertical vein, which drains into the left brachiocephalic vein. • This dilated vein, along with the dilated left brachiocephalic vein and superior vena cava, causes widening of the superior mediastinum, which is the ‘head’ of the ‘snowman’. • The body is formed by enlargement of the right atrium, producing a rounded appearance of the lower mediastinum. • There may be increased pulmonary vascularity. Partial anomalous pulmonary venous return • Partial anomalous pulmonary venous return presents later in life with symptoms similar to atrial septal defect. Transposition of the great vessels • Transposition of the great vessels presents in the first 2 weeks of life with cyanosis. • Chest radiograph shows an ‘egg-on-string’ appearance of the mediastinum with increased pulmonary vascularity. Infantile-type coarctation • Infantile-type coarctation presents with symptoms and signs of congestive heart failure in infancy. • The classic figure-3 sign seen in coarctation is often hidden by the thymus in infants.
  • 47.
    EBDR Exam MCQs& Concepts June 2025 Dr. Kareem Alnakeeb 41 104. A 73-year-old woman with previous history of myocardial infarction was referred for a chest radiograph by her GP to exclude chest infection. No infective focus was identified but a focal bulge was noticed in the left heart border, with curvilinear calcification along the edge. What is the diagnosis? A. Myocardial calcification B. Right atrial calcification C. Mitral annulus calcification D. Calcified vegetations E. Left ventricular aneurysm calcification Source: Chin, Teck Yew, et al. Get Through Final FRCR 2A: SBAs. 1st ed., CRC Press, 2017. Explanation: Cardiac False Aneurysm • A cardiac false aneurysm is defined as a rupture of the myocardium that is contained by pericardial adhesion. • It usually represents a rare complication of myocardial infarction, but it may also occur after cardiac surgery, chest trauma and endocarditis. True vs False Left Ventricular Aneurysm • True left ventricular aneurysms are discrete, dyskinetic areas of the left ventricular wall with a broad neck. • Unlike a true aneurysm, which contains some myocardial elements in its wall, the walls of a false aneurysm are composed of organized hematoma and pericardium only. • Both demonstrate focal bulge to the cardiac contour and can calcify. Diagnostic Imaging • Marked delayed enhancement of the pericardium on dynamic enhanced MRI may help in differentiating a false aneurysm from a true one. 105. Adamantinoma is a rare low-grade malignant bone tumour; in which bone does it most commonly arise? A. Femur B. Fibula C. Mandible D. Radius E. Tibia Explanation: (by Perplexity AI, generated with the OpenAI O3 model) • Around 80–90% of adamantinomas originate in the mid-diaphysis of the tibia, classically presenting as an eccentric lytic lesion with cortical thinning. • The fibula can be involved but usually secondarily or in multifocal disease, therefore less common than the tibia. • Femoral and radial occurrences are rare isolated case reports only. • The mandible is the favoured site for odontogenic ameloblastoma; true long-bone adamantinoma is histologically and genetically distinct, so mandibular involvement is not typical. 106. A 55-year-old woman develops right upper-quadrant pain and abnormal liver function tests six weeks after an uncomplicated laparoscopic cholecystectomy. What is the most appropriate initial imaging modality to investigate a suspected post-cholecystectomy bile duct problem? A. Abdominal ultrasound B. Contrast-enhanced CT abdomen C. Endoscopic retrograde cholangiopancreatography (ERCP) D. Magnetic resonance cholangiopancreatography (MRCP) E. Hepatobiliary (HIDA) scintigraphy
  • 48.
    EBDR Exam MCQs& Concepts June 2025 Dr. Kareem Alnakeeb 42 Explanation: (by Perplexity AI, generated with the OpenAI O3 model) • Ultrasound is quick, widely available, radiation-free and inexpensive, making it the usual first-line test after cholecystectomy. It can detect biliary dilatation, retained stones, intra-abdominal collections and vascular complications, guiding whether more advanced studies are needed. • MRCP (D) provides superior ductal detail but is typically reserved when ultrasound or CT raises concern or is non-diagnostic. • ERCP (C) is invasive and kept for therapeutic intervention. • Contrast CT (B) offers a broad survey but involves radiation and intravenous contrast; it is often second line. • HIDA scintigraphy (E) is highly sensitive for active bile leaks but takes longer and is not the routine starting test. 107. According to the Stanford classification of aortic dissection, which single anatomical feature distinguishes a Type A dissection from a Type B dissection? A. Primary intimal tear proximal to the left subclavian artery B. Dissection confined to the descending thoracic aorta C. Dissection involves the ascending aorta D. Dissection extends below the renal arteries E. Presence of partial thrombosis within the false lumen Explanation: (by Perplexity AI, generated with the OpenAI O3 model) • Type A in the Stanford scheme is defined by any involvement of the ascending aorta, irrespective of where the entry tear lies or how far the dissection extends distally; surgical repair is usually required. • Type B dissections spare the ascending aorta and are often managed medically or endovascularly. • Option A is incorrect because the tear may originate distally yet propagate retrograde into the ascending aorta, still making it Type A. • Option B describes a pure descending aortic dissection—Type B. • Option D concerns distal extent, which does not affect Stanford grouping. • Option E relates to luminal status, not classification. 108. An MRI of the thoracic spine in a 32-year-old man with chronic back pain shows reduced intervertebral disc height, irregular destruction of the opposing vertebral endplates and a smooth, lobulated paravertebral soft-tissue mass with minimal surrounding oedema. What is the most likely diagnosis? A. Degenerative disc disease B. Metastatic vertebral collapse C. Pyogenic spondylodiscitis D. Tuberculous spondylitis E. Osteoporotic compression fracture Explanation: (by Perplexity AI, generated with the OpenAI O3 model) • Tuberculous infection typically involves two contiguous vertebral bodies with intervening disc narrowing, irregular endplate erosion and a “cold” paravertebral abscess that produces little inflammatory reaction; these combined features are highly characteristic, making option D correct. • Degenerative change (A) causes end-plate sclerosis, osteophytes and preserved marrow signal, not soft-tissue abscess. • Metastases (B) may collapse a vertebra but rarely cross the disc or form smooth abscesses. • Acute pyogenic spondylodiscitis (C) also erodes endplates and narrows the disc but usually shows marked marrow oedema, enhancing inflammatory pannus and systemic sepsis. • Osteoporotic fracture (E) gives wedge or biconcave collapse without endplate erosion or paravertebral collection.
  • 49.
    EBDR Exam MCQs& Concepts June 2025 Dr. Kareem Alnakeeb 43 109. A non-contrast CT KUB shows a branching staghorn calculus filling the renal pelvis and calyces in a patient with chronic Proteus urinary tract infection. Which stone composition is most likely? A. Calcium oxalate monohydrate B. Uric acid C. Magnesium ammonium phosphate (struvite) D. Cystine E. Xanthine Explanation: (by Perplexity AI, generated with the OpenAI O3 model) • Infection with urease-producing organisms such as Proteus raises urinary pH and splits urea into ammonium, promoting precipitation of magnesium, ammonium and phosphate ions. The resulting magnesium ammonium phosphate (struvite) stones grow rapidly, often forming staghorn calculi that appear radiodense on CT. • Calcium oxalate stones (A) are the commonest overall but form in sterile, acidic urine. • Uric acid stones (B) are radiolucent and occur in persistently acid urine, not alkaline infection. • Cystine calculi (D) arise from a hereditary transport defect and have a characteristic hexagonal crystal appearance. • Xanthine stones (E) are rare and related to xanthine oxidase deficiency or allopurinol therapy; they are not infection-related. 110. Enchondroma vs. Chondrosarcoma: classic location for Enchondroma is: A. Hand and feet (small bones) B. Pelvis C. Ribs D. Scapula Explanation: (by Perplexity AI, generated with the OpenAI O3 model) • Enchondroma is most common in the small bones of the hands and feet; • chondrosarcoma is rare in these locations and more common in the pelvis and long bones. 111. A 45-year-old carpenter presents with a gradually enlarging, mildly tender swelling of the proximal phalanx of the index finger. Hand radiographs show an expansile intramedullary lytic lesion with endosteal scalloping that involves more than two-thirds of the cortical thickness but no cortical breach or soft-tissue mass. Which diagnosis is most likely? A. Bone infarct B. Enchondroma C. Low-grade (grade 1) chondrosarcoma D. Osteoblastoma E. Giant cell tumour Explanation: (by Perplexity AI, generated with the OpenAI O3 model) • Extensive (>⅔) endosteal scalloping in a cartilaginous lesion of the small bones strongly suggests low-grade chondrosarcoma rather than a benign enchondroma; benign lesions usually cause little or no scalloping and lack aggressive remodeling. • Bone infarct shows serpiginous sclerosis without scalloping. • Osteoblastoma produces a blastic nidus, not chondroid lucency. • Giant cell tumour is eccentric, abuts the articular surface, and lacks chondroid matrix.
  • 50.
    EBDR Exam MCQs& Concepts June 2025 Dr. Kareem Alnakeeb 44 112. A 55-year-old man undergoes staging CT for an exophytic gastric body mass suspected to be either adenocarcinoma or a gastrointestinal stromal tumour (GIST). Which CT feature most reliably favours gastric adenocarcinoma over GIST? A. Large size (>5 cm) of the primary lesion B. Central areas of low attenuation within the mass C. Marked enhancement after intravenous contrast D. Presence of perigastric lymph-node enlargement E. Exophytic growth pattern with a claw-shaped gastric wall connection Explanation: (by Perplexity AI, generated with the OpenAI O3 model) • Gastric adenocarcinoma spreads early via lymphatics, so enlarged regional lymph nodes are common and help distinguish it from GIST, which rarely involves lymph nodes (<5%); routine lymphadenectomy is therefore unnecessary in GIST. • Central low attenuation (necrosis) and strong enhancement can occur in both tumours, while size and exophytic growth pattern overlap and are not discriminatory. 113. A 28-year-old primigravida undergoes her routine 20-week anomaly scan that shows mild unilateral pyelectasis with an anteroposterior renal pelvic diameter of 5 mm. There are no other structural abnormalities. What is the most appropriate next management step? A. Immediate referral for fetal MRI B. Amniocentesis for karyotyping C. Repeat ultrasonography in 6 weeks D. Elective delivery at 37 weeks E. No further follow-up required Explanation: (by Perplexity AI, generated with the OpenAI O3 model) • Isolated mild pyelectasis (4–7 mm at 18–20 weeks) is usually a transient finding related to delayed ureteric maturation; the majority resolve spontaneously. • Current obstetric imaging guidelines recommend a single follow-up ultrasound 4–6 weeks later to assess progression or resolution. • Immediate MRI offers no added value, and invasive karyotyping is reserved for cases with additional soft markers or risk factors. • Elective pre-term delivery is unnecessary unless severe, progressive dilatation leads to complications, and discharging without surveillance risks missing significant hydronephrosis that may develop. 114. On routine 28-week fetal ultrasound, a male fetus shows bilateral renal pelvis dilatation measuring 11 mm in the anteroposterior plane. Which anteroposterior renal-pelvic diameter (APRPD) threshold at or after 28 weeks’ gestation usually prompts postnatal urological assessment and possible intervention? A. ≥5 mm B. ≥7 mm C. ≥10 mm D. ≥15 mm E. ≥20 mm Explanation: (by Perplexity AI, generated with the OpenAI O3 model) • At 28 weeks and beyond, an APRPD of 10 mm or greater is classified as severe pyelectasis/hydronephrosis. • This degree of dilatation carries a high risk of underlying pelvi-ureteric junction obstruction or vesico- ureteric reflux and therefore triggers structured postnatal follow-up, antibiotic prophylaxis and, in many centres, neonatal imaging with a view to possible surgery.
  • 51.
    EBDR Exam MCQs& Concepts June 2025 Dr. Kareem Alnakeeb 45 • Smaller measurements (5 mm or 7 mm) are considered mild to moderate and often resolve spontaneously; they are usually managed with serial antenatal scans rather than immediate postnatal intervention. • Diameters of 15 mm or 20 mm are uncommon and indicate marked obstruction, but the action line in standard UK guidelines remains 10 mm. 115. An unenhanced CT abdomen shows acute right renal vein thrombosis. Which of the following sonographic findings is most characteristic of this condition? A. Small echogenic right kidney with cortical thinning B. Large hypoechoic right kidney with loss of corticomedullary differentiation C. Normal-sized kidney with multiple simple cortical cysts D. Large echogenic kidney with prominent renal sinus fat E. Small hypoechoic kidney with increased cortical vascularity Explanation: (by Perplexity AI, generated with the OpenAI O3 model) • Acute renal vein thrombosis causes venous outflow obstruction, leading to renal congestion and interstitial oedema. • On ultrasound this manifests as an enlarged, globally hypoechoic kidney in which the normal corticomedullary differentiation is effaced, sometimes accompanied by reduced or absent venous flow on Doppler—option B. • Chronic thrombotic kidneys are typically small and echogenic (option A). • Simple cortical cysts (option C) and prominent sinus fat (option D) are unrelated incidental findings. • Increased cortical vascularity (option E) would be expected in acute pyelonephritis, not venous thrombosis. 116. Which branch of the mandibular division forms a communicating loop with the facial nerve within the parotid region, effectively linking cranial nerves V and VII? A. Inferior alveolar nerve B. Chorda tympani C. Auriculotemporal nerve D. Buccal nerve E. Lingual nerve Explanation: (by Perplexity AI, generated with the OpenAI O3 model) • The auriculotemporal nerve (V3) ascends behind the mandibular neck, then crosses the parotid gland where it gives sensory fibres to the gland and skin and sends multiple fine branches that anastomose with the facial nerve, creating a direct connection between CN V and CN VII. It also carries post-ganglionic secretomotor fibres from the otic ganglion to the parotid. • Chorda tympani (B) is a branch of the facial nerve that joins, not links, with the lingual nerve to convey taste and parasympathetic fibres; it is itself CN VII, not a V3 branch. • Inferior alveolar (A), buccal (D) and lingual (E) nerves are V3 branches without significant facial-nerve communications.
  • 52.
    EBDR Exam MCQs& Concepts June 2025 Dr. Kareem Alnakeeb 46 117. A 7-year-old boy’s jaw radiograph shows loss of alveolar bone giving several incisors a ‘floating tooth’ appearance, and skull radiographs demonstrate multiple lytic calvarial lesions with bevelled edges; which single diagnosis best explains these findings? A. Eosinophilic granuloma B. Osteomyelitis C. Acute leukaemic infiltration D. Papillon-Lefèvre syndrome E. Metastatic neuroblastoma Explanation: (by Perplexity AI, generated with the OpenAI O3 model) • Eosinophilic granuloma, the osseous form of Langerhans cell histiocytosis, classically produces sharply marginated lytic bone defects with asymmetric cortical involvement, giving the calvarial “bevelled edge” sign and selective alveolar destruction that makes teeth appear to float. • Osteomyelitis may erode cortex but usually shows periosteal reaction and lacks discrete bevelled margins. • Leukaemic infiltration causes diffuse, poorly defined medullary lucencies rather than focal punched- out skull lesions. • Papillon-Lefèvre syndrome creates generalized periodontitis without discrete calvarial lesions. • Metastatic neuroblastoma can give skull “hair-on-end” appearance or sutural widening, but floating teeth are uncommon and bevelled calvarial edges are not typical. 118. Rigler’s sign indicates: A. Bowel ischemia B. Free air on both sides of bowel wall (pneumoperitoneum) C. Gallstone ileus D. Pneumatosis intestinalis 119. On a supine abdominal radiograph the bowel wall is outlined on both its luminal and peritoneal surfaces by gas (double-wall appearance). What approximate minimum volume of intraperitoneal free air is usually required for this Rigler sign to be visible? A. 50 mL B. 100 mL C. 200 mL D. 500 mL E. 1 000 mL Explanation: (by Perplexity AI, generated with the OpenAI O3 model) • Rigler sign appears only when a large pneumoperitoneum (>1 000 mL) surrounds gas-filled loops so that gas lies on both sides of the bowel wall, accentuating it. • Smaller volumes (50–500 mL) are often sufficient to create sub-diaphragmatic free air on an erect chest film but do not envelope the bowel circumferentially, so the double-wall sign is absent. • Therefore options A–D underestimate the amount of free air needed for Rigler sign.
  • 53.
    EBDR Exam MCQs& Concepts June 2025 Dr. Kareem Alnakeeb 47 120. Which prenatal ultrasound finding is NOT typically associated with trisomy 21 (Down syndrome) in the fetus? A. Nuchal translucency thickening B. Echogenic intracardiac focus C. Duodenal atresia (“double-bubble” sign) D. Choroid plexus cyst E. Single umbilical artery Explanation: (by Perplexity AI, generated with the OpenAI O3 model) • Trisomy 21 commonly shows soft-marker findings such as increased nuchal translucency, echogenic intracardiac focus, and duodenal atresia; a single umbilical artery can also be seen because aneuploidies, including Down syndrome, are linked to umbilical-cord vessel anomalies. • Choroid plexus cysts, however, are typically isolated or linked to trisomy 18, not trisomy 21; when they appear without other abnormalities they do not raise the risk of Down syndrome, making option D the exception. 121. Salpingitis isthmica nodosa is most commonly associated with which underlying pelvic condition that causes tubal inflammation? A. Endometriosis B. Polycystic ovarian syndrome C. Pelvic inflammatory disease D. Uterine fibroids E. Cervical carcinoma Explanation: (by Perplexity AI, generated with the OpenAI O3 model) • Salpingitis isthmica nodosa (SIN) represents nodular outpouchings of tubal epithelium into the myosalpinx and is widely regarded as an acquired sequel of chronic tubal infection. • Large hysterosalpingography and surgical series show that up to 90% of tubes demonstrating SIN also have histologic or clinical evidence of pelvic inflammatory disease, supporting a strong post- inflammatory aetiology. • Chronic salpingitis therefore predisposes to the development of SIN, which in turn increases the risks of infertility and ectopic pregnancy. • Endometriosis and uterine fibroids may distort the pelvis but do not produce the characteristic diverticular scarring of the isthmic tube. • Polycystic ovarian syndrome and cervical carcinoma have no direct link to SIN. 122. A 68-year-old man with long-standing hypertension presents with sudden severe chest pain; acute aortic intramural haematoma is suspected. Which imaging test is the modality of choice for initial assessment? A. Transthoracic echocardiography B. Transoesophageal echocardiography C. Magnetic resonance angiography D. Chest radiography E. CT angiography Explanation: (by Perplexity AI, generated with the OpenAI O3 model) • CT angiography provides rapid, whole-aorta coverage with high spatial resolution, allowing confident detection of intramural haematoma, dissection flaps and associated complications within minutes. Multidetector scanners achieve sensitivities and specificities approaching 100%, and the study is available 24/7 in most emergency departments. • Transoesophageal echocardiography (B) is sensitive but semi-invasive and less comprehensive for distal aorta. • Magnetic resonance angiography (C) lacks speed and availability in unstable patients.
  • 54.
    EBDR Exam MCQs& Concepts June 2025 Dr. Kareem Alnakeeb 48 • Transthoracic echocardiography (A) has limited acoustic windows for the thoracic aorta, and chest radiography (D) is neither sensitive nor specific. 123. In children with β-thalassaemia major, premature fusion of the proximal humeral and other long-bone epiphyses is most often a consequence of which pathological process? A. Chronic iron chelation with desferrioxamine B. Extramedullary haemopoietic marrow expansion within the bone C. Endocrine iron overload causing hypothyroidism D. Secondary hyperparathyroidism from chelation-induced zinc loss E. Vitamin D deficiency osteomalacia Explanation: (by Perplexity AI, generated with the OpenAI O3 model) • Hyperactive, ineffective erythropoiesis in β-thalassaemia major markedly enlarges the marrow cavity. The expanding marrow breaches cortex, extends sub-periosteally and weakens the metaphysis. • Normal axial loading then compresses the medial physis, producing a Salter-Harris V-type injury that triggers early physeal fusion and characteristic humeral varus deformity. • Desferrioxamine can cause metaphyseal dysplasia but does not close the physis. • Endocrine iron overload, secondary hyperparathyroidism and vitamin D deficiency produce osteopenia or delayed rather than premature fusion, so options C–E are incorrect. 124. In the radionuclide technetium-99m used for routine gamma-camera imaging, what does the suffix “m” signify? A. Monoenergetic gamma emission B. Metastable nuclear isomer C. Marker that the isotope is generator-produced D. Mean photon energy of 140 keV E. Mass number rounded to the nearest integer Explanation: (by Perplexity AI, generated with the OpenAI O3 model) • The “m” in 99mTc denotes a metastable nuclear isomer, meaning the nucleus is in a long-lived excited state that later decays to its ground state by emitting 140 keV gamma rays. This metastability (half-life ≈ 6 h) allows sufficient time for imaging while keeping patient dose low. • Option A is incorrect: many isotopes emit monoenergetic photons without the “m”. • Option C confuses production method with nuclear state; generator production relates to parent 99Mo, not the “m”. • Option D cites a characteristic gamma energy but does not define the suffix. • Option E concerns atomic mass number; the mass is already given by “99”, independent of the “m”. 125. A teenager presents with painless bony swellings of the fingers, and radiographs show multiple expansile intramedullary lucencies in the phalanges; soft-tissue films demonstrate numerous phlebolith- containing haemangiomas. What is the most likely diagnosis? A. Ollier disease B. Hereditary multiple exostoses C. Maffucci syndrome D. McCune–Albright syndrome E. Klippel–Trenaunay syndrome
  • 55.
    EBDR Exam MCQs& Concepts June 2025 Dr. Kareem Alnakeeb 49 Explanation: (by Perplexity AI, generated with the OpenAI O3 model) • Maffucci syndrome is defined by the combination of multiple enchondromas and soft-tissue haemangiomas, often with phleboliths; this vascular component distinguishes it from Ollier disease, which has enchondromas alone. • Hereditary multiple exostoses produces osteochondromas, not enchondromas. • McCune–Albright shows polyostotic fibrous dysplasia plus endocrine features and café-au-lait macules, not haemangiomas. • Klippel–Trenaunay causes limb over-growth with venous and lymphatic malformations but lacks enchondromas; bone lesions, when present, are cortical hypertrophy rather than medullary cartilage tumours. 126. An abdominal radiograph taken for sudden epigastric pain shows a single, markedly dilated loop of proximal jejunum in the left upper quadrant—the classic “sentinel loop” appearance. Which underlying condition is this sign most suggestive of? A. Acute pancreatitis B. Proximal small-bowel obstruction C. Acute cholecystitis D. Perforated peptic ulcer E. Left ureteric colic Explanation: (by Perplexity AI, generated with the OpenAI O3 model) • The sentinel loop represents a localised adynamic ileus adjacent to an inflamed viscus. When it lies in the left upper quadrant, it overlies the pancreas and therefore points to acute pancreatitis. • In small-bowel obstruction (B) multiple dilated loops with air–fluid levels are typical rather than a solitary segment. • Acute cholecystitis (C) can produce a sentinel loop, but this would be in the right hypochondrium near the gall-bladder, not the LUQ. • Perforated peptic ulcer (D) usually shows pneumoperitoneum under the diaphragm. • Ureteric colic (E) gives no specific bowel gas patterns. 127. According to the modified Hinchey classification of acute diverticulitis, what stage corresponds to generalised faecal peritonitis? A. Stage Ia B. Stage Ib C. Stage II D. Stage III E. Stage IV Explanation: (by Perplexity AI, generated with the OpenAI O3 model) • Stage IV represents perforated diverticulitis with faecal contamination of the peritoneal cavity, necessitating urgent laparotomy and usually resection with stoma formation. • Stage III is limited to purulent (but not faecal) peritonitis, while Stage II is a distant intra-abdominal or pelvic abscess that can often be managed with percutaneous drainage. • Stage Ib is a confined pericolic/mesocolic abscess, and Stage Ia is limited pericolic inflammation without abscess. • Recognising the correct stage guides the urgency and type of surgical versus radiological intervention.
  • 56.
    EBDR Exam MCQs& Concepts June 2025 Dr. Kareem Alnakeeb 50 128. MRI brain in a 6-year-old boy with worsening headaches shows marked dilatation of the lateral and third ventricles, a thin web across the aqueduct of Sylvius and a normal-calibre fourth ventricle. What is the most likely diagnosis? A. Communicating hydrocephalus after meningitis B. Dandy–Walker malformation C. Aqueductal stenosis causing non-communicating obstructive hydrocephalus D. Choroid plexus papilloma E. Normal pressure hydrocephalus Explanation: (by Perplexity AI, generated with the OpenAI O3 model) • Triventricular enlargement with an intraluminal membrane at the aqueduct is classic for aqueductal stenosis, the commonest form of non-communicating (obstructive) hydrocephalus in childhood. CSF is blocked between the third and fourth ventricles, so the fourth remains normal. • Communicating hydrocephalus (A) enlarges all four ventricles because the obstruction is distal to the ventricular system. • Dandy–Walker malformation (B) features a cystic dilatation of the fourth ventricle and enlarged posterior fossa. • A choroid plexus papilloma (D) would present as an enhancing intraventricular mass, often with global ventricular enlargement. • Normal pressure hydrocephalus (E) affects elderly patients, shows proportionate ventricular expansion and an unobstructed aqueduct. 129. On sagittal MRI of the cranio-cervical junction, which imaging finding best distinguishes an Arnold-Chiari type 1.5 malformation from a classic Chiari I malformation? A. Inferior displacement of the cerebellar tonsils >5 mm below the foramen magnum B. Presence of syringomyelia within the cervical spinal cord C. Caudal descent of the medulla/obex through the foramen magnum in addition to tonsillar herniation D. Small posterior fossa with crowding of CSF spaces E. Basilar invagination with platybasia Explanation: (by Perplexity AI, generated with the OpenAI O3 model) • Chiari 1.5 is considered an “advanced” form of Chiari I in which the brain-stem (typically the medulla or obex) is pulled caudally below the foramen magnum along with the cerebellar tonsils; this brain- stem descent (option C) is the key imaging discriminator. • Tonsillar descent alone (option A) defines Chiari I and therefore is not specific. • Syringomyelia (option B) can accompany either Chiari I or 1.5. A small posterior fossa (option D) is common to several hind-brain crowding disorders but is not distinctive. • Basilar invagination (option E) may coexist yet is neither necessary nor sufficient for the Chiari 1.5 diagnosis. 130. In cardiac CT reporting, what minimum percentage luminal narrowing typically defines a “significant” coronary artery stenosis requiring further clinical correlation? A. 30% B. 50% C. 70% D. 80% E. 90%
  • 57.
    EBDR Exam MCQs& Concepts June 2025 Dr. Kareem Alnakeeb 51 Explanation: (by Perplexity AI, generated with the OpenAI O3 model) • A diameter reduction of about 70% is generally accepted as the threshold at which a coronary stenosis becomes haemodynamically significant and is therefore termed “significant” on CT or invasive angiography. • Below this, flow-limiting effects are less predictable: a 50% lesion may still be clinically silent, while >70% frequently produces ischaemia. • Higher cut-offs such as 80% or 90% describe critical or near-occlusive disease, not merely “significant.” • A 30% narrowing is considered mild and unlikely to impair perfusion. 131. A contrast-enhanced CT abdomen shows diffuse sheet-like soft-tissue infiltration of the greater omentum producing an “omental cake”. Which primary malignancy most commonly gives this appearance? A. Gastric adenocarcinoma B. Colon adenocarcinoma C. Ovarian carcinoma D. Pancreatic adenocarcinoma E. Non-Hodgkin lymphoma Explanation: (by Perplexity AI, generated with the OpenAI O3 model) • Ovarian carcinoma spreads early by trans-coelomic seeding; tumour cells gravitate to the dependent greater omentum, so diffuse omental thickening (“omental cake”) is classically and most frequently linked to advanced ovarian cancer. • Gastric and colonic adenocarcinomas can also seed the peritoneum but do so less often than ovarian tumours. • Pancreatic cancer typically causes peritoneal deposits later in the disease and far less frequently involves the omentum diffusely. • Lymphoma may infiltrate the omentum but such lymphomatous caking is rare because the omentum lacks significant lymphoid tissue. 132. Which subtype of colonic polyp has the highest likelihood of progressing to invasive colorectal carcinoma? A. Hyperplastic polyp B. Tubular adenoma C. Tubulovillous adenoma D. Villous adenoma E. Inflammatory pseudopolyp Explanation: (by Perplexity AI, generated with the OpenAI O3 model) • Villous adenomas, defined by ≥75% villous architecture, have the greatest malignant potential among colonic polyps. Reported transformation rates are 15–25% overall and approach 40% in lesions >4 cm, reflecting high rates of dysplasia and a broad sessile growth pattern that facilitates invasion. • Tubular adenomas are far more common but carry only a 1–5% risk. Tubulovillous adenomas sit between these extremes. • Hyperplastic and inflammatory polyps are non-neoplastic and rarely, if ever, become cancerous. • Hence, villous adenoma is the subtype most prone to malignant change while key distractors pose lower or negligible risk.
  • 58.
    EBDR Exam MCQs& Concepts June 2025 Dr. Kareem Alnakeeb 52 133. A 7-year-old girl undergoes abdominal ultrasound for recurrent urinary infections. A single, lobulated renal mass is seen in the midline pelvis; CT confirms complete fusion of both kidneys without intervening septum, with two non-crossing ureters draining separately into the bladder. What is the most likely congenital anomaly? A. Crossed fused renal ectopia B. Horseshoe kidney C. Pancake (shield) kidney D. Solitary pelvic kidney E. Duplex collecting system Explanation: (by Perplexity AI, generated with the OpenAI O3 model) • Pancake kidney is an extreme fusion anomaly in which the upper and lower poles of both kidneys fuse completely, forming a flattened mid-pelvic mass. Each half retains its own pelvicalyceal system and typically short, uncrossed ureters, fitting the CT description. • Crossed fused ectopia (A) has one ureter crossing the midline to the orthotopic bladder orifice. • Horseshoe kidney (B) shows fusion only at the lower poles with an isthmus and lies across the lumbar spine, not entirely in the pelvis. • A solitary pelvic kidney (D) is unfused and single. • Duplex systems (E) involve duplicated pelvis/ureter but do not produce a single fused renal mass. 134. A 9-month-old boy presents with intermittent crying episodes, drawing up of the legs, and passage of “red-currant jelly” stool. Abdominal ultrasound shows a transverse bowel segment with an outer hypoechoic rim and central echogenic core, producing a“pseudokidney” (target) sign. What is the most likely diagnosis? A. Midgut volvulus B. Meckel diverticulum C. Intussusception D. Hypertrophic pyloric stenosis E. Hirschsprung disease Explanation: (by Perplexity AI, generated with the OpenAI O3 model) • The “pseudokidney” or target sign on ultrasound represents concentric bowel-within-bowel with mesenteric fat and vessels, classic for ileocolic intussusception in infants. • Midgut volvulus shows a whirlpool sign of twisted mesentery, not a target lesion. • A Meckel diverticulum may cause bleeding or obstruction but lacks the characteristic ultrasound appearance. • Hypertrophic pyloric stenosis produces an elongated, thickened pylorus (“doughnut” sign) in the epigastrium, not in the right abdomen. • Hirschsprung disease is a colonic motility disorder diagnosed by contrast enema and rectal biopsy, without a pseudokidney appearance on ultrasound. 135. Which of the following is NOT a recognised cause of pulsatile tinnitus in clinical practice? A. Idiopathic intracranial hypertension B. Glomus jugulare tumour C. High-riding jugular bulb D. Dural arteriovenous fistula E. Vestibular schwannoma
  • 59.
    EBDR Exam MCQs& Concepts June 2025 Dr. Kareem Alnakeeb 53 Explanation: (by Perplexity AI, generated with the OpenAI O3 model) • Pulsatile tinnitus is almost always generated by conditions that alter or amplify vascular flow near the temporal bone. • Raised intracranial pressure in idiopathic intracranial hypertension produces turbulent venous flow; glomus jugulare tumours and dural arteriovenous fistulas are highly vascular lesions that transmit pulse-synchronous noise; and a high-riding jugular bulb abuts the middle ear, transmitting venous flow sounds. • In contrast, vestibular schwannoma is a benign nerve-sheath tumour that typically causes non- pulsatile tinnitus by compressing the cochleovestibular nerve; it does not create haemodynamically driven noise, so pulse synchrony is absent. 136. During a single-contrast barium meal, a lenticular pool of contrast with a convex inner margin (the Carman meniscus sign) is seen within a large ulcer on the gastric body. What is the most likely diagnosis? A. Benign peptic (non-malignant) gastric ulcer B. Malignant gastric ulcer C. Duodenal ulcer disease D. Gastric diverticulum E. Hiatal hernia Explanation: (by Perplexity AI, generated with the OpenAI O3 model) • The Carman meniscus sign represents barium trapped within an ulcer whose rigid, infiltrative margins are produced by an underlying carcinoma; the convex inner edge forms a semilunar “meniscus” that is highly specific for a malignant gastric ulcer. • Benign ulcers usually have smooth folds reaching the crater and may show Hampton’s line, not a meniscus. • Duodenal ulcers (C) occur distal to the pylorus and do not display this sign. • A gastric diverticulum (D) is an out-pouching without ulcer mound or heaped margins. • Hiatal hernia (E) alters gastric position but does not create focal ulcerative masses or the characteristic meniscus configuration. 137. Which coronary artery predominantly supplies the lateral wall of the left ventricle? A. Left anterior descending artery B. Right coronary artery C. Circumflex artery D. Posterior descending artery E. Conus branch Explanation: (by Perplexity AI, generated with the OpenAI O3 model) • The lateral (obtuse) margin of the left ventricle receives its main blood supply from obtuse marginal branches that arise from the circumflex artery, a branch of the left main stem. • The left anterior descending artery mainly perfuses the anterior septum and anterior wall, not the true lateral wall. • The right coronary artery and its posterior descending branch chiefly supply the inferior wall and posterior septum in right-dominant circulation. • The conus branch is a small vessel from the right coronary artery that supplies the right ventricular outflow tract, not the left ventricular free wall.
  • 60.
    EBDR Exam MCQs& Concepts June 2025 Dr. Kareem Alnakeeb 54 • Therefore, only the circumflex artery consistently provides the principal arterial supply to the left ventricular lateral wall. 138. Freiberg’s disease is characterized by avascular necrosis involving which part of the foot? A. Calcaneal apophysis B. Navicular bone C. Second metatarsal head D. Talus E. Fifth metatarsal base Explanation: (by Perplexity AI, generated with the OpenAI O3 model) • Freiberg’s disease (Freiberg infraction) is osteonecrosis of the second metatarsal head, typically presenting in adolescent females with forefoot pain and stiffness. • Vascular compromise leads to subchondral collapse and fragmentation of the metatarsal head, visible on radiographs and MRI. • Option A describes Sever disease, an overuse apophysitis of the calcaneus, not true AVN. • Option B is Köhler disease, AVN of the navicular. • Option D refers to talar AVN, most often post-fracture. • Option E (Jones fracture site) involves stress or acute fracture of the fifth metatarsal base, not avascular necrosis. 139. Which imaging modality is most sensitive for detecting active lower gastrointestinal bleeding with flow rates as low as 0.05–0.1 mL/min? A. 99mTc-labelled red-blood-cell scintigraphy B. Digital subtraction angiography C. Contrast-enhanced multidetector CT angiography D. Colonoscopy E. 18F-FDG PET/CT Explanation: (by Perplexity AI, generated with the OpenAI O3 model) • 99mTc-labelled RBC scintigraphy can demonstrate active bleeding at very low rates (≈0.05–0.1 mL/min) because the radiolabelled cells pool at the haemorrhage site, allowing prolonged acquisition and high sensitivity. • Angiography requires faster bleeding (≈0.5–1 mL/min) to visualise contrast extravasation, while CT angiography detects rates around 0.3–0.5 mL/min; both are therefore less sensitive than scintigraphy. • Colonoscopy often misses intermittent or slow bleeds and relies on direct visualisation rather than flow rate. • 18F-FDG PET/CT is not a recognised technique for acute GI haemorrhage localisation. 140. A 27-year-old man with haemoptysis and biopsy-proven anti-GBM (Goodpasture’s) disease undergoes high-resolution CT of the chest; which imaging pattern is most typical of his pulmonary involvement? A. Peripheral honeycombing of the lower lobes B. Diffuse bilateral ground-glass opacification with relative subpleural sparing C. Multiple centrilobular nodules with tree-in-bud spread D. Thick-walled cavitary lesion in the upper lobe E. Patchy mosaic attenuation with expiratory air-trapping
  • 61.
    EBDR Exam MCQs& Concepts June 2025 Dr. Kareem Alnakeeb 55 Explanation: (by Perplexity AI, generated with the OpenAI O3 model) • Goodpasture’s syndrome commonly causes diffuse alveolar haemorrhage. • Fresh intra-alveolar blood increases parenchymal density, producing widespread ground-glass opacities or air-space consolidation, usually bilateral and often sparing the extreme lung apices and costophrenic angles. • Honeycombing (A) reflects end-stage fibrosis, not haemorrhage. • Centrilobular tree-in-bud nodules (C) suggest endobronchial infection. • A cavitary focus (D) is characteristic of necrotising infection or tuberculosis. • Mosaic attenuation (E) points to small-airway or vascular obstruction rather than hemorrhage. 141. Which imaging feature most strongly suggests a retroperitoneal lymphatic malformation on CT? A. Well-defined multilocular cystic mass that displaces adjacent organs B. Homogeneous low-attenuation fluid collection that invades the psoas muscle C. Multilocular cystic lesion with fluid–fluid levels that insinuates between great vessels without invasion D. Solid enhancing mass arising from para-aortic soft tissues E. Unilocular cyst with peripheral coarse calcification Explanation: (by Perplexity AI, generated with the OpenAI O3 model) • Retroperitoneal lymphatic malformations are benign multilocular cystic lesions that typically contain simple or chylous fluid. They characteristically insinuate between retroperitoneal structures rather than invading them, and may show fluid–fluid levels when complicated by hemorrhage or high- protein chyle, helping differentiate them from other cystic or solid retroperitoneal masses. Option C captures these key features. • Option A lacks the classic insinuating growth pattern; most lymphatic malformations produce little true mass effect. • Option B describes invasive behavior, atypical for a lymphatic malformation. • Option D depicts a solid enhancing tumor, more in keeping with lymphoma, sarcoma or paraganglioma. • Option E (unilocular calcified cyst) better fits hydatid, epidermoid or matured hematoma rather than a multilocular lymphatic lesion. 142. In a multiphasic renal CT, a solid renal cortical mass measures 35 HU on the non-contrast scan, 120 HU in the corticomedullary phase, 100 HU in the nephrographic phase and 80 HU on 5-minute excretory images. Which renal tumour subtype is most likely? A. Papillary renal cell carcinoma B. Chromophobe renal cell carcinoma C. Clear cell renal cell carcinoma D. Mucinous tubular & spindle cell carcinoma E. Fat-poor angiomyolipoma Explanation: (by Perplexity AI, generated with the OpenAI O3 model) • Clear cell RCC is typically hypervascular, showing brisk enhancement that peaks in the corticomedullary phase (often >110 HU) and gradually washes out on later phases, exactly as in this vignette. • Papillary RCC (A) is hypovascular, seldom exceeding 70 HU even at peak and may be hypo- or iso- attenuating to cortex throughout.
  • 62.
    EBDR Exam MCQs& Concepts June 2025 Dr. Kareem Alnakeeb 56 • Chromophobe RCC (B) and mucinous tubular & spindle cell carcinoma (D) enhance less avidly than clear cell and show lower peak HU values (~70 HU and ~55 HU respectively). • Fat-poor angiomyolipoma (E) can mimic clear cell but usually displays lower early enhancement (80– 90 HU) and more rapid wash-out; presence of tumoral fat or homogeneous enhancement pattern further aids differentiation. Subtype Non-contrast HU Corticomedullary (Arterial) phase Nephrographic (Venous) phase Key Features Clear cell RCC 20–40 HU† Strong, rapid enhancement (70– 120+ HU), heterogeneous Loss of enhancement, but still hypervascular Most vascular; rapid, intense enhancement Papillary RCC 20–40 HU Mild or minimal, homogeneous (50–60 HU); remains hypoattenuating Remains low, mild delayed wash-in Typically hypovascular; most homogeneous Angiomyolipoma Often fat-density (-10 to -20 HU); may vary May enhance but usually identifiable by fat Variable Macroscopic fat is diagnostic † Values are approximate; literature shows attenuation varies, but ccRCC most often has low pre-contrast, then strong post-contrast enhancement. Pearl for FRCR 2A • Clear cell RCC: Greatest, rapid, heterogeneous arterial enhancement (“fastest & brightest”) • Papillary RCC: Mild, homogeneous enhancement, always < cortex and clear cell • Angiomyolipoma: Macroscopic fat (very low HU) is diagnostic 143. On an unenhanced CT head following blunt trauma, which intracranial hematoma classically appears biconvex and is limited by cranial suture lines? A. Acute subdural hematoma B. Intracerebral hematoma C. Extradural (epidural) hematoma D. Subarachnoid haemorrhage E. Intraventricular haemorrhage Explanation: (by Perplexity AI, generated with the OpenAI O3 model) • An extradural (epidural) haematoma accumulates between the inner table of the skull and the periosteal dura. Because the dura is firmly attached at the sutures, the collection cannot extend beyond these lines, giving the characteristic lentiform (biconvex) shape on CT. • Subdural haematomas lie beneath the dura, so they freely cross sutures but are limited by dural reflections such as the falx; they therefore appear crescentic. • Intracerebral, subarachnoid and intraventricular haemorrhages occur within brain parenchyma, cisterns or ventricles respectively and are not constrained by sutures, so their distribution differs from an extradural haematoma.
  • 63.
    EBDR Exam MCQs& Concepts June 2025 Dr. Kareem Alnakeeb 57 144. A 29-year-old motorcyclist sustains a high-speed head injury. Non-contrast CT brain performed on arrival shows a focal haemorrhage within the splenium of the corpus callosum with no significant cortical contusions or skull fracture. Which primary mechanism best explains this haemorrhage distribution? A. Direct coup–contrecoup contusion B. Rupture of lenticulostriate perforators from acute hypertension C. Shearing forces producing diffuse axonal injury D. Venous bleeding secondary to basilar skull fracture E. Microvascular fragility from cerebral amyloid angiopathy Explanation: (by Perplexity AI, generated with the OpenAI O3 model) • The corpus callosum is tethered between the falx and ventricular walls, so rapid deceleration or rotational acceleration generates high shear strains across interhemispheric white matter. These shearing forces tear small medullary vessels and axons, producing characteristic callosal haemorrhage that often accompanies diffuse axonal injury. • Direct cortical contusions (A) typically involve the frontal or temporal poles rather than deep midline structures. • Acute hypertensive vessel rupture (B) classically causes basal ganglia or thalamic bleeds. • Basilar skull fractures (D) lead to petrous or posterior fossa venous haemorrhage, not isolated callosal bleeds. • Cerebral amyloid angiopathy (E) produces lobar cortical–subcortical haemorrhages in the elderly, not midline injuries in young trauma patients. 145. On colour Doppler renal ultrasound, which artefact produces a rapidly alternating red–blue “mosaic” posterior to a 10 mm hyperechoic focus, thereby confirming a renal calculus? A. Aliasing artefact B. Mirror-image artefact C. Twinkle artefact D. Blooming artefact E. Reverberation artefact Explanation: (by Perplexity AI, generated with the OpenAI O3 model) • Twinkle artefact appears on colour Doppler as a chaotic red–blue signal immediately behind a rough, highly reflective interface such as a renal stone; it is sensitive for calculi ≥5 mm and reliably highlights a 10 mm calculus even when acoustic shadowing is subtle. • Aliasing (A) results from velocities exceeding the Nyquist limit and occurs within true vascular flow, not behind stationary stones. • Mirror-image artefact (B) duplicates structures across strong reflectors like the diaphragm, not at the site of a calculus. • Blooming (D) is colour “bleed” from over-gain and obscures rather than localises stones. • Reverberation artefact (E) produces multiple linear echoes from parallel reflectors and lacks the distinctive colour mosaic. 146. Which renal lesion is most frequently encountered as a palpable abdominal mass in neonates? A. Congenital mesoblastic nephroma B. Wilms tumour C. Multilocular cystic nephroma D. Rhabdoid tumour of the kidney E. Renal cell carcinoma
  • 64.
    EBDR Exam MCQs& Concepts June 2025 Dr. Kareem Alnakeeb 58 Explanation: (by Perplexity AI, generated with the OpenAI O3 model) • Congenital mesoblastic nephroma is the commonest renal tumour presenting in the neonatal period, accounting for the majority of solid renal masses detected before three months of age. Its benign nature and typical presentation as a unilateral solid mass make it the leading consideration when a newborn has a renal lump. • Wilms tumour is far more prevalent in toddlers and preschool children, not neonates. • Multilocular cystic nephroma is cystic rather than predominantly solid and is much rarer. • Rhabdoid tumour is aggressive but extremely uncommon, while renal cell carcinoma is exceptionally rare in infancy and usually occurs in older children or adolescents. 147. According to PI-RADS v2.1, which MRI feature upgrades a prostate lesion from PI-RADS 4 to PI-RADS 5? A. Low apparent diffusion coefficient <750 mm²/s B. Marked early enhancement on dynamic contrast imaging C. ≥1.5 cm maximum lesion diameter D. Definite extraprostatic extension or invasive behavior E. Contact with the prostatic capsule exceeding 10 mm Explanation: (by Perplexity AI, generated with the OpenAI O3 model) • PI-RADS v2.1 designates category 5 when a lesion already meeting PI-RADS 4 criteria is either ≥15 mm in greatest dimension or shows definite extraprostatic extension (EPE) or invasive features on MRI. • EPE/invasion (option D) carries the higher likelihood of clinically significant prostate cancer and therefore triggers the upgrade. • While a size ≥1.5 cm (option C) is an alternative route to PI-RADS 5, the question asks for the single feature that upgrades beyond size alone; definite EPE/invasion is that distinguishing criterion. • Low ADC (A), early enhancement (B) and capsular contact length (E) contribute to scoring but, on their own, do not automatically raise a PI-RADS 4 lesion to category 5. 148. In prostate artery embolization performed for benign prostatic hyperplasia, what is the primary therapeutic aim of the procedure? A. Prevent malignant transformation of prostatic tissue B. Shrink the prostate by reducing its arterial blood supply C. Relieve bladder neck obstruction by stent placement D. Ablate peri-urethral tissue with focused ultrasound E. Deliver intra-arterial antibiotics to treat prostatitis Explanation: (by Perplexity AI, generated with the OpenAI O3 model) • Prostate artery embolization (PAE) involves catheter-directed delivery of embolic particles into prostatic arterial branches, producing ischaemia and volume reduction of hyperplastic prostatic tissue. The consequent prostate shrinkage alleviates lower urinary tract symptoms. • Option C describes a urological stenting approach, not embolization; Option D refers to high- intensity focused ultrasound therapy; Option E is irrelevant to BPH management; Option A is incorrect because PAE targets symptom control, not cancer prevention.
  • 65.
    EBDR Exam MCQs& Concepts June 2025 Dr. Kareem Alnakeeb 59 149. During prostatic artery embolisation to treat benign prostatic hyperplasia, the catheter is usually introduced via which artery? A. Common femoral artery B. Radial artery C. Brachial artery D. External iliac artery E. Inferior epigastric artery Explanation: (by Perplexity AI, generated with the OpenAI O3 model) • The standard access for prostatic artery embolisation is a percutaneous puncture of the common femoral artery, allowing a straight course into the internal iliac arteries and their prostatic branches. • The radial artery (B) is gaining popularity but still accounts for a minority of cases and requires longer catheters with potentially higher radiation dose. • The brachial artery (C) is rarely selected because of its small calibre and higher risk of spasm. • Direct puncture of the external iliac artery (D) is not performed in routine practice; instead it is reached intraluminally from the femoral puncture. • The inferior epigastric artery (E) is a branch of the external iliac and is never used as an access site. 150. In uterine fibroid embolization, which vessel is selectively catheterised to deliver the embolic particles? A. Ovarian artery B. Internal pudendal artery C. Inferior mesenteric artery D. Uterine artery E. Superior vesical artery Explanation: (by Perplexity AI, generated with the OpenAI O3 model) • Uterine fibroid embolization (also called uterine artery embolization) works by occluding the arterial supply that feeds the fibroid plexus. • An interventional radiologist advances a catheter into each uterine artery—branches of the anterior division of the internal iliac—and injects calibrated particles until stasis is achieved, producing selective fibroid infarction while preserving surrounding myometrium. • The ovarian artery (A) is an occasional collateral but is not the routine target; internal pudendal (B) and superior vesical (E) arteries supply pelvic organs other than the uterus, and the inferior mesenteric artery (C) supplies the hind-gut. 151. On MRI evaluation of a stage IV bladder tumour, how does the muscularis propria typically appear? A. Intact continuous low-signal band on T2-weighted images B. Continuous low-signal band with focal early contrast enhancement C. Complete disruption of the low-signal band with tumour extending into perivesical fat D. Non-enhancing low-signal band on all sequences E. Continuous intermediate-signal band with high ADC values on DWI/ADC Explanation: (by Perplexity AI, generated with the OpenAI O3 model) • Stage IV (≥T4) disease implies that tumour has breached the muscularis propria and invaded perivesical fat or adjacent organs. • Consequently, the normal hypointense muscularis band seen on T2WI is completely lost and replaced by intermediate-signal tumour that protrudes beyond the bladder wall; dynamic contrast images show early enhancement through the full thickness, and DWI shows high-signal tumour outside the bladder.
  • 66.
    EBDR Exam MCQs& Concepts June 2025 Dr. Kareem Alnakeeb 60 • Options A, B and D describe preserved or only focally disturbed muscle compatible with ≤T2 disease, while option E depicts normal muscle signal characteristics rather than invasion. 152. A 3-day-old neonate presents with bilious vomiting and abdominal distension. Abdominal radiography shows a markedly dilated stomach and proximal duodenum, while Doppler ultrasound demonstrates the “whirlpool” sign of mesenteric vessels twisting around the superior mesenteric artery. What is the most likely diagnosis? A. Annular pancreas B. Duodenal atresia C. Hypertrophic pyloric stenosis D. Jejunal atresia E. Midgut volvulus Explanation: (by Perplexity AI, generated with the OpenAI O3 model) • Midgut volvulus occurs when malrotated mid-intestinal loops twist around the SMA, producing proximal obstruction (dilated stomach/duodenum) and the pathognomonic whirlpool sign on colour Doppler, making option E correct. • Annular pancreas and duodenal atresia both cause bilious vomiting but classically show the “double-bubble” without a whirlpool sign. • Hypertrophic pyloric stenosis presents later (2–8 weeks) with non-bilious projectile vomiting and no dilated duodenum. • Jejunal atresia gives multiple dilated small-bowel loops (“triple-bubble” or beyond) rather than isolated proximal dilatation and lacks vascular whirlpool. 153. Neonatal adrenal haemorrhage is more frequently seen on which side? A. Bilateral equally B. Left adrenal gland C. Right adrenal gland D. Depends on birth weight E. Depends on gestational age Explanation: (by Perplexity AI, generated with the OpenAI O3 model) • The right adrenal gland is more commonly affected because its venous drainage is directly into the inferior vena cava, making it vulnerable to pressure fluctuations during birth trauma, hypoxia or sepsis. • The left gland drains via the longer left adrenal vein into the renal vein, offering a relative pressure buffer. • Bilateral haemorrhage is possible but less common, while birth weight and gestational age influence overall risk but not lateral predilection. 154. Anteroposterior long-leg radiographs of a 3-year-old with progressive bow-legs are reviewed; which one of the following radiographic findings is NOT typical of Blount disease? A. Proximal tibial metaphyseal beaking B. Wedging of the medial tibial epiphysis C. Tibial shaft valgus D. Varus angulation of the tibia below the knee E. Metaphyseal–diaphyseal angle greater than 11°
  • 67.
    EBDR Exam MCQs& Concepts June 2025 Dr. Kareem Alnakeeb 61 Explanation: (by Perplexity AI, generated with the OpenAI O3 model) • Blount disease is a growth disturbance of the medial proximal tibial physis that produces progressive varus deformity at and just below the knee. • Classic radiographic hallmarks include medial metaphyseal beaking, depression and wedging of the medial epiphysis, and an increased metaphyseal–diaphyseal (Drennan) angle above 11°. Together these lead to overall tibial shaft varus, not valgus. • Therefore tibial shaft valgus is inconsistent with Blount disease. • The other options describe well-recognised features seen on plain films in both infantile and adolescent forms of the condition. 155. Which classification system is most widely used for categorising proximal femoral focal deficiency on plain radiographs? A. Amstutz–Wilson classification B. Hamanishi classification C. Aitken classification D. Gillespie–Torode classification E. Hillman classification Explanation: (by Perplexity AI, generated with the OpenAI O3 model) • The four-tier Aitken classification (classes A–D) is the standard radiographic system for proximal femoral focal deficiency, describing severity by the presence and relationship of the femoral head, neck and acetabulum; it guides prognosis and surgical planning. • Later systems such as Amstutz–Wilson and Hamanishi are elaborations, while Gillespie–Torode and Hillman focus on clinical groupings or specific imaging refinements, so they are less commonly adopted in everyday practice.
  • 68.
    EBDR Exam MCQs& Concepts October 2024 Dr. Kareem Alnakeeb 62 October 2024 1. A 40-year-old mother of two presents with a right lower abdominal lump near a surgical scar and with a cyclical history of pain. Ultrasound shows a 2 cm solid hypoechoic lesion in the subcutaneous tissue. Doppler shows internal vascularity. The most likely diagnosis is? F. Desmoid tumor G. Endometriosis H. Metastasis I. Lymph node J. Suture granuloma Source: Gupta, Chaitanya. 300 Single Best Answers for the Final FRCR Part A. 1st ed., Jaypee UK, 2010. Explanation: • Endometriosis can be found in surgical scars or needle tracts. • Most cases of subcutaneous endometriosis occur in Pfannenstiel incisions. • Abdominal wall endometriosis is thought to occur in up to 1% of cases. • Clinically it presents as a cyclical painful lump and can arise many years after surgery. 2. A 74-year-old man with increased urinary frequency and hesitancy is found to have an enlarged prostate on digital rectal examination. He is referred for a TRUS and biopsy. Which one of the following statements best describes the TRUS findings of benign prostatic hypertrophy (BPH)? A. Dense echogenic foci are seen at the margin of the peripheral and transitional zones. B. The central zone is enlarged. C. The peripheral zone is enlarged and appears homogeneously hypoechoic. D. The peripheral zone is enlarged and is of mixed echogenicity. E. The transitional zone is enlarged. Source: McQueen, Andrew S., et al. Grainger & Allison’s Diagnostic Radiology: Single Best Answer MCQs. 5th ed., Churchill Livingstone, 2009. Explanation: • The central zone atrophies with age while the transitional zone increases in size as it develops BPH. • Peripheral zone enlargement is not a feature of BPH. 3. HSG shows small diverticular outpouchings in the isthmic portion of the right fallopian tube with distal tube occlusion. What is the diagnosis? A. Salpingitis isthmica nodosa (SIN) B. Tubal polyps C. Adenomyosis D. Asherman’s syndrome E. Ectopic pregnancy Source: Leen, Edward. Succeeding in the New FRCR Part 2A Exam: Single Best Answer (SBA) Revision Questions for Modules 1-6. 1st ed., UNKNO, 2011. Explanation: • SIN is associated with pelvic inflammatory disease and a higher risk of ectopic pregnancy.
  • 69.
    EBDR Exam MCQs& Concepts October 2024 Dr. Kareem Alnakeeb 63 4. You are the radiology SpR consenting a 70-year-old man for a transrectal ultrasound (TRUS) prostate biopsy. Which one of the following is not a recognized complication of this procedure? A. Haematuria B. Haematospermia C. Perirectal bleeding D. Pneumoperitoneum E. Pain/discomfort post-procedure Source: McQueen, Andrew S., et al. Grainger & Allison’s Diagnostic Radiology: Single Best Answer MCQs. 5th ed., Churchill Livingstone, 2009. Explanation: • Almost all patients complain of pain/discomfort afterwards and up to 80% will experience either hematuria or haematospermia. • Perirectal bleeding (up to 37%), infection, vasovagal attack, urinary retention and epididymitis are other recognized complications. • Pneumoperitoneum should not occur because the prostate lies well below the peritoneal reflection. 5. Which of the following has the highest specificity for Non-Accidental Iniury (NAI)? A. Scapula fracture B. Vetebral fractures C. Complex skull fracture D. Digital fracture E. Epiphyseal separation injuries Source: Leen, Edward. Succeeding in the New FRCR Part 2A Exam: Single Best Answer (SBA) Revision Questions for Modules 1-6. 1st ed., UNKNO, 2011. Explanation: • B-E are moderately specific. Other high specificity signs are classic metaphyseal fractures, rib fractures, and sternal fractures. 6. A 37-year-old man with AIDS presents with confusion, lethargy and memory loss. CT of the brain demonstrates multiple supratentorial enhancing masses. Which imaging feature favors a diagnosis of toxoplasmosis rather than primary CNS lymphoma? A. subependymal distribution B. lesions hyperdense on unenhanced CT C. lesion size .3 cm D. hypovascularity on MR perfusion study E. increased uptake of thallium-201 on SPECT Source: Bydder, Megan, et al. Get Through Final FRCR Part A: SBAs for the Modular Examination. 1st ed., Hodder Education Publishers, 2009. Explanation: Toxoplasmosis • Toxoplasmosis is the most common cause of a cerebral mass lesion in patients with AIDS. • Typical appearances are of multiple, hypoattenuating, ,2 cm lesions with a predilection for the basal ganglia. Lymphoma • Lymphoma is the second commonest mass lesion, with characteristic features including hyperdense lesions (though less frequently than in non-AIDS lymphoma) in a periventricular location with subependymal spread. Common Imaging Features • Lesions in both conditions may show solid or ring enhancement. • Hemorrhage is unusual in lymphoma but may be seen in toxoplasmosis, particularly following treatment.
  • 70.
    EBDR Exam MCQs& Concepts October 2024 Dr. Kareem Alnakeeb 64 Thallium Scanning • Thallium scanning may be useful to distinguish the two if there is diagnostic uncertainty. • CNS lymphoma is thallium avid whereas toxoplasmosis does not show uptake. MR Perfusion Studies • MR perfusion studies may also help to differentiate the two conditions. • Lymphomas demonstrate increased perfusion relative to surrounding tissue, while toxoplasmosis is hypovascular. Clinical Significance • Differentiation is important, as early radiation therapy confers a significant survival advantage in CNS lymphoma. 7. Which is the cause of a cystic rather than a hemorrhagic cause of brain metastases? A. Adenocarcinoma of the lung B. Malignant melanoma C. Choriocarcinoma D. Renal cell carcinoma E. Thyroid carcinoma Source: Leen, Edward. Succeeding in the New FRCR Part 2A Exam: Single Best Answer (SBA) Revision Questions for Modules 1-6. 1st ed., UNKNO, 2011. Explanation: • Squamous cell lung cancer and adenocarcinoma of the lung cause cystic metastasis to the brain. • Answers B-E are causes of hemorrhagic metastases (Mnemonic: MR CT BB) o M: melanoma o R: renal cell carcinoma o C: choriocarcinoma o T: thyroid carcinoma, teratoma o B: bronchogenic carcinoma o B: breast carcinom 8. Which of the following best describes imaging changes in a colloid cyst? A. Typically hypodense on non-contrast CT B. Appears high SI on T1 C. Appears low SI on T2 D. Commonly widens septum pellucidi E. Most commonly causes symmetrical enlargement of lateral ventricles Source: Leen, Edward. Succeeding in the New FRCR Part 2A Exam: Single Best Answer (SBA) Revision Questions for Modules 1-6. 1st ed., UNKNO, 2011. Explanation: • Protein content/paramagnetic effect of magnesium Mg2+/ calcium Ca2+ /, Iron Fe, in a cyst cause increased T1 and T2 SI. Colloid cysts appear iso/hyperdense on NCCT. • They can occasionally widen septum pellucidum and cause asymmetrical enlargement of the lateral ventricles.
  • 71.
    EBDR Exam MCQs& Concepts October 2024 Dr. Kareem Alnakeeb 65 9. Which of the following features favor Rathke’s cleft cyst rather than craniopharyngioma? A. Absence of calcification B. Cystic element on MR C. Involvement of suprasellar and sellar regions D. Enhancement of the wall E. High signal intensity on T1 Source: Leen, Edward. Succeeding in the New FRCR Part 2A Exam: Single Best Answer (SBA) Revision Questions for Modules 1-6. 1st ed., UNKNO, 2011. Explanation: • Rathke’s cleft cysts do not calcify. They affect women to men in a 2:1 ratio and adults from 40-60 years of age. They cause variable MR appearances depending on protein content of cyst. They can rarely show enhancement. 10. Which of the following is an extra-axial posterior fossa tumor in adults? A. Choroid plexus papilloma B. Metastasis C. Haemangioblastoma D. Lymphoma E. Glioma Source: Leen, Edward. Succeeding in the New FRCR Part 2A Exam: Single Best Answer (SBA) Revision Questions for Modules 1-6. 1st ed., UNKNO, 2011. Explanation: • Other extra-axial posterior fossa masses include acoustic neuroma, meningioma, chordoma and epidermoid. 11. Which is the most common site of metastatic spread in medulloblastoma? A. Axial skeleton B. Lymph nodes C. Lung D. Subarachnoid space E. Liver Source: Leen, Edward. Succeeding in the New FRCR Part 2A Exam: Single Best Answer (SBA) Revision Questions for Modules 1-6. 1st ed., UNKNO, 2011. Explanation: • Subarachnoid space is the most common, with drop metastases occurring in 40%. 12. Following a large postpartum hemorrhage, a 25-year-old woman develops a severe headache and sudden visual field defect. What is the most likely diagnosis? A. intracerebral hemorrhage B. reversible posterior leukoencephalopathy C. subarachnoid hemorrhage D. Sheehan’s syndrome E. vertebral artery dissection Source: Bydder, Megan, et al. Get Through Final FRCR Part A: SBAs for the Modular Examination. 1st ed., Hodder Education Publishers, 2009. Explanation: Hypertension in Pregnancy • Many of the acute neurological conditions of pregnancy occur with rising blood pressure. Sheehan’s Syndrome and Hypotension • Sheehan’s syndrome results from haemorrhage-induced hypotension causing pituitary infarction. • Early on, this appears as an enlarged homogeneous pituitary with low T1 signal, high T2 signal and post-contrast ring enhancement.
  • 72.
    EBDR Exam MCQs& Concepts October 2024 Dr. Kareem Alnakeeb 66 • Later, there is an empty sella. • Clinical manifestations include visual field loss, headache, ophthalmoplegia and pituitary dysfunction (diabetes insipidus). • Hemorrhage, sepsis and pulmonary embolism cause hypotension that can cause watershed infarction as well as Sheehan’s syndrome. Reversible Posterior Leukoencephalopathy • Reversible posterior leukoencephalopathy produces cortical blindness, headaches, confusion and seizures. • Those affected are often taking immunosuppressant treatment. • Imaging features can be identical to eclampsia, peripartum cerebral angiopathy and hypertensive encephalopathy, but with a posterior predominance. • On CT, there is low attenuation change. • On MRI, there is high signal on T2W/FLAIR images. • ADC maps can differentiate between likely reversible vasogenic oedema (high signal on ADC map showing unrestricted diffusion) and cytotoxic oedema (low signal due to restricted diffusion), which is more likely to progress to infarct. Microangiopathic Haemolytic Anaemias • Microangiopathic haemolytic anaemias, such as thrombotic thrombocytopenic purpura and haemolytic uraemic syndrome, give widespread ischaemia/infarction and haemorrhagic transformation. Vasculitis in Pregnancy • There is no increased risk in pregnancy of vasculitis such as systemic lupus erythematosus, Takayasu’s syndrome or Moyamoya syndrome. Vascular Malformations and Aneurysms • Arteriovenous malformation is no more likely to bleed in pregnancy, but there is an increased risk with arterial aneurysms. 13. A 53-year-old man has an MRI of his pelvis as a staging investigation for bladder cancer. The request card also states that the prostate is mildly enlarged on digital rectal examination and the serum prostate specific antigen (PSA) level is borderline elevated. The reporting radiologist reviews the prostate in detail. Which one of the following statements best describes the MRI findings of a normal prostate gland? A. A On Tlw images, the central zone is of higher signal intensity than the peripheral zone. B. On Tlw images, the central zone is of lower signal intensity than the peripheral zone. C. On T2w images, the peripheral zone is of lower signal intensity than the central and transitional zones D. The peripheral zone is of higher signal intensity than the central zone on T2w images. E. The seminal vesicles are hypointense on T2w images Source: McQueen, Andrew S., et al. Grainger & Allison’s Diagnostic Radiology: Single Best Answer MCQs. 5th ed., Churchill Livingstone, 2009.
  • 73.
    EBDR Exam MCQs& Concepts October 2024 Dr. Kareem Alnakeeb 67 14. An elderly male patient presents with signs suggesting acute middle cerebral artery infarction. Around 21 2 hours after symptom onset, an unenhanced CT of the brain is performed. Among other subtle signs, the basal ganglia are obscured. Reduced perfusion through which of the following vessels best explains this sign? A. lenticulostriate arteries B. anterior choroidal artery C. callosomarginal artery D. recurrent artery of Heubner E. angular artery Source: Bydder, Megan, et al. Get Through Final FRCR Part A: SBAs for the Modular Examination. 1st ed., Hodder Education Publishers, 2009. Explanation: Lenticulostriate arteries • The lenticulostriate arteries are vessels arising from the M1 segment of the middle cerebral artery; there are medial and lateral groups. • Collectively, they supply the thalamus, caudate and lentiform nuclei. Anterior cerebral artery branches • The callosomarginal artery and the recurrent artery of Heubner are anterior cerebral artery branches. • The latter provides some supply to the anterior limb of the internal capsule, and parts of the caudate nucleus and globus pallidus. Middle cerebral artery cortical branch • The angular artery is a cortical branch of the middle cerebral artery. Anterior choroidal artery • The anterior choroidal artery also supplies parts of the internal capsule and basal ganglia but is a branch of the internal carotid artery. Basal ganglia anatomy • The nuclei of the basal ganglia are the amygdala, claustrum, lentiform and caudate nuclei, with the internal, external and extreme capsules being associated white matter tracts. 15. An 80-year-old man presents acutely with a dense hemiplegia. CT perfusion is performed soon after admission, which suggests that the entire involved arterial territory is beyond recovery. Which of the following options represents the most likely combination of cerebral blood flow, mean transit time and cerebral blood volume, respectively, seen within the affected brain parenchyma, compared with unaffected parenchyma? A. increased, increased, increased B. increased, increased, decreased C. increased, decreased, decreased D. decreased, decreased, decreased E. decreased, increased, decreased Source: Bydder, Megan, et al. Get Through Final FRCR Part A: SBAs for the Modular Examination. 1st ed., Hodder Education Publishers, 2009. Explanation: Clinical applications • Cerebral perfusion CT can distinguish viable but ischemic tissue (the penumbra) from tissue that is beyond recovery. • Other uses include: o evaluation of vasospasm after subarachnoid hemorrhage, o assessment of cerebrovascular reserve with acetazolamide (cerebral arteriole vasodilator) in cases of vascular stenosis,
  • 74.
    EBDR Exam MCQs& Concepts October 2024 Dr. Kareem Alnakeeb 68 o evaluation of collateral flow and cerebrovascular reserve in patients having temporary balloon occlusion and o assessment of microvascular permeability of intracranial neoplasms. Central volume principle • Cerebral perfusion CT utilizes the central volume principle. • This states that CBF=BV/MTT, where CBF is cerebral blood flow, CBV is cerebral blood volume and MTT is mean transit time. CT perfusion techniques • In practice, two CT perfusion techniques can be used. o One is perfused-blood volume mapping, in which a quantity is assigned to cerebral blood volume by subtracting unenhanced CT data from CT angiographic data. ▪ It has the advantage of imaging the whole brain. o The second technique is a dynamic, contrast-enhanced technique that acquires data from a limited number of axial slices, and monitors the first pass of an iodinated contrast agent bolus through the cerebral circulation. ▪ This requires an unenhanced CT brain, followed by a dynamic CT performed during injection of 50 ml of iodinated contrast (300 mg I/ml) at 4 ml/s. ▪ The first pass of contrast is observed in the brain. Perfusion measurement and maps • Cerebral perfusion is related to the concentration of iodinated contrast, which is directly related to the attenuation measured. • Several maps are produced, including the CBV, CBF and MTT. o MTT is derived from arterial and venous enhancement curves, measured by using regions of interest placed on an artery (one that is not occluded as part of an acute event) and a venous sinus. o CBV is the area under the enhancement curves, and o CBF is obtained from the central volume equation. Differentiating infarct and penumbra • Differentiation of infarcted brain from penumbra is important because, while penumbra can be saved by timely thrombolysis, infarcted tissue has an increased risk of bleeding from thrombolysis with no chance of recovery. • CBF is decreased in both ischemia and infarction, • MTT is longer (.6 s) in both, while • CBV is decreased in infarct but increased (or normal) in the penumbra due to cerebral autoregulatory mechanisms. Key parameters: • MTT is the most sensitive for stroke. • So this or CBF can be used to detect stroke while CBV is used to determine whether there is infarct or reversible ischemia.
  • 75.
    EBDR Exam MCQs& Concepts October 2024 Dr. Kareem Alnakeeb 69 16. Which is most likely to represent an intramedullary mass lesion? A. Ependymoma B. Meningioma C. Neurofibroma D. Arachnoid cyst E. Abscess Source: Leen, Edward. Succeeding in the New FRCR Part 2A Exam: Single Best Answer (SBA) Revision Questions for Modules 1-6. 1st ed., UNKNO, 2011. Explanation: • Other intramedullary masses include astrocytomas are dermoids (lipoma/teratomas), acutely expanding infarcts and haematoma. 17. A 38-year-old male with Human Immunodeficiency Virus (HIV) stopped taking his retrovirals 6 months ago and now presents with confusion. CT brain shows non-enhancing hypodensities, with apparent dilated perivascular spaces, although these were not present on a CT brain from 2 years ago. What is the most likely cause? A. Cryptococcus B. Progressive multifocal leukoencephelopathy C. Tuberculosis D. CMV encephalitis E. Toxoplasmosis Source: Leen, Edward. Succeeding in the New FRCR Part 2A Exam: Single Best Answer (SBA) Revision Questions for Modules 1-6. 1st ed., UNKNO, 2011. Explanation: • More commonly cryptococcus meningitis but cryptococcus or gelatinous pseudocysts reside in dilated perivascular spaces. 18. A 30-year-old male with recurrent Transient Ischemic Attacks (TIAs) and a history of migraine with aura undergoes CT brain. Subcortical infarcts are identified raising suspicion of cerebral autosomal dominant arteriopathy with subcortical infarcts (CADASIL). Which is the most characteristically involved location for subcortical infarcts? A. Anterior temporal pole B. Frontal lobe C. Centrum semiovale D. Deep grey matter structure E. Pons Source: Leen, Edward. Succeeding in the New FRCR Part 2A Exam: Single Best Answer (SBA) Revision Questions for Modules 1-6. 1st ed., UNKNO, 2011. Explanation: • A young patient with migraines, auras, TIAs or subcortical strokes should raise suspicion of CADASIL. • Subcortical infarcts are characteristically in the anterior temporal pole and external capsule but may involve C, D and E.
  • 76.
    EBDR Exam MCQs& Concepts October 2024 Dr. Kareem Alnakeeb 70 19. A 65-year-old woman is referred for a pelvic radiograph to investigate intermittent right hip pain. The radiograph shows thin lucent lines within both inferior pubic rami. Which radiographic feature would support a diagnosis of osteoporotic fracture rather than osteomalacia? A. Callus formation B. Failure to extend across the entire width of the bone C. Sclerotic margin to lucencies D. Similar appearances within the proximal femora E. Symmetrical appearance Source: McQueen, Andrew S., et al. Grainger & Allison’s Diagnostic Radiology: Single Best Answer MCQs. 5th ed., Churchill Livingstone, 2009. 20. Considering Moyamoya disease in adults: A. Infarct seen in cortical/subcortical areas B. Multiple small flow voids are characteristic C. Affects anterior circulation D. Presentation with ischemia is more common in adults than in children E. The supraclinoid MCA is spared Source: Leen, Edward. Succeeding in the New FRCR Part 2A Exam: Single Best Answer (SBA) Revision Questions for Modules 1-6. 1st ed., UNKNO, 2011. Explanation: • Multiple flow voids are due to hypertrophied lenticulostriate arteries. • Children more often present with ischemia and infarct in cortical/subcortical areas. • Adults more often present with hemorrhage than children, but when infarcts do occur, they are most often in the deep white matter. • The disease can involve the posterior circulation. • The supraclinoid MCA is the first to be involved. 21. A 36-year-old woman presents with primary infertility. A transabdominal ultrasound shows a normal anteverted uterus and bilateral adnexal masses. A subsequent MRI shows bilateral high signal ovarian masses on both T1w and T2w sequences. On fat-suppressed Tlw images, the lesions remain high signal. What is the most likely diagnosis? A. Bilateral dermoid cysts B. Bilateral endometriomas C. Bilateral ovarian fibromas D. Bilateral theca lutein cysts E. Polycystic ovaries Source: McQueen, Andrew S., et al. Grainger & Allison’s Diagnostic Radiology: Single Best Answer MCQs. 5th ed., Churchill Livingstone, 2009. Explanation: • Theca lutein cysts contain straw-colored fluid which is low signal on T1w and high signal on T2w images.
  • 77.
    EBDR Exam MCQs& Concepts October 2024 Dr. Kareem Alnakeeb 71 22. An 18 year old man undergoes a Tc MDP bone scan to investigate pain in the right hip. A ‘hot’ lesion is seen in the right proximal femur. No other lesions are seen. Which of the following lesions would appear as ‘hot’ on a Tc MDP bone scan? A. Osteopoikilosis B. Fibrous cortical defect C. Acute fracture within 12 hours of injury D. Fibrous dysplasia E. Haemangioma Source: Currie, Stuart, et al. SBAs for the FRCR 2A. 1st ed., Cambridge University Press, 2010. Explanation: • The most common site of monostotic fibrous dysplasia is the ribs, followed by proximal femur and craniofacial bones. Three-quarters of cases present before age 30. • Other benign lesions causing a ‘hot’ on bone scan include Paget’s disease, brown tumours, aneurysmal bone cysts, osteoid osteoma and chondroblastoma. • Acute fractures are not usually ‘hot’ until after the first 24–48 hours. 23. A premature baby of a diabetic mother delivered by caesarean section develops tachypnoea soon after birth. Chest radiographs show hyperinflated lungs with prominent interstitial markings and prominent horizontal fissure. These changes resolved after 3 days. The most likely diagnosis is? A. Respiratory distress syndrome B. Meconium aspiration syndrome C. Transient tachypnoea of the newborn D. Left heart failure E. Normal lung of newborn Source: Gupta, Chaitanya. 300 Single Best Answers for the Final FRCR Part A. 1st ed., Jaypee UK, 2010. Explanation: • If the processes of clearing amniotic fluid from the lungs is impaired in a new born transient tachypnoea of the newborn develops. This is associated with prematurity, caesarean section and diabetic mothers. These are typical radiographic features, which resolve in 2–3 days. 24. Which of the following indicates T3 rather than T4 lung cancer? A. Invasion of the oesophagus B. Invasion of the trachea C. Invasion of the pericardium D. Malignant pleural effusion E. Invasion of the vertebral body Source: Leen, Edward. Succeeding in the New FRCR Part 2A Exam: Single Best Answer (SBA) Revision Questions for Modules 1-6. 1st ed., UNKNO, 2011. Explanation: • T3 disease features include a tumor of any size less than 2cm from the carina, invasion of the parietal pleura, chest wall, diaphragm, mediastinal pleura, pericardium, pleural effusion or satellite nodule in the same lobe. • T4 disease is characterized by invasion of the heart, great vessels, trachea, oesophagus, vertebral body, carina or the presence of a malignant pleural effusion. • The TNM staging system was updated in 2009 (AJR, 2010).
  • 78.
    EBDR Exam MCQs& Concepts October 2024 Dr. Kareem Alnakeeb 72 25. A 70-year-old patient undergoes a staging CT for renal cell carcinoma, which shows ligamentous ossification extending from the fifth to ninth thoracic vertebrae. There is relative sparing of the left side of the vertebrae and disc space height is preserved. The apophyseal and sacroiliac joints appear normal. What is the most likely diagnosis? A. Ankylosing spondylitis B. Degenerative disc disease C. Diffuse idiopathic skeletal hyperostosis D. Metastatic disease E. Ossification of the posterior longitudinal ligament Source: McQueen, Andrew S., et al. Grainger & Allison’s Diagnostic Radiology: Single Best Answer MCQs. 5th ed., Churchill Livingstone, 2009. 26. A 75-year-old lady undergoes bone mineral density (BMD) measurements at the hip and spine by means of dual energy radiograph absorptiometry (DXA). What findings would satisfy the World Health Organisation (WHO) criteria for osteoporosis? A. BMD below the young adult reference mean B. BMD between —1 and —2.5 standard deviations below that of the young adult reference mean C. BMD more than —2.5 standard deviations below the young adult reference mean D. BMD more than —2.5 standard deviations below the young adult reference mean, with one low- energy fracture E. BMD more than —2.5 standard deviations below the young adult reference mean, with two low- energy fractures Source: McQueen, Andrew S., et al. Grainger & Allison’s Diagnostic Radiology: Single Best Answer MCQs. 5th ed., Churchill Livingstone, 2009. 27. A series of neonatal radiographs reveal a narrow thorax with short ribs, square iliac wings with horizontal acetabular roofs, short sacrosciatic notches, progressive narrowing of the interpedicular distance and posterior scalloping of the vertebral bodies. What is the most likely diagnosis? A. Achondroplasia B. Campomelic dysplasia C. Cleidocranial dysplasia D. Ellis-van Creveld syndrome E. Morquio's syndrome Source: McQueen, Andrew S., et al. Grainger & Allison’s Diagnostic Radiology: Single Best Answer MCQs. 5th ed., Churchill Livingstone, 2009. Explanation: • The iliac wings in Morquio's syndrome are characteristically flared rather than square. 28. A 33 year old male with no significant past medical history presents with headache, drowsiness and confusion. CT shows a hypodense lesion with a smooth regular wall centred over the left lentiform nucleus. There is surrounding oedema and mass effect with effacement of the ipsilateral Sylvian fissure. On T2-weighted MR imaging, the lesion is hyperintense and is surrounded by a hypointense rim and hyperintense oedema. There is peripheral enhancement post-contrast injection, and diffusion-weighted imaging demonstrates restricted diffusion within the lesion. What is the most likely diagnosis? A. Glioblastoma multiforme B. Pyogenic abscess C. Toxoplasmosis D. Lymphoma E. Metastasis Source: Currie, Stuart, et al. SBAs for the FRCR 2A. 1st ed., Cambridge University Press, 2010. Explanation: • The differential diagnosis for a solitary ring-enhancing lesion of the brain includes (‘MAGICAL DR’): Metastasis; Abscess; Glioma/Glioblastoma multiforme; Infarction; Contusion; AIDS
  • 79.
    EBDR Exam MCQs& Concepts October 2024 Dr. Kareem Alnakeeb 73 (toxoplasmosis); Lymphoma (often AIDS-related); Demyelinating disease; Resolving haematoma/Radiation necrosis. • Classically, abscesses are located at the corticomedullary junction in the frontal and temporal lobes. o The most common causative organism is Streptococcus. o The wall is generally smooth and regular with relative thinning of the medial wall secondary to a poorer blood supply from white matter (neoplastic lesions usually have a thick, nodular, irregular rim). o In this scenario, the enhancing, T2-hypointense rim suggests abscess. o Restricted diffusion is also highly suggestive of an abscess. • Lymphoma may be hyperdense on CT due to a high nuclear-to-cytoplasmic ratio and typically shows solid homogeneous enhancement in immunocompetent patients. 29. A 34-year-old man with chronic back pain is referred by his GP for thoracic and lumbar spine radiographs. The GP is concerned about the possibility of ankylosing spondylitis. Which radiological feature is atypical for ankylosing spondylitis, and might suggest an alternative diagnosis? A. Ankylosis of the apophyseal joints B. Anterior longitudinal ligament calcification C. Osteophyte formation D. Sclerosis of the anterior corners of the vertebrae E. Vertebral body squaring Source: McQueen, Andrew S., et al. Grainger & Allison’s Diagnostic Radiology: Single Best Answer MCQs. 5th ed., Churchill Livingstone, 2009. Explanation: • Syndesmophytes, rather than osteophytes, are characteristic features of ankylosing spondylitis. • They are differentiated from osteophytes by their vertical orientation, as they represent ossification of the outer border of the annulus fibrosus. • Progression and maturation of the syndesmophytes result in a ‘bamboo spine’. 30. An incidental finding on plain film is a 2-cm lucency within the diaphysis of the right humerus, which exhibits chondroid calcification. Which clinical or radiological feature would favour a diagnosis of chondrosarcoma rather than enchondroma? A. Age less than 20 years B. Circular, curvilinear or nodular calcific densities C. Periosteal reaction D. Slow growth E. Well-defined round or elliptical margin Source: McQueen, Andrew S., et al. Grainger & Allison’s Diagnostic Radiology: Single Best Answer MCQs. 5th ed., Churchill Livingstone, 2009. Explanation: • Outside the hands and feet, chondrosarcoma is five times more common than enchondroma
  • 80.
    EBDR Exam MCQs& Concepts October 2024 Dr. Kareem Alnakeeb 74 31. An 80-year-old man undergoes skeletal scintigraphy for multifocal skeletal pain, malaise and weight loss. The scintigram shows diffusely increased activity throughout die skeleton, with absent renal activity. What is the most likely diagnosis? A. Metastatic bladder cancer B. Metastatic colon cancer C. Metastatic gastric cancer D. Metastatic lung cancer E. Metastatic prostate cancer Source: McQueen, Andrew S., et al. Grainger & Allison’s Diagnostic Radiology: Single Best Answer MCQs. 5th ed., Churchill Livingstone, 2009. 32. A previously well 80-year-old woman sustains a subcapital fracture of the right neck of femur following a fall onto hard ground. The plain film reveals multiple lytic lesions within the pelvic bones and proximal femora, which are highly suspicious for bone metastases. What is the most likely occult primary lesion? A. Carcinoma of the bladder B. Carcinoma of the breast C. Carcinoma of the bronchus D. Carcinoma of the colon E. Carcinoma of the stomach Source: McQueen, Andrew S., et al. Grainger & Allison’s Diagnostic Radiology: Single Best Answer MCQs. 5th ed., Churchill Livingstone, 2009. 33. A solitary, lytic lesion with aggressive features is an unexpected incidental finding on radiography of the left knee. Which radiological feature would favor a diagnosis of metastasis rather than primary bone tumour? A. Bone expansion B. Diaphyseal location C. Florid periosteal reaction D. Tumour bone formation E. Soft tissue mass 34. A 24-year-old man is referred to the gastroenterology outpatient clinic. He describes intermittent bloody diarrhoea with abdominal pain and has lost 5 kg in weight over the past 6 months. His father and uncle both have inflammatory bowel disease. Routine laboratory investigations are remarkable for a moderately elevated CRP. A double contrast barium enema examination is performed. Which of the following findings would be more consistent with Crohn's disease than ulcerative colitis? A. Aphthous ulceration interspersed with areas of normal mucosa B. Fine granular appearance of the descending and sigmoid colon C. Isolated involvement of the rectum and sigmoid D. Shortening and narrowing of the entire colon with absence of haustral folds E. The presence of ‘collar button’ ulceration Source: McQueen, Andrew S., et al. Grainger & Allison’s Diagnostic Radiology: Single Best Answer MCQs. 5th ed., Churchill Livingstone, 2009. Explanation: • Aphthous ulceration is the earliest sign of Crohn's disease on a double contrast barium enema. • The other options are all true of ulcerative colitis—from the earliest signs of fine mucosal granularity to the ‘lead pipe’ appearance of the colon in chronic UC. • Submucosal ulceration can extend laterally in UC giving the ‘collar button’ appearance.
  • 81.
    EBDR Exam MCQs& Concepts October 2024 Dr. Kareem Alnakeeb 75 35. A 70-year-old man complains of a tense painless swelling posterior to his right knee. Ultrasound demonstrates a large cyst, which communicates with the knee joint between which two structures? A. Through the interval between semimembranosus and the lateral head of gastrocnemius B. Through the interval between semimembranosus and the medial head of gastrocnemius C. Through the interval between semimembranosus and semitendinosus D. Through the interval between semitendinosus and the lateral head of gastrocnemius E. Through the interval between semitendinosus and the medial head of gastrocnemius Source: McQueen, Andrew S., et al. Grainger & Allison’s Diagnostic Radiology: Single Best Answer MCQs. 5th ed., Churchill Livingstone, 2009. 36. An 80-year-old man has been admitted to hospital with shortness of breath and a productive, purulent cough. A CXR reveals left lower lobe consolidation. Which additional radiological finding is most likely to suggest a diagnosis of Klebsiella pneumoniae rather than Legionella pneumophildi A. Bulging fissures B. Mediastinal lymphadenopathy C. Pleural effusion D. Pneumothorax E. Septal thickening Source: McQueen, Andrew S., et al. Grainger & Allison’s Diagnostic Radiology: Single Best Answer MCQs. 5th ed., Churchill Livingstone, 2009. Explanation: • Klebsiella pneumoniae leads to an extensive exudative response leading to cavitating lobar consolidation and bulging fissures. • Legionnaire's disease, on the other hand, tends to present with multifocal lobar, homogeneous opacities with a tendency to appear like masses. 37. Regarding MRI examination of the shoulder, what are the signal characteristics of the normal supraspinatus tendon? F. High signal intensity on all sequences G. High signal on T1w, low signal on T2w H. Intermediate signal on all sequences I. Low signal on all sequences J. Low signal on T1w, high signal on T2w Source: McQueen, Andrew S., et al. Grainger & Allison’s Diagnostic Radiology: Single Best Answer MCQs. 5th ed., Churchill Livingstone, 2009. 38. A 66-year-old man undergoes screening for colorectal cancer and is found to have two positive stool samples for fecal occult blood. The patient is asymptomatic with no significant medical history. He is referred for CT colonography (CTC). Which one of the following statements is correct regarding CTC? A. As much as 0.5—1% of examinations result in colonic perforation. B. A routine examination should involve supine imaging only. C. Significant extracolonic pathology is identified in 30-40% of symptom- D. The administration of intravenous contrast (portal venous imaging) is advised for asymptomatic patients, as it improves the detection of colonic E. The use of an antispasmodic (eg Buscopan) immediately prior to gas insufflation enables optimal colonic distension. Source: McQueen, Andrew S., et al. Grainger & Allison’s Diagnostic Radiology: Single Best Answer MCQs. 5th ed., Churchill Livingstone, 2009. Explanation: • CT colonography (CTC) is accepted as a generally safe technique with a reported perforation rate in symptomatic patients of 0.03% (compared with 0.13% with optical colonoscopy).
  • 82.
    EBDR Exam MCQs& Concepts October 2024 Dr. Kareem Alnakeeb 76 • Supine and prone imaging is widely advocated in CTC to maximize colonic distension and discriminate between fecal/fluid bowel residue and genuine pathology, while the use of an antispasmodic is advised to avoid colonic spasm. • There is no strong evidence that intravenous contrast improves detection of colonic lesions, but it may be of benefit in symptomatic patients as it enables a more accurate assessment of extracolonic pathology. 39. A 74-year-old man presents with an 8-week history of altered bowel habit and rectal bleeding. A flexible sigmoidoscopy demonstrates a malignant stricture in the rectum and biopsies confirm rectal adenocarcinoma. An MRI is performed and shows an annular neoplasm at 12 cm. The mass invades 4 mm beyond the rectal wall into the perirectal fat and infiltrates the peritoneal reflection anteriorly. There is a small volume of free peritoneal fluid. What is the radiological T stage? A. TX B. T1 C. T2 D. T3 E. T4 Source: McQueen, Andrew S., et al. Grainger & Allison’s Diagnostic Radiology: Single Best Answer MCQs. 5th ed., Churchill Livingstone, 2009. Explanation: • Evidence of peritoneal invasion indicates stage T4 rectal cancer. 40. A male patient is referred to the on-call surgical team with a 3-day history of generalized abdominal pain and vomiting. The patient has not opened his bowels for 2 days. Examination reveals a distended abdomen with increased bowel sounds. An abdominal radiograph is performed and demonstrates a large dilated loop of large bowel with several loops of dilated small bowel centrally. Which other feature would make a diagnosis of caecal volvulus more likely than that of sigmoid volvulus? A. Haustrae are visible in the gas-filled viscus. B. The apex of the viscus lies in the left upper quadrant. C. The patient is 75 years old. D. The patient is in long-term institutional care. E. The viscus rises above the level of the T10 vertebral body. Source: McQueen, Andrew S., et al. Grainger & Allison’s Diagnostic Radiology: Single Best Answer MCQs. 5th ed., Churchill Livingstone, 2009. Explanation: • The presence of haustrations in a dilated viscus and gas in the appendix are key to the diagnosis of caecal volvulus.
  • 83.
    EBDR Exam MCQs& Concepts October 2024 Dr. Kareem Alnakeeb 77 41. A 23-year-old woman complains of episodes of diarrhoea and rectal bleeding. Her father died of colorectal cancer aged 39. A double contrast barium enema is performed and demonstrates more than one hundred small polyps, measuring up to 5 mm in size, throughout the colon. An upper GI endoscopy demonstrates multiple polypoid lesions in the stomach and duodenum. What is the most likely diagnosis? A. Carcinoid syndrome B. Familial adenomatous polyposis C. Hereditary non-polyposis colorectal cancer D. Juvenile polyposis E. Peutz-Jeghers syndrome Source: McQueen, Andrew S., et al. Grainger & Allison’s Diagnostic Radiology: Single Best Answer MCQs. 5th ed., Churchill Livingstone, 2009. Explanation: • Autosomal dominant condition with multiple colonic adenomas and 100% risk of colorectal carcinoma 20 years after diagnosis. Associated with hamartomas in the stomach, gastric and duodenal adenomas and periampullary carcinoma. 42. A 35-year-old woman is referred to the Radiology Department following the birth of her first child. The baby was delivered 8 days post-term and was a vaginal delivery following a prolonged labour and episiotomy. Two months later, the patient continues to experience faecal incontinence and an anal sphincter tear is suspected. Which investigation would be most useful to demonstrate anal sphincter damage? A. Barium evacuation proctogram B. CT colonography C. CT with rectal contrast media D. Endoanal ultrasound E. MRI of the pelvis with a body coil Source: McQueen, Andrew S., et al. Grainger & Allison’s Diagnostic Radiology: Single Best Answer MCQs. 5th ed., Churchill Livingstone, 2009. Explanation: • High-frequency endosonography allows an accurate assessment of the four layers of the anal wall; superficial and deep mucosa, submucosa and muscularis propria.
  • 84.
    EBDR Exam MCQs& Concepts October 2024 Dr. Kareem Alnakeeb 78 43. A 64-year-old woman presents with bloating and vague pelvic pain and is referred for a pelvic ultrasound. On transabdominal ultrasound, she is found to have a large right adnexal mass. Which one of the following sonographic findings would indicate that this mass is more likely to be malignant than benign? A. Doppler waveform with a high resistive index (> 0.8) B. Homogeneously hypoechoic mass with posterior acoustic enhancement C. Multiple septations that are approximately 1 mm thick D. Papillary projections E. Size > 4 cm Source: McQueen, Andrew S., et al. Grainger & Allison’s Diagnostic Radiology: Single Best Answer MCQs. 5th ed., Churchill Livingstone, 2009. 44. A 60-year-old nulliparous woman presents with postmenopausal bleeding. On transvaginal ultrasound, her endometrium is 8 mm thick and the endomyometrial junction appeared indistinct. The radiologist suspects invasive endometrial cancer and refers her for an MRI examination. What are the likely findings on MRI? A. On unenhanced T1w images the endometrial cancer appears of high signal intensity compared to the surrounding myometrium. B. On contrast-enhanced T1w images, endometrial cancer shows avid enhancement compared with surrounding myometrium. C. On T2w images the normally high signal junctional zone is disrupted. D. T1w fat-saturated sequences are best used to assess the junctional zone. E. The endometrial cancer demonstrates delayed/little enhancement compared to the normal surrounding myometrium on postcontrast T1w images. Source: McQueen, Andrew S., et al. Grainger & Allison’s Diagnostic Radiology: Single Best Answer MCQs. 5th ed., Churchill Livingstone, 2009. 45. Dynamic contrast-enhanced CT may be used to characterize adrenal lesions. Which one of the following statements best describes the imaging characteristics of a primary adrenal carcinoma on portal venous phase (70 s) and subsequent delayed phase (15 min) contrast-enhanced CT images? A. Early washout on delayed images B. No measurable enhancement in either phase C. Poor early enhancement, with an increase in enhancement on delayed images D. Washout by greater than 80%, compared with the early postcontrast E. Washout of less than 40% on delayed images, compared with the portal venous phase images Source: McQueen, Andrew S., et al. Grainger & Allison’s Diagnostic Radiology: Single Best Answer MCQs. 5th ed., Churchill Livingstone, 2009. Explanation: • Malignant lesions have abnormally high vascular density leading to slower flow and increased microvascular permeability. This translates to longer transit times for intravenous contrast within malignant adrenal lesions, compared with simple adenomas.
  • 85.
    EBDR Exam MCQs& Concepts October 2024 Dr. Kareem Alnakeeb 79 46. A 19-year-old female student presents with acute abdominal pain, elevated CRP and a low-grade temperature. On clinical examination, there is tenderness to light palpation in the right iliac fossa and the patient is febrile. A graded compression ultrasound examination is performed. Which one of the following statements is true? A. A transverse appendiceal diameter of 5 mm is diagnostic of acute appendicitis. B. The finding of a pelvic fluid collection makes a diagnosis of acute appendicitis unlikely. C. The presence of hyperechoic fat in the right iliac fossa makes a diagnosis of acute appendicitis unlikely. D. The sensitivity of graded compression ultrasound in suspected acute appendicitis is 75—90%. E. The specificity of graded compression ultrasound in suspected acute appendicitis is 35—50%. Source: McQueen, Andrew S., et al. Grainger & Allison’s Diagnostic Radiology: Single Best Answer MCQs. 5th ed., Churchill Livingstone, 2009. Explanation: • Graded compression ultrasound of the appendix can avoid unnecessary surgery and ionizing radiation—particularly relevant for children and women of childbearing age. • The finding of a noncompressible appendix with transverse diameter of 6 mm or greater is highly suggestive of acute appendicitis (specificity 86-100%). • Other ultrasound findings include hyperechoic fat in the right iliac fossa, periappendiceal fluid or a pelvic fluid collection (appendiceal abscess). 47. A 79-year-old woman trips and falls whilst stepping off a bus. She suffers from a fractured right neck of femur and undergoes a hemiarthroplasty the following day. Her early recovery is complicated by bronchopneumonia which resolves after 5 days of broad-spectrum antibiotics. On her tenth day in hospital she develops abdominal pain and diarrhea and pseudomembranous colitis is suspected clinically. Which one of the following statements is true regarding pseudomembranous colitis? A. A normal abdominal CT effectively excludes pseudomembranous colitis. B. Ascites is present in up to 40% of patients. C. CT carries a low positive predictive value for pseudomembranous colitis. D. Extensive pericolonic stranding is a typical feature on CT. E. The rectum is not involved in 40—50% of patients. Source: McQueen, Andrew S., et al. Grainger & Allison’s Diagnostic Radiology: Single Best Answer MCQs. 5th ed., Churchill Livingstone, 2009. Explanation: • Ascites can occur with other colitides, but is often seen in pseudomembranous colitis. • CT typically demonstrates mucosal enhancement and marked colonic wall thickening but only mild pericolonic stranding, in patients with pseudomembranous colitis. These findings have a high positive predictive value but a normal CT does not exclude pseudomembranous colitis. • Rectal sparing occurs in around 10% of patients.
  • 86.
    EBDR Exam MCQs& Concepts October 2024 Dr. Kareem Alnakeeb 80 48. A 22-year-old pregnant woman (30 weeks' gestation) presents with right flank pain. She has an abdominal ultrasound which shows dilatation of the right pelvicalyceal system. Which one of the following additional findings would suggest a diagnosis of mechanical ureteric obstruction rather than pregnancy- related dilatation? A. An elevated resistive index (RI) B. Decreased corticomedullary differentiation C. Hyperechoic renal parenchyma D. Renal parenchymal thinning E. Ureteric and pelvicalyceai dilatation Source: McQueen, Andrew S., et al. Grainger & Allison’s Diagnostic Radiology: Single Best Answer MCQs. 5th ed., Churchill Livingstone, 2009. Explanation: • Mechanical obstruction is associated with elevation of the RI. 49. A 40-year-old woman has a 15-year history of ulcerative colitis (UC). After the initial diagnosis, she suffered frequent exacerbations of colitis requiring several hospital admissions. She declined surgical intervention at that stage and has subsequently been well controlled on medical management. Recently, she has developed a change in bowel habit and a double contrast barium enema is performed. This shows a stricture in the descending colon. Which one statement is true regarding strictures in ulcerative colitis? A. Abrupt shouldering is typical of a benign stricture in UC. B. In patients with UC, colorectal carcinomas typically arise from tubular adenomas. C. The majority of strictures in UC are benign. D. There is no increased risk of colorectal carcinoma in patients with UC. E. Widening of the presacral space is pathognomonic of a rectal carcinoma. Source: McQueen, Andrew S., et al. Grainger & Allison’s Diagnostic Radiology: Single Best Answer MCQs. 5th ed., Churchill Livingstone, 2009. Explanation: • Benign strictures in UC are typically smooth and symmetrical and are due to chronic smooth muscle hypertrophy. These occur in 10—20% of patients with UC and are most common in the left colon. • Carcinomas arise from dysplastic changes within the diseased epithelium and not from adenomas as in the general population. 156. The following statements concerning esophageal carcinoma are true: A. 90% of cases are squamous cell carcinomas B. Most commonly located in the upper third of the esophagus C. Plummer-Vinson syndrome is a recognised predisposing factor D. Commonest appearance on double contrast barium swallow is of a large ulcer within a bulging mass E. It is associated with ulcerative colitis Source: Hussain, Shahid, et al. Radiology MCQs for the New FRCR Part 2A. 1st ed., TFM Publishing, 2006. Explanation: A. True B. False - 20% in the upper third, 30-40% middle third and 30-40% in lower third C. True D. False - polypoid/fungating form is commonest E. False - predisposing factors include Barrett’s oesophagus, alcohol abuse, smoking, coeliac disease, achalasia, tylosis
  • 87.
    EBDR Exam MCQs& Concepts October 2024 Dr. Kareem Alnakeeb 81 50. Regarding diverticular disease, which is False? A. Colonic diverticulosis affects 70-80% by 80 years of age B. 10-25% of individuals with colonic diverticular disease develop diverticulitis C. Rectosigmoid colon is most commonly affected D. Fistula formation occurs in 40-50% of cases complicating acute diverticulitis E. Moderate diverticulitis is present when the bowel wall is thickened >3 mm Source: Hussain, Shahid, et al. Radiology MCQs for the New FRCR Part 2A. 1st ed., TFM Publishing, 2006. Explanation: A. True B. True C. True D. False - 14% E. True 51. Which one of the following statements best describes the CT appearances of a renal oncocytoma (tubular adenoma)? A. It appears as a small, ill-defined renal mass in the majority of cases. B. It is bilateral in 60-80% of cases. C. It characteristically consists of multiple renal lesions. D. CT shows punctuate calcification in the majority of patients. E. Low attenuation (—100 to —50 HU) areas within a large lesion are consistent with an oncocytoma. Source: McQueen, Andrew S., et al. Grainger & Allison’s Diagnostic Radiology: Single Best Answer MCQs. 5th ed., Churchill Livingstone, 2009. Explanation: • Large lesions can extend into and engulf the perinephric fat, and can therefore be mistaken for angiomyolipomas (due to fat content). 52. An 83-year-old man undergoes an emergency left hip hemiarthroplasty following a fractured neck of femur. Six days after surgery, he develops increasing abdominal distension with nausea and vomiting. An abdominal radiograph is performed and demonstrates dilatation of the ascending and transverse colon with the caecum measuring 7.0 cm in diameter. The clinical team believe that the patient may have colonic pseudo-obstruction and a single contrast (instant) enema is performed using water-soluble contrast. What are the likely findings in colonic pseudo-obstruction? A. Extrinsic compression of the sigmoid colon B. Long, irregular stricture of the sigmoid colon C. Long, smooth stricture at the splenic flexure D. No stricture demonstrated E. Short ‘apple core’ stricture of the descending colon Source: McQueen, Andrew S., et al. Grainger & Allison’s Diagnostic Radiology: Single Best Answer MCQs. 5th ed., Churchill Livingstone, 2009. Explanation: • An instant enema can exclude mechanical obstruction in patients with colonic pseudo-obstruction.
  • 88.
    EBDR Exam MCQs& Concepts October 2024 Dr. Kareem Alnakeeb 82 53. A 54-year-old man has an abdominal ultrasound that shows a 3-cm hyperechoic lesion at the upper pole of the left kidney. An unenhanced CT abdomen is subsequently performed and demonstrates a left upper pole heterogeneous renal mass with central areas of low attenuation (5—10 HU). After intravenous contrast is administered, this mass enhances by more than 30 HU. What is the most likely diagnosis? A. Angiomyolipoma B. Oncocytoma C. Renal abscess D. Renal cell carcinoma E. Unilocular renal cyst Source: McQueen, Andrew S., et al. Grainger & Allison’s Diagnostic Radiology: Single Best Answer MCQs. 5th ed., Churchill Livingstone, 2009. Explanation: • Angiomyolipomas are benign, fat-containing lesions which do not enhance by more than 15 HU and contain low attenuation (—15 to —20HU) fatty areas. • Postcontrast enhancement of greater than 20 HU of a solid renal mass is highly suggestive of malignancy. 54. A 64-year-old man presents with right renal colic and a kidney ureter bladder plain radiograph (CT KUB) is performed. This demonstrates an incidental 2-cm solid right adrenal mass. On the unenhanced CT, the mass is homogeneous and has an average density of 7 HU. What is the most likely diagnosis? A. Adrenal adenoma B. Adrenal hyperplasia C. Adrenal metastasis D. Focal adrenal haemorrhage E. Primary adrenal malignancy Source: McQueen, Andrew S., et al. Grainger & Allison’s Diagnostic Radiology: Single Best Answer MCQs. 5th ed., Churchill Livingstone, 2009. Explanation: • The 10-HU threshold is now the standard by which radiologists differentiate lipid-rich adenomas from most other adrenal lesions on unenhanced CT. • The presence of substantial amounts of intracellular fat is critical in malting the specific diagnosis of adenoma. • Up to 30% of adenomas, however, do not have abundant intracellular fat and, thus, show attenuation values greater than 10 HU on unenhanced CT. • Lesions above 10 HU on an unenhanced CT are considered indeterminate and other investigations may be required. 55. A 34-year-old man is knocked off his bicycle by a car and presents to the Emergency Department with bruising over the right flank and gross hematuria. The A&E SpR suspects renal injury and requests a CT abdomen. Which one of the following findings is most likely to be seen in uncomplicated renal contusion (Grade 1 renal injury)? A. Ill-defined areas of low attenuation with irregular margins B. Subcapsular high attenuation collection C. Wedge-shaped areas of high attenuation, typically involving the renal D. Well-defined areas of low attenuation within the renal parenchyma E. Urinoma formation Source: McQueen, Andrew S., et al. Grainger & Allison’s Diagnostic Radiology: Single Best Answer MCQs. 5th ed., Churchill Livingstone, 2009.
  • 89.
    EBDR Exam MCQs& Concepts October 2024 Dr. Kareem Alnakeeb 83 56. A 24-year-old motorcyclist is involved in a high-speed accident and is brought to the Emergency Department. He has abdominal guarding and is hemodynamically unstable. An ultrasound abdomen performed in the Emergency Department demonstrates free peritoneal fluid and a laparotomy is performed. In addition to liver and splenic lacerations, the surgeon finds a left retroperitoneal hematoma. Postoperatively, the on-call urologist requests a CT abdomen to assess the left renal injury. Which one of the following findings would indicate a Grade 4 renal laceration? F. Extravasation of contrast from the pelvicalyceal system on delayed phase (5 min) images G. Large (2-cm) subcapsular hematoma H. Perinephric hematoma that extends into the pararenal spaces I. Ill-defined low attenuation change in the lower pole renal cortex J. Segmental renal infarction Source: McQueen, Andrew S., et al. Grainger & Allison’s Diagnostic Radiology: Single Best Answer MCQs. 5th ed., Churchill Livingstone, 2009. Explanation: • A deep renal laceration that extends into the collecting system is indicative of a grade 4 injury. 57. Which one of the following statements best describes the radiological appearances of a parapelvic renal cyst? A. It does not opacify during IVU. B. If hydronephrosis is present, a parapelvic cyst can be excluded. C. It shows delayed (10 min) filling on IVU. D. It may have similar appearances to calyceal diverticula on IVU. E. The majority arise from the lower renal pole. Source: McQueen, Andrew S., et al. Grainger & Allison’s Diagnostic Radiology: Single Best Answer MCQs. 5th ed., Churchill Livingstone, 2009. Explanation: • A parapelvic cyst is located near the renal hilum, does not communicate with the renal pelvis (unlike calyceal diverticula) and therefore does not opacify during IVU. • It compresses the pelvis and may cause hydronephrosis. 58. A 23-year-old man presents with a 2-day history of vomiting and generalized abdominal pain. Two years ago, he underwent a small bowel resection for an ileal stricture due to Crohn's disease. Initial blood tests reveal a raised CRP and white cell count and an abdominal radiograph demonstrates dilated loops of small bowel. Small bowel obstruction is suspected and a contrast-enhanced CT of the abdomen is performed. Which one of the following statements is true regarding the role of multidetector CT in small bowel obstruction? A. Five to 15% of small bowel obstructions are due to hernias. B. Twenty to 30% of small bowel obstructions arc due to adhesions. C. Bowel wall thickening and intramural gas indicate the presence of pneumatosis coli. D. Closed loop obstruction is less likely to result in bowel ischemia than simple obstruction. E. In small bowel obstruction due to adhesions, a transition point will not be seen. Source: McQueen, Andrew S., et al. Grainger & Allison’s Diagnostic Radiology: Single Best Answer MCQs. 5th ed., Churchill Livingstone, 2009. Explanation: • Bowel wall thickening, lack of enhancement, adjacent fluid and pneumatosis intestinalis are all CT signs of ischemia (strangulation) in small bowel obstruction. • Fifty to 80% of small bowel obstruction is attributable to adhesions while 10% is due to hernias. • In adhesions, there will usually be a history of previous abdominal surgery with CT demonstrating small bowel obstruction. • The transition point may be identified, but the actual adhesive land is usually not visualized.
  • 90.
    EBDR Exam MCQs& Concepts October 2024 Dr. Kareem Alnakeeb 84 59. A 78-year-old man has myelodysplastic syndrome and requires frequent blood transfusions. He develops progressively abnormal liver function tests and a grossly elevated ferritin level. An MRI of the liver is performed using breath hold half Fourier single shot spin echo T2w images. Which finding would make a diagnosis of hemosiderosis (iron overload from recurrent blood transfusion) more likely than haemochromatosis? F. Increased T2 signal in the liver only G. Increased T2 signal in the liver and spleen H. Reduced T2 signal in the liver only I. Reduced T2 signal in the liver and spleen J. Reduced T2 signal in the spleen only Source: McQueen, Andrew S., et al. Grainger & Allison’s Diagnostic Radiology: Single Best Answer MCQs. 5th ed., Churchill Livingstone, 2009. Explanation: • In iron overload due to recurrent transfusions, there is increased iron deposition in the reticuloendothelial system. This leads to reduced Tl, T2 and T2* signal intensity in the liver and spleen. • Haemochromatosis causes diffusely reduced T2 signal in the liver and may lead to cirrhosis, but the splenic signal intensity should remain normal. • Diffuse fatty liver will lead to increased T2 signal in the liver with signal loss during out-of-phase images. 60. Which of the following is not an angiographic sign of active bleeding? A. Contrast extravasation B. Vessel spasm C. Vessel cut-off D. Early venous filling E. Vessel dilatation Source: Chin, Teck Yew, et al. Get Through Final FRCR 2A: SBAs. 1st ed., CRC Press, 2017. Explanation: • The rest are angiographic signs of active bleeding; vessel dilatation is not. 61. A 52-year-old postmenopausal woman presents for her first screening mammogram. Within the right upper outer quadrant, there is a 2-cm well-defined, oval mass that has dense ‘popcorn’ calcification within it and is surrounded by a thin radiolucent rim. On ultrasound, the mass is well defined and hyperechoic with areas of acoustic shadowing due to contained calcification. What is the most likely diagnosis? A. Fat necrosis B. Fibroadenoma C. Hamartoma D. Oil cyst E. Papilloma Source: McQueen, Andrew S., et al. Grainger & Allison’s Diagnostic Radiology: Single Best Answer MCQs. 5th ed., Churchill Livingstone, 2009. Explanation: • Fibroadenomas may become calcified, particularly after menopause. Classically the calcifications have a coarse ‘popcorn’ appearance; however, they may also appear small and punctate. • An oil cyst typically demonstrates eggshell calcification and is the result of fat necrosis.
  • 91.
    EBDR Exam MCQs& Concepts October 2024 Dr. Kareem Alnakeeb 85 62. A 15-month-old boy is referred from his GP with a limp. Which radiological finding would be consistent with DDH? A. Accelerated epiphyseal ossification B. Decreased distance between the medial portion of the proximal femoral metaphysis and the pelvis C. Increased acetabular angle D. Inferolateral displacement of the femoral head in relation to Perkin's line E. Preservation of Shenton's line Source: McQueen, Andrew S., et al. Grainger & Allison’s Diagnostic Radiology: Single Best Answer MCQs. 5th ed., Churchill Livingstone, 2009. 63. An 18-year-old man experiences persistent symptoms following a fracture through the waist of the right scaphoid. Radiographs of the right scaphoid indicate non-union. An MRI is performed to assess the vascularity of the proximal pole. Which imaging features are consistent with a diagnosis of avascular necrosis? F. Bone marrow enhancement following administration of gadolinium G. High signal surrounding the fracture on T2w images H. High signal within the proximal pole on T1w images I. High signal within the proximal pole on STIR images J. Low signal within the proximal pole on T1w images Source: McQueen, Andrew S., et al. Grainger & Allison’s Diagnostic Radiology: Single Best Answer MCQs. 5th ed., Churchill Livingstone, 2009. Explanation: • Low signal on T1 reflects death of the adipocytes. The combination of low signal on T1w images and low or intermediate signal on T2w images accurately predicts avascular necrosis. 64. A 68-year-old man presents to his GP with a 1-month history of epigastric pain, vomiting and mild weight loss. Examination is unremarkable and the patient is referred for an upper gastrointestinal endoscopy. This demonstrates mild gastritis with biopsies positive for Helicobacter pylori and he is commenced on eradication therapy. Three months later, the symptoms have persisted and the patient has lost 5 kg in weight. A double contrast barium meal is performed and reveals a shallow ulcer on the lesser curve of the stomach. Which additional finding would make the ulcer more likely to be benign than malignant? A. Hampton's line is present. B. Nodular mucosal folds stop at the edge of the lesion. C. The ulcer does not extend beyond the gastric wall. D. The ulcer has an irregular margin. E. The ulcer measures 40 mm in size. Source: McQueen, Andrew S., et al. Grainger & Allison’s Diagnostic Radiology: Single Best Answer MCQs. 5th ed., Churchill Livingstone, 2009. Explanation: • Hampton’s line refers to a lucent line crossing the ulcer base: its presence is highly suggestive of a benign ulcer.
  • 92.
    EBDR Exam MCQs& Concepts October 2024 Dr. Kareem Alnakeeb 86 65. A 53-year-old woman is seen in the general surgical outpatient clinic. She attended her GP with a 1- month history of upper abdominal pain and was found to have a palpable, firm mass in the epigastrium. An upper gastrointestinal (GI) endoscopy is normal and the surgical team request a contrast-enhanced CT of the abdomen. This demonstrates a multicystic mass in the pancreas. Which findings would make a mucinous cystic tumor more likely than a serous cystadenoma? A. Central stellate calcification is present within the lesion. B. The mass contains 12 separate cysts. C. The smallest cystic component measures 28 mm in diameter. D. The patient has a known diagnosis of von Hippel-Lindau disease. E. The tumor is located in the head of the pancreas. Source: McQueen, Andrew S., et al. Grainger & Allison’s Diagnostic Radiology: Single Best Answer MCQs. 5th ed., Churchill Livingstone, 2009. Explanation: • Mucinous cystic pancreatic tumors (cystadenomas and cystadenocarcinomas) typically contain a few large cysts, each measuring more than 20 mm diameter. 66. A 41-year-old man has a 3-month history of weight loss and recurrent central abdominal pain. The pain is intermittent and radiates from the epigastrium through to his back. His past medical history includes excessive alcohol consumption and two previous admissions to hospital for acute pancreatitis. A contrast-enhanced CT of the abdomen is performed with pre-contrast, arterial and portal venous phase images of the upper abdomen. Which CT finding would be more suggestive of chronic pancreatitis than ductal pancreatic adenocarcinoma? A. Common bile duct dilatation B. Focal enlargement of the pancreatic head C. Intraductal pancreatic calcification D. Peripancreatic fat stranding and ascites E. Reduced enhancement of body of pancreas Source: McQueen, Andrew S., et al. Grainger & Allison’s Diagnostic Radiology: Single Best Answer MCQs. 5th ed., Churchill Livingstone, 2009. Explanation: • Intraductal calcification may be focal or diffuse and is not seen in all patients with chronic pancreatitis. When it is present, however, it is a highly reliable sign of chronic pancreatitis. 67. A 75-year-old diabetic man underwent a left below knee amputation 3 months ago for osteomyelitis of the distal tibia. Since then, he has experienced recurrent episodes of fever and malaise. MRI is contraindicated due to a metallic aortic valve. Which is the best investigation to exclude an occult focus of osteomyelitis? A. CT B. US C. Scintigraphy using gallium D. Scintigraphy using indium-labelled white cells E. Scintigraphy using technetium (Tc-99m) monodiphosphonate Source: McQueen, Andrew S., et al. Grainger & Allison’s Diagnostic Radiology: Single Best Answer MCQs. 5th ed., Churchill Livingstone, 2009. Explanation: • Although an indium-labelled white cell study is more specific, a bone scintigram using Tc-99m monodiphosphonate is a more sensitive test to exclude osteomyelitis.
  • 93.
    EBDR Exam MCQs& Concepts October 2024 Dr. Kareem Alnakeeb 87 68. A 6-year-old with spina bifida has a chest X-ray performed for possible lower respiratory tract infection. The lungs are clear but there is a well-defined, round paraspinal mass with an air–fluid level. What is the most likely diagnosis? A. Bronchogenic cyst B. Morgagni hernia C. Esophageal duplication cyst D. Cystic teratoma E. Esophageal tumor Source: Chin, Teck Yew, et al. Get Through Final FRCR 2A: SBAs. 1st ed., CRC Press, 2017. Explanation: Esophageal Duplication Cysts • Esophageal duplication cysts are rare congenital anomalies. • They are associated with vertebral anomalies (spina bifida, hemivertebrae, fusion defects). • There is also an association with esophageal atresia and small bowel duplication. • Most cysts develop in the right posteroinferior mediastinum. • CT demonstrates a well-marginated round, oval or tubular-shaped fluid-filled cystic structure that has a well-defined, thin wall. The cyst is of water attenuation with no enhancement of contents and no infiltration of surrounding structures. • Malignant degeneration is rare. Other Mediastinal Lesions • Bronchogenic cyst is the most common cystic mediastinal mass that typically lies in the middle mediastinum, not in a paraspinal location; in addition, you would not expect an air-fluid level. • Cystic teratoma is an anterior mediastinal mass. • Morgagni hernia would be unlikely to cause a solitary round lesion; multiple structures would be expected. 69. A 38-year-old woman undergoes pelvic MRI for chronic pelvic pain. Axial and coronal T2-weighted images show a tortuous, C-shaped tubular structure in the left adnexa that is uniformly hyperintense on T2 and hypointense on T1. The wall is thin and smooth; there is no mural nodule or enhancing solid tissue on post-contrast sequences. According to the ACR O-RADS MRI risk stratification system, which score and malignancy risk category should be assigned to this lesion? Option O-RADS MRI score Positive predictive value (PPV) for malignancy A. 1 <0.1% B. 2 <0.5% C. 3 5 – 10% D. 4 20 – 50% E. 5 >80% Answer: B Source: Perplexity AI Explanation: • A dilated fallopian tube containing simple fluid with a thin, smooth wall and no enhancing solid tissue is classified as O-RADS MRI 2 (“almost certainly benign”), carrying a < 0.5% likelihood of malignancy. Recognizing this typical appearance of hydrosalpinx prevents over-classification and unnecessary intervention. • Key learning point: Hydrosalpinx with simple fluid and no suspicious mural elements is an O-RADS MRI 2 lesion; only the presence of thickened folds, non-simple fluid, or enhancing solid components would raise the score to 3 or higher.
  • 94.
    EBDR Exam MCQs& Concepts October 2024 Dr. Kareem Alnakeeb 88 70. A 54-year-old woman attends a well woman clinic and is found to have abnormal liver function tests. She is referred to the hepatology outpatient clinic and an abdominal ultrasound is performed. This demonstrates diffuse increased reflectivity of the liver parenchyma but no focal parenchymal abnormality. The hepatology team request an ultrasound-guided percutaneous liver biopsy. Which statement is true regarding this procedure? A. Ten to 20% of complications occur in the first 2 hours post procedure. B. Ascites is an absolute contraindication to percutaneous liver biopsy. C. Mortality rate is 1 in 500. D. Over 90% of complications occur in the first 24 hours post procedure. E. There is no increased risk of complications with malignant liver lesions. Source: McQueen, Andrew S., et al. Grainger & Allison’s Diagnostic Radiology: Single Best Answer MCQs. 5th ed., Churchill Livingstone, 2009. Explanation: • Following an ultrasound-guided liver biopsy, nearly two-thirds of complications occur in the first 2 hours, with 96% of complications having occurred by 24 hours. 71. A 67-year-old woman undergoes surgical resection of a distal sigmoid adenocarcinoma. The surgeon performs a primary anastomosis between the descending colon and rectum and leaves a defunctioning loop colostomy. Nine days later, the patient is experiencing fevers and low abdominal pain. A contrast- enhanced CT shows a small fluid collection around the anastomosis with no definite abscess identified. The surgical team are concerned about the integrity of the anastomosis. Which investigation would you choose to look for an anastomotic leak? A. Barium enema B. Barium follow-through C. MRI pelvis with intravenous gadolinium D. Water-soluble contrast cystogram E. Water-soluble contrast enema Source: McQueen, Andrew S., et al. Grainger & Allison’s Diagnostic Radiology: Single Best Answer MCQs. 5th ed., Churchill Livingstone, 2009. Explanation: • The combination of water-soluble contrast enema and CT is used to look for anastomotic leakage and abscess formation. 72. A 68-year-old man presents to his GP with weight loss and jaundice. Liver function tests demonstrate obstructive jaundice and an abdominal ultrasound shows mild intrahepatic biliary dilatation with a common bile duct measuring 12 mm in diameter. In the pancreatic head, a 3-cm hypoechoic mass is present. An ERCP is performed with insertion of a plastic stent and brushings confirm a pancreatic ductal adenocarcinoma. A triple-phase (pre-contrast, arterial and portal venous) multidetector CT of the pancreas is performed. Which finding would indicate a nonresectable pancreatic tumor? A. Enhancing pancreatic parenchyma between the tumor and superior B. The pancreatic duct dilated to 6 mm C. The presence of a 5-mm coeliac axis lymph node D. The tumor has invaded the duodenum E. The tumor in contact with 75% of the superior mesenteric artery Source: McQueen, Andrew S., et al. Grainger & Allison’s Diagnostic Radiology: Single Best Answer MCQs. 5th ed., Churchill Livingstone, 2009. Explanation: • If the tumour is in contact with more than half of the vessel circumference, it is very unlikely to be resectable.
  • 95.
    EBDR Exam MCQs& Concepts October 2024 Dr. Kareem Alnakeeb 89 73. What level of serum glucose is generally considered acceptable when performing an FDG-PET scan? A. Less than 50 mg/dL. B. Less than 100 mg/dL. C. Less than 200 mg/dL. D. Less than 300 mg/dL. Source: Unknown Explanation: (by Perplexity AI, generated with the OpenAI O3 model) • Most professional guidelines (EANM, SNMMI, ACR) advise proceeding with an oncologic FDG-PET/CT if the fasting plasma glucose is below about 11 mmol/L (≈ 200 mg/dL); at higher levels competitive inhibition by circulating glucose can lower tumor uptake and degrade image quality. • Values under 100 mg/dL are ideal but not mandatory, whereas levels > 200 mg/dL usually prompt rescheduling or glucose-lowering measures. 74. A 56-year-old patient presents with classic carcinoid syndrome and is found to have multiple liver metastases on imaging. What is the most likely site of the primary tumor? A. Stomach B. Duodenum C. Small Bowel D. Appendix. E. Rectum Source: Unknown Explanation: • Over 40% of carcinoid tumors originate in the small intestine; the rectum (~27%), appendix (~24%) and stomach (~8%) are the next most frequent primary sites. • Duodenal carcinoid tumors are rare. • Small-intestinal carcinoid tumors frequently symptomatic. 75. A 35-year-old patient received a cadaveric renal transplant 5 days ago and now presents with worsening renal function and decreasing urine output. Which one of the following findings on a Tc-99m DTPA radionuclide scan would favor a diagnosis of acute tubular necrosis (ATN) over acute rejection? F. Delayed renal excretion G. Elevated resistive index greater than 0.7 H. Increased renal perfusion after administration of an ACEI (eg Captopril) I. Poor/impaired graft perfusion J. Preserved renal transplant perfusion Source: McQueen, Andrew S., et al. Grainger & Allison’s Diagnostic Radiology: Single Best Answer MCQs. 5th ed., Churchill Livingstone, 2009. Explanation: • ATN is an early complication in cadaveric allografts and frequently resolves spontaneously in 1—3 weeks. The radionuclide imaging findings of ATN are of preserved perfusion but poor renal function and urine excretion. • In acute rejection however, there is both impaired renal function and reduced perfusion on radionuclide imaging.
  • 96.
    EBDR Exam MCQs& Concepts October 2024 Dr. Kareem Alnakeeb 90 76. A 25-year-old doctor injures her left wrist whilst snowboarding. Initial radiographs are reported as showing no fracture, but there is clinical suspicion of a scapholunate ligament disruption. Further views are obtained. Which radiological feature would support the diagnosis? A. Scapholunate angle less than 30° B. Scapholunate distance of 2 mm C. ‘Signet ring’appearance of the scaphoid D. Rotatory subluxation of the lunate E. Wedge-shaped appearance of the lunate Source: McQueen, Andrew S., et al. Grainger & Allison’s Diagnostic Radiology: Single Best Answer MCQs. 5th ed., Churchill Livingstone, 2009. Explanation: • The ‘signet ring’appearance is clue to rotatory subluxation of the scaphoid as a result of disruption of the scapholunate ligament. 77. An athletic 19-year-old medical student presents to the Emergency Department after sustaining an injury to his right hip during training. A radiograph reveals a fracture of the anterior superior iliac spine. What is the most likely diagnosis? A. Avulsion of the adductor muscles B. Avulsion of the hamstring muscles C. Avulsion of iliopsoas D. Avulsion of rectus femoris E. Avulsion of sartorius Source: McQueen, Andrew S., et al. Grainger & Allison’s Diagnostic Radiology: Single Best Answer MCQs. 5th ed., Churchill Livingstone, 2009. Explanation: • Avulsion injuries occur at characteristic sites and are particularly common in children and adolescents. 78. A 29-year-old man has an IVU performed following an episode of haematuria. This demonstrates complete right-sided ureteric duplication. Which one of the following statements is true? F. If present, an ectopic ureterocoele is usually related to the lower moiety G. The lower moiety ureter usually obstructs at the vesicoureteric junction. H. The upper moiety calyces are prone to vesicoureteric reflux. I. The upper moiety ureter is prone to ureteric obstruction. J. The upper moiety ureter usually inserts into the bladder superior to the lower moiety ureter. Source: McQueen, Andrew S., et al. Grainger & Allison’s Diagnostic Radiology: Single Best Answer MCQs. 5th ed., Churchill Livingstone, 2009. 79. A low flat renogram curve indicates: E. Advanced nephropathy F. Complete obstruction to urine outflow G. Vesico-ureteric reflux H. Partial obstruction to urine outflow Explanation: (by Perplexity AI, generated with the OpenAI O3 model) • A very low, almost horizontal time–activity curve reflects markedly reduced tracer extraction by damaged renal parenchyma, typical of end-stage or advanced medical renal disease. • Complete or partial outflow obstruction instead produces an uptake phase followed by a rising or plateau phase; vesico-ureteric reflux alters the post-void segment rather than the primary renographic curve
  • 97.
    EBDR Exam MCQs& Concepts October 2024 Dr. Kareem Alnakeeb 91 80. Regarding gastric emptying scintigraphy A. T1/2 is the time at which gastric counts falls to its half B. Tc 99m tin colloid labeled RBCs are injected IV C. If 20 % of gastric counts remain after 1 hour, delayed gastric emptying is considered. D. It cannot detect changes in gastric emptying rate in post operative cases.
  • 98.
    EBDR Exam MCQs& Concepts July 2024 Dr. Kareem Alnakeeb 92 July 2024 Paper 1 1. A 45-year-old patient presents with progressive visual field defects. MRI demonstrates a well- circumscribed extra-axial mass with dural attachment showing avid homogeneous enhancement and a dural tail sign. What are the typical T1 and T2 signal characteristics of this lesion? A. High T1, high T2 signal B. Low T1, high T2 signal C. Isointense T1, isointense to mildly hyperintense T2 signal D. High T1, low T2 signal E. Low T1, low T2 signal Explanation: (by Perplexity AI, generated with the OpenAI O3 model) • The clinical and imaging features describe a typical meningioma. Meningiomas characteristically demonstrate isointensity to slight hypointensity relative to grey matter on T1-weighted sequences and isointensity to mild hyperintensity on T2-weighted imaging. • The microcystic subtype (1.6% of cases) is an exception, showing low T1 and high T2 signal. • Options A and D suggesting high T1 signal would be more consistent with melanotic lesions like uveal melanoma. • Option B describes typical glioma characteristics, while option E would suggest a calcified or fibrotic lesion. 2. In bilateral Wilms tumour (stage 5) the primary treatment goal after initial neoadjuvant chemotherapy is to A. perform bilateral radical nephrectomies to maximise oncological clearance B. undertake unilateral radical nephrectomy and contralateral nephron-sparing surgery C. achieve lung-directed radiotherapy before any renal surgery D. proceed immediately to renal transplantation once both kidneys are removed E. delay all surgery until post-chemotherapy radiation has completed Explanation: (by Perplexity AI, generated with the OpenAI O3 model) • Neoadjuvant chemotherapy is given first to shrink synchronous tumours in both kidneys. Surgery then aims to remove the more involved kidney completely while preserving as much functioning parenchyma as possible in the contralateral kidney, typically via nephron-sparing surgery. This balances oncological control with long-term renal function. • Bilateral radical nephrectomies (A) sacrifice all native renal tissue and would leave a child dialysis- dependent; transplantation (D) is only considered later if renal failure ensues. • Up-front lung radiotherapy (C) is reserved for metastatic pulmonary disease, not as the initial step in stage 5. • Delaying all surgery until after radiotherapy (E) risks local progression and is not standard practice. 3. According to the Children’s Oncology Group (NWTS/COG) staging system, synchronous tumours in both kidneys at presentation classify Wilms tumour as what stage? A. Stage I B. Stage II C. Stage III D. Stage IV E. Stage V
  • 99.
    EBDR Exam MCQs& Concepts July 2024 Dr. Kareem Alnakeeb 93 Explanation: (by Perplexity AI, generated with the OpenAI O3 model) • Bilateral renal involvement is automatically designated stage V in the NWTS/COG scheme, irrespective of tumour size, resectability or nodal status. • Stages I–II apply only to unilateral disease that can be completely resected, stage III to residual intra- abdominal tumour or nodal spread after surgery, and stage IV to distant metastases such as lung or liver. • Recognising stage V is crucial because management focuses on nephron-sparing chemotherapy followed by staged or partial nephrectomy to preserve renal function, rather than immediate radical nephrectomy used in lower stages. 4. A 3-day-old infant with inspiratory stridor undergoes contrast-enhanced CT, which shows the left pulmonary artery arising from the right pulmonary artery and passing between the trachea and oesophagus to reach the left hilum (pulmonary artery sling); which associated abnormality is most frequently present? A. Long-segment tracheal stenosis with complete tracheal rings B. Double aortic arch C. Patent foramen ovale D. Aberrant right subclavian artery E. Pulmonary sequestration Explanation: (by Perplexity AI, generated with the OpenAI O3 model) • The aberrant left pulmonary artery (pulmonary sling) frequently compresses and deforms the airway, and over half of patients have long-segment tracheal stenosis formed by complete cartilaginous rings, making option A correct. • Double aortic arch and aberrant right subclavian artery are vascular rings that can coexist but are far less common associations. • Patent foramen ovale is a frequent incidental cardiac finding but has no specific link to pulmonary sling. • Pulmonary sequestration affects lung parenchyma rather than the central airway and is not characteristically related to this vascular anomaly. 5. In a patient with post-ductal coarctation of the aorta who also has an aberrant right subclavian artery arising distal to the coarctation, which chest-radiograph pattern of rib notching is classically expected? A. Bilateral rib notching of ribs 3–8 B. Unilateral right-sided rib notching of ribs 3–8 C. Unilateral left-sided rib notching of ribs 3–8 D. Rib notching limited to the 1st and 2nd ribs E. No rib notching is seen at any level Explanation: (by Perplexity AI, generated with the OpenAI O3 model) • Inferior rib notching results from dilated intercostal collaterals carrying blood from a subclavian artery that is proximal to the coarctation down to the descending aorta. • When the right subclavian artery is aberrant and originates distal to the narrowed segment, only the left subclavian (which is proximal) can supply collateral flow; therefore, enlarged intercostal arteries – and rib notching – develop on the left side only, typically affecting ribs 3–8. • Right-sided notching (option B) occurs when the coarctation lies proximal to the left subclavian so the right subclavian is the collateral source. • Bilateral notching (option A) requires both subclavian arteries to originate proximal to the obstruction. • The 1st-2nd ribs are spared because their intercostal arteries do not participate in the collateral circuit, so option D is incorrect.
  • 100.
    EBDR Exam MCQs& Concepts July 2024 Dr. Kareem Alnakeeb 94 • Absence of notching (option E) is inconsistent with long-standing adult coarctation. 6. In paediatric non-WNT/SHH medulloblastoma, which post-gadolinium MRI appearance is associated with the poorest overall and event-free survival? A. No measurable enhancement B. Heterogeneous enhancement in 10–75% of tumour volume C. Extensive enhancement involving >75% of tumour volume D. Ring enhancement with central necrosis E. Peripheral nodular enhancement surrounding internal cysts Explanation: (by Perplexity AI, generated with the OpenAI O3 model) • Extensive, nearly whole-tumour gadolinium uptake signifies a highly vascular, biologically aggressive lesion in non-WNT/SHH medulloblastoma. • A series of 76 patients showed that enhancement of >75% of tumour volume independently predicted significantly worse overall and event-free survival than either weak/none or heterogeneous enhancement patterns. • Minimal enhancement (option A) and heterogeneous partial enhancement (option B) were linked to better outcomes, while ring (D) and peripheral nodular (E) patterns are described in other posterior- fossa tumours but are not established prognostic markers in medulloblastoma. 7. On MRI, a 25-year-old patient has a unilocular, T2-hyperintense cystic lesion centred in the sublingual space that tracks around the posterior margin of the mylohyoid muscle into the submandibular space without internal enhancement. What is the most likely diagnosis? A. Plunging ranula B. Thyroglossal duct cyst C. First branchial cleft cyst D. Submandibular abscess E. Cystic hygroma Explanation: (by Perplexity AI, generated with the OpenAI O3 model) • Plunging ranula arises from the sublingual gland; when it dissects through or behind a dehiscence in the mylohyoid, it presents as a well-defined, thin-walled, unilocular cyst that is high signal on T2 and shows no enhancement, exactly as described. • Thyroglossal duct cysts lie in the midline or paramedian track between foramen cecum and thyroid cartilage and rarely extend lateral to the mylohyoid. • First branchial cleft cysts occur parotid–external auditory canal region, not the floor of mouth. • A submandibular abscess would show thick enhancing walls and diffusion restriction with clinical sepsis. • Cystic hygroma (lymphatic malformation) is characteristically multiloculated and infiltrative, often in the posterior triangle, not a solitary unilocular cyst. 8. In the standard cervical lymph-node classification used in head-and-neck imaging, the jugulodigastric (subdigastric) lymph nodes belong to which nodal level? A. Level Ib B. Level IIa C. Level IIb D. Level III E. Level IV Explanation: (by Perplexity AI, generated with the OpenAI O3 model) • The jugulodigastric nodes sit just inferior to the posterior belly of the digastric muscle and immediately anterior (or inseparable from) the internal jugular vein. These anatomical boundaries place them in the upper internal jugular chain designated Level IIa.
  • 101.
    EBDR Exam MCQs& Concepts July 2024 Dr. Kareem Alnakeeb 95 • Level IIb nodes are also in the upper jugular region but lie posterior to a fat plane separating them from the vein and spinal accessory nerve. • Level Ib nodes occupy the submandibular triangle, while Levels III and IV track progressively caudal along the internal jugular vein from the hyoid to the clavicle. • Recognising the jugulodigastric group as Level IIa is essential because they are often the first echelon for oropharyngeal and tonsillar metastases and are the largest normal cervical nodes. 9. A 50-year-old intravenous drug user presents with severe mid-back pain, fever and rapidly progressive paraparesis; which imaging investigation is the most appropriate first-line test to confirm a suspected spinal epidural abscess? A. Contrast-enhanced MRI of the whole spine B. Non-contrast CT of the symptom-level spine C. Radionuclide white-cell scan D. Plain thoracic spine radiographs E. FDG PET/CT Explanation: (by Perplexity AI, generated with the OpenAI O3 model) • Spinal epidural abscess is a neurosurgical emergency; diagnosis must be secured quickly and the entire neuraxis screened for skip lesions. Gadolinium-enhanced MRI provides near-100% sensitivity and specificity, delineates the epidural collection, shows rim enhancement that distinguishes abscess from phlegmon, and accurately demonstrates cord or cauda compression—making it the gold-standard first-line study. • Non-contrast CT (B) and plain radiographs (D) lack soft-tissue contrast and miss early collections. • Radionuclide scans (C) and FDG PET/CT (E) detect infection metabolically but are slower, less specific for epidural compartment involvement and do not guide urgent surgical planning. 10. Regarding the classical CT appearance of pleural empyema, which radiological sign is most characteristic and helps distinguish it from a peripheral lung abscess? A. Air-fluid level that is centrally located B. Cavitary lesion with irregular thick walls C. Split pleura sign D. Crescentic pleural fluid layering dependently E. Tree-in-bud nodularity in the adjacent lung Explanation: (by Perplexity AI, generated with the OpenAI O3 model) • On contrast-enhanced CT, pleural empyema typically shows the split pleura sign—smooth, symmetrically thickened parietal and visceral pleura that enhance and appear separated by pleural fluid. This finding is highly sensitive and specific for empyema and is rarely seen with peripheral lung abscesses. • A lung abscess more often presents as a cavitary parenchymal lesion with irregular thick walls and a central air-fluid level (distractors A and B). • Free-flowing, crescentic effusions that layer with gravity suggest uncomplicated pleural effusion, not empyema (distractor D). • Tree-in-bud nodularity indicates endobronchial spread of infection rather than pleural disease (distractor E).
  • 102.
    EBDR Exam MCQs& Concepts July 2024 Dr. Kareem Alnakeeb 96 11. A 68-year-old man with heart failure with preserved ejection fraction undergoes cardiac MRI with gadolinium. The short-axis late-gadolinium images show difficulty nulling the myocardium and widespread heterogeneous patchy mid-myocardial and subendocardial enhancement of both ventricles. Which diagnosis best explains this enhancement pattern? A. Hypertrophic cardiomyopathy B. Cardiac sarcoidosis C. Dilated cardiomyopathy with mid-wall fibrosis D. Cardiac amyloidosis E. Previous transmural myocardial infarction Explanation: (by Perplexity AI, generated with the OpenAI O3 model) • Cardiac amyloidosis typically shows diffuse or patchy subendocardial/mid-myocardial late gadolinium enhancement with failure to null normal myocardium because amyloid infiltration shortens T1; this combination is highly suggestive of the disease. • Hypertrophic cardiomyopathy usually shows focal mid-wall or RV insertion-point enhancement; sarcoidosis produces focal patchy nodular enhancement often in the basal septum; dilated cardiomyopathy gives linear mid-septal enhancement; prior infarction shows contiguous subendocardial or transmural enhancement in a coronary territory—none match the diffuse patchy pattern with inversion-time “nulling” difficulty. 12. A 13-year-old boy presents with pleuritic chest pain and a tender palpable swelling over the right posterior rib cage. Chest radiograph shows a permeative rib lesion with a multilaminated “onion-skin” periosteal reaction and an associated soft-tissue mass. Which diagnosis is most likely? A. Langerhans cell histiocytosis B. Chronic osteomyelitis C. Chondroblastoma D. Ewing sarcoma E. Osteoid osteoma Explanation: (by Perplexity AI, generated with the OpenAI O3 model) • Rib lesions that combine a permeative pattern, a soft-tissue mass and a lamellated (“onion-skin”) periosteal reaction are classically seen in Ewing sarcoma, a malignant small-round-blue-cell tumour that typically affects children and adolescents. • Langerhans cell histiocytosis can produce laminated periosteal reaction but usually causes well- defined erosions or punched-out defects without a large soft-tissue mass. • Osteomyelitis may mimic aggressive periostitis yet commonly shows sequestra, cortical irregularity and systemic infection markers. • Chondroblastoma arises in epiphyses of long bones, not ribs, and seldom forms a sizeable soft- tissue component. • Osteoid osteoma is a benign cortical lesion producing a small nidus with solid rather than multilayered periosteal reaction and intense nocturnal pain relieved by NSAIDs.
  • 103.
    EBDR Exam MCQs& Concepts July 2024 Dr. Kareem Alnakeeb 97 13. In a patient who has undergone canal-wall-up mastoidectomy, which MRI sequence is most reliable for detecting residual or recurrent middle-ear cholesteatoma ≥3 mm? A. Post-contrast T1-weighted spin-echo B. T2-weighted fast spin-echo C. Non-echo-planar diffusion-weighted imaging D. Susceptibility-weighted imaging (SWI) E. 3D time-of-flight MR angiography Explanation: (by Perplexity AI, generated with the OpenAI O3 model) • Cholesteatoma contains tightly packed keratin, giving marked restriction of water diffusion; non- echo-planar DWI (e.g. PROPELLER or HASTE) exploits this, providing bright high-signal foci against suppressed background and avoids susceptibility artefact from temporal-bone air interfaces. • It detects lesions as small as 2–3 mm with >90% sensitivity and obviates many “second-look” surgeries. • Post-contrast T1 (A) usually shows no enhancement and may miss small lesions. • Conventional T2 FSE (B) lacks specificity because fluid and granulation tissue also appear bright. • SWI (D) is designed for paramagnetic blood products, not keratin, and TOF MRA (E) images flow in vessels, offering no benefit for cholesteatoma assessment. 14. Which of the following is a recognised advantage of magnetic resonance urography (MRU) compared with CT urography? A. Superior detection of small ureteric calculi B. Ability to combine high soft-tissue contrast imaging with functional assessment in one examination C. Shorter acquisition time D. Lower sensitivity to patient movement E. Greater availability in emergency settings Explanation: (by Perplexity AI, generated with the OpenAI O3 model) • MRU exploits T2-weighted “static-fluid” and post-contrast T1-weighted sequences, allowing simultaneous high-contrast anatomical imaging of the entire urinary tract and quantitative functional data such as renal transit times and differential renal function. This one-stop structural-plus- functional capability is unique to MRU and underpins its use in congenital anomalies, hydronephrosis and renal impairment. • CT urography offers faster scans and excellent stone detection but provides little functional information and uses ionising radiation. • Acquisition times are actually longer with MRU, small calculi are often missed, motion artefact remains problematic and MRI scanners are less widely available, so options A, C, D and E are incorrect. 15. A 38-year-old woman with metastatic, hormone-receptor-negative/HER2-negative breast cancer has widespread visceral disease 6 months after her initial diagnosis. She is otherwise fit and is being considered for palliative chemotherapy. Which single-agent intravenous regimen is recommended as first-line treatment in this setting, according to contemporary guidelines? A. Capecitabine B. Paclitaxel C. Eribulin D. Vinorelbine E. Cyclophosphamide + Methotrexate + 5-FU (CMF)
  • 104.
    EBDR Exam MCQs& Concepts July 2024 Dr. Kareem Alnakeeb 98 Explanation: (by Perplexity AI, generated with the OpenAI O3 model) • Modern ESMO and ASCO guidance advise starting fit patients who have triple-negative metastatic breast cancer with a taxane (paclitaxel or docetaxel) as first-line single-agent chemotherapy because these drugs offer the best balance of response rate, survival benefit and tolerability. • Capecitabine, vinorelbine and eribulin are commonly used later-line options after taxane or anthracycline failure. • CMF is outdated and reserved only when newer agents are unsuitable. • Therefore, weekly or 3-weekly paclitaxel is the preferred initial regimen. 16. Ductal carcinoma in situ (DCIS) is suspected on screening mammography; an MRI is performed for surgical planning and shows a non-mass clumped ductal enhancement extending over 6 cm in the upper outer quadrant of the left breast with type II (plateau) kinetic curve. According to BI-RADS MRI descriptors, what is the most appropriate category to assign? A. BI-RADS 2 (benign) B. BI-RADS 3 (probably benign) C. BI-RADS 4A (low-suspicion malignancy) D. BI-RADS 4C (high-suspicion malignancy) E. BI-RADS 5 (highly suggestive of malignancy) Explanation: (by Perplexity AI, generated with the OpenAI O3 model) • Clumped ductal non-mass enhancement covering >1 segment and showing plateau kinetics is strongly associated with high-grade DCIS or invasive cancer, carrying a positive predictive value around 30%, which fits BI-RADS 4C (51–94% likelihood of malignancy). • BI-RADS 5 requires classic malignant mass features with ≥95% likelihood, which non-mass enhancement rarely achieves. • A category 4A would underestimate risk, while BI-RADS 2 or 3 are inappropriate for suspicious morphology and kinetics. 17. Catheter-directed intra-arterial thrombolysis is most appropriately used for which pattern of lower-limb ischaemia? A. Acute limb ischaemia of less than 14 days’duration B. Chronic critical limb ischaemia with ulceration present for 2 months C. Stable intermittent claudication due to femoropopliteal stenosis D. Non-viable limb with fixed paralysis and mottling (Rutherford class III) E. Thrombosed infra-inguinal bypass graft occluded for 8 weeks Explanation: (by Perplexity AI, generated with the OpenAI O3 model) • Thrombolytic agents can dissolve fresh thrombus and are therefore recommended for acute limb ischaemia (<14 days, Rutherford I–IIa), where clot is still soft and the limb remains viable. • Chronic critical ischaemia and long-standing graft occlusions contain organised thrombus and intimal hyperplasia, so chemical lysis is ineffective, often needing surgical or endovascular reconstruction. • Intermittent claudication is treated electively; risks of thrombolysis outweigh benefits. • A non-viable limb (fixed neurological deficit) requires primary amputation or open thrombectomy, as thrombolysis is too slow and increases bleeding risk
  • 105.
    EBDR Exam MCQs& Concepts July 2024 Dr. Kareem Alnakeeb 99 18. On high-resolution CT, which pleural abnormality is considered most characteristic of previous asbestos exposure? A. Diffuse parietal pleural thickening involving the costophrenic angles B. Bilateral calcified plaques on the diaphragmatic pleura C. Smooth visceral pleural thickening over the lung apices D. Circumferential mediastinal pleural thickening without calcification E. A focal pleural mass crossing an interlobar fissure Explanation: (by Perplexity AI, generated with the OpenAI O3 model) • Calcified plaques on the diaphragmatic (and parietal) pleura are the classic sentinel sign of asbestos exposure because this site is seldom affected by other diseases; their bilateral, well-circumscribed, often calcified appearance reflects prior fibre deposition and pleural reaction. • Diffuse pleural thickening (A) can follow infection or haemothorax. • Smooth apical visceral thickening (C) is typical of post-primary TB scarring. • Non-calcified mediastinal pleural thickening (D) lacks specificity and may occur with any chronic pleuritis. • A focal pleural mass crossing a fissure (E) suggests malignant mesothelioma rather than benign asbestos-related plaque disease. 19. Which of the following conditions classically produces fibrosis that predominantly affects the lung upper lobes? A. Ankylosing spondylitis B. Asbestosis C. Sarcoidosis D. Silicosis E. Tuberculosis Explanation: (by Perplexity AI, generated with the OpenAI O3 model) • Asbestosis causes lower-lobe‐predominant interstitial fibrosis because the asbestos fibres settle in the dependent peripheral lung bases; hence it is the exception in this list. • Upper-lobe fibrosis is typical of sarcoidosis (perihilar upper-zone scarring), silicosis (nodular/progressive massive fibrosis in miners), tuberculosis (post-primary fibrocavitary disease in the apices) and, less commonly, ankylosing spondylitis (apical pleural/subpleural fibrosis). • These diseases share pathophysiological factors—higher ventilation, oxygen tension and slower lymphatic clearance—in the upper zones, predisposing them to fibrosis, whereas asbestos exposure targets the lower lobes. 20. A 12-year-old boy presents with fever and thigh pain; femoral radiograph shows a permeative diaphyseal lesion with laminated (“onion-skin”) periosteal reaction and a sizable adjacent soft-tissue mass—what is the most likely diagnosis? A. Osteoid osteoma B. Acute osteomyelitis C. Ewing sarcoma D. Langerhans cell histiocytosis E. Osteosarcoma Explanation: (by Perplexity AI, generated with the OpenAI O3 model) • The combination of a diaphyseal permeative bone destruction, lamellated periosteal reaction and bulky soft-tissue component is classic for Ewing sarcoma, a small-round-cell tumour that typically affects the mid-shaft of long bones in children and adolescents. • Osteoid osteoma is a small cortical nidus without aggressive periosteal change.
  • 106.
    EBDR Exam MCQs& Concepts July 2024 Dr. Kareem Alnakeeb 100 • Acute osteomyelitis can mimic tumour but usually lacks a large organised soft-tissue mass and shows more ill-defined periosteal lifting. • Langerhans cell histiocytosis often produces punched-out lesions or beveled edges and minimal periosteal response. • Osteosarcoma is more common in the metaphyses and characteristically shows a sunburst or cloud-like osteoid matrix rather than laminated periosteal layering. 21. A 29-year-old man with primary infertility and a palpable left-sided grade III varicocele undergoes venography showing reflux in the internal spermatic vein; which interventional radiology technique is most appropriate to occlude the refluxing vein while preserving arterial flow? A. Antegrade scrotal sclerotherapy B. Retrograde coil embolisation via the internal jugular vein C. Percutaneous balloon angioplasty D. Laparoscopic Palomo ligation E. Microsurgical sub-inguinal varicocelectomy Explanation: (by Perplexity AI, generated with the OpenAI O3 model) • Retrograde coil embolisation is the standard endovascular treatment for varicocele. Through a jugular or femoral venous approach, a catheter is advanced into the gonadal vein and coils or glue are deployed to block reflux, avoiding the testicular artery and lymphatics, with ≥90% technical success and low recurrence. • Antegrade sclerotherapy (A) is done through a scrotal incision and is less commonly used. • Balloon angioplasty (C) treats stenoses, not venous reflux. • Laparoscopic Palomo (D) and microsurgical varicocelectomy (E) are surgical, not radiological, options; both require operative ligation rather than intravascular occlusion. 22. A 7-year-old child presents with headache and Parinaud’s syndrome. Non-contrast CT shows a pineal region mass with scattered “exploded” peripheral calcifications that displace the native pineal calcification fragments outward. Which tumour most commonly exhibits this calcification pattern? A. Pineal germinoma B. Pineoblastoma C. Pineocytoma D. Metastasis E. Pineal meningioma Explanation: (by Perplexity AI, generated with the OpenAI O3 model) • Pineal parenchymal tumours (pineoblastoma and pineocytoma) characteristically blast the normal pineal calcification apart, producing multiple peripheral flecks – the classic “exploded” or peripherally dispersed pattern. • Among them, the aggressive, poorly differentiated pineoblastoma is the typical childhood tumour causing Parinaud’s syndrome and peripheral calcification. • Germinomas usually engulf the pre-existing calcification centrally, while meningioma and metastasis seldom arise in the pineal gland and lack this specific pattern.
  • 107.
    EBDR Exam MCQs& Concepts July 2024 Dr. Kareem Alnakeeb 101 23. Which type of central venous access device is most commonly inserted via a large vein in the upper arm for prolonged outpatient chemotherapy delivery? A. Tunneled Hickman line B. Implanted port (Port-a-Cath) C. Peripherally inserted central catheter (PICC) D. Non-tunnelled subclavian central line E. Midline peripheral catheter Explanation: (by Perplexity AI, generated with the OpenAI O3 model) • A peripherally inserted central catheter (PICC) is a long, flexible tube placed through a basilic, brachial or cephalic vein above the elbow and advanced so its tip lies in the superior vena cava, making it ideal for repeated chemotherapy infusions over weeks to months. • Hickman lines and Port-a-Cath devices are inserted via the chest wall or neck, not the arm, while non-tunnelled subclavian lines are intended for short-term inpatient use. • A midline catheter does not reach the central veins, so it cannot safely deliver vesicant chemotherapy. 24. On cardiac MRI, which imaging plane is obtained by angling perpendicular to the line connecting the ventricular apex and the centre of the mitral valve annulus, thereby providing stacked cross-sections through both ventricles? A. Horizontal long-axis (four-chamber) plane B. Vertical long-axis (two-chamber) plane C. Left ventricular outflow tract (three-chamber) plane D. Short-axis plane E. Coronal thoracic plane Explanation: (by Perplexity AI, generated with the OpenAI O3 model) • The short-axis plane is planned perpendicular to the heart’s long axis (drawn from apex to mitral valve centre). Sequential slices in this orientation give circular cross-sections of the left and right ventricles, allowing accurate assessment of ventricular volumes and wall motion. • The horizontal and vertical long-axis planes (options A and B) run parallel to the long axis, displaying chambers in longitudinal profile, not cross-section. • The LV outflow tract or three-chamber view (option C) is oblique, aligned with the aortic and mitral valves to show the LVOT. • A standard coronal thoracic plane (option E) is body-orientated, not heart-orientated, and lacks consistent ventricular short-axis anatomy. 25. A 58-year-old man with Child-Pugh A cirrhosis is found to have a solitary 6 cm hepatocellular carcinoma in segment VI. Which of the following locoregional therapies is generally NOT considered technically suitable or oncologically effective for a 6 cm HCC? A. Conventional trans-arterial chemo-embolisation (TACE) B. Microwave thermal ablation C. Radiofrequency thermal ablation D. Stereotactic body radiotherapy (SBRT) E. Yttrium-90 trans-arterial radio-embolisation (TARE) Explanation: (by Perplexity AI, generated with the OpenAI O3 model) • Radiofrequency ablation is regarded as curative only for tumours ≤3 cm, with substantially higher incomplete ablation and local recurrence rates once diameter exceeds about 3 – 4 cm; a 6 cm lesion is therefore unsuitable for RFA.
  • 108.
    EBDR Exam MCQs& Concepts July 2024 Dr. Kareem Alnakeeb 102 • In contrast, microwave ablation can treat lesions up to 5–7 cm because of higher intratumoural temperatures and larger ablation zones; SBRT can deliver ablative doses to tumours up to 6 – 7 cm; TACE is standard of care for intermediate-size unresectable HCCs; and Y-90 TARE is effective for larger solitary tumours or multifocal disease. • Thus, among the options listed, only radiofrequency ablation is generally inappropriate for a 6 cm HCC. 26. A cardiac MRI shows an avidly contrast-enhancing, broad-based mass infiltrating the right atrial wall and attached by a thin sessile margin; which diagnosis is most likely? A. Left-atrial myxoma B. Organized mural thrombus C. Metastatic melanoma deposit D. Primary cardiac angiosarcoma E. Papillary fibroelastoma Explanation: (by Perplexity AI, generated with the OpenAI O3 model) • Malignant cardiac tumours typically present as infiltrative, poorly marginated, vascular masses with heterogeneous or intense enhancement on post-gadolinium MRI and a broad, sessile attachment; these features best fit a right-sided primary angiosarcoma. • Myxomas are usually pedunculated, arise from the inter-atrial septum in the left atrium, and enhance less avidly. • Chronic thrombus shows no (or only peripheral) enhancement and often appears non-vascular. • Cardiac melanoma metastases can be hyper-enhancing but usually occur in patients with known widespread disease and often involve the left atrium or ventricle. • Papillary fibroelastomas are small, highly mobile, valvular masses with minimal enhancement, not infiltrative lesions. 27. A feature that favors a malignant cardiac mass on imaging is: A. A Lack of first-pass perfusion B. Thin pedunculated attachment C. Heterogeneous first-pass contrast enhancement with late gadolinium enhancement D. Absence of pericardial effusion Explanation: (by Perplexity AI, generated with the OpenAI O3 model) • Malignant tumors are vascular, infiltrative and heterogeneously enhance. • Benign myxomas classically have a thin stalk. • Thrombi lack enhancement and often have a thin attachment 28. Which primary cardiac tumour is most frequently encountered in adults on routine imaging or at surgery? A. Cardiac fibroma B. Papillary fibro-elastoma C. Cardiac hemangioma D. Cardiac myxoma E. Cardiac rhabdomyoma Explanation: (by Perplexity AI, generated with the OpenAI O3 model) • Cardiac myxomas account for about half of all primary cardiac tumours in adults and roughly 75% arise from the left atrial septum, making them the commonest lesion in this age group.
  • 109.
    EBDR Exam MCQs& Concepts July 2024 Dr. Kareem Alnakeeb 103 • Fibromas and rhabdomyomas are predominantly paediatric tumours; fibromas are intramural ventricular masses, while rhabdomyomas are strongly linked to tuberous sclerosis and often regress spontaneously. • Papillary fibro-elastomas are the most frequent valvular tumours but overall are less common than myxomas. • Cardiac hemangiomas are rare vascular malformations that represent only a small fraction of benign cardiac masses. 29. In a patient with Marfan syndrome undergoing routine echocardiographic follow-up, which cardiac valve lesion is encountered most frequently? A. Mitral stenosis B. Mitral regurgitation C. Tricuspid stenosis D. Tricuspid regurgitation E. Aortic stenosis Explanation: (by Perplexity AI, generated with the OpenAI O3 model) • Myxomatous degeneration of the mitral valve apparatus is common in Marfan syndrome, leading to mitral valve prolapse and resulting regurgitation—the typical and most prevalent valvular abnormality in these patients. • Stenotic lesions of either atrioventricular valve (Options A and C) are rare in Marfan and usually reflect unrelated rheumatic or congenital pathology. • Tricuspid regurgitation (Option D) is less frequent and, when present, is often secondary to pulmonary hypertension rather than primary valvular disease. • Aortic stenosis (Option E) is not characteristic; Marfan pathology primarily affects the aortic root, causing dilatation and regurgitation rather than obstruction. 30. On dedicated ocular MRI, which signal intensity combination is most typical for a primary uveal melanoma? A. High T1, high T2 B. High T1, low T2 C. High T1, intermediate T2 D. Low T1, low T2 E. Low T1, high T2 Explanation: (by Perplexity AI, generated with the OpenAI O3 model) • Melanin shortens both T1 and T2 relaxation times. Consequently, pigmented uveal melanomas characteristically show intrinsic T1 hyper-intensity while appearing hypo-intense on T2-weighted images. This high-T1/low-T2 pattern is well-described and helps distinguish melanoma from most other intra-ocular masses, which lack melanin and therefore remain low or iso-intense on T1 and variably hyper-intense on T2. • Options A, C and E include a high T2 signal that is uncharacteristic for pigmented melanoma; option D lacks the typical T1 hyper-intensity produced by melanin.
  • 110.
    EBDR Exam MCQs& Concepts July 2024 Dr. Kareem Alnakeeb 104 31. On MRI of a spinal intradural-extramedullary tumour, marked gadolinium enhancement of a tapering “dural (epidural) tail” is highly specific but not very sensitive for which neoplasm? A. Astrocytoma B. Ependymoma C. Hemangioblastoma D. Meningioma E. Schwannoma Explanation: (by Perplexity AI, generated with the OpenAI O3 model) • The dural (or epidural) tail sign is produced by reactive vascularised thickening of adjacent dura. Although it appears in only about 60% of meningiomas (hence limited sensitivity), when present it points very strongly toward this diagnosis, with reported specificity exceeding 90%. • Other intradural or extradural tumours such as ependymoma, astrocytoma, hemangioblastoma and schwannoma can very rarely show a similar tapering enhancement, but this is uncommon, so their presence does not match the high specificity seen with meningioma. 32. Which posterior cranial fossa tumour is usually intra-axial rather than extra-axial (extramural)? A. Metastasis B. Vestibular schwannoma C. Meningioma D. Ependymoma E. Hemangioblastoma Explanation: (by Perplexity AI, generated with the OpenAI O3 model) • Ependymomas arise from the ependymal lining of the fourth ventricle and adjacent brain parenchyma, so they lie within the neural tissue of the posterior fossa (intra-axial); they are therefore not classed as extra-axial (extramural) lesions. • Posterior-fossa extra-axial tumours include vestibular schwannoma, meningioma and dural or leptomeningeal metastases, all of which originate outside the brain parenchyma and displace it. • Hemangioblastomas are also usually intra-axial but cystic–solid morphology and vascular flow voids help separate them from ependymoma. • Key distractors therefore describe classic extra-axial masses, leaving ependymoma as the only intra- axial option. 33. At what level does the cauda equina begin in the average adult? A. Above T12 B. Below L1 C. L2/3 D. L3/4 E. L4/5 Explanation: (by Perplexity AI, generated with the OpenAI O3 model) • The cauda equina begins below the termination of the spinal cord (conus medullaris), which occurs at the L1-L2 vertebral level in the average adult. • The spinal cord tapers and ends between the first and second lumbar vertebrae, with the cauda equina consisting of nerve roots L2-S5 and the coccygeal nerve extending distally from this point. • While the conus medullaris may vary from T12 to L3 in different individuals, it typically lies around L1 level, making "below L1" the most accurate description of where the cauda equina begins. • Options C and D represent levels within the cauda equina rather than its commencement, while option A is too superior and represents spinal cord rather than cauda equina territory.
  • 111.
    EBDR Exam MCQs& Concepts July 2024 Dr. Kareem Alnakeeb 105 34. What is the most common congenital anomaly of the pancreas encountered in everyday radiology practice? A. Annular pancreas B. Dorsal pancreatic agenesis C. Pancreas divisum D. Heterotopic (ectopic) pancreas E. True congenital pancreatic cyst Explanation: (by Perplexity AI, generated with the OpenAI O3 model) • Pancreas divisum results from failure of fusion of the dorsal and ventral pancreatic ducts during the 7th gestational week. • Autopsy and MRCP studies show it in 4–14% of the population, making it the commonest congenital pancreatic anomaly. • Annular pancreas is far rarer (≈1/20,000) and typically presents with duodenal obstruction in neonates. • Dorsal agenesis is very uncommon and often associated with other anomalies. • Heterotopic pancreas and true congenital pancreatic cysts are both rare incidental findings. 35. A 9-month-old infant presents with progressive jaundice. Ultrasound shows a solid, vascular mass centred at the porta hepatis that extends into the common bile duct causing marked intra- and extrahepatic duct dilatation. Serum alpha-fetoprotein is normal. What is the most likely diagnosis? A. Embryonal rhabdomyosarcoma B. Infantile haemangioma C. Hepatoblastoma D. Choledochal cyst with dysplasia E. Neuroblastoma Explanation: (by Perplexity AI, generated with the OpenAI O3 model) • Embryonal rhabdomyosarcoma is the commonest malignant tumour of the biliary tree in infancy and typically arises at the porta hepatis, invading the bile ducts to produce obstructive jaundice; it is usually AFP-negative. • Hepatoblastoma (C) generally originates within hepatic parenchyma, elevates AFP and only secondarily compresses ducts rather than growing intraductally. • Infantile haemangioma (B) is hypervascular but does not infiltrate ducts and seldom causes obstruction. • A choledochal cyst with dysplasia (D) presents as cystic dilatation, not a solid vascular mass. • Neuroblastoma (E) arises from the adrenal or sympathetic chain, may encase vessels but rarely invades the biliary ducts or presents primarily at the porta hepatis.
  • 112.
    EBDR Exam MCQs& Concepts July 2024 Dr. Kareem Alnakeeb 106 36. A 65-year-old man presents with progressive, painless jaundice. MRI with MRCP shows a solid mass at the porta hepatis extending within the common hepatic duct and proximal common bile duct, producing an abrupt, band-like stricture (“shouldering”) and marked upstream biliary dilatation. Which single diagnosis best explains these findings? A. Cholangiocarcinoma (hilar type) B. Gallbladder carcinoma invading the bile duct C. Pancreatic head adenocarcinoma D. Primary sclerosing cholangitis E. Metastatic lymph-node compression Explanation: (by Perplexity AI, generated with the OpenAI O3 model) • Hilar (Klatskin) cholangiocarcinoma arises at the confluence of the hepatic ducts and commonly grows intraductally, creating an annular, shouldered stricture with proximal ductal dilatation but little distal duct involvement, as in this case. • Gallbladder carcinoma can extend into the bile duct but usually originates from a gallbladder mass, often with cholelithiasis, which is not described. • Pancreatic head cancer obstructs the distal common bile duct rather than the porta hepatis. • Primary sclerosing cholangitis causes multifocal, beaded narrowing, not a solitary mass. • Metastatic nodes compress ducts extrinsically without intraductal extension or a discrete porta hepatis mass. 37. Following lumbar discectomy, a patient re-presents with radicular pain. MRI with gadolinium is performed. Which MRI appearance most reliably indicates recurrent disc herniation rather than postoperative scar tissue? A. Extradural lesion that fails to enhance after contrast B. Extradural lesion that avidly enhances after contrast C. Intradural lesion that avidly enhances after contrast D. Intradural lesion that fails to enhance after contrast E. Uniform enhancement of the surgical bed and surrounding fat planes Explanation: (by Perplexity AI, generated with the OpenAI O3 model) • Recurrent nucleus pulposus remains avascular; on contrast-enhanced MRI it appears as an extradural mass that shows little or no enhancement, often surrounded by a thin enhancing rim of granulation tissue. • Postoperative epidural scar, by contrast, is vascularised granulation tissue and therefore demonstrates diffuse or nodular enhancement after gadolinium (ruling out option B). • Intradural lesions (options C and D) are anatomically incorrect for disc recurrence and instead suggest arachnoiditis or tumour. • Uniform enhancement of the operative site (option E) reflects expected postoperative granulation and fibrotic change, not disc material.
  • 113.
    EBDR Exam MCQs& Concepts July 2024 Dr. Kareem Alnakeeb 107 38. On cardiac MRI, which characteristic imaging feature most strongly suggests cardiac amyloidosis? A. Patchy mid-myocardial late gadolinium enhancement B. Focal asymmetric septal hypertrophy C. Diffuse subendocardial or transmural late gadolinium enhancement with difficulty nulling the myocardium D. Apical thinning with hyper-trabeculation E. Right-sided ventricular outflow tract obstruction Explanation: (by Perplexity AI, generated with the OpenAI O3 model) • Cardiac amyloidosis causes widespread interstitial deposition of amyloid fibrils, expanding the extracellular volume and producing diffuse subendocardial or transmural late gadolinium enhancement (LGE) on inversion-recovery sequences; the abnormal myocardium nulls at the same time as blood, making correct TI selection difficult. • Patchy enhancement (A) can occur but is less specific. Hypertrophic cardiomyopathy typically shows focal asymmetric septal hypertrophy (B). • Apical thinning with trabeculation (D) is characteristic of left ventricular non-compaction. • Right ventricular outflow obstruction (E) suggests conditions such as pulmonary stenosis or carcinoid heart disease, not amyloid infiltration. 39. In a patient with atrial fibrillation, which imaging technique is regarded as the reference standard for quantitative calculation of left-ventricular ejection fraction? A. CT coronary angiography B. MR coronary angiography C. Left ventricular cine angiography during cardiac catheterisation D. Planar radionuclide ventriculography (MUGA scan) E. PET-CT perfusion imaging Explanation: (by Perplexity AI, generated with the OpenAI O3 model) • Multigated planar radionuclide ventriculography (MUGA) remains the benchmark for precise, beat- to-beat quantification of left-ventricular ejection fraction, even when rhythm is irregular as in atrial fibrillation. • Gamma-camera acquisition over hundreds of cardiac cycles averages out cycle-length variability, giving reproducible EF with <5% inter-study error. • Cardiac MRI offers excellent spatial resolution but suffers from mis-gating in AF and longer breath- holds; CT angiography is optimised for coronary anatomy rather than ventricular volumes; invasive ventriculography provides single-beat EF only and is affected by arrhythmia-related variation; PET-CT focuses on metabolism or perfusion and does not directly compute EF..
  • 114.
    EBDR Exam MCQs& Concepts July 2024 Dr. Kareem Alnakeeb 108 40. In patients with atrial fibrillation, the most accurate non-invasive modality for quantifying LVEF is: A. Transthoracic or trans-esophageal echocardiography (2-D & 3-D) B. Cardiac CT angiography (CTA) C. Cardiac MR (CMR) D. Invasive left ventriculography E. Gated SPECT myocardial perfusion (“MUGA” or perfusion-gated) F. Gated PET-CT Explanation: (by Perplexity AI, generated with the OpenAI O3 model) Practical take-aways for patients in AF • Best overall accuracy: CMR with arrhythmia-rejection remains the reference when precise LVEF is required for therapy decisions (e.g., device eligibility). • Best balance of speed and availability: 3-D echocardiography averaged over multiple representative beats is adequate for most routine assessments; always report the number of beats averaged. • CT angiography is reliable when already indicated for coronary imaging, but use dose-modulating techniques sparingly in fast AF because they discard irregular beats. • Nuclear techniques are acceptable when simultaneous perfusion data are needed; ensure tight beat-rejection windows to avoid systematic LVEF depression. • Single-beat invasive ventriculography is now reserved for catheter-lab situations where no other modality is available.
  • 115.
    EBDR Exam MCQs& Concepts July 2024 Dr. Kareem Alnakeeb 109 41. A child who undergoes MR of the brain for clinically apparent facial abnormalities is shown to have a defect of midline cleavage of the brain. What structure is abnormal or absent in all forms of holoprosencephaly, and therefore is the most sensitive indicator of a midline cleavage abnormality? A. falx cerebri B. third ventricle C. fourth ventricle D. corpus callosum E. septum pellucidum Source: Bydder, Megan, et al. Get Through Final FRCR Part A: SBAs for the Modular Examination. 1st ed., Hodder Education Publishers, 2009. Explanation: Overview • Holoprosencephaly is failure of the primitive brain to cleave into two hemispheres, and is commonly associated with midline facial abnormalities (ranging from cyclopia to hypertelorism) and absence of many intracranial midline structures. Classification There are three types; Alobar Form • the most severe being the alobar form, which shows no cleavage at all, with absence of the falx cerebri and third ventricle, fusion of the cerebral hemispheres and thalami, and a single large lateral ventricle. Semilobar Form • The semilobar form has variable cleavage with a partially formed falx, rudimentary third ventricle, and variable cleavage of the thalami, lateral ventricles and cerebral hemispheres. Lobar Form • In the lobar type of holoprosencephaly, brain formation may be nearly normal, but the septum pellucidum is always absent, as in all forms. • The falx, corpus callosum and ventricular system may be normal in the lobar type. 42. A plain lumbar spine radiograph of a 45-year-old woman shows marked posterior scalloping of the vertebral bodies extending over several vertebral lengths. All of the following are diseases associated with this finding except K. Marfan L. Neurofibromatosis M. Ependymoma N. Achondroplasia O. Hypothyroidism Source: Chin, Teck Yew, et al. Get Through Final FRCR 2A: SBAs. 1st ed., CRC Press, 2017. Explanation: Mass-Related Causes • A common cause of localised posterior vertebral scalloping is increased intraspinal pressure secondary to an expanding mass. • Widening of the interpediculate distance and alteration of the configuration of the pedicles are associated signs. • Relatively large, slow-growing lesions that originate during a period of active skeletal growth (such as ependymomas) are most likely to give rise to posterior vertebral scalloping. Dural Ectasia and Connective-Tissue Disorders
  • 116.
    EBDR Exam MCQs& Concepts July 2024 Dr. Kareem Alnakeeb 110 • Dural ectasia is thought to cause posterior vertebral scalloping due to loss of the normal protection provided to the vertebral body by a strong, intact dura. • Dural ectasia classically occurs in association with inherited connective-tissue disorders such as Marfan syndrome (classical) and Ehlers–Danlos syndrome. Neurofibromatosis and Related Conditions • Posterior vertebral scalloping is also commonly seen in patients with neurofibromatosis, most likely due to dural ectasia but also secondary to neurofibromas or a thoracic meningocoele. • It has also been reported in patients with AS; in these cases, the development of associated arachnoid cysts may give rise to cauda equina syndrome. Endocrine Disorders • Acromegaly has been described as a further cause of diffuse posterior vertebral scalloping, probably because of a combination of soft-tissue hypertrophy in the spinal canal and increased bone resorption. 43. A 25 year old man has a routine chest radiograph prior to a work permit application. It demonstrates a well-defined, rounded mediastinal mass. Which of the following features on CT would make a diagnosis of bronchogenic cyst less likely? A. Soft-tissue density B. Thick wall C. Precarinal location D. Communication with tracheal lumen E. Unilocularity Source: Currie, Stuart, et al. SBAs for the FRCR 2A. 1st ed., Cambridge University Press, 2010. Explanation: • Bronchogenic cyst is the most common intrathoracic foregut duplication cyst. • It could have all the above features, but in a mediastinal location, the cyst walls are usually thin. Thick-walled cysts are more likely to be oesophageal. 44. On an axial section of the brain at the level of the third ventricle, which structure lies immediately lateral to the putamen? A. internal capsule B. globus pallidus C. external capsule D. thalamus E. insular cortex Source: Bydder, Megan, et al. Get Through Final FRCR Part A: SBAs for the Modular Examination. 1st ed., Hodder Education Publishers, 2009. Explanation: • The lentiform nucleus is composed of a larger lateral component (the putamen) and a smaller medial component (the globus pallidus), separated by a sheet of white matter. • The lentiform nucleus is bounded medially by the internal capsule. Lateral to the lentiform nucleus lies the white matter of the external capsule, and then the claustrum, a thin sheet of grey matter. • The extreme capsule lies lateral to the claustrum, and separates it from the insular cortex.
  • 117.
    EBDR Exam MCQs& Concepts July 2024 Dr. Kareem Alnakeeb 111 45. A 50 year old male presents with a history of occasional haemoptysis and exertional shortness of breath which has been getting progressively worse. Plain chest radiograph demonstrates bibasal reticular shadowing with volume loss. HRCT demonstrates bibasal fibrosis and traction bronchiectasis. Incidental note is made of a patulous oesophagus. Which of the following is the most likely cause? A. Tuberculosis B. SLE C. Rheumatoid arthritis D. Wegener’s granulomatosis E. Scleroderma Source: Currie, Stuart, et al. SBAs for the FRCR 2A. 1st ed., Cambridge University Press, 2010. Explanation: • Whilst haemoptysis may be a presentation in tuberculosis and Wegener’s and bibasal fibrosis maybe seen in all of the above except tuberculosis (where apical fibrosis is the more likely feature), scleroderma is the only condition resulting in a patulous lower oesophageal sphincter, oesophageal shortening and stricture formation. 46. Which of the following is NOT an indication for percutaneous nephrostomy? A. Benign obstruction of the ureter B. Urinary leakage or fistula C. Pyelonephritis in a non-dilated renal pelvis D. Malignant obstruction of the ureter 47. A 36 year old female with history of pelvic pain and severe dysmenorrhoea undergoes a pelvic ultrasound examination which reveals uterine fibroid disease. Which of the following imaging features would be associated with the best outcome following uterine artery embolisation? A. Submucosal location B. Subserosal location C. Associated adenomyosis D. Calcification E. Multiple fibroids Source: Currie, Stuart, et al. SBAs for the FRCR 2A. 1st ed., Cambridge University Press, 2010. Explanation: • Subserosal fibroids, especially pedunculated ones, may often draw their blood supply from adjacent viscera, which may be a cause of failure of the procedure. They are also associated with a higher incidence of complications. • Calcific fibroids are less vascular and may not respond well to embolisation. • Bulky and multiple fibroids may need multiple interventions or surgery. • Adenomyosis is a known cause for failure of the procedure. 48. A patient with a known collagen vascular disease has pulmonary fibrosis. HRCT reveals bilateral lower lobe bronchiectasis. Which collagen vascular disease is most likely? A. Sjogren syndrome B. Progressive systemic sclerosis C. SLE D. Rheumatoid arthritis E. Dermatomyositis Source: Currie, Stuart, et al. SBAs for the FRCR 2A. 1st ed., Cambridge University Press, 2010. Explanation: • Whilst pulmonary fibrosis is a feature of all the above conditions, bronchiectasis is most likely seen in Sjogren syndrome.
  • 118.
    EBDR Exam MCQs& Concepts July 2024 Dr. Kareem Alnakeeb 112 49. Eight days after lung transplantation for alpha-1 antitrypsin deficiency, a 45 year old man develops pyrexia, breathlessness and desaturation. HRCT reveals perihilar heterogenous opacities and ground glass changes with new pleural effusion and septal thickening. Which of the following is the most likely cause? A. Reperfusion oedema B. Acute rejection C. Anastomotic dehiscence D. Post-transplantation PCP infection E. Hyperacute rejection Source: Currie, Stuart, et al. SBAs for the FRCR 2A. 1st ed., Cambridge University Press, 2010. Explanation: • Hyperacute rejection presents within hours of the transplantation. • Reperfusion oedema usually presents within 24 hours of the transplantation, peaking by about day four. • Posttransplant infections can be broadly divided into those occurring within the first month (gram- negative bacteria, fungi (candida, aspergillosis)) and those occurring after the first month (CMV, PCP). • Anastomotic dehiscence is usually an early feature, but the presentation and features are not those described. 50. A young man presents with progressive productive cough and halitosis. He had severe pneumonia as a child. Plain chest radiograph demonstrates bronchial dilatation and bronchial wall thickening with some volume loss. Which of the following HRCT findings is the most sensitive finding for bronchiectasis? A. Air trapping B. Mucous-filled dilated bronchi C. Bronchial wall thickening D. Bronchi seen in the subpleural region E. Lack of bronchial tapering Source: Currie, Stuart, et al. SBAs for the FRCR 2A. 1st ed., Cambridge University Press, 2010. Explanation: • Whilst all the above can be seen in patients with bronchiectasis, a lack of progressive tapering of the bronchi is the most sensitive (80%). 51. A 26 year old female patient with an optic nerve tumour and café-au-lait spots presents with exertional breathlessness. Imaging of the chest is most likely to reveal which of the following? A. Multiple small lower lobe cysts B. Emphysema C. Lower zone fibrosis D. Thick-walled cavities in the upper zone E. Asymmetrical upper zone fibrosis Source: Currie, Stuart, et al. SBAs for the FRCR 2A. 1st ed., Cambridge University Press, 2010. Explanation: • The case describes neurofibromatosis I, which is associated with lower zone fibrosis and thin-walled bullae, mainly in the upper zones. Apart from the pulmonary changes, skeletal abnormalities involving the ribs and spine and mediastinal masses may also be seen. 52. A 22 year old is diagnosed with tuberculosis. Which of the following features will make a diagnosis of primary tuberculosis more likely? A. Mediastinal enlargement B. Septal thickening C. Upper zone cavitation D. Miliary nodules E. Apical consolidation Source: Currie, Stuart, et al. SBAs for the FRCR 2A. 1st ed., Cambridge University Press, 2010.
  • 119.
    EBDR Exam MCQs& Concepts July 2024 Dr. Kareem Alnakeeb 113 Explanation: • Mediastinal lymph node enlargement is a feature of primary TB. The others are seen with reactivation or fibrocavitary TB. Miliary TB can be seen in any phase with haematogenous dissemination but primary presentation is uncommon. 53. A 68 year old miner develops an irregular opacity in the upper zone on plain chest radiograph. Which imaging feature would be more in favour of malignancy than progressive massive fibrosis (PMF)? A. Peripheral enhancement on contrast-enhanced MR B. Peripheral location on axial images C. Presence of calcification D. High signal on T2-weighted images E. Avid lesion on PET-CT Source: Currie, Stuart, et al. SBAs for the FRCR 2A. 1st ed., Cambridge University Press, 2010. Explanation: • PMF has a peripheral location which moves towards the hilum on follow-up imaging. Calcification and cavitation may also be seen. PMF lesions can be FDG-avid on PET-CT. However, high signal in a mass on T2-weighted images is strongly suspicious for malignancy. 54. In acute respiratory distress syndrome; what is the first change usually seen on the chest radiograph? A. confluent consolidation B. pleural effusions C. increased heart size with globular shape D. volume loss with atelectasis E. patchy ill-defined opacities Source: Bydder, Megan, et al. Get Through Final FRCR Part A: SBAs for the Modular Examination. 1st ed., Hodder Education Publishers, 2009. Explanation: • Acute respiratory distress syndrome (ARDS) commences with interstitial oedema, progressing to congestion and extensive alveolar, and interstitial oedema and haemorrhage. The chest radiograph is often normal for the first 24 hours, before patchy opacities appear in both lungs. These progress to massive airspace consolidation over the following 24–48 hours. True volume loss, atelectasis, cardiomegaly and effusions are not seen in ARDS. 55. A middle-aged man presents with easy fatigability. CT shows an anterior mediastinal mass with areas of calcifications, invading the mediastinal structures. There are multiple small pleural masses. What is the most likely diagnosis? A. Thymoma B. Thymic lipoma C. Lymphoma D. Teratoma E. Asbestosis Source: Chin, Teck Yew, et al. Get Through Final FRCR 2A: SBAs. 1st ed., CRC Press, 2017. Explanation: Classification and Clinical Associations • Thymomas are classified as encapsulated, infiltrative and metastasizing, with pulmonary and pleural deposits and thymic carcinoma. • Half the thymomas are asymptomatic and 30% are associated with myasthenia gravis. CT Imaging Features • At CT a benign thymoma appears round, oval or lobulated. • Focal calcification is seen in 25%, which may be dense, irregular or coarse. • Benign thymomas show mild homogeneous enhancement; cystic changes are also described. • Invasive thymoma are heterogeneous in appearance; pericardial and pleural nodules suggest malignancy.
  • 120.
    EBDR Exam MCQs& Concepts July 2024 Dr. Kareem Alnakeeb 114 • Egg-shell calcification is described in invasive thymoma. • Absent fat planes between thymoma and mediastinum does not necessarily reflect invasion. 56. All of the following are causes of lower zone fibrosis, except A. Amiodarone B. Idiopathic pulmonary fibrosis C. Asbestosis D. Ankylosing spondylitis E. Neurofibromatosis I Source: Chin, Teck Yew, et al. Get Through Final FRCR 2A: SBAs. 1st ed., CRC Press, 2017. Explanation: Lower Zone Fibrosis: Causes • Causes of lower zone fibrosis include asbestosis, aspiration, cryptogenic alveolitis (IPF), neurofibromatosis I and tuberous sclerosis; connective tissue diseases like RA, scleroderma and SLE; and drug toxicity to substances like amiodarone and nitrofurantoin. 57. A 58 year old male has a CT staging scan following a diagnosis of adenocarcinoma of the body of the pancreas. The tumour is 3 cm in size and extends beyond the boundaries of the pancreas but does not invade any vessels or adjacent organs. Two 1 cm lymph nodes lie adjacent to the tumour. No other nodes, or metastatic disease in the chest, abdomen or pelvis, are identified. The tumour is best staged as which one of the following? A. T1N0M0 B. T1N1M0 C. T2N0M0 D. T3N0M0 E. T3N1M0 Source: Currie, Stuart, et al. SBAs for the FRCR 2A. 1st ed., Cambridge University Press, 2010. Explanation: • T1 tumour is disease confined to the pancreas and less than 2 cm in diameter. • T2 tumour is also confined to the pancreas but greater than 2 cm in diameter. • As the tumour extends beyond the boundary of the pancreas, it is at least T3. • Invasion of the coeliac or superior mesenteric arteries would make this a T4 tumour, but as these features are not present it is T3. • The presence of regional nodes make it N1 rather than N0 (no nodes involved), and there is no metastatic disease so it is M0. • Therefore the correct radiological stage is T3N1M0.
  • 121.
    EBDR Exam MCQs& Concepts July 2024 Dr. Kareem Alnakeeb 115 58. A 52 year old male with a metal heart valve has a transrectal ultrasound performed to stage rectal carcinoma as MRI is contraindicated. A 3 cm hypoechoic mass is identified from three to seven o’clock in the lower rectum. It extends through an inner hypoechoic layer and into the outer hypoechoic layer, but the outermost hyperechoic layer is intact and unaffected. What is the correct T staging (TNM system) based on these observations? A. T0 B. T1 C. T2 D. T3 E. T4 Source: Currie, Stuart, et al. SBAs for the FRCR 2A. 1st ed., Cambridge University Press, 2010. Explanation: • The layers of the rectum are well demonstrated at transrectal ultrasound. o The innermost hyperechoic layer represents the balloon-mucosa interface, o the middle hyperechoic layer represents the submucosa and o the outermost hyperechoic layer represents the serosa. • The tumour described in the question extends through the submucosa into the muscularis propria (outer hypoechoic layer) but does not involve the serosa. o T1 disease is limited to the submucosa, o T2 is limited to the muscularis propria, o T3 extends through the serosa and o T4 represents invasion of adjacent organs. • The correct staging for the tumour described in the question is therefore T2.
  • 122.
    EBDR Exam MCQs& Concepts July 2024 Dr. Kareem Alnakeeb 116 Paper 2 1. Concerning dislocations: Which statement is CORRECT? A. Posterior dislocation of the hip accounts for 10-20% of all hip dislocations B. Anterior dislocation of the shoulder accounts for less than 30% of glenohumeral dislocations C. A Bankart lesion is a fracture of the anterior aspect of the superior rim of the glenoid D. Dislocation of the patella is usually lateral. Source: Hussain, Shahid, et al. Radiology MCQs for the New FRCR Part 2A. 1st ed., TFM Publishing, 2006. Explanation: • Anterior dislocation of the hip accounts for 10-20% of all hip dislocations - lies medial and inferior to acetabulum on pelvis X-ray • Posterior dislocations of both radius and ulna account for 80-90% of elbow dislocations - isolated dislocation of the radial head is rare • Anterior dislocation of the shoulder accounts for more then 90% of glenohumeral dislocations - 97% are anterior dislocations. Associated with a Hill-Sachs lesion which is a defect in the posterolateral aspect of the humeral head • A Bankhart lesion is a fracture of the anterior aspect of the inferior rim of the glenoid • Dislocation of the patella is usually lateral 2. On imaging, which of the followings causes 'Bone within Bone' appearance? A. Marfan's syndrome B. Sickle cell disease C. Rickets. D. Fibrous dysplasia Source: Hussain, Shahid, et al. Radiology MCQs for the New FRCR Part 2A. 1st ed., TFM Publishing, 2006. Explanation: • The causes include: o Congenital syphilis o Infantile cortical hyperostosis o Sickle cell disease o Oxalosis o Paget's disease o Acromegaly and radiation
  • 123.
    EBDR Exam MCQs& Concepts July 2024 Dr. Kareem Alnakeeb 117 3. Which of the followings is a feature of diaphyseal aclasia (hereditary multiple exostosis)? A. Autosomal recessive inheritance B. Exostoses arise from the metaphysis and point towards the joint C. Exostoses stop growing when the nearest epiphyseal center fuses D. Malignant transformation to chondrosarcoma occurs in 35-40% Source: Hussain, Shahid, et al. Radiology MCQs for the New FRCR Part 2A. 1st ed., TFM Publishing, 2006. Explanation: • AD, presents at 2-10 years of age • Exostoses have a cap of hyaline cartilage, often with a bursa formation over the cap • Exostoses arise from metaphysis of long bones near epiphyses and point away from the joint • Exostoses stop growing when the nearest epiphyseal centre fuses • Malignant transformation to chondrosarcoma occurs in <5% 4. Malignant fibrous histiocytoma: Which statement is CORRECT? A. Is the commonest soft tissue sarcoma in females >18 years of age B. Rarely calcifies C. Is most commonly found in a retroperitoneal location. D. Angiomatoid malignant fibrous histiocytoma is frequently seen in <20-year-olds Source: Hussain, Shahid, et al. Radiology MCQs for the New FRCR Part 2A. 1st ed., TFM Publishing, 2006. Explanation: • Is the commonest soft tissue sarcoma in adults >45 years of age • Presents as a painless soft tissue mass - imaging features of low signal on T1 /high signal on T2 with variable contrast enhancement • Rarely calcifies • 75% are found in the extremities, lower limb > upper limb • Angiomatoid malignant fibrous histiocytoma is frequently seen in <20-year-olds 5. Regarding myositis ossificans: Which statement is CORRECT? A. 10-20% of lesions undergo malignant transformation B. In the acute stages, lesions undergo no contrast enhancement on MRI C. On a plain radiograph, lesions are seen to be in contact with the periosteum D. It affects the large muscles of the extremities in 80-90% of cases Source: Hussain, Shahid, et al. Radiology MCQs for the New FRCR Part 2A. 1st ed., TFM Publishing, 2006. Explanation: • Myositis ossificans is a benign, non-neoplastic condition characterized by heterotopic ossification within muscle, most commonly affecting large muscles of the limbs in 80-90% of cases. It does not undergo malignant transformation. Commoner in adults. M:F 1:1 • In the acute stage, the lesions typically show contrast enhancement on MRI due to inflammation and hyperemia. • Radiographically, lesions are separated from the periosteum by a radiolucent zone. • Burns are indeed a recognized predisposing factor because they cause muscle injury. 6. Telangiectatic osteosarcoma: Which statement is CORRECT? A. The commonest type of osteosarcoma B. Painless C. High intensity signal on T2 and low signal in T1 weighted MRI D. Most commonly found in patients 60-80 years of age Source: Hussain, Shahid, et al. Radiology MCQs for the New FRCR Part 2A. 1st ed., TFM Publishing, 2006. Explanation:
  • 124.
    EBDR Exam MCQs& Concepts July 2024 Dr. Kareem Alnakeeb 118 • Telangiectatic osteosarcoma is a highly aggressive subtype of osteosarcoma, characterized by blood-filled spaces and rapid local destruction. • It presents most often with pain and swelling, typically in adolescents and young adults (15–35 years). It is not the most common osteosarcoma variant; the conventional type is more frequent. • On MRI, telangiectatic osteosarcoma exhibits high T2 signal due to its cystic, fluid-filled nature, not low T2 signal. • It rarely occurs in those aged 60–80 years; older patients are much less affected. • Pain is the commonest presenting symptom rather than absence of pain. 7. Regarding eosinophilic granuloma: Which statement is CORRECT? A. Lesions in proximal long bones are usually metaphyseal B. The commonest site is the mandible C. Lesions rarely elicit a periosteal reaction D. It is a recognized cause of 'floating teeth' appearance Source: Hussain, Shahid, et al. Radiology MCQs for the New FRCR Part 2A. 1st ed., TFM Publishing, 2006. Explanation: • The commonest site of eosinophilic granuloma is the skull. • Lesions in proximal long bones are usually diaphyseal, but the skull remains the most common site overall. • On MRI, eosinophilic granuloma typically shows increased signal on T1-weighted images, not low signal, due to xanthomatous histiocytes. • Lesions can be expansile and lytic, often with periosteal reaction and endosteal scalloping. • Eosinophilic granuloma is a recognized cause of the 'floating teeth' appearance in the jaw. 8. Which of the following statements is CORRECT? A. Paget's disease has a prevalence of 10% in people over the age of 80 years of age B. Developmental dysplasia of the hip is more common in males C. Diffuse idiopathic skeletal hyperostosis commonly presents in children D. The highest incidence of fibrous dysplasia is between 30-50 years of age Source: Hussain, Shahid, et al. Radiology MCQs for the New FRCR Part 2A. 1st ed., TFM Publishing, 2006. Explanation: • Paget’s disease of bone becomes increasingly prevalent with age, reaching approximately 10% in those over 80 years old, and is uncommon in people under 40. • Ankylosing spondylitis is actually more common in Caucasians than Black populations, with a reported ratio of around 3:1. • Developmental dysplasia of the hip is much more common in girls, not boys. • Diffuse idiopathic skeletal hyperostosis almost always presents in adults over 50, not children. • Fibrous dysplasia has its highest incidence in childhood and adolescence, especially between ages 3 and 15, with the majority of cases presenting under age 30.
  • 125.
    EBDR Exam MCQs& Concepts July 2024 Dr. Kareem Alnakeeb 119 9. Which of the following is a feature of adamantinoma? A. Most common presentation in patients >50 years of age B. Over 90% occur in the tibia C. Osteosclerotic lesion D. Avascularity Source: Hussain, Shahid, et al. Radiology MCQs for the New FRCR Part 2A. 1st ed., TFM Publishing, 2006. Explanation: • Adamantinoma is a rare, low-grade malignant bone tumor that typically affects younger adults between 10–50 years, not those over 50. • It has a striking predilection for the tibial diaphysis, with over 90% of cases arising here and most commonly affecting the mid-shaft. • Radiologically, adamantinoma is classically osteolytic rather than sclerotic. • Although lung metastases may occur, they are seen in approximately 10% of cases—not considered ‘rare’ for this tumor type. • The lesion is not avascular; on the contrary, it typically displays prominent vascularity, which aids in distinguishing it from other bone lesions. Thus, the most characteristic feature is its marked tibial predilection. 10. Which of the following statements regarding seronegative arthritis is CORRECT? A. Ankylosing spondylitis and inflammatory bowel disease typically cause bilateral symmetrical sacroiliac joint disease B. Large joint involvement in psoriatic arthropathy is common C. The interphalangeal joint of the great toe is rarely affected in Reiter's syndrome D. Psoriatic arthropathy is associated with bilateral syndesmophytes Source: Hussain, Shahid, et al. Radiology MCQs for the New FRCR Part 2A. 1st ed., TFM Publishing, 2006. Explanation: • Bilateral symmetrical sacroiliitis is the hallmark of ankylosing spondylitis, a key seronegative spondyloarthropathy. • While psoriatic arthritis and Reiter’s syndrome often produce an asymmetrical pattern in the sacroiliac joints, ankylosing spondylitis and the spondyloarthropathy associated with inflammatory bowel disease typically cause bilateral symmetrical disease. • Psoriatic arthropathy more commonly involves small joints in a rheumatoid-like distribution and has asymmetrical, frequently unilateral syndesmophytes (not bilateral and thin). • Reiter’s syndrome more often affects the feet, especially the interphalangeal joint of the great toe, and does not show marked DIP erosions as a key feature. 11. Which of the following is a feature of periosteal osteosarcoma? A. This is the second commonest subtype of osteosarcoma B. Typically involves the diaphysis of long tubular bones C. Extension of tumor usually involves the medullary cavity D. Prognosis of this tumor is better than that of parosteal osteosarcoma Source: Hussain, Shahid, et al. Radiology MCQs for the New FRCR Part 2A. 1st ed., TFM Publishing, 2006. Explanation: • Periosteal osteosarcoma most often affects the diaphyseal regions of long bones, especially the femur and tibia. • Though aggressive, it is less common than conventional and parosteal subtypes. It is rare. Seen in 10-20-year-olds. • It rarely involves the medullary cavity, tending instead to originate and grow from the periosteal surface, with extension into adjacent soft tissues. • Its prognosis is less favourable than for parosteal osteosarcoma.
  • 126.
    EBDR Exam MCQs& Concepts July 2024 Dr. Kareem Alnakeeb 120 • Unlike osteochondromas, the tumour base is broad, not pedunculated. 12. Which of the followings is a skeletal feature of thalassemia major? A. Central nidus B. Bone sclerosis. C. Narrowing of medullary cavity D. Premature fusion of epiphysis Source: Hussain, Shahid, et al. Radiology MCQs for the New FRCR Part 2A. 1st ed., TFM Publishing, 2006. Explanation: • Erlenmeyer flask deformity, osteoporosis, and premature fusion of the epiphysis (seen in 10%) are well-recognised skeletal features in thalassaemia major. • Arthropathy can also occur, usually as a complication secondary to haemochromatosis or calcium pyrophosphate deposition disease (CPPD). However, narrowing of the medullary cavity is not a feature—instead, patients show medullary cavity expansion due to marrow hyperplasia from chronic anaemia. Recognising this distinction is key, as marrow expansion leads to characteristic bone changes in thalassaemia. 13. Concerning Brodie's abscess: Which statement is CORRECT? A. Most common in the elderly B. Tibial metaphysis is commonest location C. It is rarely associated with dense marginal sclerosis D. It has no characteristic features on MRI Source: Hussain, Shahid, et al. Radiology MCQs for the New FRCR Part 2A. 1st ed., TFM Publishing, 2006. Explanation: • Brodie’s abscess is a subacute or chronic form of osteomyelitis, most commonly affecting children and adolescents. o The tibial metaphysis is the classic location. o It appears radiographically as a lytic lesion with well-defined sclerotic margins. o Staphylococcus aureus is the most common causative organism. o MRI may show a “double-line” sign due to granulation tissue and surrounding sclerosis. • Ewing sarcoma and metastatic neuroblastoma usually lack dense marginal sclerosis. • Osteoid osteoma is typically smaller and presents with intense night pain, while chondrosarcoma is rare in this age group and site. 14. Regarding hyperparathyroidism (HPT): Which statement is CORRECT? A. Brown tumors occur more frequently in secondary HPT B. Rugger Jersey spine occurs more frequently in primary HPT C. Increased incidence of slipped upper femoral epiphysis is associated with HPT D. Abnormal bone scan in about 80% Source: Hussain, Shahid, et al. Radiology MCQs for the New FRCR Part 2A. 1st ed., TFM Publishing, 2006. Explanation: • Chondrocalcinosis, due to calcium pyrophosphate deposition, is seen in about 15-20% of patients with hyperparathyroidism, especially in secondary HPT. • Brown tumours are more common in primary HPT, not secondary. The "rugger jersey spine" sign is classically associated with secondary (renal) hyperparathyroidism due to chronic renal failure, not primary HPT. There is an increased risk of slipped upper femoral epiphysis in HPT. Most cases of HPT show abnormal bone scan findings.
  • 127.
    EBDR Exam MCQs& Concepts July 2024 Dr. Kareem Alnakeeb 121 15. Regarding ultrasound of soft tissue masses: Which statement is CORRECT? A. Ganglion cysts show sharp posterior acoustic shadowing. B. Superficial masses are best examined with a 9-13 MHz Frequency Linear Transducer C. Schwannomas can be differentiated from neurofibromas by ultrasound appearances D. Lipomas commonly have increased vascularity Source: Hussain, Shahid, et al. Radiology MCQs for the New FRCR Part 2A. 1st ed., TFM Publishing, 2006. Explanation: • Ganglion cysts classically appear as anechoic or hypoechoic lesions with well-defined margins and often demonstrate posterior acoustic enhancement, which helps distinguish them from solid masses. They frequently communicate with adjacent tendon sheaths. • Superficial soft tissue masses are best evaluated with high-frequency (9-13 MHz) linear transducers rather than low-frequency convex probes. • Schwannomas and neurofibromas generally have overlapping sonographic features and cannot reliably be differentiated by ultrasound alone. • Lipomas typically appear as hyperechoic or isoechoic masses with little or no vascularity on Doppler imaging. 16. Which of the followings is a feature of Paget's disease? A. The skull is most commonly affected B. Increased density of vertebra - 'ivory vertebra' C. Mostly unilateral D. Sarcomatous transformation in 10-15% Source: Hussain, Shahid, et al. Radiology MCQs for the New FRCR Part 2A. 1st ed., TFM Publishing, 2006. Explanation: • The pelvis is the most frequently affected site in Paget’s disease, involved in roughly 75% of patients. • Ivory vertebra (a dense vertebra) can be seen due to increased bone turnover. • Thickening of the ileopectineal line is a common radiographic finding reflecting pelvic involvement. • Candle flame lysis refers to a V-shaped lytic lesion in long bones, not sclerosis. • Sarcomatous transformation is a rare complication, occurring in less than 1% of cases—10-15% would be uncharacteristically high. 17. The radiograph of an 8-year-old boy with dietary Vitamin D deficiency reveals cupping and fraying of the distal tibial metaphysis. Which radiological finding is a recognized feature of this condition? A. Cortical sclerosis involving the margin of the epiphysis B. Expansion of the costochondral junctions C. Exuberant periosteal reaction D. Metaphyseal spurs Source: McQueen, Andrew S., et al. Grainger & Allison’s Diagnostic Radiology: Single Best Answer MCQs. 5th ed., Churchill Livingstone, 2009. Explanation: • Sclerosis of the margins of the epiphysis (Wimberger sign), metaphyseal (Pelcan) spurs, dense metaphyseal lines (white lines of Frankel) and exuberant periosteal reactions (secondary to recurrent subperiosteal bleeding) are features of scurvy. • Expansion of the costochondral junctions results in the characteristic appearance of a rachitic rosary.
  • 128.
    EBDR Exam MCQs& Concepts July 2024 Dr. Kareem Alnakeeb 122 18. A 43-year-old man is investigated for pain related to his left arm. Plain radiography demonstrates a well- defined, lytic lesion in the proximal humerus, with chondroid matrix mineralization and a wide zone of transition. What is the MOST likely diagnosis? A. Chondroblastoma B. Chondroma C. Chondrosarcoma D. Osteochondroma Source: McQueen, Andrew S., et al. Grainger & Allison’s Diagnostic Radiology: Single Best Answer MCQs. 5th ed., Churchill Livingstone, 2009. 19. What are the MOST likely radiological findings in a 'Monteggia fracture ? A. A fracture of the distal radius with an associated dislocation of the radial B. A fracture of the distal radius with an associated disruption of the distal radioulnar joint C. A fracture of the distal ulna with an associated dislocation of the radial head D. A fracture of the proximal radius with an associated disruption of the distal radioulnar joint Source: McQueen, Andrew S., et al. Grainger & Allison’s Diagnostic Radiology: Single Best Answer MCQs. 5th ed., Churchill Livingstone, 2009. Explanation (from Radiology MCQs for the new FRCR Part 2A): • Monteggia fracture - proximal ulnar fracture with dislocation of the radial head • Galleazi fracture - distal radial fracture with dislocation of distal radioulnar joint • Chauffer’s fracture - triangular fracture of radial styloid process Mnemonic: • MUGR (pronounced “MUGGER”): identifies the fractured bone. o MU: Monteggia Ulna o GR: Galeazzi Radius • Use “A to Z” to remember the location of the fracture-dislocation: o ‘A’is proximal (MonteggiA) - Radial head dislocation and proximal ulna fracture o ‘Z’ is distal (GaleazZi) - Distal radioulnar joint dislocation and distal radius fracture • Segond fracture - cortical avulsion fracture of proximal lateral tibia • Pott’s fracture - fibula fracture above an intact tibiofibular ligament 20. Which of the followings is an indicator of fracture instability of the lumbar spine? A. Compression fracture greater than 20% of vertebral body height B. Fracture of the posterior elements C. Increased inter-pediculate distance. D. Associated pre-vertebral hematoma. Explanation: (by Perplexity AI, generated with the OpenAI O3 model) • Both fracture of the posterior elements and increased inter-pediculate distance (as a sign of burst fracture) are strong indicators of lumbar spine fracture instability because they both directly or indirectly imply compromise of the middle and/or posterior columns, which are critical for spinal stability. However, "Fracture of the posterior elements" directly refers to the disruption of one of the key anatomical columns responsible for spinal integrity. 21. A 6-week-old boy has a positive family history of developmental dysplasia of the hip (DDH) and is referred for a hip ultrasound. Which imaging features would be consistent with normal (Graf type 1) hips? A. Alpha angle greater than 60° B. Beta angle greater than 77° C. Compressed cartilage roof D. Deficient bony roof Source: McQueen, Andrew S., et al. Grainger & Allison’s Diagnostic Radiology: Single Best Answer MCQs. 5th ed., Churchill Livingstone, 2009.
  • 129.
    EBDR Exam MCQs& Concepts July 2024 Dr. Kareem Alnakeeb 123 22. A 28-year-old tennis player undergoes a MR arthrogram to investigate recurrent right shoulder instability following a previous glenohumeral dislocation. The MRI reveals a tear of the anterosuperior labrum, closely related to the insertion of the biceps tendon. How are these appearances BEST described? A. Bankart lesion B. Hill-Sachs lesion C. Reverse Hill-Sachs lesion D. Superior labrum from anterior to posterior (SLAP) lesion Source: McQueen, Andrew S., et al. Grainger & Allison’s Diagnostic Radiology: Single Best Answer MCQs. 5th ed., Churchill Livingstone, 2009. 23. An asymptomatic 65-year-old woman on long-term steroids for rheumatoid disease undergoes dual energy X-ray absorptiometry (DXA). Her Z score is —2 and her T score is —2.7. What is the WHO definition of osteoporosis? A. T score less than —1 B. T score less than —2.5 C. Z score less than —2.5 D. Mean of T and Z score less than —2 Source: McQueen, Andrew S., et al. Grainger & Allison’s Diagnostic Radiology: Single Best Answer MCQs. 5th ed., Churchill Livingstone, 2009. Explanation • Bone density can be measured in relation to an age and sex-matched population (Z score) or in relation to a population of young adults of the same sex (T score). • The WHO defines osteoporosis as a T score less than —2.5, therefore relating bone mineral density to sex-matched peak bone mass. 24. A 13-year-old boy with a short history of pain and swelling around his left elbow. The radiograph reveals a 4-cm area of permeative bone destruction within the distal diaphysis of the left humerus, with a wide zone of transition. There is an extensive associated soft tissue component and evidence of a 'hair-on-end' pattern of periosteal reaction. What is MOST likely diagnosis? A. Chondroblastoma B. Chondromyxoid fibroma C. Ewing's sarcoma D. Malignant fibrous histiocytoma Source: McQueen, Andrew S., et al. Grainger & Allison’s Diagnostic Radiology: Single Best Answer MCQs. 5th ed., Churchill Livingstone, 2009. Explanation • Infection and Langerhans cell histiocytosis (LCH) should be considered in the differential diagnosis of a permeative bone lesion in a child. • Askin tumour is a rare primitive neuroectodermal tumour of the chest wall in children.
  • 130.
    EBDR Exam MCQs& Concepts July 2024 Dr. Kareem Alnakeeb 124 25. An 80-year-old man undergoes skeletal scintigraphy for multifocal skeletal pain, malaise and weight loss. The scintigram shows diffusely increased activity throughout die skeleton, with absent renal activity. What is the MOST likely diagnosis? A. Metastatic bladder cancer B. Metastatic colon cancer C. Metastatic lung cancer D. Metastatic prostate cancer Source: McQueen, Andrew S., et al. Grainger & Allison’s Diagnostic Radiology: Single Best Answer MCQs. 5th ed., Churchill Livingstone, 2009. 26. What is the MOST likely appearance of a 'hangman's fracture' on plain film? A. Fractures through the neural arch of Cl B. Fractures through the neural arch of C2 C. Fracture of the spinous process of C7 D. Transverse fracture through the base of the dens Source: McQueen, Andrew S., et al. Grainger & Allison’s Diagnostic Radiology: Single Best Answer MCQs. 5th ed., Churchill Livingstone, 2009. 27. On MRI of the shoulder joint, which pattern of imaging features is compatible with a partial thickness supraspinatus tendon? A. A gap between the distal and proximal portions of the tendon, with retraction of the proximal tendon B. Areas of increased signal on T1 and T2 images C. Areas of increased signal on Tl and T2 images, extending across the full thickness of the tendon D. Low signal on all sequences Source: McQueen, Andrew S., et al. Grainger & Allison’s Diagnostic Radiology: Single Best Answer MCQs. 5th ed., Churchill Livingstone, 2009. 28. MRI of the right knee due to clinical suspicion of an acute rupture of the ACL. The ACL is indistinct, and cannot be visualized. Which additional features would be supportive of a diagnosis of ACL rupture? A. Buckling of the PCL B. Oedema within the medial collateral ligament C. Posterior translation of the femur on the tibial condyles D. Tear of the medial meniscus Source: McQueen, Andrew S., et al. Grainger & Allison’s Diagnostic Radiology: Single Best Answer MCQs. 5th ed., Churchill Livingstone, 2009. 29. Radiographs of the left knee for 30-year-old man shows several small articular erosions and MRI reveals foci of low T2/T2* signal intensity within the synovium. Which is the MOST likely diagnosis? A. Calcium pyrophosphate arthropathy B. Pigmented villonodular synovitis C. Psoriatic arthropathy D. Synovial chondromatosis Source: McQueen, Andrew S., et al. Grainger & Allison’s Diagnostic Radiology: Single Best Answer MCQs. 5th ed., Churchill Livingstone, 2009.
  • 131.
    EBDR Exam MCQs& Concepts July 2024 Dr. Kareem Alnakeeb 125 30. Defective osteoclastic function is a predominant feature of which disease? A. Osteomalacia B. Osteopetrosis C. Osteoporosis D. Rickets Source: McQueen, Andrew S., et al. Grainger & Allison’s Diagnostic Radiology: Single Best Answer MCQs. 5th ed., Churchill Livingstone, 2009. 31. Which clinical or radiological feature would favor a diagnosis of chondrosarcoma rather than enchondroma? A. Age less than 20 years B. Circular, curvilinear or nodular calcific densities C. Periosteal reaction D. Slow growth Source: McQueen, Andrew S., et al. Grainger & Allison’s Diagnostic Radiology: Single Best Answer MCQs. 5th ed., Churchill Livingstone, 2009. Explanation • Outside the hands and feet, chondrosarcoma is five times more common than enchondroma. 32. The radiograph of a 40-year-old man with a painful knee shows multiple calcified loose bodies, each of similar size, within the joint. The joint space is preserved. What diagnosis is MOST likely? A. Gout B. Pigmented villonodular synovitis C. Rheumatoid arthritis D. Synovial osteochondromatosis Source: McQueen, Andrew S., et al. Grainger & Allison’s Diagnostic Radiology: Single Best Answer MCQs. 5th ed., Churchill Livingstone, 2009. 33. Which radiological feature would favor a diagnosis of rheumatoid rather than psoriatic arthritis? A. Early reduction in bone mineralization B. Joint ankylosis C. Pencil-in-cup deformities of the middle phalanges D. Periosteal reaction Source: McQueen, Andrew S., et al. Grainger & Allison’s Diagnostic Radiology: Single Best Answer MCQs. 5th ed., Churchill Livingstone, 2009. Explanation • Juxta-articular osteopenia is one of the earliest radiographic abnormalities in rheumatoid arthritis, distinguishing it from psoriatic arthropathy, in which bone mineral density is preserved until late in the disease process. 34. MRI ankle joint following trauma showed that the Achilles tendon has a convex anterior margin with small linear area of increased signal within the tendon on T2- and T2*-weighted images. What is the MOST likely diagnosis? A. Achilles para-tendonitis B. Achilles tendinosis C. Achilles tendinosis with complete tear D. Achilles tendinosis with partial tear Source: McQueen, Andrew S., et al. Grainger & Allison’s Diagnostic Radiology: Single Best Answer MCQs. 5th ed., Churchill Livingstone, 2009.
  • 132.
    EBDR Exam MCQs& Concepts July 2024 Dr. Kareem Alnakeeb 126 35. The plain radiograph reveals a wrist fracture extending through the epiphysis and into the metaphysis in 9-year-old boy. How would this injury be classified in the Salter-Harris classification? A. Type I B. Type II C. Туре III D. Type IV Source: McQueen, Andrew S., et al. Grainger & Allison’s Diagnostic Radiology: Single Best Answer MCQs. 5th ed., Churchill Livingstone, 2009. 36. A series of neonatal radiographs reveal a narrow thorax with short ribs, square iliac wings with horizontal acetabular roofs, short sacrosciatic notches, progressive narrowing of the interpedicular distance and posterior scalloping of the vertebral bodies. What is the MOST likely diagnosis? A. Achondroplasia B. Campomelic dysplasia C. Cleidocranial dysplasia D. Morquio's syndrome Source: McQueen, Andrew S., et al. Grainger & Allison’s Diagnostic Radiology: Single Best Answer MCQs. 5th ed., Churchill Livingstone, 2009. Explanation • The iliac wings in Morquio's syndrome are characteristically flared rather than square. 37. A 19-year-old female student presents with acute abdominal pain, elevated CRP and a low-grade temperature. On clinical examination, there is tenderness to light palpation in the right iliac fossa and the patient is febrile. A graded compression ultrasound examination is performed. Which one of the following statements is TRUE? A. A transverse appendiceal diameter of 5 mm is diagnostic of acute appendicitis. B. The finding of a pelvic fluid collection makes a diagnosis of acute appendicitis unlikely. C. The presence of hyperechoic fat in the right iliac fossa makes diagnosis of acute appendicitis unlikely. D. The sensitivity of graded compression ultrasound in suspected acute appendicitis is 75—90%. Source: McQueen, Andrew S., et al. Grainger & Allison’s Diagnostic Radiology: Single Best Answer MCQs. 5th ed., Churchill Livingstone, 2009. Explanation • Graded compression ultrasound of the appendix can avoid unnecessary surgery and ionising radiation—particularly relevant for children and women of childbearing age. • The finding of a noncompressible appendix with transverse diameter of 6 mm or greater is highly suggestive of acute appendicitis (specificity 86-100%). • Other ultrasound findings include hyperechoic fat in the right iliac fossa, periappendiceal fluid or a pelvic fluid collection (appendiceal abscess). 38. Which plain radiographic finding would be MOST suggestive of a toxic megacolon? A. Caecum measuring 4.5 cm in diameter B. Multiple mucosal islands in a dilated transverse colon C. Thickened haustra throughout the entire colon D. 'Thumbprinting' of the transverse and descending colon Source: McQueen, Andrew S., et al. Grainger & Allison’s Diagnostic Radiology: Single Best Answer MCQs. 5th ed., Churchill Livingstone, 2009. Explanation • The presence of severe ulceration leading to mucosal islands is a major sign of toxic megacolon (the other key finding is colonic dilatation > 5 cm).
  • 133.
    EBDR Exam MCQs& Concepts July 2024 Dr. Kareem Alnakeeb 127 39. Abdominal ultrasound for 37-year-old male patient with right upper quadrant pain and the patient is afebrile, US shows a 5-cm diameter cystic lesion in the right lobe of liver. The mass contains multiple septations with a large cyst centrally and multiple small cystic spaces peripherally. Echogenic debris is seen within the cystic lesion and alters in position when the patient lies on his side. What is the MOST likely diagnosis? A. Amoebic abscess B. Hydatid cyst C. Pyogenic liver abscess D. Simple liver cyst Source: McQueen, Andrew S., et al. Grainger & Allison’s Diagnostic Radiology: Single Best Answer MCQs. 5th ed., Churchill Livingstone, 2009. Explanation • A multiloculated cystic mass with daughter cysts and echogenic debris (‘hydatid sand’) is characteristic of a hydatid liver cyst. 40. A 23-year-old woman complains of episodes of diarrhea and rectal bleeding. Her father died of colorectal cancer aged 39. A double contrast barium enema is performed and demonstrates more than one hundred small polyps, measuring up to 5 mm in size, throughout the colon. An upper GT endoscopy demonstrates multiple polypoid lesions in the stomach and duodenum. What is the MOST likely diagnosis? A. Carcinoid syndrome B. Familial adenomatous polyposis C. Juvenile polyposis D. Peutz-Jegher's syndrome Source: McQueen, Andrew S., et al. Grainger & Allison’s Diagnostic Radiology: Single Best Answer MCQs. 5th ed., Churchill Livingstone, 2009. Explanation • Autosomal dominant condition with multiple colonic adenomas and 100% risk of colorectal carcinoma 20 years after diagnosis. Associated with hamartomas in the stomach, gastric and duodenal adenomas and periampullary carcinoma. 41. Which of the following findings would make a diagnosis of fibrolamellar carcinoma more likely than that of focal nodular hyperplasia (FNH)? A. A hyperechoic central scar B. Delayed enhancement of a central scar C. Punctuate calcification in the lesion D. The patient is taking the combined oral contraceptive pill Source: McQueen, Andrew S., et al. Grainger & Allison’s Diagnostic Radiology: Single Best Answer MCQs. 5th ed., Churchill Livingstone, 2009. Explanation • There is considerable overlap in the imaging appearances of these two conditions, but punctate calcification occurs in over half of patients with fibrolamellar carcinoma and is extremely unusual in FNH. 42. MRCP examination for 47-year-old woman with obstructive jaundice shows a smooth stricture in the mid- common bile duct with associated moderate intrahepatic biliary dilatation. The stricture is caused by extrinsic compression from a round filling defect within the cystic duct. What is the diagnosis? E. Acute bacterial cholangitis F. Gallbladder carcinoma G. Mirizzi syndrome H. Primary sclerosing cholangitis (PSC) Source: McQueen, Andrew S., et al. Grainger & Allison’s Diagnostic Radiology: Single Best Answer MCQs. 5th ed., Churchill Livingstone, 2009. Explanation
  • 134.
    EBDR Exam MCQs& Concepts July 2024 Dr. Kareem Alnakeeb 128 • In Mirizzi syndrome, a gallstone in the cystic duct produces mass effect on the common duct and can lead to fistula formation. 43. Which finding would indicate a nonresectable pancreatic tumor? A. The pancreatic duct dilated to 6 mm B. The presence of a 5-mm coeliac axis lymph node C. The tumor has invaded the duodenum D. The tumor in contact with 75% of the superior mesenteric artery Source: McQueen, Andrew S., et al. Grainger & Allison’s Diagnostic Radiology: Single Best Answer MCQs. 5th ed., Churchill Livingstone, 2009. Explanation • If the tumour is in contact with more than half of the vessel circumference, it is very unlikely to be resectable. 44. Which one of the following statements is TRUE regarding pseudomembranous colitis? A. A normal abdominal CT effectively excludes pseudomembranous colitis. B. Ascites is present in up to 40% of patients. C. Extensive pericolonic stranding is a typical feature on CT. D. The rectum is not involved in 40—50% of patients. Source: McQueen, Andrew S., et al. Grainger & Allison’s Diagnostic Radiology: Single Best Answer MCQs. 5th ed., Churchill Livingstone, 2009. Explanation • Ascites can occur with other colitides, but is often seen in pseudomembranous colitis. • CT typically demonstrates mucosal enhancement and marked colonic wall thickening but only mild pericolonic stranding, in patients with pseudomembranous colitis. These findings have a high positive predictive value but a normal CT does not exclude pseudomembranous colitis. • Rectal sparing occurs in around 10% of patients. 45. Which of the following features would be MOST consistent with intestinal polyp? A. The lesion contains a locule of gas at its base. B. The lesion has a mean density of — 150 HU. C. The lesion is of homogeneous attenuation. D. There are diverticulae seen in the sigmoid colon. Source: McQueen, Andrew S., et al. Grainger & Allison’s Diagnostic Radiology: Single Best Answer MCQs. 5th ed., Churchill Livingstone, 2009. Explanation • A polyp will usually demonstrate uniform soft tissue density, similar to the surrounding bowel wall. 46. A 64-year-old woman with past medical history of gallstones. Abdominal ultrasound demonstrates a 6 x 4 cm mixed echogenicity lesion in the gallbladder fossa, with the gallbladder not visualized. On CT, the gallbladder fossa mass demonstrates central low attenuation with peripheral enhancement and mild intrahepatic biliary dilatation. Low attenuation lymph nodes are present at the porta hepatis (measuring up to 1.5 cm short axis). Which diagnosis is MOST likely? A. Adenomyomatosis B. Gallbladder carcinoma C. Hepatocellular carcinoma D. Xanthogranulomatous cholecystitis Source: McQueen, Andrew S., et al. Grainger & Allison’s Diagnostic Radiology: Single Best Answer MCQs. 5th ed., Churchill Livingstone, 2009. Explanation • A gallbladder fossa mass with little/no visible normal gallbladder and hilar biliary obstruction is highly suggestive of gallbladder carcinoma.
  • 135.
    EBDR Exam MCQs& Concepts July 2024 Dr. Kareem Alnakeeb 129 47. Which CT finding would be more suggestive of chronic pancreatitis than ductal pancreatic adenocarcinoma? A. Common bile duct dilatation B. Focal enlargement of the pancreatic head C. Intraductal pancreatic calcification D. Peripancreatic fat stranding and ascites Source: McQueen, Andrew S., et al. Grainger & Allison’s Diagnostic Radiology: Single Best Answer MCQs. 5th ed., Churchill Livingstone, 2009. Explanation • Intraductal calcification may be focal or diffuse and is not seen in all patients with chronic pancreatitis. When it is present, however, it is a highly reliable sign of chronic pancreatitis. 48. Which statement regarding acute pancreatitis is CORRECT? A. Mumps is a recognized cause B. Pancreatic oedema is a late sign C. Pancreatic necrosis demonstrated on CT is associated with a mortality of 5-10% D. Right-sided pleural effusion is seen in 5% Source: Hussain, Shahid, et al. Radiology MCQs for the New FRCR Part 2A. 1st ed., TFM Publishing, 2006. Explanation: • Mumps is a recognized although rare cause of acute pancreatitis, alongside more common causes such as gallstones and alcohol. • Pancreatic oedema is often an early sign on imaging, not a late one. • Pancreatic necrosis seen on CT is linked to mortality rates exceeding 20%, not just 5–10%. • Pleural effusions in pancreatitis are typically left-sided, not right-sided. • Hemorrhagic pancreatitis is characterized on CT by hyperdense areas measuring 50–70 HU, not hypodense regions of 5–20 HU. 49. Which of the following statements regarding pancreatic carcinoma is CORRECT? A. 60-70% of pancreatic carcinomas arise in the tail B. They are usually hypovascular C. Calcification is common D. On ultrasound appears as a hyperechoic pancreatic mass Source: Hussain, Shahid, et al. Radiology MCQs for the New FRCR Part 2A. 1st ed., TFM Publishing, 2006. Explanation: • The majority of pancreatic carcinomas (60-70%) arise in the head of the pancreas, not the tail. • Pancreatic carcinomas are typically hypovascular on contrast-enhanced imaging. • Calcification is uncommon in pancreatic adenocarcinoma, occurring in only a small percentage of cases. • Contiguous organ invasion is relatively common, especially in locally advanced disease, with invasion documented in up to 40% of cases. • On ultrasound, pancreatic carcinomas usually appear hypoechoic rather than hyperechoic. 50. Regarding porcelain gallbladder: (True or False) F. It is often symptomless G. It is rarely associated with gallstones H. Oral cholecystogram shows a non-functioning gallbladder I. 60-70% develop carcinoma of the gallbladder J. Acute pancreatitis is a recognised cause Source: Hussain, Shahid, et al. Radiology MCQs for the New FRCR Part 2A. 1st ed., TFM Publishing, 2006. Explanation: • A. True
  • 136.
    EBDR Exam MCQs& Concepts July 2024 Dr. Kareem Alnakeeb 130 • B. False - 90% • C. True • D. False - 10-20% • E. False 51. Which of the following statements regarding splenic lymphoma is CORRECT? A. The spleen is involved at presentation in 30-40% of patients with non Hodgkin's lymphoma B. Focal splenic deposits are usually well defined, round lesions of increased brightness on ultrasound C. When there is lymphomatous involvement of the spleen, splenomegaly is seen in 70-80% D. Lymph nodes are seen in the splenic hilum in 50% of patients with Hodgkin's lymphoma Source: Hussain, Shahid, et al. Radiology MCQs for the New FRCR Part 2A. 1st ed., TFM Publishing, 2006. Explanation: • Slightly higher for Hodgkin's lymphoma. • Focal splenic lymphoma deposits are most commonly well-defined, rounded, hypoechoic lesions on ultrasound, reflecting their cellular composition and infiltration pattern • Splenomegaly is seen in about 50% of cases. • Splenic lymphoma deposits commonly calcify • Lymph nodes at the splenic hilum are uncommon in Hodgkin’s lymphoma. 52. Regarding Budd-Chiari syndrome: Which statement is FALSE? A. It can be caused by obstruction of the supra-hepatic IVC B. The caudate lobe is markedly atrophic C. On MRI images 'comma-shaped' intrahepatic collateral vessels are seen D. A 'spider's web' appearance at hepatic venography is characteristic Source: Hussain, Shahid, et al. Radiology MCQs for the New FRCR Part 2A. 1st ed., TFM Publishing, 2006. Explanation: • Suprehepatic IVC obstruction is Primary cause of Budd-Chiari syndrome; secondary (hepatic vein thrombosis) more common. • Early CT: central liver prominent, “Flip-flop” pattern: central enhancement early, peripheral enhancement late. • The caudate lobe is typically enlarged—not atrophic—because it drains directly into the IVC. • Intrahepatic collateral vessels are seen, not absent, and may have a ‘comma’ shape on MRI. • The ‘spider’s web’ appearance at hepatic venography is characteristic of Budd-Chiari syndrome and results from multiple small collateral channels forming due to hepatic venous outflow obstruction. 157. The following statements concerning esophageal carcinoma are true: A. 90% of cases are squamous cell carcinomas B. Most commonly located in the upper third of the esophagus C. Plummer-Vinson syndrome is a recognised predisposing factor D. Commonest appearance on double contrast barium swallow is of a large ulcer within a bulging mass E. It is associated with ulcerative colitis Source: Hussain, Shahid, et al. Radiology MCQs for the New FRCR Part 2A. 1st ed., TFM Publishing, 2006. Explanation: A. True B. False - 20% in the upper third, 30-40% middle third and 30-40% in lower third C. True D. False - polypoid/fungating form is commonest E. False - predisposing factors include Barrett’s oesophagus, alcohol abuse, smoking, coeliac disease, achalasia, tylosis
  • 137.
    EBDR Exam MCQs& Concepts July 2024 Dr. Kareem Alnakeeb 131 53. Regarding diverticular disease: Which is FALSE? A. Colonic diverticulosis affects 70-80% by 80 years of age B. 10-25% of individuals with colonic diverticular disease develop diverticulitis C. Rectosigmoid colon is most commonly affected D. Fistula formation occurs in 40-50% of cases complicating acute diverticulitis Source: Hussain, Shahid, et al. Radiology MCQs for the New FRCR Part 2A. 1st ed., TFM Publishing, 2006. Explanation: • Colonic diverticulosis is very common in the elderly, affecting 70–80% of individuals by 80 years. • Fistula formation is much less common, seen in approximately 14% of complicated diverticulitis cases, not 40–50%. • The rectosigmoid colon—not the ascending colon—is most commonly involved. • Roughly 10–25% of those with diverticulosis develop diverticulitis. • Moderate diverticulitis is typically defined by bowel wall thickening over 3mm, not 6mm. 54. The MOST common site of gastro-intestinal stromal tumors (GIST) is: A. Esophagus. B. Stomach. C. Small intestine D. Colon. Source: Hussain, Shahid, et al. Radiology MCQs for the New FRCR Part 2A. 1st ed., TFM Publishing, 2006. Explanation: • Tumor size is regarded as the most significant criterion for predicting malignant potential in gastrointestinal stromal tumors (GISTs). • The most common location for GISTs is the stomach, not the sigmoid colon. • GISTs can cause gastrointestinal bleeding but are not a common cause of hematemesis. • Contrast enhancement of GISTs is typically heterogeneous due to hemorrhage, necrosis, or cystic change. • There is a recognized association between GISTs and neurofibromatosis Type 1. 55. Chron's disease of small intestine: A. Terminal ileum is affected in 80% of case. B. Colon is affected in 10% of case. C. Treated surgically. D. Presenting with deep penetrating ulcers. Explanation: (by Perplexity AI, generated with the OpenAI O3 model) • Terminal ileum involvement ≈ 70-80% ➜ True. • Colonic disease only ≈ 20% (±50% ileocolonic) ➜ “10% colon” statement is False. • Management today is medical first (steroids, immunomodulators, biologics). Surgery only for strictures, fistulae, perforation, bleeding or refractory disease ➜ “treated surgically” is False. • Transmural inflammation gives deep, fissuring / penetrating ulcers—cobblestone mucosa ➜ True. 56. Caudate lobe to right lobe ratio that is specific for liver cirrhosis is: A. More than 0.73 B. Less than 0.55 C. Less than 0.60 D. Less than 0.45 Source: Unknown Explanation: • The right lobe and medial segment of left lobe are atrophied. The caudate lobe is not affected. • There is compensatory hypertrophy of the lateral segments of the left lobe.
  • 138.
    EBDR Exam MCQs& Concepts July 2024 Dr. Kareem Alnakeeb 132 • Caudate lobe/right lobe >0.65. • Quadrate lobe < 3 cm. • Liver enlarged in early stages and atrophied in lateral stages. The surface is nodular. • The GB angle is normally more than 40 degrees. It is less than 35 degrees in cirrhosis. 57. Fat within hepatic focal lesion on MRI is seen in: A. НСС B. Focal nodular hyperplasia C. Cholangiocarcinoma D. Hemangioma E. Metastasis Explanation: (by Perplexity AI, generated with the OpenAI O3 model) • Hepatocellular carcinoma (HCC) is the primary liver lesion most likely to show intralesional fat on MRI, due to fatty metamorphosis within the tumor. Fat can be detected as a signal drop out on out- of-phase imaging. • Focal nodular hyperplasia and hemangiomas do not characteristically contain fat (although FNH contains a central scar and hemangiomas are blood-filled). • Cholangiocarcinoma very rarely demonstrates fat, and while certain metastases may mimic this, the classic primary is HCC, especially in non-cirrhotic livers. Key distractors are common hepatic masses, but fat within them is distinctly unusual. 58. linitis plastica of the stomach is seen in: A. Gastric bezoar. B. Gastric carcinoma. C. Peutz-Jeghers syndrome. D. Gastric polyps E. Gastric lymphoma Explanation: (by Perplexity AI, generated with the OpenAI O3 model) • Linitis plastica refers to a diffuse infiltrative process that causes thickening and stiffening of the stomach wall, resulting in a rigid, nondistensible stomach often referred to as “leather bottle” stomach. This pattern is most commonly associated with diffuse-type gastric carcinoma, particularly signet ring cell carcinoma, due to malignant infiltration of the submucosa and muscularis. • Gastric bezoar and gastric polyps do not cause this diffuse infiltrative change. • Peutz-Jeghers syndrome is associated with hamartomatous polyps but not linitis plastica. • Gastric lymphoma can cause thickening but classically presents as large folds or mass-like lesions, rather than uniform rigidity. 59. The characteristic feature of hepatic focal nodular hyperplasia on MRI is: A. Central scar retains contrast in delayed phase and poor enhancement in arterial phase B. Central scar shows wash out of contrast in delayed phase C. Hyperintense central scar on Tl D. Cystic changes E. Hyperintense central scar on T2 Explanation: (by Perplexity AI, generated with the OpenAI O3 model) • The classic imaging hallmark of hepatic focal nodular hyperplasia (FNH) on MRI is a central scar that demonstrates delayed contrast retention, meaning it enhances in the delayed phase after gadolinium administration, while showing poor enhancement in the arterial phase. The scar typically appears hyperintense on T2-weighted images, but not on T1.
  • 139.
    EBDR Exam MCQs& Concepts July 2024 Dr. Kareem Alnakeeb 133 • Cystic changes are not a feature of FNH; these are more characteristic of other hepatic lesions such as hepatocellular carcinoma or cystic metastases. • "Wash out" of contrast in the delayed phase is not typical for the FNH central scar; instead, washout is more suggestive of hepatic adenomas or malignancy. 60. A 60-year-old presents with left groin pain. Ultrasound shows a 2 cm hypoechoic lesion bulging medial to the epigastric vessels on Valsalva manoeuvre and absent on rest. What is the most likely diagnosis? A. Direct inguinal hernia B. Indirect inguinal hernia C. Obturator hernia D. Spigelian hernia E. Femoral hernia Source: Gupta, Chaitanya. 300 Single Best Answers for the Final FRCR Part A. 1st ed., Jaypee UK, 2010. Explanation: • A direct inguinal hernia is seen medial to the inferior epigastric vessels whereas an indirect hernia is seen lateral to them. 61. Central dot sign is seen in: A. Caroli disease B. Primary sclerosing cholangitis C. Polycystic liver disease D. Liver hamartoma Explanation: (by Perplexity AI, generated with the OpenAI O3 model) • The central dot sign is an enhancing portal venous radicle and fibrovascular bundle seen within fluid-filled, ectatic intrahepatic bile ducts; it is virtually pathognomonic for Caroli’s disease because the ducts are massively dilated yet remain in continuity with portal triads. • Primary sclerosing cholangitis produces multifocal strictures and beading without a central enhancing dot. • Simple hepatic cysts are avascular, with thin walls and no intraluminal structures. • Hepatic mesenchymal hamartoma appears as a multicystic mass lacking patent biliary connections. • Cavernous hemangiomas show peripheral nodular enhancement progressing centrally, not a single central dot. 62. On ultrasonographic examination, diffuse thickening of gall bladder with hyperechoic shadow at neck and comet tailing is seen in: A. Xanthogranulomatous cholecystitis B. Adenomyomatosis C. Adenomyomatous polyps D. Cholesterol crystals Explanation: (by Perplexity AI, generated with the OpenAI O3 model) • Adenomyomatosis is a benign hyperplastic condition of the gallbladder characterized by mural thickening and formation of Rokitansky–Aschoff sinuses. Ultrasonography typically reveals diffuse or segmental wall thickening, with comet tail or ring-down artefact resulting from cholesterol crystals trapped within the sinuses, most often seen at the gallbladder neck. • Xanthogranulomatous cholecystitis produces heterogeneous wall thickening but lacks comet tail artefacts.
  • 140.
    EBDR Exam MCQs& Concepts July 2024 Dr. Kareem Alnakeeb 134 • Adenomyomatous polyps refer to focal instead of diffuse changes and do not exhibit classic comet tail artefact. • Cholesterol crystals can contribute to comet tailing, but they most commonly present as echogenic foci without diffuse wall thickening. 63. Retroperitoneal lymphoma: A. It is the most common retroperitoneal malignancy, accounting for up to one-third of all cases. B. On CT, it is usually seen as well-defined para-aortic or pelvic non-enhancing masses with areas of necrosis C. Vascular invasion is one of its characteristic features D. It is almost always associated with mediastinal lymphadenopathy E. It typically calcifies before treatment. Explanation: (by Perplexity AI, generated with the OpenAI O3 model) • Retroperitoneal lymphoma is indeed the most common primary retroperitoneal malignancy, responsible for up to one-third of such cases. • It usually presents as well-defined, often homogenous soft-tissue masses predominantly in para- aortic or pelvic locations. However, these masses usually ENCASE vessels rather than invade them directly, which distinguishes lymphoma from other retroperitoneal malignancies—direct vascular invasion is uncharacteristic. • Although mediastinal lymphadenopathy may occur, it is not an almost constant feature. • Areas of necrosis are uncommon unless the patient has been treated, and calcification typically occurs only after therapy rather than beforehand. 64. Large-bowel obstruction is diagnosed at CT if: A. There is colonic wall thickening and luminal dilation more than 4 cm B. The large bowel is dilated with no dilation of the small bowel C. The colon is dilated (colon diameter > 5.5 cm, cecum diameter > 10 cm) and filled with feces, gas, and fluid proximal to an abrupt transition point, after which the colon is collapsed distally D. Dilated large bowel with associated ileus Explanation: (by Perplexity AI, generated with the OpenAI O3 model) • Large-bowel obstruction on CT is characterized by pronounced colonic dilatation (colon >5.5cm, cecum >10cm), with feces, gas, and fluid accumulating proximal to a clear transition point. Distal to this point, the bowel appears collapsed. This pattern is crucial for distinguishing true obstruction from pseudo-obstruction or diffuse ileus, which may not show a transition. • Colonic wall thickening and a 4cm diameter alone are not specific for obstruction. • Lack of small bowel dilation does not exclude partial or early obstruction. • Presence of only gas without a transition point is nonspecific. 65. Early complication of post bariatric surgical technique is: A. Anastomotic narrowing. B. Abscess formation. C. Anastomotic leak. D. Hemorrhage Explanation: (by Perplexity AI, generated with the OpenAI O3 model) • Immediate (days-to-first-weeks) post-operative hazards include anastomotic/staple-line leaks and early hemorrhage, whereas abscesses and strictures tend to occur later.
  • 141.
    EBDR Exam MCQs& Concepts July 2024 Dr. Kareem Alnakeeb 135 66. On scrotal ultrasound for testicular torsion, which of the following radiological findings would suggest that the testis is still viable? E. A diffusely enlarged hypoechoic left testis F. A normal echogenicity testis on grey-scale imaging G. A small shrunken left testis with a surrounding hydrocoele and scrotal wall thickening H. Absent blood flow within the left testis on color flow Doppler but good flow within the tunica vaginalis Source: McQueen, Andrew S., et al. Grainger & Allison’s Diagnostic Radiology: Single Best Answer MCQs. 5th ed., Churchill Livingstone, 2009. Explanation: • The section on testicular torsion within the comprehensive review by Fiitterer et al provides useful additional information. 67. Which one of the following statements BEST describes the expected ultrasound findings in acute, uncomplicated epididymo-orchitis? A. A small atrophic right testis B. Multiple small echogenic foci scattered throughout the testis C. Patchy areas of increased echogenicity within the testis with reduced flow on color Doppler D. Well-defined, patchy areas of decreased echogenicity within the testis Source: McQueen, Andrew S., et al. Grainger & Allison’s Diagnostic Radiology: Single Best Answer MCQs. 5th ed., Churchill Livingstone, 2009. Explanation: • In the early phase of acute orchitis, there is oedema of the testis leading to swelling and diffuse low reflectivity on ultrasound. • The ultrasound appearances then evolve to increasingly well-defined areas of patchy low reflectivity. Colour Doppler flow is typically increased within these areas of low reflectivity. 68. Which of the following ultrasound finding would suggest a diagnosis of testicular teratoma rather than seminoma? A. A testicular mass that contains areas of calcification B. A testicular mass that demonstrates increased color Doppler flow C. A testicular mass that is homogeneously anechoic with posterior acoustic enhancement D. A testicular mass that has well-defined margins Source: McQueen, Andrew S., et al. Grainger & Allison’s Diagnostic Radiology: Single Best Answer MCQs. 5th ed., Churchill Livingstone, 2009. 69. An IVU showed complete right-sided ureteric duplication. Which one of the following statements is TRUE? A. If present, an ectopic ureterocele is usually related to the lower moiety B. The upper moiety calyces are prone to vesicoureteric reflux. C. The upper moiety ureter is prone to ureteric obstruction. D. The upper moiety ureter usually inserts into the bladder superior to the lower moiety ureter. Source: McQueen, Andrew S., et al. Grainger & Allison’s Diagnostic Radiology: Single Best Answer MCQs. 5th ed., Churchill Livingstone, 2009.
  • 142.
    EBDR Exam MCQs& Concepts July 2024 Dr. Kareem Alnakeeb 136 70. On the unenhanced CT, right adrenal mass is detected and appears homogeneous and has an average density of 7 HU. What is the MOST likely diagnosis? E. Adrenal adenoma F. Adrenal hyperplasia G. Adrenal metastasis H. Focal adrenal hemorrhage Source: McQueen, Andrew S., et al. Grainger & Allison’s Diagnostic Radiology: Single Best Answer MCQs. 5th ed., Churchill Livingstone, 2009. Explanation: • The 10-HU threshold is now the standard by which radiologists differentiate lipid-rich adenomas from most other adrenal lesions on unenhanced CT. • The presence of substantial amounts of intracellular fat is critical in malting the specific diagnosis of adenoma. Up to 30% of adenomas, however, do not have abundant intracellular fat and, thus, show attenuation values greater than 10 HU on unenhanced CT. • Lesions above 10 HU on an unenhanced CT are considered indeterminate and other investigations may be required. 71. A 35-year-old patient received a cadaveric renal transplant 5 days ago and now presents with worsening renal function and decreasing urine output. Which one of the following findings on a Tc-99m DTPA radionuclide scan would favor a diagnosis of acute tubular necrosis (ATN) over acute rejection? A. Delayed renal excretion B. Elevated resistive index greater than 0.7 C. Increased renal perfusion after administration of an ACEI (eg Captopril) D. Preserved renal transplant perfusion Source: McQueen, Andrew S., et al. Grainger & Allison’s Diagnostic Radiology: Single Best Answer MCQs. 5th ed., Churchill Livingstone, 2009. Explanation: • ATN is an early complication in cadaveric allografts and frequently resolves spontaneously in 1—3 weeks. • The radionuclide imaging findings of ATN are of preserved perfusion but poor renal function and urine excretion. • In acute rejection however, there is both impaired renal function and reduced perfusion on radionuclide imaging. 72. Which one of the following radiological findings is a recognized feature of Von Hippel Lindau (VHL) disease? A. Bilateral adrenal masses that yield a high signal on T2w sequences B. Cerebral aneurysms on CT angiography C. Evidence of calcified subependymal nodules on CT head D. Polymicrogyria and corpus callosum agenesis on MRI brain Source: McQueen, Andrew S., et al. Grainger & Allison’s Diagnostic Radiology: Single Best Answer MCQs. 5th ed., Churchill Livingstone, 2009. Explanation: • Phaeochromocytomas are hyperintense on T2w sequences and iso- or hypointense to the liver on Tlw sequences. 73. Regarding urethral injuries, which one of the following statements is CORRECT? A. Anterior urethral injury is more commonly due to iatrogenic or penetrating trauma than to blunt trauma B. Cystography should precede a retrograde urethrogram in a patient with suspected urethral injury. C. In men, on digital rectal examination the prostate is lower than normal in patients with urethral trauma. D. Urethral injuries occur in 50% of major pelvic fractures.
  • 143.
    EBDR Exam MCQs& Concepts July 2024 Dr. Kareem Alnakeeb 137 Source: McQueen, Andrew S., et al. Grainger & Allison’s Diagnostic Radiology: Single Best Answer MCQs. 5th ed., Churchill Livingstone, 2009. 74. Which one of the following findings would indicate a Grade 4 renal laceration? A. Extravasation of contrast from the pelvicalyceal system on delayed phase (5min) images B. Large (2-cm) subcapsular hematoma C. Perinephric hematoma that extends into the pararenal spaces D. Ill-defined low attenuation change in the lower pole renal cortex Source: McQueen, Andrew S., et al. Grainger & Allison’s Diagnostic Radiology: Single Best Answer MCQs. 5th ed., Churchill Livingstone, 2009. Explanation: • A deep renal laceration that extends into the collecting system is indicative of a grade 4 injury. 75. Which one of the following MRI findings would favor a diagnosis of renal angiomyolipoma? A. High signal on T1w and low signal on T2w sequences B. High signal on T1w and STIR sequences C. High signal on T1w and T2w sequences D. High signal on T2w and low signal on proton density sequences Explanation: (by Perplexity AI, generated with the OpenAI O3 model) • Renal angiomyolipomas characteristically contain macroscopic fat. On MRI, fat demonstrates high signal intensity on T1-weighted images and loses signal on fat-suppressed or STIR sequences. However, the classic radiological finding is high signal on T1-weighted images and variable (often low) signal on T2-weighted images due to the heterogeneous tissue components. • High signal on both T1w and STIR (option B) would not be typical, as fat is suppressed on STIR. • High signal on both T1w and T2w (option C) better fits fat-containing metastases or certain hemorrhagic lesions, not classic angiomyolipoma. • Option D and E do not match the characteristic fat features of angiomyolipoma. 76. Abdominal ultrasound for 22-year-old pregnant woman (30 weeks' gestation) with right loin pain shows dilatation of the right pelvicalyceal system. Which one of the following additional findings would suggest a diagnosis of mechanical ureteric obstruction rather than pregnancy-related dilatation? A. An elevated resistive index (RI) B. Decreased corticomedullary differentiation C. Renal parenchymal thinning D. Ureteric and pelvicalyceal dilatation Source: McQueen, Andrew S., et al. Grainger & Allison’s Diagnostic Radiology: Single Best Answer MCQs. 5th ed., Churchill Livingstone, 2009. Explanation: • Mechanical obstruction is associated with elevation of the RI. 77. The control film of IVU for an immunosuppressed 24-year-old man shows a gas containing, round lamellated mass within the urinary bladder. Postcontrast, there are multiple filling defects within the urinary bladder. What is the MOST likely cause of these appearances? A. Blood clot B. Bladder calculi C. Fungal ball D. Schistosomiasis Source: McQueen, Andrew S., et al. Grainger & Allison’s Diagnostic Radiology: Single Best Answer MCQs. 5th ed., Churchill Livingstone, 2009.
  • 144.
    EBDR Exam MCQs& Concepts July 2024 Dr. Kareem Alnakeeb 138 78. In which part of the prostate gland is a carcinoma MOST likely A. Central zone B. Peripheral zone C. Peri-urethral zone D. Transitional zone Source: McQueen, Andrew S., et al. Grainger & Allison’s Diagnostic Radiology: Single Best Answer MCQs. 5th ed., Churchill Livingstone, 2009. Explanation: • Roughly 70–75% of prostate adenocarcinomas arise in the peripheral zone, the posterolateral portion that is accessible on digital-rectal examination. • About 20% originate in the transitional (central gland) zone and only a small minority in the central or peri-urethral zones 79. Which one of the following statements BEST describes the MRI findings of a normal prostate gland? A. On T1 w images, the central zone is of higher signal intensity than the peripheral zone. B. On T1 w images, the central zone is of lower signal intensity than the peripheral zone. C. On T2w images, the peripheral zone is of lower signal intensity than the central and transitional zones D. The peripheral zone is of higher signal intensity than the central zone on T2w images. Source: McQueen, Andrew S., et al. Grainger & Allison’s Diagnostic Radiology: Single Best Answer MCQs. 5th ed., Churchill Livingstone, 2009. Explanation: • Zonal anatomy of the prostate is best demonstrated on T2w sequences. 80. The following stones are NOT visible on CT scan: A. Urate stones. B. Matrix stones. C. Xanthine stones. D. Cysteine stones Explanation: (by Perplexity AI, generated with the OpenAI O3 model) • Non-contrast CT detects nearly all urinary calculi (>99%), including urate, xanthine and cystine stones, because they remain denser than surrounding soft tissue. • Matrix stones, composed mainly of proteinaceous material with minimal mineral content, may be completely radiolucent on CT and therefore escape detection, making them the principal exception. 81. Which one of the following statements BEST describes the characteristic radiological features of retroperitoneal fibrosis? A. A plaque-like mass that encases the aorta and displaces it laterally, most commonly to the left B. A plaque-like mass that displaces the kidneys and ureters laterally at the L1- 2 level C. A plaque-like mass that displaces the aorta and iliac arteries anteriorly D. A plaque-like mass that narrows and medially displaces the ureters at the L4-5 level Source: McQueen, Andrew S., et al. Grainger & Allison’s Diagnostic Radiology: Single Best Answer MCQs. 5th ed., Churchill Livingstone, 2009.
  • 145.
    EBDR Exam MCQs& Concepts July 2024 Dr. Kareem Alnakeeb 139 82. Regarding benign and malignant adrenal masses. Which statement is CORRECT? A. Lesions >4 cm tend to be malignant B. Approximately one third of benign adenomas have HU of >10 on unenhanced CT C. Adenomas tend to show delayed enhancement with IV contrast D. Adenomas tend to show delayed clearance of IV contrast Source: Hussain, Shahid, et al. Radiology MCQs for the New FRCR Part 2A. 1st ed., TFM Publishing, 2006. Explanation: • Lesions larger than 4cm are more likely to be malignant. • About one third of benign adrenal adenomas have an unenhanced CT attenuation above 10 HU, so a higher HU does not exclude a benign adenoma. • Adenomas enhance rapidly, not with delay. • Adenomas characteristically show rapid washout (clearance) of IV contrast. • Chemical shift MRI uses T1-weighted sequences, not T2-weighted, to assess intracytoplasmic lipid. 83. Multicystic dysplastic kidney. Which statement is FLASE? A. Is the second commonest cause of a neonatal abdominal mass B. Is usually unilateral C. Is associated with pelvi-ureteric junction (PUJ) obstruction D. The renal cysts communicate Source: Hussain, Shahid, et al. Radiology MCQs for the New FRCR Part 2A. 1st ed., TFM Publishing, 2006. Explanation: • Multicystic dysplastic kidney is the second most common cause of a neonatal abdominal mass (after hydronephrosis) and is usually unilateral. • It is associated with pelviureteric junction (PUJ) obstruction in approximately 10–20% of cases. • The cysts in multicystic dysplastic kidneys do not communicate with each other, and there is no intervening normal renal parenchyma—rather, the normal architecture is replaced by multiple non- communicating cysts with fibrous stroma. • Bilaterality is rare and usually incompatible with life. 84. Which of the following features on US suggest a malignant rather than benign breast mass? A. It is taller than it is wide B. It is markedly hyperechoic C. It has a thin echogenic capsule D. It does not cast an acoustic shadow Explanation: (by Perplexity AI, generated with the OpenAI O3 model) • A non-parallel or “taller-than-wide” orientation indicates that the lesion is invading tissue planes perpendicular to the skin, a behavior typical of invasive carcinomas 85. A transvaginal ultrasound is performed on a 36-year-old woman with dysfunctional uterine bleeding. This demonstrates an enlarged globular uterus with a heterogeneous appearance of the myometrium. The myometrium contains diffuse echogenic nodules, subendometrial echogenic linear striations and 2- to 6-mm subendometrial cysts. What is the MOST likely diagnosis? A. Adenomyosis B. Endometrial polyposis C. Stage 1A endometrial cancer D. Uterine fibroid Source: McQueen, Andrew S., et al. Grainger & Allison’s Diagnostic Radiology: Single Best Answer MCQs. 5th ed., Churchill Livingstone, 2009.
  • 146.
    EBDR Exam MCQs& Concepts July 2024 Dr. Kareem Alnakeeb 140 86. Screening mammogram showed a 2-cm well-defined, oval mass that has dense 'popcorn' calcification within it and is surrounded by a thin radiolucent rim. On ultrasound, the mass is well defined and hyperechoic with areas of acoustic shadowing due to contained calcification. What is the MOST likely diagnosis? A. Fat necrosis B. Fibroadenoma C. Hamartoma D. Papilloma Source: McQueen, Andrew S., et al. Grainger & Allison’s Diagnostic Radiology: Single Best Answer MCQs. 5th ed., Churchill Livingstone, 2009. Explanation: • Fibroadenomas may become calcified, particularly after menopause. Classically the calcifications have a coarse ‘popcorn’ appearance; however, they may also appear small and punctate. • An oil cyst typically demonstrates eggshell calcification and is the result of fat necrosis. 87. On transvaginal ultrasound for a 60-year-old nulliparous woman presents with postmenopausal bleeding presents with postmenopausal bleeding, endometrium is 8 mm thick and the endo-myometrial junction appeared indistinct. MRI examination is indicated for the possibility of endometrial carcinoma. What are the likely findings on MRI? A. On unenhanced T1w images the endometrial cancer appears of high signal intensity compared to the surrounding myometrium. B. On contrast-enhanced T1w images, endometrial cancer shows avid enhancement compared with surrounding myometrium. C. On T2w images the normally high signal junctional zone is disrupted. D. The endometrial cancer demonstrates delayed/little enhancement compared to the normal surrounding myometrium on postcontrast Tl w images. Source: McQueen, Andrew S., et al. Grainger & Allison’s Diagnostic Radiology: Single Best Answer MCQs. 5th ed., Churchill Livingstone, 2009. 88. Which one of the following sonographic finding would indicate that the adnexal mass is more likely to be malignant than benign? A. Homogeneously hypoechoic mass with posterior acoustic enhancement B. Multiple septations that are approximately 1 mm thick C. Papillary projections D. Size > 4 cm Source: McQueen, Andrew S., et al. Grainger & Allison’s Diagnostic Radiology: Single Best Answer MCQs. 5th ed., Churchill Livingstone, 2009. 89. The HSG shows a single vagina, single cervix but two separate uterine cavities leading to separate uterine horns. What is the MOST likely diagnosis? A. Arcuate uterus B. Bicornuate uterus C. Didelphys uterus D. Septate uterus Source: McQueen, Andrew S., et al. Grainger & Allison’s Diagnostic Radiology: Single Best Answer MCQs. 5th ed., Churchill Livingstone, 2009.
  • 147.
    EBDR Exam MCQs& Concepts July 2024 Dr. Kareem Alnakeeb 141 90. Regarding MRI of the breast, which one of the following statements is CORRECT? A. Breast MRI should be performed during the middle of the menstrual cycle to improve sensitivity. B. Malignant lesions tend to show poor enhancement following intravenous contrast, compared with. surrounding breast tissue. C. MRI has a high sensitivity and specificity for the detection of invasive breast cancer. D. Post radiotherapy, abnormal enhancement patterns return to normal within 3- 6 months. Source: McQueen, Andrew S., et al. Grainger & Allison’s Diagnostic Radiology: Single Best Answer MCQs. 5th ed., Churchill Livingstone, 2009. Explanation: • Malignant breast lesions enhance postcontrast; however, normal hormonally active breast tissue can also chance, particularly during the middle of the menstrual cycle (6th—17th days). • In younger patients it may be helpful to repeat the scan carlier or later in the menstrual cycle to improve specificity. 91. Regarding ovarian cancer: Which statement is CORRECT? A. It is the commonest gynecological malignancy B. CT only has a pre-operative staging accuracy of 50% , C. CA-125 is specific for ovarian cancer D. Doppler ultrasound may help with differentiating benign from malignant disease Source: Hussain, Shahid, et al. Radiology MCQs for the New FRCR Part 2A. 1st ed., TFM Publishing, 2006. Explanation: • Ovarian cancer is associated with colorectal cancer, particularly in the context of Lynch syndrome (hereditary nonpolyposis colorectal cancer), which increases risk for several cancers including both ovarian and colorectal malignancies. • Ovarian cancer is the second most common gynecological malignancy after endometrial carcinoma. • The accuracy of CT for pre-operative staging of ovarian cancer is higher than 50%, typically ranging from 70–90%. • CA-125, while useful as a tumor marker, is not specific for ovarian cancer as it may be elevated in various benign conditions such as fibroids, endometriosis and pelvic inflammatory disease. • Doppler ultrasound can aid in distinguishing benign from malignant ovarian masses. 92. Regarding endometriosis: Which statement is CORRECT? A. 20% of infertile women are affected B. Endometrioma is rarely anechoic on ultrasound C. May present with pneumothorax D. Cystic masses seen are typically hypointense on Tl weighted images Source: Hussain, Shahid, et al. Radiology MCQs for the New FRCR Part 2A. 1st ed., TFM Publishing, 2006. Explanation: • About 20% of infertile women are affected by endometriosis, making it a common association. The condition most often involves the ovaries (around 80%) and not the fallopian tubes. • Endometriomas, also known as "chocolate cysts", classically appear as cystic masses with low-level internal echoes on ultrasound, not anechoic. • Although rare, thoracic endometriosis may present with recurrent pneumothorax. • Cystic endometriotic lesions are typically hyperintense on T1-weighted MRI due to blood products.
  • 148.
    EBDR Exam MCQs& Concepts July 2024 Dr. Kareem Alnakeeb 142 93. With regards to assessment of tubal patency, which of the following statements is CORRECT? A. Gadolinium enhanced MRI is the investigation of choice B. Normal fallopian tubes are visible on pelvic US C. Hysterosalpingography should be performed in the first half of the menstrual cycle D. lodine based contrast is used for hysterosalpingo contrast sonography Source: Unknown Explanation: • Laparoscopy and blue dye instillation (with spillage of dye into the peritoneal cavity indicating patency) is the gold standard, but requires a general anaesthetic. • MRI and standard pelvic US do not clearly demonstrate the fallopian tubes. • HSG provides an accurate indication of tubal patency but employs ionizing radiation. It should therefore be performed in the first half of the menstrual cycle to avoid irradiating a patient with possible early pregnancy. • Hysterosalpingo-Contrast-Sonography uses microbubbles to demonstrate the fallopian tubes and has the advantage of not using ionizing radiation or requiring anaesthesia but is less accurate than the other methods. 94. Ultrasound in early pregnancy: Which statement is CORRECT? A. Gestational sac is first seen in 8 weeks. B. Accurate biparietal diameter can be done at 13 weeks. C. Yolk sac is seen in 5 weeks. D. Placenta can be identified at 12 weeks Source: Unknown Explanation: • Accurate biparietal diameter measurement can be performed from 13 weeks. • The gestation sac is typically first visualized at around 5 weeks and by 10 weeks, it should occupy more than half the uterine cavity. • The placenta can usually be identified from 10 weeks onwards. • The yolk sac is usually visible from about 6 weeks. 95. What level of serum glucose is generally considered acceptable when performing an FDG-PET scan? A. Less than 50 mg/dL. B. Less than 100 mg/dL. C. Less than 200 mg/dL. D. Less than 300 mg/dL. Explanation: (by Perplexity AI, generated with the OpenAI O3 model) • Most professional guidelines (EANM, SNMMI, ACR) advise proceeding with an oncologic FDG-PET/CT if the fasting plasma glucose is below about 11 mmol/L (≈ 200 mg/dL); at higher levels competitive inhibition by circulating glucose can lower tumor uptake and degrade image quality. • Values under 100 mg/dL are ideal but not mandatory, whereas levels > 200 mg/dL usually prompt rescheduling or glucose-lowering measures. 96. A normal brain perfusion SPECT pattern: A. Appears as a completely symmetrical tracer uptake distribution in all subjects B. Is completely identical in all subjects C. Is influenced by radiopharmaceutical injected D. Cerebellum is not visualized Explanation: (by Perplexity AI, generated with the OpenAI O3 model) • Minor physiological right-to-left differences (often slightly higher left-hemisphere counts) are frequent in healthy people; perfect symmetry is not obligatory. • Normal inter-individual variability exists (age, sex, handedness and technical factors all modify uptake distribution)
  • 149.
    EBDR Exam MCQs& Concepts July 2024 Dr. Kareem Alnakeeb 143 • Tracers such as 99 mTc-HMPAO and 99 mTc-ECD have different extraction fractions and wash-out kinetics, leading to subtle but recognizable differences in regional uptake (for example higher cerebellar activity with HMPAO) • Cerebellar cortex normally shows high perfusion and is well seen; it is often chosen as a reference (“normalizer”) region 97. A low flat renogram curve indicates: A. Advanced nephropathy B. Complete obstruction to urine outflow C. Vesico-ureteric reflux D. Partial obstruction to urine outflow Explanation: (by Perplexity AI, generated with the OpenAI O3 model) • A very low, almost horizontal time–activity curve reflects markedly reduced tracer extraction by damaged renal parenchyma, typical of end-stage or advanced medical renal disease. • Complete or partial outflow obstruction instead produces an uptake phase followed by a rising or plateau phase; vesico-ureteric reflux alters the post-void segment rather than the primary renographic curve 98. Regarding gastric emptying scintigraphy A. T1/2 is the time at which gastric counts falls to its half B. Tc 99m tin colloid labeled RBCs are injected IV C. If 20 % of gastric counts remain after 1 hour, delayed gastric emptying is considered. D. It cannot detect changes in gastric emptying rate in post operative cases. Explanation: (by Perplexity AI, generated with the OpenAI O3 model) • T1/2, or half-emptying time, refers to the time at which the measured gastric radioactivity drops to 50% of its initial value, a standard parameter in gastric emptying studies. • Tc-99m sulfur colloid is typically used and is ingested with a solid meal, not injected intravenously as labeled RBCs. • Delayed gastric emptying is usually defined as >60% retention at 2 hours or >10% at 4 hours, not at the 1-hour mark. • Scintigraphy can reliably assess postoperative gastric emptying changes. 99. Regarding the scintigraphic assessment of a joint prosthesis, which of the following is CORRECT? A. Increased MDP activity over the greater trochanter is diagnostic of an inflammatory bursitis B. A cemented total hip replacement (THR) remains 'hot' for only 6-12 months after surgery C. A hot knee prosthesis 18 months after surgery is abnormal D. Prosthetic loosening results in a scan that is 'hot' on blood pool and delayed phases Explanation: (Different AI models give varying answers to this question) • Normal postoperative uptake o Cemented hip: fades to baseline within 6–12 months. o Uncemented hip: may stay active for 3–5 years. o Knee (cemented or hybrid): activity can persist for ∼3 years. • Interpretation of multiphase scans o Hot flow + blood-pool + delayed → think infection or active inflammation. o Normal flow/blood-pool but hot delayed → think aseptic loosening or stress remodeling
  • 150.
    EBDR Exam MCQs& Concepts May 2023 Dr. Kareem Alnakeeb 144 May 2023 1. A 60-year-old female presents with a history of facial pain and diplopia. Clinical examination reveals palsies of the III, IV, and VI cranial nerves, Horner’s syndrome, and facial sensory loss in the distribution of the ophthalmic and maxillary divisions of the trigeminal (V) cranial nerve. Where is the causative abnormality located? A. Dorello’s canal. B. Cavernous sinus. C. Superior orbital fissure. D. Inferior orbital fissure. E. Meckel’s cave. Source: Lindsay, Richard, et al. SBAs for the FRCR Part 2A. 1st ed., Oxford University Press, 2012. Explanation: Cavernous Sinus Anatomy • Cranial nerves III, IV, and VI, and ophthalmic (V1) and maxillary (V2) divisions of the V cranial nerve course through the cavernous sinus along with the internal carotid artery. • The V2 division of the trigeminal nerve passes through the inferior portion of the cavernous sinus and exits via the foramen rotundum. • The remainder of the cranial nerves mentioned above enter the orbit via the superior orbital fissure. Clinical Implications • Palsies of cranial nerves III, IV, and VI result in ophthalmoplegia. • Involvement of V1 and V2 divisions of the trigeminal nerve produces facial pain and sensory loss; involvement of sympathetic nerves around the internal carotid artery results in Horner’s syndrome. • This cluster of findings is found in Tolosa Hunt syndrome, an idiopathic inflammatory process involving the cavernous sinus. 2. A 40-year-old presents with a painless, slowly growing lateral neck mass located between the angle of the mandible and the carotid bifurcation. On ultrasound, the lesion is well-circumscribed, cystic, and displaces nearby structures but does not show internal vascularity. What is the most likely diagnosis? A. Second branchial cleft cyst B. Carotid body tumour C. Cystic metastatic lymph node D. Cervical thymic cyst E. Vagal schwannoma Explanation: (by Perplexity AI, generated with the OpenAI O3 model) • The most likely diagnosis is a second branchial cleft cyst, which typically presents as a well- circumscribed, cystic mass situated lateral to the neck, often at the angle of the mandible and just anterior to the sternocleidomastoid, displacing but not infiltrating vascular structures. Carotid body tumours are usually solid, hypervascular lesions at the carotid bifurcation, causing splaying of the carotid vessels. • Cystic metastatic lymph nodes are considered especially in older adults, but they often exhibit irregular walls or solid components. • Although cervical thymic cysts and vagal schwannomas can occur laterally, they are rare and have distinct imaging or clinical features. The ultrasound finding of a purely cystic, avascular mass supports a branchial cleft cyst diagnosis.
  • 151.
    EBDR Exam MCQs& Concepts May 2023 Dr. Kareem Alnakeeb 145 3. A 60-year-old woman presents with a painless, slowly growing mass in the lateral aspect of the neck. The patient is referred for imaging with a clinical diagnosis of carotid body paraganglioma. Which of the following is a distinctive feature of carotid body paraganglioma on imaging? A. Soft-tissue mass in the carotid space. B. Intense enhancement after IV contrast administration. C. High signal on T2WI. D. Splaying of the internal and external carotid arteries. E. Low signal on T2WI. Source: Lindsay, Richard, et al. SBAs for the FRCR Part 2A. 1st ed., Oxford University Press, 2012. Explanation: • Carotid body tumour or paraganglioma is the most common paraganglioma of the head and neck. It arises from the paraganglionic cells located on the medial aspect of the carotid bifurcation. • On MRI, they are of low to intermediate signal intensity on T1WI and hyperintense on T2WI. They are hypervascular and demonstrate intense enhancement after contrast administration. • Splaying of the internal and external carotid arteries and multiple flow voids producing a ‘salt and pepper’ appearance are distinctive features on imaging. 4. A newborn presents with cyanosis and a chest radiograph showing increased pulmonary vascularity (lung plethora). Which is the most likely underlying cardiac condition? A. Patent ductus arteriosus B. Atrial septal defect C. Pulmonary atresia D. Tricuspid regurgitation E. Tetralogy of Fallot Explanation: (by Perplexity AI, generated with the OpenAI O3 model) • Patent ductus arteriosus (PDA) classically leads to increased pulmonary blood flow, resulting in lung plethora visible on imaging. Cyanosis may occur if right-to-left shunting develops (e.g., with pulmonary hypertension). • Atrial septal defect (ASD) usually causes increased flow but less often cyanosis early on. • Pulmonary atresia typically causes reduced pulmonary flow and oligemic lung fields rather than plethora. • Tricuspid regurgitation can result from right heart issues but does not usually lead to marked lung plethora with cyanosis unless there is a significant shunt or other associated defects. 5. A 45-year-old asthmatic patient presents with worsening cough and expectoration of brownish mucus plugs. Chest radiograph shows branching tubular opacities in the right upper lobe resembling “finger-in- glove” appearance. Which is the most likely underlying diagnosis? A. Bronchiectasis secondary to cystic fibrosis B. Allergic bronchopulmonary aspergillosis C. Acute pulmonary embolism D. Sarcoidosis E. Primary lung carcinoma Explanation: (by Perplexity AI, generated with the OpenAI O3 model) • The “finger-in-glove” sign on chest imaging refers to branching tubular opacities representing impacted mucus plugs within dilated bronchi. • In asthma patients, this is classically seen in allergic bronchopulmonary aspergillosis (ABPA), a hypersensitivity reaction to Aspergillus colonization of the airways. • Cystic fibrosis can also show similar opacities but is less likely in a middle-aged adult without suggestive history.
  • 152.
    EBDR Exam MCQs& Concepts May 2023 Dr. Kareem Alnakeeb 146 • Acute pulmonary embolism and sarcoidosis do not typically present with branching tubular opacities on imaging. • Primary lung carcinoma can cause mass-like consolidations or obstructive changes, not the characteristic pattern described here. 6. A 26 year old man suffers a blunt injury to his chest in a road traffic accident. The most common abnormality seen on CT as a result of blunt thoracic injury is: A. Pneumothorax B. Pulmonary laceration C. Haemothorax D. Tracheo-bronchial injuries E. Pulmonary contusion Source: Currie, Stuart, et al. SBAs for the FRCR 2A. 1st ed., Cambridge University Press, 2010. Explanation: • Pulmonary contusion is the commonest manifestation of blunt trauma and indicates trauma to alveoli with alveolar haemorrhage without significant alveolar disruption. • Whilst plain film changes may not be apparent for up to six hours, CT will demonstrate changes almost immediately post-trauma and signs of resolution can be seen as early as 48 hours. • If unresolved, it may progress to ARDS. 7. A 46-year-old motorcyclist was involved in a high-speed RTA. On arrival of the paramedics, the GCS was recorded as 4/15 and the patient was intubated at the site of injury. An emergency noncontrast CT showed multiple subtle petechial hemorrhages characteristic of diffuse axonal injury. What is the most likely site of the petechial hemorrhage? A. Insular ribbon B. Watershed areas C. Periventricular white matter D. Grey-white matter junction E. Cerebellum Source: Proctor, Robin. Final FRCR Part A Modules 4-6 Single Best Answer MCQs: The SRT Collection of 600 Questions with Explanatory Answers (MasterPass). 1st ed., CRC Press, 2009. Explanation: • Diffuse axonal injury (DAI) occurs in severe trauma as a result of shearing stress along the course of the white matter tracts especially at the grey-white matter junction. • The injury is usually microscopic and initial CTs are usually normal despite profound clinical impairment. • Acute DAI may also be seen as small petechial hemorrhages at the grey-white matter junction (67%), internal/external capsule, corona radiata, corpus callosum (21%) and brainstem. • MR features depend on the age of the hemorrhage. Prognosis is poor. 8. Following renal trauma, a renal angiogram is performed to assess which of the following complications? A. Hematuria B. Urinary leakage C. Hypotension D. Hypertension E. None of the above
  • 153.
    EBDR Exam MCQs& Concepts May 2023 Dr. Kareem Alnakeeb 147 Explanation: (by Perplexity AI, generated with the OpenAI O3 model) • Renal angiography is primarily indicated after renal trauma to assess ongoing or severe hematuria, typically due to vascular injury such as pseudoaneurysm or arteriovenous fistula formation. It helps locate the source of bleeding and can be therapeutic via embolization. • While urinary leakage is best evaluated by CT urography or retrograde pyelography, and hypotension is a clinical sign rather than an imaging finding, hypertension is a late or chronic sequela that can result from vascular injury but is not acutely assessed by angiography in the trauma setting. 9. In classical transposition of the great arteries (TGA), which pattern of atrioventricular (AV) and ventriculoarterial (VA) connections is typically present? A. AV concordance, VA concordance B. AV concordance, VA discordance C. AV discordance, VA concordance D. AV discordance, VA discordance E. AV discordance, normal VA connections Explanation: (by Perplexity AI, generated with the OpenAI O3 model) • In classical (D-loop) transposition of the great arteries, the atrioventricular (AV) connections are concordant (right atrium to right ventricle, left atrium to left ventricle), but the ventriculoarterial (VA) connections are discordant—the aorta arises from the right ventricle and the pulmonary artery from the left ventricle. This anatomical configuration causes parallel rather than series circulation, resulting in severe cyanosis after birth unless there is mixing of blood. • Options with AV discordance (C, D, E) describe other complex congenital heart diseases, such as congenitally corrected transposition (L-TGA), not the classical TGA pattern. 10. A 59-year-old patient with Child-Pugh A cirrhosis is found to have a solitary 5cm hepatocellular carcinoma. Which of the following locoregional therapies is generally considered to be limited by tumour size below 5cm? A. Transarterial chemoembolization (TACE) B. Transarterial radioembolization (TARE) C. Radiofrequency ablation D. Microwave ablation E. Stereotactic body radiotherapy Explanation: (by Perplexity AI, generated with the OpenAI O3 model) • Radiofrequency ablation (RFA) is typically reserved for hepatocellular carcinoma (HCC) lesions up to 3–5cm; efficacy significantly declines for tumours larger than 5cm, and complete ablation is less likely to be achieved. • Microwave ablation (MWA) also has size limitations but can more reliably treat tumours approaching 5cm, while TACE and TARE are not strictly limited by nodule size in the same way and are used for larger or multifocal disease. • Stereotactic body radiotherapy can also be considered for tumours above this size. Therefore, RFA is the option most limited by a size threshold of <5cm.
  • 154.
    EBDR Exam MCQs& Concepts May 2023 Dr. Kareem Alnakeeb 148 11. On musculoskeletal ultrasound, a tendoachilles was assessed but distal assessment was limited due to a static procedure. The tendon appeared hyperechoic with hypoechoic areas, was hypoechoic compared to the related nerve, and the distal part could not be evaluated. What is the most likely abnormality? A. Tendon xanthoma B. Achilles tendon rupture C. Normal tendon D. Paratenonitis E. Partial thickness tear Explanation: (by Perplexity AI, generated with the OpenAI O3 model) • The described ultrasound findings—hyperechoic tendon with interspersed hypoechoic areas and inability to fully assess the distal tendon—best fit a partial thickness tear of the Achilles tendon. • Chronic tendinopathy or xanthomas tend to show diffuse thickening, but the classic pattern for tears is disruption of tendon fibers with mixed echogenicity and potential inability to visualise the distal end, especially if static imaging is used. A complete rupture usually shows clear tendon discontinuity with a gap, whereas normal tendon is uniformly echogenic and fibrillar. • Paratenonitis may show surrounding fluid or inflammation but does not typically cause hypoechoic defects within the tendon itself. 12. On intravenous pyelogram (IVP), which of the following is the most likely cause of a medially deviated ureter? A. Atrophy of psoas muscle B. Enlarged paraaortic lymph nodes C. Retrocaval ureter D. Urinoma E. Retroperitoneal fibrosis Explanation: (by Perplexity AI, generated with the OpenAI O3 model) • Retrocaval ureter results in medial deviation of the ureter on IVP due to the abnormal course of the ureter passing posterior to the inferior vena cava and then looping medially. • Atrophy of the psoas muscle may cause lateral displacement of the ureter. • Enlarged paraaortic lymph nodes usually push the ureter laterally, not medially. • Urinoma is unlikely to affect ureteral position, and retroperitoneal fibrosis typically causes lateral or irregular deviation with medial constraint rather than frank medialization. 13. What is the most common benign lesion of the phalanx, which may present with pathological fracture? A. Chondroblastoma B. Enchondroma C. Osteoid osteoma D. Giant cell tumour E. Aneurysmal bone cyst Explanation: (by Perplexity AI, generated with the OpenAI O3 model) • Enchondroma is the most common benign lesion of the phalanx and frequently affects the small bones of the hands and feet. It often presents as an incidental finding but may cause thinning of the cortex and predispose to pathological fractures. • Chondroblastoma and giant cell tumour are rare in the phalanges and typically occur at different sites. • Osteoid osteoma usually involves the diaphysis of long bones and presents with nocturnal pain.
  • 155.
    EBDR Exam MCQs& Concepts May 2023 Dr. Kareem Alnakeeb 149 • Aneurysmal bone cysts can also occur in the phalanges but are much less common than enchondroma. 14. A 25-year-old man presents with persistent knee pain. Radiographs show a well-defined lytic lesion in the epiphysis of the distal femur with surrounding chondroid-type ring-and-arc calcifications. What is the most likely diagnosis? A. Giant cell tumour B. Chondroblastoma C. Osteosarcoma D. Fibrous dysplasia E. Aneurysmal bone cyst Explanation: (by Perplexity AI, generated with the OpenAI O3 model) • Chondroblastoma characteristically arises in the epiphysis of long bones in young patients and appears as a lytic lesion often containing “ring-and-arc” (chondroid) calcifications due to the cartilaginous matrix. • This differentiates it from giant cell tumour (A), which usually lacks matrix calcification and affects older patients. • Osteosarcoma (C) often has aggressive features and produces osteoid rather than cartilage; fibrous dysplasia (D) has a ground-glass matrix; aneurysmal bone cyst (E) is expansile but lacks internal calcification and affects metaphyses. 15. In a pregnant woman with a history of previous Caesarean section and current placenta previa, what is the most important assessment during the second pregnancy? A. Retroplacental Doppler flow B. Congenital fetal anomaly C. Oligohydramnios D. Intra-placental lakes E. Placenta accreta spectrum Explanation: (by Perplexity AI, generated with the OpenAI O3 model) • The most critical assessment in a woman with a previous Caesarean section and a current diagnosis of placenta previa is for placenta accreta spectrum (PAS). The risk of abnormal placental adherence (including accreta, increta, and percreta) is markedly increased in this setting. • Ultrasound with or without MRI is performed to assess for features of PAS, which significantly impact delivery planning and maternal risk. • Retroplacental Doppler, fetal anomalies, and oligohydramnios are relevant but not as closely linked to morbidity or immediate management needs in this context. • Intra-placental lakes may be seen in PAS but are a nonspecific finding. 16. A patient presents with a tumour located in the upper anorectal canal and regional lymph node enlargement is seen. Which lymph node group is most likely to be involved? A. External iliac B. Internal iliac C. Common iliac D. Pudendal E. Obturator
  • 156.
    EBDR Exam MCQs& Concepts May 2023 Dr. Kareem Alnakeeb 150 Explanation: (by Perplexity AI, generated with the OpenAI O3 model) • The upper anorectal canal drains primarily to the internal iliac lymph nodes due to its embryological origin above the dentate line. This region follows the lymphatics of the superior rectal and middle rectal vessels, directing spread to internal iliac nodes first. • External iliac and common iliac nodes may be involved in more advanced disease but are not primary nodal groups for this location. • Pudendal and obturator nodes are less commonly involved from tumours originating in the upper anorectal canal. 17. A pregnant woman presents with acute abdominal pain and vomiting. The pregnancy was previously normal. Imaging shows a dilated bowel loop with circumferential thickening, delayed enhancement, and pneumobilia. What is the most likely diagnosis? A. Intestinal obstruction B. Volvulus C. Internal hernia D. Mesenteric ischaemia E. Gallstone ileus Explanation: (by Perplexity AI, generated with the OpenAI O3 model) • Gallstone ileus is suggested by the combination of small bowel obstruction, pneumobilia (air in the biliary tree), and a dilated bowel loop, especially with history of prior normal pregnancy and acute presentation. The classic radiological triad—small bowel obstruction, ectopic gallstone, and pneumobilia—is nearly pathognomonic. • Intestinal obstruction is a descriptive term and not a specific cause; volvulus or internal hernia could present similarly but would not explain pneumobilia. • Mesenteric ischaemia may cause bowel wall thickening and enhancement delay, but does not account for pneumobilia. • Gallstone ileus results from a fistula between the gallbladder and GI tract allowing a gallstone to enter and obstruct the bowel. 18. Which of the following is most characteristic of a flash-filling hepatic haemangioma on MRI? A. T1 hypointensity B. T2 hyperintensity C. Delayed washout D. All of the above E. None of the above Explanation: (by Perplexity AI, generated with the OpenAI O3 model) • Flash-filling hepatic haemangiomas typically appear markedly hyperintense on T2-weighted MRI sequences, which is a key distinguishing feature. • While they can be mildly hypointense on T1, this is less specific. • Delayed washout is not a feature; instead, haemangiomas show persistent enhancement after the initial rapid fill. • "All of the above" is incorrect because delayed washout is not observed, making option B the best answer.
  • 157.
    EBDR Exam MCQs& Concepts May 2023 Dr. Kareem Alnakeeb 151 19. Which of the following statements about tumefactive sludge in the gallbladder is correct? A. Shows vascularity with colour Doppler B. Looks like a mass C. Rarely occurs with gallstones D. All of the above E. None of the above Explanation: (by Perplexity AI, generated with the OpenAI O3 model) • Tumefactive sludge in the gallbladder can mimic a mass due to its echogenic and mass-like appearance on ultrasound, but it typically does not demonstrate vascularity on colour Doppler, helping to distinguish it from true neoplastic lesions. • While tumefactive sludge can occur without gallstones, it is commonly associated with gallstones and biliary stasis, making option C incorrect. Therefore, the most accurate choice is that tumefactive sludge looks like a mass but does not usually fulfill the other criteria. 20. On pelvic MRI, which imaging feature best helps differentiate a hemorrhagic ovarian cyst from an endometrioma? A. Hemorrhagic cyst shows dark spots in T2 B. Hemorrhagic cyst shows facilitated DW C. Hemorrhagic cyst shows bright ADC D. Shading is more in Hemorrhagic cyst E. Endometrioma shows high T2 signal Explanation: (by Perplexity AI, generated with the OpenAI O3 model) • Hemorrhagic cysts typically show facilitated diffusion on DWI, meaning they have high signal on ADC maps owing to the lack of true diffusion restriction, helping distinguish them from endometriomas. Endometriomas often show "T2 shading," a progressive loss of T2 signal due to repeated hemorrhage and high protein content, whereas hemorrhagic cysts do not typically exhibit this pronounced shading effect. • True restricted diffusion (low ADC) is more likely in endometriomas than in simple hemorrhagic cysts. • Hemorrhagic cysts may have variable T2 signal but do not characteristically show “dark spots” or more shading than endometriomas. 21. An 18-year-old mountain bike enthusiast is suspected of sustaining a renal injury after attempting a front wheel touch-up manoeuvre. A laceration to the right kidney is noted on CT, which demonstrates contrast enhancement during the pyelographic phase of the examination. What is the significance of this finding? A. Pre-existing angiomyolipoma. B. Active haemorrhage. C. Devascularization. D. Renal infarction. E. Urine leak. Source: Lindsay, Richard, et al. SBAs for the FRCR Part 2A. 1st ed., Oxford University Press, 2012. Explanation: Lacerations and Contrast Enhancement • Lacerations generally contain clotted blood and therefore do not enhance on scans obtained with intravenous contrast. • Intense enhancement within a laceration during the early phase indicates active haemorrhage.
  • 158.
    EBDR Exam MCQs& Concepts May 2023 Dr. Kareem Alnakeeb 152 Indications of Urine Leak and Urinary Extravasation • Contrast enhancement during the pyelographic phase of the CT examination indicates the presence of a urine leak. • A delayed scan of 10–15 minutes may show the extent of the urinary extravasation. Infarction and Contusions • Focal areas of infarction do not enhance (unlike contusions). Cortical Rim Nephrogram and Kidney Devitalization • The cortical rim nephrogram is a sign of a devascularized kidney, which occurs due to laceration of the main renal artery. 22. A 55-year-old female patient presents to your hospital with a history of recurrent UTIs and gross haematuria. Repeated urine cultures are negative, but analysis reveals copious white and red cells in the urine. The patient fails to improve with antibiotics. A CT scan of renal tracts is carried out, which shows an atrophic right kidney containing calcification. There is also an area of increased density on the unenhanced portion of the scan noted in the upper pole of the kidney, with overlying cortical thinning. There are multiple strictures noted in the ureter, with intervening areas of dilatation, giving a corkscrew appearance. There is extensive coarse calcification noted in the wall of the bladder. A CXR is carried out and is normal. Early morning urine collections finally identify mycobacterium tuberculosis in the urine, confirming the suspicion of renal and urinary tract TB. Which of these features is atypical of renal TB? A. High-density material in the calyceal system. B. Bladder calcification. C. Renal calcification. D. Normal CXR. E. Corkscrew appearance to the ureter. Source: Lindsay, Richard, et al. SBAs for the FRCR Part 2A. 1st ed., Oxford University Press, 2012. Explanation: Schistosomiasis and Tuberculosis Associations • Whilst this can be seen in TB, it is more typically associated with schistosomiasis. Tuberculosis Effects on the Urinary Tract • TB usually causes scarring and a reduced capacity bladder. • Renal calcification is typical. • The areas of increased attenuation within the calyceal system represent areas of coalescing caseating granulomas, and may have associated calcification. • Scarring can also cause stenosis of calyces, causing focal obstruction. • Occasionally a small calcified kidney is found, evidence of autonephrectomy. Spread and Imaging in Renal TB • Passage of the infection via the urine into the ureters causes focal stenoses, which can coalesce to cause a long stricture, or give a beaded or corkscrew appearance to the ureter. • Whilst renal TB results from spread from a primary pulmonary infection, the CXR is only abnormal in 25–50% of cases and is therefore not helpful.
  • 159.
    EBDR Exam MCQs& Concepts May 2023 Dr. Kareem Alnakeeb 153 23. A 45-year-old man with a history of urinary tract infections undergoes imaging for persistent haematuria. Which of the following is NOT a typical radiological feature of renal tuberculosis? A. Atrophic kidney B. Hydronephrosis C. Staghorn calculus D. Parenchymal calcification E. Bilateral polycystic change Explanation: (by Perplexity AI, generated with the OpenAI O3 model) • Renal tuberculosis typically results in an atrophic kidney due to chronic infection, may show parenchymal or lobar (putty) calcification, and often leads to hydronephrosis due to infundibular or ureteric stricture. • A staghorn calculus can also occur secondary to chronic infection. • Bilateral polycystic change is not characteristic of renal TB, which instead causes destructive, not cystic, changes to the renal parenchyma. 24. A 40-year-old woman, 4 days post-liver transplant, presents with fever. She is scheduled for a routine screening mammogram as part of breast cancer screening. What is the most appropriate step regarding her mammography appointment? F. Proceed with mammography as scheduled G. Defer mammography until resolution of fever and reassessment H. Cancel all future breast cancer screening I. Immediate breast MRI instead of mammography J. Begin annual mammography from age 50 only Explanation: (by Perplexity AI, generated with the OpenAI O3 model) • In the early post-liver transplant period, patients are immunosuppressed and at increased risk of infection. • Fever 4 days after transplantation is a red flag for possible infection or post-operative complication, and elective procedures, including screening mammography, should be deferred until the patient is clinically stable and fully assessed. • Proceeding with mammography risks exposing the patient and staff to potential infection and may delay essential medical management. • There is no indication to permanently cancel breast screening, nor to switch to MRI based only on fever or transplant status. • Guidelines support age-appropriate screening for breast cancer regardless of transplant history, but screening should be postponed during acute illness. 25. Osteoporosis circumscripta – well-defined geographic lytic lesions in the skull – represents the early stages of which condition? A. Paget’s disease B. hyperparathyroidism C. multiple myeloma D. senile osteoporosis E. sickle cell disease
  • 160.
    EBDR Exam MCQs& Concepts May 2023 Dr. Kareem Alnakeeb 154 Source: Bydder, Megan, et al. Get Through Final FRCR Part A: SBAs for the Modular Examination. 1st ed., Hodder Education Publishers, 2009. Explanation: Paget’s Disease Overview • Paget’s disease is a common progressive disorder of osteoclasts and osteoblasts resulting in bone remodelling. • It is usually polyostotic and asymmetrical, and affects 10% of those aged over 80. Initial Phase of Paget’s Disease • Osteoporosis circumscripta is seen in the initial phase of Paget’s disease, which is characterized by an aggressive, predominantly lytic process with intense osteoclastic activity causing bone resorption. • Bone marrow is replaced by fibrous tissue with large vascular channels. Radiological Features • Geographic osteoporosis is seen in the skull and long bones, where the characteristic feature is a flame-shaped radiolucency beginning in a subarticular location and progressing into the diaphysis. Disease Progression • The disease then progresses through a mixed phase to a quiescent inactive late stage where bone turnover is decreased. Skull Involvement • The skull is involved in 29–65% of cases, most commonly the anterior calvarium. 26. Which of the following is a recognised cause of a ‘bone within bone’ appearance? A. Renal osteodystrophy B. Paget’s disease C. Hyperparathyroidism D. Melorheostosis E. Osteopathia striata Source: Chin, Teck Yew, et al. Get Through Final FRCR 2A: SBAs. 1st ed., CRC Press, 2017. Explanation: • A ‘bone within bone’ appearance describes the radiographic appearance whereby a bone appears to have another bone within it, which results from endosteal new bone formation. • Recognised causes include Paget’s disease, sickle cell disease, thalassemia, Gaucher’s disease, acromegaly, hypervitaminosis D, scurvy and rickets, among many others. • It can also be a normal finding in infants, particularly in the thoracolumbar spine. 27. In a 26 year old woman with sickle cell disease, which one of the following would not be considered a typical musculoskeletal manifestation of the disease? A. Osteopaenia and trabecular thinning B. ‘Bone within bone’ appearance C. Avascular necrosis of the femoral head D. Posterior vertebral scalloping E. Fish deformity of the vertebrae Source: Currie, Stuart, et al. SBAs for the FRCR 2A. 1st ed., Cambridge University Press, 2010. Explanation: • Posterior vertebral scalloping is not a feature.
  • 161.
    EBDR Exam MCQs& Concepts May 2023 Dr. Kareem Alnakeeb 155 • The remainder are all classic features of sickle cell anaemia, along with ‘hair-on-end’ appearance of the skull due to coarse granular osteoporosis and widening of the diploe. • Osteomyelitis is a feature and is due to salmonella in over 50% of cases. 28. On pelvic MRI, which signal characteristic underlies the “shading sign” commonly seen in endometriomas? A. Bright T1, bright T2 B. Bright T1, dark T2 C. Bright T1, dark T2 due to hemosiderin content D. Dark T1, dark T2 E. Bright T1, intermediate T2 Explanation: (by Perplexity AI, generated with the OpenAI O3 model) • The MRI “shading sign” in endometriomas refers to lesions that are hyperintense on T1-weighted images and show a loss of signal (hypointensity) on T2-weighted images. This results from repeated hemorrhage and high protein content within the cyst, which causes T2 shortening—darkening the lesion despite T1 brightness. • While hemosiderin can further lower T2 signal, classic shading is not exclusively due to it, distinguishing B from C. • Options A and E are incorrect as endometriomas characteristically have dark, not bright, T2 signal. • Option D does not match the hemorrhagic content typical for endometriomas. 29. Which of the following is NOT a functioning pancreatic tumour? A. Insulinoma B. Gastrinoma C. Vipoma D. Glucagonoma E. Pancreatic cystadenoma Explanation: (by Perplexity AI, generated with the OpenAI O3 model) • Functioning pancreatic tumours are neuroendocrine neoplasms that secrete biologically active hormones, leading to characteristic clinical syndromes. • Classic examples include insulinoma (secreting insulin), gastrinoma (gastrin), VIPoma (vasoactive intestinal peptide), and glucagonoma (glucagon). These tumours present with specific hormonal syndromes such as hypoglycaemia, Zollinger-Ellison syndrome, Verner-Morrison syndrome, or necrolytic migratory erythema. • In contrast, pancreatic cystadenoma is a benign, non-functioning cystic neoplasm of the pancreas and does not produce hormones or cause endocrine syndromes—making E the correct answer. • Key distractors, such as glucagonoma, are rare but established functioning neuroendocrine tumours of the pancreas.
  • 162.
    EBDR Exam MCQs& Concepts May 2023 Dr. Kareem Alnakeeb 156 30. A 42-year-old man from an endemic region presents with persistent lower urinary tract symptoms and haematuria. Imaging reveals calcification of the urinary bladder wall. Which of the following is the most likely cause? A. Chronic cystitis B. Transitional cell carcinoma C. Schistosomiasis D. Bladder outlet obstruction E. Interstitial cystitis Explanation: (by Perplexity AI, generated with the OpenAI O3 model) • Schistosomiasis, caused by Schistosoma haematobium, is strongly associated with calcified urinary bladder walls, particularly in patients from endemic areas. The parasite’s eggs become embedded in the urothelium, leading to granulomatous inflammation and eventual calcification. • Tuberculosis (TB) can cause urinary tract calcification but is far less likely to produce a uniformly calcified bladder wall compared to schistosomiasis. • Chronic cystitis may cause irregular thickening but rarely true diffuse calcification. • Transitional cell carcinoma may create focal calcifications but does not involve the entire bladder wall. • Bladder outlet obstruction and interstitial cystitis do not cause bladder wall calcification. 31. A 56-year-old woman has a palpable left breast lump. Mammography categorises the lesion as BIRADS 2. What is the most appropriate next step? A. MRI B. Core biopsy C. Reassurance of the patient D. Excision biopsy E. Fine-needle aspiration cytology Explanation: (by Perplexity AI, generated with the OpenAI O3 model) • BIRADS 2 lesions are definitively benign by imaging criteria, requiring no further diagnostic intervention beyond routine screening. Reassurance is appropriate and no biopsy (core or excision) or MRI is indicated, since there is no suspicion of malignancy. • Biopsies (options B, D, E) are reserved for indeterminate or suspicious lesions (BIRADS 3-5). • MRI (option A) is not indicated when the lesion is unequivocally benign on standard imaging. 32. What is the primary imaging modality to differentiate solid from cystic lesions in clinical practice? A. Mammography B. Ultrasound C. CT D. MRI E. PET Explanation: (by Perplexity AI, generated with the OpenAI O3 model) • Ultrasound is the modality of choice for distinguishing solid from cystic lesions due to its real-time capability, high sensitivity in detecting fluid content, and ability to assess lesion vascularity with Doppler imaging. • Mammography provides limited information on internal lesion composition, primarily showing density.
  • 163.
    EBDR Exam MCQs& Concepts May 2023 Dr. Kareem Alnakeeb 157 • CT and MRI can characterize lesions but are less practical as first-line tools for this purpose due to cost, accessibility, and radiation (CT). • PET does not reliably distinguish cystic from solid lesions. 33. On DCE MRI of the breast, which time–signal intensity curve pattern best represents an intermediate (type II) curve, characterised by an initial signal rise of about 10% followed by a plateau phase? A. Continuous increase throughout imaging B. Initial rise and plateau after early enhancement C. Rapid rise then decrease (washout) D. No significant enhancement E. Both A & B Explanation: (by Perplexity AI, generated with the OpenAI O3 model) • The type II (intermediate) curve on DCE MRI of the breast is defined by an initial rise in signal intensity—commonly 10% or more—followed by a plateau phase, with minimal further change. This pattern is considered suspicious, as it may be seen in both benign and malignant lesions but is more indeterminate than type I (persistent/continuous increase, seen mostly in benign lesions) or type III (washout, strongly associated with malignancy). • Option B most accurately captures this classic type II (plateau) pattern. • Options A and E refer to the persistent enhancement seen in benign lesions, whereas C describes the washout (type III) typically linked to malignancy, and D is incorrect as it indicates absence of enhancement. 34. Which statement about breast cancer screening in low-risk women is most accurate? A. Screening ultrasound is preferred over mammography for women over 40 years. B. Contrast-enhanced mammography has entirely replaced standard mammography. C. MRI is routinely performed annually in all low-risk women. D. Ultrasound is the first-line screening tool for women under 40 years. E. All women should begin mammography screening by age 25. Explanation: (by Perplexity AI, generated with the OpenAI O3 model) • For low-risk women under 40 years, ultrasound is typically preferred as the first-line imaging modality due to higher breast density and the increased radiation risk in younger patients. • Mammography is the mainstay of screening from age 40, but for asymptomatic low-risk women below this age, routine screening with MRI or contrast-enhanced mammography is not recommended. • Annual MRI is reserved for high-risk categories. • Contrast-enhanced mammography is emerging but does not replace standard mammography for population screening. • Mammography before age 40–50 is not standard unless specific risk factors are present. 35. Which of the following is considered the most malignant ultrasonographic feature of lymph node malignancy? A. Increased size >10mm B. Infiltrated hilum C. Increased cortex D. Vascularity of the hilum E. Smooth, thin cortex
  • 164.
    EBDR Exam MCQs& Concepts May 2023 Dr. Kareem Alnakeeb 158 Explanation: (by Perplexity AI, generated with the OpenAI O3 model) • Infiltration or loss of the normal fatty hilum is the most malignant sonographic sign of lymph node malignancy, as it suggests tumor replacement of lymph node architecture. • While increased size (>10mm) and cortical thickness may be seen in both benign and malignant conditions, and vascularity of the hilum is typical of benign nodes, not malignant ones, the presence of a smooth, thin cortex is also a feature of benignity. • Key distractors (A, C, D) often occur with benign or reactive nodes, but infiltrated hilum (B) indicates a higher suspicion for malignancy. 36. A 60-year-old woman presents with a palpable lump in her right breast. Her recent screening mammogram 6 months previously was negative. Clinical examination reveals a subtle mass in the right lower quadrant. Which of the following mammographic findings is the most common in invasive lobular carcinoma (ILC)? A. Spiculated mass. B. Architectural distortion. C. Microcalcifi cation. D. Nipple retraction. E. Skin thickening. Source: Lindsay, Richard, et al. SBAs for the FRCR Part 2A. 1st ed., Oxford University Press, 2012. Explanation: • ILC is the second most common form of invasive breast cancer, after ductal carcinoma. It exhibits the same mammographic features as invasive ductal carcinoma, although architectural distortion is the most common mammographic finding. Due to the pattern of small cells growing around ducts ( ‘Indian files’), mammographic findings are subtle and thus ILC is the most frequently missed breast cancer. Prognosis is generally poor due to late diagnosis. 37. Which one of the following is the most characteristic mammographic feature of a malignant breast mass? A. Spiculated mass and clustered microcalcification B. Smooth margins C. Ill-defined infiltrative mass with macrocalcification D. Asymmetrical distribution and nipple retraction E. Well-circumscribed mass with uniform density Explanation: (by Perplexity AI, generated with the OpenAI O3 model) • Spiculated mass margins and clustered microcalcifications are highly suggestive of malignancy on mammography, reflecting irregular tumour infiltration and ductal carcinoma in situ components. • Smooth margins and well-circumscribed masses (options B and E) are more typical of benign lesions such as fibroadenomas or cysts. • Ill-defined infiltrative mass with macrocalcification (option C) is less specific for malignancy— macrocalcifications usually represent benign involution. • Asymmetrical distribution and nipple retraction (option D) are concerning, but on their own are less specific than the classical combination in option A.
  • 165.
    EBDR Exam MCQs& Concepts May 2023 Dr. Kareem Alnakeeb 159 38. A 54-year-old female patient presents with anaemia and haematuria. A CT of abdomen confi rms renal cell carcinoma of the right kidney, but there is also enlargement of the right adrenal gland. Which of the following CT characteristics is most consistent with a benign adrenal adenoma? A. A pre-contrast attenuation of 50. B. An immediate post-contrast attenuation of 50. C. A relative percentage washout (RPW) of 60%. D. Lesion size of 50 mm. E. Heterogeneity of the lesion. Source: Lindsay, Richard, et al. SBAs for the FRCR Part 2A. 1st ed., Oxford University Press, 2012. Explanation: Adrenal adenoma diagnostic criteria • Findings consistent with an adrenal adenoma are: a pre-contrast attenuation of 10 HU or less, an absolute percentage washout (APW) of 60% or greater, or an RPW of 40% or greater. Percentage washout calculations • The percentage washout is calculated by comparing the attenuation value at 15 minutes post contrast (delayed H), to the value in the portal venous phase (enhanced H), and in the case of APW, the pre-contrast value. • RPW = 100 × (enhanced H – delayed H)/enhanced H • APW = 100 × (enhanced H – delayed H)/(enhanced H – pre-contrast H) Practical imaging considerations • In practice, an unenhanced scan is not usually performed and thus only the RPW is calculated. Adrenal cortical carcinoma indicators • Adrenal cortical carcinomas usually have an RPW of less than 40% although exceptions have been reported. • Their large size (usually greater than 6 cm), heterogeneity pre-contrast (necrosis), and heterogeneous enhancement are more reliable indicators of the diagnosis. Other lesions that may mimic adenoma • Phaeochromocytomas and hypervascular metastases may mimic adenomas, but most metastases show RPW < 40% and APW < 60%. 39. A 14-year-old girl presents with sudden onset of severe right-sided lower abdominal pain, nausea, and vomiting. An ultrasound of the pelvis is requested with a suspected diagnosis of ovarian torsion. Which of the following is the most constant ultrasound finding in ovarian torsion? A. Enlarged ovary. B. Absent ovarian blood flow. C. Pelvic free fluid. D. Twisted ovarian pedicle. E. ‘String of pearls’ sign. Source: Lindsay, Richard, et al. SBAs for the FRCR Part 2A. 1st ed., Oxford University Press, 2012. Explanation: Prevalence and Risk Factors • Ovarian torsion can occur in all age groups, with the highest prevalence in the reproductive age group. • Large heavy cysts and cystic neoplasms commonly predispose to ovarian torsion. • Torsion of normal ovaries is unusual but more common in adolescents. • Benign cystic teratoma is the most common tumour predisposed to ovarian torsion.
  • 166.
    EBDR Exam MCQs& Concepts May 2023 Dr. Kareem Alnakeeb 160 Clinical Presentation • Clinical symptoms are often nonspecific and therefore imaging is routinely requested. Ultrasound Findings • On ultrasound, an enlarged ovary is the most constant finding. • The enlarged ovary may be heterogeneous due to haemorrhage and oedema. • Other findings include multiple small cysts aligned in the periphery of the enlarged ovary (string of pearls sign), coexistent mass, pelvic free fluid, and twisted ovarian pedicle. Doppler Findings • Absent arterial flow is described as the classic colour Doppler finding. • However, the most frequent Doppler finding is decreased or absent venous flow with or without reduced arterial flow. • A twisted ovarian pedicle with a ‘whirlpool’ sign is a useful finding on colour Doppler. 40. What is the most common soft tissue mass found in the hand and wrist? A. Hemangioma B. Giant cell tumor of tendon sheath C. Lipoma D. Epidermoid cyst E. Ganglion cyst Explanation: (by Perplexity AI, generated with the OpenAI O3 model) • Ganglion cysts are the most frequent soft tissue masses in the hand and wrist, encountered in both adult and paediatric populations. They typically arise near joint capsules or tendon sheaths, presenting as firm, well-defined swellings—often on the dorsal wrist. • In contrast, lipomas are rare in this location, hemangiomas are more common in childhood, and giant cell tumours of tendon sheath—while possible—are less prevalent than ganglion cysts. • Epidermoid cysts are also uncommon in this anatomical area. Identification relies on clinical history, examination, and imaging, especially ultrasound or MRI for atypical cases. 41. Which anatomical site is most commonly affected by Freiberg’s avascular necrosis in the foot? A. Navicular B. Scaphoid C. Second metatarsal head D. Talus E. Fifth metatarsal base Explanation: (by Perplexity AI, generated with the OpenAI O3 model) • Freiberg’s disease refers to avascular necrosis (AVN) of the metatarsal head, most classically affecting the second metatarsal. This is due to its length and relative vulnerability to altered biomechanics and blood supply, especially in adolescent females. • The navicular and scaphoid are both prone to AVN, but in different contexts: the navicular is classically affected in Kohler’s disease, and the scaphoid (in the wrist) in the context of fracture. • The talus and fifth metatarsal base are not commonly associated with Freiberg’s AVN.
  • 167.
    EBDR Exam MCQs& Concepts May 2023 Dr. Kareem Alnakeeb 161 42. Which of the following is NOT a recognized radiological sign of intestinal tuberculosis? A. Fleischner sign B. String sign C. Stierlin's sign D. Comb sign Explanation: (by Perplexity AI, generated with the OpenAI O3 model) • The comb sign (engorged mesenteric vessels) is typically seen in Crohn’s disease, not intestinal tuberculosis. • Fleischner sign (thickened loop due to spasm), string sign (narrow, rigid segment), and Stierlin's sign (rapid passage through ulcerated segment with minimal mucosal coating) are all described in intestinal TB and help differentiate it from other pathologies. • The comb sign is a classic finding in active Crohn’s, reflecting hypervascularity, and is not associated with tuberculosis. 43. A 36-year-old female presents with progressive lower abdominal distension and menorrhagia. On examination, there is a large, firm, irregular pelvic mass arising from the pelvis. What is the most likely diagnosis? A. Endometriosis B. Uterine fibroid C. Dermoid cyst D. Mesenteric cyst E. Ovarian carcinoma Explanation: (by Perplexity AI, generated with the OpenAI O3 model) • The most likely diagnosis is a uterine fibroid (leiomyoma), which typically presents as a large, firm, irregular pelvic mass associated with menorrhagia in reproductive-age women. • Fibroids are hormone-dependent benign smooth muscle tumors of the uterus and classically cause heavy menstrual bleeding and mass effect symptoms. • Endometriosis may cause pelvic pain and dysmenorrhoea but rarely presents with a palpable mass of this size. • Dermoid cysts (mature teratomas) are ovarian in origin but usually not associated with menorrhagia or irregular, firm uterine masses. • Mesenteric cysts are rare, extrapelvic, and non-gynecological, and do not cause uterine-related symptoms. • Ovarian carcinoma often presents later in life and with non-specific symptoms or systemic features. 44. What is the most common primary malignant retroperitoneal tumor encountered in adults? A. Neuroblastoma B. Liposarcoma C. Leiomyosarcoma D. Malignant lymphoma E. Synovial sarcoma Explanation: (by Perplexity AI, generated with the OpenAI O3 model) • Liposarcoma is the most common primary malignant retroperitoneal tumor in adults. It accounts for around a third of all retroperitoneal sarcomas. The large retroperitoneal space allows these tumors to grow to a significant size before becoming symptomatic. • Leiomyosarcoma is the second most common but occurs less frequently than liposarcoma. • Neuroblastoma is a pediatric tumor seen predominantly in children.
  • 168.
    EBDR Exam MCQs& Concepts May 2023 Dr. Kareem Alnakeeb 162 • Malignant lymphoma can involve the retroperitoneum but is not a primary retroperitoneal sarcoma. • Synovial sarcoma is rare in this location. 45. A 25-year-old woman presents with an incidental abdominal mass on ultrasound. CT reveals a well- defined, unilocular cystic lesion involving multiple retroperitoneal compartments, with no enhancement or calcification. Which is the most likely diagnosis? A. Lymphatic malformation B. Pancreatic pseudocyst C. Retroperitoneal sarcoma D. Paraganglioma E. Mesenteric cyst Explanation: (by Perplexity AI, generated with the OpenAI O3 model) • Lymphatic malformations in the retroperitoneum typically present as well-defined unilocular or multilocular cystic lesions that may cross anatomical boundaries or compartments. They lack enhancement and are usually not associated with calcification or solid components. • Pancreatic pseudocysts generally relate to a history of pancreatitis and usually do not cross compartments. • Retroperitoneal sarcomas are more often solid, heterogeneous, and may enhance. • Paragangliomas are vascular and enhancing masses, not cystic. • Mesenteric cysts are usually confined to the mesentery and do not typically involve multiple retroperitoneal compartments. 46. An 18-year-old male fractures his pelvis following a motorcycle accident. He is suspected of sustaining a bladder injury and undergoes CT cystography. This reveals ill-defi ned contrast medium within the peri- vesical space with a ‘molar-tooth’ appearance. What is the signifi cance of this fi nding? A. Interstitial bladder injury. B. Intraperitoneal rupture. C. Extraperitoneal rupture. D. Combined intra- and extraperitoneal rupture. E. Bladder contusion. Source: Lindsay, Richard, et al. SBAs for the FRCR Part 2A. 1st ed., Oxford University Press, 2012. Explanation: Epidemiology and Imaging • Over 70% of patients with traumatic bladder injury have a coexisting pelvic fracture. • CT cystography is considered to be as accurate as conventional cystography. Extraperitoneal Rupture • Extraperitoneal rupture accounts for 80% of all cases of traumatic bladder injury. • It occurs as a result of shearing forces or penetrating injury from bony fragments at the base of the bladder. • Contrast can also track down into the scrotum or thigh. Intraperitoneal Rupture • Intraperitoneal rupture (15% of cases) follows a direct blow to a distended bladder, with the tear involving the bladder dome. • Contrast will be seen to outline small bowel loops. Other Injury Types • Combined injuries occur in 5%. • Interstitial injuries are rare and are detected by contrast dissecting into the bladder wall.
  • 169.
    EBDR Exam MCQs& Concepts May 2023 Dr. Kareem Alnakeeb 163 Bladder Contusion • Imaging is frequently normal in the setting of bladder contusion. 47. A 66-year-old man is diagnosed with a pancreatic body mass involving the celiac axis on CT. According to widely used resectability criteria, which imaging feature confirms the tumour is unresectable? A. Less than 180° tumour contact with the celiac axis B. Greater than 180° tumour contact with the celiac axis without aortic involvement C. Isolated splenic artery encasement D. Less than 180° tumour contact with the common hepatic artery E. Greater than 180° tumour contact with the celiac axis with aortic involvement Explanation: (by Perplexity AI, generated with the OpenAI O3 model) • Tumour contact of greater than 180° with the celiac axis involving the aorta (encasement) renders a pancreatic tumour unresectable according to major guidelines, as this reflects extensive arterial invasion not amenable to curative surgery. • Tumours with less extensive encasement (>180° celiac axis contact without aortic involvement) may still be approached with advanced surgical techniques in select cases after neoadjuvant therapy, and isolated splenic artery or less common hepatic artery involvement do not preclude resection. • Only E features definitive local arterial extension prohibiting surgery. 48. A 56-year-old man with a history of chronic gastroesophageal reflux presents with progressive dysphagia. Endoscopy reveals a granular-appearing mucosa in the distal esophagus and a mid-esophageal stricture. Which is the most likely underlying histopathological diagnosis? A. Squamous cell carcinoma B. Eosinophilic esophagitis C. Barrett’s esophagus with intestinal metaplasia D. Infective esophagitis E. Achalasia Explanation: (by Perplexity AI, generated with the OpenAI O3 model) • Barrett’s esophagus is characterized by replacement of the normal squamous epithelium with metaplastic columnar epithelium, typically in the distal esophagus, usually due to chronic reflux. Granular mucosa described on endoscopy is a typical finding, and chronic acid injury can rarely lead to strictures even in the mid-esophagus. • Squamous cell carcinoma may also cause strictures but would more likely present as an ulcerated or mass-forming lesion. • Eosinophilic esophagitis often causes rings and linear furrows, not granular Barrett’s mucosa. • Infective esophagitis presents with ulceration, and achalasia is a motility disorder rarely producing these mucosal changes.
  • 170.
    EBDR Exam MCQs& Concepts May 2023 Dr. Kareem Alnakeeb 164 49. A 67 year old man is referred for a barium swallow from the surgical outpatient department with a history of dysphagia to solids. A mid-oesophageal stricture is demonstrated. Which one of the following causes is unlikely to be in the differential? A. Barrett’s oesophagus B. Squamous cell carcinoma of the oesophagus C. Schatzki ring D. Caustic substance ingestion E. Epidermolysis bullosa Source: Currie, Stuart, et al. SBAs for the FRCR 2A. 1st ed., Cambridge University Press, 2010. Explanation: • All are reasonable differentials for a mid-oesophageal stricture, albeit with varying degrees of frequency, with the exception of a Schatzki ring which is found in the lower oesophagus. • It occurs near the squamocolumnar junction and is associated with reflux. • It is nondistensible and best seen in the prone position on barium swallow examinations. • Schatzki rings are often asymptomatic, but oesophageal dilatation may be required where dysphagia is severe. 50. Which of the following combinations is required to make the diagnosis of hydrops fetalis? A. Pleural effusion and mild polyhydramnios B. Skin edema and pleural effusion C. Single-organ ascites D. Increased nuchal translucency alone E. Pericardial effusion alone Explanation: (by Perplexity AI, generated with the OpenAI O3 model) • Hydrops fetalis is defined by the abnormal accumulation of fluid in at least two fetal compartments, such as skin edema, pleural effusion, pericardial effusion, or fetal ascites. • Therefore, the presence of both skin edema and pleural effusion (Option B) satisfies this definition. Single findings alone (e.g., E or C) and isolated non-compartmental fluid (e.g., nuchal translucency or polyhydramnios) do not meet the diagnostic threshold. 51. A 60-year-old man with a history of prior myocardial infarction undergoes myocardial perfusion scintigraphy, which shows decreased perfusion alongside reduced contractility in the affected region. What is the most likely explanation for these findings? A. Myocardial stunning B. Hibernating myocardium C. Acute infarction D. Hypertrophic cardiomyopathy E. Restrictive cardiomyopathy Explanation: (by Perplexity AI, generated with the OpenAI O3 model) • Hibernating myocardium refers to chronic, viable myocardial tissue that demonstrates both decreased perfusion and reduced contractility due to prolonged but reversible ischaemia. This state persists until adequate perfusion is restored, at which point function may improve. • Myocardial stunning (A) presents with reduced contractility but normal perfusion following transient ischaemia. • Acute infarction (C) often results in irreversible loss of contractility and perfusion due to cell death. • Hypertrophic (D) and restrictive (E) cardiomyopathy commonly cause abnormal contractility but are not typically associated with focal perfusion deficits related to reversible ischaemia.
  • 171.
    EBDR Exam MCQs& Concepts May 2023 Dr. Kareem Alnakeeb 165 52. A 55-year-old woman with long-standing systemic sclerosis presents with worsening dysphagia; a recent barium swallow (shown) demonstrates a smoothly dilated, aperistaltic distal two-thirds of the esophagus with rapid emptying into the stomach. Which imaging pattern is most characteristic of this finding? A. Achalasia (“bird-beak” taper) B. Scleroderma oesophagus C. Diffuse oesophageal spasm (“corkscrew” oesophagus) D. Zenker diverticulum E. Oesophageal carcinoma causing stricture Explanation: (by Perplexity AI, generated with the OpenAI O3 model) • In systemic sclerosis, atrophy and fibrosis of smooth muscle cause loss of peristalsis and lower oesophageal sphincter incompetence, producing a smooth, tubular dilatation of the distal oesophagus that empties freely into the stomach—the classic scleroderma pattern. Achalasia instead shows a narrowed distal “bird-beak” with poor emptying; diffuse spasm gives a corkscrew appearance; Zenker diverticulum is a pharyngeal pouch, not distal; malignant stricture produces an irregular, shouldered narrowing, not uniform dilatation. 53. A patient has a dual phase subtraction study for investigation of hyperparathyroidism and a focus of uptake is seen on the Tc-99m-mibi scan with a corresponding area of increased uptake on the I-123 study in the region of upper pole of the right lobe of the gland. What is the most likely cause for this finding? A. Functioning parathyroid adenoma B. Papillary thyroid tumour C. Multinodular goitre with prominent nodule D. Solitary functioning thyroid nodule E. Submandibular salivary gland Source: Proctor, Robin. Final FRCR Part A Modules 4-6 Single Best Answer MCQs: The SRT Collection of 600 Questions with Explanatory Answers (MasterPass). 1st ed., CRC Press, 2009. Explanation: • Multinodular goitre can give rise to heterogeneous uptake in both types of scan as a functioning thyroid nodule will take up both tracers. • The presence of smooth uniform uptake on the thyroid scan would have been more consistent with a functioning parathyroid adenoma. • Salivary gland uptake is seen with Tc-99m pertechnetate but not I131 . • Thyroid malignancy presents as a cold (nonfunctioning) nodule in 90% of scans. 54. Regarding transient osteoporosis of the femoral head, which of the following statements is INCORRECT? A. It is more common in females. B. MRI typically demonstrates bone-marrow oedema. C. Radiographs show a grade III avascular necrosis appearance of the femoral head. D. An associated hip joint effusion is often present. E. The condition is self-limiting and usually resolves within months. Explanation: (by Perplexity AI, generated with the OpenAI O3 model) • Transient osteoporosis (TOH) most frequently affects middle-aged men and women in the third- trimester of pregnancy, so overall prevalence is higher in males, not females; MRI shows diffuse marrow oedema with low T1 and high T2/STIR signal, and hip effusion is common, while the disease follows a benign, self-limiting course resolving within 6–12 months. • In contrast, a grade III avascular necrosis radiographic appearance (crescent sign/collapse) indicates structural failure of subchondral bone and is not a feature of TOH, thereby making option C incorrect.
  • 172.
    EBDR Exam MCQs& Concepts May 2023 Dr. Kareem Alnakeeb 166 • The other options describe recognised characteristics of TOH. 55. A 40-year-old male with a history of haematuria undergoes CT urography. Initial non-contrast scan demonstrates right-sided medullary nephrocalcinosis. Following intravenous contrast administration, a striated ‘paintbrush’ appearance of the renal medulla is noted. The left kidney is unremarkable. What is the diagnosis? A. Hyperparathyroidism. B. Renal tubular acidosis. C. Medullary sponge kidney. D. Sarcoidosis. E. Multiple myeloma. Source: Lindsay, Richard, et al. SBAs for the FRCR Part 2A. 1st ed., Oxford University Press, 2012. Explanation: Causes of Medullary Nephrocalcinosis • Hyperparathyroidism, renal tubular acidosis, and medullary sponge kidney are the three most common causes of medullary nephrocalcinosis. • The former two conditions are associated with hypercalciuria that results in uniform medullary nephrocalcinosis. • Sarcoidosis and multiple myeloma are associated with hypercalcemia resulting in bilateral nephrocalcinosis. Medullary Sponge Kidney Characteristics • Medullary sponge kidney can affect the kidney segmentally, unilaterally, or bilaterally, therefore unilateral nephrocalcinosis is suggestive of medullary sponge kidney. • Medullary sponge kidney is characterized by cystic dilatation of collecting tubules. • Urine stasis within the dilated tubules predisposes to infection and calculus formation within the dilated tubules or urinary tracts. • On excretory urogram, contrast within the dilated tubules produces a striated ‘paintbrush’ appearance of the renal pyramids. 56. An 18-year-old male fractures his pelvis following a motorcycle accident. He is suspected of sustaining a bladder injury and undergoes CT cystography. This reveals ill-defi ned contrast medium within the peri- vesical space with a ‘molar-tooth’ appearance. What is the signifi cance of this fi nding? A. Interstitial bladder injury. B. Intraperitoneal rupture. C. Extraperitoneal rupture. D. Combined intra- and extraperitoneal rupture. E. Bladder contusion. Source: Lindsay, Richard, et al. SBAs for the FRCR Part 2A. 1st ed., Oxford University Press, 2012. Explanation: Epidemiology and Fracture Association • Over 70% of patients with traumatic bladder injury have a coexisting pelvic fracture. Diagnostic Imaging • CT cystography is considered to be as accurate as conventional cystography. Extraperitoneal Rupture • Extraperitoneal rupture accounts for 80% of all cases of traumatic bladder injury. • It occurs as a result of shearing forces or penetrating injury from bony fragments at the base of the bladder. • Contrast can also track down into the scrotum or thigh.
  • 173.
    EBDR Exam MCQs& Concepts May 2023 Dr. Kareem Alnakeeb 167 Intraperitoneal Rupture • Intraperitoneal rupture (15% of cases) follows a direct blow to a distended bladder, with the tear involving the bladder dome. • Contrast will be seen to outline small bowel loops. Combined and Interstitial Injuries • Combined injuries occur in 5%. • Interstitial injuries are rare and are detected by contrast dissecting into the bladder wall. Bladder Contusion • Imaging is frequently normal in the setting of bladder contusion. 57. A 1.5 cm well-defined hepatic lesion shows markedly hyperintense T2 signal, low T1 signal, and intense, homogeneous arterial-phase enhancement that persists without wash-out on dynamic MRI. What is the most likely diagnosis? A. Hepatocellular carcinoma B. Flash-filling hepatic haemangioma C. Focal nodular hyperplasia D. Hypervascular metastasis (renal cell carcinoma) E. Cholangiocarcinoma Explanation: (by Perplexity AI, generated with the OpenAI O3 model) • Flash-filling (capillary) haemangiomas are typically small (<2 cm) lesions that are T1 hypointense and T2 markedly hyperintense. They “flash-fill” with brisk, uniform arterial enhancement that matches blood pool and retain contrast on portal and delayed phases, showing no true wash-out—distinct from hypervascular malignancies such as HCC or metastases, which usually become hypo- or iso- intense later. • Focal nodular hyperplasia shows a central scar and becomes iso-enhancing on delayed images, while cholangiocarcinoma is usually peripheral, fibrous, and shows progressive enhancement rather than rapid homogeneous fill-in. 58. On ultrasound, gall-bladder “smudge” artefact is noted as an echogenic focus on the wall with internal colour Doppler flow. Which is the MOST appropriate interpretation of this finding in routine practice? A. Gall-bladder wall adenomyomatosis B. Impacted cholesterol calculus C. Early gall-bladder carcinoma D. Hyperplastic cholecystosis (strawberry gall-bladder) E. Biliary sludge ball Explanation: (by Perplexity AI, generated with the OpenAI O3 model) • The “smudge” or “comet-tail” appearance with intralesional Doppler flow represents a Rokitansky- Aschoff sinus containing cholesterol crystals in segmental or focal adenomyomatosis; colour flow is due to twinkling artefact, not vascularity. • Stones and sludge (options B and E) usually cast an acoustic shadow and demonstrate no internal Doppler signal. • Hyperplastic cholecystosis (option D) produces diffuse mucosal speckling without a discrete wall- based focus. • Gall-bladder carcinoma (option C) manifests as a mural mass or irregular wall thickening and genuine vascular flow, but true malignancy is rarely linked to the isolated “smudge” sign.
  • 174.
    EBDR Exam MCQs& Concepts May 2023 Dr. Kareem Alnakeeb 168 59. Which statement regarding gallbladder carcinoma is CORRECT in relation to patients under 40 years of age? A. It is common and almost always associated with gallstones B. It is common and rarely associated with gallstones C. It is rare but still almost always associated with gallstones D. It is rare and gallstones are present in only about half of cases E. It never occurs because gallstones are absent at young age Explanation: (by Perplexity AI, generated with the OpenAI O3 model) • Gallbladder carcinoma is distinctly uncommon in patients younger than 40 years, but when it does present, cholelithiasis is identified in roughly 50% of cases. Thus, the disease is both rare in the young and not universally linked to stones. • Option A is incorrect because although stones are a recognised risk factor, the tumour is not common in this age group. • Option B is wrong: stone association is not rare. • Option C overstates the frequency of stone presence, while option E is false because gallbladder cancer, though infrequent, can occur despite the lower prevalence of stones in youth. 60. A 58-year-old man presents with cramping abdominal pain and vomiting. CT enterography shows a 4 cm, well-circumscribed exophytic mass arising from the jejunal wall and projecting both intraluminally and extraluminally; mottled enhancement and central areas of low attenuation are noted. Which is the most likely diagnosis? A. Adenocarcinoma of the small bowel B. Gastrointestinal stromal tumour (GIST) C. Carcinoid tumour D. Small-bowel lymphoma E. Ectopic pancreatic rest Explanation: (by Perplexity AI, generated with the OpenAI O3 model) • GISTs are the second commonest benign small-bowel tumours after lipomas and typically originate from the muscularis propria, producing mixed transmural–exophytic growth that can both obstruct the lumen and extend outward. On CT they appear as well-defined, enhancing soft-tissue masses that may contain necrotic or cystic areas, exactly as in this case. • Primary small-bowel adenocarcinomas usually arise in the mucosa, are irregular and infiltrative rather than well-circumscribed. • Carcinoid tumours tend to be submucosal, small, intensely enhancing nodules often accompanied by desmoplastic mesenteric stranding. • Lymphoma often shows diffuse or segmental bowel wall thickening with bulky lymphadenopathy and rarely causes obstruction due to its pliable nature. • An ectopic pancreatic rest is a small submucosal nodule with central umbilication and rarely exceeds 2 cm.
  • 175.
    EBDR Exam MCQs& Concepts May 2023 Dr. Kareem Alnakeeb 169 61. Regarding eosinophilic granuloma, which of the following are true? (True or False) A. Bone lesions are usually solitary B. Vertebral pedicles are typically affected C. The cranial vault is rarely involved D. The mandible is rarely involved E. A periosteal reaction is not seen on plain film Source: Bell, J., and N. Davies. MCQs in Clinical Radiology: A Revision Guide for the FRCR. 1st ed., Remedica Pub Ltd, 2004. Explanation: A. True - About 75% of cases are monostotic. B. False - Eosinophilic granuloma typically involves the vertebral body, most commonly the thoracic spine, causing vertebra plana. It rarely involves the posterior elements and the disc spaces are preserved. C. False - The skull is the most frequent site of involvement. Typically there is a round or oval lucency within the skull vault with a bevelled edge. D. False - It often involves the mandible, causing a 'floating teeth' appearance. E. False - Although a periosteal reaction is almost never seen in flat bones, a prominent periosteal reaction is not unusual in the axial skeleton. This may simulate a neoplastic lesion. 62. In cervical spine trauma, which fracture pattern is associated with the highest risk of catastrophic spinal cord injury and thus considered the most serious? A. Flexion teardrop fracture B. Hangman fracture C. Extension teardrop fracture D. Burst fracture of C1 (Jefferson fracture) E. Clay-shoveller fracture Explanation: (by Perplexity AI, generated with the OpenAI O3 model) • Flexion teardrop fractures occur after high-energy flexion-compression, fracturing the anteroinferior vertebral body corner and typically disrupting the posterior ligamentous complex. This often produces severe kyphotic angulation and retropulsion of a large osseous fragment into the spinal canal, leading to a high incidence of complete spinal cord injury. • Hangman fractures (bilateral C2 pars fractures) usually spare the spinal canal, so neurological deficit is uncommon. • Extension teardrop fractures are avulsion injuries of the anterior inferior corner and, while unstable, carry a lower neurological risk than flexion variants. • Jefferson fractures are burst injuries of C1 with widened ring fragments that rarely cause cord damage because the canal diameter increases. • Clay-shoveller fractures are isolated lower cervical/thoracic spinous process avulsions and are stable with negligible neurological risk.
  • 176.
    EBDR Exam MCQs& Concepts May 2023 Dr. Kareem Alnakeeb 170 63. A 34-year-old woman with long-standing Hashimoto thyroiditis presents with biochemical thyrotoxicosis. Pelvic ultrasound shows a 6 cm complex cystic-solid lesion in the left ovary; colour Doppler is unremarkable. Which diagnosis best explains both the ovarian mass and her thyroid overactivity? A. Dysgerminoma B. Granulosa cell tumour C. Mature cystic teratoma containing struma ovarii D. Serous cystadenocarcinoma E. Theca-lutein cyst Explanation: (by Perplexity AI, generated with the OpenAI O3 model) • Struma ovarii is a rare monodermal variant of mature cystic teratoma composed predominantly of thyroid tissue; functioning lesions can secrete thyroid hormone and cause thyrotoxicosis. Imaging typically shows a complex cystic-solid ovarian mass with variable echogenic components. • Dysgerminomas and granulosa cell tumours do not contain thyroid tissue and are not linked to hyperthyroidism. • Serous cystadenocarcinoma is usually multilocular with papillary projections but likewise lacks thyroid tissue and endocrine effects. • Theca-lutein cysts arise from gonadotropin overstimulation, are usually bilateral and cystic, and do not produce thyroid hormones. 64. A 24-year-old woman attends A&E with lower abdominal pain and vaginal bleeding. A pregnancy test is positive. She is hemodynamically stable, and an ultrasound is requested to confirm the presumed diagnosis of an ectopic pregnancy. Which of the following is the most common location for an ectopic pregnancy? A. Cervix B. Ovary C. Abdominal cavity D. Ampullary portion of the fallopian tube E. Interstitial portion of the fallopian tube Source: Currie, Stuart, et al. SBAs for the FRCR 2A. 1st ed., Cambridge University Press, 2010. Explanation: • The most common site of implantation is the fallopian tube, which accounts for over 90% of ectopic pregnancies. • Ovarian and abdominal sites account for only approximately 3% and 1%, respectively. • Within the fallopian tube the most common site is the ampulla (73%) followed by the fimbrial and interstitial regions. 65. A 24 year old man is referred for an ultrasound examination following blunt trauma to the scrotum. Which of the following is not true? A. The left testis is more susceptible to blunt trauma B. Intratesticular hematomas need to be followed up until resolution C. Penetrating injuries are more likely to be bilateral compared to blunt injuries D. An ultrasound finding of an intact tunica albuginea allows the confident exclusion of a testicular rupture in the absence of a hematocoele E. An atrophic testis is more likely to dislocate
  • 177.
    EBDR Exam MCQs& Concepts May 2023 Dr. Kareem Alnakeeb 171 Source: Currie, Stuart, et al. SBAs for the FRCR 2A. 1st ed., Cambridge University Press, 2010. Explanation: • The testis suffers blunt trauma against the thigh or the symphysis pubis and the right testis, being higher, is more susceptible. • Intratesticular haematomas should be followed up to resolution to rule out an underlying neoplasm and also rule out any ensuing complications such as abscess formation which may necessitate orchidectomy. 66. A 36 year old man suffers pelvic fracture following a road traffic accident. On examination, blood is noted at the urethral meatus and the patient has urinary retention. Regarding urothelial injuries: A. Associated bladder injuries are seen in 50% of patients B. Anterior urethral injuries are commoner with pelvic fractures C. They are more commonly associated with pelvic fractures in females rather than males D. Posterior urethral injuries can be seen in up to 20% of pelvic fractures in males E. Impotence is a rare complication of male urethral injury Source: Currie, Stuart, et al. SBAs for the FRCR 2A. 1st ed., Cambridge University Press, 2010. Explanation: • Urethral injuries are seen in up to 20% of male patients following pelvic fractures. • They are much less common in women. • The posterior urethra is the commonest site; impotence can develop in up to 40% of these patients. 67. A 40 year old female presents with bitemporal hemianopia. CT brain shows a large, slightly hyperdense suprasellar lesion. The mass contains several lucent foci and there is bone erosion of the sella floor. There is enhancement post-contrast. T1-weighted MR imaging shows a predominantly isointense mass causing sella expansion and compression of the optic chiasm. The mass contains foci of low and high signal intensity. What is the most likely diagnosis? A. Craniopharyngioma B. Meningioma C. Rathke’s cleft cyst D. Giant internal carotid aneurysm E. Pituitary adenoma Source: Currie, Stuart, et al. SBAs for the FRCR 2A. 1st ed., Cambridge University Press, 2010. Explanation: Pituitary Adenoma Classification • Pituitary adenomas are divided into microadenomas (<1 cm) and macroadenomas (>1 cm). • Macroadenomas may present with endocrine dysfunction but are generally less active than microadenomas. Clinical Presentation of Macroadenomas • Macroadenomas often present with symptoms of mass effect on the optic chiasm. • If there is lateral extension into the cavernous sinuses patients may present with other local cranial nerve palsies (III, IV, VI). Differential Diagnosis of a Suprasellar Mass (‘SATCHMO’) • The differential diagnosis of a suprasellar mass includes: Suprasellar extension of pituitary adenoma/sarcoid; Aneurysm/arachnoid cyst; TB/teratoma (other germ-cell tumours); Craniopharyngioma; Hypothalamic glioma or hamartoma; Meningioma/metastases (especially breast); and Optic/chiasmatic glioma. Imaging Findings in This Case
  • 178.
    EBDR Exam MCQs& Concepts May 2023 Dr. Kareem Alnakeeb 172 • In this case, the sellar is widened and the floor is eroded suggesting the mass arises from the pituitary itself. • Low-density/low-intensity regions on CT/T1 MRI correspond to necrotic areas and high-signal foci on T1 MRI (found relatively frequently) represent areas of recent haemorrhage. 68. A 49 year old African male presents to the outpatient urology clinic with a five-month history of macroscopic hematuria. A plain KUB X-ray is requested, which reveals thin arcuate calcification outlining the bladder and the distal ureters. Which one of the following causes is most likely? A. Transitional cell carcinoma B. Squamous cell carcinoma C. Schistosomiasis D. E. coli cystitis E. Proteus cystitis Source: Currie, Stuart, et al. SBAs for the FRCR 2A. 1st ed., Cambridge University Press, 2010. Explanation: • The differential for bladder calcification includes tuberculosis, post-radiotherapy cystitis, urachal carcinoma, TCC, and squamous cell carcinoma. • However, schistosomiasis is the commonest cause, especially in the African population, where it is often endemic. • The bladder is usually a normal size and shape, with thin curvilinear calcifications. • Ureteric strictures, inflammatory pseudopolyps and vesicoureteric reflux are seen in addition to bladder and ureteric calcification. 69. Triple rule out MDCT exam is used for diagnosis of A. Pulmonary embolism B. Coronary artery stenosis C. Aortic dissection D. Aortic aneurysmal dilatation E. All of the above Explanation: (by Perplexity AI, generated with the OpenAI O3 model) Triple-rule-out (TRO) multidetector CT • A TRO MDCT scan is designed to evaluate three potentially fatal causes of acute chest pain in one contrast-enhanced acquisition: o pulmonary embolism in the pulmonary arteries o coronary artery stenosis (acute coronary syndrome) in the coronary arteries o aortic pathology—most importantly acute aortic dissection, but the same acquisition also shows aortic aneurysmal dilatation • Because the single scan simultaneously interrogates all three vascular territories, it is intended to “rule out” all of them at once. • Aortic aneurysmal dilatation (choice D) is usually evident on the same dataset, but TRO is conceptually aimed at acute life-threatening causes of chest pain; aneurysmal dilatation alone is not an acute emergency and is not one of the three traditional targets. • Therefore the correct option is: E. All of the above – but only because the scan inevitably depicts an aneurysm if present, even though the classic “triple” refers to embolism, dissection, and coronary stenosis.
  • 179.
    EBDR Exam MCQs& Concepts May 2023 Dr. Kareem Alnakeeb 173 70. MRI Steady State Free Precession sequence (SSFP) is used for: A. Evaluation of left ventricular function B. Evaluation of night ventricular function C. Diagnosis of myocardial wall hypokinesia D. None of the above E. All of the above Explanation: (by Perplexity AI, generated with the OpenAI O3 model) • Balanced steady-state free precession (SSFP) cine sequences are the routine “work-horse” of cardiac MRI because they deliver bright-blood images with high temporal resolution. They are therefore simultaneously used for o quantitative left ventricular (LV) functional analysis (volumes, mass, ejection fraction) , o quantitative right ventricular (RV) functional analysis (volumes, ejection fraction) , and o visual identification of regional wall-motion abnormalities such as hypokinesia, akinesia or dyskinesia 71. Anomalous right upper pulmonary vein draining within into SVC Is associated with: A. Patent foramen oval B. ASD (septum primum Type) C. VID D. ASD (sinus venosus type) E. None of the above Explanation: (by Perplexity AI, generated with the OpenAI O3 model) • Partial anomalous pulmonary venous return in which the right upper (or right superior) pulmonary vein drains into the superior vena cava is classically linked to a superior (high) sinus venosus atrial septal defect. • Case series and reviews report that approximately 80%–90% of these PAPVR cases coexist with a sinus venosus ASD, whereas associations with patent foramen ovale, septum primum ASD, or ventricular septal defect are uncommon or incidental. 72. Transposition of great arteries can be defined as A. AV concordance B. VA concordance C. VA discordance D. AV discordance E. None of the above Explanation: (by Perplexity AI, generated with the OpenAI O3 model) Definition of Transposition of the Great Arteries (TGA) • Transposition of the great arteries is defined by ventriculo-arterial (VA) discordance—the aorta arises from the morphologic right ventricle and the pulmonary artery arises from the morphologic left ventricle, creating parallel rather than serial circulations. • (The atrioventricular [AV] connection is usually normal, so the hallmark lesion is discordance between ventricles and great arteries.)
  • 180.
    EBDR Exam MCQs& Concepts May 2023 Dr. Kareem Alnakeeb 174 Transposition of the Great Arteries: Segmental Anatomy Cheat-Sheet Variant Atrioventricular (AV) Connection Ventriculo-Arterial (VA) Connection Circulatory Physiology D-TGA (“complete” or dextro-TGA) Concordance (RA→RV, LA→LV) Discordance (RV→Aorta, LV→PA) Parallel circuits → cyanosis at birth; mixing needs PFO/ASD, VSD, or PDA L-TGA (“congenitally corrected” or levo- TGA) Discordance (RA→LV, LA→RV) Discordance (LV→PA, RV→Aorta) “Physiologically corrected” series flow; systemic work done by RV Key points to remember • Transposition is defined by VA discordance—the aorta springs from the morphologic right ventricle and the pulmonary artery from the morphologic left ventricle. • D-TGA = VA discordance + AV concordance. A patent ductus arteriosus (PDA) or other shunt is often life-saving until surgery. • L-TGA = VA discordance + AV discordance (“double discordance”). Because the blood crosses two wrong connections, overall flow is in the correct direction, hence “congenitally corrected.” 73. Non-Transmural myocardial infarction involves A. <25% transmurality B. 50% transmurality C. <75% transmurality D. 90% transmurality E. None of the above Explanation: (by Perplexity AI, generated with the OpenAI O3 model) Non-transmural (sub-endocardial) myocardial infarction • By definition, a non-transmural MI is confined to the inner layers of the ventricular wall and does not extend through the full thickness of the myocardium. • In standard pathology and imaging texts, the cut-off generally used is less than 50% of the wall thickness. • Infarcts that involve ≥50% of the wall are classified as transmural. 74. Left ventricle non-compaction is best defined as: A. Spongy left ventricle B. Endomyocardial with prominent trabeculation C. Pseudo-thickening of the lateral wall D. Unclassified congenital cardiomyopathy E. All of the above Explanation: (by Perplexity AI, generated with the OpenAI O3 model) Left-ventricular non-compaction (LVNC) is typically described with several overlapping phrases: • Spongy left ventricle – reflects the embryonic, sponge-like myocardial meshwork that fails to compact. • Endomyocardium with prominent trabeculation and deep recesses – the hallmark imaging and pathologic criterion.
  • 181.
    EBDR Exam MCQs& Concepts May 2023 Dr. Kareem Alnakeeb 175 • Pseudo-thickening of the lateral wall – on echocardiography or MRI the heavily trabeculated inner layer can mimic wall hypertrophy, producing apparent thickening. • Unclassified (or non-classified) congenital cardiomyopathy – listed as such in the European Society of Cardiology scheme; the American Heart Association places LVNC among genetic cardiomyopathies. Because each option represents a commonly used component of the standard definition, the most inclusive and therefore correct choice is “All of the above.” 75. Black blood Imaging is A. Used for assessment of cardiac anatomy B. Used in cardiac CINE imaging C. Used as a cardiac fat suppression technique D. None of the above E. All of the above Explanation: (by Perplexity AI, generated with the OpenAI O3 model) Black-blood cardiac MRI: typical uses Black-blood sequences (most commonly double-inversion recovery fast spin-echo) null the signal from flowing blood so that the blood pool appears dark while the myocardium remains bright. They are employed for three principal purposes: 1. Anatomic delineation – because the dark lumen sharply outlines the ventricular walls, valves, great vessels, and pericardium, black-blood imaging is the work-horse sequence for static assessment of cardiac anatomy and morphology. 2. Tissue characterization – varying the echo time produces T1- or T2-weighted black-blood images that highlight edema, fibrosis, or infiltration; fat-suppressed variants such as STIR combine blood nulling with fat suppression for detecting myocardial edema or masses. 3. Not for cine function – dynamic (cine) evaluation of ventricular motion is instead performed with bright-blood/SSFP sequences; black-blood techniques are inherently single-shot or segmented still images and are not used for routine cine acquisition. They are not a dedicated “cardiac fat-suppression technique” (although fat suppression can be added) and are unnecessary for standard cine imaging. 76. White blood imaging is A. Used for assessment of cardiac anatomy B. Used in cardiac CINE imaging C. Used as a cardiac fat suppression technique D. None of the above E. All of the above Explanation: (by Perplexity AI, generated with the OpenAI O3 model) White-blood (bright-blood) cardiac MRI “White-blood” sequences—principally gradient-echo and steady-state free-precession (SSFP) cine imaging— render fast-flowing blood bright, allowing clear visualization of intracardiac cavities throughout the cardiac cycle. Key applications • Cine functional imaging: SSFP is the gold-standard for quantifying ventricular volumes, ejection fraction, wall motion, and valve dynamics because its high temporal resolution captures motion in real time.
  • 182.
    EBDR Exam MCQs& Concepts May 2023 Dr. Kareem Alnakeeb 176 • Anatomic survey: Although cines do depict chamber and valvular anatomy, static high-resolution anatomic detail is usually obtained with black-blood spin-echo; “white-blood” itself is not the primary static morphologic sequence. • Not a fat-suppression technique: Fat suppression can be combined with various sequences, but “white-blood” refers to bright-blood contrast, not to selective nulling of fat signal. Answer B. Used in cardiac CINE imaging White-blood imaging’s defining role is dynamic (cine) assessment of cardiac function; it is not chiefly a fat- suppression method, and static anatomic assessment is better served by black-blood imaging. 77. Cardiac sarcoidosis involves A. Patchy enhancement of LV myocardium B. Preferential involvement to the base of the heart C. Bad prognostic value D. All of the above E. None of the above Explanation: (by Perplexity AI, generated with the OpenAI O3 model) Cardiac sarcoidosis on imaging and prognosis Cardiac sarcoidosis typically shows three characteristic features: • Patchy late-gadolinium enhancement (LGE) of the left-ventricular myocardium, often mid-wall or sub-epicardial rather than sub-endocardial. • Predominant involvement of the basal segments—especially the basal interventricular septum and the ventricular free wall—on MRI and autopsy studies. • Adverse prognosis compared with extracardiac sarcoidosis, with higher risks of arrhythmia, heart failure and sudden cardiac death; cardiac involvement is therefore regarded as a negative prognostic marker. 78. Regarding cardiac amyloidosis A. Patchy enhancement of LV myocardium is typical B. Global epicardial enhancement is characteristic C. Subendocardial enhancement is the rule D. All of the above E. None of the above Explanation: (by Perplexity AI, generated with the OpenAI O3 model) Cardiac amyloidosis: late-gadolinium enhancement (LGE) patterns Cardiac amyloid deposits expand the interstitium, producing circumferential subendocardial or diffuse transmural enhancement on cardiac MRI: • Multiple series identify a global or circumferential subendocardial LGE pattern as the characteristic and most common appearance, especially in light-chain (AL) amyloidosis. • More extensive transmural (epicardium-to-endocardium) LGE can develop in advanced disease— seen more often in transthyretin (ATTR) amyloidosis—but pure epicardial-predominant enhancement is not typical. • Patchy mid-myocardial foci are uncommon and, when present, suggest alternative diagnoses such as sarcoidosis rather than amyloid.
  • 183.
    EBDR Exam MCQs& Concepts May 2023 Dr. Kareem Alnakeeb 177 79. A 76-year-old male patient with chronic inflammatory disease and known history of secondary generalized multisystem amyloidosis showed an abnormal appearance of the heart on echocardiography. Dynamic enhanced cardiac MR imaging was advised for further characterization. All of the following are imaging findings seen with cardiac amyloidosis, except A. Left ventricular wall hypertrophy B. Subendocardial delayed myocardial hyperenhancement C. Systolic dysfunction D. Granular echogenic myocardium E. Interatrial septal thickening Source: Chin, Teck Yew, et al. Get Through Final FRCR 2A: SBAs. 1st ed., CRC Press, 2017. Explanation: Pathophysiology • In cardiac amyloidosis, the amyloid protein is deposited in the myocardium, which leads to diastolic dysfunction that progresses to restrictive cardiomyopathy. • Because amyloidosis is a systemic process, involvement of all four chambers is common; thus, an increase in the thickness of the interatrial septum and right atrial free wall by more than 6 mm has been shown to be a specific finding for cardiac amyloidosis. Imaging Findings • Through the use of dynamic enhanced cardiac MRI, a distinct pattern of late enhancement, which was distributed over the entire subendocardial circumference, has been shown to have high specificity and sensitivity for cardiac amyloidosis. • Echocardiogram shows concentric LV hypertrophy, with hyperechoic granular sparkling of the ventricular wall. 80. Macleod's syndrome: A. Affects the small bronchi B. Bronchiectasis is seen C. Submucosal fibrosis from 4 generation bronchus D. Pulmonary artery branches are hypoplastic E. All of the above Explanation: (by Perplexity AI, generated with the OpenAI O3 model) MacLeod’s syndrome (Swyer–James–MacLeod) This acquired post-infectious lung disorder shows a triad of pathologic changes that explain its imaging features: 1. Bronchiolar injury from bronchiolitis obliterans → fibrosis, narrowing and even obliteration of the small (membranous and respiratory) bronchioles, beginning about the fourth-generation airway and beyond. 2. Secondary bronchiectasis—cylindrical or saccular dilatation of affected bronchi is common, though not universal. 3. Marked reduction in calibre and number of ipsilateral pulmonary-artery branches, sometimes frank hypoplasia or agenesis, leading to the classic unilateral hyperlucent, hypoperfused lung on radiographs and CT.
  • 184.
    EBDR Exam MCQs& Concepts May 2023 Dr. Kareem Alnakeeb 178 81. In patients with rheumatoid arthritis, what is the commonest pulmonary finding seen on the chest radiograph? A. pleural effusion B. fibrosis C. pulmonary nodules D. bronchiectasis E. heart failure Source: Bydder, Megan, et al. Get Through Final FRCR Part A: SBAs for the Modular Examination. 1st ed., Hodder Education Publishers, 2009. Explanation: Pulmonary Manifestations of Rheumatoid Arthritis • Between 2% and 54% of patients with rheumatoid arthritis have pulmonary abnormalities. • Pleural abnormalities are most frequent, being either an effusion (unilateral in 92% of cases) or pleural thickening (usually bilateral). • Fibrosis occurs in 30% of patients with pulmonary involvement. • Nodules are unusual and seen in advanced disease. • They are usually peripheral and may cavitate. • Bronchial abnormalities are seen in 30% of patients with rheumatoid lung, and include bronchiectasis and bronchiolitis obliterans. • Other findings include pulmonary arterial hypertension and heart failure secondary to carditis/pericarditis. 82. A 56 year old female is found to have a small, well-defined anterior mediastinal mass on a chest radiograph which demonstrates homogeneous soft-tissue density with some peripheral calcification on CT. On MRI it is isointense to skeletal muscle on T1-weighted images and slightly increased signal on T2- weighted images. It is most likely to be: A. Thymic cyst B. Thymoma C. Thymolipoma D. Thymic hyperplasia E. Thymic carcinoma Source: Currie, Stuart, et al. SBAs for the FRCR 2A. 1st ed., Cambridge University Press, 2010. Explanation: • This case describes the typical features of a thymoma. Thymic hyperplasia and thymic carcinoma are usually ill-defined abnormalities. The signal from the lesion is not typical for a thymic cyst or thymolipoma.
  • 185.
    EBDR Exam MCQs& Concepts May 2023 Dr. Kareem Alnakeeb 179 83. A 40-year-old schoolteacher presented with a non-productive cough, dyspnea and low-grade pyrexia. She has never smoked. The CXR demonstrated several bilateral areas of patchy consolidation which were confirmed on HRCT and shown to be in a mainly sub-pleural distribution. In addition, there are also patchy ground-glass change and small (<5 mm) centrilobular nodules). What is the most likely diagnosis? A. Bronchoalveolar cell carcinoma B. Histoplasmosis C. Sarcoidosis D. Cryptogenic organizing pneumonia E. Multifocal streptococcal pneumonia Source: Proctor, Robin. Final FRCR Part A Modules 1–3 Single Best Answer MCQs: The SRT Collection of 600 Questions with Explanatory Answers (MasterPass). 1st ed., CRC Press, 2009. Explanation: • These imaging findings are classical of COP. Effusions and adenopathy are also present in up to one third of patients. • Bronchoalveolar cell carcinoma is an important differential, but is most commonly solitary and centrilobular nodules are not a feature, and there is a strong smoking association. 84. A 32 year old male front seat passenger is involved in a road traffic accident and sustains blunt abdominal trauma. He is admitted via the emergency department and CT reveals a splenic laceration with subcapsular haematoma. Which one of the following associated injuries is most likely to be found? A. Diaphragmatic rupture B. Injury to the liver C. Injury to the left kidney D. Ipsilateral rib fractures E. Injury to the small bowel mesentery Source: Currie, Stuart, et al. SBAs for the FRCR 2A. 1st ed., Cambridge University Press, 2010. Explanation: • All are potential associated injuries and should be actively searched for in the context of blunt abdominal trauma. • Rib fractures are found in up to 50% of patients with splenic injuries and as such are the most common association. • The left kidney is injured in 10% of patients with splenic injury, and diaphragm rupture is even rarer. • Diaphragm rupture may be difficult to appreciate on axial slices, and may be more evident on coronal reformats. 85. An 83-year-old man with a history of bladder cancer and myocardial infarction was referred for radiofrequency ablation (RFA) because of his co-morbidities. All of the following are true regarding RFA, except A. RFA uses high frequency alternating current to generate heat and high temperature to cause cell death. B. Cell death occurs by denaturation of proteins (coagulative necrosis). C. The tip of the electrode is placed in the centre of the lesion. D. The heat sink effect results in a poor outcome in larger lesions. E. Cell death starts at 49 degrees. Source: Chin, Teck Yew, et al. Get Through Final FRCR 2A: SBAs. 1st ed., CRC Press, 2017. Explanation: Radiofrequency Ablation Fundamentals • In RFA, a high-frequency, alternating current with a wavelength of 460–500 kHz is emitted through an electrode placed within the targeted tissue.
  • 186.
    EBDR Exam MCQs& Concepts May 2023 Dr. Kareem Alnakeeb 180 • Grounding pads applied to the patient’s thighs complete the electrical circuit. • Cell death starts at 49 degrees. • Temperature above 60 degrees causes immediate cell death, and tissue charring occurs at 105 degrees. • Cell death is induced by the denaturation of proteins. Percutaneous Imaging-Guided Technique • For percutaneous imaging-guided RFA, the energy is delivered into the target tissue by means of needle-like electrodes. • Unlike in a typical biopsy, the electrode tip should be advanced to the deep margin of the tumour. Imaging Follow-Up and Residual Disease • On follow-up CT, ablated tumours often have internal areas of increased attenuation or increased signal intensity at CT and MRI. • Areas of contrast enhancement (>10 HU or >15% with CT and MRI, respectively) are indicative of residual viable RCC. • Residual viable tumour can be treated with additional ablation sessions. Heat-Sink Phenomenon • Heat-sink phenomena refers to the reduction in tissue temperature due to the conductive effects of adjacent vessels or airways. • It is an explanation for distortion of the ablation zone and poor outcome in larger lesions. • The heat-sink effect can be overcome by pharmacologically reducing blood flow, intra-arterial embolisation, intravascular balloon occlusion, Pringle manoeuvre or reducing treatment zone. 86. A 45 year old woman with severe portal hypertension and variceal bleeding is referred for a trans-jugular intrahepatic porto-systemic shunt (TIPSS) procedure following the failure of endoscopic procedures in controlling the bleeding. Which of the following is the most appropriate regarding TIPSS? A. The middle hepatic vein is the preferred route of access to the portal vein B. The right portal vein is usually posterior to the right hepatic vein C. Flow of contrast towards the porta hepatis usually indicates puncture of the biliary tree D. The gradient across the shunt should be less than 20mmHg E. Stenosis tends to occur in the portal vein Source: Currie, Stuart, et al. SBAs for the FRCR 2A. 1st ed., Cambridge University Press, 2010. Explanation: • Usually the right hepatic vein (RHV) is the preferred route of access to the right portal vein, which lies anterior to the RHV. • Flow of contrast towards the porta, and especially if it remains there, usually indicates biliary puncture. • Puncture of portal vein and hepatic artery usually result in contrast flowing to the periphery. • The shunt gradient should be less than 12mm of mercury. • Stenoses usually tend to occur in the hepatic vein or the shunt itself.
  • 187.
    EBDR Exam MCQs& Concepts May 2023 Dr. Kareem Alnakeeb 181 87. A 36 year old female with history of pelvic pain and severe dysmenorrhea undergoes a pelvic ultrasound examination which reveals uterine fibroid disease. Which of the following imaging features would be associated with the best outcome following uterine artery embolization? A. Submucosal location B. Subserosal location C. Associated adenomyosis D. Calcification E. Multiple fibroids Source: Currie, Stuart, et al. SBAs for the FRCR 2A. 1st ed., Cambridge University Press, 2010. Explanation: • Subserosal fibroids, especially pedunculated ones, may often draw their blood supply from adjacent viscera, which may be a cause of failure of the procedure. They are also associated with a higher incidence of complications. • Calcific fibroids are less vascular and may not respond well to embolisation. • Bulky and multiple fibroids may need multiple interventions or surgery. • Adenomyosis is a known cause for failure of the procedure. 88. A 60 year old man who recently suffered a hemorrhagic stroke develops pulmonary emboli. He is referred for an IVC filter insertion and angiography is performed prior to this. The usual reasons for doing so would be all of the following except: A. To identify the renal veins B. To identify the hepatic veins C. To size the IVC D. To rule out the presence of a left IVC E. To evaluate for the presence of an IVC thrombus Source: Currie, Stuart, et al. SBAs for the FRCR 2A. 1st ed., Cambridge University Press, 2010. Explanation: • The hepatic veins do not need to be identified routinely prior to filter insertion. • Most filters are deployed in an infrarenal position, unless there is IVC thrombus which would preclude this, in which case the filter is positioned in the suprarenal position. • A left iliac injection is performed to rule out a left IVC, which could be a cause of filter failure. 89. While reporting plain films in a paediatric radiology setting, you look at the plain abdominal film of an 8- year-old child admitted with abdominal pain and vomiting. The film demonstrates multiple gas-filled bowel loops throughout the abdomen, with paucity of gas and a small calcific density in right lower quadrant. The right psoas outline is not clear. What is the likely diagnosis? A. Acute appendicitis B. Acute cholecystitis C. Renal colic D. Abdominal teratoma E. Chronic peritonitis Source: Chin, Teck Yew, et al. Get Through Final FRCR 2A: SBAs. 1st ed., CRC Press, 2017. Explanation: Acute Appendicitis • The plain film findings and clinical history are suggestive of acute appendicitis, which is the most common reason for abdominal surgery in children. • It is one of the most common causes of intestinal obstruction in children.
  • 188.
    EBDR Exam MCQs& Concepts May 2023 Dr. Kareem Alnakeeb 182 • Other causes include adhesions, intussusception, incarcerated inguinal hernia, malrotation with volvulus and Meckel’s diverticulum. • On plain films, a fecolith or appendicolith with small bowel obstruction (seen in 10% of cases) and displacement of bowel gas from the right iliac fossa are all typical signs of acute appendicitis. • However, an abdominal US examination is the preferred imaging modality of choice for investigation of acute appendicitis in children rather than abdominal radiograph. Imaging Modalities • The primary imaging modality in suspected acute appendicitis remains controversial. • Several authors advocate plain films, US and CT as primary diagnostic sets, with various arguments. Other Pediatric Abdominal Conditions • Acute cholecystitis and renal colic are not common diagnoses in children. • Intussusception most often occurs between 3 months and 1 year of age. 90. A 66-year-old joiner presents to his GP with jaundice and abdominal discomfort. He was subsequently referred to a gastroenterologist who requests a liver biopsy due to deranged liver function tests. Which of the following options is not a contraindication for percutaneous liver biopsy? A. INR above 1.6 B. Platelets less than 60,000/mm3 C. Tense ascites D. Extra-hepatic biliary obstruction E. Suspected hemangioma Source: Chin, Teck Yew, et al. Get Through Final FRCR 2A: SBAs. 1st ed., CRC Press, 2017. Explanation: Contraindications for liver biopsy include the following: A. Uncooperative patient B. Extrahepatic biliary duct dilatation (except if benefit outweighs the risk) C. Bacterial cholangitis (relative contraindication due to risk of septic shock) D. Abnormal coagulation indices (having a normal INR or PT is not a reassurance that the patient will not bleed; however, there is increased incidence of bleeding with INR above 1.5) E. Thrombocytopenia (platelet count below 60,000/mm3) 91. A 27-year-old female patient undergoes urgent neuroimaging following loss of consciousness as a result of an RTA. CT is unremarkable. MRI reveals multiple small areas of increased signal on T2WI in the white matter near the grey–white matter junction within the frontal and temporal lobes. In the same locations, DWI reveals areas of increased signal on the B1000 image and reduced signal on the ADC map. What is the most likely diagnosis? A. Subarachnoid haemorrhage. B. Extradural haematoma. C. Subdural haematoma. D. Hypoxic brain injury. E. Diffuse axonal injury. Source: Lindsay, Richard, et al. SBAs for the FRCR Part 2A. 1st ed., Oxford University Press, 2012. Explanation: Computed Tomography (CT) • CT is initially often normal (up to 80% of cases) in DAI.
  • 189.
    EBDR Exam MCQs& Concepts May 2023 Dr. Kareem Alnakeeb 183 • If positive, it may reveal small low attenuation foci (oedema) or high attenuation foci of petechial haemorrhage. • The gray/white matter interface of the frontotemporal lobes, corpus callosum (especially the splenium), and brainstem are the most commonly involved sites in DAI. Magnetic Resonance Imaging (MRI) • MRI is much more sensitive and is the investigation of choice. • The signal on MRI depends on the age of the lesion and whether haemorrhage is present, but classically hyperintense foci on T2WI sequences are seen acutely. • In the more chronic phase, the lesions may only be detected as hypointense foci at characteristic locations on GE sequences: this appearance may remain for years. • DWI reveals hyperintense foci of restricted diffusion on B1000 images, with corresponding low signal on the ADC map. Differential Findings • The findings on DWI are easily distinguishable from extradural haematoma/subarachnoid haemorrhage/subdural haematoma/generalized oedema, which are discussed in other questions in this chapter. 92. A 69-year-old lady was admitted 10 days ago following an acute intracerebral haematoma diagnosed on CT. What are the most likely radiological findings on the follow-up MRI scan of brain? A. Haematoma hypointense to grey matter on T1WI, hyperintense on T2WI. B. Haematoma hyperintense to grey matter on both T1WI and T2WI. C. Haematoma hyperintense to grey matter on T1WI, hypointense on T2WI. D. Haematoma hypointense to grey matter on both T1WI and T2WI. E. Haematoma isointense to grey matter on both T1WI and T2WI. Source: Lindsay, Richard, et al. SBAs for the FRCR Part 2A. 1st ed., Oxford University Press, 2012. Explanation: • The MRI appearances of intracranial haemorrhage are determined primarily by the state of the haemoglobin (Hb), which evolves with age. This can be staged as hyperacute (first few hours), acute (1–3 days), early subacute (3–7 days), late subacute (4–7 days to 1 month), or chronic (1 month to years). Table 6.1 The sequential signal intensity changes of the evolving intracerebral haematoma Phase State of Hb Magnetic properties T1 signal intensity T2 signal intensity Hyperacute Intracellular oxy-Hb Diamagnetic ↔ / ↓ ↑ Acute Intracellular deoxy-Hb Paramagnetic ↔ / ↓ ↓ Early subacute Intracellular met-Hb Paramagnetic ↑↑ ↓↓ Late subacute Extracellular met-Hb Paramagnetic ↑↑ ↑↑ Chronic Haemosiderin Superparamagnetic ↔ / ↓ ↓↓
  • 190.
    EBDR Exam MCQs& Concepts May 2023 Dr. Kareem Alnakeeb 184 93. A 52-year-old male presents with dyspnea and cough. A chest radiograph shows an ill-defined opacity in the right mid-zone, obscuring the heart border. A lateral view shows a thin wedge-shaped opacity with base in contact with the pleura anteroinferiorly, and pointing posterosuperiorly. What is the most likely diagnosis? A. right middle lobe collapse B. right middle lobe consolidation C. right lower lobe collapse D. right lower lobe consolidation E. encysted pleural fluid Source: Bydder, Megan, et al. Get Through Final FRCR Part A: SBAs for the Modular Examination. 1st ed., Hodder Education Publishers, 2009. Explanation: Radiographic Findings in Right Middle and Lower Lobe Pathology • In right middle lobe collapse, the horizontal fissure and lower half of the oblique fissure converge. • This creates a wedge-shaped opacity on the lateral chest radiograph. • On the frontal chest radiograph, there is an ill-defined mid-zone opacity. • With right middle lobe consolidation, there is a mid-zone opacity with a well-defined superior margin, as the horizontal fissure remains in a normal position and is tangential to the radiograph beam. • Both obscure the right heart border. • Lower lobe collapse and consolidation cause basal opacity with loss of clarity of the right hemidiaphragm. • The lateral view shows a triangular opacity at the right base posteriorly, larger in consolidation than collapse. 94. A 20-year-old female with a history of neurofibromatosis presents with reduced visual acuity in the right eye. She subsequently has CT and MR imaging of the orbits to assess for a tumour relating to the right optic nerve. Which of the following findings on imaging would be more suggestive of the presence of an optic nerve glioma, rather than a meningioma arising from the optic nerve sheath? A. Presence of the ‘tram-track’ sign. B. Presence of optic canal widening. C. Presence of marked intense tumour enhancement. D. Presence of calcification. E. Presence of bony hyperostosis. Source: Lindsay, Richard, et al. SBAs for the FRCR Part 2A. 1st ed., Oxford University Press, 2012. Explanation: Optic Nerve Canal & Lesion Prevalence • The presence of a widened optic nerve canal occurs in up to 90% of cases of optic nerve glioma. • While it can also occur in meningioma, it is more common in glioma and some cases of meningioma may even have a narrowed canal secondary to bony hyperostosis. Imaging Characteristics • The ‘tram-track’ sign is typically associated with meningioma and refers to the more avidly enhancing meningioma surrounding the non-enhancing optic nerve on axial CT and MR imaging of the orbit. • Although both meningioma and glioma enhance following intravenous contrast, it is meningioma that is more typically associated with marked intense enhancement. Calcification & Bony Changes • Calcification and bony hyperostosis are features associated with meningioma. • Calcification is rare in gliomas, unless they have previously undergone radiotherapy.
  • 191.
    EBDR Exam MCQs& Concepts May 2023 Dr. Kareem Alnakeeb 185 95. A 45 year old woman undergoes investigation for conductive hearing loss. History reveals several previous ear infections. Direct visualisation with an otoscope shows a mass behind an intact tympanic membrane. Coronal CT imaging demonstrates a soft-tissue mass located between the lateral attic wall and the head of the malleus. There is blunting of the scutum. The mass does not enhance post-contrast. What is the most likely diagnosis? A. Chronic otitis media B. Cholesterol granuloma C. Cholesteatoma D. Rhabdomyosarcoma E. Squamous cell carcinoma Source: Currie, Stuart, et al. SBAs for the FRCR 2A. 1st ed., Cambridge University Press, 2010. Explanation: Definition • A cholesteatoma consists of a sac lined with stratified squamous epithelium and filled with keratin – essentially ‘skin growing in the wrong place’. Types and Origin • They can be acquired (98%) or congenital (2%). • Most acquired cholesteatomas arise in the superior portion of the tympanic membrane (pars flaccida) and extend into Prussak’s space where they can cause medial displacement of the head of the malleus and erosion of the bony scutum. Imaging Characteristics • The characteristic imaging feature of a cholesteatoma is bone erosion associated with a non- enhancing soft-tissue mass. Complications • Intratemporal: ossicular destruction, facial nerve paralysis, labyrinthine fistula, complete hearing loss, automastoidectomy. • Intracranial: meningitis, sinus thrombosis, abscess, CSF rhinorrhea. 96. You are reporting a CT scan of neck in a patient with a head and neck cancer. You see an enlarged necrotic jugulo-digastric lymph node on the right side and wish to describe the appropriate level of this lymph node in your report. What is the correct level? A. I. B. II. C. III. D. IV. E. V. F. VI. G. VII. Source: Lindsay, Richard, et al. SBAs for the FRCR Part 2A. 1st ed., Oxford University Press, 2012. Explanation: Overview of Neck Lymph-Node Levels • Lymph nodes in the neck have been divided into seven levels, generally for the purpose of squamous cell carcinoma staging. • This is, however, not all inclusive, as the parotid nodes and retropharyngeal space nodes are not included in this system. • Lymph node levels of the neck. Radiopaedia.org.
  • 192.
    EBDR Exam MCQs& Concepts May 2023 Dr. Kareem Alnakeeb 186 Level I: Below mylohyoid to hyoid bone anteriorly. • Level Ia: Submental. • Level Ib: Submandibular. Level II: Jugulodigastric (base of skull to hyoid). Level III: Deep cervical (hyoid to cricoid). Level IV: Virchow (cricoid to clavicle). Level V: Posterior triangle groups. • Level Va: Accessory spinal (posterior triangle), superior half. • Level Vb: Accessory spinal (posterior triangle), inferior half. Level VI: Prelaryngeal/pretracheal/Delphian node. Level VII: Superior mediastinal (between common carotid arteries (CCAs), below top of manubrium). 97. A 12-year-old boy is investigated via MRI brain for headache, nystagmus, and ataxia. Which of the following radiological fi ndings would suggest a diagnosis of Chiari I malformation as opposed to Chiari II? A. Lacunar skull. B. Myelomeningocoele. C. Elongation of the fourth ventricle. D. Caudal displacement of the cerebellar tonsils. E. Cervicomedullary kinking. Source: Lindsay, Richard, et al. SBAs for the FRCR Part 2A. 1st ed., Oxford University Press, 2012. Explanation: • Chiari II is seen in all patients with open spinal dysraphisms, such myelomeningocoele. • Lacunar skull (luckenshadel) is also associated with Chiari II. • Cervicomedullary kinking is common to both, although more so with Chiari II. • Caudal displacement of the cerebellar tonsils is a feature of Chiari I, whereas in Chiari II the vermis herniates into the foramen magnum and the tonsils are lateral to the medulla. 98. A 36 year old woman with known polycystic kidney disease presents with a history of sudden onset headache and has signs of meningism. A CT brain reveals subarachnoid haemorrhage with haematoma within the septum pallucidum. What is the most likely site for an intracerebral aneurysm? A. Anterior communicating artery B. Posterior communicating artery C. A2 segment of an anterior cerebral artery D. Tip of the basilar artery E. Middle cerebral artery Source: Currie, Stuart, et al. SBAs for the FRCR 2A. 1st ed., Cambridge University Press, 2010. Explanation: • A clot in the septum pallucidum is virtually diagnostic of an aneurysm of the anterior communicating artery. • Aneurysms of the distal anterior cerebral artery are less common.
  • 193.
    EBDR Exam MCQs& Concepts May 2023 Dr. Kareem Alnakeeb 187 99. A 6-year-old girl is brought to your local paediatric outpatients with a history of night sweats, tiredness, and new onset wheeze not responding to bronchodilators. A CXR is done which shows increased mediastinal soft tissue noted superiorly. The paravertebral lines are maintained. The aortic knuckle is not visible. A lateral CXR has been carried out at the request of the paediatrician, which shows increased soft tissue displacing the trachea posteriorly, causing mild narrowing. What is the most likely diagnosis? A. Tuberculosis. B. Lymphangioma. C. Bronchogenic cyst. D. Thymic/nodal malignant infiltration. E. Teratoma. Source: Lindsay, Richard, et al. SBAs for the FRCR Part 2A. 1st ed., Oxford University Press, 2012. Explanation: Lesion Localization • The first step in this question is to localize the lesion. • The posterior displacement of the trachea indicates an anterior mediastinal position. Differential Diagnosis of Anterior Mediastinal Masses in Children • It is then necessary to consider the common causes of anterior mediastinal masses in children, which are: normal thymus and thymic/nodal infiltration (leukaemia, lymphoma), with other causes (lymphangioma and teratoma) being much less common. • A normal thymus would not be expected to cause significant posterior displacement of the trachea. • On plain film it would not be possible to differentiate thymic infiltration from anterior mediastinal nodal infiltration. • TB in children would be uncommon in the anterior mediastinum, especially when no abnormality is noted in the hila. 100. A 24-year-old woman who is 28 weeks pregnant is admitted with suspected pulmonary embolism. As the on-call radiologist, her obstetrician contacts you seeking advice regarding further management. An admission CXR is normal. What investigation do you advise initially? A. Venous ultrasound. B. Low-dose CTPA. C. Reduced dose lung scintigraphy. D. MRA. E. Catheter pulmonary angiography. Source: Lindsay, Richard, et al. SBAs for the FRCR Part 2A. 1st ed., Oxford University Press, 2012. Explanation: Venous Ultrasound • For pregnant patients, venous ultrasound is recommended before imaging tests with ionizing radiation are performed. • Up to 29% of pregnant patients with PE will have a positive venous ultrasound, obviating the need for further imaging. V/Q Scanning vs CTPA • The majority of the PIOPED II investigators currently recommend V/Q scanning over CTPA in the evaluation of PE in pregnant patients. • The foetal dose with V/Q is similar to that with CTPA, although the effective dose per breast is much greater with CTPA. MRI and Contrast Agents
  • 194.
    EBDR Exam MCQs& Concepts May 2023 Dr. Kareem Alnakeeb 188 • MRI requires further evaluation and gadolinium-based contrast agents have not been proven to be safe in pregnancy. Catheter Angiography • The role of catheter angiography is probably limited to those patients requiring mechanical thrombectomy. Cumulative Radiation Exposure • It should be noted that even a combination of CXR, lung scintigraphy, CTPA, and pulmonary angiography exposes the foetus to approximately 1.5 mGy of radiation, which is well below the accepted limit of 50 mGy for the induction of deterministic effects in the foetus. 101. A 62 year old man presents with right shoulder pain which radiates down his arm. A plain radiograph confirms the presence of a right apical mass with destruction of the surrounding ribs. CT-guided biopsy is performed and is likely to reveal: A. Large cell lung cancer B. Squamous cell cancer C. Small cell lung cancer D. Adenocarcinoma E. Carcinoid Source: Currie, Stuart, et al. SBAs for the FRCR 2A. 1st ed., Cambridge University Press, 2010. Explanation: • The case describes a Pancoast tumor for which squamous is the most common cell type. 102. A 3-month-old child presents to the paediatric outpatient clinic with a history of recurrent respiratory distress. The child had an uneventful delivery, but has had recurrent problems since birth. The child had a CXR taken prior to discharge home, aged 2 days, which showed a density in the left upper lobe, felt by the paediatrician to represent the thymus. Whilst the infant has never required admission, the mother is concerned due to recurrent coughing and dyspnoea. A CXR obtained at the clinic shows a large hyperlucent area in the left upper lobe. What is the most likely diagnosis? A. Congenital lobar emphysema. B. Congenital cystic adenomatoid malformation. C. Pulmonary sequestration. D. Persistent PIE. E. Congenital diaphragmatic hernia. Source: Lindsay, Richard, et al. SBAs for the FRCR Part 2A. 1st ed., Oxford University Press, 2012. Explanation: Lobar Predilection and Presentation • This has a lobar predilection with around 40% being found in the left upper lobe. • Most present in the neonatal period with respiratory distress, but they can present later. Congenital Lobar Emphysema (CLE) • Congenital lobar emphysema initially presents as an area of soft tissue density due to retained foetal pulmonary fluid. • This resolves and is replaced by hyperlucency. Congenital Cystic Adenomatoid Malformation (CCAM) • Congenital cystic adenomatoid malformations (CCAM) do not show a lobar predilection, but can be found anywhere. • They can be either air or fluid filled and consist of multiple cysts.
  • 195.
    EBDR Exam MCQs& Concepts May 2023 Dr. Kareem Alnakeeb 189 • CCAM are graded on the size of the cysts with type 1 lesions containing one or more large cysts, type 2 have numerous small cysts, and type 3 contain microscopic cysts, but appear solid at imaging. Congenital Diaphragmatic Hernias • Congenital diaphragmatic hernias are also initially solid on plain radiography and only contain air if there is bowel present within the hernia and this contains air. • This would obviously have continuity with the diaphragm and not be contained entirely within the upper lobe. Persistent Pulmonary Interstitial Emphysema (PIE) • Persistent PIE occurs when PIE fails to resolve after 1 week. • As PIE is almost always seen in infants with surfactant deficiency being ventilated, it would not be in the differential in this case. Sequestrations • Sequestrations are usually solid masses on plain film radiography, unless there has been a history of infection within the sequestration. 103. A 16-year-old male with a history of epilepsy is investigated via MRI. Axial T2WI demonstrates a cystic space within the left frontal lobe isointense to CSF. This is causing local mass effect and there is adjacent enlargement of the left lateral ventricle. What is the most likely diagnosis? A. Porencephalic cyst. B. Arachnoid cyst. C. Schizencephaly. D. Hydranencephaly. E. Ependymal cyst. Source: Lindsay, Richard, et al. SBAs for the FRCR Part 2A. 1st ed., Oxford University Press, 2012. Explanation: Porencephaly • Porencephaly is a congenital/acquired cystic cavity within the brain parenchyma with adjacent enlargement of the lateral ventricle. • They develop in utero or early infancy. Arachnoid Cysts • Arachnoid cysts are also CSF isointense, but are extra-axial, displacing the brain away from the adjacent skull. Ependymal Cysts • Ependymal cysts are intraventricular and the surrounding brain is usually normal. Schizencephaly • Schizencephaly is characterized by an intraparenchymal cleft extending from the ventricular surface to the brain surface lined by gray matter. Hydranencephaly • Hydranencephaly results from an early destructive process of the developing brain. • The cranial vault is CSF filled with absence of the cortical mantle and ventricles (water-bag brain). • Death in infancy is typical.
  • 196.
    EBDR Exam MCQs& Concepts May 2023 Dr. Kareem Alnakeeb 190 104. Which of the following is not an angiographic sign of active bleeding? A. Contrast extravasation B. Vessel spasm C. Vessel cut-off D. Early venous filling E. Vessel dilatation Source: Chin, Teck Yew, et al. Get Through Final FRCR 2A: SBAs. 1st ed., CRC Press, 2017. Explanation: • The rest are angiographic signs of active bleeding; vessel dilatation is not. 105. An 80-year-old man presents with hemoptysis and a mass on chest radiograph. A biopsy shows non- small-cell lung cancer. CT of chest shows a 4 cm, right middle lobe mass with pleural tethering but no chest wall invasion. Lymph nodes are seen at the right hilum (17 mm short axis), in the subcarinal space (20 mm short axis) and in the aortopulmonary space (8 mm short axis). No other abnormalities are seen. What is the TNM stage? A. T2 N1 M0 B. T2 N2 M0 C. T2 N3 M0 D. T3 N1 M0 E. T3 N2 M0 Source: Bydder, Megan, et al. Get Through Final FRCR Part A: SBAs for the Modular Examination. 1st ed., Hodder Education Publishers, 2009. Explanation: • The T stage is T2, as the lesion is over 3cm but there is no chest wall or mediastinal invasion or other associated feature. • The nodes at the right hilum (N1) and in the subcarinal space (N2) are significantly enlarged, whereas the node in the aortopulmonary space (N3) is not (,10mm short axis), hence the N stage is N2. 106. A 35-year-old man involved in a major RTA undergoes a lateral view of the cervical spine in the resus on arrival. All of the following are features associated with atlanto-occipital dislocation, except A. Soft-tissue swelling anterior to C2 by >10 mm. B. Basion dens interval >12 mm. C. Odd’s ratio >1. D. X-ray can often be normal. E. Incongruity of articular surface of atlas and occipital condyles. Source: Chin, Teck Yew, et al. Get Through Final FRCR 2A: SBAs. 1st ed., CRC Press, 2017. Explanation: Lateral Radiograph Findings • Atlanto-occipital dislocation shows the following on lateral radiograph of the cervical spine: • 10 mm soft-tissue swelling anterior to C2, with pathological convexity (80%), • basion-dens interval of >12 mm, • odds ratio (distance between the basion and the posterior arch of the atlas divided by opisthion and anterior arch of atlas) >1, and • basion–posterior axial line interval >12 mm anterior/>4 mm posterior to axial line. Direct Radiographic Signs • Direct signs include loss of congruity of articular surfaces of atlas and occipital condyle. Diagnostic Caveat • Normal X-ray in the presence of atlanto-occipital dislocation is rare.
  • 197.
    EBDR Exam MCQs& Concepts May 2023 Dr. Kareem Alnakeeb 191 107. You have been asked to review a chest radiograph by a junior doctor. The image demonstrates subtle hazy opacification of the upper part of the lower zone of the right lung. The right atrial border is indistinct and the horizontal fissure runs from the right hilum to the eighth rib in the mid axillary line. What is the most plausible explanation for these findings? A. Middle lobe collapse B. Middle lobe consolidation C. Pectus excavatum D. Right lower lobe mediobasal segment consolidation E. Right lower lobe anteriobasal segment consolidation Source: Proctor, Robin. Final FRCR Part A Modules 1–3 Single Best Answer MCQs: The SRT Collection of 600 Questions with Explanatory Answers (MasterPass). 1st ed., CRC Press, 2009. Explanation: • The findings are those of middle lobe collapse. • Signs on the frontal radiograph can be subtle, and it is more easily seen on the lateral radiograph. • In this case the loss of clarity of the right atrial border indicates the pathology is located in the middle lobe. • There is loss of volume (the normal horizontal fissure runs from the hilum to the sixth rib in the mid axillary line), therefore collapse of the middle lobe, rather than consolidation, is the likely cause for these appearances. 108. In acute respiratory distress syndrome what is the first change usually seen on the chest radiograph? A. confluent consolidation B. pleural effusions C. increased heart size with globular shape D. volume loss with atelectasis E. patchy ill-defined opacities Source: Bydder, Megan, et al. Get Through Final FRCR Part A: SBAs for the Modular Examination. 1st ed., Hodder Education Publishers, 2009. Explanation: • Acute respiratory distress syndrome (ARDS) commences with interstitial oedema, progressing to congestion and extensive alveolar, and interstitial oedema and hemorrhage. • The chest radiograph is often normal for the first 24 hours, before patchy opacities appear in both lungs. • These progress to massive airspace consolidation over the following 24–48 hours. • True volume loss, atelectasis, cardiomegaly and effusions are not seen in ARDS. 109. A 16-year-old male presents with sudden shortness of breath. A chest radiograph shows multiple, bilateral nodules measuring up to 3 cm, some of which are calcified. There is a moderate left pneumothorax. The patient has been undergoing treatment for a malignant tumor. What is the most likely diagnosis? A. metastases secondary to Wilms’ tumor B. metastases secondary to osteosarcoma C. metastases secondary to testicular tumor D. abscesses secondary to immunosuppression E. varicella pneumonia secondary to immunosuppression
  • 198.
    EBDR Exam MCQs& Concepts May 2023 Dr. Kareem Alnakeeb 192 Source: Bydder, Megan, et al. Get Through Final FRCR Part A: SBAs for the Modular Examination. 1st ed., Hodder Education Publishers, 2009. Explanation: • Osteosarcoma pulmonary metastases are uncommon (seen in 2% of cases) and present as multiple masses which may calcify. There is a high incidence of associated pneumothorax. • Wilms’ tumors may also produce multiple pulmonary masses and may be associated with pneumothorax, but are not known to calcify. • Testicular tumors may produce calcified lung metastases, but are not associated with pneumothorax. • Varicella pneumonia shows patchy consolidation in the acute phase, with multiple, small, calcified nodules in the chronic phase. • Abscesses may present as multiple masses but rarely calcify and often cavitate. 110. A 35 year old with asthma presents with malaise, flu-like illness and cough. Previous similar episodes have occurred. A chest radiograph shows patchy airspace opacification in the mid and upper zones. Which feature on high-resolution CT would make allergic bronchopulmonary aspergillosis a more likely diagnosis than extrinsic allergic alveolitis? A. widespread centrilobular micronodules ,3 mm B. tubular finger-like opacities C. bronchiectasis D. upper-zone fibrosis E. pleural effusion Source: Bydder, Megan, et al. Get Through Final FRCR Part A: SBAs for the Modular Examination. 1st ed., Hodder Education Publishers, 2009. Explanation: • Allergic bronchopulmonary aspergillosis (ABPA) is hypersensitivity to aspergillus in people with asthma. Typical features are of a migratory pneumonitis, predominantly in the upper lobes. • It may cause bronchiectasis and upper-zone fibrosis, which are features also seen in extrinsic allergic alveolitis (EAA). • Tubular opacities, indicating mucus plugging, are seen in ABPA, but not in EAA. • Centrilobular nodules are seen in EAA, along with mosaic perfusion and patchy ground-glass change. • Pleural effusions are rarely seen in EAA and not in ABPA. 111. A 56-year-old male presents with wheezing, cough and recurrent chest infections. A chest radiograph shows right middle lobe consolidation. CT of the chest shows a 3 cm mass arising within the right middle lobe bronchus with distal collapse and consolidation. Which feature of the mass would make hamartoma more likely than carcinoid? A. central location B. presence of calcification C. cavitation D. presence of fat E. prominent enhancement
  • 199.
    EBDR Exam MCQs& Concepts May 2023 Dr. Kareem Alnakeeb 193 Source: Bydder, Megan, et al. Get Through Final FRCR Part A: SBAs for the Modular Examination. 1st ed., Hodder Education Publishers, 2009. Explanation: • Hamartomas are usually seen in the periphery of the lungs (two-thirds) with 10% being endobronchial. Calcification is seen in 15%, often popcorn type. Cavitation is rare but fat is seen in 50%. • Carcinoids are usually located centrally and are endobronchial. Calcification is seen in one-third and they rarely cavitate. They do not contain fat and show prominent enhancement following contrast, as they are vascular. 112. A 50-year–old male is admitted under the surgical team having presented with upper abdominal pain and raised inflammatory markers. Suspecting acute cholecystitis, an ultrasound is requested, but due to large body habitus there is poor visualization of his gallbladder. To further evaluate hepatobiliary scintigraphy using 99mTc-labelled iminodiacetic acid is arranged. Which of the following findings are consistent with acute cholecystitis? A. Non-visualization of the gallbladder at 1 and 4 hours. B. Non-visualization of the gallbladder at 1 hour but seen at 4 hours. C. Visualization of the gallbladder at 1 hour. D. Visualization of the gallbladder at 30 minutes after morphine administration. E. Hepatobiliary scintigraphy is not appropriate for investigation of acute cholecystitis. Source: Lindsay, Richard, et al. SBAs for the FRCR Part 2A. 1st ed., Oxford University Press, 2012. Explanation: Indications and Purpose • Hepatobiliary scintigraphy is most commonly used to evaluate suspected acute cholecystitis. Patient Preparation • A minimum of 2 hours fasting is required. Radiotracer Dynamics • Following prompt uptake by the liver, the radiotracer is excreted into the biliary system and drains into the small bowel. • Activity should be demonstrated within the gallbladder by 1 hour. • Morphine can be used during the scan to relax the sphincter of Oddi, thus pushing radiolabelled bile into the gallbladder. Diagnostic Criteria • Acute cholecystitis is characterized by non-visualization of the gallbladder at both 1 and 4 hours or at 30 minutes following morphine administration. • Non-visualization of the gallbladder at 1 hour, but seen at 4 hours, is indicative of chronic cholecystitis. • A false-positive diagnosis of acute cholecystitis can occur with previous cholecystectomy, gallbladder agenesis, and tumor obstructing the cystic duct. 113. A radiologist is reporting a 99mTc bone scan and describes it as a ‘superscan’. He can say this because of reduced uptake in the: A. brain B. skeleton C. kidneys D. bowel E. myocardium.
  • 200.
    EBDR Exam MCQs& Concepts May 2023 Dr. Kareem Alnakeeb 194 Source: Lindsay, Richard, et al. SBAs for the FRCR Part 2A. 1st ed., Oxford University Press, 2012. Explanation: Definition and Causes • A superscan refers to a 99mTc-labelled technetium IBS where there is diffuse increased osseous uptake with apparent reduced renal and soft tissue uptake. • The appearance is commonly due to widespread osteoblastic bony metastases (e.g. prostate or breast carcinoma), but is also caused by non-malignant disease (e.g. renal osteodystrophy, hyperparathyroidism, osteomalacia, myelofibrosis, Paget’s disease). • In metastatic disease there is usually higher uptake in the axial than the appendicular skeleton. Normal IBS and FDG-PET Uptake Patterns • In IBS uptake is normally seen in bone, kidneys, and bladder, soft tissues (low levels), breasts (particularly in young women), and epiphyses (skeletally immature patients). • Uptake is seen in the myocardium (high), brain (high), and bowel (moderate) in FDG-PET scanning, not IBS; however myocardial uptake on IBS can be seen in cases of recent myocardial infarction and amyloidosis. Pitfalls and False Results • Note that poor renal function can often demonstrate reduced or absent renal visualization producing an appearance similar to a superscan (false positive), whereas urinary tract obstruction in prostatic carcinoma can increase renal activity and lead to false negative scans. 114. A 5-year-old boy presents with a history of walking difficulty. On examination he is noted to have an antalgic gait and lower limb length discrepancy, with the right limb being shorter than the left. Plain radiographs of the right leg show lobular ossific masses arising from the distal femoral epiphysis and the talus, which resemble osteochondromas. What is the most likely underlying diagnosis? A. Dysplasia epiphysealis hemimelica (Trevor disease). B. Multiple epiphyseal dysplasia. C. Diaphyseal aclasis. D. Dyschondrosteosis (Leri–Weil disease). E. Klippel–Trenaunay–Weber syndrome. Source: Lindsay, Richard, et al. SBAs for the FRCR Part 2A. 1st ed., Oxford University Press, 2012. Explanation: • This is an uncommon developmental disorder relating to the formation of an osteochondroma-type lesion at the epiphyses of usually a single lower extremity. • The epiphyses most commonly involved are those on either side of the knee or ankle. • Typically it is only the medial or lateral side of the epiphyses affected (medial:lateral 2:1). • The disease is usually recognized at a young age because of an antalgic gait, palpable mass, varus or valgus deformity, or limb length discrepancy. 115. A 5-year-old boy with bilateral wrist pain undergoes a plain fi lm which reveals several pedunculated bony outgrowths from the metaphysis of both radii, which point away from the adjacent joints. What is the most likely diagnosis? A. Ollier disease. B. Maffucci syndrome. C. Morquio syndrome. D. Diaphyseal aclasia. E. Hunter syndrome.
  • 201.
    EBDR Exam MCQs& Concepts May 2023 Dr. Kareem Alnakeeb 195 Source: Lindsay, Richard, et al. SBAs for the FRCR Part 2A. 1st ed., Oxford University Press, 2012. Explanation: Description and Etiology • The description is classic for multiple hereditary osteochondromas/exostoses, also known as diaphyseal aclasia. • Osteochondromas are the result of displaced growth plate cartilage, which causes lateral bone growth from the metaphysis. • They typically point away from the epiphysis. • There is continuity of the normal marrow, cortex, and periosteum between the exostosis and the host bone. • The cartilage cap, which is the source of growth, may have some chondroid matrix, but the appearance is otherwise of a deformed but normal bone. Distribution and Growth • They are normally found in the extremities, with 36% around the knee. • Their growth normally ceases at skeletal maturity. Clinical Manifestations • Symptoms are related to pressure effects on adjacent neural or vascular structures. Malignant Transformation • Less than 1% of solitary osteochondromas undergo malignant transformation to chondrosarcoma. • Findings that should alert to this are destruction of exostosis bone, destruction of matrix in the cartilage cap, irregular or thick (>2 cm in adults, >3 cm in children) cap, or growth of the cap after skeletal maturity. Multiple Hereditary Osteochondromatosis • Multiple hereditary osteochondromatosis is an uncommon autosomal dominant condition. • Patients present with multiple osteochondromas, which cause short stature. • The elbow and wrist joints are often deformed. • There is a higher risk of malignant transformation than in solitary osteochondromas, probably 2–5%. Differential Diagnoses • Ollier disease is the presence of multiple enchondromas and Mafucci syndrome requires, in addition, multiple soft-tissue haemangiomas. • Morquio and Hunter syndromes are mucopolysaccharidoses, with their own musculoskeletal abnormalities, which often make an appearance in exams. 116. An 80-year-old man undergoes CT brain for confusion. A well-defined intramedullary bony lesion containing speckled calcification is identified in the left frontal diploë. He has multiple small, rounded, benign skin tumours since youth. Which diagnosis best explains the imaging and clinical findings? A. Frontal osteochondroma B. Frontal osteoma C. Frontal meningioma D. Frontal chondrosarcoma E. Frontal osteoid osteoma Explanation: (by Perplexity AI, generated with the OpenAI O3 model) • Intradiploic frontal osteomas are dense, slow-growing, intramedullary bone lesions that show mature lamellar bone with variable calcification on CT. They are classically associated with multiple
  • 202.
    EBDR Exam MCQs& Concepts May 2023 Dr. Kareem Alnakeeb 196 cutaneous sebaceous cysts or epidermoid cysts in Gardner syndrome, matching this patient’s longstanding skin lesions. • Osteochondromas (A) arise from the metaphyseal surface of long bones, not the skull. • Meningiomas (C) may calcify but are extra-axial soft-tissue masses, typically showing hyperostosis rather than forming a discrete intramedullary bony nidus. • Chondrosarcomas (D) of the skull base show “rings-and-arcs” chondroid calcification and an aggressive lytic pattern, unlike the benign sclerotic appearance here. • Osteoid osteoma (E) produces a small cortical nidus causing intense night pain and is exceedingly rare in the calvarium. 117. A 40-year-old man of short stature is found to have multiple metaphyseal osteochondromas in the long bones. What is the approximate risk of malignant transformation of these lesions? A. Less than 5% B. Less than 10% C. Less than 20% D. Less than 30% E. Around 50% Explanation: (by Perplexity AI, generated with the OpenAI O3 model) • Osteochondromas are benign exostoses that arise from the metaphyses of long bones. • In solitary lesions, malignant change to secondary peripheral chondrosarcoma is rare (≈1%). • In patients with multiple hereditary exostoses—classically associated with short stature—the pooled literature shows a transformation risk of about 5%–10%. • The other options over- or underestimate this figure: <5% reflects solitary cases; ≥20% is far above the accepted range for multiple lesions. 118. An adult’s hand X-ray shows a well-defined exophytic ossified mass stuck onto the cortical surface of a proximal phalanx without corticomedullary continuity. Histology confirms a bizarre osteochondromatous parosteal proliferation (Nora lesion). Which statement about this entity is CORRECT? A. It arises within the cortex and medulla of long bones B. It is confined to the medullary cavity of short tubular bones C. It is characteristically seen in the hands and feet D. It always behaves as an overtly malignant tumour E. It never recurs after marginal excision Explanation: (by Perplexity AI, generated with the OpenAI O3 model) • Bizarre parosteal osteochondromatous proliferation (BPOP) is a benign surface‐based lesion that most frequently affects the small bones of the hands and feet, accounting for more than half of reported cases. It originates on the periosteal surface and typically lacks medullary continuity. • Long-bone involvement is unusual, so option A is incorrect. • Option B is wrong because the lesion is parosteal, not intramedullary. • Although overwhelmingly benign, rare malignant transformation after multiple recurrences is documented, so option D overstates its behaviour. • Recurrence rates approach 30%–50% after simple excision, making option E incorrect.
  • 203.
    EBDR Exam MCQs& Concepts September 2022 Dr. Kareem Alnakeeb 197 September 2022 1. Which of the following characteristics is seen in a sequestered disk? A. Peripheral enhancement B. Continuity with a disk extrusion C. Posterior location in the spinal canal D. Hemorrhage E. Foraminal location Source: Riascos-Castaneda, Roy F., et al., editors. RadCases Plus Q&A: Neuro Imaging. 2nd ed., Thieme, 2018. Explanation: • Sequestered disks show peripheral enhancement in the inflammatory phase. 2. Which of the following conditions presents with lesions that are cigar-shaped on sagittal imaging, wedge- shaped on axial imaging, are eccentrically located, and are less than or equal to two vertebral bodies in length? A. Multiple sclerosis B. Neuromyelitis optica (NMO) C. Transverse myelitis D. Spinal cord infarction E. Subacute combined degeneration Source: Riascos-Castaneda, Roy F., et al., editors. RadCases Plus Q&A: Neuro Imaging. 2nd ed., Thieme, 2018. Explanation: • MS lesions in the spinal cord typically have these features on MR imaging. 3. Regarding Chance fracture, which statement is true? A. There is no distraction of the posterior elements. B. There is fracture of the pedicles and variable involvement of the posterior elements. C. It is more frequently centered in the craniocervical junction. D. The anterior ligamentous complex is frequently involved. Source: Riascos-Castaneda, Roy F., et al., editors. RadCases Plus Q&A: Neuro Imaging. 2nd ed., Thieme, 2018. Explanation: • There is fracture of the pedicles and variable involvement of the posterior elements. • Chance fractures are more frequently centered at the thoracolumbar (not craniocervical) junction. • Distraction of the posterior elements and involvement of the posterior (not anterior) ligamentous complex are common. 4. Which is true regarding Duret hemorrhages? A. Occur secondary to upward transtentorial herniation B. Result in tonsillar herniation C. Are more common in the medulla D. Are generally centrally located Source: Riascos-Castaneda, Roy F., et al., editors. RadCases Plus Q&A: Neuro Imaging. 2nd ed., Thieme, 2018. Explanation: • Duret hemorrhages are typically centrally located. • They are caused by downward transtentorial herniation, they do not necessarily result in tonsillar herniation and are more common in the midbrain and pons.
  • 204.
    EBDR Exam MCQs& Concepts September 2022 Dr. Kareem Alnakeeb 198 5. Which of the following is a feature required for diagnosis of a Chiari I malformation? A. Small posterior fossa B. Cervical cord syrinx C. Myelomeningocele D. Cerebellar tonsils below the level of the foramen magnum Source: Riascos-Castaneda, Roy F., et al., editors. RadCases Plus Q&A: Neuro Imaging. 2nd ed., Thieme, 2018. Explanation: • Answers a and care characteristics of Chiari II malformations. • Although cervical cord syrinx is a common finding in Chiari I, it is not necessary to make the diagnosis. 6. Which of the following posterior fossa cystic abnormalities is associated with a small vermian size? A. Dandy-Walker malformation B. Blake pouch cyst C. Megacistema magna D. Posterior fossa arachnoid cyst Source: Riascos-Castaneda, Roy F., et al., editors. RadCases Plus Q&A: Neuro Imaging. 2nd ed., Thieme, 2018. Explanation: • Blake pouch cyst, megacisterna magna, and posterior fossa arachnoid cysts typically course with a normal cerebellar vermis. 7. Which imaging modality is considered the gold standard for the evaluation of vasculitis? A. CT angiography B. MR angiography C. Digital subtraction angiography (DSA) D. Vessel wall MRI Source: Riascos-Castaneda, Roy F., et al., editors. RadCases Plus Q&A: Neuro Imaging. 2nd ed., Thieme, 2018. Explanation: • Although occasionally primary angiitis of the central nervous system (PACNS) is not seen on imaging, DSA is currently considered the gold standard for evaluation of PACNS due to its high spatial resolution. 8. Which is true regarding advanced imaging in radiation necrosis? A. Increased N-acetylaspartate (NM) on MR spectroscopy (MRS) images B. Hypometabolism on fluorodeoxyglucose-positron emission tomography (FDG-PET) scan C. Increased relative cerebral blood volume (rCBV) on MR perfusion images D. Myoinositol peak on MRS images Source: Riascos-Castaneda, Roy F., et al., editors. RadCases Plus Q&A: Neuro Imaging. 2nd ed., Thieme, 2018. Explanation: • Radiation necrosis is generally hypometabolic on FOG-PET. NM decreases, rCBV is low or normal, and there is no myoinositol peak on MRS images.
  • 205.
    EBDR Exam MCQs& Concepts September 2022 Dr. Kareem Alnakeeb 199 9. Imaging findings of mesial temporal sclerosis (MTS) include which of the following? A. Small or atrophic hippocampus ipsilateral to the seizure focus with increased T1 signal. B. Loss of internal architecture and loss of hippocampal head digitations. C. Dilatation of the contralateral temporal horn. D. Decreased T2 signal intensity of the ipsilateral amygdala. E. Atrophy of contralateral mammillary body and fornix. Source: Riascos-Castaneda, Roy F., et al., editors. RadCases Plus Q&A: Neuro Imaging. 2nd ed., Thieme, 2018. Explanation: • Findings of MTS include loss of internal architecture and loss of hippocampal head digitations. • Findings also include small or atrophic hippocampus ipsilateral to the seizure focus with increased T2 (not Tl) signal, dilatation of the ipsilateral temporal horn, increased (not decreased) T2 signal intensity of the ipsilateral amygdala, and atrophy of ipsilateral (not contralateral) mammillary body and fornix. 10. Which of the following is a characteristic of the typical presentation of epidural hematomas? A. Will not cross the sutures B. Crescent in shape C. Content is venous blood D. Will not cross the dural folds E. 50% are infratentorial secondary to skull base fractures Source: Riascos-Castaneda, Roy F., et al., editors. RadCases Plus Q&A: Neuro Imaging. 2nd ed., Thieme, 2018. Explanation: • Epidural hematomas typically will not cross the sutures and they have a biconvex shape, the content is arterial blood, they will cross the dural folds, and > 90% are supratentorial in location. 11. Which is a characteristic of pyogenic spondylodiskitis? A. Sparing of the intervertebral disk B. Prevertebral soft tissue edema C. Involvement of the posterior elements D. Noncontiguous spread of infection Source: Riascos-Castaneda, Roy F., et al., editors. RadCases Plus Q&A: Neuro Imaging. 2nd ed., Thieme, 2018. Explanation: • Prevertebral edema is a common feature of pyogenic spondylodiskitis and aids in differentiating it from degenerative spondylosis. Sparing of the disk, involvement of the posterior elements, and noncontiguous spread of infection are more common in tuberculous spondylitis. 12. A dark rim is noted on susceptibility images in a cystic lesion of the sella/suprasellar region. Which of the following is the most likely diagnosis? A. Chronic pituitary apoplexy B. Rathke cleft cyst C. Suprasellar meningioma D. Papillary craniopharyngioma E. Cystic degeneration of a pituitary adenoma
  • 206.
    EBDR Exam MCQs& Concepts September 2022 Dr. Kareem Alnakeeb 200 Source: Riascos-Castaneda, Roy F., et al., editors. RadCases Plus Q&A: Neuro Imaging. 2nd ed., Thieme, 2018. Explanation: • Chronic pituitary apoplexy will present as a cystic lesion with a hemosiderin rim better seen on the susceptibility sensitive images. • None of the other options would present like this. • Adamantinomatous craniopharyngiomas are more frequently seen in the pediatric population and tend to calcify. 13. Which of the following temporal lobe lesions typically present as a cystic mass with an enhancing nodule? A. Supratentorial ependymoma B. Central neurocytoma C. Ganglioglioma D. DNET E. Embryonal tumor with multilayered rosettes Source: Riascos-Castaneda, Roy F., et al., editors. RadCases Plus Q&A: Neuro Imaging. 2nd ed., Thieme, 2018. Explanation: • Gangliogliomas present as cystic masses in the temporal lobe with a mural nodule. • Supratentorial ependymomas tend to be extraventricular and show a heterogenous appearance, such as embryonal tumors with multilayered rosettes. 14. Apical pseudotumor of the orbit extending to the cavernous sinus associated with painful ophthalmoplegia is known as which one of the following? A. Ramsay Hunt syndrome B. Tolosa-Hunt syndrome C. Horner syndrome D. Gradenigo syndrome E. Bell's palsy Source: Riascos-Castaneda, Roy F., et al., editors. RadCases Plus Q&A: Neuro Imaging. 2nd ed., Thieme, 2018. Explanation: • This is the definition of TolosaHunt syndrome. • Ramsay Hunt syndrome is varicella zoster virus infection affecting the sensory fibers of cranial nerves VII and VIII. • Horner syndrome is an interruption of the cervical sympathetic pathway. • Gradenigo syndrome is petrous apicitis that presents with retro-orbital pain and cranial nerve palsies. • Bell's palsy is facial nerve paralysis secondary to herpes simplex infection. 15. Central restricted diffusion is present in which of the following conditions? A. Metastasis B. Abscess C. Cyst D. Primary brain neoplasm
  • 207.
    EBDR Exam MCQs& Concepts September 2022 Dr. Kareem Alnakeeb 201 Source: Riascos-Castaneda, Roy F., et al., editors. RadCases Plus Q&A: Neuro Imaging. 2nd ed., Thieme, 2018. Explanation: • High diffusion-weighted imaging signal with low apparent diffusion coefficient values are typically present centrally within cerebral abscesses, representing true restricted diffusion. 16. Which of the following leukodystrophies typically enhances? A. Canavan disease B. X-linked adrenoleukodystrophy C. Alexander disease D. Pelizaeus-Merzbacher disease Source: Riascos-Castaneda, Roy F., et al., editors. RadCases Plus Q&A: Neuro Imaging. 2nd ed., Thieme, 2018. Explanation: • X-linked adrenoleukodystrophy (ALD) is characterized by an intermediate zone that represents disruption of the blood-brain barrier that typically enhances. • The other white matter diseases in the question do not enhance. 17. Which of the following mitochondrial disorders classically presents with cortical involvement? A. Leigh syndrome B. Myoclonus epilepsy with ragged red fibers (MERRF) C. Mitochondrial encephalopathy with lactic acidosis and strokelike lesions (MELAS) D. d) Keams-Sayre syndrome Source: Riascos-Castaneda, Roy F., et al., editors. RadCases Plus Q&A: Neuro Imaging. 2nd ed., Thieme, 2018. Explanation: • MELAS is a mitochondrial disorder that presents with cortical and white matter involvement in a nonvascular distribution, mimicking stroke. 18. Which of the following is typical of Joubert syndrome? A. Normal decussation of the corticospinal tracts B. A large fourth ventricular cyst C. Elongated superior cerebellar peduncles D. Normal cerebellar vermis Source: Riascos-Castaneda, Roy F., et al., editors. RadCases Plus Q&A: Neuro Imaging. 2nd ed., Thieme, 2018. Explanation: • Joubert syndrome features lack of decussation of the both the corticospinal tracts and the superior cerebellar peduncles. • Typical features also include elongated superior cerebellar peduncles and a small cerebellar vermis. • A large fourth ventricular cyst is not a characteristic of Joubert syndrome. 19. Which of the following lesions can show increased T1 signal in the sellar region? A. Pituitary microadenoma B. Pituitary macroadenoma C. Rathke cleft cyst D. Hamartoma of the tuber cinereum
  • 208.
    EBDR Exam MCQs& Concepts September 2022 Dr. Kareem Alnakeeb 202 Source: Riascos-Castaneda, Roy F., et al., editors. RadCases Plus Q&A: Neuro Imaging. 2nd ed., Thieme, 2018. Explanation: • Rathke deft cyst, pituitary apoplexy, sellar aneurysms, and hemorrhagic sellar metastasis can typically demonstrate increased T1 signal. 20. Concerning renal lymphoma, which one is TRUE? A. Multiple or solitary focal nodular masses are the most common form. B. It demonstrates uniform, hyperintense enhancement after IV gadolinium. C. Direct extension to and involvement of the psoas muscle is more characteristic of primary renal cell carcinoma than of renal lymphoma. D. Tumor thrombus commonly occurs in renal lymphoma. Source: ACR. Explanation: A. Correct. There are 3 basic patterns of renal involvement by lymphoma: 1) direct invasion by adjacent nodal disease, 2) focal masses that may be solitary or multiple (most common), and 3) diffuse infiltration. B. Incorrect. Renal lymphoma typically enhances minimally to a mildly heterogenous pattern. C. Incorrect. Renal lymphoma can commonly extend to and involve the adjacent psoas muscle. This feature is rare in primary renal carcinoma. D. Incorrect. Renal lymphoma rarely causes tumor thrombus. This is a common feature of renal carcinoma. 21. Concerning prostate carcinoma, which one of the following is CORRECT? A. 30% of prostate cancers arise from the peripheral zone of the prostate. B. T1-weighted images provide the best contrast for detecting most prostate carcinomas. C. Most prostate cancers demonstrate increased enhancement on immediate post gadolinium fat- saturated T1 images. D. Prostate cancer metastasizes early along the gonadal vein/lymphatic pathway to the periaortic and pericaval region near the level of the kidneys. Source: ACR. Explanation: A. Incorrect. 70% of prostate cancers arise from the peripheral zone, the remainder from the transitional and central zones B. Incorrect. Prostate carcinomas in the peripheral zone are generally isointense to surrounding prostate tissue on T1-weighted images. T2-weighted images demonstrate prostate cancers as low signal intensity compared to the surrounding normal high signal intensity peripheral zone. C. Correct. Prostate cancer in the peripheral zone (where the majority of prostate cancers arise) demonstrates increased enhancement compared to the normal peripheral zone tissue. D. Incorrect. This metastatic pathway is characteristic of testicular neoplasms, not prostatic. Lymph node metastases from prostate carcinoma are generally first to the obturator, external and internal iliac chains. 22. Concerning renal medullary carcinoma, which one is TRUE? A. Usually peripheral in location B. Commonly seen in diabetic females C. Common in patients with sickle trait D. Often very small at presentation
  • 209.
    EBDR Exam MCQs& Concepts September 2022 Dr. Kareem Alnakeeb 203 Source: ACR. Explanation: A. Incorrect. They are usually central. B. Incorrect. Commonly seen in African American patients with sickle trait; more commonly male. There is no association with diabetes. C. Correct. Renal medullary carcinoma typically is seen as an infiltrative mass in patients with sickle trait. D. Incorrect. They are usually large at presentation. 23. Concerning congenital ureteropelvic junction (UPJ) obstruction, which one of the following is TRUE? A. It is an uncommon cause of hydronephrosis in children. B. Urinary tract infection is the most common presentation. C. Females and males are affected equally. D. The presence of crossing vessels decreases the success rate of pyeloplasty. Source: ACR. Explanation: A. Incorrect. It is the MOST common cause of hydronephrosis in children. B. Incorrect. o UPJ obstruction is being discovered increasingly in the prenatal period due to frequent use of obstetric ultrasound. o When detected due to symptoms or signs, congenital ureteropelvic junction obstruction most often presents in infancy or childhood with an abdominal mass, flank or abdominal pain, failure to thrive, or nonspecific gastrointestinal complaints. o Infection, hypertension, hematuria, and stone formation less commonly are the cause for the child to come to medical attention. o In a significant number of cases, the disorder is clinically silent into adulthood, when hematuria, flank pain, fever, or rarely, hypertension, are the presenting clinical symptoms. o Pain in adults is often episodic and in some cases may only present by high urine flow rates such as those produced by beer drinking. C. Incorrect. Males are affected more than females by 2:1. D. Correct. Crossing vessels are seen in only 15%-20% of cases but significantly reduce the success of pyeloplasty. Thus, many advocate the use of CT for preoperative planning. 24. What is the Bosniak classification of a renal cyst with complex septations and dense calcification? A. Bosniak I B. Bosniak II C. Bosniak III D. Bosniak IV Source: ACR. Explanation: A. Incorrect. Bosniak I cysts are simple cysts and have no septations or calcifications. These require no further evaluation B. Incorrect. Bosniak II cysts have some atypical features, but are most likely benign. This group of cysts can have thin septations or calcifications but not complex septations or dense calcifications. Some lesions in this group are followed (subgroup IIF). Hyperdense, nonenhancing cysts are included in the Bosniak II category.
  • 210.
    EBDR Exam MCQs& Concepts September 2022 Dr. Kareem Alnakeeb 204 C. Correct. Bosniak III cysts can have dense calcifications, complex septations, and multiloculated cysts. This group cannot be distinguished from malignancy, and often these lesions require surgical exploration D. Incorrect. Bosniak IV cystic masses have features which strongly suggest malignancy, such as an enhancing solid component or thick irregular walls. Lesions in this category are treated as presumed renal carcinomas. 25. Concerning blunt trauma to the bladder, which one of the following is TRUE? A. Intraperitoneal rupture accounts for the majority of cases. B. Less than 20% of extraperitoneal ruptures have pelvic fractures. C. Intraperitoneal rupture is typically treated with surgical repair. D. CT with intravenous contrast can exclude major bladder injury. Source: ACR. Explanation: A. Incorrect. Extraperitoneal bladder ruptures account for 80%-90% of major bladder injuries. Intraperitoneal ruptures account for 10%-20% of major bladder injuries. B. Incorrect. Extraperitoneal bladder ruptures are almost always associated with pelvic fractures and many are thought to be due to bladder laceration by the fracture fragments. (Although other causes of extraperitoneal bladder injury have also been suggested, such as stress applied to the puboprostatic ligaments causing the bladder wall to tear.) C. Correct. Intraperitoneal bladder rupture is typically treated with surgical repair of the tear and diverting vesicostomy. D. Incorrect. Even delayed images of the bladder with CT and intravenous contrast are not adequate to exclude major bladder injury. This is because there is inadequate distension of the bladder. At least 300 ml of fluid is required to adequately distend the bladder and evaluate for extravasation. 26. Concerning renal angiomyolipoma’s, which one finding is MOST diagnostic? A. Fluid/fluid levels B. Fat C. Homogeneous soft tissue D. Large irregular calcification Source: ACR. Explanation: A. Incorrect. These lesions may occasionally hemorrhage but are usually incidental masses with mixed amounts of soft tissue and macroscopic fat. B. Correct. While other lesions such as renal cell carcinoma, oncocytoma, Wilm’s and metastasis have also been reported with areas of fat within these tumors, these cases are rare. C. Incorrect. Angiomyolipomas have varying amounts of fat and soft tissue. Some have no fat visible by CT and a solid soft tissue renal mass in such a case is indistinguishable from renal cell carcinoma and should be treated as such. D. Incorrect. Calcification in angiomyolipomas is unusual but may occur if there has been prior hemorrhage. 27. Which of the following is associated with testicular microlithiasis? A. Testicular torsion B. Epididymo-orchitis C. Right-sided varicocele D. Testicular neoplasm
  • 211.
    EBDR Exam MCQs& Concepts September 2022 Dr. Kareem Alnakeeb 205 Source: ACR. Explanation: A. Incorrect. Microlithiasis is not typically seen in testicular torsion. B. Incorrect. While the calcifications may be the result of prior infection, it does not have an increased association with infection C. Incorrect. There is no increased incidence of varicocele with testicular microlithiasis. D. Correct. While testicular microlithiasis is often incidental, there is an increased incidence of testicular neoplasm, most of which are germ cell tumors. 28. Regarding intravaginal testicular torsion, which one of the following is TRUE? A. Color Doppler is more sensitive than power Doppler for detecting flow. B. It is associated with an abnormal mesenteric attachment bilaterally. C. It accounts for 70 % of cases of acute scrotal pain in adolescents. D. Symmetric homogeneous echogenicity of the testes excludes the diagnosis. Source: ACR. Explanation: A. Incorrect. Power Doppler is more sensitive than color Doppler for detecting flow, especially in neonates and young boys. Power Doppler shows superiority in demonstrating intratesticular vessels. Power Doppler is limited somewhat by being more sensitive to patient motion than color Doppler. B. Correct. Cases of intravaginal torsion are caused by a bell-clapper deformity of attachment of the mesentery to the testis. The abnormality is bilateral in nearly all cases. C. Incorrect. Testicular torsion only accounts for 30% of cases of scrotal pain in boys age 12-18. Epididymoorchitis or torsion of an appendix testis/epididymis are much more common causes of scrotal pain. D. Incorrect. In early torsion (when most critical to detect torsion to permit salvaging the testicle), testes may have normally preserved gray-scale appearance. Later gray-scale ultrasound may demonstrate decreased echogenicity of the testis, testicular swelling or reactive hydrocele. Early on, the sonographic diagnosis of testicular torsion relies on the demonstration of decreased or absent flow in the torsed testis on color or power Doppler. 29. Concerning adrenal cortical carcinoma, which one is TRUE? A. It is the most common cause of an adrenal mass. B. It most often displays areas of macroscopic fat. C. It usually presents with <10 H.U. on non-contrast CT. D. It usually presents as a large heterogeneous soft tissue mass. Source: ACR. Explanation: A. Incorrect. Adrenal adenoma and metastatic disease are much more common than primary adrenal cortical carcinoma. B. Incorrect. While fat can rarely be seen in these tumors, macroscopic fat in an adrenal lesion is almost always in a myelolipoma C. Incorrect. Adrenal adenomas are more likely to present with the above characteristics. D. Correct. Most adrenal cortical carcinomas are > 6 cm and often have central necrosis. Calcification is seen in approximately 30% of these lesions.
  • 212.
    EBDR Exam MCQs& Concepts September 2022 Dr. Kareem Alnakeeb 206 30. Concerning gonadal vein thrombosis, which one is TRUE? A. Most common on the right side in post-partum women B. Best study for diagnosis is excretory urography C. Usually treated surgically D. Commonly seen in diabetic males Source: ACR. Explanation: A. Correct. TRUE. Gonadal vein thrombosis is in the differential for cause of fever in postpartum woman. B. Incorrect. FALSE. CT or MR are most sensitive in detection of gonadal vein thrombosis. The diagnosis may also be made with US. IVU would not be expected to be helpful in this diagnosis. C. Incorrect. FALSE. Patients are usually treated with anticoagulation and antibiotics. D. Incorrect. FALSE. There is no association with diabetes; gonadal vein thrombosis is most commonly seen in postpartum women (answer A). 31. What is the MOST common location of metastatic peritoneal implants? A. Right paracolic gutter B. Medial border of the cecum C. Superior border of the sigmoid colon D. Rectovesical space Source: ACR. Explanation: A. Right paracolic gutter 18%. B. Medial border of the cecum 41%. C. Superior border of the sigmoid colon 21%. D. Rectovesical space in 56%. It is the most dependent portion of the peritoneal cavity in both the upright and supine positions. 32. Which of the following statements about localized fibrous tumors of the pleura is TRUE? A. They are associated with asbestos exposure. B. They are associated with hypertrophic pulmonary osteoarthropathy. C. They account for the majority of pleural tumors. D. Most of these tumors arise from parietal pleura. Source: ACR. Explanation: A. Incorrect. B. Correct. Localized fibrous tumor of the pleura are relatively rare tumors of the pleura. About 80% of them arise from the visceral pleura. They affect male and female patients equally. They are not associated with smoking, asbestos exposure or other environmental pollutants. About half of the patients are asymptomatic when the tumor is discovered incidentally. They occasionally reach very large size and produce symptoms of cough, dyspnea and chest pain. Paraneoplastic syndromes such as hypoglycemia and hypertrophic osteoarthropathy are present in 4-5 % of the cases. C. Incorrect. D. Incorrect.
  • 213.
    EBDR Exam MCQs& Concepts September 2022 Dr. Kareem Alnakeeb 207 33. Concerning Morton’s neuroma, which one of the following is TRUE? A. It involves the digital branch of the plantar nerves. B. It is most common between the second and third toes. C. Histology demonstrates scattered mitosis and hypercellularity. D. It has diffuse high-signal intensity on T2-weighted spin-echo images. Source: ACR. Explanation: A. Morton’s neuroma is a non-neoplastic condition (compression neuropathy) representing neural degeneration and perineural fibrosis secondary to entrapment of the digital branch of the medial or lateral plantar nerves of the foot at the transverse intermetatarsal ligament. There may be associated inflammation. Patients may experience pain and numbness. Lesions are typically unilateral. There is a marked female predilection, as high as 18:1. B. The second web space is the second most common location. It is most common at the third web space between the third and fourth toes. C. Because it is caused by impingement, histology shows perineural fibrosis, edema of the endoneurium, and axonal degeneration and necrosis. D. On T2W images, it is characterized by isointensity or lower signal intensity relative to fat. This helps to differentiate a Morton’s neuroma from a true neuroma or fluid at the intermetatarsal bursa, which has high signal intensity. 34. Concerning tunica albuginea cysts, which one is TRUE A. They are intratesticular in location. B. They are not palpable. C. They range from 2 mm to 5 mm in size. D. They are located in the posterior and inferior aspect of testis. Source: ACR. Explanation: A. Incorrect. Tunica Albuginea cyst are extra testicular in location, however when large in size may mimic an intratesticular cyst. B. Incorrect. These cysts are palpable and patients present with a palpable lump. C. Correct. These cysts are of mesothelial origin and range from 2-5mm in size. D. Incorrect. Their characteristic location is at the upper anterior or lateral aspect of the testicle. 35. Which of the following measurements provides the BEST estimate of gestational age in a normal mid- first-trimester pregnancy? A. Yolk sac diameter B. Crown-rump length C. Mean sac diameter D. Biparietal diameter Source: ACR. Explanation: A. Yolk sac diameter is not an accepted measure of gestational age B. An embryo is normally visible beginning at 6 weeks' gestation, and crown-rump length provides the best estimate of embryonic age C. The mean sac diameter may be used in the early first trimester but is not as accurate as crown-rump length in the mid first trimester, once an embryo is visible. D. Biparietal diameter should be used beginning at 12 weeks' gestation, but cannot be obtained in a 6-7 week embryo.
  • 214.
    EBDR Exam MCQs& Concepts September 2022 Dr. Kareem Alnakeeb 208 36. What is the MOST common ultrasound finding in acute pyelonephritis? A. Perirenal fluid collection B. Normal-appearing kidney C. Loss of corticomedullary differentiation D. Poorly marginated, hypoechoic mass Source: ACR. Explanation: A. Incorrect. A perirenal fluid collection may be seen as a complication of acute pyelonephritis, but in most cases there are no complications and the ultrasound findings are normal. B. Correct. A normal appearance is the most common ultrasound finding. C. Incorrect. Pyelonephritis may alter the echo texture of the renal parenchyma, but in most cases the ultrasound findings are normal. D. Incorrect. Pyelonephritis may produce areas of increased as well as decreased echogenicity which may simulate a mass, but in most cases the ultrasound findings are normal. 37. Concerning serous cystadenoma of the pancreas, which of the following is TRUE? A. Individual cysts are usually larger than 2 cm. B. The majority have a calcified central stellate scar. C. It is common in von Hippel-Lindau disease. D. It has a propensity to occur in young women. Source: ACR. Explanation: • Although most patients with serous cystadenoma do not have von Hippel-Lindau (VHL) disease, serous cystadenoma is more prevalent among patients with VHL particularly multiple lesions. In sporadic cases, serous cystadenoma is usually a solitary lesion. 38. Regarding toxic megacolon, which one of the following is TRUE? A. CT is usually required for diagnosis. B. Peritonitis can occur without perforation. C. It is due to severe and extensive submucosal inflammation. D. Transverse colonic dilatation with normal haustra is diagnostic. Source: ACR. Explanation: A. Diagnosis is typically made with conventional abdominal radiograph. B. The inflammatory exudate of the colon seeps through the serosa and may cause peritonitis without frank perforation. C. It is due to severe and extensive transmural inflammation. D. The profound inflammation and extensive ulceration of toxic megacolon always abolish the haustral pattern, so the presence of haustral folds excludes the diagnosis. 39. Concerning focal hepatic fatty sparing, what is its appearance on imaging studies? A. Hypodense on CT relative to remaining liver B. Hyperechoic on US relative to remaining liver C. Demonstrates no signal loss on fat-suppressed MR images D. Heterogeneous enhancement Source: ACR. Explanation: A. Incorrect. Areas of fatty sparing are hyperdense. B. Incorrect. Fatty sparing is hypoechoic. C. Correct.
  • 215.
    EBDR Exam MCQs& Concepts September 2022 Dr. Kareem Alnakeeb 209 D. Incorrect. Enhancement pattern is similar to the entire liver. 40. What is the MOST common malignant primary hepatic tumor? A. Hepatocellular carcinoma B. Lymphoma C. Angiosarcoma D. Intrahepatic cholangiocarcinoma Source: ACR. Explanation: A. Hepatocellular carcinoma is the most common primary hepatic malignancy. B. Liver involvement may be an extrahepatic manifestation of hematologic malignancy. C. Angiosarcoma is rare, and has been associated with exposure to Thorotrast and vinyl chloride. D. Cholangiocarcinoma is the second most common primary hepatic malignancy. 41. Concerning cervical carcinoma, what stage is a lesion that is confined to the upper two thirds of the vagina on clinical exam and that shows right hydroureter to the level of a poorly defined cervical soft tissue mass on CT exam? A. Stage II A B. Stage II B C. Stage III A D. Stage III B Source: ACR. Explanation: A. Incorrect. At stage II A the tumor has spread beyond the cervix but has no obvious parametrial involvement, is confined to the upper two thirds of the vagina and no invasion of the ureter or bladder. B. Incorrect. Stage II B has obvious parametrial involvement but does not extend to the pelvic side wall. C. Incorrect. Stage III A extends to the lower third of the vagina but not the pelvic sidewall and does not obstruct the ureters or invade adjacent organs. D. Correct. Stage III B tumors extend to pelvic sidewall and/or causes hydronephrosis or non- functioning kidney. 42. Concerning Mirizzi syndrome, which one is TRUE? A. Mirizzi syndrome is caused by gallstone impaction in the common hepatic duct. B. Mirizzi syndrome is facilitated by an anatomic variant. C. Cholecystocolonic fistula can complicate Mirizzi syndrome. D. Bile duct injury at surgery is less likely in cases of Mirizzi syndrome. Source: ACR. Explanation: A. Incorrect. Mirizzi syndrome is a complication of longstanding cholelithiasis. It is caused by impaction of a gallstone in the cystic duct or in the gallbladder neck. Extrinsic mass effect of the stone in the cystic duct on the common hepatic duct (CHD) or associated inflammatory changes extending to the CHD causes obstruction of the extrahepatic biliary tree B. Correct. Mirizzi syndrome is defined as extrinsic obstruction of the common hepatic duct (CHD), usually caused by mass effect from a stone lodged in the adjacent cystic duct. CHD compression is more likely in patients with low cystic duct insertions, because the cystic duct runs more parallel and in closer proximity to the CHD when this variant anatomy is present. Inflammatory changes extending from the cystic duct or gallbladder neck can also cause narrowing of the bile duct
  • 216.
    EBDR Exam MCQs& Concepts September 2022 Dr. Kareem Alnakeeb 210 C. Incorrect. A cholecystocholedochal (not cholecystocolonic) fistula is a complication of Mirizzi syndrome in 9 – 39% of cases. Fistula repair is required at the time of surgery. This requires either choledochoplasty or hepaticojejunostomy. D. Incorrect. Because of adhesions, inflammation and variant bile duct anatomy, the surgical dissection is more difficult in Mirizzi syndrome. Inadvertent injury to the biliary tree or hepatic artery is more likely. Although laparoscopic cholecystectomy can be performed successfully in patients with Mirizzi syndrome, more require open cholecystectomy than in cases of uncomplicated cholecystitis. 43. Concerning intrahepatic cholangiocarcinoma, which one is TRUE? [not sure] A. It is usually hyperdense during the arterial phase of contrast enhancement. B. It demonstrates washout during delayed CT imaging. C. Post obstructive hepatic atrophy can be a prominent feature. D. Extrahepatic spread of intrahepatic cholangiocarcinoma is uncommon. Source: ACR. Explanation: A. Incorrect. In the absence of primary sclerosing cholangitis, intrahepatic cholangiocarcinoma usually presents as a bulky mass. The lesion tends to be hypodense to background liver on non contrast CTs. Following the dynamic administration of IV contrast, intrahepatic cholangiocarcinoma tends to remain relatively hypovascular (not hyperdense), particularly centrally. B. Incorrect. Increased patchy enhancement can be observed during the portal venous phase. Retention of contrast within the extracellular space of the central stoma at delayed CT imaging (5 – 10 minutes) is relatively characteristic for these lesions. C. Correct. High-grade obstruction from intrahepatic and hilar cholangiocarcinomas can cause atrophy of the hepatic parenchyma surrounding the pathologically dilated biliary tree. Capsular retraction occurs in about 20% of cases. This retraction likely reflects the atrophy of a small volume of hepatic parenchyma beneath Glisson’s capsule. Surprisingly marked segmental and lobar atrophy can occur when cholangiocarcinoma affects more central intrahepatic or hilar bile ducts D. Incorrect. The extrahepatic spread of peripheral, intrahepatic cholangiocarcinoma is not uncommon. In autopsy series, it is noted in about 50 – 70% of cases. Metastatic disease to regional celiac and left gastric nodes occurs frequently. The prevalence of microscopic nodal disease with its tendency to spread back to the liver makes the preoperative diagnosis of cholangiocarcinoma a contraindication for hepatic transplantation. 44. Concerning pseudocyst of the pancreas, which of the following is TRUE? A. Most common cystic pancreatic lesion B. Has an epithelial cell lining C. Does not communicate with the pancreatic duct D. Can be distinguished from mucinous cystic neoplasms by imaging Source: ACR. Explanation: • Pseudocyst is the most common pancreatic cystic lesion. Pseudocyst accounts for 85-90% of all cystic lesions of the pancreas. • True cysts and cystic neoplasms are not as common, and represent only 10-15% of pancreatic cystic lesions. 45. Concerning islet cell tumors of the pancreas, which one of the following is TRUE? A. Gastrinoma is the most common type of functioning islet cell tumor. B. Zollinger-Ellison Syndrome is caused by gastrinoma.
  • 217.
    EBDR Exam MCQs& Concepts September 2022 Dr. Kareem Alnakeeb 211 C. Functioning islet cell tumors usually present as large masses. D. Functioning islet cell tumors are usually hypovascular. Source: ACR. Explanation: A. Incorrect. Insulinomas are the most common functioning islet cell tumor with gastrinoma being the second most common. B. Correct. Zollinger-Ellison syndrome is caused by a gastrinoma. Peptic ulcer disease is seen in 90% of patients with gastrinoma, usually with more extensive involvement of the duodenum than with typical peptic ulcer disease. Many patients have diarrhea due to gastric hypersecretion. C. Incorrect. Most functioning islet cell tumors present as small (1 – 2 cm) hypervascular pancreatic masses D. Incorrect. Most functioning islet cell tumors are hypervascular. 46. Concerning gastric lymphoma, which one is TRUE? A. Accounts for 25% of all gastrointestinal lymphomas. B. Vast majority are Hodgkin’s lymphoma. C. Low-grade MALT lymphomas usually present as large bulky masses on upper gastrointestinal barium studies. D. Advanced gastric lymphoma is usually large and involves the body and antrum. Source: ACR. Explanation: A. Incorrect. Lymphoma involves the stomach more frequently than any other portion of the gastrointestinal tract and accounts for 50% of all gastrointestinal lymphomas. B. Incorrect. Almost all gastric lymphomas are non-Hodgkin’s lymphoma of B-cell origin. C. Incorrect. Low-grade MALT lymphomas usually present as varied sized, rounded nodules on double contrast barium studies. D. Correct. Advanced lymphoma is a large bulky mass and most commonly involves the gastric body and antrum. 47. In routine adult practice, which underlying condition most frequently produces biliary obstruction? A. Choledocholithiasis. B. Cholangiocarcinoma C. Carcinoma of the pancreatic head D. Primary sclerosing cholangitis E. Mirizzi syndrome Explanation: (by Perplexity AI, generated with the OpenAI O3 model) • Migrated gallstones obstructing the common bile duct (choledocholithiasis) account for the majority of biliary obstruction cases encountered in day-to-day UK radiology. • Malignant strictures—chiefly pancreatic head carcinoma and cholangiocarcinoma—are important but each presents less often than stones. • Primary sclerosing cholangitis causes multifocal biliary narrowing rather than a single dominant blockage and is comparatively uncommon. • Mirizzi syndrome is a rare variant of gallstone disease involving extrinsic compression of the common hepatic duct by a cystic-duct stone and is far less frequent than straightforward choledocholithiasis.
  • 218.
    EBDR Exam MCQs& Concepts September 2022 Dr. Kareem Alnakeeb 212 48. Concerning jaundice, which of the following is the MOST common etiology? A. Choledocholithiasis. B. Pancreatitis. C. Benign stricture. D. Pancreatic carcinoma. Source: ACR. Explanation: A. Incorrect. Choledocholithiasis accounts for 20% of cases of biliary obstruction. B. Incorrect. Pancreatitis accounts for 8%. C. Correct. Benign stricture form surgery, trauma, or biliary intervention accounts for almost half of the cases of biliary obstruction. D. Incorrect. Pancreatic cancer does cause biliary obstruction, but less commonly than benign stricture. 49. What of the following is associated with primary sclerosing cholangitis (PSC)? A. Cholangiocarcinoma B. Choledochal cyst C. Choledochocele D. Recurrent pyogenic cholangitis Source: ACR. Explanation: A. Correct. Primary sclerosing cholangitis (PSC) is a significant risk factor for cholangiocarcinoma. Among patients with PSC, the lifetime risk of cholangiocarcinoma is 10-15%, with an annual risk of 1.0-1.5%. B. Incorrect. Choledochal cyst is considered to be congenital in etiology. It is postulated to be related to anomalous development of the junction between the common bile duct and the pancreatic duct. In patients with pancreaticobiliary maljunction (PBM), gallbladder cancers occur in 15% and bile duct cancers occur in 5%. However, carcinogenesis in patients with choledochal cyst or PBM is not related to PSC. C. Incorrect. Most authors consider a choledochocele to be a Type III choledochal cyst. As such, it is not related to PSC. D. Incorrect. Recurrent pyogenic cholangitis (Oriental cholangiohepatitis) is an infectious disease, with no particular association with PSC. 50. Concerning renal cystic disease, which one is TRUE? A. Autosomal recessive polycystic disease typically presents as multiple bilateral cysts in adulthood. B. Autosomal dominant polycystic disease typically presents as enlarged hyperechoic kidneys in the neonatal period. C. Acquired cystic renal disease in chronic renal failure patients on dialysis is indistinguishable from autosomal dominant polycystic disease. D. Autosomal dominant polycystic disease has a higher incidence of associated hepatic cysts than does autosomal recessive polycystic disease. Source: ACR. Explanation: A. Incorrect. Autosomal dominant polycystic disease usually presents with multiple bilateral simple renal cysts between ages 20-39 years. Autosomal recessive polycystic disease has a spectrum of presentation ages but is typically seen from the neonatal through childhood periods rather than adulthood.
  • 219.
    EBDR Exam MCQs& Concepts September 2022 Dr. Kareem Alnakeeb 213 B. Incorrect. This description is more typical of the appearance of the infantile form of ARPKD. C. Incorrect. The kidneys are typically small and atrophic with multiple cysts in acquired cystic renal disease of dialysis as compared to markedly enlarged kidneys in ADPCD. D. Correct. ADPCD typically has multiple hepatic cysts in over 50% of cases. Autosomal recessive polycystic disease is associated with hepatic fibrosis particularly in the juvenile onset form. 51. The staging chest CT of a 40 year old man with a known primary malignancy demonstrates cavitating pulmonary metastases. The least likely type of primary lesion would be: A. Squamous cell carcinoma B. Malignant melanoma C. Renal cell cancer D. Sarcomas E. Colonic carcinoma Source: Currie, Stuart, et al. SBAs for the FRCR 2A. 1st ed., Cambridge University Press, 2010. 52. An 18 year old woman who is 32 weeks pregnant is referred for an obstetric ultrasound for ongoing abdominal pain. She is shown to have a small placenta relative to gestational age. Which one of the following would be a possible cause? A. Molar pregnancy B. Maternal diabetes C. Umbilical vein obstruction D. Pre-eclampsia E. Maternal anaemia Source: Currie, Stuart, et al. SBAs for the FRCR 2A. 1st ed., Cambridge University Press, 2010. Explanation: • Pre-eclampsia, IUGR, chromosomal abnormality and intrauterine infection can all cause a decrease in placental size. • Enlargement of the placenta is defined as a measurement of >5 cm when obtained at right angles to the long axis of the placenta. • The causes of placentomegaly include maternal diabetes, chronic intrauterine infection (e.g. syphilis), maternal anaemia, thalassaemia and twin–twin transfusion syndrome. • Fetal chromosomal abnormalities may cause either a large or small placenta. 53. On the 20-week fetal anomaly scan, it is noticed that there is less than 1 mm of hypoechoic myometrium between placenta and echo-bright uterine serosa. An MRI is performed. On T2W images, the placenta is heterogeneous and bright, and causes junctional zone interruption and marked focal myometrial thinning. The serosa looks intact. These findings describe which of the following? A. Placenta accreta. B. Placenta increta. C. Placenta percreta. D. Placenta praevia. E. Placental abruption. Source: Bydder, Megan, et al. Get Through Final FRCR Part A: SBAs for the Modular Examination. 1st ed., Hodder Education Publishers, 2009. Explanation: Placenta Accreta Spectrum • The normal decidua forms a barrier between chorionic villi and uterus, preventing deep invasion of placental material.
  • 220.
    EBDR Exam MCQs& Concepts September 2022 Dr. Kareem Alnakeeb 214 • An underdeveloped or absent decidua permits direct contact of chorionic villi with the myometrium, known as placenta accreta. • When the villi invade the myometrium, it becomes placenta increta; if the serosa is penetrated, it is placenta percreta. • Diagnosis is difficult on ultrasound scan, but MRI can help. Risk Factors • Risk factors are previous caesarean section and myomectomy, multiparity and increasing maternal age. Complications and Management • Complications include maternal haemorrhage, premature delivery, intrauterine growth retardation and 5% chance of perinatal death. • To protect the mother, balloon catheters can be placed over the internal iliac arteries prior to caesarean delivery. 54. You are asked to perform an antenatal ultrasound examination and note that the placenta has an unusual morphology. You see an additional lobule, which is separate from the main bulk of the placenta. What is this variant of placental morphology known as? A. Circumvallate placenta. B. Bilobed placenta. C. Placenta membranacea. D. Succenturiate placenta. E. Placenta accrete Source: Lindsay, Richard, et al. SBAs for the FRCR Part 2A. 1st ed., Oxford University Press, 2012. Explanation: Accessory Placental Variants • This is an additional lobule separate from the main bulk of the placenta. • The significance of this variant is the rupture of vessels connecting the two components or retention of the accessory lobe with resultant post-partum haemorrhage. Circumvallate Placenta • Circumvallate placenta has a chorionic plate smaller than the basal plate, with associated rolled placental edges. • There is known to be an increased risk of placental abruption and haemorrhage with this type of placenta. Bilobed Placenta • A bilobed placenta is a placenta with two evenly sized lobes connected by a thin bridge of placental tissue. • This has no known increased risk of morbidity. Placenta Membranacea • Placenta membranacea is a thin membranous structure circumferentially occupying the entire periphery of the chorion. • There is an increased risk of placenta praevia, as a portion of the placenta completely covers the internal os. Abnormal Placental Invasion • Placenta accreta is not a variant of placental morphology.
  • 221.
    EBDR Exam MCQs& Concepts September 2022 Dr. Kareem Alnakeeb 215 • It occurs when there is superficial invasion of the chorionic villi of the placenta into the basalis layer of the uterine wall. • Deeper invasion of the myometrium is termed ‘placenta increta’. • Even deeper invasion involving the serosa or adjacent pelvic organs is termed ‘placenta percreta’. • The risk of this is catastrophic intrapartum haemorrhage at the time of placental delivery. 55. A 23 year old woman undergoes investigation for dyspareunia. Pelvic ultrasound was unremarkable. MRI demonstrates a 1 cm thin-walled ovoid cystic lesion at the anterolateral aspect of the upper vagina. It is homogeneously hypointense on T1 and shows marked hyperintensity on T2. What is the most likely diagnosis? A. Bartholin cyst B. Nabothian cyst C. Cervical fibroid D. Gartner duct cyst E. Cervical polyp Source: Currie, Stuart, et al. SBAs for the FRCR 2A. 1st ed., Cambridge University Press, 2010. Explanation: Vaginal and Cervical Lesions • Gartner’s duct cysts are remnants of mesonephric ducts and have a reported incidence of 1–2%. • They are ovoid, thin-walled cysts located at the anterolateral aspect of the upper vagina and generally measure less than 2 cm. • They may contain proteinaceous material, making them slightly hyperintense on T1. • They may be associated with Herlyn–Werner–Wunderlich syndrome (ipsilateral renal agenesis and ipsilateral blind vagina) and ectopic ureter inserting into the cyst. • Bartholin cysts are located at the lateral introitus adjacent to the labia minora. • Nabothian cysts are epithelial inclusion cysts which develop in the endocervical canal and are most commonly found in the perimenopausal period. • Cervical fibroids and cervical polyps show mainly as solid lesions. 56. On CT perfusion, which of the following is true? A. The CBV and MTT are decreased and CBF is increased in the infarct. B. The CBV, CBF, and MTT are decreased in the ischemic area. C. The CBV and CBF are decreased and MTT is increased in infarct area. D. CBV and MTT are decreased and CBF is normal or increased in ischemic area. Explanation: (by Perplexity AI, generated with the OpenAI O3 model) • In the core of an infarct the cerebral blood volume (CBV) falls because capillary flow has ceased, cerebral blood flow (CBF) is markedly reduced, and mean transit time (MTT) prolongs due to sluggish residual flow. • Penumbral tissue shows different patterns (often preserved CBV with low CBF and prolonged MTT), so options describing preserved or increased CBV/CBF relate to penumbra, not established infarction. • Option B has increased CBF—opposite to infarct physiology; option C wrongly suggests raised CBV; option D fits luxury reperfusion after ischemia, not the infarct core; option E describes isolated transit delay without volume or flow loss, typical of benign oligemia rather than infarct.
  • 222.
    EBDR Exam MCQs& Concepts September 2022 Dr. Kareem Alnakeeb 216 57. Which anatomic structure of the brain herniates in descending transtentorial (uncal) herniation? A. Cerebellar tonsils B. Cerebellar hemispheres C. Cingulate gyrus D. Uncus and hippocampus Explanation: (by Perplexity AI, generated with the OpenAI O3 model) • In transtentorial herniation, the cingulate gyrus is displaced downward because it sits just above and slightly lateral to the tentorial edge. • Descending transtentorial herniation is usually precipitated by a mass-effect lesion in the supratentorial compartment. The medial temporal lobe—the uncus with the adjacent hippocampal head—herniates downward over the tentorial edge, compressing the ipsilateral third cranial nerve and midbrain. • Cerebellar tonsils and hemispheres (A, B) are involved in tonsillar or upward cerebellar herniation, not transtentorial descent. • The cingulate gyrus (C) slides beneath the falx in subfalcine herniation. 58. Regarding fluoroscopically-guided air reduction for the treatment of intussusception in a child. Which of the following is most likely to predict a successful outcome? A. 24 hour history of symptoms. B. Intussusception seen in the sigmoid colon on US. C. US demonstrates fluid in the lumen around the intussusceptum. D. US shows blood flow in the intussusceptum. E. The child is aged >12 months. Explanation: (by Perplexity AI, generated with the OpenAI O3 model) • Preserved arterial and venous Doppler signals within the intussusceptum indicate that the bowel wall remains viable and not strangulated, correlating strongly with a high success rate for pneumatic (air) reduction. • Fluid between the intussusceptum and intussuscipiens (option C) is more often associated with oedema or early ischemia and predicts a lower success rate. • A longer symptom duration of 24 hours (option A) and age over 12 months (option E) are both linked to higher failure and perforation risk. • If the intussusception already reaches the sigmoid colon (option B), a long segment is involved, also reducing enema success. 59. You perform a transvaginal US and identify a pelvic mass with ground glass internal texture. Which of the following is the most likely diagnosis? A. Epithelial ovarian tumor B. Endometrioma C. Dermoid cyst D. Corpus luteum cyst Explanation: (by Perplexity AI, generated with the OpenAI O3 model) • Endometriomas contain chronic hemorrhagic debris, producing uniform low-level internal echoes that give a classic “ground-glass” appearance on ultrasound. • Malignant epithelial tumors more often show multiloculation, solid components or papillary excrescences. • Dermoids display echogenic Rokitansky nodules, acoustic shadowing and fat-fluid levels rather than homogeneous echoes.
  • 223.
    EBDR Exam MCQs& Concepts September 2022 Dr. Kareem Alnakeeb 217 • Functional corpus luteum cysts are usually thin-walled, may have a “ring of fire” peripheral Doppler signal, and frequently resolve within weeks, lacking the persistent ground-glass texture. 60. Which of the following is the most accurate sonographic measurement used for dating pregnancy in the first trimester? A. Biparietal diameter B. Crown rump length C. Humerus length D. Head circumference E. Abdominal circumference Explanation: (by Perplexity AI, generated with the OpenAI O3 model) • Crown-rump length (CRL) is the gold-standard biometric for dating a first-trimester pregnancy because early embryonic growth is highly linear and shows minimal biological variation, yielding a ±3–5-day error. • Biparietal diameter and head circumference become reliable only after about 12–13 weeks when ossification and head shape stabilise, but they are less accurate early on. • Long-bone (e.g. humerus) and abdominal measurements are used in the second and third trimesters; in the first trimester limb buds and abdominal contour are too small and variable for dependable dating. 61. A 24-year-old woman attends A&E with lower abdominal pain and vaginal bleeding. A pregnancy test is positive. She is hemodynamically stable, and an ultrasound is requested to confirm the presumed diagnosis of an ectopic pregnancy. Which of the following is the most common location for an ectopic pregnancy? A. Cervix B. Ovary C. Abdominal cavity D. Ampullary portion of the Fallopian tube E. Interstitial portion of the fallopian tube Source: Currie, Stuart, et al. SBAs for the FRCR 2A. 1st ed., Cambridge University Press, 2010. Explanation: • The most common site of implantation is the fallopian tube, which accounts for over 90% of ectopic pregnancies. • Ovarian and abdominal sites account for only approximately 3% and 1%, respectively. • Within the fallopian tube the most common site is the ampulla (73%) followed by the fimbrial and interstitial regions. 62. A 25-year-old has an ultrasound at 39 weeks gestation of a singleton pregnancy. The amniotic fluid volume is less than 500 mL. What is the most likely underlying cause for this? A. Severe growth restriction B. Maternal diabetes mellitus C. Trans-oesophageal fistula (TOF) D. Duodenal atresia E. Cystic adenomatoid lung Source: Proctor, Robin. Final FRCR Part A Modules 4-6 Single Best Answer MCQs: The SRT Collection of 600 Questions with Explanatory Answers (MasterPass). 1st ed., CRC Press, 2009. Explanation: • Oligohydramnios is when there is less than 500 mL of amniotic fluid at term.
  • 224.
    EBDR Exam MCQs& Concepts September 2022 Dr. Kareem Alnakeeb 218 • It is associated with a 20 times increase in fetal abnormalities and occurs with renal anomalies, intrauterine growth restriction (IUGR) and most commonly with premature rupture of the membranes. • The other options are associated with polyhydramnios (amniotic fluid volume >1500-2000mL at term). 63. In an acyanotic child with an enlarged heart and an enlarged main pulmonary artery, which is the diagnosis? A. Transposition of the great vessels B. Ventricular Septal Defect (VSD) C. Truncus arteriosis D. TAPVR E. Tricuspid atresia Source: Leen, Edward. Succeeding in the New FRCR Part 2A Exam: Single Best Answer (SBA) Revision Questions for Modules 1-6. 1st ed., UNKNO, 2011. Explanation: • The other options are causes of an enlarged heart in a cyanotic child with a concave main pulmonary artery. 64. Concerning splenic trauma, which one is TRUE? A. The spleen is injured in 35% of penetrating abdominal trauma. B. The spleen is the second most common solid organ injured in blunt trauma. C. Grading splenic trauma is a reliable way to predict whether a patient will need splenectomy. D. Embolization techniques can be used to control splenic hemorrhage. Source: ACR. Explanation: • Splenic artery embolisation is widely accepted for haemorrhage control in haemodynamically stable patients and reduces splenectomy rates by preserving splenic tissue. • Penetrating trauma much less commonly involves the spleen than the 35% quoted, making option A incorrect. • The spleen is actually the most frequently injured solid organ in blunt abdominal trauma, not the second, so option B is wrong. • CT grading correlates imperfectly with clinical course; management decisions depend on hemodynamics rather than grade alone, so option C is false. 65. Which of the following is the most likely diagnosis of a high-density mass on head CT? A. Lymphoma B. Astrocytoma C. Schwannoma D. Paraganglioma Explanation: (by Perplexity AI, generated with the OpenAI O3 model) • Because of densely packed cells, lymphomas appear hyperdense on CT. • Primary CNS lymphoma is typically hyperdense but is intraparenchymal and lacks a dural tail. • High-grade astrocytoma is intra-axial, often heterogeneous and hypodense centrally due to necrosis. • Vestibular schwannoma arises at the cerebellopontine angle canal and is usually isodense to brain on non-contrast CT. • Paraganglioma rarely occurs intracranially and more often shows marked vascularity rather than uniform hyperdensity.
  • 225.
    EBDR Exam MCQs& Concepts September 2022 Dr. Kareem Alnakeeb 219 66. A 26-year-old pregnant woman attends for an obstetric ultrasound at 37 weeks. She is shown to have polyhydramnios. Which of the following would be a possible cause? A. Cystic adenoid malformation B. Ventricular septal defect C. Infantile polycystic kidney disease D. Posterior urethral valves E. Intrauterine growth retardation Source: Currie, Stuart, et al. SBAs for the FRCR 2A. 1st ed., Cambridge University Press, 2010. Explanation: • The remainder of the conditions listed above will cause oligohydramnios. • Polyhydramnios is defined as amniotic fluid volume >1500–2000 cm3 at term. • Most cases are due to maternal factors, with diabetes causing the majority of these. • Oligohydramnios is defined as an amniotic fluid volume of <500 cm3 at term; the most common causes include demise of the fetus, drugs and renal anomalies. 67. A well-circumscribed, round, 15 mm mass is identified in the breast on first-round screening mammography. It has no associated calcification. From the following, choose the most appropriate management: A. Repeat mammography at the normal screening interval B. repeat mammography in 6 months C. MRI of the breast D. wide local excision of the lesion E. Ultrasound examination of the mass Source: Bydder, Megan, et al. Get Through Final FRCR Part A: SBAs for the Modular Examination. 1st ed., Hodder Education Publishers, 2009. Explanation: • Ultrasound scan is useful in determining whether mass lesions seen on the mammogram are cystic or solid. 68. At a breast cancer multidisciplinary team meeting, the case of a 60-year-old female patient is discussed. Following clinical examination, she is thought to have multifocal breast cancer, but this is not supported by the ultrasound and mammography findings. Which of the following is the most appropriate next investigation? A. Repeat ultrasound scan B. Repeat mammography with additional views C. MRI D. CT E. 18 FDG PET Source: Bydder, Megan, et al. Get Through Final FRCR Part A: SBAs for the Modular Examination. 1st ed., Hodder Education Publishers, 2009. Explanation: Treatment Planning Implications • Multifocal/multicentric cancer in the breast may alter treatment choice and when clinically suspected should be investigated with MRI. • MRI can also be used to assess the extent of residual disease in the breast after breast conservation surgery in cases where the surgical resection margins are positive. Recommended Breast MRI Protocol
  • 226.
    EBDR Exam MCQs& Concepts September 2022 Dr. Kareem Alnakeeb 220 • An acceptable series of sequences for breast MRI would be: 4 mm slice-thickness, transverse, spin echo T2W images of both breasts; 4 mm-thick, sagittal, spin echo T2W images of the affected breast; 4 mm-thick, sagittal, dynamic contrast-enhanced T1W gradient echo with fat saturation of the affected breast; and a delayed post-contrast sequence with the same parameters. 69. A 45-year-old man is admitted after a road traffic accident in which he sustained abdominal injuries. After fluid resuscitation he undergoes CT of the abdomen and pelvis with intravenous contrast. This demonstrates a serpiginous area of attenuation value 130 HU at the splenic hilum with surrounding lower-attenuation material. What is this most likely to represent? A. active arterial extravasation B. acute clotted blood C. acute unclotted blood D. splenic arterial calcification E. ascites Source: Bydder, Megan, et al. Get Through Final FRCR Part A: SBAs for the Modular Examination. 1st ed., Hodder Education Publishers, 2009. Explanation: Attenuation Values in Haemoperitoneum Evaluation • In the evaluation of haemoperitoneum by CT, attenuation values can help differentiate ascites, unclotted blood, active bleeding and haematoma. • Blood usually has a higher measured attenuation than other body fluids, but its appearance depends on the age, extent and location of haemorrhage. • Unclotted blood has an attenuation value of 30–45 HU, but this may be lower in patients with a lower serum haematocrit and if the haemorrhage is more than 48 hours old. • Clotted blood has an attenuation value of 45–70 HU, and identification of the area of highest attenuation haematoma (sentinel clot) on CT indicates the site of bleeding. • Active arterial extravasation is seen as an area of higher attenuation resembling that in the aorta, ranging from 85 HU to 370 HU. • It may be surrounded by lower-attenuation haematoma. • This finding indicates the need for urgent embolization or surgical treatment. 70. A 48-year-old female non-smoker presents to the Accident & Emergency Department with acute dyspnoea and chest pain. The chest radiograph shows bilateral basal airspace shadowing. Chest CT shows diffuse basal consolidation and air-bronchograms within a background of ground-glass opacity. There is septal thickening and bilateral pleural effusions. The most likely diagnosis is? A. Desquamative interstitial pneumonitis B. Lymphocytic interstitial pneumonitis C. Acute interstitial pneumonia D. Usual interstitial pneumonitis E. Cryptogenic organising pneumonia Source: Gupta, Chaitanya. 300 Single Best Answers for the Final FRCR Part A. 1st ed., Jaypee UK, 2010. Explanation: • This clinically presents as adult respiratory distress syndrome and has high mortality. • It has a fulminant course leading to respiratory failure and requiring mechanical ventilation with a mortality of > 50%. • CT findings are non-specific but include bilateral, diffuse ground-glass opacity with consolidation and air bronchograms.
  • 227.
    EBDR Exam MCQs& Concepts September 2022 Dr. Kareem Alnakeeb 221 • Honeycombing and traction bronchiectasis may be seen in advanced cases after recovery. 71. A 24-year-old male patient presents following a head injury with GCS of 13. There is bruising over the right temporal region. A CT scan shows no intracranial haemorrhage but does identify a longitudinal fracture through the petrous temporal bone. What complication should be considered? A. sensorineural hearing loss B. conductive hearing loss C. vertigo D. carotid artery injury E. sigmoid sinus injury Source: Bydder, Megan, et al. Get Through Final FRCR Part A: SBAs for the Modular Examination. 1st ed., Hodder Education Publishers, 2009. Explanation: Temporal Bone Fracture Types • Longitudinal fractures of the temporal bone represent 75% of temporal bone fractures and run parallel to the axis of the petrous pyramid. • Transverse fractures of the temporal bone are associated with sensorineural hearing loss and vertigo. Auditory Consequences • Longitudinal fractures may cause dislocation of the auditory ossicles, usually the incus, causing a conductive deafness. • Sensorineural hearing loss is associated with transverse fractures of the temporal bone, as is vertigo. Facial Nerve Involvement • Facial nerve palsy is seen in both fracture types, but is less common in longitudinal fractures, where it frequently recovers spontaneously. Vascular Considerations • Carotid artery and major sinus injuries are not directly associated with petrous temporal fractures. 72. In a patient presenting with a Le Fort fracture following facial trauma, which bone is almost invariably fractured as part of the injury pattern? A. Zygomatic arch B. Frontal sinus wall C. Nasal septum D. Pterygoid plates E. Lacrimal bone Explanation: (by Perplexity AI, generated with the OpenAI O3 model) • Le Fort fractures are defined by the involvement of the pterygoid plates—fracture lines must traverse these plates for the injury to qualify as any Le Fort type. • Their posterior location anchors the mid-face to the skull base; disruption allows displacement of the maxillary complex. • Other facial bones may be involved variably: the zygomatic arch (A) and frontal sinus wall (B) are not required components, nasal septum (C) may be spared, and the thin lacrimal bone (E) is inconsistently affected. 73. Concerning the LeFort classification of facial injuries, which one is TRUE? A. All types involve the pterygoid plates and nasal region. B. All types involve the orbital floors and zygomas. C. The Type III injury is characterized by a free-floating palate (trans-maxillary fracture). D. The type II injury is characterized by cranial-facial dissociation.
  • 228.
    EBDR Exam MCQs& Concepts September 2022 Dr. Kareem Alnakeeb 222 Source: ACR. Explanation: • In the Le Fort system every pattern (I, II and III) must include bilateral fractures of the pterygoid plates and traverse the nasal region—this unites them as mid-face fractures. • Option B is incorrect because only Le Fort II and III extend through the orbital floors and involve the zygomas; Le Fort I is limited below the nasal floor. • Option C misattributes the “free-floating palate” feature, which actually defines Le Fort I, not III. • Option D confuses Le Fort II with Le Fort III; cranio-facial (orbital-zygomatic) separation is the hallmark of Le Fort III. 74. In an infant suspected of hypertrophic pyloric stenosis, which sonographic measurement is diagnostic of the condition? A. Pyloric canal length of 19 mm B. Transverse pyloric diameter of 11 mm C. Pyloric muscle thickness of 2 mm D. Pyloric canal length of 7 mm E. Transverse pyloric diameter of 6 mm Explanation: (by Perplexity AI, generated with the OpenAI O3 model) • Hypertrophic pyloric stenosis is confirmed on ultrasound when the pyloric canal measures ≥15–17 mm in length and the muscle wall is ≥3 mm thick; a length of 19 mm therefore fulfils diagnostic criteria. • A transverse diameter of 11 mm can be seen in normal infants and is not specific. • Muscle thickness of 2 mm and canal length of 7 mm both lie within normal limits, while a diameter of 6 mm is clearly normal, making these distractors incorrect. 75. A six week old child has an ultrasound scan of the abdomen performed for non-bilious projectile vomiting. Which one of the following features would support a diagnosis of infantile pylorospasm over a diagnosis of hypertrophic pyloric stenosis? A. Pyloric muscle wall thickness of 2 mm B. Pyloric canal length of 19 mm C. Target sign D. Antral nipple sign E. Transverse pyloric diameter of 14 mm Source: Currie, Stuart, et al. SBAs for the FRCR 2A. 1st ed., Cambridge University Press, 2010. Explanation: Clinical Presentation • Hypertrophic pyloric stenosis presents between four and six weeks of life with non-bilious vomiting, typically in first-born males. Physical Examination • A palpable olive-shaped mass is a sign with reported sensitivity of up to 80%, but ultrasound is the most frequently used imaging modality. Ultrasound Characteristics • Typical ultrasound features include the target sign (central hyperechoic mucosa with surrounding hypoechoic pyloric muscle), the nipple sign (pyloric mucosa indenting the gastric antrum), pyloric canal length >16 mm, transverse pyloric diameter >13 mm and pyloric muscle wall thickness >3 mm. Differential Diagnosis • Pyloric stenosis can be difficult to differentiate radiologically from infantile pylorospasm.
  • 229.
    EBDR Exam MCQs& Concepts September 2022 Dr. Kareem Alnakeeb 223 • Typically with pylorospasm the appearances change with time, and so if the pyloric muscle thickness is measured at less than 3 mm this makes infantile pylorospasm the more likely diagnosis. 76. A 4-week-old male neonate presents with nonbilious vomiting and a hypochloraemic alkalosis. Hypertrophic pyloric stenosis is suspected and an ultrasound is performed. Which one of the following ultrasound findings would confirm the diagnosis? A. A pylorus that does not open B. Pyloric canal length of greater than 11 mm C. Pyloric muscle wall thickness of 1 mm D. Reduced gastric peristalsis E. Transverse pyloric diameter of greater than 11 mm. Source: McQueen, Andrew S., et al. Grainger & Allison’s Diagnostic Radiology: Single Best Answer MCQs. 5th ed., Churchill Livingstone, 2009. Explanation: • Ultrasound criteria include canal length >16 mm, transverse pyloric diameter >11 mm, muscle wall thickening > 2.5 mm and increased gastric motility. • A pylorus that does not open is associated with hypertrophic stenosis; however, it may also be seen in pylorospasm. 77. A 26-year-old patient with a positive B- HCG undergoes pelvic ultrasound examination. Which finding on ultrasound is most likely to indicate a nonviable pregnancy? A. The intradecidual sign B. Non-visualisation of cardiac activity when crown-rump length (CRL) is 7mm C. Visualisation of the yolk sac when the gestational sac is 8 mm D. Gestational sac present at 32 days E. Asymmetry of the echogenic ring surrounding the gestational sac at five weeks. Source: Proctor, Robin. Final FRCR Part A Modules 4-6 Single Best Answer MCQs: The SRT Collection of 600 Questions with Explanatory Answers (MasterPass). 1st ed., CRC Press, 2009. Explanation: • At 6.5 weeks the CRL is approximately 5 mm and cardiac movement can be identified. • The intradecidual sign is seen in intrauterine pregnancy. • The gestational sac can be seen at the fundus from five weeks and is surrounded by an echogenic ring which can be asymmetric. • The yolk sac is seen at approximately five to seven weeks when the gestational sac is 6-9 mm. 78. A 63 year old man with an elevated PSA is diagnosed with prostate cancer following needle biopsy. He undergoes an MRI examination to help stage his disease. Which of the following sequences would be most helpful in identifying location and local extent of the tumour? A. T1-weighted B. T1-weighted with gadolinium C. T2-weighted D. Fat-suppressed T1-weighted E. Fat-suppressed T2-weighted Explanation: (by Perplexity AI, generated with the OpenAI O3 model) • Prostate cancer shows low signal intensity against the normally high-signal peripheral zone on high- resolution T2-weighted images, allowing clear delineation of tumour focus, assessment of capsular bulge, neurovascular bundle involvement, and seminal vesicle invasion.
  • 230.
    EBDR Exam MCQs& Concepts September 2022 Dr. Kareem Alnakeeb 224 • T1-weighted sequences (with or without gadolinium) lack soft-tissue contrast for zonal anatomy; gadolinium adds little for local staging. Fat suppression on T1 or T2 degrades prostate-parenchymal contrast and is mainly used for post-biopsy haemorrhage detection or whole-body marrow imaging, not for primary lesion mapping. 79. On an erect chest radiograph, what is the approximate minimum volume of intraperitoneal free air that can usually be detected beneath the diaphragm? A. 100 ml B. 5 ml C. 250 ml D. 500 ml E. 1,000 ml Explanation: (by Perplexity AI, generated with the OpenAI O3 model) • As little as about 100 ml of intraperitoneal gas will typically outline the underside of the hemidiaphragm on an erect chest X-ray, creating the classic sub-diaphragmatic crescent. • Smaller volumes (≈5 ml) are generally below the threshold of plain-film detection and instead require more sensitive techniques such as computed tomography. • Larger volumes—250 ml or more—are obviously visible; however, waiting for that amount risks delayed diagnosis. • Therefore, 100 ml represents the accepted lower limit on standard erect radiographs, making option A correct, while options B, C, D and E are respectively too low or unnecessarily high. 80. A runner presents with pain in the third web space of the foot; ultrasound confirms a Morton neuroma. Which nerve is characteristically involved? A. Common plantar digital nerve B. Deep peroneal nerve C. Lateral plantar nerve D. Medial plantar nerve E. Sural nerve Explanation: (by Perplexity AI, generated with the OpenAI O3 model) • Morton neuroma is a perineural fibrosis affecting the common plantar digital nerve as it passes beneath the intermetatarsal ligament, most frequently between the third and fourth metatarsal heads. • The deep peroneal nerve (B) supplies the first web space on the dorsum and is not implicated. • The lateral plantar nerve (C) divides into proper and common plantar branches more laterally and does not traverse the usual site of a Morton neuroma. • The medial plantar nerve (D) courses medially and is seldom affected. • The sural nerve (E) runs along the lateral foot and is unrelated to intermetatarsal neuromas. 81. A 48-year-old man with previous pulmonary tuberculosis presents with life-threatening massive haemoptysis. Bronchoscopy localises bleeding to the right upper lobe and urgent endovascular treatment is planned. Which arterial system should be selectively catheterised and assessed first during the embolisation procedure? A. Pulmonary arteries B. Internal mammary arteries C. Superior thyroid arteries D. Bronchial arteries E. Subclavian arteries
  • 231.
    EBDR Exam MCQs& Concepts September 2022 Dr. Kareem Alnakeeb 225 Explanation: (by Perplexity AI, generated with the OpenAI O3 model) • The vast majority (≈90%) of massive haemoptysis arises from hypertrophied bronchial arteries because post-tuberculous inflammation increases their calibre and pressure; therefore, these vessels are the primary target for embolisation. • Pulmonary arterial bleeding is much less common and usually lower pressure, so pulmonary angiography is reserved for persistent haemorrhage after negative bronchial studies. • Internal mammary, superior thyroid and subclavian arteries can supply non-bronchial systemic collaterals but are secondary considerations explored only if bronchial angiography is inconclusive or bleeding recurs. 82. Where along its course is the Achilles tendon most commonly ruptured in adults presenting with an acute tear? A. 0–2 cm proximal to its calcaneal insertion B. 2–6 cm proximal to its calcaneal insertion C. At the calcaneal insertion itself D. In the mid‐substance more than 6 cm from the calcaneus E. At the musculotendinous junction with gastrocnemius Explanation: (by Perplexity AI, generated with the OpenAI O3 model) • The typical site of Achilles tendon rupture lies 2–6 cm above its calcaneal insertion, corresponding to a relatively hypovascular “watershed” zone that predisposes the tendon to degeneration and tearing. • Ruptures directly at the insertion (A) are uncommon and usually reflect enthesopathic disease; complete avulsions from bone (C) remain rare. • Tears occurring more proximally within the mid‐substance beyond 6 cm (D) or at the musculotendinous junction (E) are seen mainly in younger athletes after severe trauma but represent a minority of cases. 83. On a contrast-enhanced CT, a mantle of homogeneous soft-tissue density is seen encasing the abdominal aorta at the level of L4–L5 and medially displacing both mid-ureters, resulting in bilateral mild hydronephrosis. Which diagnosis best explains these findings? A. Abdominal aortic aneurysm with mural thrombus B. Malignant lymphoma C. Idiopathic retroperitoneal fibrosis D. Paraganglioma of the organ of Zuckerkandl E. Metastatic peri-aortic lymphadenopathy Explanation: (by Perplexity AI, generated with the OpenAI O3 model) • Idiopathic retroperitoneal fibrosis classically presents as a plaque-like soft-tissue rind that envelops the distal aorta and common iliac arteries without anterior displacement of the vessels and pulls the ureters medially, causing obstructive uropathy. • Lymphoma and metastatic nodes typically form discrete nodal masses that displace the aorta anteriorly and do not characteristically medialise the ureters. • An aortic aneurysm with thrombus demonstrates a dilated aortic lumen surrounded by thrombus rather than a periaortic mantle. • Paraganglioma appears as an avidly enhancing focal mass at the aortic bifurcation, not a longitudinal plaque.
  • 232.
    EBDR Exam MCQs& Concepts September 2022 Dr. Kareem Alnakeeb 226 84. A CT scan shows a heterogeneously enhancing liver mass with a tumour thrombus extending from the hepatic vein into the inferior vena cava and up to the right atrium. Which primary malignancy most commonly produces this pattern of intravascular spread? A. Pancreatic ductal adenocarcinoma B. Hepatocellular carcinoma C. Intrahepatic cholangiocarcinoma D. Adrenocortical carcinoma E. Renal cell carcinoma Explanation: (by Perplexity AI, generated with the OpenAI O3 model) • Hepatocellular carcinoma (HCC) has a well-recognised propensity to invade hepatic veins, progressing into the inferior vena cava and occasionally the right atrium; this occurs in 1.4–4.9% of HCC cases and is a classic imaging pattern. • Pancreatic adenocarcinoma and cholangiocarcinoma more often involve the portal or spleno- mesenteric veins, while direct caval thrombus reaching the atrium is exceptional. • Renal and adrenocortical carcinomas can extend up the IVC but typically arise below the liver and do not involve hepatic veins. Therefore, the described cranio-caudal tumour thrombus originating in a hepatic lesion most strongly indicates HCC. 85. On ultrasound, which description best matches the appearance of the central scar within a focal nodular hyperplasia (FNH) lesion? A. Markedly hyperechoic linear focus B. Homogeneously isoechoic to surrounding liver C. Irregular anechoic cavity with posterior enhancement D. Mildly hypoechoic focus compared with adjacent parenchyma E. Prominent calcified echogenic focus with shadowing Explanation: (by Perplexity AI, generated with the OpenAI O3 model) • FNH often contains a central fibrous scar. On grayscale ultrasound the scar is usually mildly hypoechoic relative to the surrounding liver, reflecting its fibrous composition and lower cellularity, while the remainder of the lesion may be isoechoic or slightly hyperechoic. • It is seldom brightly echogenic (ruling out A) and not typically the same echogenicity as liver (B). • It is solid rather than cystic, so an anechoic cavity with through-transmission (C) is incorrect. • Calcification and acoustic shadowing are rare in FNH scars, eliminating option E. 86. A 3-year-old boy presents with abdominal distension and raised serum α-fetoprotein. Contrast- enhanced CT of the liver shows a solitary right-lobe mass with heterogeneous arterial enhancement and patchy calcification. What is the most likely diagnosis? A. Focal nodular hyperplasia B. Hepatoblastoma C. Hepatocellular carcinoma D. Mesenchymal hamartoma E. Metastatic neuroblastoma Explanation: (by Perplexity AI, generated with the OpenAI O3 model) • Hepatoblastoma is the commonest primary malignant liver tumour in children under 5 years. It typically manifests as a solitary right-lobe mass showing heterogeneous arterial enhancement due to mixed epithelial and mesenchymal elements, often containing calcifications and markedly elevating α-fetoprotein.
  • 233.
    EBDR Exam MCQs& Concepts September 2022 Dr. Kareem Alnakeeb 227 • Hepatocellular carcinoma is rarer at this age and usually arises in cirrhotic livers. • Focal nodular hyperplasia is exceptional in toddlers and shows a central scar rather than calcification. • Mesenchymal hamartoma occurs in infants, has cystic components and does not elevate α- fetoprotein. • Metastatic neuroblastoma may calcify but classically presents with multiple subcapsular lesions and normal α-fetoprotein. 87. A 25-year-old woman on long-term combined oral contraceptive pills presents with acute right-upper- quadrant pain. CT shows a solitary, well-circumscribed 8 cm hepatic mass containing areas of high attenuation consistent with intratumoural haemorrhage. Which is the most likely diagnosis? A. Focal nodular hyperplasia B. Hepatocellular carcinoma C. Hepatic adenoma D. Cavernous haemangioma E. Hepatic metastasis Explanation: (by Perplexity AI, generated with the OpenAI O3 model) • Oral-contraceptive–related hepatic adenomas typically occur in young women and are prone to haemorrhage, so a painful, hyperattenuating lesion on CT strongly favours this diagnosis. • Focal nodular hyperplasia usually has a central scar and rarely bleeds. • Hepatocellular carcinoma is uncommon in a healthy young woman without cirrhosis and often shows arterial wash-out rather than isolated haemorrhage. • Cavernous haemangiomas demonstrate peripheral nodular enhancement with centripetal fill-in and seldom rupture. • Metastases are unlikely when there is a single lesion with haemorrhage and no known primary. 88. On MRCP, which imaging sign best differentiates a congenital choledochal cyst from a common bile duct dilatation caused by downstream obstruction? A. Abrupt irregular cutoff at the distal common bile duct B. Beaded appearance of the intra-hepatic ducts C. Smooth, gradual tapering between dilated and normal duct calibre D. Mural nodularity within the dilated segment E. Multiple filling defects within the dilated duct Explanation: (by Perplexity AI, generated with the OpenAI O3 model) • Choledochal cysts are true congenital dilatations of the extra-hepatic bile duct. They characteristically merge with the normal calibre duct distally via a smooth, gradual (conical) taper, reflecting unobstructed bile flow. • In contrast, malignant or calculous obstruction produces an abrupt irregular cutoff (option A) or meniscoid/step-off appearance, not a taper. • Sclerosing cholangitis gives a beaded contour (option B). Mural nodularity (option D) suggests cholangiocarcinoma. • Multiple intraductal filling defects (option E) are typical of choledocholithiasis or sludge rather than a congenital cyst.
  • 234.
    EBDR Exam MCQs& Concepts September 2022 Dr. Kareem Alnakeeb 228 89. A newborn boy presents with a markedly distended, wrinkled abdomen, bilateral non-palpable testes and ultrasound evidence of severe bilateral hydroureteronephrosis with a massively dilated bladder. Which diagnosis best explains this triad of findings? A. Abdominal compartment syndrome B. Necrotising enterocolitis C. Posterior urethral valves D. Prune-belly (Eagle-Barrett) syndrome E. Pseudo-Prune-belly syndrome Explanation: (by Perplexity AI, generated with the OpenAI O3 model) • Prune-belly syndrome is defined by the classic triad of deficient anterior abdominal wall musculature (producing a lax, “prune-like” abdomen), profound urinary tract dilatation/obstruction and bilateral cryptorchidism. The combination of wrinkled abdominal skin, megacystis with hydroureteronephrosis and undescended testes therefore points directly to this diagnosis. • Posterior urethral valves (C) cause bladder outlet obstruction but do not account for absent abdominal muscles or cryptorchidism. • Abdominal compartment syndrome (A) produces a tense, not lax, abdomen and lacks urogenital anomalies. • Necrotising enterocolitis (B) manifests with pneumatosis and sepsis rather than urinary findings. • Pseudo-Prune-belly syndrome (E) shows partial or unilateral abdominal wall deficiency and usually lacks the full triad. 90. A neonate fails a nasogastric tube-passing test; the tube coils in the upper mediastinum on radiograph and a gas-filled stomach is visible. What is the most likely diagnosis? A. Pure oesophageal atresia without fistula B. Oesophageal atresia with distal tracheo-oesophageal fistula C. Oesophageal atresia with proximal tracheo-oesophageal fistula D. H-type tracheo-oesophageal fistula without atresia E. Congenital oesophageal stenosis Explanation: (by Perplexity AI, generated with the OpenAI O3 model) • Coiling of the tube in a blind-ending upper pouch indicates oesophageal atresia. The presence of gastric bubble signifies air passage into the stomach via a distal tracheo-oesophageal fistula, making the classic type C lesion (atresia with distal fistula) most likely. • Pure atresia (A) shows no gas in the abdomen. • A proximal fistula (C) would not aerate the stomach. • An isolated H-type fistula (D) has no atresia, so the tube reaches the stomach. • Congenital stenosis (E) allows tube passage, though with resistance. 91. An 11-day-old jaundiced infant has persistent pale stools and conjugated hyperbilirubinaemia. Ultrasound shows an absent gallbladder and a 4 mm echogenic “triangular cord” anterior to the portal vein bifurcation. What is the most likely diagnosis? A. Neonatal hepatitis B. Choledochal cyst C. Biliary atresia D. Alagille syndrome E. Inspissated bile syndrome
  • 235.
    EBDR Exam MCQs& Concepts September 2022 Dr. Kareem Alnakeeb 229 Explanation: (by Perplexity AI, generated with the OpenAI O3 model) • The echogenic “triangular cord” sign represents fibrotic residual bile duct tissue at the porta hepatis and, together with a non-visualised or abnormal gallbladder, is highly specific for biliary atresia. • Early recognition is critical because Kasai porto-enterostomy before 6 weeks markedly improves outcome. • Neonatal hepatitis, Alagille syndrome and inspissated bile may produce cholestasis but lack the triangular cord and typically show a normal-sized gallbladder. • Choledochal cysts present with cystic biliary dilatation, not an absent gallbladder with fibrotic remnant. 92. A 6-week-old boy presents with persistent jaundice and hepatosplenomegaly. Ultrasound shows a normal-appearing gallbladder and a triangular fibrotic echogenic band at the porta hepatis (the “triangular cord” sign). Which is the most likely underlying diagnosis? A. Alagille syndrome B. Biliary atresia C. Neonatal hepatitis secondary to CMV infection D. Choledochal cyst (type I) E. Alpha-1-antitrypsin deficiency Explanation: (by Perplexity AI, generated with the OpenAI O3 model) • The echogenic “triangular cord” anterior to the portal vein represents fibrotic remnants of the extra- hepatic bile duct and is highly specific for biliary atresia. • Although some infants with biliary atresia can have a small or even normal-looking gallbladder, the porta hepatis fibrotic band clinches the diagnosis. • Alagille syndrome may show a small or absent gallbladder but lacks the triangular cord and is usually accompanied by characteristic facies and cardiac anomalies. • Neonatal hepatitis (including CMV) and alpha-1-antitrypsin deficiency cause diffuse parenchymal liver disease without an echogenic hilar band. • A choledochal cyst would demonstrate a cystic dilatation of the common bile duct rather than a solid fibrotic band. 93. In an adult patient with a fusiform dilatation of the extra-hepatic common bile duct on MRCP, which Todani classification type of choledochal cyst is most likely? A. Type IA B. Type IB C. Type IC D. Type II E. Type III Explanation: (by Perplexity AI, generated with the OpenAI O3 model) • Todani type IC describes a long, smooth, fusiform dilatation that involves the entire extra-hepatic bile duct, often extending up to the common hepatic duct; this imaging pattern fits the vignette. o Type IA appears as a saccular (cystic) dilatation of the CBD, not fusiform. o Type IB is a segmental, focal outpouching of the distal CBD. o Type II is a true diverticulum of the extra-hepatic duct. o Type III (choledochocele) is located within the duodenal wall. Hence these alternatives do not match the described fusiform morphology.
  • 236.
    EBDR Exam MCQs& Concepts September 2022 Dr. Kareem Alnakeeb 230 94. On MRI, a cystic dilatation of the segment 2/3 intra-hepatic ducts shows a tiny enhancing “central dot” with smooth tapering of the extra-hepatic bile duct. Which diagnosis best fits these imaging findings? A. Primary sclerosing cholangitis B. Recurrent pyogenic cholangitis C. Choledochal cyst type I D. Caroli disease E. Klatskin (hilar) cholangiocarcinoma Explanation: (by Perplexity AI, generated with the OpenAI O3 model) • The “central dot sign” represents an enhancing portal vein radicle within a dilated intra-hepatic duct and, along with smooth, unobstructed extra-hepatic ducts, is characteristic of Caroli disease—a congenital ductal-plate malformation causing saccular or beaded ectasia of intra-hepatic bile ducts. • Primary sclerosing cholangitis (A) and recurrent pyogenic cholangitis (B) produce multifocal strictures without the central dot. • Choledochal cyst type I (C) causes fusiform dilatation of the common bile duct, not intra-hepatic cystic ectasia with a dot. • A Klatskin tumour (E) gives irregular biliary obstruction and shouldering, not smooth tapering or a central dot. 95. A 27-year-old woman is incidentally found to have a solitary 3 cm hepatic lesion on ultrasound. Triple- phase CT shows intense homogeneous arterial enhancement becoming isodense to liver on portal venous phase with a central scar that demonstrates delayed enhancement. Which is the most likely diagnosis? A. Hepatocellular adenoma B. Fibrolamellar hepatocellular carcinoma C. Focal nodular hyperplasia D. Hypervascular metastasis E. Cavernous haemangioma Explanation: (by Perplexity AI, generated with the OpenAI O3 model) • Focal nodular hyperplasia classically presents in young women as a well-circumscribed lesion that shows brisk, uniform arterial phase enhancement and remains isoattenuating on the portal venous phase; a central scar with delayed contrast uptake is typical. • Hepatocellular adenoma (A) enhances heterogeneously and often contains fat or haemorrhage with no delayed scar enhancement. • Fibrolamellar HCC (B) may have a scar but usually occurs in adolescents/young adults of either sex, shows heterogeneous signal, and the scar does not enhance early or may calcify. • Hypervascular metastases (D) are usually multiple and demonstrate wash-out on portal/delayed phases. • Cavernous haemangioma (E) shows peripheral nodular discontinuous enhancement with progressive centripetal fill-in, not a central scar.
  • 237.
    EBDR Exam MCQs& Concepts September 2022 Dr. Kareem Alnakeeb 231 96. On a contrast-enhanced CT performed in the portal-venous phase, which pancreatic tumour classically appears as a focal low-attenuation mass within the pancreatic head causing upstream duct dilatation? A. Pancreatic neuroendocrine tumour B. Solid pseudopapillary neoplasm C. Pancreatic ductal adenocarcinoma D. Serous cystadenoma E. Intraductal papillary mucinous neoplasm Explanation: (by Perplexity AI, generated with the OpenAI O3 model) • Pancreatic ductal adenocarcinoma typically presents on portal-venous phase CT as a hypovascular (low-attenuation) mass relative to the enhancing pancreatic parenchyma, most frequently in the head, and often results in “double-duct” sign from obstruction of the pancreatic and common bile ducts, making C correct. • Neuroendocrine tumours (A) are usually hypervascular and enhance avidly, not low attenuation. • Solid pseudopapillary neoplasms (B) show mixed cystic-solid components with variable enhancement, often in young women. • Serous cystadenomas (D) are microcystic, giving a honeycomb appearance rather than solid low attenuation. • Intraductal papillary mucinous neoplasms (E) are cystic lesions arising from or communicating with the pancreatic ducts and are not solid hypovascular masses. 97. A full-term neonate presents with a midline abdominal wall defect covered by a membrane at the umbilicus. Which associated abnormality is most frequently seen with this condition? A. Meckel diverticulum B. Malrotation C. Trisomy 18 D. Duodenal atresia E. Hirschsprung disease Explanation: (by Perplexity AI, generated with the OpenAI O3 model) • Omphalocele is a midline, membrane-covered herniation at the umbilical ring. • Roughly 50% of affected infants have additional anomalies, and chromosomal disorders—especially trisomies 13 and 18—are the single commonest group. • Trisomy 18 is classically quoted as the most frequent association, often co-existing with cardiac and other systemic defects. • Gastrointestinal malformations such as malrotation, Meckel diverticulum, duodenal atresia, and Hirschsprung disease do occur but each individually accounts for a much smaller proportion of cases, making them less likely than a chromosomal trisomy in an unknown neonate with omphalocele. 98. On CT adrenal wash-out studies, which statement regarding typical lipid-rich adrenal cortical adenomas is MOST accurate? A. They are less common than malignant adrenal tumours B. More than 90% of adenomas still measure >30 HU at 10-min delayed imaging C. Around 70% demonstrate >60% absolute contrast wash-out by 10 min D. Fewer than 20% contain intracellular lipid giving ≤10 HU on non-contrast CT E. Most show progressive enhancement rather than wash-out on delayed scans
  • 238.
    EBDR Exam MCQs& Concepts September 2022 Dr. Kareem Alnakeeb 232 Explanation: (by Perplexity AI, generated with the OpenAI O3 model) • Adrenal cortical adenomas are common incidental findings and classically enhance rapidly then lose contrast quickly. • Approximately two-thirds to three-quarters achieve an absolute contrast wash-out ≥60% within 10– 15 min, a highly specific benign feature. • Malignant lesions usually wash-out much more slowly. • Adenomas are far more common than primary or secondary malignant adrenal masses, making option A incorrect. • Over 80% of adenomas fall to <30 HU by 10 min, not remain above it, so option B is false. • About 70% are lipid-rich with ≤10 HU on unenhanced CT, not fewer than 20%, invalidating option D. • Adenomas do not show progressive enhancement; they wash-out, so option E is wrong. 99. On grayscale ultrasound, which description most closely matches the typical appearance of a hepatocellular carcinoma (HCC) in a cirrhotic liver? A. Small, well-defined, uniformly hyperechoic nodule B. Ill-defined, heterogeneous, predominantly hypoechoic mass C. Round, sharply marginated, anechoic lesion with posterior enhancement D. Homogeneous, isoechoic lesion containing a central stellate scar E. Well-circumscribed, heterogeneously echogenic mass with a hypoechoic rim seen only in non- cirrhotic young women Explanation: (by Perplexity AI, generated with the OpenAI O3 model) • HCCs usually arise in a background of cirrhosis and, on standard B-mode ultrasound, classically appear as ill-defined, heterogeneous, predominantly hypoechoic masses; larger lesions often show mixed echogenicity because of necrosis or hemorrhage. • Option A describes a small hyperechoic nodule more typical of a hemangioma. • Option C fits a simple cyst. • Option D suggests focal nodular hyperplasia, characterised by a central scar. • Option E corresponds to hepatic adenoma, typically occurring in young women on the oral contraceptive pill, appearing as a well-circumscribed heterogeneous lesion that may have a surrounding hypoechoic halo but generally in a non-cirrhotic liver. 100. In a neonate with bilateral hydronephrosis, ultrasound shows marked tapering of the distal 1–2 cm of each ureter just before the vesicoureteric junction with proximal ureteric dilatation; what is the most likely underlying diagnosis? A. Vesicoureteric reflux B. Posterior urethral valves C. Primary obstructive megaureter D. Ureterocele within the bladder E. Prune-belly syndrome Explanation: (by Perplexity AI, generated with the OpenAI O3 model) • Primary obstructive megaureter is characterised by an intrinsically aperistaltic or stenotic segment in the distal 1–2 cm of the ureter immediately proximal to the bladder, producing a smooth tapered lower third and upstream dilatation. • Vesicoureteric reflux (A) shows refluxing contrast without a true distal taper and often has a non- dilated ureter when the bladder is empty.
  • 239.
    EBDR Exam MCQs& Concepts September 2022 Dr. Kareem Alnakeeb 233 • Posterior urethral valves (B) cause bladder-outlet obstruction with a dilated, trabeculated bladder and dilated posterior urethra rather than a focal distal ureteric narrowing. • A ureterocele (D) appears as a cystic filling defect inside the bladder, not as tapering of the distal ureter. • Prune-belly syndrome (E) presents with flaccid, massively dilated ureters and bladder but lacks the short, narrowed intramural segment. 101. A 52-year-old woman presents with progressive exertional breathlessness and intermittent dizziness. Transthoracic echocardiography demonstrates a mobile, echogenic mass attached by a narrow stalk to the interatrial septum, prolapsing through the mitral valve during diastole. What is the most likely diagnosis? A. Papillary fibroelastoma B. Cardiac myxoma C. Thrombus in transit D. Rhabdomyoma E. Metastatic melanoma deposit Explanation: (by Perplexity AI, generated with the OpenAI O3 model) • Cardiac myxomas are the commonest primary cardiac tumours in adults and typically arise from the fossa ovalis region of the interatrial septum, attached by a pedicle; their mobility can cause obstruction of the mitral inflow, explaining positional dyspnoea and syncope. • Papillary fibroelastomas usually originate from valvular endocardium rather than the septum. • An intracardiac thrombus lacks a discrete stalk and is less likely to be highly mobile on a narrow pedicle. • Rhabdomyomas predominate in infants and involve ventricular myocardium. • Metastatic melanoma deposits are seldom pedunculated and usually present as multiple masses or pericardial involvement. 102. A 16-year-old boy with lifelong short stature presents for hand radiographs, which show bilateral shortening of the fourth and fifth metacarpals. Routine biochemistry reveals hypocalcaemia, hyperphosphataemia and markedly elevated parathyroid hormone levels. Which single diagnosis best explains this combination of skeletal and biochemical findings? A. Idiopathic hypoparathyroidism B. Pseudohypoparathyroidism (Albright hereditary osteodystrophy) C. Primary hyperparathyroidism D. Renal osteodystrophy secondary to chronic kidney disease E. Vitamin-D–dependent rickets Explanation: (by Perplexity AI, generated with the OpenAI O3 model) • Shortened fourth and fifth metacarpals together with short stature and other skeletal stigmata constitute the Albright hereditary osteodystrophy phenotype. • When this phenotype co-exists with high PTH, low calcium and high phosphate, the underlying problem is end-organ resistance to PTH—pseudohypoparathyroidism type 1a. • Idiopathic hypoparathyroidism (A) shows low or absent PTH, not elevated. • Primary hyperparathyroidism (C) produces high calcium and low phosphate. • Renal osteodystrophy (D) causes secondary hyperparathyroidism but typically shows high phosphate with normal metacarpals and no Albright features.
  • 240.
    EBDR Exam MCQs& Concepts September 2022 Dr. Kareem Alnakeeb 234 • Vitamin-D–dependent rickets (E) presents with rachitic bone changes and low–normal phosphate, not the classic metacarpal shortening. 103. In a middle-aged patient with a vividly enhancing extra-axial mass abutting the falx on contrast MRI (no image shown), which imaging feature most reliably favours a haemangiopericytoma over a meningioma? A. Presence of a dural tail B. Hyperostotic bone reaction of the adjacent calvarium C. Irregular bone erosion of the inner table D. Calcification within the lesion on CT E. Broad-based dural attachment Explanation: (by Perplexity AI, generated with the OpenAI O3 model) • Haemangiopericytomas are aggressive, vascular tumours that tend to erode adjacent skull with scalloped or irregular lytic defects, whereas meningiomas more often incite hyperostosis or leave the bone intact. • A dural tail and broad dural base are classic but non-specific signs frequently seen in meningiomas; they can be absent in haemangiopericytoma. • Intratumoural calcification is common in meningioma but rare in haemangiopericytoma. Therefore, irregular bone erosion is the best discriminator in this scenario. 104. Which fracture most frequently leads to avascular necrosis of the femoral head? A. Intertrochanteric femur fracture B. Subtrochanteric femur fracture C. Femoral neck (intracapsular) fracture D. Shaft of femur fracture E. Greater trochanter avulsion fracture Explanation: (by Perplexity AI, generated with the OpenAI O3 model) • A femoral neck (intracapsular) fracture disrupts the retinacular vessels within the capsule, making osteonecrosis of the femoral head the commonest vascular complication. • Intertrochanteric and subtrochanteric fractures lie extracapsular, preserving the main blood supply, so osteonecrosis is rare. • Diaphyseal (shaft) and greater-trochanter avulsion fractures are even further from the femoral head and do not compromise its arterial inflow, hence they seldom cause avascular necrosis. 105. Which of the following conditions is a recognised cause of a high-probability V/Q mismatch on scintigraphy that proves false-positive for pulmonary embolism at CT pulmonary angiography? A. Pulmonary veno-occlusive disease B. Asthma C. Bacterial pneumonia D. Hypersensitivity pneumonitis E. Pulmonary Langerhans cell histiocytosis Explanation: (by Perplexity AI, generated with the OpenAI O3 model) • Pulmonary veno-occlusive disease narrows medium- and large-sized pulmonary veins. Downstream venous obstruction reduces regional blood flow, so technetium-labelled macro-aggregated albumin fails to lodge in affected segments while ventilation remains normal. The scan therefore shows multiple mismatched segmental perfusion defects that mimic thromboembolic occlusion, yet CT or
  • 241.
    EBDR Exam MCQs& Concepts September 2022 Dr. Kareem Alnakeeb 235 conventional angiography demonstrates patent pulmonary arteries, confirming a false-positive result. • Asthma and bacterial pneumonia usually produce matched or reverse-matched defects because airway or parenchymal disease affects both ventilation and perfusion. • Hypersensitivity pneumonitis and pulmonary Langerhans cell histiocytosis typically cause diffuse, mottled, or matched abnormalities rather than discrete segmental mismatches, so they rarely give a high-probability pattern. 106. A 52-year-old man presents with progressive exertional dyspnoea. Chest radiograph shows a large homogenous opacity occupying most of the right hemithorax. Contrast-enhanced CT demonstrates a well-circumscribed, lobulated, enhancing mass arising from the parietal pleura and displacing adjacent lung. There is no chest wall invasion or effusion. Which is the most likely diagnosis? A. Malignant mesothelioma B. Metastatic sarcoma C. Solitary fibrous tumour D. Liposarcoma E. Pleural lymphoma Explanation: (by Perplexity AI, generated with the OpenAI O3 model) • Solitary fibrous tumours typically appear as well-defined, lobulated, contrast-enhancing pleural masses that displace but rarely invade lung or chest wall, and they often present without pleural effusion, matching the described CT findings. • Malignant mesothelioma (A) usually shows diffuse, nodular pleural thickening that encases the lung and is frequently associated with effusion. • Metastatic sarcoma (B) and pleural lymphoma (E) more often produce multiple or infiltrative pleural deposits rather than a solitary mass. • Liposarcoma (D) characteristically contains areas of macroscopic fat on CT, which are not reported here. 107. A 52-year-old lifelong non-smoker presents with a new 2.8 cm spiculated nodule in the right upper lobe on CT thorax without significant mediastinal lymphadenopathy. Which histological subtype of primary lung cancer is most likely? A. Large-cell carcinoma B. Squamous cell carcinoma C. Small-cell carcinoma D. Adenocarcinoma E. Typical carcinoid Explanation: (by Perplexity AI, generated with the OpenAI O3 model) • Adenocarcinoma is the commonest primary lung cancer in never-smokers and typically arises peripherally in the lung parenchyma, often appearing as a solitary spiculated nodule on CT. • Squamous cell and small-cell carcinomas have the strongest association with tobacco exposure and usually present as central or hilar masses rather than peripheral nodules. • Large-cell carcinoma is less common overall and still linked to smoking. • Typical carcinoid tumours are neuroendocrine lesions that more often occur centrally within bronchi and tend to be smaller, well-defined masses rather than spiculated nodules.
  • 242.
    EBDR Exam MCQs& Concepts September 2022 Dr. Kareem Alnakeeb 236 108. Which of the following ultrasound criteria is most appropriate for diagnosing hypertrophic pyloric stenosis in a premature neonate presenting with non-bilious vomiting? A. Muscle thickness ≥ 2 mm B. Muscle thickness ≥ 3 mm C. Muscle length ≥ 10 mm D. Muscle length ≥ 16 mm E. Pyloric canal diameter ≥ 10 mm Explanation: (by Perplexity AI, generated with the OpenAI O3 model) • In preterm infants the pyloric muscle is smaller than in term babies, so lower sonographic cut-offs are used. A muscle thickness of 3 mm or more is widely accepted as the most reliable indicator of hypertrophic pyloric stenosis in this group, even when muscle length remains below the classic 17 mm seen in term infants. • Thickness ≥2 mm (A) lacks specificity and overlaps with pylorospasm. • Muscle length thresholds of 10 mm (C) and 16 mm (D) are too short; length usually enlarges later and can stay normal in premature babies. • Canal diameter ≥10 mm (E) is not a standard metric for diagnosis. 109. In a 25-year-old man with chronic liver disease, MRI brain shows bilateral low-signal intensity confined to the globus pallidus on T2-weighted and susceptibility sequences. Which trace‐metal deposition disorder most likely explains this finding? A. Wilson disease B. Manganese toxicity C. Chronic lead exposure D. Pantothenate-kinase-associated neurodegeneration (PKAN) E. Fahr disease Explanation: (by Perplexity AI, generated with the OpenAI O3 model) • The globus pallidus is highly sensitive to paramagnetic metals. Excess manganese—typically from portal-systemic shunting or prolonged parenteral nutrition—accumulates preferentially in the pallida, causing marked T2 and T2* hypointensity with corresponding T1 hyperintensity. • In Wilson disease copper deposition gives mixed or high T2 signal, not uniform low signal. • Lead and Fahr disease produce calcification leading to CT hyperdensity and signal void on all MRI sequences rather than selective T2 hypointensity. • PKAN shows the classic “eye-of-the-tiger” sign: central T2 hyperintensity surrounded by a hypointense rim, different from the homogeneous low signal seen here. 110. Hallervorden–Spatz disease (now termed neurodegeneration with brain iron accumulation type 1) is characterised pathologically by which abnormal cerebral deposition? A. Aluminium B. Copper C. Iron D. Manganese E. Magnesium Explanation: (by Perplexity AI, generated with the OpenAI O3 model) • Hallervorden–Spatz disease shows excessive iron deposition, especially in the globus pallidus and substantia nigra, producing the characteristic “eye-of-the-tiger” MR appearance.
  • 243.
    EBDR Exam MCQs& Concepts September 2022 Dr. Kareem Alnakeeb 237 • Copper accumulation typifies Wilson disease, while manganese deposition is seen in chronic liver failure or parenteral nutrition. • Aluminium and magnesium are not associated with this basal-ganglia neurodegeneration pattern. 111. A 60-year-old man presents with history of chronic cough. The chest radiograph shows a 5 cm subpleural mass in the right lower lobe. There is a curvilinear opacity from the lower pole of the mass and the mass courses towards the hilum. CT confirms the mass lesion and demonstrates the bronchovascular bundles converging into the mass in a curvilinear fashion. In addition, there are multiple pleural plaques but no lymphadenopathy. The most likely diagnosis is? A. Bronchogenic carcinoma B. Round atelectasis C. Large parenchymal metastasis D. Lymphoma E. Arteriovenous malformation Source: Gupta, Chaitanya. 300 Single Best Answers for the Final FRCR Part A. 1st ed., Jaypee UK, 2010. Explanation: • Also called ‘folded lung’ or ‘asbestos pseudotumour’. The lesion forms acute angles with the pleura indicating its parenchymal location. • Pleural thickening is usually an associated finding. • It usually affects the lower lobes and there is volume loss. • The characteristic sign of round atelectasis is the ‘comet tail‘ sign. • As the lung collapses, the bronchovascular bundle is pulled into the region. • As they reach the mass they diverge and arch around the surface to merge with the inferior pole of the mass. This is typically well demonstrated on CT. 112. A 58-year-old man has an incidental, well-circumscribed 2.3 cm solitary pulmonary nodule in the right mid-zone on chest radiograph. CT confirms a peripheral lesion with internal fat and coarse “popcorn” calcification, but no contrast enhancement. What is the most likely diagnosis? A. Granulomatous tuberculoma B. Peripheral carcinoid tumour C. Peripheral pulmonary hamartoma D. Sclerosing haemangioma (pneumocytoma) E. Metastatic osteogenic sarcoma deposit Explanation: (by Perplexity AI, generated with the OpenAI O3 model) • Intralesional fat together with chunky “popcorn” calcification in a small, non-enhancing, well- defined peripheral nodule is classic for a pulmonary hamartoma, the commonest benign lung neoplasm. • Tuberculomas may calcify but do not contain macroscopic fat and usually enhance. • Carcinoid tumours enhance strongly and rarely show fat or coarse calcification. • Sclerosing haemangiomas can calcify lightly but typically lack fat and often enhance; 18 F-FDG uptake tends to be higher than in hamartoma. • Metastatic osteogenic sarcoma deposits calcify or ossify but internal fat is not a feature and they often enhance.
  • 244.
    EBDR Exam MCQs& Concepts September 2022 Dr. Kareem Alnakeeb 238 113. A 10-year-old girl presents with intermittent limb pain and low-grade fevers; radiographs show bilateral symmetrical metaphyseal sclerosis in the femora and tibiae without periosteal reaction, and laboratory cultures are repeatedly negative. Which condition best explains these findings? A. Chronic recurrent multifocal osteomyelitis B. Lead deposition in growing bone C. Acute bacterial osteomyelitis D. Ewing sarcoma E. Rickets Explanation: (by Perplexity AI, generated with the OpenAI O3 model) • Chronic recurrent multifocal osteomyelitis (CRMO) is a sterile autoinflammatory bone disorder of childhood causing episodic limb pain, low-grade fevers and multifocal symmetrical metaphyseal sclerosis on radiographs; cultures remain negative and there is no periosteal reaction, matching the vignette. • Lead deposition produces dense sclerotic metaphyseal lines (“lead lines”) but lacks pain and fevers. • Acute bacterial osteomyelitis is usually unilateral, lytic in early stages and culture-positive. • Ewing sarcoma presents with an aggressive permeative pattern and layered periosteal reaction. • Rickets causes metaphyseal cupping and fraying, not sclerosis. 114. Which imaging investigation most reliably detects a suspected pituitary micro-adenoma in a patient with Cushing’s disease? A. High-resolution CT sella with contrast B. Dynamic gadolinium-enhanced MRI pituitary protocol C. Standard non-contrast MRI brain D. Inferior petrosal sinus sampling with venography E. 18 F-FDG PET/CT of the skull base Explanation: (by Perplexity AI, generated with the OpenAI O3 model) • Dynamic contrast-enhanced MRI acquires rapid sequential images during and immediately after gadolinium administration, highlighting the normal gland’s early intense enhancement while the adenoma remains relatively hypo-enhancing; this maximises lesion-to-gland contrast and achieves sensitivities around 90% for micro-adenomas. • Conventional non-contrast MRI (C) and routine contrasted CT (A) miss many lesions <5 mm. • Inferior petrosal sinus sampling (D) is invasive and used when MRI is negative to confirm a central ACTH source, not as the primary imaging test. • FDG PET/CT (E) lacks spatial resolution for millimetric sellar lesions and is reserved for atypical or metastatic disease work-up. 115. A 28-year-old motorcyclist presents 3 weeks after a closed head injury with progressive orbital pain, pulsatile proptosis and a “whooshing” bruit over the right eye. CT angiography shows an enlarged right superior ophthalmic vein and early cavernous sinus opacification. What is the most likely underlying lesion? A. Dural arteriovenous malformation of the transverse sinus B. Direct carotid–cavernous fistula (Barrow type A) C. Indirect dural carotid–cavernous fistula (Barrow type D) D. Cavernous sinus thrombosis E. Traumatic pseudo-aneurysm of the middle meningeal artery
  • 245.
    EBDR Exam MCQs& Concepts September 2022 Dr. Kareem Alnakeeb 239 Explanation: (by Perplexity AI, generated with the OpenAI O3 model) • Head trauma causing a tear in the cavernous segment of the internal carotid artery produces a high- flow direct carotid–cavernous fistula (type A). The arterialised cavernous sinus drains anteriorly, dilating the superior ophthalmic vein and giving the classic triad of pulsatile proptosis, bruit and ocular pain. • Indirect dural fistulas (C) are usually low-flow, often spontaneous and present more insidiously. • Cavernous sinus thrombosis (D) causes chemosis, ophthalmoplegia and fever rather than pulsatile proptosis. • A transverse sinus malformation (A) would not enlarge the superior ophthalmic vein. • A middle meningeal pseudo-aneurysm (E) lies extracavernously and typically manifests with epidural haematoma or scalp bruit, not superior ophthalmic vein dilatation. 116. A preterm neonate on positive-pressure ventilation develops progressive respiratory distress and a chest radiograph (shown) demonstrates diffuse bilateral granular opacities with air bronchograms. Which condition best explains this “wet lung” appearance? A. Meconium aspiration B. Transient tachypnoea of the newborn C. Neonatal pneumonia D. Hyaline membrane disease E. Congenital lobar emphysema Explanation: (by Perplexity AI, generated with the OpenAI O3 model) • Transient tachypnoea of the newborn (TTN) is caused by delayed clearance of fetal lung fluid, giving a “wet lung” film with diffuse hazy opacities, fluid in fissures and prominent vascular markings. It typically affects term or near-term infants after Caesarean delivery and resolves within 48–72 hours. • Hyaline membrane disease (D) also shows diffuse granular opacities but occurs in markedly preterm infants and is associated with low lung volumes. • Meconium aspiration (A) produces patchy coarse infiltrates and hyperinflation, not uniform haziness. • Neonatal pneumonia (C) can mimic TTN yet usually shows asymmetric consolidation and pleural effusion. • Congenital lobar emphysema (E) presents with lobar overinflation rather than diffuse opacification. 117. Progressive massive fibrosis is most characteristic of which occupational lung disease involving upper- lobe conglomerate masses sometimes surrounded by ground-glass change? A. Asbestosis B. Coal workers’pneumoconiosis C. Chronic hypersensitivity pneumonitis D. Idiopathic pulmonary fibrosis E. Pulmonary Langerhans cell histiocytosis Explanation: (by Perplexity AI, generated with the OpenAI O3 model) • Progressive massive fibrosis (PMF) refers to coalescent fibrotic masses—classically bilateral and upper-lobe predominant—arising as the advanced form of coal workers’ pneumoconiosis. CT shows large irregular opacities with strand-like arms and may demonstrate adjacent ground-glass opacity from dust-related alveolitis. • Asbestosis (A) usually produces lower-zone subpleural fibrosis and pleural plaques, not conglomerate upper-lobe masses.
  • 246.
    EBDR Exam MCQs& Concepts September 2022 Dr. Kareem Alnakeeb 240 • Chronic hypersensitivity pneumonitis (C) can mimic fibrotic patterns but lacks the signature mass- like lesions of PMF. • Idiopathic pulmonary fibrosis (D) causes a basal, peripheral usual interstitial pneumonia pattern. • Pulmonary Langerhans cell histiocytosis (E) affects upper lobes but shows nodules and cysts rather than confluent fibrotic masses. 118. A 35-year-old man with progressive dyspnoea and cough undergoes high-resolution CT of the chest, which shows bilateral perihilar “crazy-paving”—ground-glass opacification with superimposed thickened inter- and intralobular septa. What is the most likely underlying diagnosis? A. Pneumocystis jirovecii pneumonia B. Sarcoidosis C. Pulmonary alveolar proteinosis D. Acute respiratory distress syndrome E. Lipoid pneumonia Explanation: (by Perplexity AI, generated with the OpenAI O3 model) • Pulmonary alveolar proteinosis classically produces the “crazy-paving” pattern because lipoproteinaceous material fills the alveoli (ground-glass) while adjacent septa become thickened. • Pneumocystis infection can mimic ground-glass change but septal thickening is usually mild and diffuse rather than perihilar dominant. • Sarcoidosis favours upper-lobe nodularity and fibrosis, not widespread ground-glass. • Early ARDS gives dependent ground-glass opacities without the characteristic septal lines. • Lipoid pneumonia tends to show low-attenuation consolidation and fat-density foci rather than uniform crazy-paving. 119. Which statement best describes the usual presentation of a peripheral pulmonary hamartoma? A. Multiple lobulated nodules arising in childhood B. Endobronchial mass causing obstructive symptoms in young adults C. Solitary, well-circumscribed parenchymal nodule detected incidentally in middle-aged patients D. Rapidly enlarging cavitating lesion with surrounding consolidation E. Calcified mediastinal mass associated with lymphadenopathy Explanation: (by Perplexity AI, generated with the OpenAI O3 model) • Pulmonary hamartoma is the commonest benign lung tumour. More than 90% occur peripherally within the lung parenchyma and appear as a single, smoothly marginated “coin lesion” that is often found on imaging performed for unrelated reasons. Peak incidence is in the fifth to sixth decades; occurrence in children or young adults is rare. • Endobronchial hamartomas (option B) account for only about 5–10% and usually present with cough or obstruction, not as the typical peripheral nodule. • Multiple lesions (option A) and rapid growth with cavitation (option D) are atypical and should raise alternative diagnoses such as metastases or infection. • Mediastinal location with lymphadenopathy (option E) is exceedingly uncommon for hamartoma. 120. In interstitial lung disease, which specific histopathological pattern confers the HIGHEST relative risk for subsequent bronchogenic carcinoma development? A. Desquamative interstitial pneumonia B. Respiratory bronchiolitis–associated interstitial lung disease
  • 247.
    EBDR Exam MCQs& Concepts September 2022 Dr. Kareem Alnakeeb 241 C. Cryptogenic organising pneumonia D. Nonspecific interstitial pneumonia E. Usual interstitial pneumonia Explanation: (by Perplexity AI, generated with the OpenAI O3 model) • Usual interstitial pneumonia (UIP), the pathological correlate of idiopathic pulmonary fibrosis, is strongly linked to an increased incidence of bronchogenic carcinoma because of repetitive epithelial injury and fibrosis that promote oncogenic change. The risk is several-fold higher than in the general population. • Cryptogenic organising pneumonia shows patchy, reversible intra-alveolar granulation tissue with no proven rise in lung cancer incidence. • Desquamative interstitial pneumonia and respiratory bronchiolitis–associated ILD are smoking- related conditions with comparatively little data demonstrating a cancer association. • Nonspecific interstitial pneumonia, often seen in connective-tissue disease, has lower fibrotic distortion than UIP and has not shown the same carcinogenic propensity. 121. Which histological subtype is most frequently encountered in cases of primary orbital rhabdomyosarcoma in children? A. Alveolar B. Pleomorphic C. Embryonal D. Spindle-cell sclerosing E. Botryoid variant Explanation: (by Perplexity AI, generated with the OpenAI O3 model) • Embryonal rhabdomyosarcoma accounts for roughly 70–80% of orbital RMS in the paediatric age group and carries the most favourable prognosis. • Alveolar lesions are less common in the orbit and typically affect older children or adults, while pleomorphic disease is largely restricted to older adults. • The spindle-cell sclerosing form is rare and usually arises in paratesticular or head-and-neck sites outside the orbit. • The botryoid pattern is a variant of embryonal RMS seen mainly in mucosal sites such as the bladder and nasopharynx, not the orbit. 122. A 32-year-old woman develops heavy vaginal bleeding 20 minutes after an uncomplicated vaginal delivery. What is the most common cause of primary postpartum haemorrhage? A. Retained placental tissue B. Uterine atony C. Genital tract laceration D. Uterine inversion E. Uterine artery pseudoaneurysm Explanation: (by Perplexity AI, generated with the OpenAI O3 model) • Uterine atony—failure of the myometrium to contract effectively after placental delivery—is responsible for the majority (≈70-80%) of primary postpartum haemorrhages, because the flaccid uterus cannot compress the spiral arteries. • Retained placental tissue and genital tract lacerations are significant but less frequent causes. Uterine inversion is rare and usually dramatic.
  • 248.
    EBDR Exam MCQs& Concepts September 2022 Dr. Kareem Alnakeeb 242 • Uterine artery pseudoaneurysm is an uncommon iatrogenic cause that typically presents days to weeks postpartum rather than immediately. 123. A 22-year-old woman undergoes echocardiography for exertional dyspnoea. The study shows marked enlargement of the right atrium and right ventricle with a normal-sized left atrium; colour Doppler demonstrates left-to-right shunting at the atrial level. Which congenital abnormality most commonly explains this combination of findings? A. Secundum atrial septal defect without associated venous anomaly B. Sinus venosus atrial septal defect with partial anomalous drainage of the right upper pulmonary vein C. Ostium primum atrial septal defect D. Isolated partial anomalous pulmonary venous return of a left upper lobe vein to the brachiocephalic vein E. Total anomalous pulmonary venous return (supracardiac type) Explanation: (by Perplexity AI, generated with the OpenAI O3 model) • Sinus venosus ASD typically occurs high in the inter-atrial septum near the superior vena cava and is frequently accompanied by partial anomalous pulmonary venous return of the right upper lobe vein into the SVC or right atrium. The combined defects create a significant left-to-right shunt that dilates the right atrium and ventricle while leaving the left atrium normal. • Secundum and ostium primum ASDs (A, C) can enlarge right-sided chambers but are not usually linked to anomalous pulmonary veins. • Isolated PAPVR of a left upper vein (D) causes a smaller shunt and is less likely to produce pronounced right-heart enlargement in an otherwise healthy young adult. • Total anomalous pulmonary venous return (E) presents in infancy with cyanosis and involves all pulmonary veins, not the partial pattern described. 124. Which intracranial ring-enhancing lesion classically shows marked central restricted diffusion on diffusion-weighted MRI? A. High-grade glioma B. Metastatic adenocarcinoma C. Pyogenic brain abscess D. Cerebral toxoplasmosis E. Radiation necrosis Explanation: (by Perplexity AI, generated with the OpenAI O3 model) • Pyogenic brain abscesses contain viscous, cellular pus that markedly impedes Brownian water motion, producing a bright core on DWI with low ADC—an imaging sign often used to differentiate them from other ring-enhancing masses. • Necrotic tumours such as high-grade glioma and metastatic adenocarcinoma usually demonstrate facilitated diffusion because their fluid centres are less cellular. • Cerebral toxoplasmosis typically lacks restricted diffusion owing to necrotic rather than purulent contents. • Radiation necrosis consists of liquefaction and necrotic debris, so diffusion is not restricted. Thus, only a pyogenic abscess reliably shows central restricted diffusion.
  • 249.
    EBDR Exam MCQs& Concepts September 2022 Dr. Kareem Alnakeeb 243 125. A 25-year-old patient presents with headaches and obstructive hydrocephalus. MRI demonstrates a well- defined intraventricular mass at the foramen of Monro showing multiple tiny “soap-bubble” cysts, punctate calcification and mild heterogeneous enhancement. Which of the following tumours is the most likely diagnosis? A. Ependymoma B. Central neurocytoma C. Choroid plexus papilloma D. Subependymal giant cell astrocytoma E. Medulloblastoma Explanation: (by Perplexity AI, generated with the OpenAI O3 model) • The combination of location (lateral ventricle/foramen of Monro in a young adult), well-circumscribed margins and the characteristic “soap-bubble” appearance caused by numerous tiny intratumoral cysts strongly points to central neurocytoma. These tumours typically calcify and enhance mildly, matching the vignette. o Ependymomas may be intraventricular but are more common in children and often show extension through the ventricular walls with less distinct borders. o Choroid plexus papillomas usually occur in children, are frond-like, intensely enhance and often arise in the trigone. o Subependymal giant cell astrocytoma occurs almost exclusively in tuberous sclerosis and shows a solid mass at the foramen of Monro that is hyperenhancing and often calcified but lacks the bubbly cystic pattern. o Medulloblastoma is a posterior fossa tumour, not intraventricular in the lateral ventricles in young adults. 126. Which intracranial tumour is classically associated with neurofibromatosis type 2 but not with neurofibromatosis type 1? A. Optic pathway glioma B. Vestibular schwannoma C. Pilocytic astrocytoma D. Intraventricular meningioma E. High-grade glioblastoma Explanation: (by Perplexity AI, generated with the OpenAI O3 model) • Neurofibromatosis type 2 (NF2) predisposes to multiple extra-axial tumours, notably bilateral vestibular schwannomas and meningiomas. Intraventricular meningiomas arise from arachnoid cap cells within the lateral ventricles and occur in up to half of NF2 patients, whereas they are not a recognised feature of neurofibromatosis type 1. • Optic pathway gliomas (A) and pilocytic astrocytomas (C) are typical of NF1, not NF2. • Vestibular schwannoma (B) is indeed common in NF2 but also occurs sporadically, so it is not as discriminating. • High-grade glioblastoma (E) shows no specific association with either phakomatosis.
  • 250.
    EBDR Exam MCQs& Concepts September 2022 Dr. Kareem Alnakeeb 244 127. A patient’s cardiac MRI shows late gadolinium enhancement involving 50% of the mid-septal left ventricular myocardium. Which coronary artery territory is most likely affected and still contains viable myocardium? A. Right coronary artery, non-viable B. Right coronary artery, viable C. Left circumflex artery, viable D. Left anterior descending artery, viable E. Left circumflex artery, non-viable Explanation: (by Perplexity AI, generated with the OpenAI O3 model) • Late gadolinium enhancement (LGE) of ≤50% wall thickness indicates that at least half of the myocytes are intact and capable of functional recovery after revascularisation. The interventricular septum—particularly its anterior two-thirds—is classically supplied by septal perforators from the left anterior descending (LAD) artery. Therefore a 50% mid-septal infarct points to the LAD territory with residual viability. o Option B: the right coronary artery mainly supplies the inferior septum; an RCA lesion would less commonly give mid-septal LGE, and viability is possible but the anatomy makes this choice less likely. o Option C/E: the left circumflex artery rarely supplies the interventricular septum except in some left-dominant hearts. o Option A: “non-viable” contradicts the ≤50% LGE threshold, which denotes viability. 128. A 55-year-old man with exertional angina undergoes CT coronary calcium scoring that reports an Agatston score of 750. Which statement best reflects the likelihood of significant coronary artery stenosis in this patient? A. A calcium score above 400 virtually excludes ≥50% stenosis B. A score ≥250 has high specificity for ≥50% stenosis on CT angiography C. Any non-zero score mandates invasive coronary angiography irrespective of symptoms D. A calcium score of 0 reliably rules out ≥70% stenosis in all age groups E. Calcium scoring is unsuitable for estimating stenosis risk in symptomatic patients Explanation: (by Perplexity AI, generated with the OpenAI O3 model) • A calcium score threshold of about 250 demonstrates 100% specificity and positive predictive value for ≥50% luminal narrowing on coronary CT angiography in symptomatic cohorts, so a score of 750 makes significant stenosis very likely. • Option A is incorrect because scores >400 increase—rather than exclude—the probability of obstructive disease. • Option C overstates management; many patients are first imaged with CT angiography or functional tests. • Option D is wrong: younger patients can have obstructive, non-calcified plaque despite a zero score. • Option E is incorrect because extensive data show calcium scoring helps stratify symptomatic patients, with higher scores correlating with increasing stenosis severity.
  • 251.
    EBDR Exam MCQs& Concepts September 2022 Dr. Kareem Alnakeeb 245 129. In congenitally corrected transposition of the great arteries, which combination of cardiac connections is present? A. Atrioventricular concordance with ventriculoarterial discordance B. Atrioventricular discordance with ventriculoarterial concordance C. Atrioventricular and ventriculoarterial concordance D. Double-inlet left ventricle with malposed great arteries E. Atrioventricular and ventriculoarterial discordance Explanation: (by Perplexity AI, generated with the OpenAI O3 model) • Congenitally corrected transposition (also called “double discordance”) features atrioventricular (AV) discordance—right atrium draining to the morphologic left ventricle and left atrium to the morphologic right ventricle—and simultaneous ventriculoarterial (VA) discordance—the morphologic left ventricle ejects into the pulmonary artery while the morphologic right ventricle ejects into the aorta. The two discordant connections “cancel out” physiologically, so systemic venous blood still reaches the lungs and oxygenated blood reaches the body. Option E therefore describes the hallmark anatomical arrangement. • Key distractors: o Option A lacks AV discordance, so blood flow would not be physiologically corrected. o Option B describes the rare entity of isolated AV discordance with normal great-artery relations, not congenitally corrected TGA. o Option C depicts the normal heart. o Option D refers to a univentricular connection, an entirely different malformation. 130. Following chemoradiotherapy for a glioblastoma, an MRI including multivoxel proton MR-spectroscopy is performed to distinguish true tumour progression from pseudoprogression. Which spectroscopic change is most sensitive for active tumour progression? A. Fall in lipid peak B. New alanine resonance C. Decrease in choline/N-acetylaspartate (NAA) ratio D. Rise in choline/NAA ratio E. Isolated increase in creatine Explanation: (by Perplexity AI, generated with the OpenAI O3 model) • Active glioma progression shows heightened cell-membrane turnover and cellularity, producing a marked rise in choline with concurrent loss of neuronal NAA, so the choline/NAA ratio increases (often >1.4), the most sensitive MR-spectroscopic marker of viable tumour. Pseudoprogression or radiation change typically demonstrates low choline and often elevated lipid-lactate from necrosis, not an increased choline/NAA ratio. o Option A is incorrect: lipid peaks usually rise in treatment-related necrosis rather than active tumour. o Option B (alanine) characterises meningioma, not glioma assessment. o Option C describes the opposite of tumour behaviour; a falling ratio suggests necrosis. o Option E: creatine changes are nonspecific and less sensitive than choline/NAA alterations.
  • 252.
    EBDR Exam MCQs& Concepts September 2022 Dr. Kareem Alnakeeb 246 131. A 32-year-old heavy smoker presents with progressive dry cough and exertional dyspnoea. High- resolution CT shows numerous 2–5 mm centrilobular nodules, some cavitating, and multiple small thin- walled cysts predominantly in the upper and mid-zone lungs with relative costophrenic sparing. Which diagnosis best explains this pattern? A. Respiratory bronchiolitis–interstitial lung disease B. Pulmonary Langerhans cell histiocytosis C. Lymphangioleiomyomatosis D. Sarcoidosis E. Hypersensitivity pneumonitis Explanation: (by Perplexity AI, generated with the OpenAI O3 model) • Pulmonary Langerhans cell histiocytosis (PLCH) almost exclusively affects young adult smokers and classically shows a mixture of centrilobular nodules that may cavitate and evolve into irregular, thin- walled cysts concentrated in the upper and middle lung zones with sparing of lung bases. • Respiratory bronchiolitis–ILD (A) produces faint ground-glass haziness rather than discrete nodules/cysts. • Lymphangioleiomyomatosis (C) occurs in women, features uniform round cysts of equal size, and lacks nodules. • Sarcoidosis (D) shows perilymphatic nodules and upper-zone fibrosis without cystic change. • Hypersensitivity pneumonitis (E) gives diffuse ground-glass opacities and mosaic attenuation, not cavitating nodules evolving into cysts. 132. A 65-year-old woman presents six months after pelvic radiotherapy for cervical cancer with irritative lower-urinary-tract symptoms. Cystoscopy is unremarkable. MRI pelvis shows diffuse concentric thickening of the urinary bladder wall with low T2 signal intensity and homogeneous post-contrast enhancement. Which is the most likely diagnosis? A. Recurrent cervical tumour invading the bladder B. Radiation-induced cystitis C. Schistosomiasis of the bladder D. Transitional-cell carcinoma of the bladder E. Tuberculous cystitis Explanation: (by Perplexity AI, generated with the OpenAI O3 model) • Radiation-induced cystitis typically appears within months to a few years after pelvic irradiation and manifests as diffuse, symmetric bladder wall thickening that is low on T2-weighted MRI and enhances homogeneously—reflecting fibrosis and submucosal vascular changes. There is no discrete mass or focal mural defect. • Recurrent cervical tumour (A) would more often produce an eccentric soft-tissue mass extending from the cervix into the bladder rather than smooth concentric thickening. • Schistosomiasis (C) gives irregular calcified or nodular thickening, often with mural calcification. • Transitional-cell carcinoma (D) usually presents as a focal polypoid mass or eccentric asymmetric thickening, not diffuse symmetric changes. • Tuberculous cystitis (E) classically shows a contracted, small-capacity bladder with irregular wall thickening and sometimes calcification, usually years after infection rather than shortly after radiotherapy.
  • 253.
    EBDR Exam MCQs& Concepts September 2022 Dr. Kareem Alnakeeb 247 133. In patients with pulmonary sarcoidosis, up to what percentage will progress to irreversible pulmonary fibrosis despite treatment? A. 5% B. 10% C. 20% D. 35% E. 50% Explanation: (by Perplexity AI, generated with the OpenAI O3 model) • Most individuals with sarcoidosis experience either remission or a stable chronic course. However, a minority develop sarcoidosis-associated pulmonary fibrosis, which carries a markedly worse prognosis. • Contemporary epidemiological and pathological reviews consistently show that approximately one- fifth—around 20%—of patients progress to established fibrotic lung disease even when managed with corticosteroids or steroid-sparing agents. • Lower figures such as 5% or 10% underestimate this well-documented risk, whereas 35% and 50% overstate it and are not supported by current data. 134. Regarding transitional cell carcinoma (TCC) of the urinary tract, which statement is MOST accurate? A. Primary TCC occurs more frequently in the upper ureter than in the urinary bladder. B. Squamous metaplasia is a recognised premalignant change preceding TCC in the bladder. C. Multifocality is common because the urothelium is exposed to carcinogens along its entire length. D. Hydronephrosis is uncommon with upper-tract TCC owing to its infiltrative growth pattern. E. Smoking is not a significant risk factor for upper-tract TCC. Explanation: (by Perplexity AI, generated with the OpenAI O3 model) • Transitional cell carcinoma arises from urothelium anywhere from renal pelvis to urethra. Exposure of the whole lining to urinary carcinogens (“field change”effect) results in synchronous or metachronous tumours, so multifocality is typical—making option C correct. • Upper-tract (renal pelvis and ureter) tumours are much less common than bladder TCC, not more common, invalidating option A. • Keratinising squamous metaplasia predisposes to squamous, not transitional, cell cancer, so option B is wrong. • Obstructive hydronephrosis is actually frequent with ureteric or pelvic TCC because the mass narrows the lumen, refuting option D. • Cigarette smoking is a major risk factor for both bladder and upper-tract TCC, so option E is false. 135. A 35-year-old man presents with progressive dyspnoea and palpitations. Cardiac MRI shows a globally dilated left ventricle with systolic dysfunction (LVEF 30%), diffuse hypokinesia and patchy sub-epicardial late gadolinium enhancement in the lateral wall. Which is the most likely diagnosis? A. Acute myocardial infarction B. Dilated cardiomyopathy secondary to chronic alcohol abuse C. Acute viral myocarditis D. Hypertrophic cardiomyopathy E. Cardiac sarcoidosis
  • 254.
    EBDR Exam MCQs& Concepts September 2022 Dr. Kareem Alnakeeb 248 Explanation: (by Perplexity AI, generated with the OpenAI O3 model) • Sub-epicardial or mid-wall late gadolinium enhancement sparing the endocardium, particularly in the lateral wall, is characteristic of acute viral myocarditis and reflects inflammation and necrosis of the outer myocardial layers. Dilated cardiomyopathy from alcohol typically shows mid-wall, not sub- epicardial, enhancement and requires a history of heavy intake. • Acute infarction produces sub-endocardial or transmural enhancement in a coronary artery territory. • Hypertrophic cardiomyopathy shows hypertrophied, non-dilated ventricles with patchy mid-wall enhancement in hypertrophied segments. • Cardiac sarcoidosis favours multifocal mid-wall or transmural enhancement, often with right- ventricular involvement and associated lymphadenopathy. 136. In a patient with acute myocarditis, transthoracic echocardiography shows a markedly reduced left- ventricular ejection fraction (LVEF 28%) with global hypokinesis. Which term best describes this finding? A. Diastolic dysfunction B. Systolic dysfunction C. Concentric hypertrophy D. Restrictive physiology E. Cardiac tamponade Explanation: (by Perplexity AI, generated with the OpenAI O3 model) • The hallmark of systolic dysfunction is an abnormally low LVEF reflecting impaired ventricular contractility; acute myocarditis can cause diffuse myocardial inflammation and global hypokinesis, reducing the ejection fraction to <40%. • Diastolic dysfunction (A) involves impaired relaxation with preserved or mildly reduced LVEF. • Concentric hypertrophy (C) refers to increased wall thickness with normal cavity size, not a low ejection fraction. • Restrictive physiology (D) denotes normal systolic function but severely impaired ventricular filling. • Cardiac tamponade (E) produces chamber compression and pulsus paradoxus rather than a primary fall in LVEF. 137. Which inherited cardiomyopathy that predisposes to ventricular tachyarrhythmia is characterised pathologically by progressive fibrofatty replacement of the right ventricular free wall? A. Dilated cardiomyopathy B. Hypertrophic cardiomyopathy C. Arrhythmogenic right ventricular cardiomyopathy D. Restrictive cardiomyopathy E. Left ventricular non-compaction Explanation: (by Perplexity AI, generated with the OpenAI O3 model) • Arrhythmogenic right ventricular cardiomyopathy (ARVC) is an autosomal-dominant desmosomal disorder in which normal myocytes are gradually replaced by fibrofatty tissue, especially within the “triangle of dysplasia” of the right ventricle. This provides the substrate for re-entrant ventricular tachycardia and sudden death. • Dilated cardiomyopathy shows chamber dilation and fibrosis rather than fatty infiltration; hypertrophic cardiomyopathy features myocyte hypertrophy and disarray; restrictive cardiomyopathy is defined by diastolic dysfunction with normal ventricular thickness; left ventricular non-
  • 255.
    EBDR Exam MCQs& Concepts September 2022 Dr. Kareem Alnakeeb 249 compaction results from arrested myocardial development with deep trabeculations—none display the hallmark fibrofatty replacement seen in ARVC. 138. A 45-year-old woman underwent total thyroidectomy for papillary thyroid carcinoma. Six months later her stimulated serum thyroglobulin is 48 ng/mL, but a diagnostic ^123I whole-body scan shows no abnormal uptake. Which ONE of the following is the most appropriate next management step? A. Empiric high-dose 131 I ablation therapy B. 18 F-FDG PET/CT to localise iodine-negative metastases C. Neck ultrasound with fine-needle aspiration of any suspicious nodes D. Repeat diagnostic 123 I scan in 6 months while on TSH suppression E. Begin tyrosine-kinase inhibitor therapy (e.g. sorafenib) Explanation: (by Perplexity AI, generated with the OpenAI O3 model) • A markedly elevated stimulated thyroglobulin after thyroidectomy indicates residual or metastatic differentiated thyroid cancer even when radio-iodine imaging is negative. Current guidelines recommend targeted structural imaging—starting with high-resolution neck ultrasound—because cervical nodal disease is the commonest source and is potentially curable surgically. Empiric high- dose 131 I (Option A) may be considered only after structural evaluation, as many iodine-negative lesions will not respond and the radiation burden is significant. • 18 F-FDG PET/CT (Option B) is useful if ultrasound is negative or non-diagnostic but is not first-line. • Simply repeating the 123 I scan (Option D) delays definitive evaluation and rarely converts to positive. • Tyrosine-kinase inhibitors (Option E) are reserved for progressive, inoperable, radio-iodine-refractory disease, not as initial investigation. 139. A 7-year-old boy presents with seizures, global developmental delay and multiple hypomelanotic macules on skin examination. Which inherited condition best explains the constellation of neurological and cutaneous findings? A. Neurofibromatosis type 1 B. Tuberous sclerosis complex C. Sturge–Weber syndrome D. Ataxia-telangiectasia E. Phenylketonuria Explanation: (by Perplexity AI, generated with the OpenAI O3 model) • Tuberous sclerosis complex is an autosomal-dominant phakomatosis characterised by cortical tubers causing epilepsy and learning difficulties, alongside cutaneous manifestations such as hypomelanotic “ash-leaf” macules and shagreen patches. • Neurofibromatosis 1 produces café-au-lait spots and neurofibromas but not cortical tubers or ash- leaf lesions. • Sturge–Weber gives facial port-wine stains and leptomeningeal angiomatosis rather than hypomelanotic macules. • Ataxia-telangiectasia features progressive cerebellar ataxia and oculocutaneous telangiectasias, not seizures from cortical tubers. • Phenylketonuria leads to intellectual impairment if untreated but lacks the specific skin lesions and brain hamartomas seen here.
  • 256.
    EBDR Exam MCQs& Concepts September 2022 Dr. Kareem Alnakeeb 250 140. A 55-year-old man presents with digital clubbing, joint pains and plain radiographs showing smooth, bilateral periosteal new bone along the diaphyses of the tibiae and radii with no underlying destructive lesion. Which underlying condition most commonly produces this skeletal reaction? A. Benign fibroma of bone B. Deep (desmoid-type) fibromatosis of the abdominal wall C. Primary lung carcinoma D. Crohn’s disease–related enteric fistulae E. Cyanotic congenital heart disease Explanation: (by Perplexity AI, generated with the OpenAI O3 model) • Hypertrophic osteoarthropathy (HOA) is characterised by digital clubbing and symmetric, smooth periosteal reaction affecting long bones. The classic and commonest secondary cause is a primary intrathoracic malignancy—especially non-small-cell lung carcinoma—because tumour-derived vascular and humoral factors drive periosteal new bone. • Cyanotic heart disease and inflammatory bowel disease can cause HOA but far less frequently. • Benign bony fibromas and desmoid-type fibromatosis do not generate the diffuse periosteal response seen in HOA; they produce focal cortical remodeling or soft-tissue masses without systemic skeletal changes. 141. A 42-year-old marathon runner presents with progressive knee discomfort and intermittent swelling. Radiographs show multiple smooth, well-defined, round intra-articular calcified bodies; MRI confirms numerous ossified nodules within the synovium but an otherwise normal joint. Which diagnosis best explains these imaging findings? A. Pigmented villonodular synovitis B. Synovial chondromatosis C. Intra-articular osteoid osteoma D. Popliteal artery arteriovenous malformation E. Hydroxyapatite deposition disease Explanation: (by Perplexity AI, generated with the OpenAI O3 model) • Synovial chondromatosis causes metaplasia of synovial membrane into cartilaginous nodules that calcify, producing multiple smooth “loose bodies” evident on radiographs and MRI—exactly as seen here. Pigmented villonodular synovitis gives blooming low-signal hemosiderin but rarely calcifies. • Osteoid osteoma forms a single cortical nidus, not numerous intra-articular bodies. • Arteriovenous malformations show serpiginous flow voids or enhancing vascular channels, not ossified nodules. • Hydroxyapatite deposition disease typically affects tendons (e.g. rotator cuff) creating amorphous peri-tendinous calcification, not discrete intra-articular ossicles. 142. In a young patient with sickle-cell trait, CT shows an infiltrative, hypo-enhancing solid mass centred in the renal medulla causing caliectasis and retroperitoneal nodes. Which diagnosis best fits these findings? A. Papillary renal cell carcinoma B. Collecting (Bellini) duct carcinoma C. Renal medullary carcinoma D. Transitional cell carcinoma of the renal pelvis E. Oncocytoma
  • 257.
    EBDR Exam MCQs& Concepts September 2022 Dr. Kareem Alnakeeb 251 Explanation: (by Perplexity AI, generated with the OpenAI O3 model) • Renal medullary carcinoma is a rare, highly aggressive tumour almost exclusive to patients with sickle-cell trait. Imaging typically demonstrates a central medullary mass that is infiltrative, poorly marginated and markedly hypo-enhancing compared with renal cortex, often accompanied by caliectasis and regional lymphadenopathy. • Collecting duct carcinoma can mimic the pattern but lacks the strong sickle-cell association. • Papillary RCCs are usually well circumscribed and hypovascular cortical tumours. • Transitional cell carcinoma arises from the urothelium and appears as a filling defect or discrete pelvic mass rather than a parenchymal infiltrative lesion. • Oncocytomas are benign, usually cortical and frequently show a characteristic central scar, not medullary infiltration. 143. In a patient with congenitally corrected transposition of the great arteries (ccTGA), which segmental arrangement is most characteristic of the condition, showing atrioventricular discordance with ventriculoarterial discordance but normal atrial situs? A. {S, D, D} B. {I, D, L} C. {S, L, L} D. {I, L, D} E. {S, D, L} Explanation: (by Perplexity AI, generated with the OpenAI O3 model) • ccTGA shows normal atrial situs solitus (S), left-looped ventricles (L) producing atrioventricular discordance, and left-looped great arteries (L) causing ventriculoarterial discordance, giving the classic {S, L, L} segmental set. • {S, D, D} (option A) is the normal heart without discordance. • {I, D, L} (option B) begins with atrial situs inversus, not typical for ccTGA. • {I, L, D} (option D) has both situs inversus and a normally related great-artery arrangement. • {S, D, L} (option E) would have atrioventricular concordance with only ventriculoarterial discordance (d-TGA pattern), not the double discordance of ccTGA. 144. A 56-year-old man presents with progressive dyspnoea and dry cough. High-resolution CT shows bibasal ground-glass opacities with superimposed inter- and intralobular septal thickening (“crazy-paving”) and small bilateral pleural effusions. Which is the most likely diagnosis? A. Pneumocystis jirovecii pneumonia B. Cryptogenic organising pneumonia C. Acute interstitial pneumonia D. Lymphocytic interstitial pneumonia E. Pulmonary alveolar proteinosis Explanation: (by Perplexity AI, generated with the OpenAI O3 model) • Pulmonary alveolar proteinosis classically produces diffuse or bibasal ground-glass opacities with a crazy-paving pattern due to intra-alveolar surfactant accumulation, and minor pleural effusions may be present. • Lymphocytic interstitial pneumonia (option D) usually shows thin-walled cysts mixed with ground- glass change rather than uniform crazy-paving.
  • 258.
    EBDR Exam MCQs& Concepts September 2022 Dr. Kareem Alnakeeb 252 • Pneumocystis pneumonia (option A) can cause ground-glass opacities but pleural effusions are uncommon and septal thickening less pronounced. • Cryptogenic organising pneumonia (option B) favours peripheral consolidations and bronchocentric nodules, not crazy-paving. • Acute interstitial pneumonia (option C) presents with diffuse alveolar damage leading to widespread consolidation, lacking the characteristic crazy-paving appearance seen here. 145. A 25-year-old woman with facial angiofibromas and epilepsy undergoes abdominal CT, which shows multiple bilateral renal masses containing macroscopic fat consistent with angiomyolipomas. The renal lesions and skin findings are most characteristic of which phakomatosis? A. Neurofibromatosis type 1 B. Sturge–Weber syndrome C. Tuberous sclerosis complex D. Von Hippel–Lindau disease E. Ataxia-telangiectasia Explanation: (by Perplexity AI, generated with the OpenAI O3 model) • Renal angiomyolipomas that are multiple and bilateral are a classic extracranial manifestation of tuberous sclerosis complex (TSC); up to 80% of TSC patients develop them, often alongside facial angiofibromas and epilepsy. Sporadic angiomyolipoma is usually solitary, while the other phakomatoses listed rarely produce fat-containing renal tumours. • Neurofibromatosis type 1 can give dermal neurofibromas but not angiomyolipomas. • Sturge–Weber causes leptomeningeal angiomata and port-wine stains, not renal fat tumours. • Von Hippel–Lindau is linked to renal cell carcinoma and cysts, not angiomyolipoma. • Ataxia-telangiectasia has immunodeficiency and telangiectasias without renal involvement. 146. In acute cervical spine trauma, which ligament’s disruption is most confidently detected on sagittal fat- saturated (STIR) MRI sequences? A. Anterior longitudinal ligament B. Posterior longitudinal ligament C. Ligamentum flavum D. Interspinous ligament E. Supraspinous ligament Explanation: (by Perplexity AI, generated with the OpenAI O3 model) • Fat-saturated sagittal T2/STIR images accentuate fluid and oedema, making the normally low-signal ligamentum flavum stand out when torn or swollen; its elastic fibres show bright high signal against suppressed fatty marrow, so disruption is readily recognised. • The anterior and posterior longitudinal ligaments (A, B) are thin ventral structures best assessed on routine T2 without fat sat, while the interspinous and supraspinous ligaments (D, E) lie posteriorly in fatty tissue where STIR can help but partial-volume effects and obliquity often obscure focal injury, reducing confidence compared with the ligamentum flavum.
  • 259.
    EBDR Exam MCQs& Concepts September 2022 Dr. Kareem Alnakeeb 253 147. A 75-year-old woman presents with acute left hip pain after a fall; pelvic radiograph shows the fracture line running just below the femoral head and above the femoral neck’s midpoint. What type of proximal femoral fracture is this? A. Basicervical fracture B. Intertrochanteric fracture C. Subcapital fracture D. Subtrochanteric fracture E. Transcervical fracture Explanation: (by Perplexity AI, generated with the OpenAI O3 model) • The fracture described lies immediately distal to the femoral head within the femoral neck’s most proximal zone, classically termed a subcapital fracture. Subcapital fractures disrupt the retinacular vessels and carry the highest risk of avascular necrosis, guiding urgent surgical fixation. • A basicervical fracture (A) occurs at the base of the femoral neck near the intertrochanteric line, not just below the head. • Intertrochanteric fractures (B) extend between the greater and lesser trochanters, sparing the femoral neck. • Subtrochanteric fractures (D) lie 5 cm or less distal to the lesser trochanter. • A transcervical fracture (E) passes through the mid-portion of the femoral neck, lying more distal than the subcapital location. 148. On a coronal T2-weighted MRI of the femoral head, which imaging feature is classically described as the “double-line sign” of osteonecrosis? A. An inner bright serpiginous line with an outer dark serpiginous line at the necrosis–viable bone interface B. A single low-signal subchondral crescent parallel to the articular surface C. Diffuse marrow oedema with loss of normal fatty signal in the femoral neck D. Patchy high T1 signal within the metaphysis due to fatty infiltration E. Focal cartilage thinning with underlying subchondral cysts Explanation: (by Perplexity AI, generated with the OpenAI O3 model) • The double-line sign is highly specific for osteonecrosis. It appears on fluid-sensitive sequences as two adjacent serpiginous rims: an inner high T2 signal rim reflecting hyperaemic granulation tissue and an outer low T2 signal rim representing reactive sclerosis at the border between necrotic and viable bone, thus fulfilling option A. • Option B describes the crescent sign of subchondral collapse, not the double-line sign. • Option C represents non-specific marrow oedema seen in many pathologies including transient osteoporosis. • Option D refers to fatty marrow replacement rather than osteonecrosis. • Option E depicts degenerative osteoarthritis, unrelated to the classic MRI sign of osteonecrosis.
  • 260.
    EBDR Exam MCQs& Concepts September 2022 Dr. Kareem Alnakeeb 254 149. A 68-year-old ventilated patient has a right internal jugular venous line inserted for haemodynamic monitoring. A chest radiograph taken immediately after the procedure is shown. Which anatomical structure should the tip of a correctly placed Swan–Ganz catheter project over on a frontal chest radiograph? A. Superior vena cava B. Right atrium C. Main pulmonary artery D. Inferior vena cava E. Right ventricular outflow tract Explanation: (by Perplexity AI, generated with the OpenAI O3 model) • The thermodilution (Swan–Ganz) catheter is designed to measure pulmonary artery pressures and cardiac output; correct positioning requires the balloon-tipped tip to advance through the right heart and lie within the main pulmonary artery, usually just beyond its bifurcation. On the frontal chest radiograph the tip should project over the hilum at the level of the left or right main pulmonary artery shadow. • If the tip remains in the superior vena cava or right atrium (options A and B) it cannot measure pulmonary artery pressures and risks arrhythmia; in the inferior vena cava (option D) it has not entered the right heart; placement in the right ventricular outflow tract (option E) may produce ventricular ectopics and gives falsely elevated systolic readings. 150. A 3-month-old infant presents with fever, irritability and firm swelling over the right mandible; radiographs show symmetrical, dense subperiosteal new bone along the mandibular body sparing the condyles. Which diagnosis best explains the imaging findings and clinical picture? A. Acute osteomyelitis B. Caffey’s disease (infantile cortical hyperostosis) C. Langerhans cell histiocytosis D. Hypervitaminosis A E. Non-accidental injury Explanation: (by Perplexity AI, generated with the OpenAI O3 model) • Caffey’s disease produces a triad of fever, soft-tissue swelling and characteristic lamellated periosteal new bone confined to diaphyseal cortices, most frequently involving the mandible (≈75% of cases) with metaphyseal sparing. • Osteomyelitis (A) usually shows focal cortical destruction and subperiosteal abscess rather than smooth, symmetric cortical thickening. • Langerhans cell histiocytosis (C) manifests as lytic “punched-out” lesions, not diffuse hyperostosis. • Hypervitaminosis A (D) may cause periostitis but typically affects long bones in older children and lacks mandibular predilection. • Non-accidental injury (E) demonstrates mixed-age fractures and metaphyseal “corner” injuries, not uniform subperiosteal new bone.
  • 261.
    EBDR Exam MCQs& Concepts September 2022 Dr. Kareem Alnakeeb 255 151. Infarction of the medial frontal and superior medial parietal cortices most commonly indicates occlusion of which intracranial artery territory? A. Anterior cerebral artery B. Middle cerebral artery C. Posterior cerebral artery D. Anterior choroidal artery E. Superior cerebellar artery Explanation: (by Perplexity AI, generated with the OpenAI O3 model) • The anterior cerebral artery (ACA) supplies the medial aspects of the frontal lobe—including the paracentral lobule—and the superior medial parietal lobe. Infarcts in this vascular bed therefore present with weakness and sensory loss mainly in the contralateral lower limb, reflecting involvement of the paracentral cortex. • The middle cerebral artery perfuses the lateral frontal, parietal and temporal lobes, not the medial surfaces, so MCA infarction is a distractor. • Posterior cerebral artery strokes involve the occipital and inferior temporal regions. • The anterior choroidal artery supplies deep structures such as the posterior limb of the internal capsule and hippocampus, not cortical medial surfaces. • The superior cerebellar artery only supplies the superior cerebellar hemispheres and midbrain, not supratentorial cortex. 152. A 35-year-old man presents with recurrent lower respiratory tract infections since childhood. Chest CT shows a well-defined mass of non-functioning lung tissue in the medial basal segment of the left lower lobe, supplied by an anomalous artery arising from the descending thoracic aorta and draining via the pulmonary veins. What is the most likely diagnosis? A. Bronchogenic cyst B. Intralobar pulmonary sequestration C. Extralobar pulmonary sequestration D. Congenital pulmonary airway malformation (CPAM) type II E. Pulmonary arteriovenous malformation Explanation: (by Perplexity AI, generated with the OpenAI O3 model) • An intralobar sequestration consists of aberrant lung tissue within the visceral pleura of an otherwise normal lobe, classically fed by a systemic arterial branch (usually from the thoracic or abdominal aorta) and draining into pulmonary veins—features shown on the CT. • Extralobar sequestrations are separate from normal lung and have their own pleural covering, typically presenting in infancy and often located near the posterior costophrenic angle. • Bronchogenic cysts are fluid-filled, not vascular-supplied masses. • CPAM type II is a multicystic lesion communicating with airways and supplied by the pulmonary circulation. • Pulmonary arteriovenous malformations feature a direct artery-to-vein communication without non- functioning lung tissue.
  • 262.
    EBDR Exam MCQs& Concepts September 2022 Dr. Kareem Alnakeeb 256 153. A 45-year-old man with exertional dyspnoea undergoes cardiac MRI. Cine four-chamber and short-axis steady-state free-precession images (not shown) demonstrate focal systolic hypokinesia and marked late gadolinium enhancement confined to the mid-anteroseptal left-ventricular wall; maximal wall thickness elsewhere is 14 mm. Which diagnosis best explains these findings? A. Acute myocarditis B. Dilated cardiomyopathy C. Hypertrophic cardiomyopathy with myocardial scar D. Ischaemic infarction of left-anterior-descending territory E. Restrictive cardiomyopathy Explanation: (by Perplexity AI, generated with the OpenAI O3 model) • Hypertrophic cardiomyopathy (HCM) can present with focal regions of hypertrophy or scar, often in the anteroseptum, producing focal contractile impairment on cine imaging and dense mid- myocardial late enhancement on post-contrast sequences; overall wall thickness may be only mildly increased, as here. • Acute myocarditis (A) typically shows subepicardial or patchy mid-wall LGE in a non-coronary distribution and is rarely limited to a single septal segment. • Dilated cardiomyopathy (B) produces global systolic dysfunction with diffuse rather than focal mid- wall fibrosis. • Ischaemic infarction (D) gives subendocardial or transmural LGE in a coronary vascular territory and is accompanied by wall thinning, not hypertrophy. • Restrictive cardiomyopathy (E) manifests with bi-atrial enlargement and diffuse fibrosis, lacking the discrete septal scar pattern seen here. 154. Which imaging modality is most sensitive for detecting early skeletal metastases during whole-body oncologic staging? A. Plain radiography B. Whole-body MRI C. 18 F-FDG PET/CT D. 99 mTc-MDP planar bone scintigraphy E. CT skeletal survey Explanation: (by Perplexity AI, generated with the OpenAI O3 model) • Whole-body MRI directly visualises tumour marrow infiltration, giving the highest lesion-level sensitivity (≈94%) for bone metastases and outperforming both 99mTc-MDP bone scans and FDG PET/CT, particularly for early marrow-based or predominantly lytic deposits. • Bone scintigraphy (option D) is highly sensitive for osteoblastic activity but misses many early or purely lytic lesions and has lower overall sensitivity (≈62-78%). • FDG PET/CT (option C) is excellent for metabolically active disease but is less sensitive for osteoblastic metastases and may be inferior to MRI on a per-patient basis in several cancers. • CT surveys (option E) and plain radiographs (option A) require 30-50% bone mineral loss before lesions are visible, making them least sensitive for early metastatic spread.
  • 263.
    EBDR Exam MCQs& Concepts September 2022 Dr. Kareem Alnakeeb 257 155. A term neonate presents with mild tachypnoea; chest radiograph shows marked over-inflation of the left upper lobe without mediastinal shift. What is the most appropriate initial management of suspected congenital lobar emphysema in this scenario? A. Emergency lobectomy B. High-flow nasal oxygen and urgent CT scan C. Observation with serial clinical and radiographic follow-up D. Insertion of an intercostal chest drain E. Continuous positive airway pressure ventilation Explanation: (by Perplexity AI, generated with the OpenAI O3 model) • Mild or asymptomatic congenital lobar emphysema (CLE) often stabilises or improves as the infant grows; therefore, conservative management with close observation and follow-up imaging is recommended. Surgery (lobectomy) is reserved for progressive respiratory distress or significant mediastinal shift, making option A inappropriate here. • High-flow oxygen and CT (option B) may expose the infant to unnecessary radiation and are not first- line in a stable patient. • Chest drains (option D) are contraindicated because CLE is not a pneumothorax and drainage risks worsening the condition. • CPAP (option E) can further over-distend the affected lobe and is avoided. 156. Which histologically benign pleural tumour is well‐known for local recurrence and late distant metastases despite apparently complete surgical excision? A. Desmoplastic mesothelioma B. Solitary fibrous tumour of the pleura C. Pleural lipoma D. Pleural calcifying fibrous tumour E. Pleural schwannoma Explanation: (by Perplexity AI, generated with the OpenAI O3 model) • Solitary fibrous tumour of the pleura is usually benign on histology yet behaves unpredictably, with up to one-third of cases showing local recurrence and a smaller proportion developing haematogenous metastases years after resection. • Desmoplastic mesothelioma (A) is frankly malignant from the outset. • Pleural lipoma (C), calcifying fibrous tumour (D) and pleural schwannoma (E) are all benign lesions that very rarely, if ever, recur or metastasise following complete excision. 157. Which imaging feature is most typically seen when a granular cell tumour of the breast mimics an aggressive primary carcinoma? A. Clustered micro-calcification B. Spiculated margin with posterior acoustic shadowing C. Circumscribed oval mass with posterior enhancement D. Complex cystic-solid mass with internal septations E. Hypervascular mass on colour Doppler Explanation: (by Perplexity AI, generated with the OpenAI O3 model) • Granular cell tumours are benign neural-derived lesions but on mammography and ultrasound they frequently appear malignant. The common aggressive appearance is a small, irregular or spiculated mass that produces marked posterior acoustic shadowing, closely resembling an invasive ductal carcinoma.
  • 264.
    EBDR Exam MCQs& Concepts September 2022 Dr. Kareem Alnakeeb 258 • Calcifications (A) are uncommon; most lesions are non-calcified. • Well-circumscribed masses with enhancement (C) represent the less typical benign pattern. • Complex cystic-solid architecture (D) and prominent Doppler vascularity (E) are not characteristic and would prompt alternative differentials such as papillary or phyllodes tumours. 158. A 29-year-old woman presents with chest discomfort. Chest radiograph shows a well-circumscribed anterior mediastinal mass containing both fat density and an air–fluid level. Which is the most likely diagnosis? A. Bronchogenic cyst B. Mature teratoma C. Thymic cyst D. Pericardial cyst E. Lymphoma Explanation: (by Perplexity AI, generated with the OpenAI O3 model) • Mature (benign) teratomas classically arise in the anterior mediastinum and often contain a mix of fat, fluid and occasionally teeth or bone. When internal fat interfaces with secreted fluid, an air–fluid or fat–fluid level may be seen, a highly characteristic feature pointing toward a teratoma. • Bronchogenic and thymic cysts are purely fluid-attenuation without macroscopic fat. • Pericardial cysts sit at the cardiophrenic angle and also lack fat. • Lymphoma appears soft-tissue attenuating and rarely demonstrates cystic change or fat, making it the least likely diagnosis here. 159. Which congenital coronary artery anomaly is considered“malignant” because its inter-arterial course predisposes to sudden cardiac death in young adults? A. Retro-aortic left circumflex artery B. Prepulmonic left anterior descending artery C. Inter-arterial right coronary artery from the left sinus D. Conus branch arising separately from right sinus E. High take-off coronary ostia Explanation: (by Perplexity AI, generated with the OpenAI O3 model) • A right coronary artery that originates from the left coronary sinus and runs between the aorta and pulmonary trunk has an inter-arterial (malignant) course. Exercise-related expansion of the great vessels can compress the slit-like proximal segment, causing ischaemia and sudden cardiac death. • Retro-aortic and prepulmonic courses (options A and B) lie away from the great vessels and are regarded as benign. • A separate conus branch (option D) and high ostial take-off (option E) are normal variants that do not confer the same risk profile. 160. Following blunt head injury, CT brain shows effacement of the basal cisterns without midline shift. Which intracranial lesion most commonly produces this appearance? A. Acute epidural haematoma B. Acute subdural haematoma C. Cerebral contusion D. Intracerebral laceration E. Diffuse cerebral oedema
  • 265.
    EBDR Exam MCQs& Concepts September 2022 Dr. Kareem Alnakeeb 259 Explanation: (by Perplexity AI, generated with the OpenAI O3 model) • Effacement of the suprasellar and perimesencephalic cisterns in the absence of midline shift indicates raised intracranial pressure from globally swollen brain rather than from a space-occupying focal haematoma. • Diffuse cerebral oedema causes symmetrical brain swelling, narrowing of ventricles and cisterns, and can progress to brain-stem compression. • Epidural and subdural haematomas usually create mass effect with midline shift; contusions and lacerations are focal parenchymal injuries that rarely obliterate basal cisterns unless extensive, and when large enough they also shift midline structures. 161. A 48-year-old man with longstanding bronchiectasis presents with worsening wrist pain; radiographs show bilateral, symmetrical periosteal new bone along the distal radius and ulna shafts. What is the most likely underlying mechanism for this skeletal finding? A. Autoimmune synovial inflammation B. Deposition of urate crystals in periosteum C. Hypertrophic pulmonary osteoarthropathy secondary to intrapulmonary shunting D. Direct metastatic spread from bronchogenic carcinoma E. Chronic steroid-induced osteoporosis Explanation: (by Perplexity AI, generated with the OpenAI O3 model) • Hypertrophic pulmonary osteoarthropathy (HPOA) results from circulating vasodilatory and growth factors produced in chronic intrapulmonary shunting, leading to periosteal new-bone formation along long-bone cortices—classically in patients with bronchiectasis or other chronic lung disease. • Autoimmune synovitis (A) causes joint space narrowing and erosions rather than smooth laminated periosteal reaction. • Urate deposition (B) produces punched-out juxta-articular erosions with overhanging edges, not diffuse periostitis. • Metastatic spread (D) gives focal lytic or sclerotic lesions, not uniform cortical layering. • Steroid-induced osteoporosis (E) causes cortical thinning and fractures, not periosteal apposition. 162. A full-term neonate develops respiratory distress shortly after birth; chest radiograph (frontal view) shows patchy bilateral opacification with areas of lucency and a flattened hemidiaphragm. What is the most likely underlying diagnosis? A. Transient tachypnoea of the newborn B. Meconium aspiration syndrome C. Respiratory distress syndrome (hyaline membrane disease) D. Neonatal pneumonia E. Congenital lobar overinflation Explanation: (by Perplexity AI, generated with the OpenAI O3 model) • Patchy coarse opacities intermixed with hyperlucent, over-inflated lung fields and depression of the diaphragms in a term baby are classic for meconium aspiration syndrome, reflecting airway obstruction by particulate meconium. • Transient tachypnoea typically shows prominent vascular markings and fluid in fissures without over- inflation. • Hyaline membrane disease occurs in preterm infants and gives uniform reticulogranular opacities with air bronchograms, not focal hyperinflation.
  • 266.
    EBDR Exam MCQs& Concepts September 2022 Dr. Kareem Alnakeeb 260 • Neonatal pneumonia may mimic hyaline membrane disease or show lobar consolidation but lacks widespread over-inflation. • Congenital lobar overinflation produces unilateral lobar hyperlucency rather than diffuse bilateral changes. 163. A 46-year-old man with post-tuberculous lung destruction presents with sudden massive haemoptysis. Trans-femoral catheter angiography is planned for emergency embolisation. Which vessel should be catheterised and assessed first? A. Internal mammary artery B. Bronchial artery C. Pulmonary artery D. Intercostal artery E. Superior thoracic artery Explanation: (by Perplexity AI, generated with the OpenAI O3 model) • In >90% of massive haemoptysis, including that caused by tuberculosis, bleeding arises from the high-pressure systemic bronchial circulation rather than the low-pressure pulmonary arteries. • Bronchial artery embolisation is therefore the primary lifesaving procedure, and selective bronchial arteriography must be performed first to identify hypertrophied or ectopic bronchial branches for embolisation. • The pulmonary artery is an uncommon source; internal mammary, intercostal and superior thoracic arteries act as collateral feeders but are usually interrogated only if bronchial vessels do not account for the bleeding. 164. A 24-year-old man with cystic fibrosis attends clinic for routine assessment. High-resolution CT of the thorax shows widespread upper-lobe bronchiectasis but only mild lower-zone disease. Which pulmonary function parameter would most closely correlate with the extent of bronchiectasis seen on CT? A. Total lung capacity (TLC) B. Forced expiratory volume in 1 s (FEV₁) C. Diffusion capacity for carbon monoxide (DLCO) D. Peak expiratory flow rate (PEFR) E. Residual volume (RV) Explanation: (by Perplexity AI, generated with the OpenAI O3 model) • In cystic fibrosis, the severity of structural lung changes—especially bronchiectasis—demonstrated on HRCT correlates best with the degree of airflow obstruction, quantified by FEV₁. • Studies show a strong inverse relationship between CT bronchiectasis score and percent-predicted FEV₁ because dilated, inflamed airways cause expiratory airflow limitation. • TLC and RV reflect hyperinflation rather than airway damage, and DLCO usually remains near normal until advanced parenchymal disease. • PEFR is more effort-dependent and less sensitive to small-airway loss, so it correlates poorly with CT extent.
  • 267.
    EBDR Exam MCQs& Concepts May 2022 Dr. Kareem Alnakeeb 261 May 2022 1. A 35 year old with asthma presents with malaise, flu-like illness and cough. Previous similar episodes have occurred. A chest radiograph shows patchy airspace opacification in the mid and upper zones. Which feature on high-resolution CT would make allergic bronchopulmonary aspergillosis a more likely diagnosis than extrinsic allergic alveolitis? A. widespread centrilobular micronodules ,3 mm B. tubular finger-like opacities C. bronchiectasis D. upper-zone fibrosis E. pleural effusion Source: Bydder, Megan, et al. Get Through Final FRCR Part A: SBAs for the Modular Examination. 1st ed., Hodder Education Publishers, 2009. Explanation: • Allergic bronchopulmonary aspergillosis (ABPA) is hypersensitivity to aspergillus in people with asthma. Typical features are of a migratory pneumonitis, predominantly in the upper lobes. • It may cause bronchiectasis and upper-zone fibrosis, which are features also seen in extrinsic allergic alveolitis (EAA). • Tubular opacities, indicating mucus plugging, are seen in ABPA, but not in EAA. • Centrilobular nodules are seen in EAA, along with mosaic perfusion and patchy ground-glass change. • Pleural effusions are rarely seen in EAA and not in ABPA. 2. A 56-year-old male presents with wheezing, cough and recurrent chest infections. A chest radiograph shows right middle lobe consolidation. CT of the chest shows a 3 cm mass arising within the right middle lobe bronchus with distal collapse and consolidation. Which feature of the mass would make hamartoma more likely than carcinoid? A. central location B. presence of calcification C. cavitation D. presence of fat E. prominent enhancement Source: Bydder, Megan, et al. Get Through Final FRCR Part A: SBAs for the Modular Examination. 1st ed., Hodder Education Publishers, 2009. Explanation: • Hamartomas are usually seen in the periphery of the lungs (two-thirds) with 10% being endobronchial. Calcification is seen in 15%, often popcorn type. Cavitation is rare but fat is seen in 50%. • Carcinoids are usually located centrally and are endobronchial. Calcification is seen in one-third and they rarely cavitate. They do not contain fat and show prominent enhancement following contrast, as they are vascular.
  • 268.
    EBDR Exam MCQs& Concepts May 2022 Dr. Kareem Alnakeeb 262 3. An 80-year-old man presents with hemoptysis and a mass on chest radiograph. A biopsy shows non- small-cell lung cancer. CT of chest shows a 4 cm, right middle lobe mass with pleural tethering but no chest wall invasion. Lymph nodes are seen at the right hilum (17 mm short axis), in the subcarinal space (20 mm short axis) and in the aortopulmonary space (8 mm short axis). No other abnormalities are seen. What is the TNM stage? A. T2 N1 M0 B. T2 N2 M0 C. T2 N3 M0 D. T3 N1 M0 E. T3 N2 M0 Source: Bydder, Megan, et al. Get Through Final FRCR Part A: SBAs for the Modular Examination. 1st ed., Hodder Education Publishers, 2009. Explanation: • The T stage is T2, as the lesion is over 3cm but there is no chest wall or mediastinal invasion or other associated feature. • The nodes at the right hilum (N1) and in the subcarinal space (N2) are significantly enlarged, whereas the node in the aortopulmonary space (N3) is not (,10mm short axis), hence the N stage is N2. 4. In acute respiratory distress syndrome what is the first change usually seen on the chest radiograph? A. confluent consolidation B. pleural effusions C. increased heart size with globular shape D. volume loss with atelectasis E. patchy ill-defined opacities Source: Bydder, Megan, et al. Get Through Final FRCR Part A: SBAs for the Modular Examination. 1st ed., Hodder Education Publishers, 2009. Explanation: • Acute respiratory distress syndrome (ARDS) commences with interstitial oedema, progressing to congestion and extensive alveolar, and interstitial oedema and hemorrhage. • The chest radiograph is often normal for the first 24 hours, before patchy opacities appear in both lungs. • These progress to massive airspace consolidation over the following 24–48 hours. • True volume loss, atelectasis, cardiomegaly and effusions are not seen in ARDS. 5. A 16-year-old male presents with sudden shortness of breath. A chest radiograph shows multiple, bilateral nodules measuring up to 3 cm, some of which are calcified. There is a moderate left pneumothorax. The patient has been undergoing treatment for a malignant tumor. What is the most likely diagnosis? A. metastases secondary to Wilms’ tumor B. metastases secondary to osteosarcoma C. metastases secondary to testicular tumor D. abscesses secondary to immunosuppression E. varicella pneumonia secondary to immunosuppression
  • 269.
    EBDR Exam MCQs& Concepts May 2022 Dr. Kareem Alnakeeb 263 Source: Bydder, Megan, et al. Get Through Final FRCR Part A: SBAs for the Modular Examination. 1st ed., Hodder Education Publishers, 2009. Explanation: • Osteosarcoma pulmonary metastases are uncommon (seen in 2% of cases) and present as multiple masses which may calcify. There is a high incidence of associated pneumothorax. • Wilms’ tumors may also produce multiple pulmonary masses and may be associated with pneumothorax, but are not known to calcify. • Testicular tumors may produce calcified lung metastases, but are not associated with pneumothorax. • Varicella pneumonia shows patchy consolidation in the acute phase, with multiple, small, calcified nodules in the chronic phase. • Abscesses may present as multiple masses but rarely calcify and often cavitate. 6. A 56-year-old female patient presents with shortness of breath. A chest radiograph is unremarkable. A high-resolution CT scan is performed which shows mosaic perfusion with no air trapping on expiratory scan. What is the most likely diagnosis? A. bronchiolitis obliterans B. cystic fibrosis C. hypersensitivity pneumonitis D. chronic thromboembolic disease E. asthma Source: Bydder, Megan, et al. Get Through Final FRCR Part A: SBAs for the Modular Examination. 1st ed., Hodder Education Publishers, 2009. Explanation: • Mosaic perfusion is caused by abnormalities of ventilation, or vascular obstruction. • Expiratory scans help to distinguish causes by establishing whether there is air trapping. • With no air trapping present, pulmonary emboli of any cause are most likely. • Air trapping would suggest airway disease such as bronchiolitis obliterans, or other causes of small airway obstruction such as bronchiectasis or cystic fibrosis. 7. A known MS patient has presented to the neurologist with clinical features of involvement of the spinal cord. An MRI of the whole spine has been requested with a view towards assessment of the cord for possible multiple sclerosis (MS) plaques. MS lesions in the spinal cord occur most commonly in the A. Cervical segment. B. Thoracic segment. C. Lumbar segment. D. Sacral segment. E. All segments are equally affected. Source: Chin, Teck Yew, et al. Get Through Final FRCR 2A: SBAs. 1st ed., CRC Press, 2017. Explanation: Multiple Sclerosis Spinal Cord Lesions • MS can show multiple lesions in the spinal cord. • Typical spinal cord lesions in MS are relatively small and peripherally located. • They are most often found in the cervical cord and are usually less than two vertebral segments in length.
  • 270.
    EBDR Exam MCQs& Concepts May 2022 Dr. Kareem Alnakeeb 264 8. A 30-year-old male smoker presents with a history of acute dyspnea. CXR shows bilateral reticulo- nodular interstitial changes, predominantly in the upper and mid zones, with preservation of lung volume. There is a right sided apical pneumothorax and a small right pleural effusion. HRCT of chest shows complex thin- and thick-walled cysts and irregular centrilobular nodules in a similar distribution with sparing of the bases. The intervening lung appears normal. What is the diagnosis? A. Lymphangioleiomyomatosis. B. Bronchiectasis. C. Metastases. D. Pulmonary Langerhans cell histiocytosis. E. Idiopathic pulmonary fibrosis. Source: Lindsay, Richard, et al. SBAs for the FRCR Part 2A. 1st ed., Oxford University Press, 2012. Explanation: Overview and Symptoms • This is a rare isolated form of Langerhans cell histiocytosis that primarily affects young adult smokers. • Most patients are symptomatic and the most frequent symptoms are non-productive cough (50–70% of cases) and dyspnea (35–87%). • Less common symptoms include fatigue, weight loss, pleuritic chest pain, and fever. Chest X-Ray Findings • The most common finding on CXR is small irregular nodules, usually bilaterally symmetric, with upper lobe predominance and sparing of the costophrenic angles. • Coarse reticular and reticulo-nodular pattern is seen in later stages. Complications and Lung Volumes • Pneumothorax occurs in up to 25% (may be recurrent). • Pleural effusion is uncommon, but may occur with pneumothorax. • Lung volumes are normal or increased in most patients. High-Resolution CT Findings • HRCT of chest demonstrates the cysts and nodules in a characteristic distribution with normal intervening lung. • Interstitial fibrosis and honeycombing are seen in advanced stages. Treatment • Treatment consists of smoking cessation; steroids may be useful in selected patients. • Chemotherapeutic agents and lung transplantation may be offered in advanced disease. Prognosis • The prognosis is variable. • Stable disease is seen in up to 50%. • Spontaneous regression is reported in up to 25%. • A variably progressive, deteriorating course is seen in up to 25%.
  • 271.
    EBDR Exam MCQs& Concepts May 2022 Dr. Kareem Alnakeeb 265 9. A 75-year-old man presents with worsening shortness of breath. He was a mine worker. A chest radiograph shows multiple nodules in the upper zones with a large upper-zone mass on the left. CT confirms multiple small nodules up to 5 mm with a sausage-shaped mass paralleling the mediastinum. What is the most likely diagnosis? A. coal worker’s pneumoconiosis with bronchogenic carcinoma B. coal worker’s pneumoconiosis with progressive massive fibrosis C. tuberculosis D. primary lung carcinoma with metastases E. chronic extrinsic allergic alveolitis Source: Bydder, Megan, et al. Get Through Final FRCR Part A: SBAs for the Modular Examination. 1st ed., Hodder Education Publishers, 2009. Explanation: Occupational Lung Diseases • The history of mining (dust exposure) with small nodules in the upper zones is typical of pneumoconiosis. • The sausage-shaped mass is characteristic of progressive massive fibrosis (PMF), although malignancy cannot be excluded. PMF is often seen to have reduced nodularity surrounding it, as it incorporates the surrounding nodules and migrates towards the hilum. Infectious Conditions • Tuberculosis usually produces a generalized distribution of 2–3 mm nodules in its miliary form, often with lymphadenopathy. Neoplastic Processes • Metastases from primary lung cancer are not uniformly small (though they can be in thyroid cancer and melanoma). Hypersensitivity-Related Disorders • Chronic extrinsic allergic alveolitis produces fibrotic changes in the mid and lower zones. 10. A 67-year-old man has been rushed to the stroke unit with features of acute stroke. All of the following are true about acute stroke imaging, except A. CT source images correlate with infarct volume. B. Matched CBV (Cerebral blood volume) and CBF (Cerebral blood flow) represent salvageable brain. C. Diffusion-weighted MR imaging assesses the infarct core. D. Mismatch between PWI (Perfusion weighted imaging) and DWI (Diffusion weighted imaging) volumes represents salvageable brain. E. T2 shine through is seen as bright on DWI. Source: Chin, Teck Yew, et al. Get Through Final FRCR 2A: SBAs. 1st ed., CRC Press, 2017. Explanation: CT Perfusion and Angiography • An important advance in stroke imaging is the development of CT perfusion imaging. • CT angiography source images (CTA-SI) represent cerebral blood volume that is reduced in the core infarct and correlates with infarct volume as seen on DWI (Diffusion Weighted Imaging). Perfusion Parameters • CBF (cerebral blood flow), CBV (cerebral blood volume), and MTT (mean transit time) are three parameters that can distinguish infarcted tissue from potentially salvageable penumbra.
  • 272.
    EBDR Exam MCQs& Concepts May 2022 Dr. Kareem Alnakeeb 266 • Ischemic but non-infarcted tissue will have decreased CBF, elevated MTT, and normal or high CBV (mismatch). • Once infarcted, there will also be a persistent decrease in CBV (matched defect). Diffusion-Weighted Imaging (DWI) • Sensitivity and specificity of DWI for stroke detection is very high. • DWI bright signals do not necessarily represent irreversibly infarcted tissue but reflect redistribution of water from the extracellular to the intracellular space in ischemic tissue. • It is necessary to analyze maps of ADC (apparent diffusion coefficient) to distinguish the effects of reduced water diffusibility (dark on ADC) from T2 ‘shine-through’ (bright on ADC). • Both features lead to the DWI bright signals seen in ischemia. Perfusion–Diffusion Mismatch • The volumetric mismatch between the PWI and DWI volumes is a marker of potentially salvageable tissue at risk. Overall DWI provides the best estimate of infarcted core. 11. The absence of which of the following indicates a diagnosis of Dandy-Walker variant rather than Dandy- Walker malformation? A. Dysgenesis of the corpus callosum B. Holoprosencephaly C. Cerebellar heterotopia D. Enlargement of the pituitary fossa E. Cerebellar gyri malformation Source: Leen, Edward. Succeeding in the New FRCR Part 2A Exam: Single Best Answer (SBA) Revision Questions for Modules 1-6. 1st ed., UNKNO, 2011. Explanation: • The other features are common to both. 12. A CT brain of a 25-year-old male with a head injury but no focal neurology shows no acute abnormality. A thin cerebrospinal fluid (CSF) density is noted between the frontal horns of the lateral ventricles in the midline. Which is the diagnosis? A. Cavum septi pellucidi B. Cavum vergae C. Cavum veli interpositii D. Colloid cyst E. Arachnoid cyst Source: Leen, Edward. Succeeding in the New FRCR Part 2A Exam: Single Best Answer (SBA) Revision Questions for Modules 1-6. 1st ed., UNKNO, 2011. Explanation: • Occurs in 80% of term infants and 15% of adults. • Rarely may dilate and cause obstructive hydrocephalus.
  • 273.
    EBDR Exam MCQs& Concepts May 2022 Dr. Kareem Alnakeeb 267 13. MRI brain in a neonate shows multiple abnormalities including an anterior interhemispheric fissure adjoining a high riding third ventricle, an enlarged foramen of monro and a sunburst gyral pattern. An interhemispheric cyst is also seen. Which is the diagnosis? A. Arachnoid cyst B. Agenesis of the corpus callosum C. Prominent cavum septum pellucidi and vergae D. Chiari II malformation E. Dandy-Walker malformation Source: Leen, Edward. Succeeding in the New FRCR Part 2A Exam: Single Best Answer (SBA) Revision Questions for Modules 1-6. 1st ed., UNKNO, 2011. Explanation: • Absence of septum pellucidum, corpus callosum and cavum septum pellucidi, and wide separation of the lateral ventricles, are other features of agenesis of the corpus callosum. 14. A 50-year-old man who presented with progressive dyspnea had a chest radiograph that demonstrated multiple opacities between 0.5 and 2 mm in size, which were noted to be more dense than soft tissue. Which of the following diagnoses is most likely? A. Fungal infection such as histoplasmosis B. Coal miners' pneumoconiosis C. Sarcoidosis D. Acute extrinsic allergic alveolitis E. Silicosis Source: Proctor, Robin. Final FRCR Part A Modules 1–3 Single Best Answer MCQs: The SRT Collection of 600 Questions with Explanatory Answers (MasterPass). 1st ed., CRC Press, 2009. Explanation: • All the other conditions would produce nodules of soft tissue density. • In pure silicosis the nodules are very well defined and very dense. • There is also relative sparing of the bases and apices with septal lines on HRCT. 15. A 45-year-old female presented with progressive dyspnoea. A chest radiograph demonstrated reticular densities with preserved lung volumes. An HRCT showed uniform cysts throughout the lung with normal intervening lung. What is the most likely diagnosis? A. Tuberous sclerosis B. Lymphangiomyomatosis C. Histiocytosis D. Tuberculosis E. Cystic fibrosis Source: Proctor, Robin. Final FRCR Part A Modules 1–3 Single Best Answer MCQs: The SRT Collection of 600 Questions with Explanatory Answers (MasterPass). 1st ed., CRC Press, 2009. Explanation: • LAM is seen exclusively in women, typically those of child-bearing age. • The typical appearances are of thin-walled cysts with normal intervening lung. • There may also be small pneumothoraces and chylous effusions.
  • 274.
    EBDR Exam MCQs& Concepts May 2022 Dr. Kareem Alnakeeb 268 16. You have been asked to review a chest radiograph by a junior doctor. The image demonstrates subtle hazy opacification of the upper part of the lower zone of the right lung. The right atrial border is indistinct and the horizontal fissure runs from the right hilum to the eighth rib in the mid axillary line. What is the most plausible explanation for these findings? F. Middle lobe collapse G. Middle lobe consolidation H. Pectus excavatum I. Right lower lobe mediobasal segment consolidation J. Right lower lobe anteriobasal segment consolidation Source: Proctor, Robin. Final FRCR Part A Modules 1–3 Single Best Answer MCQs: The SRT Collection of 600 Questions with Explanatory Answers (MasterPass). 1st ed., CRC Press, 2009. Explanation: • The findings are those of middle lobe collapse. • Signs on the frontal radiograph can be subtle, and it is more easily seen on the lateral radiograph. • In this case the loss of clarity of the right atrial border indicates the pathology is located in the middle lobe. • There is loss of volume (the normal horizontal fissure runs from the hilum to the sixth rib in the mid axillary line), therefore collapse of the middle lobe, rather than consolidation, is the likely cause for these appearances. 17. You are asked to review a chest radiograph following a pacemaker insertion. The leads have been placed via the left subclavian approach and pass down the left mediastinal border before forming a loop with the tip projected over the right ventricle. What is the most likely explanation? A. Partial anomalous pulmonary venous return B. An atrial septal defect C. A persistent left superior vena cava D. A ventricular septal defect E. Normal appearance, no abnormality Source: Proctor, Robin. Final FRCR Part A Modules 1–3 Single Best Answer MCQs: The SRT Collection of 600 Questions with Explanatory Answers (MasterPass). 1st ed., CRC Press, 2009. Explanation: • A persistent left superior vena cava (SVC) courses along the left mediastinal border and enters the coronary sinus, which is usually dilated. There is more often than not a right SVC as well. 18. A 64-year-old non-smoker presents to his GP with progressive dyspnea. His chest radiograph demonstrates a peripheral lung mass. What is the most likely histological type of carcinoma? A. Squamous cell carcinoma B. Small cell lung carcinoma C. Bronchoalveolar cell carcinoma D. Large cell carcinoma E. Adenocarcinoma Source: Proctor, Robin. Final FRCR Part A Modules 1–3 Single Best Answer MCQs: The SRT Collection of 600 Questions with Explanatory Answers (MasterPass). 1st ed., CRC Press, 2009. Explanation: • This is simply a question of incidence. It is the most common type of lung carcinoma and is also the most common in non-smokers.
  • 275.
    EBDR Exam MCQs& Concepts May 2022 Dr. Kareem Alnakeeb 269 19. A 62 year old man presents with right shoulder pain which radiates down his arm. A plain radiograph confirms the presence of a right apical mass with destruction of the surrounding ribs. CT-guided biopsy is performed and is likely to reveal: F. Large cell lung cancer G. Squamous cell cancer H. Small cell lung cancer I. Adenocarcinoma J. Carcinoid Source: Currie, Stuart, et al. SBAs for the FRCR 2A. 1st ed., Cambridge University Press, 2010. Explanation: • The case describes a Pancoast tumor for which squamous is the most common cell type. 20. A 60 year old female underwent a right pneumonectomy for bronchogenic carcinoma. Which feature on plain chest radiograph would be a cause of worry seven days after surgery? A. A sequential increase in the fluid level B. Shift of the previously central trachea to the right C. Shift of the previously central trachea to the left D. Elevation of the right hemi-diaphragm E. Shift of the cardiac silhouette to the right Source: Currie, Stuart, et al. SBAs for the FRCR 2A. 1st ed., Cambridge University Press, 2010. Explanation: • All the other changes are expected changes at this stage following a pneumonectomy. • However, contralateral shift of the trachea may be indicative of a post-surgical bronchopleural fistula. 21. A 52-year-old woman is involved in a RTA. Trauma series radiographs reveal a complex pelvic fracture. There are no other appreciable injuries. She is hemodynamically unstable, and fluid resuscitation is commenced. Which of the following steps is the next most important? A. Trauma protocol CT scan with angiographic sequences B. Immediate surgery under the orthopedic surgeons C. Discussion with interventional radiologists with view to catheter angiography and possible intervention D. Placement of a pelvic wrap device E. Blood transfusion Source: Chin, Teck Yew, et al. Get Through Final FRCR 2A: SBAs. 1st ed., CRC Press, 2017. Explanation: Pelvic Wrap Device • All of these steps may be required, but the most important is to place a pelvic wrap device. • The purpose of this device is to stabilize the pelvis. Hemorrhage Management • If the patient is hypotensive as a result of venous bleeding, a pelvic wrap should stabilize the pelvis sufficiently to cause significant reduction or cessation in the venous hemorrhage and thus avoid unnecessary endovascular intervention. • If this fails to achieve sufficiently prompt hemodynamic stability, there is a significant chance that there is arterial hemorrhage; hence endovascular intervention is likely to be necessary. Imaging and Endovascular Planning
  • 276.
    EBDR Exam MCQs& Concepts May 2022 Dr. Kareem Alnakeeb 270 • Many centers use angiographic sequences as part of the trauma CT scan to identify such a bleeding point as part of planning stage of endovascular management. • CT would be the next step once the pelvic binder is in situ. 22. A 30-year-old teacher presents with thyroid swelling and proptosis. Which of the following is true regarding Grave’s ophthalmopathy? A. It commonly involves the medial rectus first. B. It commonly involves the tendons of the eye muscles. C. There is associated dilatation of the superior ophthalmic vein. D. There is decreased density of the orbital fat. E. It is an autoimmune disease unrelated to thyroid function. Source: Chin, Teck Yew, et al. Get Through Final FRCR 2A: SBAs. 1st ed., CRC Press, 2017. Explanation: Overview • Graves’ disease of the orbit is also known as thyroid ophthalmopathy or endocrine exophthalmos. • It is produced by long-acting thyroid-stimulating hormone, probably due to cross-reactivity against antigens shared by thyroid and orbital tissue. • Signs and symptoms usually develop within 1 year of hyperparathyroidism. • It is the most common cause of unilateral/bilateral proptosis in adults. Extraocular Muscle Involvement • It commonly involves the inferior rectus first (mnemonic – I’M SLOW). o Inferior rectus o Medial rectus o Superior rectus o Lateral rectus o Obliques Orbital and Vascular Changes • The superior ophthalmic vein is dilated due to compromised orbital venous drainage at the orbital apex. • There is increased density of the orbital fat late in the disease. 23. A 33-year-old asymptomatic patient has been found to have cervical carcinoma and is undergoing a staging MRI examination. What finding on MRI is most reliable in excluding parametrial invasion? A. Absence of abnormal pelvic lymphadenopathy B. No hydronephrosis or hydroureter on either side C. Tumor volume of less than 90 cu cm D. Hypointense rim of cervical stroma E. Fine nodularity of parametrial tissue Source: Proctor, Robin. Final FRCR Part A Modules 4-6 Single Best Answer MCQs: The SRT Collection of 600 Questions with Explanatory Answers (MasterPass). 1st ed., CRC Press, 2009. Explanation: • A continuous hypointense rim representing the cervical ring measuring >3 mm is most reliable in excluding parametrial invasion with a quoted specificity of 96-99%. • Nodularity and thickening of the parametrial tissue are signs of frank invasion.
  • 277.
    EBDR Exam MCQs& Concepts May 2022 Dr. Kareem Alnakeeb 271 24. Which structure lies immediately posterior to the standard fenestration site during an endoscopic third ventriculostomy for aqueductal stenosis? A. Basilar artery apex B. Optic chiasm C. Anterior commissure D. Fornix columns E. Pineal gland Explanation: (by Perplexity AI, generated with the OpenAI O3 model) • The stoma is fashioned in the floor of the third ventricle just anterior to the paired mammillary bodies; the interpeduncular cistern—and crucially the basilar artery apex with its perforating branches—lies directly beneath this thin ventricular floor. Awareness of this vascular landmark prevents catastrophic hemorrhage. • The optic chiasm (B) sits anterior-superior to the fenestration site, the anterior commissure (C) is rostral within the third ventricle roof, the fornix columns (D) run in the anterior roof, and the pineal gland (E) is posterior to the aqueduct, not beneath the floor fenestration • Third ventriculostomy: o type of surgical treatment for obstructive hydrocephalus, especially when obstruction is located at the level of the aqueduct of Sylvius (e.g. aqueduct stenosis). o A permanent defect is created in the floor of the third ventricle anterior to the mammillary bodies, thus connecting the third ventricle with the interpeduncular cistern. 25. The gold standard diagnostic modality to evaluate the coronary arteries grafts is A. Percutaneous coronary angiography B. Echocardiography C. CT coronary angiography D. MR Coronary angiography Explanation: (by Perplexity AI, generated with the OpenAI O3 model) • Percutaneous (invasive) coronary angiography – it remains the gold-standard test for assessing the patency and anatomy of coronary artery bypass grafts 26. The reason that coronary bypass grafts are superior to native coronary arteries for CТА is: A. They are less influenced by cardiac motion B. They have a wider luminal diameter C. They contain less luminal calcification D. They follow a straighter course E. All of the above Explanation: (by Perplexity AI, generated with the OpenAI O3 model) • Bypass grafts run in the mediastinum, away from the rapidly moving epicardial surface, so motion artefact is markedly reduced. Their surgical caliber (≈4–6 mm) is larger than that of native coronary vessels (≈2–4 mm), making intraluminal contrast opacification more homogeneous and partial- volume artefact less problematic. • In addition, graft walls rarely develop the heavy, nodular calcification typical of long-standing atherosclerotic coronary arteries, improving luminal visualization. • While each individual feature aids image quality, the combination of reduced motion, larger caliber and lesser calcification together accounts for the consistently higher diagnostic accuracy of CT graft studies, making “all of the above” the single best answer.
  • 278.
    EBDR Exam MCQs& Concepts May 2022 Dr. Kareem Alnakeeb 272 • Key distractors: options A–C are each true but incomplete explanations; option D is plausible yet less impactful than the three principal factors above. 27. When performing CT coronary angiography to evaluate stent patency, which technical adjustment is most effective at reducing blooming artefacts from the metallic stent struts? A. Increase the tube voltage to 140 kV B. Reconstruct images with a sharp convolution kernel C. Acquire the study with retrospective ECG gating D. Use prospective ECG-triggered high-pitch acquisition E. Reconstruct slices at 1.5 mm thickness Explanation: (by Perplexity AI, generated with the OpenAI O3 model) • Blooming arises mainly from partial-volume averaging and beam-hardening around the highly attenuating stent struts. Applying a high-spatial-frequency (sharp) reconstruction kernel narrows the point-spread function, improving edge definition and markedly reducing apparent strut thickness, so the true lumen is better visualized. • Raising kV (Option A) increases photon penetration but does little for spatial resolution. • Retrospective (Option C) and prospective (Option D) ECG strategies influence motion artefact and radiation dose, not blooming. • Thicker slices (Option E) exacerbate partial-volume effects and increase blooming, doing the opposite of what is desired. 28. The specific technical issue in CT imaging of coronary stents is: A. Blooming artifacts due to beam hardening B. Respiratory motion artifacts C. Cardiac motion artifacts, D. The use of high-density contrast material Explanation: (by Perplexity AI, generated with the OpenAI O3 model) • CT assessment of the stent lumen is primarily hampered by blooming artifacts—an apparent enlargement of the metallic struts produced by partial-volume effects and beam-hardening, which obscures the true lumen diameter and can mimic or hide in-stent restenosis. 29. A young patient is followed up for a fractured tibia at the outpatient clinic. A repeat radiograph is acquired, which shows abnormal healing and callus formation at the fracture site. All the following are possible causes, except A. Cushing’s syndrome B. Osteogenesis imperfecta C. Osteopoikilosis D. Paralytic state E. Asthmatic on steroids Source: Chin, Teck Yew, et al. Get Through Final FRCR 2A: SBAs. 1st ed., CRC Press, 2017. Explanation: Causes of Impaired Fracture Healing • Patients with comorbidities like diabetes, anemia and malnutrition can suffer from impaired bone fracture healing. • Drug therapy like corticosteroids and NSAIDs can also produce similar problems. Osteogenesis Imperfecta • Osteogenesis imperfecta is a connective-tissue disorder with resultant abnormal bone density and structure, resulting in poor mineralization and fragile, brittle bones.
  • 279.
    EBDR Exam MCQs& Concepts May 2022 Dr. Kareem Alnakeeb 273 Osteopoikilosis • Osteopoikilosis is a benign condition and usually found incidentally. • It is a form of sclerosing bone dysplasia with multiple enostoses. • It is not associated with impaired fracture healing. 30. A male infant is born at 39 + 3 weeks gestation. Prenatal ultrasound demonstrated a partly cystic, partly echogenic mass in the right upper lobe. Shortly after delivery the infant is in respiratory distress. Initial chest X-ray demonstrates dense lungs bilaterally with increased volume on the right. On Day 2, a repeat chest X-ray demonstrates multiple air-filled cystic masses of varying sizes within the right upper lobe with mediastinal shift to the left. What is the most likely diagnosis? A. Bronchogenic cyst B. Morgagni hernia C. Congenital cystic adenomatoid malformation D. Congenital lobar emphysema E. Hyaline membrane disease Source: Chin, Teck Yew, et al. Get Through Final FRCR 2A: SBAs. 1st ed., CRC Press, 2017. Explanation: Definition • Congenital cystic adenomatoid malformation is a developmental hamartomatous abnormality of lung with adenomatoid proliferation of cysts resembling bronchioles. Pathogenesis • It is thought to be caused by focal arrest in fetal lung development before the seventh week of gestation. Incidence • Congenital cystic adenomatoid malformation represents 25% of all congenital lung lesions. Postnatal Chest X-ray • A CXR on Day 1 of life usually demonstrates dense lungs with increased volume on the affected side. • On Day 2, a CXR usually demonstrates resorption of fluid from affected areas of lung, which are then replaced with air-containing spaces. Anatomical Characteristics • Communication with the tracheobronchial tree is maintained and the vascular supply and drainage are to the pulmonary circulation. • There is a slight predilection for the upper lobes. Clinical Presentation • Newborns often present with respiratory distress secondary to mass effect and pulmonary compression or hypoplasia. • The chest is dull to percussion with decreased air entry. Prenatal Ultrasound Findings • Prenatal ultrasound shows a partly cystic, partly echogenic mass. 31. A 74-year-old man presents with neck pain, with right upper-arm pain and radicular symptoms at the lateral aspect of the forearm and tingling in the thumb. What is the most likely finding on the MRI? A. Central disc bulge at C3/4 with severe cord compression B. Right foraminal disc osteophyte at C2/3 C. Right foraminal disc osteophyte at C4/5 D. Right foraminal disc osteophyte at C5/6 E. Right foraminal disc osteophyte at C3/4
  • 280.
    EBDR Exam MCQs& Concepts May 2022 Dr. Kareem Alnakeeb 274 Source: Chin, Teck Yew, et al. Get Through Final FRCR 2A: SBAs. 1st ed., CRC Press, 2017. Explanation: Dermatomes and Nerve Root Impingement • Lateral aspect of the forearm and the thumb corresponds to the C6 dermatome. • Foraminal osteophyte at C5/6 will impinge upon the exiting C6 nerve root. • (cf. foraminal osteophyte at a thoracic or lumbar level, e.g., T4/5 or L4/5, which will impinge upon the exiting T4 or L4 nerve roots, subject to the discrepancy between number of cervical vertebrae and cervical roots. Note that the exiting root at C7/T1 is C8.) 32. A 35-year-old man involved in a major RTA undergoes a lateral view of the cervical spine in the resus on arrival. All of the following are features associated with atlanto-occipital dislocation, except F. Soft-tissue swelling anterior to C2 by >10 mm. G. Basion dens interval >12 mm. H. Odd’s ratio >1. I. X-ray can often be normal. J. Incongruity of articular surface of atlas and occipital condyles. Source: Chin, Teck Yew, et al. Get Through Final FRCR 2A: SBAs. 1st ed., CRC Press, 2017. Explanation: Lateral Radiograph Findings • Atlanto-occipital dislocation shows the following on lateral radiograph of the cervical spine: • 10 mm soft-tissue swelling anterior to C2, with pathological convexity (80%), • basion-dens interval of >12 mm, • odds ratio (distance between the basion and the posterior arch of the atlas divided by opisthion and anterior arch of atlas) >1, and • basion–posterior axial line interval >12 mm anterior/>4 mm posterior to axial line. Direct Radiographic Signs • Direct signs include loss of congruity of articular surfaces of atlas and occipital condyle.
  • 281.
    EBDR Exam MCQs& Concepts May 2022 Dr. Kareem Alnakeeb 275 Diagnostic Caveat • Normal X-ray in the presence of atlanto-occipital dislocation is rare. 33. Chest X-ray of a boy shows shift of the heart and mediastinum to the right. There is also a tubular structure parallel to the right heart border with its maximum width close to the diaphragm. The finding suggests A. ASD B. Scimitar syndrome C. Total anomalous pulmonary venous return D. Intralobar sequestration E. Inhaled foreign body Source: Chin, Teck Yew, et al. Get Through Final FRCR 2A: SBAs. 1st ed., CRC Press, 2017. Explanation: Overview • Hypogenetic lung syndrome, also known as congenital venolobar syndrome or scimitar syndrome, is primarily a complex developmental lung abnormality with anomalous venous return. Key Features • The most common features are lung hypoplasia, anomalous pulmonary venous return to IVC, pulmonary artery hypoplasia, bronchial anomalies and systemic arterial supply to hypoplastic lung. Epidemiology • It almost always occurs on the right side and is slightly more common in women. Venous Drainage Patterns • One constant component of this syndrome is an anomalous pulmonary vein or veins draining at least a part or the entire affected lung most commonly to the inferior vena cava just above or below the diaphragm. • Uncommonly, the anomalous vein may drain into hepatic, portal, azygos veins; the coronary sinus; or the right atrium. Imaging Findings • A scimitar vein is a vertical curvilinear opacity in the right mid-lower lung, running along the right heart border inferomedially towards the diaphragm to join the IVC. A scimitar vein present on a frontal chest radiograph is called the scimitar sign. 34. A 67-year-old woman with 5.5 cm atherosclerotic abdominal aortic aneurysm is being worked up for a potential aortic endograft repair. All of the following are important imaging observations to be determined prior to treatment, except A. Tortuosity of the aorta B. Diameter of aortic aneurysms C. Non-thrombosed residual lumen of the aneurysm D. Flow characteristic at the aneurysm neck E. Length of proximal and distal landing zones Source: Chin, Teck Yew, et al. Get Through Final FRCR 2A: SBAs. 1st ed., CRC Press, 2017. Explanation: Aneurysm Definition and Repair Thresholds • An aneurysm occurs when a vessel diameter exceeds 1.5 times its normal size. In the abdomen this corresponds to 3 cm. • These aneurysms should be repaired when the diameter exceeds 5–5.5 cm or the aneurysm expands more than 1 cm per year.
  • 282.
    EBDR Exam MCQs& Concepts May 2022 Dr. Kareem Alnakeeb 276 • Study results suggest higher complications for aneurysms larger than 5.5–6.5 cm. The shape can be described as saccular or fusiform. The residual lumen through the aneurysm should measure approximately 18 mm to allow passage and proper deployment of the device. Imaging Considerations • The greatest benefit of 3D volume-rendered imaging is the depiction and precise measurement of angulation in aneurysms with marked tortuosity. Proximal Landing Zone (Neck) • The proximal landing zone consists of the region from the inferior-most renal artery to the beginning of the aneurysm. • The maximal acceptable neck diameter is 32 mm. The length of the neck should be at least 15 mm (although one device allows a 7 mm neck). • The angle between the superior portion of the aneurysm neck and the suprarenal aorta is preferably less than 60°. Distal Landing Zone (Iliac) • The preferred distal landing zone is the common iliac artery. • Evaluation is similar to that of the proximal neck with assessment of diameter, length, tortuosity and degree of calcification and thrombus. • The common iliac artery diameter should not be larger than 25 mm, and at least 10 mm of length is required for an adequate seal. 35. A 72-year-old man with a known malignancy undergoes a spinal MRI for characterization of multilevel vertebral collapse identified on plain radiograph. All of the following are true about malignant vs osteoporotic causes of vertebral fractures, except A. The involved vertebra are low on T1W images. B. Paraspinal mass is useful in differentiating metastatic from benign fracture. C. DWI can differentiate between malignant and benign compression fracture. D. Posterior bulging of collapsed vertebral body suggests metastasis. E. Acute osteoporotic fractures show intense enhancement post-contrast. Source: Chin, Teck Yew, et al. Get Through Final FRCR 2A: SBAs. 1st ed., CRC Press, 2017. Explanation: MR Imaging Features for Differentiating Metastatic and Acute Osteoporotic Compression Fractures • Distinction between metastatic and acute osteoporotic compression fractures could be made on the basis of MR imaging findings. • A convex posterior border of the vertebral body is more frequent in metastatic compression fractures than acute osteoporotic compression fractures. • A higher frequency of abnormal signal intensity of the pedicle or posterior element has been observed in metastatic compression fractures. • Epidural soft-tissue mass is suggestive of malignant vertebral collapse. • A paraspinal mass is not helpful in differentiation of the cause of vertebral collapse but is more commonly encountered in the setting of metastatic compression, where it is typically focal rather than diffuse. • Signal intensity abnormalities in the marrow of vertebrae other than the collapsed vertebrae are more frequently seen in metastatic compression fractures than acute osteoporotic compression fractures. • Enhancement on post-contrast T1-weighted FS images is not useful in differentiation of acute osteoporotic fractures from malignant compression fracture, but it may be useful for old or chronic fractures, which will not show intense enhancement.
  • 283.
    EBDR Exam MCQs& Concepts May 2022 Dr. Kareem Alnakeeb 277 • Moreover, on diffusion-weighted imaging (DWI) vertebral metastases with compression fractures can be safely distinguished from vertebra with benign compression fractures based on significantly different ADC values. 36. A 4-year-old child presents with short stature and failure to grow. Plain radiographs reveal multiple abnormalities, including generalized increased density of long bones with thickened cortices, widened cranial sutures, Wormian bones, a hypoplastic mandible and shortened pointed distal phalanges. Which of the following is the most likely diagnosis? A. Pyknodysostosis B. Osteopetrosis C. Cleidocranial dysostosis D. Osteosclerosis E. Kinky hair syndrome Source: Chin, Teck Yew, et al. Get Through Final FRCR 2A: SBAs. 1st ed., CRC Press, 2017. Explanation: • Overview o Pyknodysostosis is a congenital abnormality that should be considered in the differential diagnosis of osteosclerosis. • Clinical Features o The patients are typically short, have hypoplastic mandibles, widened cranial sutures, Wormian bones, brachycephaly, clavicular dysplasia, thick skull base and hypoplasia or nonpneumatisation of the paranasal sinuses. • Distinguishing Radiologic Features o The distinguishing feature is acro-osteolysis with sclerosis. o The distal phalanges appear as if they have been put in a pencil sharpener – they are pointed and dense. 37. The causes of medullary nephrocalcinosis include all, except A. Hyperparathyroidism B. Renal tubular acidosis C. Medullary sponge kidney D. Hypervitaminosis D E. Alport’s syndrome Source: Chin, Teck Yew, et al. Get Through Final FRCR 2A: SBAs. 1st ed., CRC Press, 2017. Explanation: • Medullary Nephrocalcinosis o Causes of medullary nephrocalcinosis include hyperparathyroidism, sarcoidosis, myelomatosis, primary or secondary hyperoxaluria (Crohn’s disease), hyperthyroidism, osteoporosis, idiopathic hypercalciuria, renal tubular acidosis, medullary sponge kidney and drug-induced (hypervitaminosis D, milk-alkali syndrome). • Alport’s Syndrome o Alport’s syndrome is an autosomal dominant condition also called chronic hereditary nephritis, associated with ocular abnormalities, deafness, small kidneys, cortical calcification and progressive renal failure without hypertension.
  • 284.
    EBDR Exam MCQs& Concepts May 2022 Dr. Kareem Alnakeeb 278 38. A 66-year-old joiner presents to his GP with jaundice and abdominal discomfort. He was subsequently referred to a gastroenterologist who requests a liver biopsy due to deranged liver function tests. Which of the following options is not a contraindication for percutaneous liver biopsy? A. INR above 1.6 B. Platelets less than 60,000/mm3 C. Tense ascites D. Extra-hepatic biliary obstruction E. Suspected haemangioma Source: Chin, Teck Yew, et al. Get Through Final FRCR 2A: SBAs. 1st ed., CRC Press, 2017. Explanation: • Contraindications for liver biopsy include the following: 1. Uncooperative patient 2. Extrahepatic biliary duct dilatation (except if benefit outweighs the risk) 3. Bacterial cholangitis (relative contraindication due to risk of septic shock) 4. Abnormal coagulation indices (having a normal INR or PT is not a reassurance that the patient will not bleed; however, there is increased incidence of bleeding with INR above 1.5) 5. Presence of ascites 6. Cystic lesion 39. 57. A 40-year-old man who is a known hypothyroid patient, presents with weight loss and dull pain in the flank and back. He undergoes an abdominal CT. Regarding retroperitoneal fibrosis, all of the following is seen on imaging, except A. Medial deviation of the ureters in the middle third, typically bilateral. B. CT shows soft-tissue mass displacing the aorta anteriorly. C. T2W MRI shows variable signal. D. PET CT has high sensitivity. E. Hydronephrosis is evident on CT urogram. Source: Chin, Teck Yew, et al. Get Through Final FRCR 2A: SBAs. 1st ed., CRC Press, 2017. Explanation: Intravenous Urography • Intravenous urography usually demonstrates the classic triad of medial deviation of the middle third of the ureters, tapering of the lumen of one or both ureters in the lower lumbar spine or upper sacral region, and proximal unilateral or bilateral hydroureteronephrosis with delayed excretion of contrast material. CT and MRI Diagnosis • CT and MRI is the mainstay of non-invasive diagnosis of Retroperitoneal fibrosis (RPF). • CT Imaging Features • CT allows comprehensive evaluation of the morphology, location and extent of RPF and involvement of adjacent organs and vascular structures. • Moreover, abdominal CT allows detection of diseases often associated with idiopathic RPF (e.g., autoimmune pancreatitis) or demonstrating an underlying cause in cases of secondary RPF (e.g., malignancy). • CT shows a well-defined mass, usually anterior and lateral to the aorta, sparing the posterior aspect and not causing aortic displacement. MRI Characteristics • Idiopathic RPF typically has low signal intensity on T1-weighted images.
  • 285.
    EBDR Exam MCQs& Concepts May 2022 Dr. Kareem Alnakeeb 279 • The signal intensity on T2-weighted images is variable and reflects the degree of associated active inflammation (hypercellularity and oedema). Contrast Enhancement and PET • After administration of contrast material, early soft-tissue enhancement mirrors the degree of inflammatory activity observed at T2-weighted imaging. • The sensitivity of 18F-FDG PET is very high, which allows detection and quantification of the metabolic activity of retroperitoneal lesions. • Although sensitivity is high, specificity is low and aortic wall in the elderly can show FDG uptake. 40. A 9-year-old girl was taken to her family doctor with fever and painful knee and wrists. The GP noticed a skin rash, hepatosplenomegaly and lymphadenopathy. Plain X-ray of the knee and wrist shows expansion of bones around the knee and advanced carpometacarpal arthritis. What is the likely diagnosis? A. Still disease B. Hemophilia C. Sickle-cell disease D. Psoriasis E. Lyme disease Source: Chin, Teck Yew, et al. Get Through Final FRCR 2A: SBAs. 1st ed., CRC Press, 2017. Explanation: • Still Disease (Polyarticular Juvenile Rheumatoid Arthritis) o Still disease is a clinical manifestation of polyarticular juvenile rheumatoid arthritis characterized by fever, rash, hepatosplenomegaly and pericarditis. o There is periosteal reaction of the hand phalanges and broadening of bones with cortical thickening. • Radiological and Clinical Compatibility o The presence of advanced arthropathy in the hands at such a young age along with the other clinical findings would be compatible with this condition. • Lyme Disease Comparison o Lyme disease tends to follow a monoarticular pattern with involvement of the large joints, usually the knee, with the radiological findings not as profound as that of juvenile rheumatoid arthritis. 41. A 40-year old with recurrent pulmonary emboli is due to have a hip replacement and it is decided to deploy a temporary inferior vena cava (IVC) filter. What is the preferred site of deployment of an IVC filter? A. Suprarenal IVC B. Infrarenal IVC C. Proximal to the clot load, no matter the level D. At the confluence of common iliac vessels E. At the junction of IVC and right atrium Source: Chin, Teck Yew, et al. Get Through Final FRCR 2A: SBAs. 1st ed., CRC Press, 2017. Explanation: • Placement Considerations o In the presence of normal anatomy, IVC filters should be placed inferior to the renal veins. o If the patient has aberrant anatomy such as double IVC, a single suprarenal filter or twin IVC filters can be placed. • Rationale o This is to avoid potential clot propagation and renal vein thrombosis.
  • 286.
    EBDR Exam MCQs& Concepts May 2022 Dr. Kareem Alnakeeb 280 • Access and Imaging o Access is via a right internal jugular puncture or a right femoral vein puncture. o A cavogram is performed to visualize the renal veins and to look for aberrant anatomy as described above. • Indications o Indications for IVC filter include deep vein thrombosis where anticoagulation is contraindicated, where the patient is non-compliant with medical treatment or when there is free-floating thrombus in the IVC. • Device Management o Retrievable devices are available and should be removed within 14 days from insertion. • Complications o Pulmonary embolism can still occur despite an IVC filter with an incidence of 2.7%–4%. 42. Chest radiograph of a 12-year-old boy shows a cystic lesion with air–fluid level in the right upper lobe. CT scan confirms the presence of a thin-walled cystic lesion. Rest of the lungs are clear. There is no lymphadenopathy in the chest. Quantiferon test was negative, and there are no features of infection or signs of inflammation. What is the diagnosis? A. TB B. Intrapulmonary bronchogenic cyst C. Hydatid cyst D. Infected bulla E. Congenital lobar emphysema Source: Chin, Teck Yew, et al. Get Through Final FRCR 2A: SBAs. 1st ed., CRC Press, 2017. Explanation: Location and Prevalence • Bronchogenic cysts (BCs) are congenital lesions. • They are usually found in the mediastinum or pulmonary parenchyma and, less commonly, cysts may be found in the neck, pericardium, pleura, diaphragm or abdominal cavity. • Intrapulmonary cysts are most common in the lower lobes. Imaging Characteristics • Intrapulmonary BCs are usually sharply defined, solitary, non-calcified, round or oval opacities confined to a single lobe. • These can present as a homogeneous water density, an air-filled cyst, or with an air–fluid level. • Signal on MRI depends on the content, and fluid-containing lesions are low on T1-weighted and high on T2-weighted images; however, proteinaceous content makes them high on T1-weighted imaging. Differential Diagnosis • The differential diagnosis of intraparenchymal BCs must include acquired cystic lesions, such as a lung abscess, a hydatid cyst, infection with Nocardia, an infected bulla, congenital lobar emphysema, fungal diseases and tuberculosis, especially when the lesions manifest as air-filled or have an air–fluid level.
  • 287.
    EBDR Exam MCQs& Concepts May 2022 Dr. Kareem Alnakeeb 281 43. A 35-year-old non-smoking woman presents with progressive exertional dyspnea, and high-resolution CT of the chest shows innumerable thin-walled cysts of varying size diffusely involving both lungs with normal intervening parenchyma; which single diagnosis best explains these findings? A. Pulmonary Langerhans cell histiocytosis B. Lymphangioleiomyomatosis C. Centrilobular emphysema D. Idiopathic pulmonary fibrosis E. Pulmonary sarcoidosis Explanation: (by Perplexity AI, generated with the OpenAI O3 model) • Lymphangioleiomyomatosis (LAM) classically affects women of child-bearing age and produces uniform, thin-walled cysts scattered throughout all lung zones while preserving the background lung architecture, matching the vignette. • Langerhans cell histiocytosis usually affects young smokers and shows irregular cysts and nodules predominating in the upper and mid-zones, not a uniform diffuse pattern. • Emphysema creates areas of low attenuation without a visible wall and is centrilobular or paraseptal rather than truly cystic. • Idiopathic pulmonary fibrosis demonstrates basal-predominant reticulation, honeycombing and traction bronchiectasis, not isolated cysts. • Sarcoidosis typically has perilymphatic nodules and fibrosis; diffuse thin-walled cysts are uncommon, making LAM the most appropriate choice. 44. A 29-year-old heavy smoker presents with persistent dry cough. High-resolution CT demonstrates numerous irregular lung cysts mixed with small centrilobular nodules, predominantly in the upper and mid-zones with relative sparing of the costophrenic angles. Which single diagnosis best explains this pattern? A. Lymphangioleiomyomatosis B. Pulmonary Langerhans cell histiocytosis C. Centrilobular emphysema D. Metastatic adenocarcinoma E. Idiopathic pulmonary fibrosis Explanation: (by Perplexity AI, generated with the OpenAI O3 model) • Pulmonary Langerhans cell histiocytosis (PLCH) characteristically affects young adult smokers and shows a combination of centrilobular nodules and bizarre-shaped cysts that favor the upper/mid-lungs while sparing the bases, exactly as in this vignette. • Lymphangioleiomyomatosis produces uniformly thin-walled round cysts distributed diffusely without nodules and almost exclusively affects women, so option A is unlikely. • Centrilobular emphysema causes areas of low attenuation without definable walls and lacks nodules, ruling out option C. • Cystic metastases are rare, usually lower-lobe and accompanied by dominant masses; upper-zone nodular-cystic disease in a smoker favors PLCH over option D. • Idiopathic pulmonary fibrosis presents with basal-predominant reticulation, honeycombing and traction bronchiectasis rather than nodules with cysts, excluding option E.
  • 288.
    EBDR Exam MCQs& Concepts May 2022 Dr. Kareem Alnakeeb 282 45. Which of the following is NOT recognized as a cause of a “blue ear drum” seen on otoscopy? A. Cholesterol granuloma B. Glioma C. Otitis media with effusion D. Idiopathic haemotympanum E. Temporal bone fracture Explanation: (by Perplexity AI, generated with the OpenAI O3 model) • A blue-coloured tympanic membrane typically reflects blood or blood-stained fluid in the middle ear. • Cholesterol granuloma often presents with a blue or “chocolate” eardrum because of altered blood products behind an intact membrane. Otitis media with effusion (glue ear) can impart a bluish hue when the effusion is thick or haemorrhagic. • Idiopathic haemotympanum is defined by painless bluish discoloration due to recurrent bleeding in a poorly ventilated middle ear. • Acute haemotympanum after temporal bone fracture likewise stains the drum blue-black. • In contrast, glioma (a congenital glial rest extending from the intracranial compartment) appears as a reddish-pink mass in the external auditory canal or middle ear and does not characteristically produce the uniform blue tympanic membrane seen in haemotympanum. 46. On brain diffusion-weighted MRI, which of the following entities is typically NOT associated with low apparent diffusion coefficient (ADC) values? A. Acute cerebral infarction B. Primary central nervous system lymphoma C. Intracavitary pus within a cerebral abscess D. Necrotic tumor core E. Epidermoid cyst Explanation: (by Perplexity AI, generated with the OpenAI O3 model) • Restricted diffusion—manifesting as high signal on DWI and low ADC—occurs when water motion is impeded by high cellularity or high viscosity. • Acute infarction causes cytotoxic oedema that traps water within swollen cells, producing low ADC. • Lymphoma is densely cellular, again restricting diffusion. • Pus is viscous and cellular, giving low ADC within abscess cavities. • Epidermoid cysts contain keratin and cholesterol crystals that markedly restrict diffusion. • By contrast, tumoral necrosis involves cellular breakdown and increased extracellular water, resulting in facilitated diffusion and therefore relatively high ADC, so it is not associated with low ADC. 47. On diffusion-weighted MRI of the brain, what is the typical signal pattern seen in the centre of a mature pyogenic abscess? A. High signal on DWI with low ADC values B. Low signal on DWI with high ADC values C. Isointense on both DWI and ADC maps D. High signal on DWI with high ADC values E. Low signal on DWI with low ADC values Explanation: (by Perplexity AI, generated with the OpenAI O3 model) • Pus within a pyogenic abscess is highly cellular and viscous, restricting Brownian water motion. This produces true restricted diffusion: bright signal on DWI and correspondingly low ADC values.
  • 289.
    EBDR Exam MCQs& Concepts May 2022 Dr. Kareem Alnakeeb 283 • Options B and C show facilitated or neutral diffusion typical of necrotic tumors or cysts. • Option D represents “T2 shine-through”, where high DWI signal is accompanied by high ADC and therefore not true restriction. • Option E (low signal on both DWI and ADC) is not a recognized pattern for abscesses and would more likely reflect susceptibility artefact or hemorrhage rather than pus. 48. The commonest primary cardiac tumor in adults is: A. Myxoma B. Haemangioma C. Rhabdomyoma D. Fibroma Explanation: (by Perplexity AI, generated with the OpenAI O3 model) • > 50% of adult primary cardiac neoplasms are left-atrial myxomas. 49. The commonest primary cardiac tumor in children is: A. Myxoma B. Haemangioma C. Rhabdomyoma D. Fibroma Explanation: (by Perplexity AI, generated with the OpenAI O3 model) • Rhabdomyomas constitute more than 60% of all primary cardiac tumors diagnosed in infants and children, making them by far the most common benign cardiac neoplasm in this age group. 50. Which of the following is not associated with cardiac mass? A. Carney triad B. Carney complex C. Tuberous sclerosis D. Neurofibromatosis (NF1) Explanation: (by Perplexity AI, generated with the OpenAI O3 model) Carney triad lacks cardiac tumors • Carney triad classically links three extracardiac tumors—gastrointestinal stromal tumor, pulmonary chondroma, and extra-adrenal paraganglioma—and does not include any cardiac mass in its diagnostic constellation. • Therefore, among the options listed, Carney triad is the only condition not associated with cardiac tumors. 51. Endoscopic third ventriculostomy creates a new communication Between the third ventricle and: A. fourth ventricle B. ambient cistern C. interpeduncular cistern Explanation: (by Perplexity AI, generated with the OpenAI O3 model) • Endoscopic third ventriculostomy (ETV) creates a surgical opening in the floor of the third ventricle—just in front of the mammillary bodies—so cerebrospinal fluid can bypass an obstruction and flow directly into the interpeduncular (prepontine) cistern in the basal cisterns. o Therefore, the new communication produced by a third ventriculostomy is: o Between the third ventricle and the interpeduncular cistern. • (The other options—fourth ventricle and ambient cistern—are not the structures opened into during an ETV.)
  • 290.
    EBDR Exam MCQs& Concepts May 2022 Dr. Kareem Alnakeeb 284 52. Which of the following is not an angiographic sign of active bleeding? F. Contrast extravasation G. Vessel spasm H. Vessel cut-off I. Early venous filling J. Vessel dilatation Source: Chin, Teck Yew, et al. Get Through Final FRCR 2A: SBAs. 1st ed., CRC Press, 2017. Explanation: • The rest are angiographic signs of active bleeding; vessel dilatation is not. 53. On sagittal MRI of an acutely collapsed lumbar vertebral body, which imaging feature is most characteristic of a malignant compression fracture rather than an osteoporotic fracture? A. Intravertebral fluid (“cleft”) sign B. Band-like preservation of normal marrow in the posterior third of the vertebral body C. Low signal intensity extending into the ipsilateral pedicle D. Wedge deformity with intact posterior wall and no soft-tissue component E. Uniform low T1 signal confined to the anterior two-thirds of the body Explanation: (by Perplexity AI, generated with the OpenAI O3 model) • Malignant infiltration frequently spreads from the vertebral body into the pedicle, producing continuous low T1 and T2 signal in the posterior elements; this spread is uncommon in purely osteoporotic fractures, which usually spare the pedicles and posterior wall. • Benign osteoporotic fractures often show the intravertebral fluid cleft (A) and a horizontal band of preserved marrow (B). • A simple wedge shape with an intact posterior cortex and no paraspinal mass (D) further supports a benign cause. • Uniform low T1 signal limited to the body (E) lacks specificity, as both malignant and acute osteoporotic fractures can appear diffusely hypointense shortly after collapse. 54. Which of the following is not a feature of holoprosencephaly? A. Single ventricle B. Fused thalami C. Absent corpus callosum D. Tectal beaking E. Hypoplasia of the optic nerves Source: Chin, Teck Yew, et al. Get Through Final FRCR 2A: SBAs. 1st ed., CRC Press, 2017. Explanation: Holoprosencephaly • Holoprosencephaly results from a lack of normal cleavage of the forebrain. • Holoprosencephaly may be divided into alobar, semilobar and lobar forms depending on the degree of abnormality. • Single ventricle, fused thalami, absent corpus callosum and hypoplasia of the optic nerves are all features of the various forms of holoprosencephaly. Septum Pellucidum • The septum pellucidum is always absent in this condition. • In its mildest form, lobar holoprosencephaly, absence of the septum pellucidum may be the only abnormality. Associated Disorders and Severity • The lobar form may be associated with septo-optic dysplasia and the two conditions overlap.
  • 291.
    EBDR Exam MCQs& Concepts May 2022 Dr. Kareem Alnakeeb 285 • The degree of facial abnormality and mental retardation mirrors the severity of the intracranial abnormality. Chiari II Sign • Tectal beaking is a feature of Chiari II. 55. Which of the following are true of Holoprosencephaly? (True or False) A. It results from absence of the supraclinoid internal carotid artery (ICA) system B. Trisomy 13 is the most common associated chromosomal abnormality C. The septum pellucidum can be seen in the lobar subtype D. The falx cerebri may be seen anteriorly in the semilobar type E. The cerebellum is structurally normal in the alobar type Source: Bell, J., and N. Davies. MCQs in Clinical Radiology: A Revision Guide for the FRCR. 1st ed., Remedica Pub Ltd, 2004. Explanation: A. False This abnormality results in Hydranencephaly, in which there is no anterior cerebral mantle or facial anomaly and the falx cerebri and thalami are normal. B. True C. False D. False E. True • b-e) Overview • Holoprosencephaly results from partial/complete lack of cleavage of the developing forebrain and is divided into three types: alobar, semilobar and lobar. Alobar type • The alobar type is most severe and is characterised by fused cerebral hemispheres with an anterior cup-shaped brain, a single monoventricle, fused thalami and absent corpus callosum, fornix, optic tracts and olfactory bulbs (the midbrain, brain stem and cerebellum are normal). Semilobar type • The semilobar type is of intermediate severity and is characterised by partial cleavage into hemispheres, monoventricles with rudimentary occipital and temporal horns. • The falx cerebri may be present posteriorly and the thalami are variably fused. Lobar type • The lobar type is the least severe and is characterized by lateral ventricles that are almost normal, but frontal horns point inferiorly and may be 'squared'. Shared characteristics • The septum pellucidum is absent in all three subtypes. Incidence and associations • The incidence is equal in males and females and is associated with: chromosome abnormalities (trisomy 13 and 18 are most common), polyhydramnios (60%), and renal and cardiac anomalies. Facial anomalies • Facial anomalies are also common, ranging from cyclopia to hypotelorism, and they usually correlate with the severity of the brain abnormality. Related condition
  • 292.
    EBDR Exam MCQs& Concepts May 2022 Dr. Kareem Alnakeeb 286 • Septo-optic dysplasia (absence of the septum pellucidum and hypoplastic anterior optic pathways) may be considered a mild form of lobar holoprosencephaly. 56. A 27-year-old woman presents to the Accident & Emergency Department with headaches. A CT scan of the head shows widely spaced lateral ventricles, dilatation of the trigones and occipital horns of lateral ventricles with an upward displacement of the dilated 3rd ventricle. The underlying abnormality in the brain is? A. Midline arachnoid cyst B. Agenesis of the corpus callosum C. Prominent cavum septum pellucidum D. Hydrocephalus E. Lobar holoprosencephaly Source: Gupta, Chaitanya. 300 Single Best Answers for the Final FRCR Part A. 1st ed., Jaypee UK, 2010. Explanation: • This is associated with parallel, widely spaced lateral ventricles that may appear crescent shaped. • There is dilatation of trigones and the occipital horn of lateral ventricles, along with a high riding 3rd ventricle. • Callosal agenesis is associated with Dandy–Walker syndrome, Chiari malformations and fetal alcohol syndrome. 57. A 66-year-old joiner presents to his GP with jaundice and abdominal discomfort. He was subsequently referred to a gastroenterologist who requests a liver biopsy due to deranged liver function tests. Which of the following options is not a contraindication for percutaneous liver biopsy? F. INR above 1.6 G. Platelets less than 60,000/mm3 H. Tense ascites I. Extra-hepatic biliary obstruction J. Suspected hemangioma Source: Chin, Teck Yew, et al. Get Through Final FRCR 2A: SBAs. 1st ed., CRC Press, 2017. Explanation: Contraindications for liver biopsy include the following: • Uncooperative patient • Extrahepatic biliary duct dilatation (except if benefit outweighs the risk) • Bacterial cholangitis (relative contraindication due to risk of septic shock) • Abnormal coagulation indices (having a normal INR or PT is not a reassurance that the patient will not bleed; however, there is increased incidence of bleeding with INR above 1.5) • Thrombocytopenia (platelet count below 60,000/mm3)
  • 293.
    EBDR Exam MCQs& Concepts May 2022 Dr. Kareem Alnakeeb 287 58. A patient with known tuberous sclerosis had a routine follow-up CT. A 3 x 2-cm partly calcified heterogeneously enhancing lesion was seen at the level of the foramen of Monro. What is the most likely pathology? A. Colloid cyst B. Subependymal giant cell astrocytoma C. Intraventricular D. Meningioma E. Germinoma Source: Proctor, Robin. Final FRCR Part A Modules 4-6 Single Best Answer MCQs: The SRT Collection of 600 Questions with Explanatory Answers (MasterPass). 1st ed., CRC Press, 2009. Explanation: • 15% of patients with tuberous sclerosis develop subependymal astrocytomas. They typically occur at the foramen of Monro and are usually a well-defined rounded mass with some calcification. They usually enhance uniformly with contrast and can degrade to a high-grade astrocytoma. • 95% of tuberous sclerosis patients have subependymal hamartomas. These occur in the periventricular region, are isointense to white matter on Ti and calcified on CT. • 55% of patients have cortical tubers, which are high signal on T2-weighted imaging. 59. A five year old with seizures and cognitive impairment had an MRI scan. This revealed features highly suggestive of heterotopia. What are the likely findings on the MRI? A. CSF lined cleft extending from the ependymal surface to cortical pia B. Shallow Sylvian fissures and agyric cortex C. Bilateral nodular subependymal grey matter D. Squared appearance of the frontal horns and an absent septum pellucidum E. Poor brain sulcation with intraparenchymal calcification Source: Proctor, Robin. Final FRCR Part A Modules 4-6 Single Best Answer MCQs: The SRT Collection of 600 Questions with Explanatory Answers (MasterPass). 1st ed., CRC Press, 2009. Explanation: • Heterotopic grey matter occurs secondary to developmental arrest of migrating neuroblasts from the ventricular walls to the surface of the brain. Nodular and laminar forms are described. Signal is isointense to grey matter on all sequences. 60. A 62-year-old man with known hepatocellular carcinoma on a background of long-standing liver cirrhosis is scheduled to have a TACE procedure. Which one of the following is an absolute contraindication to TACE therapy for hepatocellular carcinoma in a cirrhotic patient? A. Contrast medium allergy B. Replacement of 25% of the liver by the tumour C. Total bilirubin greater than 2 mg/dL D. Biliary tree obstruction E. Child–Pugh Class C cirrhosis Source: Chin, Teck Yew, et al. Get Through Final FRCR 2A: SBAs. 1st ed., CRC Press, 2017.
  • 294.
    EBDR Exam MCQs& Concepts May 2022 Dr. Kareem Alnakeeb 288 Explanation: Absolute and relative contraindications for conventional TACE in patients with HCC are as follows. Absolute contraindications: Relative contraindications: 1. Decompensated cirrhosis (Childs–Pugh C or higher) 1. Tumour size >10 cm. 2. Jaundice 2. Co-morbidities involving compromised organ function such as cardiovascular and lung disease. 3. Clinical encephalopathy 3. Untreated varices present a high risk of bleeding. 4. Refractory ascites 4. Bile duct occlusion or incompetent papilla due to stent or surgery. 5. Extensive tumour with massive replacement of both lobes 6. Severely reduced portal vein flow 7. Technical contraindications to hepatic intra-arterial treatment 8. Renal insufficiency (creatinine clearance <30 mL/min) 61. A 52-year-old woman is involved in a RTA. Trauma series radiographs reveal a complex pelvic fracture. There are no other appreciable injuries. She is haemodynamically unstable, and fluid resuscitation is commenced. Which of the following steps is the next most important? A. Trauma protocol CT scan with angiographic sequences B. Immediate surgery under the orthopaedic surgeons C. Discussion with interventional radiologists with view to catheter angiography and possible intervention D. Placement of a pelvic wrap device E. Blood transfusion Source: Chin, Teck Yew, et al. Get Through Final FRCR 2A: SBAs. 1st ed., CRC Press, 2017. Explanation: Application and Purpose of Pelvic Wrap • All of these steps may be required, but the most important is to place a pelvic wrap device. • The purpose of this device is to stabilize the pelvis. • If the patient is hypotensive as a result of venous bleeding, a pelvic wrap should stabilize the pelvis sufficiently to cause significant reduction or cessation in the venous hemorrhage and thus avoid unnecessary endovascular intervention. Indications for Endovascular Intervention • If this fails to achieve sufficiently prompt hemodynamic stability, there is a significant chance that there is arterial hemorrhage; hence endovascular intervention is likely to be necessary. Imaging for Bleeding Source • Many centers use angiographic sequences as part of the trauma CT scan to identify such a bleeding point as part of planning stage of endovascular management. • CT would be the next step once the pelvic binder is in situ.
  • 295.
    EBDR Exam MCQs& Concepts May 2022 Dr. Kareem Alnakeeb 289 62. CT brain of a 10 year old girl shows a large cyst in the posterior fossa. All of the following favour pilocystic astrocytoma over haemangioblastoma, except A. Size greater than 5 cm B. Calcifications C. Smaller nodule D. Thicker-walled lesion E. No angiographic contrast blush of the mural nodule Source: Chin, Teck Yew, et al. Get Through Final FRCR 2A: SBAs. 1st ed., CRC Press, 2017. Explanation: Pilocytic Astrocytoma • Pilocytic astrocytoma is the most common pediatric cerebellar neoplasm and the most common pediatric glioma. Differentiation from Hemangioblastoma • They can be differentiated from hemangioblastoma on the following basis: • Astrocytomas are more likely to be larger than 5 cm, contain calcification, have a larger mural nodule, are thick-walled lesions, do not show angiographic contrast blush to the mural nodule and are not associated with erythrocythemia.
  • 296.
    EBDR Exam MCQs& Concepts October 2021 Dr. Kareem Alnakeeb 290 October 2021 Paper 1 1. Which imaging feature is classically associated with an oligodendroglioma on CT or MRI of the brain? A. Cortical and subcortical location B. Lack of contrast enhancement C. Presence of calcification D. Intraventricular origin E. Hyperperfusion on arterial spin-labelling Explanation: (by Perplexity AI, generated with the OpenAI O3 model) • Oligodendrogliomas most often arise in the cerebral hemispheres at the cortex–subcortical junction and frequently show coarse or gyriform calcifications on CT; this striking tendency to calcify (seen in up to 90% of cases) makes calcification the hallmark imaging clue. • Although some low-grade lesions may enhance little or not at all, contrast enhancement is variable and therefore not reliably diagnostic. • They are parenchymal tumors, not intraventricular masses (intraventricular tumors with similar calcification are more likely central neurocytomas). • Hyperperfusion is neither typical nor specific. 2. Which imaging combination is most typical of an intracranial epidermoid cyst? A. Extra-axial lesion showing T2 shine-through on DWI and low attenuation on CT B. Extra-axial lesion with true restricted diffusion, low attenuation on CT and incomplete suppression on FLAIR C. Intra-axial lesion with T2 shine-through on DWI and low attenuation on CT D. Extra-axial lesion with T2 shine-through on DWI and complete suppression on FLAIR E. Intra-axial lesion with restricted diffusion and hyperdensity on CT Explanation: (by Perplexity AI, generated with the OpenAI O3 model) • Epidermoid cysts are usually extra-axial, appear hypodense (CSF-like) on CT and fail to fully suppress on FLAIR, giving a “dirty CSF” appearance. • On diffusion-weighted imaging they are markedly bright because of true restricted diffusion rather than mere T2 shine-through; the high cellular keratin content limits water motion and differentiates them from arachnoid cysts. • Options with intra-axial location, full FLAIR suppression or only shine-through lack this classic triad and are therefore incorrect. 3. Regarding juvenile nasopharyngeal angiofibroma (shown in the CT image of a vascular nasopharyngeal mass), which statement is TRUE? A. It is a malignant tumor B. It is avascular C. It originates at the foramen rotundum D. It may erode adjacent bone E. It affects females more often than males Explanation: (by Perplexity AI, generated with the OpenAI O3 model) • Juvenile nasopharyngeal angiofibroma is a benign yet highly vascular tumor that arises near the sphenopalatine foramen in adolescent males. Although histologically benign, its aggressive growth
  • 297.
    EBDR Exam MCQs& Concepts October 2021 Dr. Kareem Alnakeeb 291 often produces pressure-related bone erosion and remodeling of the pterygoid plates, sphenoid bone and adjacent sinus walls, a key imaging clue. • The lesion is not malignant (so option A is wrong) and is anything but avascular (option B). • Its site of origin is the posterior choanal/sphenopalatine foramen region rather than the foramen rotundum (option C). • It shows an overwhelming male predilection, making option E incorrect. 4. On MRI for a suspected middle-ear cholesteatoma, which imaging combination is most characteristic? A. Central contrast enhancement with restricted diffusion B. Central contrast enhancement without restricted diffusion C. No central enhancement and no restricted diffusion D. No central enhancement but restricted diffusion E. Rim enhancement with facilitated diffusion Explanation: (by Perplexity AI, generated with the OpenAI O3 model) • Acquired and residual cholesteatomas typically appear non-enhancing on post-contrast T1- weighted images because the keratin matrix is avascular, yet they show marked hyperintensity on non-echo-planar diffusion-weighted imaging with low ADC values, reflecting true restricted diffusion of keratin debris. • Enhancement within the lesion suggests vascular granulation tissue or abscess (options A, B, E). • Absence of both enhancement and diffusion restriction (option C) favors simple fluid or mucosal thickening rather than cholesteatoma. 5. Which of the following features favor Rathke’s cleft cyst rather than craniopharyngioma? A. Absence of calcification B. Cystic element on MR C. Involvement of suprasellar and sellar regions D. Enhancement of the wall E. High signal intensity on T1 Source: Leen, Edward. Succeeding in the New FRCR Part 2A Exam: Single Best Answer (SBA) Revision Questions for Modules 1-6. 1st ed., UNKNO, 2011. Explanation: • Rathke’s cleft cysts do not calcify. They affect women to men in a 2:1 ratio and adults from 40-60 years of age. They cause variable MR appearances depending on protein content of cyst. They can rarely show enhancement. 6. Which is the most common imaging finding in neurosarcoidosis? A. Leptomeningeal contrast enhancement B. Hyperintense white matter T2 lesion C. Grey matter lesions enhancing on MR D. Involvement of the hypothalamus E. Focal epidural masses Source: Leen, Edward. Succeeding in the New FRCR Part 2A Exam: Single Best Answer (SBA) Revision Questions for Modules 1-6. 1st ed., UNKNO, 2011. Explanation: • Basilar meninges often involved. • B-E are recognized imaging findings. • Spinal disease is less common than brain disease and findings include intramedullary lesions and intrathecal nodular masses.
  • 298.
    EBDR Exam MCQs& Concepts October 2021 Dr. Kareem Alnakeeb 292 7. Which WHO grade 1 meningioma subtype most often appears isointense to cortical grey matter on T2- weighted MRI? A. Angiomatous meningioma B. Fibroblastic meningioma C. Psammomatous meningioma D. None of the above E. Microcystic meningioma Explanation: (by Perplexity AI, generated with the OpenAI O3 model) • T2 signal in meningiomas depends chiefly on their collagen and water content. Fibroblastic (fibrous) meningiomas are rich in dense collagen, giving them low proton mobility and therefore a low- to-intermediate (commonly isointense) T2 signal relative to cortex. • Angiomatous and microcystic variants contain abundant vascular channels or microcysts with high fluid content, so they are typically T2 hyperintense rather than isointense. • Psammomatous tumours often show coarse calcification; on MRI they are frequently T2 hypointense or mixed, not predominantly isointense. • Thus an extra-axial mass that is T2 isointense should raise suspicion for the fibrous subtype. 8. In a patient with a cystic intracranial mass, which MRI sequence most reliably differentiates a pyogenic cerebral abscess from a pilocytic astrocytoma? A. FLAIR B. Diffusion-weighted imaging (DWI) C. Gradient-echo (T2*) D. Post-contrast T1-weighted E. T2 spin-echo Explanation: (by Perplexity AI, generated with the OpenAI O3 model) • Pus within a cerebral abscess is highly cellular and viscous, causing marked restriction of water motion; this appears as very high signal on DWI with corresponding low ADC values, making DWI the most specific sequence for identifying an abscess. • Pilocytic astrocytomas, although often cystic, generally show facilitated or only mildly restricted diffusion, so they are easily distinguished on this sequence. • FLAIR highlights oedema but cannot separate these entities. • Gradient-echo is sensitive to hemorrhage or calcification rather than pus. • Ring enhancement on post-contrast T1 may be seen in both lesions, and conventional T2 signal characteristics overlap, so neither provides definitive discrimination. 9. On digital subtraction angiography, which single angio-architectural feature best distinguishes a true arteriovenous malformation (AVM) from an arteriovenous fistula (AVF)? A. Presence of an angiodense nidus B. Identifiable feeding artery C. Overall size of the lesion D. Pattern of venous drainage E. Patient age at presentation Explanation: (by Perplexity AI, generated with the OpenAI O3 model) • An AVM is defined by a compact tangle of abnormal vessels—the nidus—interposed between feeding arteries and draining veins. Its angiographic appearance is therefore an “angiodense” nidus, a key hallmark that is absent in an AVF12.
  • 299.
    EBDR Exam MCQs& Concepts October 2021 Dr. Kareem Alnakeeb 293 • AVFs show a single, direct artery-to-vein connection without an intervening nidus; although feeders and drainers are visible, their mere presence (options B and D) does not differentiate the two entities3. • Lesion size (option C) varies widely in both AVMs and AVFs and is not a defining criterion. • Age at presentation (option E) can overlap and offers no reliable discrimination. 10. Bilateral globus pallidus injury manifest radiologically as high signal on T2W MR sequences is indicative of poisoning by which of the following substances? A. lead B. methanol C. carbon monoxide D. carbon dioxide E. mercury Source: Bydder, Megan, et al. Get Through Final FRCR Part A: SBAs for the Modular Examination. 1st ed., Hodder Education Publishers, 2009. Explanation: Basal Ganglia Involvement in Carbon Monoxide Poisoning • Carbon monoxide poisoning results in irreversible formation of carboxyhemoglobin in the blood, causing anoxic ischemic encephalopathy. • These changes are usually bilateral and affect the basal ganglia, most commonly the globus pallidus. • Areas less commonly affected acutely are the putamen (which is characteristically involved in methanol poisoning) and caudate nucleus. • Involvement elsewhere can occur but is less common than basal ganglia changes. MRI Signal Characteristics • Injury is demonstrated as high signal on T2W and FLAIR images, and shows restricted diffusion on DWI. Delayed Post-Anoxic Changes • Delayed post-anoxic encephalopathy may develop several weeks after carbon monoxide poisoning, and MRI then shows further high T2 signal changes in the corpus callosum, subcortical U fibres, and internal and external capsules, with low T2 signal changes in the thalamus and putamen. 11. Regarding carotid–cavernous fistulas (CCFs), which of the following statements is TRUE? A. They are most commonly produced by rupture of a cavernous carotid aneurysm. B. Spontaneous CCFs are usually associated with fibromuscular dysplasia. C. A direct (Barrow type A) fistula is typically treated by a trans-arterial endovascular approach. D. Fistulas supplied by multiple arterial branches are preferably treated through an arterial route. E. Indirect (dural) fistulas never close spontaneously. Explanation: (by Perplexity AI, generated with the OpenAI O3 model) • Direct CCFs (high-flow Barrow type A), produced by a tear in the cavernous segment of the internal carotid artery—usually traumatic—are most effectively treated by a trans-arterial route using detachable balloons, coils, or flow-diverters, aiming to occlude the fistulous site while preserving carotid patency. • Most CCFs arise after head trauma; formation from rupture of a cavernous carotid aneurysm is distinctly uncommon, accounting for <5% of cases, so option A is incorrect. • Fibromuscular dysplasia is a rare predisposing factor for spontaneous dural CCFs, not a usual association, making option B wrong.
  • 300.
    EBDR Exam MCQs& Concepts October 2021 Dr. Kareem Alnakeeb 294 • When a fistula is fed by multiple small dural branches (indirect types B–D), trans-venous embolization via the inferior petrosal/superior ophthalmic vein is preferred; hence option D is incorrect. • Indirect CCFs can undergo spontaneous thrombosis, so option E is false. 12. Which is the most common site of metastatic spread in medulloblastoma? A. Axial skeleton B. Lymph nodes C. Lung D. Subarachnoid space E. Liver Source: Leen, Edward. Succeeding in the New FRCR Part 2A Exam: Single Best Answer (SBA) Revision Questions for Modules 1-6. 1st ed., UNKNO, 2011. Explanation: • Subarachnoid space is the most common, with drop metastases occurring in 40%. 13. A 30-year-old man presents with a lump in the left cheek. Ultrasound examination shows an 8 mm hypoechoic and lobulated lesion with a hyperechoic center. The most likely cause of the lesion is? A. Parotid duct stone B. Lymph node C. Warthin’s tumor D. Pleomorphic adenoma E. Abscess Source: Gupta, Chaitanya. 300 Single Best Answers for the Final FRCR Part A. 1st ed., Jaypee UK, 2010. Explanation: • Typical appearances of intraglandular lymph nodes are of a hypoechoic periphery with a fatty hyperechoic center. 14. In adult head-injury patients with suspected diffuse axonal injury, what is the most common initial appearance on non-contrast CT? A. Small hemorrhagic foci B. Normal study C. Small hemorrhagic foci with marked cerebral oedema D. Conspicuous lesions involving brainstem and corpus callosum E. Extensive subarachnoid hemorrhage Explanation: (by Perplexity AI, generated with the OpenAI O3 model) • The shearing forces of diffuse axonal injury (DAI) usually produce microscopic axonal disruption without macroscopic bleeding. Consequently, up to 50–80% of patients show no visible abnormality on their first CT scan, making a “normal” study the commonest presentation. • Hemorrhagic petechiae (option A) are classic but occur in a minority; when present with diffuse oedema (option C) they indicate more severe injury. • Early CT rarely shows the characteristic corpus callosum or brainstem lesions (option D); these are better detected on MRI. • Extensive subarachnoid hemorrhage (option E) is not a typical feature of isolated DAI.
  • 301.
    EBDR Exam MCQs& Concepts October 2021 Dr. Kareem Alnakeeb 295 15. Optochiasmatic arachnoiditis most commonly develops as a complication of which intracranial infection? A. Toxoplasmosis B. Neurocysticercosis C. Optic nerve meningioma D. Tuberculous meningitis E. Herpes simplex encephalitis Explanation: (by Perplexity AI, generated with the OpenAI O3 model) • Thick basal exudates from tuberculous meningitis tend to accumulate in the suprasellar and interpeduncular cisterns, encasing the optic nerves and chiasm and provoking optochiasmatic arachnoiditis with rapid, often bilateral visual loss. • Other infections such as neurocysticercosis can cause basal arachnoiditis, but chiasmal involvement is far less typical. • Optic nerve meningioma produces a fusiform dural-based mass rather than diffuse leptomeningeal inflammation, while toxoplasmosis and herpes encephalitis characteristically affect parenchyma, not the leptomeninges, so they do not produce this arachnoiditic pattern. 16. On MRI, how do demyelinating brainstem plaques of multiple sclerosis typically appear? A. Central and symmetrical B. Peripheral (abutting CSF spaces) C. Spare the cerebellum completely D. Both B and C E. Diffuse and confluent throughout the pons Explanation: (by Perplexity AI, generated with the OpenAI O3 model) • Multiple sclerosis characteristically produces small, well-defined T2/FLAIR hyperintense plaques that lie at the outer or inner borders of the brainstem, contiguous with the subarachnoid space or ventricular surface. • This peripheral predilection reflects perivenular inflammation along penetrating vessels and helps distinguish MS from vascular small-vessel disease, whose pontine foci are usually central and symmetric. • Cerebellar hemispheres are frequently involved in MS, so they are not spared (making options C and D incorrect). • Large confluent or diffuse pontine lesions (option E) are uncharacteristic for typical MS plaques. 17. Which statement about cystic lymphangioma is TRUE? A. It is most often encountered in the pediatric age group B. It is confined to the neck and never arises in the retroperitoneum C. It typically shows rim (“ring”) enhancement on contrast-enhanced CT D. Magnetic resonance imaging is mandatory to make the diagnosis E. It commonly calcifies on plain radiography Explanation: (by Perplexity AI, generated with the OpenAI O3 model) • Cystic lymphangiomas are congenital malformations of lymphatic channels; 80–90% present before age 2, making childhood their commonest age group of occurrence. • Although the neck is the classic site, up to 5% arise in the abdomen or retroperitoneum, so neck-only restriction is incorrect. • On CT they appear as multiloculated, fluid-attenuation masses that show little or no mural enhancement; a conspicuous enhancing rim is not typical. • MRI delineates extent well but is not obligatory; ultrasound or CT often suffice for diagnosis and treatment planning, so option D is false. • Calcification is rare, not common, in lymphangiomas; hence option E is wrong.
  • 302.
    EBDR Exam MCQs& Concepts October 2021 Dr. Kareem Alnakeeb 296 18. On plain chest radiography, which pulmonary lesion classically appears as a solitary nodule with a central “umbilication” or notch sign? A. Hydatid cyst B. Pulmonary metastasis C. Both A and B D. Rounded atelectasis E. Pulmonary hamartoma Explanation: (by Perplexity AI, generated with the OpenAI O3 model) • The “umbilicated” or notched margin of a solitary lung nodule was first described in malignant lesions, especially metastatic deposits from primaries such as colorectal or renal carcinoma. The central indentation represents tumor infiltration along pulmonary vessels producing a focal retraction of the nodule edge, a feature rarely seen in benign masses. • Hydatid cysts usually present as smooth, spherical cystic opacities; when complicated they show air–fluid signs (crescent, water-lily) rather than a focal umbilication, so option A is incorrect. • Rounded atelectasis forms a pleural-based mass that blends with thickened pleura and displays the comet-tail sign, not an umbilicated border; therefore option D is wrong. • Pulmonary hamartomas characteristically contain “popcorn” calcification or fat and do not show a notched margin. 19. Which radiographic features are typically seen in round atelectasis of the lung? A. Crowded lung markings B. Pleural thickening C. Peripheral mass or nodule D. All of the above E. None of the above Explanation: (by Perplexity AI, generated with the OpenAI O3 model) • Round atelectasis is characterized by a triad of imaging findings on chest radiograph and CT: 1. a peripheral mass or nodule, typically in the lower lobes; 2. adjacent pleural thickening, often related to prior pleural disease such as effusion or asbestos exposure; and 3. crowding or curving of pulmonary vessels and bronchi toward the lesion, producing the “comet tail” sign (which reflects crowding of lung markings). • All individual features listed in options A–C are essential to its recognition, making “all of the above” the correct choice. • Isolated crowding or thickening without a mass does not constitute round atelectasis. 20. Round atelectasis: (True or False) A. has ill-defined edges. B. is always pleurally based. C. is always associated with pleural thickening. D. has a ‘comet tail’ appearance. E. caused by a previous pleural transudate. Source: Scoffings, Daniel. Get Through FRCR 2A: Practice Papers for the Modular Examination. 1st ed., CRC Press, 2004. Explanation: A. True B. True C. True D. True
  • 303.
    EBDR Exam MCQs& Concepts October 2021 Dr. Kareem Alnakeeb 297 E. False — exudate. 21. Which imaging appearances have been described as showing an intralesional “crescent sign” of peripheral air outlining internal material? A. Cyst containing hemorrhage and debris B. Contained or frank rupture of a cyst with escape of debris C. Intracavitary pulmonary mycetoma (aspergilloma) D. All of the above E. None of the above Explanation: (by Perplexity AI, generated with the OpenAI O3 model) • When a pulmonary hydatid or other cyst partially fills with air while still containing blood or debris, a crescent-shaped rim of gas may separate the cyst wall from the internal content, producing the classic crescent (meniscus) sign on radiographs or CT. • If the cyst ruptures into a bronchus, additional air dissects between the pericyst and endocyst, again generating an air crescent before complete collapse; debris may be visible in the dependent portion. • A mature aspergilloma forms a mobile fungus ball within a pre-existing cavity; air immediately above the intracavitary mass creates the identical crescent/meniscus configuration (also called the Monod sign). Because the same aerated meniscus can be produced in all three scenarios, “all of the above” is the single best answer. 22. In HRCT terminology, which term best describes a gas-filled pulmonary space whose wall is thicker than 3 mm? A. Emphysematous bulla B. Pulmonary cyst C. Tuberculous cavity D. Pleural bleb E. Pneumatocele Explanation: (by Perplexity AI, generated with the OpenAI O3 model) • A pulmonary cavity is defined by the Fleischner Society as a gas-filled space within lung parenchyma, consolidation or a nodule whose wall is usually thicker than 4 mm; clinically, post- primary tuberculosis is the classic cause, so a “TB cavity” fits this description. • Bullae and blebs have hairline-thin walls (<1 mm) and arise from emphysema or along the pleura, respectively. • A pulmonary cyst is also thin-walled (<2 mm) and usually round. • A pneumatocele is a transient, thin-walled, traumatic or infective air collection. • Thus, wall thickness >3 mm rules out cyst, bulla, bleb and pneumatocele, leaving a cavity— commonly tuberculous—as the correct choice. 23. On a routine chest radiograph, which of the following is NOT a recognized sign of lung hyperinflation? A. Horizontal ribs B. Low, flattened diaphragm C. Narrow retrosternal clear space D. Tubular “ribbon” heart E. Widened intercostal spaces
  • 304.
    EBDR Exam MCQs& Concepts October 2021 Dr. Kareem Alnakeeb 298 Explanation: (by Perplexity AI, generated with the OpenAI O3 model) • Classic radiographic clues to pulmonary hyperinflation—typically seen in emphysema—include low, flat hemidiaphragms and more horizontal ribs caused by increased intrathoracic volume. • The anteroposterior (AP) diameter increases, so the heart appears elongated and tubular (“ribbon heart”) on the frontal film, and the intercostal spaces widen. • In contrast, the retrosternal clear space on the lateral view becomes wider, not narrower, because the over-inflated lungs encroach anteriorly. • Therefore a “narrow” retrosternal space would argue against hyperinflation, making option C the exception. 24. In systemic staging of primary lung carcinoma, which organ is the most frequent target of hematogenous metastasis seen at diagnosis or autopsy studies? A. Brain B. Contralateral lung C. Skeleton D. Adrenal gland E. Liver Explanation: (by Perplexity AI, generated with the OpenAI O3 model) • Autopsy series and large imaging datasets consistently show that blood-borne spread from lung cancer most often deposits in the adrenal cortex, with reported involvement in 30–40% of cases. • Although brain and bone metastases are also common, their incidence is lower. • Contralateral pulmonary nodules usually represent intrapulmonary spread rather than true distant hematogenous metastasis, and hepatic deposits rank below adrenal involvement in frequency. • Recognising the adrenal glands as the prime hematogenous “landing site” is critical because small, silent adrenal metastases upstage the tumor to stage IV and alter management. 25. On chest radiography showing innumerable 1–3 mm miliary nodules that are densely calcified, which one of the following causes is LEAST likely? A. Silicosis B. Healed histoplasmosis C. Active miliary tuberculosis D. Broncholithiasis E. Metastatic papillary thyroid cancer Explanation: (by Perplexity AI, generated with the OpenAI O3 model) • Dense calcification in tiny, randomly distributed miliary nodules classically reflects healed or chronic processes that produce dystrophic calcium deposition. • Pneumoconiosis such as silicosis and healed granulomatous infections like histoplasmosis frequently leave innumerable calcified micronodules. • Calcified airway‐extruded granulomas (broncholithiasis) can scatter calcified fragments into adjacent lung, also giving a miliary pattern. • Certain metastases (not asked originally but included here for option balance) from papillary or medullary thyroid carcinoma may calcify in a miliary fashion. • In contrast, active miliary tuberculosis usually presents with non-calcified micronodules; calcification, if it occurs, is a late sequela after healing. Therefore active miliary TB is the least compatible with a calcified miliary appearance.
  • 305.
    EBDR Exam MCQs& Concepts October 2021 Dr. Kareem Alnakeeb 299 26. On chest imaging, which radiographic feature best helps distinguish a pyogenic lung abscess from a recently-ruptured pulmonary hydatid cyst? A. Associated parenchymal consolidation B. Pleural effusion C. Air–fluid interface: straight in abscess, wavy or undulating in ruptured hydatid cyst D. Typical basal location Explanation: (by Perplexity AI, generated with the OpenAI O3 model) • A lung abscess usually forms a thick-walled cavity containing pus; its fluid settles evenly, so the air– fluid margin is flat and horizontal on erect films or CT. • When a hydatid cyst ruptures into a bronchus, collapsed membranes float on residual fluid producing an irregular, wavy“water-lily” interface rather than a straight line; this undulating margin is a classic clue to cyst rupture. • Consolidation (A) and reactive pleural effusion (B) can accompany either condition and are therefore non-specific. • Both lesions may appear anywhere in the lungs, so site preference (D) is unreliable for differentiation. 27. On HRCT, a diffuse “crazy-paving” pattern of ground-glass opacities with superimposed inter- and intralobular septal thickening is classically associated with which pulmonary disorder? A. Pulmonary alveolar proteinosis B. Post-primary tuberculosis C. Malignant pleural mesothelioma D. Invasive pulmonary aspergillosis E. Pneumonic‐type adenocarcinoma Explanation: (by Perplexity AI, generated with the OpenAI O3 model) • Pulmonary alveolar proteinosis (PAP) was the condition in which the crazy-paving sign was first described and it remains the prototypical cause; 80–90% of PAP cases show this striking reticular network over ground-glass opacity. • Tuberculosis, aspergillosis and a long list of other infections can occasionally mimic the sign, but they do so far less frequently and typically with additional focal nodules or cavities that break the uniform crazy-paving appearance. • Mesothelioma is a pleural tumor; parenchymal crazy paving is not a recognized feature. 28. In cardiac MRI, cine balanced steady-state free-precession (SSFP) sequences are most commonly used for which of the following purposes? A. Quantitative assessment of left-ventricular function B. Quantitative assessment of right-ventricular function C. Detection of regional myocardial hypokinesia D. All of the above E. Coronary artery stenosis grading Explanation: (by Perplexity AI, generated with the OpenAI O3 model) • Cine SSFP yields high blood-to-myocardium contrast and excellent temporal resolution, allowing accurate measurement of ventricular volumes and ejection fractions for both the left and right ventricles. The bright-blood depiction and crisp endocardial borders also make wall motion abnormalities such as hypokinesia or akinesia readily visible on the dynamic images. • Coronary artery lumen assessment, however, generally requires dedicated MR angiography or CT, not standard cine SSFP. Thus options A, B and C are all correct, making D the single best answer.
  • 306.
    EBDR Exam MCQs& Concepts October 2021 Dr. Kareem Alnakeeb 300 29. On chest CT, which finding is considered the most specific diagnostic clue for a pulmonary aspergilloma within a pre-existing cavity? A. Crescent (Monod) sign B. Demonstrable mobility of the intracavitary fungal ball between different patient positions C. Pericavitary consolidation with pleural thickening D. Predilection for the right upper lobe E. Tree-in-bud nodularity Explanation: (by Perplexity AI, generated with the OpenAI O3 model) • A true aspergilloma is a non-invasive “fungus ball” that lies loose inside an old tuberculous or other cavitary lesion. The hallmark is that the mycetoma is free-moving; when the patient changes from supine to prone or erect, the soft-tissue mass settles to the new dependent part, proving there is no attachment—this positional mobility is the single most specific sign. • The air-crescent (Monod) sign (A) is common but can also appear with cavitating cancer or abscess, so it is less specific. • Pericavitary consolidation or pleural thickening (C) merely indicates inflammation and is neither sensitive nor specific. • Although most aspergillomas occur in upper-lobe cavities, side predilection (D) is variable and therefore not diagnostic. • Tree-in-bud changes (E) suggest endobronchial infection, not an aspergilloma. 30. With respect to pulmonary aspergillosis: A. an aspergilloma is usually associated with pleural thickening. B. a fungal ball may be seen to change position within a cavity. C. allergic bronchopulmonary aspergillosis (ABPA) causes the ‘gloved finger’ sign. D. ABPA is the commonest cause of pulmonary eosinophilia in the UK. E. With repeated episodes of ABPA, the changes are more severe peripherally. Source: Scoffings, Daniel. Get Through FRCR 2A: Practice Papers for the Modular Examination. 1st ed., CRC Press, 2004. Explanation: A. True B. True C. True — dilated branching tubular opacities on the PA chest radiograph are due to mucoid impaction in dilated bronchi. D. True E. False — centrally. 31. In ultrasound-guided thoracocentesis, which statement is correct? A. The needle should be introduced below the rib. B. Pleural fluid should be withdrawn rapidly. C. A chest radiograph after tube insertion is mandatory. D. Both A and C. Explanation: (by Perplexity AI, generated with the OpenAI O3 model) • Real-time ultrasound marks a safe window; once the drain is sited a post-procedure chest X-ray is recommended to confirm position and exclude pneumothorax, especially when a catheter or tube remains in situ.
  • 307.
    EBDR Exam MCQs& Concepts October 2021 Dr. Kareem Alnakeeb 301 • The needle must traverse the upper border of the lower rib to avoid the intercostal vessels and nerve that run in the costal groove immediately below each rib—placing it “below the rib”greatly increases bleeding risk, so option A is wrong. • Therapeutic taps should be slow, intermittent and limited (≤1.5 L at a time) to minimize re- expansion pulmonary oedema; rapid aspiration is unsafe, making option B incorrect. 32. In aortic dissection, which statement about the Stanford classification is correct? A. It has completely replaced the DeBakey system. B. “Stanford type A” refers to dissections limited to the descending thoracic aorta. C. The classification is unrelated to treatment strategy. D. None of the above. Explanation: (by Perplexity AI, generated with the OpenAI O3 model) • The Stanford system co-exists with (but has not replaced) the older DeBakey scheme, offering a simpler two-group approach. • Type A includes any dissection that involves the ascending aorta, whether or not it extends distally; type B is confined to the descending aorta distal to the left subclavian artery. Therefore option B is incorrect. • The key virtue of the Stanford system is its link to management: type A dissections usually require urgent surgery, whereas most uncomplicated type B dissections are managed medically at first. Hence option C is wrong. • With all three statements incorrect, “None of the above”is the only accurate choice. 33. Which plain chest radiograph finding is classically associated with a tension pneumothorax? A. Ipsilateral transient hemithorax B. Ipsilateral depressed diaphragm C. Ipsilateral mediastinal shift D. Lung collapse E. Rib “splinting” Explanation: (by Perplexity AI, generated with the OpenAI O3 model) • A tension pneumothorax builds positive intrapleural pressure that pushes the diaphragm downward on the affected side, producing an ipsilateral depressed or flattened hemidiaphragm—one of the key radiographic clues. • Lung collapse (D) and contralateral (not ipsilateral) mediastinal shift are typical; option C is therefore incorrect. • “Transient” hemithorax (A) is not a recognised term, and rib splinting (E) is clinical, not radiographic. 34. Which of the following conditions is NOT a recognised cause of thin-walled cystic lesions in the lungs? A. Lymphangioleiomyomatosis B. Pulmonary Langerhans cell histiocytosis C. Congestive heart failure D. Cystic bronchiectasis E. Birt–Hogg–Dubé syndrome Explanation: (by Perplexity AI, generated with the OpenAI O3 model) • True pulmonary cysts are produced by a limited group of diffuse lung diseases such as lymphangioleiomyomatosis (LAM), pulmonary Langerhans cell histiocytosis (PLCH) and genetic disorders like Birt–Hogg–Dubé syndrome.
  • 308.
    EBDR Exam MCQs& Concepts October 2021 Dr. Kareem Alnakeeb 302 • Cystic bronchiectasis represents markedly dilated bronchi that mimic cysts on CT; it is therefore included in the differential list of cyst-appearing lung lesions. • Congestive heart failure causes interstitial oedema, Kerley lines and pleural effusions; it does not generate parenchymal cysts, making option C the exception. 35. A 60-year-old man with a 30-year heavy-smoking history presents with digital clubbing, hypertrophic osteoarthropathy, hypercalcemia and cerebellar ataxia; CT shows a centrally located hilar lung mass. What is the most likely histological subtype? A. Adenocarcinoma B. Small-cell carcinoma C. Large-cell carcinoma D. Squamous-cell carcinoma E. Bronchoalveolar carcinoma Explanation: (by Perplexity AI, generated with the OpenAI O3 model) • Squamous-cell carcinoma typically arises from a lobar or main bronchus, so it appears as a central hilar mass on CT. It is the lung cancer most strongly linked to secretion of parathyroid-hormone- related peptide, giving rise to paraneoplastic hypercalcemia. • Clubbing and hypertrophic osteoarthropathy are common non-small-cell manifestations and can accompany squamous tumors. Although paraneoplastic cerebellar degeneration is classically described with small-cell carcinoma, it may occur with any histology; in this vignette the powerful combination of central location and hypercalcemia outweighs that pointer. • Adenocarcinoma and large-cell carcinoma are usually peripheral, while small-cell carcinoma rarely causes hypercalcemia. 36. A young boy presents with signs of right-sided heart failure and mediastinal shift; chest radiograph shows a tubular shadow along the right cardiac border that is widest near the diaphragm, giving a “snowman” (figure-of-eight) appearance. Which congenital lesion most commonly produces this radiographic sign? A. Atrial septal defect B. Aortic aneurysm C. Total anomalous pulmonary venous return (supracardiac type) D. Pulmonary sequestration E. Persistent left superior vena cava Explanation: (by Perplexity AI, generated with the OpenAI O3 model) • The classic “snowman” or figure-of-eight sign on a frontal chest X-ray is highly characteristic of supracardiac total anomalous pulmonary venous return (TAPVR). • The upper “head” of the snowman is formed by dilated vertical and innominate veins on the left and an enlarged superior vena cava on the right; the lower “body” is the enlarged right atrium and ventricle. • The tubular right-paracardiac mass that tapers superiorly and is broadest inferiorly represents the confluence of these vessels, often causing mediastinal widening and right-sided volume overload. • An isolated atrial septal defect may enlarge the right heart but does not create the snowman configuration. • Aortic pathology, pulmonary sequestration, or a persistent left SVC project differently and lack the characteristic figure-of-eight outline.
  • 309.
    EBDR Exam MCQs& Concepts October 2021 Dr. Kareem Alnakeeb 303 37. Chest X-ray of a boy shows shift of the heart and mediastinum to the right. There is also a tubular structure parallel to the right heart border with its maximum width close to the diaphragm. The finding suggests A. ASD B. Scimitar syndrome C. Total anomalous pulmonary venous return D. Intralobar sequestration E. Inhaled foreign body Source: Chin, Teck Yew, et al. Get Through Final FRCR 2A: SBAs. 1st ed., CRC Press, 2017. Explanation: Hypogenetic Lung Syndrome Overview • Hypogenetic lung syndrome, also known as congenital venolobar syndrome or scimitar syndrome, is primarily a complex developmental lung abnormality with anomalous venous return. Hallmark Anatomical Features • One constant component of this syndrome is an anomalous pulmonary vein or veins draining at least a part or the entire affected lung most commonly to the inferior vena cava just above or below the diaphragm. • Uncommonly, the anomalous vein may drain into hepatic, portal, azygos veins; the coronary sinus; or the right atrium. Common Clinical and Radiographic Findings • The most common features are lung hypoplasia, anomalous pulmonary venous return to IVC, pulmonary artery hypoplasia, bronchial anomalies and systemic arterial supply to hypoplastic lung. It almost always occurs on the right side and is slightly more common in women. • A scimitar vein is a vertical curvilinear opacity in the right mid-lower lung, running along the right heart border inferomedially towards the diaphragm to join the IVC. A scimitar vein present on a frontal chest radiograph is called the scimitar sign. 38. A 10-year-old boy presents with fever and eosinophilia. MRI of the head shows thickening of the infundibular stalk and a markedly enhancing mass in the superior aspect of the stalk. There is also enhancement in the sella extending along the left petrous temporal bone with poorly defined borders. The features are consistent with A. Meningioma B. Petrous apicitis C. Histiocytosis X D. Craniopharyngioma E. Neuroblastoma metastasis Source: Chin, Teck Yew, et al. Get Through Final FRCR 2A: SBAs. 1st ed., CRC Press, 2017. Explanation: Hypothalamic–Neurohypophyseal Axis Involvement • Space-occupying lesions affect the hypothalamic-neurohypophyseal axis, which is the central nervous system site most commonly and often earliest involved in Langerhans cell histiocytosis. • MRI findings have been correlated with symptoms of diabetes insipidus, which is a clinical hallmark of the condition. • Typically, the formation of Langerhans cell histiocytosis granulomas leads to a loss in the normally high signal intensity of the posterior neurohypophysis on T1-weighted images.
  • 310.
    EBDR Exam MCQs& Concepts October 2021 Dr. Kareem Alnakeeb 304 • Furthermore, the hypothalamus, the pituitary stalk or both are frequently enlarged and demonstrate gradually increasing homogeneous enhancement after an intravenous injection of gadolinium, without subsequent washout. Differential Diagnosis of Infundibular Lesions • The differential diagnosis includes other infundibular diseases, such as adenohypophysitis, which can be differentiated from Langerhans cell histiocytosis by a sharp increase in contrast enhancement and rapid washout after the administration of the intravenous contrast medium. • Granulomatous diseases such as sarcoidosis, Wegener disease and leukaemia must also be considered in the differential. • Rarer differentials are germ cell tumours (germinoma, teratoma) and haemangioblastoma. • These produce the same MRI features, with the same pattern of enhancement at dynamic imaging. Neurodegenerative Intra-Axial Pattern • The second most frequent pattern of central nervous system involvement in Langerhans cell histiocytosis is characterised by intra-axial neuro-degenerative changes. • Bilateral symmetric lesions in the cerebellum, especially the dentate nucleus, basal ganglia, or brainstem, are most often observed. Differential Diagnosis of Neurodegenerative Pattern • The differential diagnosis includes ADEM, acute multiphasic disseminated encephalitis, disseminated encephalitis, various metabolic and degenerative disorders, leukoencephalopathy secondary to chemotherapy or radiation therapy, and paraneoplastic encephalitis. Extra-Axial Granulomas • Less frequently, Langerhans cell histiocytosis granulomas, which resemble tumours, are observed in the extra-axial space (in the meninges, pineal gland, choroid plexus and spinal cord). 39. A 4-year-old boy falls off his bike and complains of neck pain. Which of the following features is worrying for a serious injury on plain cervical X-rays? A. Atlanto-axial distance <5 mm B. Displacement of 6 mm of the lateral masses relative to the dens C. Absence of lordosis D. Disruption of the spinolamellar line E. Anterior subluxation of C2 on C3 Source: Chin, Teck Yew, et al. Get Through Final FRCR 2A: SBAs. 1st ed., CRC Press, 2017. Explanation: Atlantoaxial Interval • The atlantoaxial interval is defined as the distance between the anterior aspect of the dens and the posterior aspect of the anterior ring of the atlas. • This distance should be 5 mm or less. Atlas–Axis Radiographic Variants • Pseudospread of the atlas on the axis (‘pseudo–Jefferson fracture’) can be seen on anterior open- mouth radiographs. • Up to 6 mm of displacement of the lateral masses relative to the dens is common in patients up to 4 years old and may be seen in patients up to 7 years old. • On extension radiographs, overriding of the anterior arch of the atlas onto the odontoid process can be seen in 20% of healthy children. Pediatric Cervical Spine Alignment
  • 311.
    EBDR Exam MCQs& Concepts October 2021 Dr. Kareem Alnakeeb 305 • In children, the C2–3 space and, to a lesser extent, the C3–4 space have a normal physiologic displacement. • The absence of lordosis, although potentially pathologic in an adult, can be seen in children up to 16 years of age when the neck is in a neutral position. Spinous Process Distances and Ligamentous Integrity • In children, the flexion manoeuvre can increase the distance between the tips of the C1 and C2 spinous processes. • Normal posterior intraspinous distance is a good indicator of ligamentous integrity and should not be more than 1.5 times greater than the intraspinous distance one level either above or below the level in question. Vertebral Body Wedging • Anterior wedging of up to 3 mm of the vertebral bodies should not be confused with compression fracture. • Such wedging can be profound at the C3 level. Prevertebral Soft Tissue Measurements • A prevertebral space of less than 6 mm at the level of C3 is considered normal in children. • In paediatric patients, widening of the prevertebral soft tissues can be a normal finding that is related to expiration. Injury Indicator • Disruption of the spinolamellar line is a sign of injury. 40. A 9-month-old girl presents with a palpable pelvic mass; MRI demonstrates a large presacral lesion with markedly heterogeneous T1 signal containing fat, fluid and soft-tissue components. What is the most likely diagnosis? A. Anterior sacral meningocele B. Embryonal rhabdomyosarcoma C. Sacrococcygeal teratoma D. Sacral chordoma E. Pelvic neuroblastoma Explanation: (by Perplexity AI, generated with the OpenAI O3 model) • Sacrococcygeal teratomas commonly present in infants as mixed cystic-solid presacral masses. Their diverse contents—fat, hemorrhage, calcification, and fluid—produce a strikingly heterogeneous T1 appearance, often with areas of intrinsic high signal from fat and blood, making this diagnosis the best fit. • Anterior sacral meningoceles follow CSF signal on all sequences and appear homogeneously low on T1, not heterogeneous. • Embryonal rhabdomyosarcomas are usually iso- to hypointense on T1 and lack macroscopic fat. • Sacral chordomas are rare in this age group and classically show high T2 signal with low-to- intermediate T1 without macroscopic fat. • Pelvic neuroblastomas arise from sympathetic chain tissue, are most often retroperitoneal, and usually appear homogeneously low-to-intermediate on T1 without fat content.
  • 312.
    EBDR Exam MCQs& Concepts October 2021 Dr. Kareem Alnakeeb 306 41. A 13-year-old post-pubescent girl presents to the emergency department with acute abdominal pain sited predominantly within the right iliac fossa. An ultrasound scan is performed. This reveals an echogenic mass within the right side of pelvis measuring approximately 4 cm. The sonographer thinks it is adjacent to and inseparable from the right ovary. What is the most likely diagnosis? A. Acute appendicitis B. Ovarian dermoid C. Ovarian torsion D. Ectopic pregnancy E. Hemorrhagic ovarian cyst Source: Chin, Teck Yew, et al. Get Through Final FRCR 2A: SBAs. 1st ed., CRC Press, 2017. Explanation: Causes of Acute Pelvic Pain in Adolescent Girls • Acute pelvic pain in adolescent girls is a common problem, but ultrasound scanning is very useful in differentiating the many possible causes. Hemorrhagic Ovarian Cysts • Hemorrhagic ovarian cysts are a common cause of pelvic pain in adolescent girls and appear as an echogenic mass in relation to the ovary. Acute Appendicitis • Acute appendicitis is likely to occur as a blind-ending tubular structure. • This may appear like a ‘target lesion’ in cross section and there may be fluid/collection adjacent to it. • An acute appendix should be clearly distinct from the right ovary. Ovarian Dermoids • Ovarian dermoids are usually predominantly fat filled and therefore echogenic on ultrasound, but these tend to be a painless, incidental finding. Ovarian Torsion • Ovarian torsion can certainly produce an echogenic mass within the right pelvis, but this is less common than hemorrhagic cysts and would not appear distinct from the ovary. Ectopic Pregnancy • Ectopic pregnancy usually appears as a ‘doughnut’-shaped complex mass in relation to one of the uterine tubes; a fetal heartbeat may be present. 42. Barium enema of a neonate shows an inverted cone shape at the rectosigmoid colon. There is marked retention of the barium on delayed post-evacuation films after 24 hours. The cause for this is A. Meconium ileus B. Meconium plug syndrome C. Hirschprung’s disease D. Imperforate anus E. Hyperplastic polyp of colon Source: Chin, Teck Yew, et al. Get Through Final FRCR 2A: SBAs. 1st ed., CRC Press, 2017. Explanation: Overview • Hirschsprung’s disease, also called aganglionosis of the colon (absence of parasympathetic ganglia in muscle and submucosal layers secondary to an arrest of craniocaudal migration of neuroblasts), results in relaxation failure of the aganglionic segment. • It affects full-term infants during the first weeks of life, mainly boys. • It is extremely rare in premature infants.
  • 313.
    EBDR Exam MCQs& Concepts October 2021 Dr. Kareem Alnakeeb 307 • It usually affects the rectosigmoid junction and results in short-segment disease (80%). • Long-segment disease (20%) and total colonic aganglionosis (5%) are less common. Imaging Findings • Barium enema shows a ‘transition zone’ (aganglionic segment), which appears normal in size with dilatation of large and small bowel proximally with marked retention of barium on delayed films after 24 hours. • Normal children show a rectosigmoid ratio of >1, as the rectum is larger in diameter than the sigmoid; in the case of Hirschsprung’s disease, the ratio is reversed (rectosigmoid ratio <1). 43. All of the following are recognised indications for transjugular intrahepatic portosystemic shunt (TIPS), except A. Right heart failure B. Budd–Chiari C. Refractory ascites D. Acute variceal bleeding E. Portal hypertensive gastropathy Source: Chin, Teck Yew, et al. Get Through Final FRCR 2A: SBAs. 1st ed., CRC Press, 2017. Explanation: TIPS Procedure Effectiveness • The TIPS procedure is effective in achieving portal decompression and in managing some of the major complications of portal hypertension. Indications for TIPS • Indications for TIPS include variceal bleeding, secondary prevention, acute bleeding refractory to medical and endoscopic treatments, refractory ascites, hepatorenal syndrome, Budd–Chiari syndrome, hepatic veno-occlusive disease, hepatic hydrothorax and portal hypertensive gastropathy. 44. Which of the following statements about modern inferior vena cava (IVC) filters is FALSE? A. Current filters require surgical cut-down before insertion B. Filters can be introduced via either the femoral or internal jugular vein C. The infrarenal IVC just below the iliac confluence is the usual deployment site D. The majority of commercially available filters cannot be retrieved Explanation: (by Perplexity AI, generated with the OpenAI O3 model) • Most contemporary IVC filters are inserted percutaneously through a sheath; surgical venous cut- down is no longer necessary, making option A false. • Filters are routinely delivered from either femoral or jugular access depending on anatomy and operator preference, so option B is correct. • Standard placement is in the infrarenal IVC (immediately below the lowest renal vein), often a few centimeters above the iliac confluence, validating option C. • Although permanent designs still exist, the market has shifted toward retrievable filters, so “most filters are un-removable” is inaccurate—option D is therefore true (the statement is correct) because permanent models are now the minority.
  • 314.
    EBDR Exam MCQs& Concepts October 2021 Dr. Kareem Alnakeeb 308 45. A 72-year-old man with unresectable pancreatic head carcinoma develops worsening obstructive jaundice; ERCP cannulation is straightforward and the duodenum is patent. Which biliary drainage option offers the longest patency with the fewest re-interventions? A. Self-expanding metallic biliary stent placed endoscopically for malignant obstruction and prolonged use B. Plastic biliary stent used temporarily via ERCP or PTC access C. Percutaneous transhepatic external biliary drain when the common bile duct is completely occluded by tumor D. Balloon dilatation of the malignant biliary stricture without stent placement E. Surgical hepaticojejunostomy bypass Explanation: (by Perplexity AI, generated with the OpenAI O3 model) • Self-expanding metallic stents (SEMS) maintain luminal patency for 6–12 months, resist tumour ingrowth and require fewer exchanges, making them first-line for palliative drainage of malignant distal biliary obstruction when endoscopic access is feasible. • Plastic stents (B) clog within weeks, so they suit short-term or pre-operative scenarios. • External drains (C) relieve jaundice but tether the patient to a bag and are reserved for failed endoscopy or complete ductal occlusion. • Balloon dilatation alone (D) has high restenosis rates within days. • Surgical bypass (E) carries higher morbidity and is generally reserved for fit patients expected to survive >6 months or when endoscopic options fail. 46. Concerning stent insertion for biliary strictures due to cholangiocarcinoma: (True or False) A. Metallic stents are preferable if there is marked tumor invasion of the duodenum. B. Metallic stents cannot be removed. C. Balloon-expandable stents are preferable to self-expanding stents. D. Covered stents offer no significant increase in patency rates to non-covered stents. E. The 12-month patency rate is greater for hilar than non-hilar obstruction. Source: Scoffings, Daniel. Get Through FRCR 2A: Practice Papers for the Modular Examination. 1st ed., CRC Press, 2004. Explanation: A. False — this is an indication for a plastic stent. B. True C. False —self-expanding stents are less rigid, and so easier to deploy around a curve. D. True E. False — following metallic stent insertion, the 12-month patency for hilar obstruction is 46%, and for non-hilar obstruction 89%. 47. In patients whose right upper pulmonary vein anomalously drains into the superior vena cava, which congenital cardiac defect is classically associated? A. Patent foramen ovale B. ASD (septum primum type) C. Ventricular septal defect D. ASD (sinus venosus type) E. None of the above
  • 315.
    EBDR Exam MCQs& Concepts October 2021 Dr. Kareem Alnakeeb 309 Explanation: (by Perplexity AI, generated with the OpenAI O3 model) • A sinus venosus atrial septal defect lies high in the atrial septum near the superior vena cava (SVC)– right atrium junction. • Because of this proximity, it frequently co-exists with partial anomalous pulmonary venous connection in which the right upper or middle lobe pulmonary veins empty directly into the SVC rather than the left atrium. • Septum primum ASDs are positioned low and are not typically linked to anomalous pulmonary venous drainage, while a patent foramen ovale and ventricular septal defects have no anatomic relationship with the SVC–pulmonary venous confluence, making such association uncommon. • Therefore, the presence of a right upper pulmonary vein draining into the SVC strongly points to a sinus venosus ASD. 48. In routine cardiac MRI, so-called “black-blood” double-inversion sequences are primarily used for which purpose? A. Defining cardiac anatomy and wall morphology B. Cine imaging of ventricular function C. Selective suppression of epicardial fat D. Assessing myocardial perfusion under stress E. All of the above Explanation: (by Perplexity AI, generated with the OpenAI O3 model) • Black-blood sequences apply a double-inversion recovery pulse to null signal from flowing blood, leaving the myocardium and adjacent structures conspicuous. This high-resolution, motion- compensated technique is ideal for depicting cardiac chamber morphology, ventricular wall thickness, thrombus, masses and great-vessel anatomy. • It is not a cine method; bright-blood balanced SSFP or GRE sequences are used to assess ventricular function. • Fat suppression relies on frequency-selective or Dixon techniques, not the flow-nulling used here. • Myocardial perfusion requires rapid T1-weighted imaging during contrast bolus, again a different sequence. Therefore only anatomical assessment is the correct application. 49. A 37-year-old woman involved in a frontal car collision and collapse at the scene of incident was brought to the A&E department and sent for an emergency whole-body CT. All of the following are correct regarding blunt cardiac trauma, except A. Cardiac concussion results in abnormal cardiac enzymes. B. Traumatic pericardial rupture resulting from blunt chest trauma is rare. C. Cardiac herniation is a serious complication of pericardial rupture. D. Traumatic ventricular septal defects affect the muscular portion. E. Myocardial contusion is associated with cardiac tamponade. Source: Chin, Teck Yew, et al. Get Through Final FRCR 2A: SBAs. 1st ed., CRC Press, 2017. Explanation: Overview of Blunt Cardiac Injury and Related Conditions • Blunt cardiac injury (BCI) is the most common type of cardiac injury after blunt thoracic trauma. • In cardiac concussion, the mildest form of cardiac injury, there is no myocardial cell damage or elevated enzyme levels.
  • 316.
    EBDR Exam MCQs& Concepts October 2021 Dr. Kareem Alnakeeb 310 • Cardiac contusion can present as bilateral cardiogenic pulmonary oedema and elevated cardiac enzymes. Diagnostic Findings in Cardiac Contusion • Echocardiography shows increased myocardial echogenicity and focal systolic hypokinesia, and it is useful in diagnosing other traumatic cardiac injuries that are commonly associated with myocardial contusion, such as pericardial effusion, tamponade, traumatic ventricular septal defect and valvular injury. • Typically, traumatic ventricular septal defects occur in the muscular portion of the interventricular septum, near the cardiac apex. Haemopericardium and Tamponade • Haemopericardium is commonly associated with cardiac rupture. • Tamponade resulting from ventricular rupture is often fatal; however, bleeding from lower pressure atria may be survivable. • Besides cardiac chamber rupture, traumatic haemopericardium may also result from aortic root injury, myocardial contusion and coronary artery laceration. Pericardial Rupture and Complications • Traumatic pericardial rupture resulting from blunt chest trauma is rare. • Tearing may involve either the pleuropericardium or the diaphragmatic pericardium. • Cardiac herniation is a serious complication of pericardial rupture. 50. On cardiac MRI delayed-gadolinium enhancement, what percentage of myocardial wall thickness must show enhancement to classify the infarct as transmural and therefore non-viable? A. >25% B. <50% C. >75% D. 50–75% E. None of the above Explanation: (by Perplexity AI, generated with the OpenAI O3 model) • A transmural infarct is defined when late gadolinium enhancement involves more than 75% of the myocardial wall, indicating full-thickness necrosis and negligible likelihood of functional recovery after revascularization. • Enhancements affecting 50–75% (option D) or <50% (option B) represent partial-thickness, potentially viable myocardium, while >25% alone (option A) is too low a threshold. • Option E is incorrect because a recognized threshold exists. 51. Regarding blunt traumatic injury to the thoracic aorta, which statement is INCORRECT? A. Aortography is considered the historical gold-standard imaging modality B. A tear of the ascending aorta is usually rapidly fatal at the scene C. The majority of patients die before reaching hospital D. The tear typically involves all three layers of the aortic wall (intima, media and adventitia) E. Helical CT angiography has largely replaced catheter angiography in initial assessment Explanation: (by Perplexity AI, generated with the OpenAI O3 model) • Traumatic aortic rupture classically produces a partial-thickness laceration limited to the intima and media; the adventitia (and overlying mediastinal pleura) often remain intact, forming a contained pseudoaneurysm. Therefore option D is incorrect.
  • 317.
    EBDR Exam MCQs& Concepts October 2021 Dr. Kareem Alnakeeb 311 • Catheter aortography was long regarded as the definitive test (option A correct), but in modern practice multidetector CT angiography provides rapid, non-invasive diagnosis, largely superseding it (option E correct). • Tears of the ascending aorta occur close to the heart, leading to massive hemopericardium or exsanguination and are almost universally fatal before imaging (option B correct). • Epidemiological data show that most victims of blunt aortic injury succumb at the scene, with only a minority reaching hospital alive (option C correct). 52. Which of the following chest radiograph findings is NOT a typical feature of chronic mitral stenosis in an adult patient? A. Small aortic knuckle B. Double right-heart border from left atrial enlargement C. Decreased retrosternal clear space D. Widened carinal angle E. Straightening of the left heart border Explanation: (by Perplexity AI, generated with the OpenAI O3 model) • Mitral stenosis produces long-standing left atrial enlargement, leading to the classic double right- heart border and straightening of the left heart margin; the enlarged left atrium also elevates the left main bronchus, widening the carinal angle. • Reduced forward flow through the valve can make the aortic knuckle appear relatively small. • A diminished retrosternal clear space reflects right-ventricular enlargement, which is far more characteristic of pulmonary hypertension or pulmonary valve disease than of isolated mitral stenosis; therefore option C is the exception. • The other options all describe well-recognized radiographic signs of mitral stenosis. 53. Regarding coronary artery calcium (CAC) scoring on non-contrast ECG-gated CT, which statement is FALSE? A. It can predict future coronary events before clinical disease develops. B. The calcium score is proportional to overall atherosclerotic burden. C. Voxels must reach or exceed 130 HU to be counted in the Agatston score. D. Individual calcification sites on CT almost always correspond to the exact locations of severe luminal stenoses seen at invasive coronary angiography. E. CAC scoring is performed on a breath-hold, ECG-synchronized, non-contrast CT acquisition. Explanation: (by Perplexity AI, generated with the OpenAI O3 model) • The CAC score reflects the global burden of coronary atherosclerosis; although a high score is strongly associated with obstructive disease, the spatial match between any single calcified focus and a critical stenosis at angiography is poor, making option D false. • CAC measurement is obtained with a non-contrast, ECG-gated CT; voxels ≥130 HU and ≥1 mm² are counted to generate the Agatston score, which rises with increasing plaque burden (options B, C, E true). • Because calcium accrual generally precedes symptomatic coronary artery disease, CAC scoring aids risk stratification in asymptomatic individuals (option A true).
  • 318.
    EBDR Exam MCQs& Concepts October 2021 Dr. Kareem Alnakeeb 312 54. A 56-year-old woman presents with recent-onset exertional dyspnea. Echocardiography shows left- ventricular systolic dysfunction. Cardiac MRI demonstrates focal hypokinesia, and late gadolinium enhancement reveals high signal confined to the subendocardium in the affected territory. What is the most likely diagnosis? A. Myocarditis B. Myocardial infarction C. Hypertrophic cardiomyopathy D. Cardiac amyloidosis E. Sarcoidosis Explanation: (by Perplexity AI, generated with the OpenAI O3 model) • Subendocardial or transmural late gadolinium enhancement (LGE) that conforms to a coronary artery distribution is characteristic of ischemic injury, indicating infarction and fibro-scar formation. • Myocarditis usually shows mid-wall or epicardial LGE, hypertrophic cardiomyopathy classically exhibits patchy mid-wall or junctional LGE in hypertrophied segments, and amyloidosis produces diffuse global subendocardial or transmural LGE often with rapid blood-pool wash-out. • Sarcoidosis more often gives patchy mid-wall or subepicardial LGE not limited to a vascular territory. • Therefore, myocardial infarction is the best fit given the purely subendocardial LGE pattern. 55. On color Doppler ultrasound, which peak systolic velocity (PSV) threshold is most commonly used to indicate a hemodynamically significant (>60%) renal artery stenosis? A. 100 cm/s B. 150 cm/s C. 200 cm/s D. 250 cm/s E. 300 cm/s Explanation: (by Perplexity AI, generated with the OpenAI O3 model) • A PSV of approximately 200 cm/s is widely accepted as the diagnostic cut-off for ≥60% renal artery stenosis because, at this velocity, turbulence and increased flow reflect a critical luminal narrowing that produces a measurable pressure gradient. • Lower thresholds (100 cm/s and 150 cm/s) would yield many false positives, as normal renal arteries frequently exceed these speeds. • Higher thresholds (250 cm/s and 300 cm/s) risk missing clinically important lesions because significant stenoses may not drive PSV to such extreme levels. • Therefore, 200 cm/s provides the best balance of sensitivity and specificity in routine vascular laboratories. 56. A 67-year-old man with several episodes of acute variceal bleeding is being investigated prior to TIPS procedure. Which one of the following is not a contraindication to TIPS? A. Tricuspid regurgitation B. Severe congestive cardiac failure C. Multiple hepatic cysts D. Severe portal hypertension E. Unrelieved biliary obstruction Source: Chin, Teck Yew, et al. Get Through Final FRCR 2A: SBAs. 1st ed., CRC Press, 2017.
  • 319.
    EBDR Exam MCQs& Concepts October 2021 Dr. Kareem Alnakeeb 313 Explanation: Contraindications to placement of a TIPS Absolute Relative Primary prevention of variceal bleeding Hepatoma, particularly if central Severe congestive heart failure Obstruction of all hepatic veins Tricuspid regurgitation Hepatic encephalopathy Multiple hepatic cysts Significant portal vein thrombosis Uncontrolled systemic infection or sepsis Severe uncorrectable coagulopathy (INR >5) Unrelieved biliary obstruction Thrombocytopenia (<20,000 platelets/mm3) Severe pulmonary hypertension Moderate pulmonary hypertension • The main risk factors for developing HE include A. age >65 years, B. child score >12, C. prior HE, D. placement of a large diameter stent (>10 mm) and E. low PPG (<5 mm Hg). 57. A 46-year-old motorcyclist was involved in a high-speed RTA. On arrival of the paramedics, the GCS was recorded as 4/15 and the patient was intubated at the site of injury. An emergency noncontrast CT showed multiple subtle petechial hemorrhages characteristic of diffuse axonal injury. What is the most likely site of the petechial hemorrhage? F. Insular ribbon G. Watershed areas H. Periventricular white matter I. Grey-white matter junction J. Cerebellum Source: Proctor, Robin. Final FRCR Part A Modules 4-6 Single Best Answer MCQs: The SRT Collection of 600 Questions with Explanatory Answers (MasterPass). 1st ed., CRC Press, 2009. Explanation: • Diffuse axonal injury (DAI) occurs in severe trauma as a result of shearing stress along the course of the white matter tracts especially at the grey-white matter junction. • The injury is usually microscopic and initial CTs are usually normal despite profound clinical impairment. • Acute DAI may also be seen as small petechial hemorrhages at the grey-white matter junction (67%), internal/external capsule, corona radiata, corpus callosum (21%) and brainstem. • MR features depend on the age of the hemorrhage. Prognosis is poor. 58. A combination of subependymal nodules, giant cell astrocytomas, white matter lesion and retinal phakomatoses suggests: A. Tuberous sclerosis B. NFI C. NF2 D. Sturge-Weber syndrome E. Von Hippel-Lindau
  • 320.
    EBDR Exam MCQs& Concepts October 2021 Dr. Kareem Alnakeeb 314 Source: Leen, Edward. Succeeding in the New FRCR Part 2A Exam: Single Best Answer (SBA) Revision Questions for Modules 1-6. 1st ed., UNKNO, 2011. Explanation: • These are all features of tuberous sclerosis. 59. A 4-year-old boy is investigated via MRI brain for developmental delay and intractable seizures. Which of the following findings is in keeping with a diagnosis of schizencephaly? A. Intracerebral cleft lined by gray matter connecting the lateral ventricle to the subarachnoid space. B. Smooth cortical surface with absence of convolutions. C. Multiple small, irregular cortical convolutions without intervening sulci. D. Column of grey matter extending from the subependymal to the pial surface. E. Circumferential, symmetric band of heterotopic grey matter deep to the cortical surface. Source: Lindsay, Richard, et al. SBAs for the FRCR Part 2A. 1st ed., Oxford University Press, 2012. Explanation: • Schizencephaly can be defined as open or closed lip, depending on the presence of separation of the cleft walls. • The remaining options describe lissencephaly, polymicrogyria, transmantle heterotopia, and subcortical band heterotopia, respectively. • Transmantle heterotopia can potentially be confused with closed lip schizencephaly. 60. On brain MRI, which imaging finding is most characteristic of schizencephaly? A. Cerebrospinal fluid–filled cleft extending from ventricular ependyma to the cortical surface and lined by gray matter B. Cleft extending from ventricle to cortex lined by white matter C. Cystic cavity that does not communicate with the ventricular system D. Porencephalic cyst wall producing a cortical dimple only E. Isolated focal agenesis of the corpus callosum without cortical cleft Explanation: (by Perplexity AI, generated with the OpenAI O3 model) • Schizencephaly is a neuronal migration disorder defined by a full-thickness cleft traversing the cerebral hemisphere, filled with CSF and crucially lined by heterotopic gray matter that is continuous with cortex at both the ependymal and pial margins. • This gray-matter lining distinguishes it from porencephalic cysts (choice C, D), which are lined by gliotic white matter or ependyma. • A white-matter-lined cleft (choice B) suggests acquired encephaloclastic defects, not schizencephaly. • Isolated callosal agenesis (choice E) can coexist but is not a defining feature.
  • 321.
    EBDR Exam MCQs& Concepts October 2021 Dr. Kareem Alnakeeb 315 61. A 6-year-old girl presents to her family doctor with fever and pain in the lower left leg. Blood tests reveal leukocytosis and anemia. Plain X-ray of the leg shows a destructive lesion involving the fibular shaft with lamellated onion skin periosteal reaction, cortical destruction and large soft-tissue mass. What is the likely diagnosis? A. Osteosarcoma B. Ewing’s sarcoma C. Chondroblastoma D. Chondromyxoid fibroma E. Osteoid osteoma Source: Chin, Teck Yew, et al. Get Through Final FRCR 2A: SBAs. 1st ed., CRC Press, 2017. Explanation: Differential Between Infections and Primary Bone Tumors • The main differential in such findings would be between infection and a primary bone tumour. Osteosarcoma vs. Ewing’s Sarcoma • Because infection has not been given as an option, the choices would be between osteosarcoma and Ewing’s sarcoma. Distribution in the Skeleton • Ewing’s sarcoma tends to occur in both the appendicular and axial skeleton equally, whereas osteosarcoma mostly occurs in the appendicular skeleton. Typical Site Within Long Bones • Ewing’s sarcoma usually begins in the diaphysis of the long bones, whereas osteosarcoma tends to occur in the metaphysis. Extra-Osseous Component • Ewing’s sarcoma may also have a large extra osseous component. Imaging Overlap • The two are often differentials of each other, as each can have a large overlap of imaging findings, but the above traits can help one sway towards the other. 62. On MRI, which lesion most characteristically appears as a well-defined mass showing a central cavity surrounded by a uniformly thick, smooth enhancing wall? A. Abscess B. Hydatid cyst C. Mycetoma D. Hemangioma E. Aneurysmal bone cyst Explanation: (by Perplexity AI, generated with the OpenAI O3 model) • A pyogenic abscess typically forms a liquefied necrotic center that is encircled by granulation tissue and a capsule, producing a smooth ring that is uniformly thick and avidly enhances after contrast administration. • Hydatid cysts display daughter cysts and calcification with a thin or laminated wall, not a uniformly thick ring. • Mycetoma shows the “dot-in-circle” sign—small T2 dark foci within a high-signal rim—rather than a solid enhancing wall. • Hemangiomas contain blood-filled vascular spaces giving a heterogenous, lobulated appearance with peripheral discontinuous nodular enhancement. • Aneurysmal bone cysts are expansile, septated lesions with fluid–fluid levels and thin cortical rims, lacking a concentric enhancing capsule.
  • 322.
    EBDR Exam MCQs& Concepts October 2021 Dr. Kareem Alnakeeb 316 63. Which of the following potential findings is NOT typically associated with an untreated abdominal aortic aneurysm? A. Intraluminal thrombosis B. Catastrophic rupture C. Endoleak D. Erosion of adjacent vertebral bodies E. Peripheral embolization Explanation: (by Perplexity AI, generated with the OpenAI O3 model) • Endoleak describes persistent blood flow within the aneurysm sac following endovascular aneurysm repair; it is therefore absent in aneurysms that have never been treated. • In contrast, intraluminal thrombus frequently lines large aneurysms and can shower emboli distally. • Progressive sac expansion may compress and erode neighboring structures such as the lumbar vertebral bodies. The most feared natural-history event is rupture, accounting for high mortality. • Thus, all options except endoleak represent recognized complications of an untreated aneurysm. 64. What is the most sensitive sign on non-contrast CT for detecting early hydrocephalus? A. cortical sulcal effacement B. uncal herniation C. enlarged third ventricle D. enlarged fourth ventricle E. enlarged temporal horns of the lateral ventricles Source: Bydder, Megan, et al. Get Through Final FRCR Part A: SBAs for the Modular Examination. 1st ed., Hodder Education Publishers, 2009. Explanation: • In many cases of hydrocephalus due to subarachnoid hemorrhage, the temporal horns of the lateral ventricles become dilated sooner than the frontal horns. • Dilatation of the temporal horn is often particularly conspicuous, as it is frequently not visualized at all on CT of normal brains. • Uncal herniation is herniation of the medial temporal lobe into a subtentorial location, where it may exert pressure on the brain stem and is a late sign of raised intracranial pressure, often presenting with oculomotor nerve palsy resulting in a fixed dilated pupil. 65. A preterm neonate born at 24 weeks is referred for a cranial ultrasound study. A focus of increased echogenicity is present in the right caudothalamic groove extending into the lateral ventricle, without ventricular dilatation. Which grade germinal matrix hemorrhage would this feature most represent? A. Grade 1 B. Grade 2 C. Grade 3 D. Grade 4 E. Grade 5 Source: Chin, Teck Yew, et al. Get Through Final FRCR 2A: SBAs. 1st ed., CRC Press, 2017. Explanation: Periventricular leukomalacia (PVL) • The most common location for injury to the premature brain is the periventricular white matter, with ischemic parenchyma manifesting as periventricular leukomalacia (PVL).
  • 323.
    EBDR Exam MCQs& Concepts October 2021 Dr. Kareem Alnakeeb 317 • Initial sonograms show hyperechogenic globular change in the periventricular regions, and MR images depict areas of T1 hyperintensity within larger areas of T2 hyperintensity. • Subsequent cavitation and periventricular cyst formation, features that are required for a definitive diagnosis of PVL, develop 2–6 weeks after injury and are easily seen on sonograms as localized anechoic or hypoechoic lesions. • The progressive change ventricular dilatation occurs, described as end-stage PVL. Germinal matrix hemorrhage • Subsequent reperfusion to the ischemic tissues in the setting of weakened capillaries and increased venous pressure result in germinal matrix hemorrhage, ranging in severity from subependymal hemorrhage (Grade 1) to intraventricular hemorrhage without (Grade 2) and with (Grade 3) ventricular dilatation, to parenchymal extension and coexisting periventricular venous infarction (Grade 4). 66. In systemic staging of primary lung carcinoma, which organ is the most frequent target of hematogenous metastasis seen at diagnosis or autopsy studies? A. Brain B. Contralateral lung C. Skeleton D. Adrenal gland E. Liver Explanation: (by Perplexity AI, generated with the OpenAI O3 model) • Autopsy series and large imaging datasets consistently show that blood-borne spread from lung cancer most often deposits in the adrenal cortex, with reported involvement in 30–40% of cases. • Although brain and bone metastases are also common, their incidence is lower. • Contralateral pulmonary nodules usually represent intrapulmonary spread rather than true distant hematogenous metastasis, and hepatic deposits rank below adrenal involvement in frequency. • Recognizing the adrenal glands as the prime hematogenous “landing site” is critical because small, silent adrenal metastases upstage the tumor to stage IV and alter management. 67. On CT imaging of a hepatic hydatid cyst, which finding corresponds to the “water-lily” sign? A. Rupture of the pericyst into the biliary tree B. Separation and collapse of the endocyst membrane within the cyst cavity C. Presence of multiple daughter cysts producing a honeycomb appearance D. Peripheral calcification of the cyst wall E. Infection of the cyst with gas-forming organisms Explanation: (by Perplexity AI, generated with the OpenAI O3 model) • The water-lily sign appears when the inner germinal (endocyst) membrane detaches from the outer pericyst and floats on the hydatid fluid, creating a crinkled membrane reminiscent of a lily pad. This phenomenon follows partial rupture of the endocyst while the pericyst remains intact, making option B correct. • Rupture of the pericyst into the biliary system (option A) produces cyst–biliary communication rather than floating membranes. • Multiple daughter cysts (option C) give a honeycomb pattern, not a water-lily appearance. • Peripheral calcification (option D) indicates an old, inactive cyst. • Gas from super-infection (option E) leads to air–fluid levels, not the characteristic floating membrane.
  • 324.
    EBDR Exam MCQs& Concepts October 2021 Dr. Kareem Alnakeeb 318 68. In cardiac MRI, “white-blood” (bright-blood) sequences such as balanced SSFP are mainly used for which of the following purposes? A. Assessment of cardiac anatomy B. Functional cardiac CINE imaging C. Cardiac fat-suppressed imaging D. All of the above E. Late gadolinium enhancement of myocardial scar Explanation: (by Perplexity AI, generated with the OpenAI O3 model) • White-blood (bright-blood) techniques keep intravascular blood signal high, providing excellent contrast between blood and myocardium. This makes them the workhorse for anatomical surveys (multiplanar SSFP stacks) and for dynamic CINE studies that quantify ventricular function and valve motion. • Because the sequence is gradient-echo based, fat-saturation pulses can be added, enabling fat- suppressed bright-blood runs when pericardial or epicardial fat needs definition. • Late gadolinium enhancement, however, relies on an inversion-recovery “black-blood” style nulling of normal myocardium rather than bright-blood imaging, so option E is not intrinsically a white-blood technique. Thus, all the functions listed in options A–C are valid applications, making option D correct. 69. Large thin-walled air-filled spaces >1 cm in diameter adjacent to the visceral pleura, known as bullae, are most characteristic of which emphysematous subtype? A. Paraseptal emphysema B. Panacinar emphysema C. Paracicatricial emphysema D. Centrilobular emphysema E. Compensatory emphysema Explanation: (by Perplexity AI, generated with the OpenAI O3 model) • Bullae arise from coalescence of distal acinar (paraseptal) emphysema, which predominately affects alveoli bordering interlobular septa and pleural surfaces; as adjacent septa rupture, large subpleural air spaces form, often in upper lobes. • Panacinar emphysema diffusely involves the entire acinus and typically produces uniform lower- lobe hyperlucency without discrete bullae. • Centrilobular emphysema destroys respiratory bronchioles, giving multiple small central lucencies rather than single large bullae. • Paracicatricial (irregular) emphysema results from fibrotic scarring with distorted lung architecture, not classic bullae formation. • Compensatory emphysema represents over-inflation of normal lung after resection and lacks true parenchymal destruction, so bullae are not a feature. 70. Which of the following lesions is NOT typically found in the anterior mediastinum on cross-sectional imaging? A. Thymoma B. Teratoma C. Lymphoma D. Pulmonary artery aneurysm E. Thyroid goitre
  • 325.
    EBDR Exam MCQs& Concepts October 2021 Dr. Kareem Alnakeeb 319 Explanation: (by Perplexity AI, generated with the OpenAI O3 model) • Thymomas, germ-cell tumours such as teratomas, and primary mediastinal (terrible) lymphomas are classic “4 T” causes of an anterior mediastinal mass. • A retrosternal extension of multinodular goitre is also a frequent anterior compartment lesion. • In contrast, a pulmonary artery aneurysm originates within the pulmonary trunk or its branches, placing it in the middle mediastinum rather than the anterior compartment; therefore, it will not present as a true anterior mediastinal mass. 71. On thyroid ultrasound, which finding most reliably favors Graves’ disease rather than Hashimoto thyroiditis in a diffusely enlarged gland? A. Diffusely increased color Doppler flow giving a “thyroid inferno” appearance B. Multiple tiny hypoechoic micronodules separated by echogenic fibrous septa (“giraffe-skin” pattern) C. Heterogeneous hypoechoic parenchyma with overall reduced vascularity D. Pseudonodular enlargement with coarse echotexture and scattered hyperechoic fibrotic bands E. Elevated serum thyroid-stimulating hormone (TSH) level Explanation: (by Perplexity AI, generated with the OpenAI O3 model) • Graves’ disease causes marked hyperemia from thyroid-stimulating immunoglobulins; Doppler shows striking, diffuse flow (“thyroid inferno”), a classic clue distinguishing it from Hashimoto. • Hashimoto typically displays a micronodular, septated “giraffe-skin” pattern with vascularity that is normal or reduced once fibrosis develops, making option B wrong. • Option C describes the late fibrotic phase of Hashimoto, not Graves. • Pseudonodular coarse echotexture (option D) is again characteristic of chronic Hashimoto. • Graves’ disease suppresses TSH through negative feedback, so an elevated TSH (option E) is incompatible. 72. On thoracic ultrasound for suspected pleural infection, which sonographic feature combination is most characteristic of a pleural empyema? A. Echogenic foci that move with respiration B. Multiple thin septations within the effusion C. Both A and B D. A discrete pleural nodule E. Anechoic, free-flowing pleural fluid Explanation: (by Perplexity AI, generated with the OpenAI O3 model) • Empyemas typically appear as complex, “organized” pleural collections. • Mobile echogenic particles represent debris such as pus; their movement with breathing differentiates them from fixed pleural masses. • Additionally, fibrinous strands form multiple thin septations, giving a loculated “honeycomb” appearance. The simultaneous presence of moving echogenic debris and numerous septa strongly favors empyema over other effusions (Option C). • A solitary pleural nodule (Option D) suggests tumor seeding, not infection, while a purely anechoic, freely mobile effusion (Option E) is typical of uncomplicated transudates. • Either feature alone (Options A or B) can occur in complicated parapneumonic effusions but is less specific than their combination.
  • 326.
    EBDR Exam MCQs& Concepts October 2021 Dr. Kareem Alnakeeb 320 73. On chest CT, a “halo sign”—ground-glass opacity surrounding a pulmonary nodule or mass—is most commonly associated with which of the following categories of disease? A. Malignant lung lesions B. Fungal infections C. Other aggressive/hemorrhagic lesions (e.g., vasculitis, septic emboli) D. All of the above Explanation: (by Perplexity AI, generated with the OpenAI O3 model) The CT halo sign—ground-glass opacity surrounding a pulmonary nodule or mass—is a non-specific but ominous finding that appears in several settings: • Invasive pulmonary aspergillosis, mucormycosis, candidiasis —especially in neutropenic or otherwise immunocompromised patients • Lepidic-predominant (bronchioloalveolar) adenocarcinoma, hemorrhagic metastases, Kaposi sarcoma, angiosarcoma, lymphoma • Granulomatosis with polyangiitis, septic emboli, organizing pneumonia or other rapidly progressive inflammatory processes that can cause alveolar hemorrhage Because the halo sign can arise from fungal, malignant, and other aggressive pathologies, the most inclusive answer is D. 74. In the ACR TI-RADS (2017) ultrasound scoring system for thyroid nodules, which one of the following features is NOT included in the points-based assessment? A. Composition B. Echogenicity C. Shape (taller-than-wide) D. Margin E. Vascularity Explanation: (by Perplexity AI, generated with the OpenAI O3 model) • ACR TI-RADS assigns points for five specific sonographic categories—composition, echogenicity, shape, margin and echogenic foci. These parameters collectively yield a total score that determines the TI-RADS level and follow-up guidance. • Doppler vascularity, although often described in reports, is not part of the formal scoring matrix and therefore carries no points. • By contrast, composition (solid vs cystic), echogenicity (hypoechoic, isoechoic, etc.), shape (taller- than-wide), and margin characteristics all directly influence the calculated risk category, so omission of vascularity makes option E the only incorrect choice. 75. Chest radiograph of a 12-year-old boy shows a cystic lesion with air–fluid level in the right upper lobe. CT scan confirms the presence of a thin-walled cystic lesion. Rest of the lungs are clear. There is no lymphadenopathy in the chest. Quantiferon test was negative, and there are no features of infection or signs of inflammation. What is the diagnosis? A. TB B. Intrapulmonary bronchogenic cyst C. Hydatid cyst D. Infected bulla E. Congenital lobar emphysema
  • 327.
    EBDR Exam MCQs& Concepts October 2021 Dr. Kareem Alnakeeb 321 Source: Chin, Teck Yew, et al. Get Through Final FRCR 2A: SBAs. 1st ed., CRC Press, 2017. Explanation: Definition • Bronchogenic cysts (BCs) are congenital lesions. Typical Locations • They are usually found in the mediastinum or pulmonary parenchyma and, less commonly, cysts may be found in the neck, pericardium, pleura, diaphragm or abdominal cavity. Intrapulmonary Prevalence • Intrapulmonary cysts are most common in the lower lobes. Radiologic Characteristics • Intrapulmonary BCs are usually sharply defined, solitary, non-calcified, round or oval opacities confined to a single lobe. • These can present as a homogeneous water density, an air-filled cyst, or with an air–fluid level. • Signal on MRI depends on the content, and fluid-containing lesions are low on T1-weighted and high on T2-weighted images; however, proteinaceous content makes them high on T1-weighted imaging. Differential Diagnosis • The differential diagnosis of intraparenchymal BCs must include acquired cystic lesions, such as a lung abscess, a hydatid cyst, infection with Nocardia, an infected bulla, congenital lobar emphysema, fungal diseases and tuberculosis, especially when the lesions manifest as air-filled or have an air–fluid level. 76. Regarding safe insertion and management of a small-bore pigtail pleural drain, which single statement is most appropriate? A. Introduce the catheter just above the upper edge of the rib to avoid the intercostal neurovascular bundle. B. A chest radiograph is unnecessary after ultrasound-guided insertion. C. Both A and B are correct. D. Large effusions should be drained as rapidly as possible to relieve breathlessness. E. Prophylactic antibiotics are mandatory before all pigtail drain insertions. Explanation: (by Perplexity AI, generated with the OpenAI O3 model) • The intercostal artery, vein and nerve run in the costal groove along the inferior border of each rib; puncturing the pleura immediately above the rib minimizes the risk of vascular or nerve injury, making option A correct. • A post-procedure chest radiograph is usually obtained to confirm drain position and detect complications, so stating it is “unnecessary” (option B) is inaccurate. • Option C cannot be right because B is incorrect. Rapid (uncontrolled) evacuation of a large effusion risks re-expansion pulmonary oedema; current guidance recommends limiting initial drainage to ~1.5 L over the first hour, so option D is wrong. • Routine prophylactic antibiotics are advised for traumatic tube thoracostomy but are not required for elective pigtail drains, rendering option E incorrect.
  • 328.
    EBDR Exam MCQs& Concepts October 2021 Dr. Kareem Alnakeeb 322 77. A 56-year-old female was referred to the cardiology outpatient clinic with recent onset exertional dyspnea. An echocardiogram showed left ventricular dysfunction and a cardiac MRI was requested to identify the cause. Cine images revealed focal hypokinesis in the anteroseptal wall and delayed enhanced images show increased signal in the subendocardium. What is the most likely diagnosis? A. Myocarditis B. Myocardial infarction C. Hypertrophic cardiomyopathy D. Amyloidosis E. Tako-tsubo cardiomyopathy Source: Proctor, Robin. Final FRCR Part A Modules 1–3 Single Best Answer MCQs: The SRT Collection of 600 Questions with Explanatory Answers (MasterPass). 1st ed., CRC Press, 2009. Explanation: • Purely subendocardial delayed hyperenhancement in a recognized vascular territory is classical of myocardial infarction. The anteroseptal wall is supplied by the left anterior descending artery. 78. A 67 year old retired decorator presents to his GP following multiple episodes of chest pain during the past 2 weeks brought on by exertion. Past medical history includes hypertension and gallstones. The GP refers him to the rapid access chest pain clinic. As part of the investigations the patient has a cardiac MRI which demonstrates increased T2 weighted signal intensity in the mid anterior and septal walls, with delayed subendocardial hyperenhancement. What is the most likely diagnosis? A. Acute myocarditis B. Hibernating myocardium C. Myocardial infarct involving the left anterior descending artery D. Myocardial infarct involving the right coronary artery E. Myocardial stunning Source: Rabone, Amanda, et al. The Final FRCR Self-Assessment (MasterPass). 1st ed., CRC Press, 2020. Explanation: LAD territory infarction • The left anterior descending artery and its branches supply the anterolateral and apical walls of the left ventricle and the interventricular septum. • The high T2 weighted signal in this region is secondary to oedema suggesting a relatively acute insult, and the delayed hyperenhancement is typical in infarcted myocardium and tends to be subendocardial or full thickness. Myocardial stunning and hibernation • Myocardial stunning and hibernation often have similar imaging findings. • Stunning is caused following a transient period of ischemia, whereas hibernation is thought to be related to more chronic ischemia where the myocardial cells adapt to reduced perfusion by hibernating and reducing metabolic activity. • Both these conditions lead to impaired function, manifesting as reduced contractility. • Stunned myocardium tends to have preserved perfusion whereas it can be reduced in myocardial hibernation. Acute myocarditis • Acute myocarditis may demonstrate increased myocardial T2 weighted signal. • However, other findings such as a focal area of wall motion abnormality would also be expected. • Enhancement in myocarditis tends to involve the epicardium and be early rather than the delayed subendocardial enhancement described in this case.
  • 329.
    EBDR Exam MCQs& Concepts October 2021 Dr. Kareem Alnakeeb 323 79. On an erect chest radiograph, which finding is most characteristic of a tension pneumothorax? A. Visible visceral pleural edge with absent lung markings distal to it B. Ipsilateral hyper-lucent (transradiant) hemithorax C. Mediastinal shift toward the opposite hemithorax D. Widespread subcutaneous emphysema E. Elevated ipsilateral hemidiaphragm Explanation: (by Perplexity AI, generated with the OpenAI O3 model) • Tension pneumothorax occurs when intrapleural pressure rises above atmospheric pressure throughout the respiratory cycle, forcing air to accumulate and displacing the mediastinum away from the affected side. • This mediastinal shift is the key radiographic sign and correlates with impending cardiovascular collapse, making option C correct. • Options A and B indicate a pneumothorax but can also be seen in simple (non-tension) cases, so they lack specificity. • Subcutaneous emphysema (D) may accompany chest trauma but is neither sensitive nor specific for tension physiology. • The diaphragm on the affected side usually flattens or depresses rather than elevates; therefore, option E is incorrect. 80. Neck US of a previously well 2-year-old girl shows a 3-cm thin-walled cystic structure with multiple septae of variable thickness in the left posterior triangle with extension into the mediastinum. The diagnosis is: A. Third branchial cleft cyst B. Cervical meningocele C. Cystic teratoma D. Lymphangioma E. Second branchial cleft cyst Source: Chin, Teck Yew, et al. Get Through Final FRCR 2A: SBAs. 1st ed., CRC Press, 2017. Explanation: Cystic Hygroma • A cystic hygroma is the most common form of lymphangioma and constitutes about 5% of all benign tumours of infancy and childhood. • On US scans, most cystic hygromas manifest as a multilocular predominantly cystic mass with septa of variable thickness. • The echogenic portions of the lesion correlate with clusters of small, abnormal lymphatic channels. • Fluid–fluid levels can be observed with a characteristic echogenic, haemorrhagic component layering in the dependent portion of the lesion. • Prenatal US may demonstrate a cystic hygroma in the posterior neck soft tissues. • On CT images, cystic hygromas tend to appear as poorly circumscribed, multiloculated, hypoattenuated masses. • They typically have characteristic homogeneous fluid attenuation. • Usually, the mass is centred in the posterior triangle or in the submandibular space. Branchial Cleft Cysts Third Branchial Cleft Cyst
  • 330.
    EBDR Exam MCQs& Concepts October 2021 Dr. Kareem Alnakeeb 324 • A third branchial cleft cyst most commonly appears as a unilocular cystic mass centred in the posterior cervical space on CT and MRI. Second Branchial Cleft Cyst • At US, a second branchial cleft cyst is seen as a sharply marginated, round to ovoid, centrally anechoic mass with a thin peripheral wall that displaces the surrounding soft tissues. • The ‘classic’ location of these cysts is at the anteromedial border of the sternocleidomastoid muscle. First Branchial Cleft Cyst • The first branchial cleft cyst appears as a cystic mass either within, superficial to, or deep to the parotid gland. 81. Plain radiograph of a 9-month-old baby girl shows a large soft-tissue mass in the pelvis with punctate calcification. MRI reveals a large, lobulated, sharply demarcated tumour with extremely heterogeneous signal on T1W images. What is the diagnosis? A. Anterior sacral meningocoele B. Sacrococcygeal teratoma C. Caudal regression syndrome D. Rhabdomyosarcoma E. Rectal duplication Source: Chin, Teck Yew, et al. Get Through Final FRCR 2A: SBAs. 1st ed., CRC Press, 2017. Explanation: Sacrococcygeal Teratoma • Sacrococcygeal teratoma is the most common presacral germ cell tumour in children and the most common solid tumour in neonates. Benign Sacrococcygeal Teratoma • The benign form accounts for 60% of all sacrococcygeal teratomas. • Benign teratomas are predominantly cystic; have attenuation similar to fluid on CT; and may include bone, fat and calcification. • Cystic areas appear low on T1-weighted and high on T2-weighted MRI. • Fatty tissue demonstrates high signal intensity on T1-weighted images, whereas calcification is depicted as a signal void. • The coccyx is always involved, even in benign sacrococcygeal teratoma, and must be resected with the tumour. • Approximately 50% of benign teratomas contain calcification, whereas it is seldom seen in malignant tumours. Malignant Sacrococcygeal Teratoma • Malignant teratomas are more solid, and haemorrhage and necrosis are common. • Malignant teratomas may metastasise. Anterior Sacral Meningocoele • Anterior sacral meningocoele is a congenital abnormality that arises from herniation of the CSF-filled dura mater through a sacral foramen or a defect in the sacral bone. • Eccentric defect in sacrum results in a scimitar appearance on plain film.
  • 331.
    EBDR Exam MCQs& Concepts October 2021 Dr. Kareem Alnakeeb 325 82. A 62-year-old man with known hepatocellular carcinoma on a background of long-standing liver cirrhosis is scheduled to have a TACE procedure. Which one of the following is an absolute contraindication to TACE therapy for hepatocellular carcinoma in a cirrhotic patient? A. Contrast medium allergy B. Replacement of 25% of the liver by the tumour C. Total bilirubin greater than 2 mg/dL D. Biliary tree obstruction E. Child–Pugh Class C cirrhosis Source: Chin, Teck Yew, et al. Get Through Final FRCR 2A: SBAs. 1st ed., CRC Press, 2017. Explanation: Absolute and relative contraindications for conventional TACE in patients with HCC are as follows. Absolute contraindications: Relative contraindications: 1. Decompensated cirrhosis (Childs–Pugh C or higher) 1. Tumour size >10 cm. 2. Jaundice 2. Co-morbidities involving compromised organ function such as cardiovascular and lung disease. 3. Clinical encephalopathy 3. Untreated varices present a high risk of bleeding. 4. Refractory ascites 4. Bile duct occlusion or incompetent papilla due to stent or surgery. 5. Extensive tumour with massive replacement of both lobes 6. Severely reduced portal vein flow 7. Technical contraindications to hepatic intra-arterial treatment 8. Renal insufficiency (creatinine clearance <30 mL/min) 83. Which coronary branch, arising from the left circumflex artery and running along the left ventricular free wall toward the apex, is most commonly termed the obtuse marginal artery? A. Diagonal branch of the left anterior descending artery B. Posterior descending artery C. Left marginal (lateral) branch of the circumflex artery D. Acute marginal branch of the right coronary artery E. Conus branch of the right coronary artery Explanation: (by Perplexity AI, generated with the OpenAI O3 model) • The obtuse marginal artery (OM) is the principal lateral branch of the left circumflex (LCx) artery; it travels along the obtuse (left) margin of the heart toward the apex and supplies the lateral wall of the left ventricle, making option C correct. • The diagonal branches (A) originate from the left anterior descending artery, not the LCx. • The posterior descending artery (B) usually arises from the right coronary or, in left-dominant hearts, the LCx, but it supplies the inferior interventricular septum, not the obtuse margin. • The acute marginal branch (D) is a right coronary artery offshoot that courses along the acute (right) margin. • The conus branch (E) is an early right coronary branch supplying the right ventricular outflow tract, unrelated to the left circumflex system.
  • 332.
    EBDR Exam MCQs& Concepts October 2021 Dr. Kareem Alnakeeb 326 84. Which of the following are True regarding laryngoceles? (True or False) A. They typically arise in the vestibule B. They are bilateral in less than 5% of cast’s. C. They may appear as hypodense masses on unenhanced CT D. Carcinoma is a recognized complication E. They rarely extend beyond the thyrohyoid membrane Source: Bell, J., and N. Davies. MCQs in Clinical Radiology: A Revision Guide for the FRCR. 1st ed., Remedica Pub Ltd, 2004. Explanation: A. False - They arise at the saccule of the ventricle, secondary to obstruction of the laryngeal ventricles. Occasionally this is due to cancer. B. False - they are bilateral in 25% of cases. C. True - They may contain air or fluid. D. True - Carcinoma occurs in less than 1 % of cases. Other complications include infection or mucocoele formation. E. False - In relation to the thyrohyoid membrane, 30% are internal, 26% are external and 44% are mixed. 85. A 1-year-old infant is admitted with acute stridor. A viral cause is suspected. On AP chest radiography no foreign body is identified, but there is an inverted V appearance of the subglottic trachea. Which of the following is the most likely diagnosis? A. Foreign body B. Acute laryngotracheobronchitis C. Whooping cough D. Tracheobronchomalacia E. Epiglottitis Source: Chin, Teck Yew, et al. Get Through Final FRCR 2A: SBAs. 1st ed., CRC Press, 2017. Explanation: Croup • Croup (laryngotracheobronchitis) most commonly affects children between 6 months and 3 years and presents with acute stridor, usually following viral infection. • A subglottic inverted V sign is seen on plain film, but the epiglottis and aryepiglottic folds are usually normal. Epiglottitis • In contrast, epiglottitis is a life-threatening condition affecting 3–6-year-olds, with a lateral soft- tissue neck radiograph showing thickening of the epiglottis and aryepiglottic folds described as the ‘thumb sign’. 86. A 20-year-old man presents with swelling around his left eye. A CT scan shows a high attenuation mass lesion which expands the ethmoid air cells with bony erosion. There are small punctate calcifications seen within the mass. On MRI, the mass returns low signal on T1 and T2. The most likely diagnosis is? A. Fungal sinusitis B. Chronic sinonasal polyposis C. Nasopharyngeal carcinoma D. Juvenile angiofibroma E. Chronic sinusitis
  • 333.
    EBDR Exam MCQs& Concepts October 2021 Dr. Kareem Alnakeeb 327 Source: Gupta, Chaitanya. 300 Single Best Answers for the Final FRCR Part A. 1st ed., Jaypee UK, 2010. Explanation: • CT findings are typical, showing a hyperdense lesion with calcifications and bony erosion. The ethmoid sinus is most commonly involved and bony expansion with erosion is characteristic. • On MRI, the lesion is low signal on T1 and T2 due to high fungal mycelial iron, magnesium and manganese from amino acid metabolism 87. A right-sided aortic arch with mirror-image branching is most frequently associated with which congenital cardiac abnormality? A. pulmonary atresia and ventricular septal defect B. truncus arteriosus C. uncomplicated ventricular septal defect D. Fallot’s tetralogy E. corrected transposition of the great vessels Source: Bydder, Megan, et al. Get Through Final FRCR Part A: SBAs for the Modular Examination. 1st ed., Hodder Education Publishers, 2009. Explanation: • There is a 98% incidence of associated congenital heart disease with a right-sided aortic arch with mirror-image branching. Nearly all of these cases will be tetralogy of Fallot. • All of the given options are associated with right-sided aortic arch, as well as dextrocardia with situs inversus and double-outlet right ventricle. • Right-sided aortic arch with left subclavian artery is associated with only a 12% incidence of congenital heart disease, again with Fallot’s tetralogy being the most commonly associated abnormality. 88. A newborn undergoes a chest radiograph which shows a right sided aortic arch. What underlying condition are they most likely to have? A. Ebstein anomaly B. Tetralogy of Fallot C. Transposition of the great arteries D. Tricuspid atresia E. Truncus arteriosus Source: Rabone, Amanda, et al. The Final FRCR Self-Assessment (MasterPass). 1st ed., CRC Press, 2020. Explanation: • A right-sided aortic arch with mirror imaging branching is the most common subtype of a right-sided aortic arch and is nearly always associated with congenital heart disease. Of these, 900/o are associated with tetralogy of Fallot. Therefore other radiological findings to look for are a `boot- shaped' heart and pulmonary oligemia. • A right-sided aortic arch with an aberrant left subclavian artery is the second most common subtype, and the persistent ductus ligament can cause tracheal compression. A Kommerell diverticulum is also a feature, which manifests as dilatation of the aberrant left subclavian artery at the right aortic arch origin.
  • 334.
    EBDR Exam MCQs& Concepts October 2021 Dr. Kareem Alnakeeb 328 89. In persistent left-sided superior vena cava, drainage usually occurs into which structure? A. left atrium B. right atrium C. normal right superior vena cava D. hemiazygos vein E. coronary sinus Source: Bydder, Megan, et al. Get Through Final FRCR Part A: SBAs for the Modular Examination. 1st ed., Hodder Education Publishers, 2009. Explanation: • Persistent left-sided superior vena cava occurs in 0.3% of the general population and in 4.3–11% of patients with congenital heart disease. • It is associated with atrial septal defects and azygos continuation of the inferior vena cava. • It lies lateral to the aortic arch and anterior to the left hilum. It usually drains into the coronary sinus, but rarely drains into the left atrium, causing a left-to-right shunt. • The normal right-sided superior vena cava is absent in 10–18% of cases of left-sided superior vena cava.
  • 335.
    EBDR Exam MCQs& Concepts October 2021 Dr. Kareem Alnakeeb 329 Paper 2 1. A lateral thoracolumbar radiograph of a 45-year-old patient on long-term haemodialysis shows broad bands of sclerosis along the superior and inferior vertebral endplates with a relatively lucent centre, producing the classic “rugger-jersey spine” appearance. Which underlying disorder most commonly causes this finding? A. Hunter syndrome B. Secondary hyperparathyroidism C. Ankylosing spondylitis D. Osteomalacia E. Paget disease of bone Explanation: (by Perplexity AI, generated with the OpenAI O3 model) • The “rugger-jersey” spine sign is virtually pathognomonic for renal osteodystrophy attributable to secondary hyperparathyroidism. • Chronic kidney disease leads to phosphate retention and hypocalcaemia, stimulating parathyroid hormone release; the resulting sub-periosteal resorption and reactive endplate sclerosis generate the alternating dense-lucent-dense pattern. • Hunter syndrome produces dysostosis multiplex but not endplate sclerosis. • Ankylosing spondylitis shows vertebral squaring and marginal syndesmophytes, not alternating bands. • Osteomalacia causes generalized osteopenia and Looser zones, while Paget disease gives cortical thickening and “picture-frame” vertebrae rather than the horizontal sclerotic bands of a rugger-jersey spine. 2. In infants with suspected developmental dysplasia of the hip (DDH), MRI is useful for all of the following purposes except: A. Demonstrate an inverted acetabular labrum B. Identify a hypertrophic pulvinar C. Detect early avascular necrosis of the femoral head D. Assess the integrity of Shenton line E. Visualize the cartilaginous acetabular roof morphology Explanation: (by Perplexity AI, generated with the OpenAI O3 model) • MRI provides high-contrast visualization of non-ossified structures around the infant hip. It clearly shows an inverted labrum, enlarged pulvinar tissue and early marrow signal changes of avascular necrosis before they become apparent on radiographs, and it also delineates the cartilaginous acetabular roof. • Shenton line, however, is a purely radiographic arc drawn on an anteroposterior pelvic X-ray to judge femoral head displacement; it cannot be assessed on cross-sectional MRI images, making option D the exception. • Options A, B, C and E all exploit MRI’s soft-tissue and cartilage sensitivity, thereby representing appropriate indications.
  • 336.
    EBDR Exam MCQs& Concepts October 2021 Dr. Kareem Alnakeeb 330 3. A 10-year-old schoolboy had a fall and bruised his right knee badly. There was an open wound that was not treated until the following day. After a further two days he became systemically unwell with a fever. His knee was extremely tender, swollen, and the movement was restricted. Which of the following is a feature of septic arthritis? A. Usually due to Hemophilus B. Periarticular, soft tissue swelling is rare C. Blurring of the periarticular fat planes is common D. The joint space widens after a few weeks E. A joint effusion is not usually present Source: Proctor, Robin. Final FRCR Part A Modules 1–3 Single Best Answer MCQs: The SRT Collection of 600 Questions with Explanatory Answers (MasterPass). 1st ed., CRC Press, 2009. Explanation: • Septic arthritis usually occurs in hip, knee, shoulder, elbow and ankle. • Staphylococcus aureus, followed by group A Streptococcus, are the most common causes. • Other radiographic features include periarticular soft-tissue swelling, an effusion, periarticular osteopenia and, later, joint space narrowing. • Ultrasound may help identify septic arthritis before cartilage lysis occurs. • The hallmark is joint effusion in a patient with signs of a joint infection. 4. On musculoskeletal ultrasound of a painful, swollen knee suspected of septic arthritis, which sonographic finding is most frequently seen? A. Joint effusion B. Thickened synovial capsule C. Echogenic intra-articular debris D. Hyperemia on power Doppler within the synovium E. Cartilage surface irregularity Explanation: (by Perplexity AI, generated with the OpenAI O3 model) • Joint effusion is the earliest and most common ultrasound manifestation of septic arthritis because bacterial inflammation rapidly drives fluid accumulation within the affected joint. • Detecting and aspirating this fluid is critical both for diagnosis and for guiding antimicrobial therapy. • Thickening of the synovial capsule and power Doppler hyperemia are markers of synovitis; they occur variably and may be absent early in infection. • Echogenic debris represents pus or fibrin strands but is not present in every effusion and therefore is less reliable than simply identifying excess fluid. • Cartilage irregularity is a late feature reflecting destructive change and, when present, suggests delayed presentation rather than a typical initial finding. 5. Which skeletal dysplasia is typically lethal in the perinatal period? A. Thanatophoric dysplasia B. Achondroplasia C. Cleidocranial dysplasia D. Fibrous dysplasia E. Spondyloepiphyseal dysplasia congenita
  • 337.
    EBDR Exam MCQs& Concepts October 2021 Dr. Kareem Alnakeeb 331 Explanation: (by Perplexity AI, generated with the OpenAI O3 model) • Thanatophoric dysplasia results from FGFR3 mutations causing marked micromelia, narrow bell- shaped thorax and severe pulmonary hypoplasia, leading to stillbirth or death within hours of birth; hence it is considered uniformly lethal. • Achondroplasia produces rhizomelic short stature but normal life expectancy, while cleidocranial dysplasia and fibrous dysplasia are usually compatible with long, near-normal lives. • Spondyloepiphyseal dysplasia congenita causes short-trunk dwarfism and cervical spine instability, yet most affected individuals survive into adulthood. • Only thanatophoric dysplasia consistently leads to perinatal lethality, making it the correct choice. 6. An adult presents after a fall onto the outstretched hand; elbow radiographs show an isolated joint effusion with no obvious cortical break. Which occult fracture is most commonly responsible for this finding? A. Coronoid process fracture B. Medial humeral epicondyle fracture C. Lateral humeral condyle fracture D. Radial head fracture E. Olecranon fracture Explanation: (by Perplexity AI, generated with the OpenAI O3 model) • Intracapsular elbow effusion (positive fat-pad sign) indicates hemarthrosis. • In adults, axial load through the radius during a fall frequently causes an undisplaced radial head fracture. These fractures are often hairline and intra-articular, so they may be radiographically occult while still producing a hemarthrosis detectable as a joint effusion. • Coronoid fractures are rare and usually accompany elbow dislocations. • Medial or lateral condylar injuries are classically paediatric. • Olecranon fractures lie largely extra-articular; an isolated effusion without visible cortical disruption would therefore be unusual for this injury. Hence an occult radial head fracture is the most likely diagnosis. 7. Regarding the Ficat-Arlet staging of avascular necrosis of the femoral head, which statement is correct? A. Stage I disease shows bone marrow oedema on MRI with a normal plain radiograph. B. Stage II disease demonstrates acetabular cartilage loss on pelvic radiograph. C. Stage III disease shows a serpiginous double-line sign on T2-weighted MRI with an intact head contour. D. Stage III disease is defined by joint-space narrowing and complete collapse of the femoral head. E. Stage IV disease shows a subchondral crescent sign without secondary osteoarthritis. Explanation: (by Perplexity AI, generated with the OpenAI O3 model) • Stage I AVN is radiographically occult; MRI or bone scan reveals early marrow changes such as oedema and the characteristic double-line sign, hence option A is correct. • Stage II presents with sclerosis or cysts but no collapse, and acetabular involvement (option B) is absent until Stage IV. • Although the double-line sign can appear early, an intact head contour with that sign does not define Stage III (option C). • Stage III is marked by a subchondral crescent sign and early collapse, but significant joint-space narrowing signals Stage IV, making option D incorrect.
  • 338.
    EBDR Exam MCQs& Concepts October 2021 Dr. Kareem Alnakeeb 332 • Option E is wrong because secondary osteoarthritis, including joint-space loss, is integral to Stage IV disease. 8. A 14-year-old boy presents with night pain relieved by NSAIDs. Radiographs show a small, round lucency with central mineralization completely surrounded by thickened cortex in the mid-tibia cortex. What is the most likely diagnosis? A. Osteoblastoma B. Osteoid osteoma C. Osteochondroma D. Non-ossifying fibroma E. Ewing sarcoma Explanation: (by Perplexity AI, generated with the OpenAI O3 model) • Osteoid osteoma classically arises within cortical bone, appearing as a nidus <1.5 cm surrounded by reactive sclerotic cortex; pain is characteristically relieved by NSAIDs, reflecting prostaglandin production. • Osteoblastoma can look similar but is usually >2 cm, less painful at night, and often occurs in the spine. • Osteochondroma is a medullary-continuity exostosis projecting from the bone surface, not a purely cortical intracortical lesion. • Non-ossifying fibroma is a metaphyseal, cortically based but eccentric lytic defect with a sclerotic rim, not a tiny nidus with surrounding sclerosis. • Ewing sarcoma is an aggressive permeative diaphyseal tumor with soft-tissue mass and systemic symptoms, not a benign cortical nidus. 9. Which underlying condition is classically responsible for the cavernous transformation (portal cavernoma) seen on Doppler ultrasound or contrast-enhanced CT of the porta hepatis? A. Acute portal vein thrombosis B. Chronic portal vein thrombosis C. Budd–Chiari syndrome D. Hepatobiliary fistula E. Hepatocellular carcinoma Explanation: (by Perplexity AI, generated with the OpenAI O3 model) • Cavernous transformation represents a network of peri-portal collateral veins that slowly develop to bypass an obstructed main portal vein. This collateralization takes weeks to months, so it is virtually pathognomonic for chronic (long-standing) portal vein thrombosis. • In acute thrombosis (A), the thrombus is still fresh and the portal vein appears dilated rather than replaced by collaterals. • Budd–Chiari syndrome (C) affects hepatic veins, not the portal vein, so cavernoma is absent. • A hepatobiliary fistula (D) is an abnormal communication between bile ducts and adjacent structures and has no relationship with portal venous flow. • Hepatocellular carcinoma (E) may invade the portal vein but produces tumour thrombus, not a cavernous network.
  • 339.
    EBDR Exam MCQs& Concepts October 2021 Dr. Kareem Alnakeeb 333 10. A 55-year-old patient undergoes a barium meal that shows a diffusely narrowed, rigid stomach with loss of peristalsis, producing the classic “leather-bottle” (linitis plastica) appearance. Which underlying lesion most commonly causes this radiological pattern? A. Gastrointestinal stromal tumour (GIST) B. Hyperplastic gastric polyp C. Peutz-Jeghers hamartomatous polyposis D. Gastric phytobezoar E. Diffuse infiltrative (scirrhous) adenocarcinoma Explanation: (by Perplexity AI, generated with the OpenAI O3 model) • Linitis plastica results from diffuse submucosal infiltration that stiffens the gastric wall; the archetypal cause is scirrhous (diffuse) adenocarcinoma, often producing a “leather-bottle” stomach. • GISTs grow as focal submucosal masses that typically expand outward rather than circumferentially. • Hyperplastic and Peutz-Jeghers polyps create discrete intraluminal filling defects without transmural infiltration. • Bezoars are intraluminal conglomerates of indigestible material; they cast a mottled filling defect but do not alter mural pliability. Thus, only diffuse adenocarcinoma explains the uniform rigid narrowing that defines linitis plastica. 11. Which of the following statements concerning congestive cardiomegaly on an erect chest radiograph is correct? A. The portal vein is characteristically dilated. B. A right-sided pleural effusion is a common associated finding. C. The condition is unrelated to left-sided heart failure. D. Kerley B lines are typically absent. E. All of the above. Explanation: (by Perplexity AI, generated with the OpenAI O3 model) • Congestive cardiomegaly reflects elevated left atrial and pulmonary venous pressures; serous fluid often tracks into the pleural space, classically producing a right-sided or bilateral effusion, so option B is true. • Portal vein calibre is usually unaffected—hepatic and inferior vena cava dilation predominate— rendering option A incorrect. • Because the process is a direct consequence of left-sided (and frequently global) heart failure, option C is false. • Interstitial oedema from pulmonary venous hypertension gives rise to Kerley B lines; their absence would be atypical, so option D is wrong. • As only one statement is correct, option E (“all of the above”) is also incorrect. 12. In a supine polytrauma patient undergoing a focused assessment with sonography for trauma (FAST), which intraperitoneal recess is most commonly the first site where free fluid accumulates? A. Paracolic gutter B. Pouch of Douglas C. Morrison’s pouch (hepatorenal recess) D. Vesicouterine pouch (anterior to the uterus) E. Splenorenal recess
  • 340.
    EBDR Exam MCQs& Concepts October 2021 Dr. Kareem Alnakeeb 334 Explanation: (by Perplexity AI, generated with the OpenAI O3 model) • In the supine position, intraperitoneal blood gravitates to the most dependent right upper-quadrant space, Morrison’s pouch, which lies between the liver and right kidney. This recess is therefore scanned first in the FAST examination and is the most sensitive early indicator of hemoperitoneum. • The paracolic gutters fill only after larger volumes accumulate, while the Pouch of Douglas becomes dependent chiefly when the patient is upright or in Trendelenburg. • The vesicouterine pouch is shallower and rarely the initial reservoir. • The splenorenal recess is less frequently the first site because splenic injury often tamponades and left-sided free fluid disperses preferentially to the pelvis. 13. What is the most common site of involvement in tuberculosis of the gastrointestinal tract? A. Stomach B. Duodenum C. Ileocaecal region D. Splenic flexure E. Rectum Source: Bydder, Megan, et al. Get Through Final FRCR Part A: SBAs for the Modular Examination. 1st ed., Hodder Education Publishers, 2009. Explanation: • Tuberculosis of the gastrointestinal tract may occur through ingestion of infected sputum, or by hematogenous spread to submucosal lymph nodes from a pulmonary tuberculous focus. • It most commonly affects the ileocecal region due to its abundance of lymphoid tissue and relative stasis of gut contents. • Typical features at this site include circumferential thickening of the terminal ileum and caecum, a thickened ileocecal valve and ulceration following the orientation of lymphoid follicles (longitudinal in the terminal ileum and transverse in the colon). • Marked enlargement of adjacent mesenteric lymph nodes with central areas of low attenuation may be seen. 14. On contrast-enhanced CT evaluation of a pancreatic head adenocarcinoma, which one of the following findings does NOT make the tumor unresectable? A. Encasement of the hepatic artery B. Encasement of the celiac axis C. Direct invasion of the stomach wall D. Tumor extension into the peripancreatic fat planes E. >180° encasement of the superior mesenteric artery Explanation: (by Perplexity AI, generated with the OpenAI O3 model) • Surgical resection is precluded when a pancreatic cancer involves major arteries (celiac axis, hepatic artery, SMA) or invades adjacent hollow viscera such as the stomach, because clear margins cannot be achieved and vascular reconstruction is rarely feasible. • Peripancreatic fat infiltration, however, is common and does not necessarily indicate vascular or organ invasion; many resectable tumors show stranding in the fat but have intact peri-arterial planes, so resection can still proceed. • Hence option D is the only feature that does not, by itself, define unresectability, whereas options A, B, C and E each denote locally advanced disease beyond surgical cure.
  • 341.
    EBDR Exam MCQs& Concepts October 2021 Dr. Kareem Alnakeeb 335 15. Bilateral suprarenal (adrenal) lesions: A. 10% are adenomas B. Colonic metastases are the commonest cause C. They are often associated with adrenal hemorrhage D. All of the above Explanation: (by Google NotebookLM AI “based on FRCR books answers”) Regarding bilateral suprarenal (adrenal) lesions is their association with adrenal hemorrhage: • Bilateral adrenal lesions are often associated with adrenal hemorrhage. In cases of shock, hemorrhage is more likely to be bilateral. Non-traumatic hemorrhage of both adrenals may also be seen, often caused by perinatal stressors. Various conditions, including surgery, sepsis, burns, hypotension, pregnancy, cardiovascular disease, and steroid use, are associated with non-traumatic adrenal hemorrhage, which can lead to bilateral involvement. Adrenal hemorrhage is also listed as a recognized cause of bilateral large adrenals. • Regarding other options: o While adrenal adenomas are common (occurring in 1-2% of the population), and some benign adenomas can have specific CT characteristics, their specific prevalence as bilateral lesions is not provided as 10%. o Adrenal metastases can originate from various primary sites, including lung, breast, melanoma, gastrointestinal tract, and renal cancer. 16. On emergency CT of the perineum, which imaging feature best distinguishes Fournier’s gangrene from uncomplicated scrotal cellulitis? A. Scrotal wall oedema B. Relative sparing of the perineum C. Air densities within the subcutaneous tissues D. Heterogeneous testicular echotexture E. Small reactive hydrocele Explanation: (by Perplexity AI, generated with the OpenAI O3 model) • In Fournier’s gangrene, rapidly spreading necrotizing fasciitis allows gas-forming organisms to produce visible air pockets in the subcutaneous and fascial planes; their presence on CT is a hallmark and strongly supports the diagnosis over simple cellulitis. • Scrotal wall oedema (A) and minor perineal sparing (B) occur in both conditions and therefore lack specificity. • The testes themselves are usually spared in Fournier’s gangrene because the gonadal artery arises proximal to the infected fascial planes, so heterogeneous testicular echotexture (D) points instead to orchitis or infarction. • A small reactive hydrocele (E) is a non-specific secondary finding and does not differentiate the two entities. 17. Which hepatic condition classically demonstrates the “central dot sign” on contrast-enhanced CT or MRI? A. Caroli’s disease B. Primary sclerosing cholangitis C. Simple hepatic cysts D. Hepatic mesenchymal hamartoma E. Hepatic cavernous haemangioma
  • 342.
    EBDR Exam MCQs& Concepts October 2021 Dr. Kareem Alnakeeb 336 Explanation: (by Perplexity AI, generated with the OpenAI O3 model) • The central dot sign is an enhancing portal venous radicle and fibrovascular bundle seen within fluid-filled, ectatic intrahepatic bile ducts; it is virtually pathognomonic for Caroli’s disease because the ducts are massively dilated yet remain in continuity with portal triads. • Primary sclerosing cholangitis produces multifocal strictures and beading without a central enhancing dot. • Simple hepatic cysts are avascular, with thin walls and no intraluminal structures. • Hepatic mesenchymal hamartoma appears as a multicystic mass lacking patent biliary connections. • Cavernous hemangiomas show peripheral nodular enhancement progressing centrally, not a single central dot. 18. A 4-year-old boy falls off his bike and complains of neck pain. Which of the following features is worrying for a serious injury on plain cervical X-rays? A. Atlanto-axial distance <5 mm B. Displacement of 6 mm of the lateral masses relative to the dens C. Absence of lordosis D. Disruption of the spinolamellar line E. Anterior subluxation of C2 on C3 Source: Chin, Teck Yew, et al. Get Through Final FRCR 2A: SBAs. 1st ed., CRC Press, 2017. 19. During colonoscopy a 55-year-old man is found to have several colonic polyps. Which polyp subtype carries the highest risk of malignant transformation? A. Hyperplastic polyp B. Inflammatory polyp C. Tubular adenoma D. Villous adenoma E. Peutz-Jeghers hamartomatous polyp Explanation: (by Perplexity AI, generated with the OpenAI O3 model) • Villous adenomas have a broad, villous architecture that harbours high-grade dysplasia more frequently than other polyp types, giving them the greatest propensity to progress to invasive colorectal carcinoma. • Tubular adenomas are common but contain less villous tissue, so the malignant risk is lower unless the lesion is large. • Hyperplastic and inflammatory (pseudopolyp) types are generally considered non-neoplastic and rarely undergo malignant change. • Hamartomatous polyps in Peutz-Jeghers syndrome can be large but malignant conversion of the polyp itself is uncommon; the syndrome’s cancer risk arises from associated epithelial tumours elsewhere, not the polyp.
  • 343.
    EBDR Exam MCQs& Concepts October 2021 Dr. Kareem Alnakeeb 337 20. A 30-year-old man is admitted with lower abdominal pain following a road traffic accident in which he was an unrestrained passenger. Blood is seen in his urine and a bladder injury is suspected. Regarding extraperitoneal bladder rupture, which of the following is incorrect? A. It is more common than intraperitoneal rupture. B. A flame-shaped extravasation of contrast can often be seen. C. It is usually caused by puncture from a pelvic fracture. D. Contrast most commonly extravasates into the retropubic space of Retzius. E. The bladder dome is the most common site of injury. Source: Chin, Teck Yew, et al. Get Through Final FRCR 2A: SBAs. 1st ed., CRC Press, 2017. Explanation: • Extraperitoneal bladder rupture accounts for 80% of bladder ruptures. • A flame-shaped extravasation of contrast is a classic finding and can be seen to extend into the perivesical fat and into the retropubic space of Retzius, anterior abdominal wall, upper thigh or scrotum. • The most common site of injury is close to the anterolateral aspect of the bladder base. It is usually caused by puncture from pelvic fractures. 21. Regarding traumatic bladder rupture, which statement is INCORRECT? A. Extraperitoneal rupture is more common than intraperitoneal rupture. B. Pelvic fractures are more frequently associated with intraperitoneal than extraperitoneal rupture. C. Motor vehicle collisions are the commonest mechanism of injury. D. On CT cystography, intraperitoneal rupture results in contrast outlining the bladder dome. E. Intraperitoneal rupture usually requires surgical repair. Explanation: (by Perplexity AI, generated with the OpenAI O3 model) • Around 60–70% of traumatic bladder ruptures are extraperitoneal, typically occurring with pelvic ring fractures after high-energy mechanisms such as motor vehicle collisions. • These fractures tear the anterolateral bladder wall, causing “flame-shaped” contrast in the perivesical space, and most cases are managed with catheter drainage alone. • Intraperitoneal rupture occurs at the bladder dome when a full bladder is suddenly compressed; CT cystography shows contrast freely outlining bowel loops and the bladder dome and mandates operative repair to prevent peritonitis. • Therefore, pelvic fractures are much more strongly linked to extraperitoneal, not intraperitoneal, rupture, making option B incorrect. • Options A, C, D and E accurately describe epidemiology, imaging and management. 22. Antenatal ultrasound shows an abnormal facial contour with a large cyst without any cortical mantle of cerebral tissue anteriorly. Septum pellucidum, falx cerebri and optic tracts are not identified with evidence of a fused midline thalamus. A single large ventricle is identified. Normal brain stem, midbrain and cerebellum are noted. What is the diagnosis? A. Alobar holoprosencephaly B. Lobar holoprosencephaly C. Hydranencephaly D. Anencephaly E. Congenital hydrocephalous
  • 344.
    EBDR Exam MCQs& Concepts October 2021 Dr. Kareem Alnakeeb 338 Source: Chin, Teck Yew, et al. Get Through Final FRCR 2A: SBAs. 1st ed., CRC Press, 2017. Explanation: Holoprosencephaly Overview • Holoprosencephaly (HPE) is considered the most common malformation of the brain and face in humans. Alobar Holoprosencephaly • In alobar HPE, prosencephalic cleavage fails, resulting in a single midline forebrain with a primitive monoventricle often associated with a large dorsal cyst. • The olfactory bulbs and tracts, the corpus callosum and anterior commissure, the cavum septum pellucidum and the interhemispheric fissure are absent, whereas the optic nerves may be normal, fused or absent. • The basal ganglia, hypothalamic and thalamic nuclei are typically fused in the midline, resulting in absence of the third ventricle. Lobar Holoprosencephaly • In lobar HPE, the interhemispheric fissure is present along nearly the entire midline, and the thalami are completely or almost completely separated. • The corpus callosum may be normal or incomplete, but the cavum septum pellucidum is always absent. Hydranencephaly • Hydranencephaly is the result of a vascular insult (anterior circulation) with the cerebral hemispheres variably replaced by fluid covered with leptomeninges and dura. • Falx cerebri is present. • The cerebellum, midbrain, thalami, basal ganglia, choroid plexus and portions of the occipital lobes, all fed by the posterior circulation, are typically preserved. • It is differentiated from hydrocephalus by absence of an intact rim of cortex (seen with even the most severe hydrocephalus). 23. A 37-year-old woman in her third trimester presented to the labour ward with acute onset of pain in the lower central abdomen. She had had two previous caesarean sections and an appendectomy with unremarkable recovery. On clinical examination, there was definite tenderness at the site of a previous caesarean scar. The best imaging modality for evaluation of caesarean section scar dehiscence is A. CT B. Transabdominal US C. MRI D. Angiography E. Transvaginal US Source: Chin, Teck Yew, et al. Get Through Final FRCR 2A: SBAs. 1st ed., CRC Press, 2017. Explanation: • Uterine dehiscence is characterized by incomplete rupture of the uterine wall, usually involving the endometrium and myometrium but with an intact overlying serosal layer. • MR imaging may be better than CT in evaluating for uterine dehiscence because of its multiplanar capability and greater soft-tissue contrast and its ability to help identify an intact serosal layer.
  • 345.
    EBDR Exam MCQs& Concepts October 2021 Dr. Kareem Alnakeeb 339 24. A 23-year-old woman with history of pelvic inflammatory disease, 2 months amenorrhea, left lower- abdominal pain, weakness and occasional spotting showed lower-than-expected levels of HCG for pregnancy on urine assay. As far as US of the pelvis is concerned, which one of the following sonographic findings definitively distinguishes ectopic pregnancy from corpus luteum? A. Visualization of a yolk sac B. ‘Ring of fire’ appearance C. Low-impedance flow on Doppler to the ring of fire D. Visualization of a heartbeat E. Presence of echogenic pelvic fluid Source: Chin, Teck Yew, et al. Get Through Final FRCR 2A: SBAs. 1st ed., CRC Press, 2017. Explanation: Prevalence and Location • Ninety-five percent of ectopic pregnancies are tubal; they occur mostly in the ampulla (70%). Adnexal Mass Findings • An adnexal mass that is separate from the ovary is the most common finding of a tubal pregnancy and is seen on US images. • Although not common, an adnexal mass is more specific for an ectopic pregnancy when it contains a yolk sac or a living embryo. Tubal Ring Sign • The tubal ring sign is the second most common sign of a tubal pregnancy. • The tubal ring sign describes a hyperechoic ring surrounding an extrauterine gestational sac. Ring of Fire Sign • A related finding is the ‘ring of fire’ sign, which is recognized by peripheral hypervascularity of the hyperechoic ring. • The term previously described the high-velocity, low-impedance flow surrounding an ectopic adnexal pregnancy. • Peripheral hypervascularity is a non-specific finding of the ring of fire sign and may also be seen surrounding a normal maturing follicle or a corpus luteal cyst. • Location of the ring of fire in an ovary distinguishes a corpus luteal cyst from an ectopic pregnancy where the ring is extra-ovarian. Intrauterine Findings • Intrauterine findings of an ectopic pregnancy include a ‘normal endometrium’, a pseudo–gestational sac, a trilaminar endometrium and a thin-walled decidual cyst. Extrauterine Findings • Extrauterine findings of ectopic pregnancy include pelvic free fluid, hematosalpinx and hemoperitoneum. 25. A fit and healthy 25-year-old woman presents to the breast clinic with a small mobile non-tender breast lump that she noticed incidentally. An ultrasound is deemed as the first-line investigation; it reveals an extremely well-defined homogenous, hypoechoic oval mass with posterior acoustic shadowing. What is the most likely diagnosis? A. Cystosarcoma phylloides B. Fibroadenoma C. Complex breast cyst D. Invasive lobular carcinoma E. Fat necrosis
  • 346.
    EBDR Exam MCQs& Concepts October 2021 Dr. Kareem Alnakeeb 340 Source: Chin, Teck Yew, et al. Get Through Final FRCR 2A: SBAs. 1st ed., CRC Press, 2017. Explanation: • Fibroadenomas are the most common cause of benign solid mass in the breast. • On US, they appear round or oval, wider than tall, hypoechoic, well-defined and mostly homogenous and show a ‘hump and dip’ sign (small focal bulge to the contour with a contagious small sulcus), a thin echogenic pseudocapsule and rarely either posterior acoustic enhancement (17%–25%) or posterior acoustic shadow (9%–11%). 26. An 18 year old man undergoes a Tc MDP bone scan to investigate pain in the right hip. A ‘hot’ lesion is seen in the right proximal femur. No other lesions are seen. Which of the following lesions would appear as ‘hot’ on a Tc MDP bone scan? A. Osteopoikilosis B. Fibrous cortical defect C. Acute fracture within 12 hours of injury D. Fibrous dysplasia E. Hemangioma Source: Currie, Stuart, et al. SBAs for the FRCR 2A. 1st ed., Cambridge University Press, 2010. Explanation: • The most common site of monostotic fibrous dysplasia is the ribs, followed by proximal femur and craniofacial bones. 3/4 of cases present before age 30. • Other benign lesions causing a ‘hot’ on bone scan include Paget’s disease, brown tumours, aneurysmal bone cysts, osteoid osteoma and chondroblastoma. • Acute fractures are not usually ‘hot’ until after the first 24–48 hours. 27. Regarding a thyroglossal duct (tract) cyst, which of the following statements is/are true? A. The cyst may contain ectopic thyroid tissue. B. Any thyroid tissue present can be functionally active and show uptake on radioiodine (I-131 ) scintigraphy. C. Such functioning tissue can also concentrate technetium-99m pertechnetate on thyroid scans. D. All of the above. Explanation: (by Perplexity AI, generated with the OpenAI O3 model) Key points about thyroglossal duct cysts and the thyroid tissue they may contain: • The cyst often harbors heterotopic thyroid tissue, with reports indicating it is present in roughly one- half of cases. • When present, this ectopic tissue can be functionally active and concentrate radioiodine, demonstrating uptake on I-131 scintigraphy. • The same functioning tissue will also take up technetium-99m pertechnetate, the tracer commonly used in routine thyroid scans. 28. In a 60-year-old patient undergoing Tc-99m HIDA scintigraphy, both hepatic parenchymal uptake and gallbladder visualization are markedly delayed. Which interpretation is most accurate? A. A reliable sign of acute cholecystitis B. Typical for cystic duct obstruction by gallstones C. Not a reliable sign of acute or chronic cholecystitis D. Suggestive of congenital gallbladder agenesis E. Indicative of biliary atresia
  • 347.
    EBDR Exam MCQs& Concepts October 2021 Dr. Kareem Alnakeeb 341 Explanation: (by Perplexity AI, generated with the OpenAI O3 model) • Normal HIDA criteria for acute cholecystitis are non-visualization of the gallbladder despite normal hepatic uptake and prompt biliary excretion. • When tracer extraction by hepatocytes is also reduced, delivery to the cystic duct and gallbladder is proportionally delayed, producing false-positive non-visualization. • Therefore, concurrent poor hepatic uptake makes the finding unreliable for diagnosing acute or chronic cholecystitis. • Gallstone obstruction (B) classically shows absent GB activity but preserved liver uptake; gallbladder agenesis (D) and biliary atresia (E) likewise retain normal or increased hepatic extraction, distinguishing them from the combined hepatic-gallbladder delay seen here. 29. Which skeletal dysplasia is classically associated with generalized osteoporosis and fragile bones? A. Achondroplasia B. Osteogenesis imperfecta C. Developmental hip dysplasia D. Fibrous dysplasia E. Morquio syndrome Explanation: (by Perplexity AI, generated with the OpenAI O3 model) • Osteogenesis imperfecta (OI) is a collagen-I disorder producing thin cortices, low bone mineral density and recurrent fractures – the textbook picture of generalised osteoporosis. • Achondroplasia causes short-limb dwarfism with normal or increased bone density, not osteoporosis. • Developmental hip dysplasia is a local acetabular malformation without systemic bone loss. • Fibrous dysplasia replaces focal bone with fibrous tissue; overall bone mass is often normal, and generalised osteoporosis is not typical. • Morquio syndrome leads to dysostosis multiplex with thickened bones rather than osteoporotic ones. 30. CT of the pelvis in a 37-year-old woman who was undergoing ovulation induction showed massive cysts in the pelvis surrounding a core of central ovarian stroma with relatively higher attenuation. More cephalad images demonstrated ascites. All of the following are features of ovarian hyperstimulation syndrome, except A. Enlarged multicystic ovaries B. Free intraperitoneal fluid C. Low serum estradiol levels D. Pleural or pericardial effusion E. Risk of deep vein thrombosis Source: Chin, Teck Yew, et al. Get Through Final FRCR 2A: SBAs. 1st ed., CRC Press, 2017. Explanation: Etiology and Occurrence • Ovarian hyperstimulation syndrome is usually iatrogenic secondary to ovarian stimulant drug therapy for infertility but may occur as a spontaneous event in pregnancy and is associated with raised serum estradiol levels. Clinical Presentation and Complications • The syndrome consists of ovarian enlargement with extravascular accumulation of exudates leading to weight gain, ascites, pleural effusions, intravascular volume depletion with hemoconcentration and oliguria in varying degrees, with increased risk of thrombosis and stroke.
  • 348.
    EBDR Exam MCQs& Concepts October 2021 Dr. Kareem Alnakeeb 342 • Pain, abdominal distention, nausea and vomiting are frequently seen. Imaging Findings • The imaging findings are similar at US, CT and MR imaging and reflect ovarian enlargement by distended corpora luteal cysts of varying sizes. • Because the enlarged follicles are often peripheral in location, a spoked wheel appearance has been described. • Ascites, pleural effusion and pericardial effusion are also described. Differential Diagnosis • Familiarity with ovarian hyperstimulation syndrome and the appropriate clinical setting should help avoid the incorrect diagnosis of an ovarian cystic neoplasm. 31. In primary osteoarthritis of the knee, which tissue is the principal site of degenerative change? A. Joint capsule B. Tendon C. Synovium D. Articular cartilage E. Subchondral bone Explanation: (by Perplexity AI, generated with the OpenAI O3 model) • Osteoarthritis is characterized by progressive loss of the smooth, hyaline articular cartilage that lines the joint surfaces. • Mechanical and biochemical stresses lead to cartilage softening, fibrillation and eventual erosion, producing joint space narrowing on radiographs. • Secondary changes—synovial inflammation, capsular thickening, tendon irritation and subchondral bone sclerosis—can occur, but these are consequences rather than the primary pathology. • The joint capsule (A) and synovium (C) may show reactive changes, while tendons (B) are only indirectly affected. Subchondral bone (E) develops eburnation and cysts after cartilage loss, not before it. 32. An ultrasound screening of a 6-week-old infant shows a coronal static view of the right hip with an alpha angle of 68° and a beta angle of 48°. What is the Graf classification of this hip? A. Immature hip B. Normal hip C. Acetabular deficiency D. Dislocation E. Subluxation Explanation: (by Perplexity AI, generated with the OpenAI O3 model) • A Graf type I (normal) hip has an alpha angle >60° and a beta angle <55°, indicating a well-developed bony acetabular roof and a properly covered femoral head. • The measurements in this case (alpha 68°, beta 48°) meet these criteria, so option B is correct. • An immature (type IIa) hip (option A) would have an alpha of 50–59°; acetabular deficiency/dysplasia (option C) presents with alpha 43–49° and higher beta angles. • A frankly dislocated hip (option D) demonstrates alpha <43° with the femoral head displaced; subluxation (option E) shows alpha 43–49° with partial lateralization. • Thus, all distractors have measurement ranges inconsistent with the values provided.
  • 349.
    EBDR Exam MCQs& Concepts October 2021 Dr. Kareem Alnakeeb 343 33. A 42-year-old woman was found to have a mass in the right lobe of the liver on US and sent for an MRI liver for further characterization. All of the following are expected MR features of FNH, except A. High signal intensity of the central scar on T2W images B. Uniform enhancement of the mass in the arterial phase C. Lack of capsular enhancement in the arterial phase D. Hypointense to surrounding liver on the enhanced portal venous phase E. Maximum intensity of central scar on enhanced delayed phase images Source: Chin, Teck Yew, et al. Get Through Final FRCR 2A: SBAs. 1st ed., CRC Press, 2017. Explanation: MRI Signal and Enhancement • Typically, FNH is iso- or hypointense on T1-weighted images, is slightly hyper- or isointense on T2- weighted images and has a hyperintense central scar on T2-weighted images. • FNH shows intense homogeneous enhancement in the arterial phase and enhancement of the central scar in the later phases of gadolinium-enhanced imaging. Pseudocapsule • FNH does not have a tumour capsule, although the pseudocapsule surrounding some FNH lesions may be quite prominent. • The pseudocapsule of FNH results from compression of the surrounding liver parenchyma by the FNH, perilesion vessels and inflammatory reaction. • The pseudocapsule is usually a few millimetres thick and typically shows high signal intensity on T2- weighted images. • The pseudocapsule may show enhancement on delayed contrast-enhanced images. Central Scar • A central scar is present at imaging in most patients with FNH. • The amount of scar tissue within FNH and the size of the central scar may vary. • The central scar is typically high in signal intensity on T2-weighted images and low in signal intensity on T1-weighted images. • It shows visible enhancement on delayed contrast-enhanced images. • High signal intensity of the central scar may be caused by the inflammatory reaction around the ductular proliferation as well as the vessels within the septa and central scar. • The central scar is not a specific finding of FNH and can be seen in a variety of other focal liver lesions, such as giant haemangiomas. 34. A 30-year-old woman undergoes MRI for an incidental 3 cm liver lesion. Which MRI feature is most characteristic of focal nodular hyperplasia? A. Central scar demonstrates high signal on T1-weighted imaging B. Central scar is hypoenhancing in arterial phase and shows delayed contrast retention C. Lesion contains cystic degeneration D. Central scar shows avid arterial phase enhancement E. Lesion demonstrates a thick enhancing capsule on portal venous phase Explanation: (by Perplexity AI, generated with the OpenAI O3 model) • Focal nodular hyperplasia (FNH) contains a fibrous central scar that is low signal on T1 and enhances only on delayed post-gadolinium images because contrast slowly diffuses into the fibrous tissue.
  • 350.
    EBDR Exam MCQs& Concepts October 2021 Dr. Kareem Alnakeeb 344 • This pattern—poor arterial enhancement with delayed retention—is a classic sign, making option B correct. • The scar is not bright on T1 (A) and does not show rapid arterial enhancement (D). • Cystic change (C) and a thick capsule (E) are seen in adenoma or hepatocellular carcinoma, not in FNH. 35. A 35-year-old man presents with thigh pain; radiographs show a well-defined, expansile lytic lesion with cortical thinning and focal breach centred in the meta-epiphysis of the distal femur. A. Simple bone cyst B. Plasmacytoma C. Giant cell tumour D. Fibrous dysplasia E. Chondroblastoma Explanation: (by Perplexity AI, generated with the OpenAI O3 model) • Giant cell tumour (GCT) characteristically arises in skeletally mature patients aged 20–40 years, abutting the articular surface and spanning the epiphysis into the adjacent metaphysis. It appears as a well-defined, non-sclerotic lytic lesion that expands and may erode cortex, matching this case. Simple bone cysts are purely metaphyseal, usually in children, and seldom breach cortex. • Plasmacytoma occurs in older adults and classically involves the axial skeleton rather than the distal femur. • Fibrous dysplasia presents as a ground-glass, intramedullary lesion with intact cortex and does not typically localise to the epiphysis. • Chondroblastoma is epiphyseal but usually affects adolescents and shows chondroid matrix calcification rather than cortical destruction. 36. Regarding colonic diverticulitis, which of the following statements is correct? A. It predominantly affects young adults. B. It commonly presents with mechanical large-bowel obstruction. C. Non-contrast CT is superior to contrast-enhanced CT for detecting complications. D. None of the above. Explanation: (by Perplexity AI, generated with the OpenAI O3 model) • Colonic diverticulitis typically occurs in middle-aged or older patients; while incidence in those under 50 years is rising, they remain a minority. • Intestinal obstruction is an uncommon presentation; perforation, abscess or fistula are far more frequent complications. • Modern studies show non-contrast CT is overall non-inferior—but not superior—to contrast- enhanced CT; contrast improves sensitivity for abscess or perforation, so it remains preferred when feasible. • Therefore, options A-C are incorrect, making “None of the above” the best choice.
  • 351.
    EBDR Exam MCQs& Concepts October 2021 Dr. Kareem Alnakeeb 345 37. On MRI of a patient with chronic back pain, which of the following findings is most characteristic of Pott disease (spinal tuberculosis) affecting the lumbar spine? A. Vertebral body bone destruction involving two adjacent levels B. Iliopsoas abscess tracking along the muscle belly C. Preservation of intervertebral disc height at the diseased level D. Advanced degenerative disc desiccation with Modic I change E. Marginal osteophyte formation with vacuum phenomenon Explanation: (by Perplexity AI, generated with the OpenAI O3 model) • Tuberculous spondylitis typically spreads beneath the anterior longitudinal ligament to involve adjacent vertebral bodies, then tracks into the iliopsoas sheath, producing a cold iliopsoas abscess, often large and relatively asymptomatic, making it a classic imaging clue. • Although vertebral destruction (A) and relative disc preservation (C) can be seen, they are not as specific because similar patterns occur in other low-virulence infections or early pyogenic disease. • Degenerative disc changes (D) and osteophyte formation with vacuum phenomenon (E) are features of spondylosis, not infection. 38. A 45-year-old woman with suspected early rheumatoid arthritis undergoes high-frequency ultrasound of the 2nd metacarpophalangeal joint; which finding is most specific for the diagnosis? A. Cortical erosion B. Thickened capsule C. Synovitis D. Joint effusion E. Tenosynovitis Explanation: (by Perplexity AI, generated with the OpenAI O3 model) • Cortical erosion represents a breach in the bright cortical line and is highly specific for inflammatory arthropathies such as rheumatoid arthritis; ultrasound detects these erosions earlier than plain radiography, making them a key marker of early disease. • Synovitis, joint effusion and tenosynovitis are sensitive indicators of inflammation but occur in many arthritides and even in non-inflammatory conditions, limiting specificity. • A thickened joint capsule is a non-specific feature that may accompany chronic degeneration or prior injury rather than active rheumatoid pathology. Therefore, demonstration of a definite cortical erosion best supports an early diagnosis of rheumatoid arthritis. 39. On high-resolution ultrasound of a metacarpophalangeal joint, which finding is regarded as the most characteristic of rheumatoid arthritis? A. Cortical bone erosion B. Simple joint effusion C. Synovial hypertrophy (active synovitis) D. Capsular thickening E. Marginal osteophyte formation Explanation: (by Perplexity AI, generated with the OpenAI O3 model) • Cortical bone erosion represents loss of the normally smooth hyperechoic bone cortex caused by pannus-mediated destruction and is highly specific for rheumatoid arthritis, often appearing before erosions are visible on radiographs. Ultrasound can depict these sub-millimetre cortical breaks with great sensitivity, making them a key diagnostic marker.
  • 352.
    EBDR Exam MCQs& Concepts October 2021 Dr. Kareem Alnakeeb 346 • Simple effusion (B) and synovial hypertrophy (C) are sensitive but occur in many inflammatory or degenerative arthritides. • Capsular thickening (D) is a non-specific chronic change. • Marginal osteophytes (E) are typical of osteoarthritis, not rheumatoid disease. 40. A nuclear medicine bone scan shows intense radiotracer uptake confined to the left hemithorax on both anterior and posterior whole-body projections. Which single condition best explains this pattern? A. Pleural effusion B. Unilateral hyperinflated lung C. Prior mastectomy D. All of the above E. None of the above Explanation: (by Perplexity AI, generated with the OpenAI O3 model) • After a unilateral mastectomy, radiopharmaceutical soft-tissue pooling occurs in the post-surgical chest wall and axilla, producing conspicuous, hemithoracic tracer activity on both anterior and posterior views. • Pleural effusion typically causes peripheral or crescentic uptake only on the anterior image because fluid layers dependently, making the posterior view less intense. • Hyperinflated lung contains predominantly air; it shows reduced, not increased, soft-tissue tracer concentration, so it would appear photopenic rather than “hot.” • Therefore mastectomy is the sole listed cause giving symmetric anterior-posterior hemithorax uptake; the other options are incorrect. 41. Which of the following conditions is least likely to produce vertebra plana on spinal imaging? A. Primary spinal lymphoma B. Fibrous dysplasia C. Vertebral metastasis from breast carcinoma D. Langerhans cell histiocytosis E. Spinal tuberculosis Explanation: (by Perplexity AI, generated with the OpenAI O3 model) • Vertebra plana refers to near-complete collapse of a vertebral body with preserved disc spaces. It is classically associated with Langerhans cell histiocytosis, but similar flattening can arise from malignant infiltration (lymphoma, metastasis, myeloma), infectious destruction such as tuberculosis, or severe trauma. In these entities, bone is weakened by infiltration or lytic destruction, predisposing to collapse. • Fibrous dysplasia, however, characteristically causes medullary replacement by fibro-osseous tissue, leading to bone expansion and cortical thickening rather than flattening; true vertebra plana is exceptional in this disorder, making it the least likely cause among the options. 42. Which nerve passes directly posterior to the medial epicondyle of the humerus, making it vulnerable to injury with a “funny-bone” knock? A. Axillary nerve B. Median nerve C. Radial nerve D. Ulnar nerve E. Musculocutaneous nerve
  • 353.
    EBDR Exam MCQs& Concepts October 2021 Dr. Kareem Alnakeeb 347 Explanation: (by Perplexity AI, generated with the OpenAI O3 model) • The ulnar nerve courses in the ulnar groove immediately behind the medial epicondyle before entering the cubital tunnel, so it is the structure struck in a classic “funny-bone” injury. • The axillary nerve winds around the surgical neck of the humerus, not the epicondyle. • The median nerve lies anterior to the elbow within the cubital fossa. • The radial nerve passes anterior to the lateral epicondyle as the posterior interosseous branch. • The musculocutaneous nerve terminates in the forearm and does not relate to the epicondyle. 43. Which CT feature is NOT typically associated with acute intestinal infarction? A. Free intraperitoneal air B. Mural gas C. Bowel wall thickening D. Thumb-printing E. Pneumatosis intestinalis Explanation: (by Perplexity AI, generated with the OpenAI O3 model) • Acute intestinal infarction most often shows intramural pneumatosis (pneumatosis intestinalis) and associated portal or mesenteric venous gas, producing the classic “mural gas” appearance. Ischemic oedema leads to circumferential bowel wall thickening, while submucosal hemorrhage or oedema gives rise to the radiological “thumb-printing” sign. • Free intraperitoneal air indicates bowel perforation rather than ischemia itself and is therefore not a direct sign of intestinal infarction, making option A the exception. • Options B, C and D represent well-recognized CT manifestations of ischemic injury. 44. Which anatomical site is least commonly the primary location for a paediatric neuroblastoma? A. Retroperitoneum B. Adrenal gland C. Anterior mediastinum D. Pelvis E. Neck Explanation: (by Perplexity AI, generated with the OpenAI O3 model) • More than 90% of neuroblastomas arise in the sympathetic chain of the abdomen, most frequently within the adrenal medulla or adjacent retroperitoneum. • Thoracic primaries, especially posterior mediastinal masses, account for a smaller but recognised proportion; anterior mediastinal origin is far rarer but still documented. • Cervical tumours are uncommon yet well described. • True pelvic primaries are distinctly rare, representing the least frequent site among the options, making pelvis the correct choice. • Distractors: adrenal and retroperitoneal sites are the commonest; anterior mediastinal and neck primaries, though uncommon, are encountered more often than pelvic lesions. 45. In a pre-term neonate presenting with abdominal distension, bilious vomiting and systemic instability, which statement regarding necrotizing enterocolitis is CORRECT? A. It is classified as a “critical case” in the Royal College of Radiologists’ pediatric imaging guidelines B. Sonographic features usually allow reliable antenatal diagnosis C. It commonly leads to pneumoperitoneum visible on abdominal radiographs D. All of the above
  • 354.
    EBDR Exam MCQs& Concepts October 2021 Dr. Kareem Alnakeeb 348 Explanation: (by Perplexity AI, generated with the OpenAI O3 model) • Pneumoperitoneum develops in 15–20% of infants with necrotizing enterocolitis when bowel perforation occurs, and free intraperitoneal gas is a classic radiographic sign guiding urgent surgical referral. • The RCR “Pediatric Digital Radiography” guidance lists NEC imaging as high priority rather than the highest “critical” category, so option A is inaccurate. • Antenatal ultrasound may show non-specific dilated bowel loops but does not reliably diagnose NEC, making option B incorrect. Therefore, only statement C is correct. 46. Q: In an otherwise healthy adult undergoing CT for nonspecific abdominal pain, bilateral adrenal masses are detected. Which statement regarding their etiology is most accurate? A. Simple adrenal adenomas account for about 10% of bilateral lesions B. Bilaterality strongly points to metastatic disease C. Acute bilateral adrenal hemorrhage is a recognized cause D. Both A and C E. None of the above Explanation: (by Perplexity AI, generated with the OpenAI O3 model) • Bilateral adrenal enlargement has a broad differential. While solitary adenomas are common, approximately 10% of adrenal adenomas are bilateral, so adenoma remains possible. • Acute stress-related or anticoagulation-related hemorrhage can also involve both glands, producing transient bilateral masses. • Although metastases (e.g. lung, breast) often seed both adrenals, bilaterality is not specific and therefore cannot be assumed to “strongly point” to metastatic disease; option B overstates this association. Therefore both statements A and C are correct. 47. Regarding colonic diverticulitis, which statement is most accurate for clinical practice? A. Non-contrast CT is preferred over contrast-enhanced CT for diagnosis B. It predominantly affects patients under 30 years of age C. It can present with large-bowel obstruction as a complication D. It never recurs after uncomplicated medical treatment E. All of the above statements are correct Explanation: (by Perplexity AI, generated with the OpenAI O3 model) • Contrast-enhanced CT is the imaging modality of choice because intravenous contrast better delineates inflamed bowel wall, abscesses and complications; non-contrast studies are less sensitive. Diverticulitis is classically a disease of older adults; incidence rises sharply after 40 years, so it is uncommon in the young. Complicated diverticulitis may lead to segmental colonic strictures causing acute or chronic large-bowel obstruction, making option C the correct choice. • Recurrence is noted in up to 20–30% of cases even after uncomplicated episodes, so option D is incorrect. Options A, B and D are therefore false.
  • 355.
    EBDR Exam MCQs& Concepts October 2021 Dr. Kareem Alnakeeb 349 48. An intravenous urography (IVU) series in a 55-year-old diabetic man with acute flank pain shows multiple filling defects producing an irregularly interrupted outline along the papillary tips of both kidneys (“ring- shadow” sign). What is the most likely diagnosis? A. Acute papillary necrosis B. Chronic pyelonephritis C. Renal transitional cell carcinoma D. Angiomyolipoma E. Medullary sponge kidney Explanation: (by Perplexity AI, generated with the OpenAI O3 model) • Acute papillary necrosis causes sloughed or infarcted renal papillae that detach and appear as ring-shaped filling defects on IVU, giving an interrupted or “ring-shadow” appearance of the normal papillary line. • Chronic pyelonephritis produces calyceal clubbing and cortical scarring, not discrete ring defects. • Transitional cell carcinoma typically shows an irregular solitary filling defect or narrowing of the collecting system, not multiple ring shadows. • Angiomyolipoma is usually detected as a fat-containing mass on cross-sectional imaging and is not outlined on IVU. • Medullary sponge kidney manifests as brush-like medullary striations due to ectatic collecting ducts, not ring shadows or papillary interruption. 49. A 16-year-old boy presents with persistent knee pain. Radiographs reveal a well-defined lucent lesion with sclerotic margin centered in the distal femoral epiphysis. What is the most typical anatomical site for a chondroblastoma? A. Epiphyseal B. Diaphyseal C. Metadiaphyseal D. Metaphyseal E. Meta-epiphyseal Explanation: (by Perplexity AI, generated with the OpenAI O3 model) • Chondroblastoma is a benign cartilaginous tumor that characteristically arises in the epiphysis (or apophysis) of long bones in skeletally immature patients, most often around the knee, proximal humerus or proximal femur. Epiphyseal location is therefore the hallmark and guides diagnosis. • Lesions purely in the metaphysis (D) or diaphysis (B) suggest other entities such as non-ossifying fibroma or eosinophilic granuloma, while metadiaphyseal (C) and meta-epiphyseal (E) sites are atypical and less specific, making these options distractors. 50. A 35-year-old woman with a palpable nodule in the left lobe of the thyroid gland showed a corresponding area of low activity on nuclear medicine study consistent with a cold nodule. How would you investigate this patient further? A. MRI neck B. CT neck with contrast C. US neck D. US neck with FNA E. Sialogram Source: Chin, Teck Yew, et al. Get Through Final FRCR 2A: SBAs. 1st ed., CRC Press, 2017.
  • 356.
    EBDR Exam MCQs& Concepts October 2021 Dr. Kareem Alnakeeb 350 Explanation: Thyroid Scanning and Nodule Assessment • Thyroid scanning using pertechnetate (99MTc) is traditionally used to screen thyroid nodules for malignancy. • The finding of a hyperfunctioning or ‘hot’ nodule (uptake of tracer within the nodule with suppression of uptake in the surrounding normal thyroid tissue) excludes malignancy in almost all patients. • A non-functioning or ‘cold’ nodule was thought to indicate increased risk of malignancy, with 5%– 15% of these being malignant. Investigations for Thyroid Nodules • FNA should be the first-line investigation for assessment of all solitary nodules or a dominant nodule in a multinodular goitre. • US is well established as a primary investigation for patients presenting with a lump in the neck; moreover, the cost-effectiveness and diagnostic accuracy of FNA can be increased by using US guidance and the presence of an on-site cytopathologist. 51. A 46-year-old woman is found to have a solitary cold nodule in the right thyroid lobe on technetium-99m pertechnetate scintigraphy. What is the most appropriate next management step? A. Repeat ultrasound in 12 months B. Ultrasound-guided fine-needle aspiration C. Contrast-enhanced CT neck D. MRI neck with gadolinium E. Thyroid-stimulating hormone suppression trial Explanation: (by Perplexity AI, generated with the OpenAI O3 model) • Cold (non-functioning) thyroid nodules carry a higher malignancy risk than “hot” nodules. • Current guidelines recommend targeted ultrasound to characterize echogenicity, margins and vascularity, immediately coupled with ultrasound-guided FNA for cytology when the nodule is ≥1 cm or has suspicious sonographic features. • Cross-sectional CT or MRI offers no first-line diagnostic advantage and may delay tissue diagnosis; iodine-containing contrast also interferes with subsequent radioiodine imaging. • Simple interval ultrasound follow-up alone is inappropriate in a potentially malignant cold nodule, and TSH suppression therapy is not recommended because it neither diagnoses nor reliably treats malignancy. 52. In which of the following clinical situations is detailed fetal echocardiography specifically recommended according to current obstetric-cardiology guidelines? A. Routine screening at 14 weeks’ gestation B. All twin pregnancies regardless of other risk factors C. Sustained fetal heart rhythm abnormality detected on antenatal scan D. Maternal age above 35 years as the sole indication E. Normal first-trimester ultrasound with nuchal translucency <2.5 mm Explanation: (by Perplexity AI, generated with the OpenAI O3 model) • Sustained or recurrent fetal arrhythmia (e.g. supraventricular tachycardia or complete heart block) warrants targeted fetal echocardiography to assess for structural heart disease and guide in-utero or perinatal management. • Routine screening at 14 weeks (option A) is too early for comprehensive cardiac assessment; standard anomaly scans with cardiac views are performed at 18-22 weeks.
  • 357.
    EBDR Exam MCQs& Concepts October 2021 Dr. Kareem Alnakeeb 351 • Twin pregnancy alone (option B) is not an absolute indication unless additional risk factors exist such as monochorionicity or abnormal findings. • Advanced maternal age (option D) increases aneuploidy risk but does not, by itself, mandate fetal echocardiography. • A completely normal early ultrasound (option E) does not justify further cardiac imaging. 53. Which hepatic focal lesion typically demonstrates true intratumoral fat on imaging? A. Hepatocellular carcinoma B. Cholangiocarcinoma C. Focal nodular hyperplasia D. Cavernous hemangioma Explanation: (by Perplexity AI, generated with the OpenAI O3 model) Intralesional fat is seen in a subset of hepatic focal lesions, but its frequency varies markedly among the major tumor types: • Hepatocellular carcinoma (HCC) – Fat is common enough to be listed as an “ancillary feature favoring HCC” in LI-RADS. Histologic series show microscopic or macroscopic fat in roughly 15–35% of small HCCs under 3 cm. • Focal nodular hyperplasia (FNH) – Fat can be demonstrated histologically in about 20% of resected FNHs, but only half of those cases show detectable signal loss on chemical-shift MRI, so visible fat on imaging is distinctly uncommon. • Cholangiocarcinoma – Intratumoral fat is essentially absent; when apparent, it almost always represents entrapped steatotic liver or focal fat deposition adjacent to the mass rather than fat inside the tumor itself. • Cavernous hemangioma – True fat within the vascular spaces is not a feature; hemangiomas may sit inside a background of fatty liver or show a rim of “peritumoral fat-sparing,” but the lesion itself does not contain fat. Therefore, among the four choices the hepatic focal lesion in which fat is a recognized intrinsic component is: Hepatocellular carcinoma (HCC) 54. On abdominal ultrasound, which one of the following sonographic features most reliably supports the diagnosis of congestive hepatomegaly secondary to right-sided heart failure? A. Dilated, non-pulsatile portal veins B. Pleural effusion without ascites C. Normal-calibre hepatic veins with monophasic Doppler flow D. Simultaneous dilatation of the inferior vena cava and all three hepatic veins with loss of normal triphasic Doppler waveform E. Markedly increased liver echogenicity suggestive of fatty infiltration Explanation: (by Perplexity AI, generated with the OpenAI O3 model) • Passive hepatic congestion occurs when raised right-atrial pressure is transmitted to the liver. The most consistent ultrasound findings are a dilated inferior vena cava (>21 mm) together with enlarged hepatic veins (>8–9 mm) showing dampened or reversed (‘to-and-fro’) flow and loss of the normal triphasic waveform; these changes directly reflect elevated central venous pressure, making option D correct. • Isolated portal-vein dilatation (A) is non-specific and may be seen in portal hypertension.
  • 358.
    EBDR Exam MCQs& Concepts October 2021 Dr. Kareem Alnakeeb 352 • Small pleural effusions (B) are common in cardiac failure but are an extra-hepatic sign and not diagnostic of hepatic congestion. • Normal-caliber hepatic veins with monophasic flow (C) argue against congestion. • Increased hepatic echogenicity (E) suggests steatosis rather than venous congestion. 55. In the context of bone tumors and infection, which statement regarding a Codman triangle is FALSE? A. It forms between the elevated periosteum and underlying cortex B. It represents an aggressive periosteal reaction C. It may be demonstrated in a non-ossifying fibroma D. It is classically associated with osteosarcoma E. It is usually seen in acute osteomyelitis Explanation: (by Perplexity AI, generated with the OpenAI O3 model) • A Codman triangle arises when rapidly growing pathology lifts the periosteum away from the cortex, producing a triangular edge of new bone (option A true). • Because such rapid elevation reflects aggressive disease, the reaction is considered aggressive (option B true). • Although most typical of high-grade malignancies like osteosarcoma (option D true), any fast- expanding lesion—including some benign entities such as a large non-ossifying fibroma—can create the appearance (option C true). • Acute osteomyelitis often shows a solid or lamellated periosteal response; the Codman triangular pattern is uncommon, making option E the false statement. 56. A 38-year-old woman presents with chronic menorrhagia. Pelvic MRI shows a 4 cm left adnexal lesion that is uniformly low signal on T1-weighted images and markedly low signal on T2-weighted images. Which is the most likely diagnosis? A. Endometrioma B. Leiomyoma C. Hemorrhagic cyst D. Ovarian fibroma E. Serous cystadenoma Explanation: (by Perplexity AI, generated with the OpenAI O3 model) • Ovarian fibromas are solid, fibrous stromal tumors that appear low signal on both T1 and T2 sequences because of their dense collagen content; they may mimic uterine fibroids but are usually separate from the uterus. • Endometriomas and hemorrhagic cysts typically show high T1 signal from blood products, making options A and C incorrect. • Leiomyomas (uterine fibroids) can appear low on T1/T2 but would arise from the uterus, not the adnexa, so B is less likely when a distinct ovarian mass is seen. • Serous cystadenomas are predominantly cystic with fluid signal (low T1, high T2), unlike the uniformly low T2 lesion described, excluding option E.
  • 359.
    EBDR Exam MCQs& Concepts October 2021 Dr. Kareem Alnakeeb 353 57. In myocardial hibernation, which statement best describes the underlying pathophysiological change? A. Irreversible myocyte necrosis with normal coronary flow B. Transient coronary spasm causing acute stunning C. Persistently reduced myocardial perfusion with preserved viability D. Normal perfusion with idiopathic cardiomyopathy E. Patchy myocardial fibrosis following myocarditis Explanation: (by Perplexity AI, generated with the OpenAI O3 model) • Myocardial hibernation refers to chronically under-perfused but still viable myocardium that down- regulates contractile function to match the reduced blood supply; when adequate coronary flow is restored (e.g. by revascularization) contractility can recover. Therefore, the key feature is a persistent decrease in perfusion while cellular viability is maintained (Option C). • Option A is false because hibernation is reversible and does not involve necrosis. • Option B describes myocardial stunning, a different phenomenon due to acute ischemia with rapid flow restoration. • Option D has normal perfusion, so does not fit the definition. • Option E relates to post-inflammatory scarring, which is non-viable tissue and cannot improve with revascularization. 58. A 29-year-old woman is referred for pelvic ultrasound because of intermittent lower-abdominal pain. Transvaginal imaging shows a 3 cm well-defined, thin-walled adnexal cyst that is uniformly hyperechoic with posterior acoustic shadowing; serum β-hCG is negative. Which is the most likely diagnosis? A. Hemorrhagic corpus luteum cyst B. Endometrioma C. Mature cystic (dermoid) teratoma D. Serous cystadenoma E. Para-ovarian cyst Explanation: (by Perplexity AI, generated with the OpenAI O3 model) • Mature cystic teratomas typically appear as small, well-circumscribed adnexal masses containing echogenic sebaceous material and hair that produce marked acoustic shadowing—the classic “tip- of-the-iceberg” sign—matching the uniformly hyperechoic lesion here. • Hemorrhagic corpus luteum cysts are usually reticular or crenulated with internal echoes and evolve over weeks; endometriomas display homogeneous low-level echoes (“ground-glass”) rather than dense echogenicity. • Serous cystadenomas and para-ovarian cysts are thin-walled, anechoic and do not cast shadowing. 59. A 15-year-old girl presents with acute pelvic pain. Beta-human chorionic gonadotrophin (β-hCG) is normal. US demonstrates a thin-walled 5 cm echogenic adnexal mass with posterior acoustic enhancement. There is no color internal Doppler signal. Follow-up imaging after 3 months fails to demonstrate the mass. What is the most likely cause? A. Appendix abscess B. Hemorrhagic ovarian cyst C. Ovarian dermoid D. Ectopic pregnancy E. Ovarian torsion
  • 360.
    EBDR Exam MCQs& Concepts October 2021 Dr. Kareem Alnakeeb 354 Source: Chin, Teck Yew, et al. Get Through Final FRCR 2A: SBAs. 1st ed., CRC Press, 2017. Explanation: Right Hemipelvic Mass Presentations • All of the above may present as a right hemipelvic mass. Hemorrhagic Ovarian Cyst • Hemorrhage into an ovarian follicular cyst is the most common of these, and it usually resolves after one or two menstrual cycles. • Several patterns of ultrasound findings have been described, including an echogenic mass, a ground- glass pattern (diffuse low-level echoes), a whirled pattern of mixed echogenicity and a ‘fishnet weave’ pattern (fine septations or reticular echoes). Appendiceal Abscess • Appendix abscess would be thick-walled. Ovarian Dermoid • Ovarian dermoid may present as an echogenic mass, although acoustic shadowing would be more typical owing to internal calcifications and would not resolve in this fashion. Ectopic Pregnancy • Ectopic pregnancy may present as an echogenic ‘tubal mass’, although elevated β-hCG would be a feature. Ovarian Torsion • Ovarian torsion may appear as an enlarged echogenic ovary (owing to oedema) and, like hemorrhagic cyst, often lacks internal color Doppler flow, although it is less common than hemorrhagic cyst. • It would also necessitate urgent surgery rather than follow-up imaging. 60. Q: Which statement regarding Peutz–Jeghers syndrome is correct? A. It is inherited in an autosomal dominant pattern B. It presents with mucocutaneous pigmentation and gastrointestinal hamartomatous polyps C. It follows an autosomal recessive inheritance pattern D. Both statements B and C are correct E. There is no increased risk of malignancy compared with the general population Explanation: (by Perplexity AI, generated with the OpenAI O3 model) • Peutz–Jeghers syndrome is classically characterized by multiple hamartomatous polyps throughout the gastrointestinal tract together with distinctive mucocutaneous melanotic macules, especially around the lips and oral mucosa, making option B correct. • The condition is inherited in an autosomal dominant fashion due to pathogenic variants in the STK11 (LKB1) gene, so option A is incorrect. • It is not autosomal recessive, rendering options C and D incorrect. • Patients have a markedly increased lifetime risk of several malignancies (pancreatic, gastrointestinal, breast, others), so option E is wrong.
  • 361.
    EBDR Exam MCQs& Concepts October 2021 Dr. Kareem Alnakeeb 355 61. Which of the following extra-renal abnormalities is most characteristically associated with autosomal dominant polycystic kidney disease (ADPKD)? A. Bladder diverticulum B. Seminal vesicle cysts C. Coronary artery aneurysm D. Portal cavernoma E. Choledochal cyst Explanation: (by Perplexity AI, generated with the OpenAI O3 model) • Up to 40% of adults with ADPKD develop cystic dilatation of the seminal vesicles because polycystin defects affect epithelial-lined ducts beyond the kidneys. • Bladder diverticula can occur in raised intravesical pressure but have no specific link to ADPKD. • Coronary aneurysms are more typical of connective-tissue disorders such as Kawasaki disease. • Portal cavernoma reflects chronic portal vein thrombosis, unrelated to cystic kidney disease. • Choledochal cysts are congenital bile duct anomalies without proven association with ADPKD. 62. A 28-year-old woman being investigated for primary infertility undergoes hysterosalpingography, which shows two endometrial canals that diverge superiorly; the intercornual angle is measured at 120° and the external uterine fundus demonstrates a 10 mm deep cleft. Which Müllerian duct anomaly is most likely? A. Arcuate uterus B. Bicornuate uterus C. Septate uterus D. Uterus didelphys E. Unicornuate uterus Explanation: (by Perplexity AI, generated with the OpenAI O3 model) • On HSG, a bicornuate uterus shows incomplete fusion of the Müllerian ducts, producing two endometrial cavities separated by myometrium. • Key diagnostic features are a wide intercornual angle (>105°) and an external fundal cleft deeper than 1 cm, both present here. • A septate uterus (C) has a normal or mildly indented fundal contour and an acute intercornual angle (<75°). • Uterus didelphys (D) has two completely separate horns with two cervices and usually a vaginal septum; the external cleft is much deeper. • An arcuate uterus (A) shows only a shallow (<1 cm) mid-fundal indentation, while a unicornuate uterus (E) has a single, elongated uterine cavity. 63. Q: Which anatomical variant most commonly predisposes to extra-articular subacromial shoulder impingement? A. High coracoacromial ligament insertion B. Type II curved acromion C. Type III hooked acromion D. Os acromiale E. Prominent coracoid process Explanation: (by Perplexity AI, generated with the OpenAI O3 model) • A type III hooked acromion projects inferiorly, narrowing the subacromial space and repeatedly abrading the supraspinatus tendon, making it the classic variant associated with mechanical (extra- articular) impingement.
  • 362.
    EBDR Exam MCQs& Concepts October 2021 Dr. Kareem Alnakeeb 356 • A type II curved acromion (B) is common but less strongly linked to symptomatic impingement. • An os acromiale (D) may alter acromial tilt yet is a rarer cause. • Coracoid variants such as a high coracoacromial ligament insertion (A) or a prominent coracoid process (E) relate to subcoracoid, not subacromial, impingement. 64. Regarding the MRI enhancement pattern of focal nodular hyperplasia (FNH) in the liver, which of the following statements is most accurate? A. The central scar retains gadolinium contrast on delayed phase images. B. The lesion demonstrates peripheral wash-out on portal venous phase. C. FNH shows marked diffusion restriction on high b-value DWI. D. The lesion is T1 hyperintense relative to spleen on pre-contrast images. E. The enhancing rim persists on equilibrium phase imaging. Explanation: (by Perplexity AI, generated with the OpenAI O3 model) • FNH typically enhances rapidly and homogeneously in the arterial phase with a characteristic central scar that becomes hyperintense on delayed (hepatobiliary) or equilibrium phases because the fibrous tissue gradually retains gadolinium, making option A correct. • Unlike hepatocellular carcinoma, FNH does not exhibit peripheral wash-out on portal venous phase, so option B is wrong. • Diffusion restriction is usually absent or mild in FNH; marked restriction suggests malignant lesions, invalidating option C. • On non-contrast T1-weighted images, FNH is usually iso- or mildly hypo-intense to liver, not frankly hyperintense as in option D. • An enhancing rim is typical of hemangioma or metastasis, not FNH, making option E incorrect. 65. Q: Which imaging finding is classically termed the “double-wall” (Rigler) sign on an abdominal radiograph? A. Linear lucency outlining both sides of the bowel wall due to intraluminal and extraluminal gas B. Air-fluid level within the gallbladder lumen C. Crescent of subdiaphragmatic free gas beneath the right hemidiaphragm D. Mottled gas within the biliary tree branching towards the liver periphery E. Multiple step-ladder air-fluid levels within dilated small-bowel loops Explanation: (by Perplexity AI, generated with the OpenAI O3 model) • The Rigler sign appears when air is present on each side of the intestinal wall—inside the bowel lumen and in the peritoneal cavity—producing two parallel lucent lines that outline both serosal and mucosal surfaces. It is a reliable indicator of pneumoperitoneum. • Option B describes emphysematous cholecystitis; option C is the classic subdiaphragmatic free-gas sign but not the double-wall sign; option D represents pneumobilia; option E depicts small-bowel obstruction, not pneumoperitoneum. 66. In a patient with a suspected bronchogenic carcinoma, which radiographic sign describes an indentation on the edge of a peripheral lung mass produced by a supplying pulmonary vessel? A. Cervicothoracic sign B. Silhouette sign C. Air crescent sign D. Pleural tail sign E. Regel (notch) sign
  • 363.
    EBDR Exam MCQs& Concepts October 2021 Dr. Kareem Alnakeeb 357 Explanation: (by Perplexity AI, generated with the OpenAI O3 model) • The Regel notch sign refers to a small concave indentation on the margin of a peripheral lung tumour where a feeding pulmonary vessel enters, helping differentiate a vascular-supplied malignant mass from a non-vascular lesion. • The cervicothoracic sign (A) localises mediastinal masses above the clavicles, not peripheral tumours. • The silhouette sign (B) is loss of normal cardiomediastinal borders due to adjacent opacity. • The air crescent sign (C) describes a peripheral rim of air around a fungal ball or resolving invasive aspergillosis. • The pleural tail sign (D) is a thin linear extension from a subpleural mass toward the pleura, typical of pleural-based metastasis or mesothelioma, not a vascular notch. 67. A 38-year-old woman with a history seat belt injury in a road traffic accident 1 year ago, presents with a right breast lump. Mammography shows a ‘hollow’ spherical abnormality measuring about 4 cm with a rim of thin curvilinear area of calcification in the right breast. What is the most likely diagnosis? A. Vascular calcification B. Fat necrosis C. Secretory calcifications in ducts D. Milk of calcium E. Ductal carcinoma in situ Source: Gupta, Chaitanya. 300 Single Best Answers for the Final FRCR Part A. 1st ed., Jaypee UK, 2010. Explanation: • Egg shell’ calcifications are seen in patients with fat necrosis. This can be secondary to blunt trauma or it can be post-surgical. 68. On musculoskeletal ultrasound of the hands, which finding is considered most characteristic of active rheumatoid arthritis? A. Hyperechoic intra-articular loose bodies B. Anechoic joint effusion without synovial thickening C. Power-Doppler signal within hypoechoic synovial hypertrophy D. Peri-tendinous calcific foci along extensor tendons E. Subcutaneous tophaceous deposits over extensor surfaces Explanation: (by Perplexity AI, generated with the OpenAI O3 model) • Doppler-detectable vascular flow inside a hypoechoic, thickened synovium (power-Doppler positivity) indicates active synovial inflammation and is highly characteristic of active rheumatoid arthritis, correlating with disease activity and predicting future erosive damage. • Simple effusion (option B) may be seen in many arthritides and lacks specificity. • Hyperechoic loose bodies (option A) are typical of osteoarthritis or synovial chondromatosis, not RA. • Calcific deposits around tendons (option D) suggest calcific tendinopathy. • Subcutaneous tophi (option E) are a hallmark of gout, not rheumatoid arthritis.
  • 364.
    EBDR Exam MCQs& Concepts October 2021 Dr. Kareem Alnakeeb 358 69. Which of the following is the LEAST likely underlying cause of small-bowel intussusception in adults? A. Post-infective mucosal oedema following bacterial gastroenteritis B. Ingested foreign body acting as a lead point C. Small-bowel lymphoma D. Intramural lipoma E. Pneumatosis intestinalis Explanation: (by Perplexity AI, generated with the OpenAI O3 model) • Adult intussusception almost always has a structural lead point. • Neoplasms such as lymphoma or benign tumors like lipomas account for most cases because they disturb peristalsis and drag the proximal segment into the distal lumen. • Less commonly, an indigestible foreign body can serve as a lead point. • Bacterial enteritis may cause transient mucosal oedema and hyper-peristalsis sufficient to telescope the bowel, particularly in children, but it is still a recognized mechanism in adults. • Pneumatosis intestinalis, however, consists of intramural gas collections and does not usually create a discrete mass or traction focus, making it an unlikely precipitant of intussusception. 70. During transrectal ultrasonography of the prostate, which sonographic finding is most suggestive of prostate carcinoma in the peripheral zone? A. Uniformly isoechoic but markedly hypervascular nodule B. Well-defined hyperechoic lesion with peripheral flow only C. Ill-defined hypoechoic area with increased internal Doppler flow D. Cystic lesion with posterior acoustic enhancement E. Diffuse heterogeneous enlargement without focal lesion Explanation: (by Perplexity AI, generated with the OpenAI O3 model) • Prostate cancer most often arises in the peripheral zone and typically appears as an ill-defined hypoechoic focus on B-mode imaging because malignant tissue replaces the normally echogenic glandular stroma. Malignant neovascularity produces chaotic, increased internal Doppler flow, supporting the diagnosis. • Isoechoic or hyperechoic nodules (A, B) are less characteristic of carcinoma; hyperechoic foci usually represent calcification or fibrosis, while isoechoic lesions are difficult to detect sonographically. • Cysts with through-transmission (D) indicate benign cystic change. • Diffuse heterogeneous enlargement without a discrete lesion (E) is more typical of benign prostatic hyperplasia than carcinoma. 71. According to CT size criteria, lymphadenopathy is defined when which measurement threshold is exceeded? A. Lymph node short-axis diameter >10 mm B. Lymph node short-axis diameter >8 mm C. Lymph node short-axis diameter >5 mm D. Lymph node long-axis diameter >15 mm E. Lymph node volume >1 cm³ Explanation: (by Perplexity AI, generated with the OpenAI O3 model) • On contrast-enhanced CT, most radiology guidelines label a lymph node as pathologically enlarged (lymphadenopathy) when its short-axis diameter surpasses 10 mm. This cutoff balances sensitivity and specificity across nodal stations.
  • 365.
    EBDR Exam MCQs& Concepts October 2021 Dr. Kareem Alnakeeb 359 • Nodes with short axes of 5–8 mm (option B and C) are usually considered within normal limits unless morphologically suspicious. • Using the long-axis dimension alone (option D) misses many round but enlarged nodes. • Routine volumetric thresholds (option E) are not part of standard CT reporting criteria. 72. Chron's disease of small intestine: A. Terminal ileum is affected in 80% of case. B. Colon is affected in 10% of case. C. Treated surgically. D. Presenting with deep penetrating ulcers. Explanation: (by Perplexity AI, generated with the OpenAI O3 model) • Terminal ileum involvement ≈ 70-80% ➜ True. • Colonic disease only ≈ 20% (±50% ileocolonic) ➜ “10% colon” statement is False. • Management today is medical first (steroids, immunomodulators, biologics). Surgery only for strictures, fistulae, perforation, bleeding or refractory disease ➜ “treated surgically” is False. • Transmural inflammation gives deep, fissuring / penetrating ulcers—cobblestone mucosa ➜ True. 73. A 35-year-old man has distal femur pain. Radiographs demonstrate a well-defined, expansile meta- epiphyseal lytic lesion breaching the cortex; MRI shows uniformly low signal on both T1- and T2-weighted images. What is the most likely diagnosis? A. Simple bone cyst B. Plasma cell myeloma C. Langerhans cell histiocytosis D. Giant cell tumor E. Chondrosarcoma Explanation: (by Perplexity AI, generated with the OpenAI O3 model) • Giant cell tumor typically arises in skeletally mature adults (20–40 years), abuts the articular surface in the meta-epiphysis, appears expansile and well-defined, and may erode the cortex. It characteristically shows low to intermediate signal on both T1 and T2 MRI because of abundant solid cellular stroma and hemosiderin, matching this case. • Simple bone cysts are metaphyseal, centrally located in children and show high T2 signal. • Plasma cell myeloma is rare in a solitary form at this age and gives high T2 signal. • Langerhans cell histiocytosis usually affects children and produces poorly marginated lesions with impermeant cortices. • Chondrosarcoma presents later in life and shows chondroid matrix with very high T2 signal. 74. A 13 years old boy presented with pain in his right upper thigh for 6 months, mainly at night and relieved by analgesics. Plain x-ray revealed a small lucent cortical lesion at the upper femoral shaft with surrounding sclerosis. What is the most likely diagnosis? A. Non-ossifying fibroma B. Osteoid osteoma C. Bone metastasis D. Brodie’s abscess E. Ewing sarcoma Explanation: (by Perplexity AI, generated with the OpenAI O3 model) • Osteoid osteoma classically affects adolescents, producing nocturnal pain that responds dramatically to NSAIDs. Imaging typically demonstrates a tiny cortical lucent nidus (<1.5 cm) surrounded by dense reactive sclerosis in long-bone diaphyses, especially the femur.
  • 366.
    EBDR Exam MCQs& Concepts October 2021 Dr. Kareem Alnakeeb 360 • Non-ossifying fibroma appears as an eccentric metaphyseal lesion with a multilobulated outline and lacks intense pain relief with analgesia. • Bone metastases are rare at this age and usually multifocal without a characteristic nidus. • Brodie’s abscess (subacute osteomyelitis) may mimic the lesion but usually presents with a larger, metaphyseal, oval lucency with a sclerotic rim and often systemic or inflammatory signs. • Ewing sarcoma produces a permeative pattern with soft-tissue mass rather than a well-defined nidus. 75. On ultrasound, which gallbladder condition typically shows diffuse wall thickening with intramural echogenic foci at the neck producing reverberation “comet-tail” artefacts and distal shadowing? A. Xanthogranulomatous cholecystitis B. Adenomyomatosis C. Gallbladder polyps D. Impacted gallstone E. Acute calculous cholecystitis Explanation: (by Perplexity AI, generated with the OpenAI O3 model) • Adenomyomatosis is a benign hyperplastic condition where Rokitansky–Aschoff sinuses become filled with cholesterol crystals. The intramural crystals create multiple closely spaced interfaces, generating characteristic comet-tail reverberation and short acoustic shadowing on ultrasound, most pronounced in the gallbladder fundus or neck. • Xanthogranulomatous cholecystitis (A) produces heterogeneous wall thickening but lacks the crisp comet-tail artefact. • Gallbladder polyps (C) appear as non-shadowing intraluminal lesions attached to the wall. • An impacted gallstone (D) casts a clean acoustic shadow rather than a reverberation tail. • Acute calculous cholecystitis (E) shows wall oedema and gallstones with posterior shadowing but not the intramural comet-tail pattern. 76. In ultrasound assessment of chronic liver disease, what caudate-to-right-lobe (C/RL) ratio is considered specific for cirrhosis? A. More than 0.73 B. Less than 0.55 C. Less than 0.60 D. Less than 0.35 E. More than 1.00 Explanation: (by Perplexity AI, generated with the OpenAI O3 model) • The caudate lobe enlarges out of proportion to the right lobe in cirrhosis because its drainage is preserved while the rest of the liver undergoes fibrosis and atrophy; a C/RL ratio >0.73 strongly suggests cirrhosis and has high specificity. • Ratios <0.60 (options B and C) or <0.35 (option D) indicate a relatively small caudate lobe and are not typical of cirrhosis. • A ratio >1.00 (option E) is possible but far less commonly used as the diagnostic threshold and lacks supporting evidence.
  • 367.
    EBDR Exam MCQs& Concepts October 2021 Dr. Kareem Alnakeeb 361 77. The MOST common site of gastrointestinal stromal tumours (GISTs) is: A. Esophagus B. Stomach C. Small intestine D. Colon E. Rectum Explanation: (by Perplexity AI, generated with the OpenAI O3 model) • Around 55–60% of GISTs arise in the stomach owing to the high density of interstitial cells of Cajal in the gastric wall, which are believed to be their cells of origin. • The small intestine is the next most frequent site (about 30%), while colonic, rectal and esophageal tumors are uncommon. • Hence stomach is the single most common location, making options relating to other bowel segments incorrect. 78. A 2-year-boy presents with a 2-week history of melena culminating in an acute episode of bright red blood per rectum. Ultrasound was unremarkable. Upper gastrointestinal endoscopy was negative. Technetium pertechnetate demonstrates increased uptake in the left upper and right lower quadrants. What is the most likely diagnosis? A. Acute appendicitis B. Intussusception C. Meckel’s diverticulum D. Gastrinoma E. Non-specific inflammatory bowel disease Source: Chin, Teck Yew, et al. Get Through Final FRCR 2A: SBAs. 1st ed., CRC Press, 2017. Explanation: Meckel’s Diverticulum and Technetium Pertechnetate Scan Findings • The findings on this technetium pertechnetate scan are typical of a Meckel’s diverticulum containing ectopic gastric mucosa. • Secretions from ectopic gastric tissue in a Meckel’s diverticulum can cause ulceration of the diverticulum or adjacent small bowel and can lead to bleeding, which, if profuse, can simulate an upper gastrointestinal bleed. • Meckel’s diverticulum may simulate acute appendicitis on clinical examination; however, a pertechnetate scan is only performed if a Meckel’s diverticulum is suspected clinically. Interpretation of Tracer Uptake • The uptake in the left upper quadrant should not mislead the radiologist as it is the result of normal tracer uptake in the stomach mucosa. Gastrinoma Location • A gastrinoma is usually located in the pancreatic islet cells, not the right iliac fossa.
  • 368.
    EBDR Exam MCQs& Concepts October 2021 Dr. Kareem Alnakeeb 362 79. Which is the most commonly used imaging modality for the diagnosis of gastrointestinal tract (GIT) carcinoid tumors? A. CT scan B. Indium-111 Octreotide scintigraphy C. SHIAA D. HIDA scan E. PET-CT Explanation: (by Perplexity AI, generated with the OpenAI O3 model) • Indium-111 Octreotide scintigraphy (OctreoScan) is the investigation of choice for detecting GIT carcinoid tumors because these neuroendocrine tumors express somatostatin receptors, which the radiolabeled octreotide binds to with high sensitivity. • CT scan is widely used for anatomical localization and staging, but is less sensitive for detecting small or functionally active tumors. • SHIAA (5-Hydroxyindoleacetic acid) is a urinary metabolite used for biochemical diagnosis, not localization. • HIDA scan is used in biliary pathology and is not useful in carcinoid detection. • PET-CT may be useful but is not the standard method, especially compared to OctreoScan. 80. Which of the following is NOT a recognised radiological sign of intestinal tuberculosis? E. Fleischner sign F. String sign G. Stierlin's sign H. Comb sign Explanation: (by Perplexity AI, generated with the OpenAI O3 model) • The comb sign (engorged mesenteric vessels) is typically seen in Crohn’s disease, not intestinal tuberculosis. • Fleischner sign (thickened loop due to spasm), string sign (narrow, rigid segment), and Stierlin's sign (rapid passage through ulcerated segment with minimal mucosal coating) are all described in intestinal TB and help differentiate it from other pathologies. • The comb sign is a classic finding in active Crohn’s, reflecting hypervascularity, and is not associated with tuberculosis. 81. A 13-year-old post-pubescent girl presents to the emergency department with acute abdominal pain sited predominantly within the right iliac fossa. An ultrasound scan is performed. This reveals an echogenic mass within the right side of pelvis measuring approximately 4 cm. The sonographer thinks it is adjacent to and inseparable from the right ovary. What is the most likely diagnosis? A. Acute appendicitis B. Ovarian dermoid C. Ovarian torsion D. Ectopic pregnancy E. Hemorrhagic ovarian cyst Source: Chin, Teck Yew, et al. Get Through Final FRCR 2A: SBAs. 1st ed., CRC Press, 2017. Explanation: Causes of Acute Pelvic Pain in Adolescent Girls • Acute pelvic pain in adolescent girls is a common problem, but ultrasound scanning is very useful in differentiating the many possible causes. Hemorrhagic Ovarian Cysts
  • 369.
    EBDR Exam MCQs& Concepts October 2021 Dr. Kareem Alnakeeb 363 • Hemorrhagic ovarian cysts are a common cause of pelvic pain in adolescent girls and appear as an echogenic mass in relation to the ovary. Acute Appendicitis • Acute appendicitis is likely to occur as a blind-ending tubular structure. • This may appear like a ‘target lesion’ in cross section and there may be fluid/collection adjacent to it. • An acute appendix should be clearly distinct from the right ovary. Ovarian Dermoids • Ovarian dermoids are usually predominantly fat filled and therefore echogenic on ultrasound, but these tend to be a painless, incidental finding. Ovarian Torsion • Ovarian torsion can certainly produce an echogenic mass within the right pelvis, but this is less common than hemorrhagic cysts and would not appear distinct from the ovary. Ectopic Pregnancy • Ectopic pregnancy usually appears as a ‘doughnut’-shaped complex mass in relation to one of the uterine tubes; a fetal heartbeat may be present. 82. Which of the following statements about plantar fasciitis is correct? A. The plantar fascia is not normally attached to the calcaneus B. Plantar fasciitis may be associated with ankylosing spondylitis C. The presence of a calcaneal spur is diagnostic on X-ray D. Plantar fasciitis shows high T1 signal around the fascia E. Plantar fasciitis never causes morning pain Explanation: (by Perplexity AI, generated with the OpenAI O3 model) • Plantar fasciitis can be associated with seronegative spondyloarthropathies such as ankylosing spondylitis, making option B correct. • Option A is incorrect because the plantar fascia is normally attached to the calcaneus. • Option C is wrong—calcaneal spurs are often seen on X-ray, but their presence is not specific or diagnostic, as they can occur in asymptomatic individuals as well. • Option D is incorrect because plantar fasciitis typically shows low T1 and high T2 signal around the fascia on MRI, reflecting oedema. • Option E is incorrect as plantar fasciitis classically causes more pain in the morning or after periods of rest. 83. The most common site of gastric ulcer is: A. Greater curvature B. Lesser curvature C. Pyloric antrum D. Gastric fundus Explanation: (by Perplexity AI, generated with the OpenAI O3 model) The most common site of gastric ulcer is the lesser curvature of the stomach. Specifically, gastric ulcers are frequently found along the lesser curve, often near the incisura angularis, which is an area of relative mucosal weakness. To clarify the options: • Greater curvature: Ulcers are less commonly located here. • Lesser curvature: Most common site for gastric ulcers4.
  • 370.
    EBDR Exam MCQs& Concepts October 2021 Dr. Kareem Alnakeeb 364 • Pyloric antrum: While ulcers can occur here, especially in the pyloric channel, it is not the most common location. • Gastric fundus: Rare site for gastric ulcers. Therefore, the correct answer is Lesser curvature. 84. A 42-year-old woman is referred to the breast clinic and is due an ultrasound scan to evaluate a suspected lump in the breast. All of the following are ultrasonographic features of a benign breast mass, except A. Feeding central vessel on Doppler imaging B. Well-defined smooth margins C. Three or fewer lobulations D. Circumferential blood flow pattern on Doppler imaging E. Uniform hyperechogenicity Source: Chin, Teck Yew, et al. Get Through Final FRCR 2A: SBAs. 1st ed., CRC Press, 2017. Explanation: Ultrasound Features of Benign Lesions • US features characteristic of benign lesions have been described. • These include hyperechogenicity compared to fat, an oval or well-defined, lobulated, gently curving shape and the presence of a thin echogenic pseudocapsule. • Doppler examination of benign lesions shows displacement of normal vessels around the edge of the lesion. Ultrasound Features of Malignant Lesions • In contrast, malignant lesions show abnormal vessels that are irregular and centrally penetrating. 85. An 85-year-old male presented with abdominal pain and vomiting for 1 week. CT shows a dilated, C- shaped loop with circumferential wall thickening, increased attenuation, delayed uneven wall enhancement, and a few air loculi within the bowel walls. What is the most likely diagnosis? A. Small bowel obstruction B. Strangulated small bowel C. Internal hernia D. Volvulus Explanation: (by Perplexity AI, generated with the OpenAI O3 model) • The described CT features—C-shaped dilated loop, circumferential wall thickening, increased attenuation, delayed uneven wall enhancement, and intramural gas—are classic for strangulated small bowel. These findings suggest compromised bowel vascularity and evolving ischaemia, rather than simple obstruction (which typically lacks evident wall ischaemia). • Volvulus or internal hernia may cause obstruction but usually require additional specific anatomical features (e.g., whirl sign, mesenteric vessels converging on a point) and do not by themselves explain the extent of mural changes and pneumatosis described here. 86. Regarding ovarian torsion, which of the following is NOT true? A. It can occur during early pregnancy. B. The left ovary is affected more than the right. C. It has the highest incidence during ovulation induction. D. It may be due to enlargement of the ovary. E. It often presents with acute pelvic pain.
  • 371.
    EBDR Exam MCQs& Concepts October 2021 Dr. Kareem Alnakeeb 365 Explanation: (by Perplexity AI, generated with the OpenAI O3 model) • Ovarian torsion most commonly affects the right ovary rather than the left, likely due to differences in pelvic anatomy and the presence of the sigmoid colon on the left, which restricts mobility. • It can occur in early pregnancy, particularly because of corpus luteum cysts. • Ovulation induction (e.g., in fertility treatments) significantly increases the risk due to ovarian enlargement and increased mobility. • Any process causing ovarian enlargement—including cysts or tumors—predisposes to torsion. • Acute pelvic pain is a classic presenting symptom. • The left ovary being more commonly affected is incorrect. 87. A 60-year-old presents with left groin pain. Ultrasound shows a 2 cm hypoechoic lesion bulging medial to the epigastric vessels on Valsalva maneuver and absent on rest. What is the most likely diagnosis? A. Direct inguinal hernia B. Indirect inguinal hernia C. Obturator hernia D. Spigelian hernia E. Femoral hernia Source: Gupta, Chaitanya. 300 Single Best Answers for the Final FRCR Part A. 1st ed., Jaypee UK, 2010. Explanation: • A direct inguinal hernia is seen medial to the inferior epigastric vessels whereas an indirect hernia is seen lateral to them. 88. The following are signs of a normal gestational sac, except A. Intradecidual sign. B. Cardiac activity seen with a CRL (crown-rump length) of 6 mm. C. Double decidual sign. D. Mean sac diameter increases by 1 mm/day. E. Embryo seen with a mean sac diameter of 10 mm. Source: Chin, Teck Yew, et al. Get Through Final FRCR 2A: SBAs. 1st ed., CRC Press, 2017. Explanation: • The gestational sac is first identifiable on transvaginal ultrasound at 4.5 weeks. It appears as a round 2–3 mm fluid collection. It is located in the central echogenic part of the endometrium (decidua). In some cases, it is surrounded by two echogenic rings corresponding to the two layers of decidua, described as the double decidual sac sign of intrauterine pregnancy. Sometimes the gestational sac is eccentrically located on one side of a thin white line corresponding to the collapsed uterine cavity, called the intradecidual sign. • The yolk sac is the first structure visualized on TVS (trans vaginal scan) within the sac at 5.5 weeks. Yolk sac is evident when sac diameter is 10 mm. Heartbeat is evident when crown–rump length (CRL) is 5 mm. • On TVS, an embryo is seen when the mean sac diameter is 18 mm. Mean sac diameter increases by approximately 1 mm per day. Lack of fetal pole in a gestational sac with diameter more than 20 mm is suggestive of an anembryonic or nonviable pregnancy. Summary of key points: • Gestational sac first visible by TVS: 4.5–5 weeks • Sac size at first detection: ~2–3 mm round fluid collection
  • 372.
    EBDR Exam MCQs& Concepts October 2021 Dr. Kareem Alnakeeb 366 • Double decidual sac sign: two echogenic rings around sac, supporting intrauterine location • Yolk sac visible at ~5.5 weeks, when sac diameter ~10 mm • Embryo seen at sac diameter ~18 mm, heartbeat visible when CRL ~5 mm • Mean sac diameter increases about 1 mm per day • Absence of fetal pole when sac diameter >20 mm suggests nonviable pregnancy 89. A woman presents with infertility and undergoes a hysterosalpingogram. This demonstrates a uterus with two converging horns. A wide angle is seen at the roof of the uterus. Which uterine anomaly does the patient have? A. Uterine didelphys B. Septate uterus C. Arcuate uterus D. Bicornuate uterus E. Unicornuate uterus Source: Chin, Teck Yew, et al. Get Through Final FRCR 2A: SBAs. 1st ed., CRC Press, 2017. Explanation: Imaging Modalities for Uterine Anomalies • While the presence of a divided rather than triangular uterine cavity at Hysterosalpingogram (HSG) may suggest the presence of a Mullerian duct anomaly (MDA), it is not possible to differentiate between subtypes. • MRI and US provide greater anatomic detail; both of these imaging methods provide information on the external uterine contour, which is an important diagnostic feature of MDAs. • Furthermore, both MRI and US may be used to assess for concomitant renal anomalies; renal anomalies occur at a higher rate among MDA patients. Unicornuate Uterus • Unicornuate uterus appears as a small, oblong, off-midline structure on US and MRI. Uterus Didelphys • Uterus didelphys results from complete failure of Müllerian duct fusion. • Each duct develops fully with duplication of the uterine horns, cervix and proximal vagina. • A fundal cleft greater than 1 cm has been reported to be 100% sensitive and specific in differentiation of fusion anomalies (didelphys and bicornuate) from reabsorption anomalies (septate and arcuate). Bicornuate Uterus • Bicornuate uterus involves duplication of the uterus with possible duplication of the cervix (bicornuate unicollis or bicornuate bicollis). • HSG demonstrates opacification of two symmetric fusiform uterine cavities (horns) and fallopian tubes. • Historically, an intercornual angle of greater than 105° was used for diagnosis of bicornuate uterus. Septate Uterus • Septate uterus is the most common form of MDA, accounting for approximately 55% of cases. • Historically, an angle of less than 75° between the uterine horns has been reported to be suggestive of a septate rather than bicornuate uterus. • However, considerable overlap occurs between septate and bicornuate uteri; as such, the angle measurement is not a reliable diagnostic feature. Arcuate Uterus
  • 373.
    EBDR Exam MCQs& Concepts October 2021 Dr. Kareem Alnakeeb 367 • Arcuate uterus at HSG shows a single uterine cavity with a broad saddle-shaped indentation at the uterine fundus. 90. An obese 25-year-old man presents with atypical chest pain. Cardiac MR demonstrates asymmetrical hypertrophy of the interventricular septum, primarily affecting the anteroinferior portion. What is the most likely diagnosis? A. Hypertrophic obstructive cardiomyopathy B. Restrictive cardiomyopathy C. Myocardial infarction D. Dilated cardiomyopathy E. Constrictive pericarditis Source: Chin, Teck Yew, et al. Get Through Final FRCR 2A: SBAs. 1st ed., CRC Press, 2017. Explanation: Definition • Hypertrophic cardiomyopathy (HCM) is defined as a diffuse or segmental left-ventricular hypertrophy with a non-dilated and hyperdynamic chamber, in the absence of another cardiac or systemic disease explaining the degree of cardiac muscle hypertrophy. Functional Hallmarks and Symptoms • Dyspnea on exertion is the most common symptom because the key functional hallmark of hypertrophic cardiomyopathy is an impaired diastolic function with impaired LV filling in the presence of preserved systolic function. • Systolic dysfunction occurs at end-stage disease. Morphological Variants • Asymmetric involvement of the interventricular septum is the most common form of the disease, accounting for an estimated 60%–70% of the cases of HCM. • Other variants include apical, symmetric, midventricular, mass-like and non-contiguous HCM is typically associated with hypertrophy of the muscle to 15 mm or thicker and a ratio of thickened myocardium to normal left-ventricular basal myocardium of 1.3–1.5. Imaging Characteristics • With MRI and multidetector computed tomography (CT), apical HCM has a characteristic spade-like configuration of the LV cavity at end diastole, appreciated on vertical long-axis views. 91. A 45-year-old woman presents with a rapidly enlarging mildly painful breast mass over a period of few months. An urgent ultrasound is performed. The ultrasound shows that the mass measures 7 cm, filling up almost the entire breast with fluid-filled clefts in the tumor. What is the diagnosis? A. Inflammatory carcinoma B. Cystosarcoma phylloides C. Complex breast cyst D. Invasive lobular carcinoma E. Breast lymphoma Source: Chin, Teck Yew, et al. Get Through Final FRCR 2A: SBAs. 1st ed., CRC Press, 2017. Explanation: Overview of Phylloides Tumour (PT) • Phylloides tumour (PT) is a rare breast fibroepithelial neoplasm. • It is now generally accepted that PTs can be classified as benign, borderline or malignant. Imaging Features and Diagnosis Challenges
  • 374.
    EBDR Exam MCQs& Concepts October 2021 Dr. Kareem Alnakeeb 368 • Mammography and ultrasound are notorious for their inability to distinguish the benign or malignant histologic nature of PTs. • On US, they can be indistinguishable from fibroadenoma. • They appear as an inhomogeneous, solid-appearing mass. • A solid mass containing single or multiple, round or cleft-like cystic spaces and demonstrating posterior acoustic enhancement strongly suggests a diagnosis of PTs. • Solid components of the tumour show vascularity on Doppler. MRI Characteristics of PTs • On MRI, well-defined margins with a round or lobulated shape and a septate inner structure have been described as characteristic morphologic signs. • They are usually low on T1-weighted images and vary from low to very high signal on T2-weighted images. • Some have described a slit-like pattern on MRIs of benign PTs; these appear as hyperintense slit-like fluid-filled spaces on T2-weighted images, with a low signal after enhancement. • Solid areas of the tumor show enhancement with contrast. 92. A fit and healthy 25-year-old woman presents to the breast clinic with a small mobile non-tender breast lump that she noticed incidentally. An ultrasound is deemed as the first-line investigation; it reveals an extremely well-defined homogenous, hypoechoic oval mass with posterior acoustic shadowing. What is the most likely diagnosis? A. Cystosarcoma phylloides B. Fibroadenoma C. Complex breast cyst D. Invasive lobular carcinoma E. Fat necrosis Source: Chin, Teck Yew, et al. Get Through Final FRCR 2A: SBAs. 1st ed., CRC Press, 2017. Explanation: Characteristics of Fibroadenomas • Fibroadenomas are the most common cause of benign solid mass in the breast. • On US, they appear round or oval, wider than tall, hypoechoic, well-defined and mostly homogeneous and show a ‘hump and dip’ sign (small focal bulge to the contour with a contiguous small sulcus), a thin echogenic pseudocapsule and rarely either posterior acoustic enhancement (17%–25%) or posterior acoustic shadow (9%–11%). 93. A 30-year-old, nulliparous woman with Stein–Leventhal syndrome is being treated for subfertility with clomiphene. She develops abdominal pain, distension, nausea and vomiting. Ultrasound examination of the abdomen reveals both ovaries to be larger than 7 cm in length and packed with large follicles, and also reveals an ovarian cyst 12 cm in diameter. Ascites and a pleural effusion are also seen. What is the most likely diagnosis? A. Endometriosis B. Ovarian Cyst Torsion C. Ovarian Hyperstimulation Syndrome D. Ovarian Serous Cystadenoma E. Corpus Luteum Of Menstruation
  • 375.
    EBDR Exam MCQs& Concepts October 2021 Dr. Kareem Alnakeeb 369 Source: Bydder, Megan, et al. Get Through Final FRCR Part A: SBAs for the Modular Examination. 1st ed., Hodder Education Publishers, 2009. Explanation: • Ovarian hyperstimulation syndrome is more commonly seen with human menopausal gonadotrophin therapy but can also be seen with clomiphene. • Severe complications relate to volume depletion, such as hypovolaemia, oliguria, electrolyte imbalance and thromboembolic events. Intra-abdominal haemorrhage is also reported. 94. All of the following statements are true with regard to stress fractures in young athletic children, except A. Children with limb misalignment are at greater risk. B. Stress fracture of the femoral neck involves the superior surface. C. Shin splints show linear oedema limited to the medial tibia. D. Distal femoral metaphysis are recognised sites for stress fracture. E. Pars interarticularis fractures are due to repetitive extension and torsion. Source: Chin, Teck Yew, et al. Get Through Final FRCR 2A: SBAs. 1st ed., CRC Press, 2017. Explanation: Causes and Risk Factors of Stress Fractures in Children • The most common cause of stress fractures is a chronic and repeated workload. • Children with extremity malalignment or abnormal weight-bearing also are at increased risk. Common Locations of Stress Fractures in Children • Typical locations of stress fractures in children include the tibia, fibula, femur and tarsal and metatarsal bones. Diagnostic Modality • MR imaging is currently the best diagnostic modality for stress fractures. Characteristics of Femur Stress Fractures • Stress fractures of the femur tend to occur after skeletal maturity and resemble adult injuries, affecting the inferior surface of the neck, the shaft and the distal metaphysis. • This injury is most common in endurance athletes, such as runners, triathletes or soccer players but also occurs in association with abnormal weight-bearing, such as with a coxa vara deformity. Tibial Stress Fractures in Adolescent Athletes • The most common site for stress fractures in the adolescent athlete is the tibia. • Tibial stress fractures occur with activities requiring sudden stops or changes in direction, such as football, soccer and tennis. Stress Response and Shin Splints • Stress also can result in shin splints, which are probably an early stress response secondary to periosteal traction. • In cases of stress fractures, MR imaging shows diffuse and irregular bone marrow oedema, whereas in shin splints, the area of high signal intensity often is more linear and is limited to the medial aspect of the tibia. Spondylolysis as a Stress Injury • Spondylolysis is a stress injury of the pars interarticularis that is due to repetitive extension and torsion of the trunk. • Usually occurring in the lower segments of the lumbar spine, stress injuries of the pars interarticularis have been observed in young female gymnasts, college football players and wrestlers.
  • 376.
    EBDR Exam MCQs& Concepts May 2021 Dr. Kareem Alnakeeb 370 May 2021 Paper 1 1. In mandibular trauma, which fracture site most commonly results in inferior alveolar nerve injury? A. Angle of the mandible B. Condylar neck C. Symphysis menti D. Coronoid process E. Body of the mandible Explanation: (by Perplexity AI, generated with the OpenAI O3 model) • The inferior alveolar nerve runs within the mandibular canal from the mandibular foramen to the mental foramen. Fractures at the mandibular angle frequently disrupt this canal, making nerve injury common. • Condylar neck and coronoid fractures lie superior to the canal, so neuropathy is rare. • Symphysis fractures involve the midline and typically spare the canal, while body fractures posterior to the mental foramen are less common sites of nerve damage compared with angle breaks. 2. Which of the following cerebral veins belongs to the deep venous drainage system of the brain? A. Superficial Sylvian (middle cerebral) vein B. Vein of Labbe C. Vein of Trolard D. Internal cerebral veins E. Superior sagittal sinus Explanation: (by Perplexity AI, generated with the OpenAI O3 model) • The two internal cerebral veins run posteriorly beneath the splenium of the corpus callosum and unite to form the vein of Galen; they drain the thalami, basal ganglia, internal capsule, and deep white matter, making them key components of the deep venous system. • In contrast, the superficial Sylvian vein, vein of Labbe (inferior anastomotic vein), and vein of Trolard (superior anastomotic vein) are all part of the superficial cortical drainage and empty into dural sinuses rather than the deep system. • The superior sagittal sinus is a dural venous sinus, not a cerebral vein; it receives blood from superficial cortical veins but is not itself classified within either the superficial or deep cerebral venous groups 3. In advanced rheumatoid arthritis of the cervical spine, which of the following complications is most characteristically seen? A. Abscess B. Osteophytes C. Atlanto-axial subluxation D. Calcification E. Spinal cord cavernoma Explanation: (by Perplexity AI, generated with the OpenAI O3 model) • Chronic pannus formation and ligamentous laxity in longstanding rheumatoid arthritis preferentially weaken the transverse ligament at the atlanto-axial joint, leading to anterior atlanto-axial subluxation that may compress the upper cervical cord.
  • 377.
    EBDR Exam MCQs& Concepts May 2021 Dr. Kareem Alnakeeb 371 • Suppurative abscesses (A) are not a typical direct consequence of rheumatoid disease. • Osteophyte formation (B) is more typical of osteoarthritis than rheumatoid arthritis. • Diffuse ligamentous calcification (D) is unrelated to inflammatory pannus. • Cavernomas (E) are vascular malformations unlinked to rheumatoid pathology. 4. In subacute combined degeneration of the spinal cord due to vitamin B12 deficiency, which spinal tract classically undergoes the earliest and most pronounced degenerative change? A. Lateral corticospinal tract B. Dorsal (posterior) columns C. Anterior horn cells D. Posterior horn cells E. Spinothalamic tracts Explanation: (by Perplexity AI, generated with the OpenAI O3 model) • Subacute combined degeneration characteristically produces symmetric demyelination of the dorsal columns—especially the gracile and cuneate fasciculi—resulting in loss of proprioception and vibration sense. • As the disease progresses, degeneration also affects the lateral corticospinal tracts, leading to spastic paresis, but dorsal column involvement dominates early and is most severe. • Anterior and posterior horn cells are typically spared, and spinothalamic tracts remain largely intact, so options C, D and E are incorrect. • Option A describes a tract affected later rather than primarily. 5. Evaluating the four statements about the prestyloid parapharyngeal space (PPS) A. T1-weighted MRI without contrast is best to see fat invasion in the prestyloid PPS. B. The most common lesion in the prestyloid PPS is a schwannoma. C. Post-styloid compartment equals the alar (carotid) space D. The prestyloid compartment contains branches of the vagus nerve Explanation: (by Perplexity AI, generated with the OpenAI O3 model) • Normal prestyloid PPS is largely fat; on non-contrast T1 it shows high signal that outlines masses. Loss or streaking of this bright fat is the earliest sign of tumor or infection, so radiologists routinely review plain T1 images first for fat obliteration • Most prestyloid tumors arise from salivary tissue (deep-lobe parotid or ectopic minor salivary glands). Pleomorphic adenoma is the single most frequent lesion, accounting for 40-50% of all PPS tumors and 80-90% of salivary tumors in the space. Schwannomas dominate the poststyloid (carotid) compartment, not the prestyloid. • The tensor-vascular-styloid fascia behind the styloid process separates the ‘true’ prestyloid PPS from the posterior carotid (alar/poststyloid) space that contains the internal carotid artery, internal jugular vein, cranial nerves IX-XII and sympathetic chain. Radiology texts often refer to the poststyloid PPS simply as the carotid space. • Vagus nerve and its branches travel inside the carotid sheath of the poststyloid compartment. The prestyloid space contains mainly fat, salivary tissue, small vessels, and minor branches of the mandibular division of the trigeminal nerve—not vagal branches
  • 378.
    EBDR Exam MCQs& Concepts May 2021 Dr. Kareem Alnakeeb 372 6. During endoscopic sinus surgery, which anatomical dehiscence is most frequently implicated when severe haemorrhage occurs due to direct arterial injury? A. Lamina papyracea with injury to the olfactory bulb B. Sphenoid sinus wall with injury to the intracavernous internal carotid artery C. Cribriform plate with injury to the optic nerve D. Retained Haller cells causing obstruction of frontal sinus drainage E. Posterior maxillary wall with injury to the sphenopalatine artery Explanation: (by Perplexity AI, generated with the OpenAI O3 model) • The lateral wall of the sphenoid sinus may be thin or dehiscent, placing the adjacent intracavernous segment of the internal carotid artery at particular risk during functional endoscopic sinus surgery (FESS). • Penetration here can lead to catastrophic haemorrhage, making this the most significant dehiscence-related complication. Lamina papyracea breaches mainly threaten the orbit and medial rectus, not major arteries; cribriform plate injury causes CSF leak rather than arterial bleeding; retained Haller cells obstruct drainage but are not a bleeding source; the sphenopalatine artery runs in the posterior maxillary wall but is smaller and usually controlled endoscopically. 7. In a postpartum woman with a new sellar mass, MRI shows symmetrical, homogeneously enhancing enlargement of the pituitary gland and thickened stalk; which statement best characterises lymphocytic hypophysitis? A. It is easily distinguished from a non-functioning pituitary adenoma on routine MRI. B. It is most often triggered by direct cranial trauma. C. It is commonly induced by standard medications unrelated to immunotherapy. D. It represents an inflamed gland in which the normal anterior pituitary tissue is present but compressed. E. It typically spares the infundibulum. Explanation: (by Perplexity AI, generated with the OpenAI O3 model) • Lymphocytic hypophysitis is an autoimmune inflammation of the gland; lymphocytic infiltrates enlarge the pituitary but do not destroy all parenchyma, so normal tissue is usually present and merely compressed (hence option D is correct). • Imaging overlap with non-functioning adenoma is considerable—multiple studies stress that routine MRI cannot reliably differentiate the two (so A is false). • Reported aetiologies include pregnancy-related autoimmunity and immune-checkpoint inhibitors; blunt head trauma is not recognised as a typical cause (B is false), and ordinary medications rarely precipitate it outside of immunotherapy agents (C is false). • Stalk thickening is a hallmark finding, meaning it is frequently involved rather than spared (E is false). 8. In patients with an aberrant internal carotid artery (ICA) coursing through the middle ear cavity, which persistent fetal carotid–basilar anastomosis is most frequently associated? A. Persistent hypoglossal artery B. Persistent otic artery C. Persistent stapedial artery D. Persistent trigeminal artery E. Persistent dorsal ophthalmic artery
  • 379.
    EBDR Exam MCQs& Concepts May 2021 Dr. Kareem Alnakeeb 373 Explanation: (by Perplexity AI, generated with the OpenAI O3 model) • An aberrant ICA in the temporal bone represents a collateral pathway that substitutes for an absent or hypoplastic cervical ICA. • It is most commonly accompanied by a persistent trigeminal artery, the commonest of the carotid- basilar embryonic connections, because both lesions reflect arrested regression of early vascular channels within the petrous apex. • The persistent hypoglossal and otic arteries are far rarer embryonic channels, and the stapedial artery, although it traverses the middle ear, typically involutes without relation to an aberrant ICA. • The dorsal ophthalmic artery is an early orbital branch that does not course near the petrous carotid canal and is not linked to aberrant ICA formation. 9. For women with adequately treated ductal carcinoma in situ (DCIS), what is the approximate breast- cancer–specific survival at 20 years? A. 20% B. 50% C. 90% D. 95% E. 100% Explanation: (by Perplexity AI, generated with the OpenAI O3 model) • Large population-based studies tracking >100,000 women show that, after surgery with or without radiotherapy, DCIS carries an excellent prognosis: breast-cancer–specific mortality remains about 3– 5% at 20 years, equating to roughly 95% survival. • Option D reflects this figure, making it correct. • Option C underestimates survival, while A and B dramatically underestimate it, conflicting with published long-term data. • Complete (100%) survival (option E) is unrealistic because a small risk of invasive recurrence or metastasis persists despite optimal management. 10. A 46-year-old woman’s screening mammogram shows new grouped micro-calcifications in the upper outer quadrant of the right breast; targeted ultrasound is normal. Six-month mammographic surveillance demonstrates the calcifications are unchanged in morphology and extent, with no associated mass. What is the most appropriate next step in management? A. Routine annual screening mammography B. Surgical excision of the calcified area C. Digital breast tomosynthesis only D. Stereotactic-guided vacuum-assisted biopsy E. Short-term (6-month) mammographic follow-up Explanation: (by Perplexity AI, generated with the OpenAI O3 model) • Persistently grouped micro-calcifications that are new at baseline and remain stable after a short- interval follow-up still carry a clinically significant risk of ductal carcinoma in situ; BI-RADS recommends moving from Category 3 (probably benign) surveillance to tissue diagnosis if stability is confirmed but suspicion persists. • Stereotactic-guided vacuum-assisted biopsy obtains representative samples with minimal morbidity and is therefore the investigation of choice. Routine annual screening (A) overlooks the need for definitive diagnosis. • Surgical excision (B) is more invasive than necessary when percutaneous biopsy suffices.
  • 380.
    EBDR Exam MCQs& Concepts May 2021 Dr. Kareem Alnakeeb 374 • Tomosynthesis alone (C) improves lesion conspicuity but does not provide a tissue diagnosis. • Continued short-term follow-up (E) delays diagnosis without reducing cancer risk. 11. On screening mammography, which feature is considered the classic hallmark of ductal carcinoma in situ (DCIS)? A. Microcalcifications B. A well-defined solid mass C. Spontaneous bloody nipple discharge D. Global breast asymmetry E. Axillary lymph-node enlargement Explanation: (by Perplexity AI, generated with the OpenAI O3 model) • DCIS most often manifests as clusters of fine linear or pleomorphic microcalcifications because malignant ductal cells produce necrotic debris that calcifies; this pattern alerts radiologists to non- invasive disease before a palpable mass forms. • Solid masses (option B) are more typical of invasive carcinomas or benign lesions such as fibroadenomas. • Nipple discharge (option C) is a clinical symptom that can arise from multiple intraductal processes and is not pathognomonic on imaging. • Global asymmetry (option D) is a nonspecific finding seen with many benign conditions, while axillary nodal enlargement (option E) is uncommon in pure DCIS since the disease is non-invasive and lacks metastatic potential. 12. A 37-year-old woman presents with spontaneous unilateral bloody nipple discharge. What is the most common underlying cause? A. Ductal carcinoma in situ B. Intraductal papilloma C. Atypical ductal hyperplasia D. Duct ectasia E. Invasive ductal carcinoma Explanation: (by Perplexity AI, generated with the OpenAI O3 model) • Intraductal papilloma is the single most frequent lesion causing spontaneous bloody nipple discharge in women; the vascular stalk of the papilloma is prone to bleeding within a dilated duct. • Ductal carcinoma in situ and invasive carcinoma are important differentials but together account for fewer cases than papilloma in this presentation. • Duct ectasia typically produces multicoloured or green discharge rather than frank blood, while atypical ductal hyperplasia is generally an incidental microscopic finding and rarely manifests with discharge. 13. Regarding breast implant rupture on imaging, which statement is FALSE? A. Peri-implant fluid can suggest extracapsular silicone spread B. Subcapsular fluid producing the “subcapsular rim sign” indicates intracapsular rupture C. The“inverted key-hole sign” is typical of intact implants D. The “salad-oil sign” on ultrasound represents intracapsular rupture E. A collapsed envelope with “linguine sign” on MRI confirms intracapsular rupture
  • 381.
    EBDR Exam MCQs& Concepts May 2021 Dr. Kareem Alnakeeb 375 Explanation: (by Perplexity AI, generated with the OpenAI O3 model) • Intracapsular rupture traps silicone between the implant shell and fibrous capsule, giving appearances such as the subcapsular rim (option B), salad-oil or echo-free fluid–silicone layers on ultrasound (option D), and the classic collapsed shell or “linguine sign” on MRI (option E). • Peri-implant fluid outside the capsule (option A) suggests extracapsular leakage but may accompany rupture. • The “inverted key-hole sign” describes silicone tracking between shell folds after rupture, not an intact prosthesis, so option C is false. 14. In a woman diagnosed with breast carcinoma during pregnancy, when is adjuvant radiotherapy ideally delivered to minimise fetal radiation exposure? A. During the first trimester B. After the first trimester C. After the second trimester D. Post-delivery E. At any gestational age with abdominal shielding Explanation: (by Perplexity AI, generated with the OpenAI O3 model) • Radiotherapy to the breast imparts scattered dose to the uterus that can exceed teratogenic thresholds, especially in early gestation. • Because fetal shielding cannot reduce internal scatter from the maternal thorax to safe levels, standard guidance is to defer breast or chest-wall irradiation until after childbirth, scheduling surgery and systemic therapy antenatally instead. • Administering radiotherapy in the first or second trimester risks organogenesis damage or neurocognitive deficits, while third-trimester exposure still threatens growth and haematopoiesis. • Hence options A–C are contraindicated. Option E is incorrect because shielding alone is insufficient for thoracic fields. 15. Regarding male breast carcinoma, which of the following statements is INCORRECT? A. Tumours are typically oestrogen-receptor negative B. BRCA2 mutation markedly increases risk C. A family history of breast cancer is a recognised risk factor D. A family history of prostate cancer is a recognised risk factor E. Klinefelter syndrome confers an increased risk Explanation: (by Perplexity AI, generated with the OpenAI O3 model) • Most male breast cancers express oestrogen receptors; ER positivity exceeds 80%, guiding the use of endocrine therapy, so option A is false. • BRCA2 germ-line mutations (option B) confer a substantially elevated lifetime risk. • First-degree relatives with breast cancer (option C) double to triple the risk through shared genetics. • Familial clustering with prostate cancer (option D) reflects overlapping BRCA2 and other DNA-repair gene mutations, so it is a recognised association. Klinefelter syndrome (option E) raises risk 10-20- fold due to oestrogen excess.
  • 382.
    EBDR Exam MCQs& Concepts May 2021 Dr. Kareem Alnakeeb 376 16. A 40-year-old woman presents with a retro-areolar, ill-defined 8 cm hypoechoic breast mass containing irregular tubular tracts, with surrounding skin inflammation and discharging sinuses on clinical examination. What is the most appropriate next management step? A. Ultrasound-guided aspiration and core biopsy B. Dynamic contrast-enhanced breast MRI C. Commence empirical systemic antibiotics and steroids D. Wide local surgical excision E. Immediate mastectomy Explanation: (by Perplexity AI, generated with the OpenAI O3 model) • The constellation of a large, poorly defined hypoechoic mass with sinus-tract formation in a young woman is most suggestive of granulomatous mastitis. • First-line management is image-guided aspiration and core biopsy to confirm the diagnosis and exclude malignancy or specific infection such as tuberculosis. • MRI (B) is useful for extension mapping but should follow histological confirmation. • Empirical medical therapy (C) without tissue diagnosis risks inappropriate treatment. • Surgical excision (D) or mastectomy (E) is reserved for refractory or complicated cases after biopsy- proven diagnosis; performing surgery up-front carries unnecessary morbidity. 17. In which scenario does the incidence of a metachronous contralateral breast cancer NOT significantly rise after treatment of a first breast tumour? A. Positive first-degree family history of breast cancer B. Multicentric index tumour in the treated breast C. Invasive ductal carcinoma as the index tumour D. Invasive lobular carcinoma as the index tumour E. BRCA1/2 germline mutation carrier status Explanation: (by Perplexity AI, generated with the OpenAI O3 model) • Risk rises notably when there is a strong family history, a multicentric/multifocal index lesion, lobular histology or an underlying high-penetrance mutation such as BRCA1/2. • Multiple studies show that invasive lobular carcinoma confers a higher contralateral risk than ductal carcinoma because of its bilateral propensity, while multicentricity implies a field change affecting both breasts. • Family history and BRCA mutations also double to triple the risk. • By contrast, invasive ductal carcinoma without the above factors behaves as the reference baseline; its presence alone does not appreciably increase contralateral cancer incidence, so option C is the exception. 18. In a patient with invasive ductal carcinoma of the breast, which imaging modality is most sensitive for detecting an extensive intraductal component before surgery? A. Mammography B. Ultrasound C. Magnetic resonance imaging (MRI) D. PET-CT E. Digital breast tomosynthesis Explanation: (by Perplexity AI, generated with the OpenAI O3 model) • MRI is the most sensitive test for mapping the true intraductal spread of invasive ductal carcinoma because gadolinium-enhanced sequences highlight both the enhancing invasive focus and contiguous non-mass ductal enhancement typical of extensive intraductal components.
  • 383.
    EBDR Exam MCQs& Concepts May 2021 Dr. Kareem Alnakeeb 377 • Mammography may miss non-calcified ductal disease, and ultrasound is limited to mass-like areas, often under-estimating intraductal extension. • PET-CT lacks spatial resolution for small intraductal foci, and tomosynthesis improves detection of architectural distortion but is still less sensitive than MRI for non-calcified ductal spread. 19. After neoadjuvant chemotherapy for a breast carcinoma, which imaging modality is most sensitive for assessing tumour response? A. Breast ultrasound B. Mammography C. Dynamic contrast-enhanced breast MRI D. 18 F-FDG PET-CT E. Digital breast tomosynthesis Explanation: (by Perplexity AI, generated with the OpenAI O3 model) • Dynamic contrast-enhanced breast MRI detects early vascular and cellular changes, allowing the most accurate measurement of residual tumour size and extent after neoadjuvant therapy, with pooled sensitivities exceeding 90%. • PET-CT identifies metabolic response but has lower spatial resolution and variable sensitivity for small residual lesions. • Ultrasound is operator-dependent and less reliable for multifocal disease. • Mammography and tomosynthesis primarily show structural change (e.g. calcification or density reduction) and underestimate residual viable tumour, giving lower sensitivities than MRI. 20. For which breast cancer scenario is neoadjuvant chemotherapy most appropriately recommended? A. Small, clearly operable T1 lesion B. Locally advanced but non-metastatic, initially inoperable carcinoma C. Widespread metastatic disease at presentation D. Solitary mass measuring exactly 2 cm without nodal involvement E. Screen-detected microcalcifications without palpable mass Explanation: (by Perplexity AI, generated with the OpenAI O3 model) • Neoadjuvant chemotherapy is primarily indicated when the tumour is too large or locally advanced to permit immediate surgery but potentially resectable after downsizing; giving systemic therapy first can convert an inoperable cancer to operable status and allows early treatment of micrometastases. • Option B describes this classic setting. • Option A already meets criteria for upfront surgery; neoadjuvant therapy offers no advantage. • Option C requires systemic therapy for palliation, not for surgical conversion. • Option D is resectable at diagnosis (T2 ≤5 cm with no adverse factors), so surgery first is preferred. • Option E represents ductal carcinoma in situ or early invasive disease detected on screening, also managed surgically before considering adjuvant therapy. 21. In stereotactic-guided core biopsy for a cluster of suspicious breast microcalcifications, which practice optimises diagnostic accuracy? A. Obtaining at least 10 core samples B. Approaching the lesion craniocaudally in every case C. Sampling only the radiographic centre of the calcification cluster D. Relying on a single 14-gauge true-cut core E. Using vacuum aspiration rather than core needles
  • 384.
    EBDR Exam MCQs& Concepts May 2021 Dr. Kareem Alnakeeb 378 Explanation: (by Perplexity AI, generated with the OpenAI O3 model) • Accuracy rises sharply when ≥10 cores are taken with a 14-gauge needle, giving >95% retrieval of microcalcifications and reducing underestimation of ductal carcinoma in situ. • A craniocaudal approach (B) is chosen when the lesion is best accessed that way, but lateral decubitus or caudocranial paths are equally acceptable depending on location. • Limiting sampling to the cluster centre (C) risks missing adjacent DCIS. • A single true-cut core (D) lacks sufficient tissue and has high false-negative rates. • Vacuum devices (E) improve yield but the question specifies core biopsy; when using standard core needles, increasing core number is the key modifiable factor. 22. In breast cancer surgery, which of the following is NOT an accepted indication for proceeding to formal axillary lymph-node dissection? A. Suspicious lymph nodes on pre-operative axillary ultrasound B. Negative intra-operative sentinel lymph-node biopsy C. Clinically palpable, hard axillary lymph nodes on examination D. Proven metastatic involvement on needle biopsy of an axillary node E. Positive sentinel lymph-node biopsy showing macrometastasis Explanation: (by Perplexity AI, generated with the OpenAI O3 model) • Axillary dissection is generally reserved for patients with proven or highly suspected nodal disease. • It is routinely performed when axillary nodes are clinically abnormal, imaging suggests malignancy, or needle biopsy confirms metastasis (options A, C, D) and when the sentinel node contains macrometastasis (option E). • Conversely, a negative sentinel lymph-node biopsy indicates absence of nodal spread, so completion axillary dissection is unnecessary; patients instead continue with breast-conserving surgery or mastectomy plus radiotherapy as appropriate. 23. A 65-year-old man with a 30-pack-year smoking history undergoes chest CT that shows a 5.5 cm right upper-lobe mass extending into the visceral pleura with an enlarged ipsilateral hilar lymph node. What is the TNM stage of this lung cancer? A. T1 N1 M0 B. T2a N1 M0 C. T2b N1 M0 D. T3 N1 M0 E. T4 N1 M0 Explanation: (by Perplexity AI, generated with the OpenAI O3 model) • Primary tumour: size >5 cm but ≤7 cm and direct invasion of visceral pleura both qualify as T3 under the 8th edition TNM classification. • Nodal status: an ipsilateral hilar node is N1. • No distant metastases are described, so M0. Hence the correct stage is T3 N1 M0. • Key distractors: o T2a and T2b are limited to tumours ≤5 cm (T2a) or >5–7 cm without pleural invasion; therefore options B and C are undersized. o T1 applies only to lesions ≤3 cm (option A). o T4 (option E) requires involvement of mediastinal organs, vertebral body, or separate tumour nodule in a different lobe, none of which are present.
  • 385.
    EBDR Exam MCQs& Concepts May 2021 Dr. Kareem Alnakeeb 379 24. Which of the following is NOT a recognised acquired cause of pulmonary vein stenosis in adults? A. Mediastinal neoplastic infiltration B. Blunt or penetrating thoracic trauma C. Radio-frequency catheter ablation for atrial fibrillation D. Fibrosing mediastinitis E. Sarcoidosis Explanation: (by Perplexity AI, generated with the OpenAI O3 model) • Radio-frequency catheter ablation is currently the commonest iatrogenic cause of acquired pulmonary vein stenosis, and mediastinal conditions such as fibrosing mediastinitis, sarcoidosis and neoplastic infiltration/compression are well-documented non-iatrogenic causes. • Direct injury from external chest trauma, however, is not described as a mechanism for progressive pulmonary vein lumen narrowing; trauma may lacerate a vein but does not lead to chronic stenosis, making option B the incorrect statement. 25. Which of the following is NOT a typical imaging feature of a cerebral hydatid cyst? A. A clear cyst with no septations or mural enhancement B. A cyst wall that is calcified C. Absence of significant surrounding edema D. Presence of multiple intracranial cysts E. Fluid signal identical to cerebrospinal fluid on all MRI sequences Explanation: (by Perplexity AI, generated with the OpenAI O3 model) • Cerebral hydatid cysts are classically single, thin-walled, spherical lesions whose contents mirror cerebrospinal fluid on CT/MRI and show no internal septations, mural nodules, or post-contrast enhancement. • They usually produce little or no perilesional edema unless infected or ruptured. • Calcification of the cyst wall is rare in active lesions and generally indicates an involuted or previously ruptured/treated cyst, making “calcified wall”the feature that does not belong to the standard criteria. • Multiple cysts (secondary hydatidosis) can occur after rupture or hematogenous spread but remain an accepted presentation of the disease; therefore, option D is not the best choice for “except.” 26. Which of the following best describes the key skeletal abnormality in caudal regression syndrome seen on radiographs? A. Partial or complete absence of the sacrum B. Vertebral agenesis above T12, usually incompatible with life C. Hemivertebrae throughout the thoracic spine D. Posterior element fusion of cervical vertebrae E. Lytic destruction of lumbar pedicles Explanation: (by Perplexity AI, generated with the OpenAI O3 model) • Caudal regression syndrome is a spectrum of congenital anomalies characterized primarily by partial or complete absence of the sacrum and, variably, lower lumbar vertebrae. • This deficiency underlies the classic clinical features such as lower-limb deformities and neurogenic bladder. • Vertebral agenesis above T12 is not typical; involvement that high is exceedingly rare and most affected fetuses are non-viable, making option B incorrect. • Hemivertebrae (option C) are more suggestive of congenital scoliosis.
  • 386.
    EBDR Exam MCQs& Concepts May 2021 Dr. Kareem Alnakeeb 380 • Posterior element fusion of cervical vertebrae (option D) describes Klippel–Feil syndrome. • Lytic destruction of lumbar pedicles (option E) is characteristic of metastatic or infectious processes, not a congenital malformation. 27. A 4-year-old child presents with chronic central cyanosis and marked pulmonary plethora on chest radiograph; which congenital cardiac lesion best explains this combination? A. Tetralogy of Fallot B. Patent ductus arteriosus C. Total anomalous pulmonary venous return (TAPVR) D. Pulmonary valve stenosis E. Transposition of the great arteries Explanation: (by Perplexity AI, generated with the OpenAI O3 model) • Total anomalous pulmonary venous return produces right-to-left mixing at atrial level, causing cyanosis, while the anomalous pulmonary veins drain into the systemic venous circulation, leading to a large left-to-right shunt and pulmonary over-circulation that gives a plethoric (hypervascular) lung pattern on imaging. • Tetralogy of Fallot and pulmonary stenosis both show cyanosis but have reduced pulmonary blood flow, so lungs appear oligaemic. • Patent ductus arteriosus causes pulmonary plethora but is not typically cyanotic unless Eisenmenger reversal develops. • Transposition of the great arteries is cyanotic yet usually shows normal or slightly decreased pulmonary vascularity unless a large VSD or PDA is present. 28. A 6-year-old girl presents with headache, neck pain and vomiting. MRI shows a cystic lesion in the posterior fossa with an enhancing wall and a homogeneously, intensely enhancing mural nodule. Which is the most likely diagnosis? A. Arachnoid cyst B. Ependymoma C. Hemangioblastoma D. Medulloblastoma E. Pilocytic astrocytoma Explanation: (by Perplexity AI, generated with the OpenAI O3 model) • Pilocytic astrocytoma is the commonest paediatric posterior fossa tumour after medulloblastoma. It classically appears as a cyst with a vividly enhancing mural nodule; the cyst wall may also enhance. • Hemangioblastoma can look similar but is rare in children and usually occurs in adults with von Hippel–Lindau disease. • Medulloblastomas are typically solid, midline masses that may show heterogeneous rather than focal mural enhancement. • Ependymomas often arise from the floor of the fourth ventricle and show mixed solid-cystic appearance with calcification. • Arachnoid cysts follow CSF signal on all sequences and do not enhance, so an enhancing nodule would not be seen.
  • 387.
    EBDR Exam MCQs& Concepts May 2021 Dr. Kareem Alnakeeb 381 29. A 6-year-old child presents with a 9 cm abdominal mass seen on ultrasound. Which of the following findings would favour Wilms tumour over neuroblastoma? A. Punctate coarse calcifications within the mass B. Multiple pulmonary metastases on chest radiograph C. Osteolytic lesions in the femora D. The mass displaces, rather than encases, the ipsilateral kidney E. Involvement of the posterior mediastinum on staging MRI Explanation: (by Perplexity AI, generated with the OpenAI O3 model) • Wilms tumours typically arise from renal parenchyma and therefore push and displace adjacent renal tissue and vessels, often forming a pseudocapsule; they rarely encase structures. • In contrast, neuroblastomas originate from sympathetic tissue, tend to wrap around and encase the kidney and major vessels. • Calcification is common in neuroblastoma but far less frequent in Wilms, so option A disfavors Wilms. • Pulmonary metastases (option B) occur in both, though more often sought in Wilms staging, so this is not a distinguishing feature. • Bone metastases (option C) and posterior mediastinal spread (option E) are characteristic of neuroblastoma rather than Wilms. 30. Which of the following statements about long-segment Hirschsprung disease is INCORRECT? A. Skip (segmental) aganglionosis is a recognised feature B. It shows a marked male predominance C. It is occasionally associated with neuroblastoma D. A contrast enema typically demonstrates a reversed rectosigmoid ratio E. Aganglionosis always begins in the rectum and extends proximally Explanation: (by Perplexity AI, generated with the OpenAI O3 model) • Long-segment Hirschsprung disease still begins in the rectum and extends proximally, giving the classic reversed rectosigmoid ratio on contrast enema because the rectum is narrower than the dilated sigmoid (Option D true). • Like all forms of Hirschsprung disease, it is more common in males (≈4:1) (Option B true) and, as a neurocristopathy, it can coexist with neuroblastoma or other neural-crest tumours (Option C true). • Aganglionosis is continuous from the distal rectum; true skip lesions are extremely rare and not a typical feature, so Option A is the exception. • Option E is correct, reinforcing that the disease always starts distally and spreads proximally. 31. In which arterial locations is clinically significant atherosclerosis most commonly encountered? A. Suprarenal abdominal aorta B. Infrarenal abdominal aorta and iliac arteries C. Branching points such as coronary artery bifurcations D. Upper-extremity arteries E. Intracranial arteries Explanation: (by Perplexity AI, generated with the OpenAI O3 model) • Turbulent flow and low shear stress at arterial bifurcations promote endothelial injury and lipid deposition, making branching points—particularly the coronary, carotid and aorto-iliac bifurcations—the prime sites for clinically significant atherosclerosis. • The infrarenal abdominal aorta can be involved but less frequently than branch points.
  • 388.
    EBDR Exam MCQs& Concepts May 2021 Dr. Kareem Alnakeeb 382 • The suprarenal aorta and upper-extremity arteries are relatively spared because of higher laminar flow and distinct wall composition. • Intracranial arteries are affected predominantly by different vasculopathies (e.g. small-vessel lipohyalinosis) rather than classic atherosclerosis. 32. According to the classic pathological classification, how many histological types of fibromuscular dysplasia are recognised? A. 8 B. 6 C. 3 D. 4 E. 5 Explanation: (by Perplexity AI, generated with the OpenAI O3 model) • Fibromuscular dysplasia (FMD) is historically divided by the arterial wall layer affected into three distinct histological sub-types: intimal (focal), medial (multifocal) and adventitial (perimedial) fibroplasia. • All share the hallmark non-atherosclerotic, non-inflammatory arterial wall thickening but differ in location of fibroplasia, imaging appearance and clinical behaviour. • Options suggesting more than three types are incorrect; although several angiographic patterns (e.g. “string-of-beads”, focal stenosis) are described, they still map back to only these three underlying pathological categories. 33. Which statement regarding carotico-cavernous fistulae is correct? A. They are most commonly caused by rupture of a cavernous internal carotid artery aneurysm B. Fibromuscular dysplasia is the usual underlying aetiology C. An indirect fistula is best embolised via a trans-arterial approach D. High-flow fistulae fed by multiple arterial branches are usually treated through a trans-venous route E. All types present with a classic “tram-track” sign on MR imaging Explanation: (by Perplexity AI, generated with the OpenAI O3 model) • High-flow direct fistulae with multiple arterial feeders often have tortuous arterial access, so microcatheters are advanced through the inferior petrosal sinus or superior ophthalmic vein for coil or liquid embolisation, making the trans-venous route the preferred approach. • Direct carotico-cavernous fistulae are classically due to traumatic rent in the cavernous ICA, not rupture of an aneurysm (A wrong). • Fibromuscular dysplasia relates to renal and carotid stenoses, not cavernous fistula formation (B wrong). • Indirect (dural) fistulae are low-flow shunts from meningeal branches and are most successfully occluded via a trans-venous route rather than arterial (C wrong). • The “tram-track” sign describes optic nerve sheath meningioma, not carotico-cavernous fistulae (E wrong).
  • 389.
    EBDR Exam MCQs& Concepts May 2021 Dr. Kareem Alnakeeb 383 34. Which imaging feature is NOT typically associated with a cerebral cavernous angioma (“cavernoma”) on MRI? A. Prominent mass effect caused by surrounding vasogenic edema B. Complete low-signal rim produced by hemosiderin deposition C. “Popcorn” appearance from blood products in various stages D. Lack of significant contrast enhancement E. Detection is best on susceptibility-weighted or GRE sequences Explanation: (by Perplexity AI, generated with the OpenAI O3 model) • Cavernous angiomas are low-flow vascular malformations whose MRI hallmark is a heterogeneous “popcorn” core of mixed-stage hemorrhage encircled by a complete hypointense hemosiderin rim; gradient-echo and susceptibility-weighted imaging accentuate this rim, making these sequences most sensitive (options B, C and E are true). • They usually show little or no contrast enhancement, further distinguishing them from neoplasms (option D is true). • In the absence of an acute, large hemorrhage, cavernomas exert minimal mass effect or surrounding vasogenic edema; therefore prominent edema-related mass effect is uncharacteristic, making option A the false statement. 35. In a patient with a suspected cerebral arteriovenous malformation (AVM), which imaging feature is most characteristic and therefore most helpful in making the diagnosis? A. Catheter angiography is superior because it always identifies the nidus B. A serpiginous “bag-of-worms” appearance is typically demonstrated on contrast-enhanced CT C. MRI is the most reliable technique for detecting the nidus in all cases D. Focal arterial stenosis immediately proximal to the nidus is virtually diagnostic E. CT perfusion invariably shows increased cerebral blood flow in the lesion Explanation: (by Perplexity AI, generated with the OpenAI O3 model) • Cerebral AVMs comprise a tangle of abnormal vessels (“nidus”) producing multiple tortuous, serpiginous channels that enhance intensely with contrast, giving the classic “bag-of-worms” appearance on CT or MR angiography. • This striking vascular pattern (Option B) is often the first clue on cross-sectional imaging. • While digital subtraction angiography (DSA) remains the gold standard for detailed angio- architecture, it can occasionally miss very small niduses and is invasive, so it is not “always” superior (Option A wrong). • High-resolution MRI can depict flow voids but may fail to show the entire nidus in fast-flow lesions (Option C overstates accuracy). • Proximal arterial stenosis is not a specific feature and may be absent (Option D wrong). • CT perfusion may demonstrate high flow but this is neither universal nor specific (Option E wrong). 36. Which of the following conditions is LEAST likely to cause long-segment narrowing of the pulmonary artery? A. Takayasu arteritis B. Behçet disease C. Idiopathic pulmonary fibrosis D. Primary pulmonary artery sarcoma E. Tuberculous mediastinitis
  • 390.
    EBDR Exam MCQs& Concepts May 2021 Dr. Kareem Alnakeeb 384 Explanation: (by Perplexity AI, generated with the OpenAI O3 model) • Takayasu arteritis and Behçet disease are large-vessel vasculitides that can produce concentric, long-segment stenoses of the main and branch pulmonary arteries. • Primary pulmonary artery sarcoma typically manifests as an intraluminal mass but may also lead to extensive segmental narrowing as it infiltrates longitudinally. • Chronic fibrosing mediastinitis due to tuberculosis can encase and constrict the pulmonary arteries over several centimetres. • In contrast, idiopathic pulmonary fibrosis primarily affects lung parenchyma with peripheral, basilar interstitial fibrosis; it does not directly involve or narrow the pulmonary arterial lumen, so long- segment pulmonary artery stenosis is not characteristic. 37. On contrast-enhanced MRI of an asymptomatic adult, physiological enhancement of the facial nerve is most consistently seen in which intratemporal segment? A. Cisternal (pontine) segment B. Parotid (extratemporal) segment C. Tympanic (horizontal) segment D. Geniculate ganglion E. Internal auditory canal (canalicular) segment Explanation: (by Perplexity AI, generated with the OpenAI O3 model) • The facial nerve possesses a rich circumneural arteriovenous plexus. Normal gadolinium uptake is therefore common within the facial canal, but the geniculate ganglion enhances in almost every healthy individual, reported in 96–100% of nerves on modern 3D T1-weighted sequences. • By contrast, the tympanic and mastoid segments enhance less frequently, while enhancement of the cisternal, parotid and canalicular segments is usually absent or only mild and inconsistent. • Hence, symmetrical bright post-contrast signal at the geniculate ganglion is regarded as physiological; marked enhancement elsewhere should prompt a search for pathology. 38. A 35-year-old woman presents to the breast clinic with unilateral, non-bloody nipple discharge and normal mammography. What is the most common benign pathology responsible for this presentation? A. Ductal carcinoma in situ B. Intraductal papilloma C. Invasive ductal carcinoma D. Mammary duct ectasia E. Fibroadenoma Explanation: (by Perplexity AI, generated with the OpenAI O3 model) • The leading cause of spontaneous, unilateral, serous or serosanguinous nipple discharge in women with otherwise normal imaging is a solitary intraductal papilloma. • Papillomas arise within a large subareolar duct and frequently cause intermittent discharge when they bleed or obstruct the duct. • Ductal carcinoma in situ can also present with discharge but is far less common in this clinical context; its hallmark on imaging is microcalcification rather than an isolated discharge episode. • Invasive ductal carcinoma is a rarer cause and would typically present with a mass. • Mammary duct ectasia usually produces bilateral, multicoloured discharge in perimenopausal women, not a single-duct, serous pattern. • Fibroadenoma does not cause nipple discharge.
  • 391.
    EBDR Exam MCQs& Concepts May 2021 Dr. Kareem Alnakeeb 385 39. Which of the following best explains the diffuse smooth pachymeningeal enhancement seen on post- contrast brain MRI in a patient with spontaneous intracranial hypotension? A. Venous engorgement secondary to low CSF pressure B. Congenital dural developmental anomaly C. Post-operative change after lumbar puncture D. Leptomeningeal carcinomatosis E. Hypertrophic pachymeningitis Explanation: (by Perplexity AI, generated with the OpenAI O3 model) • Intracranial hypotension lowers CSF volume, producing compensatory dilation of dural venous sinuses and epidural veins. • The resulting venous engorgement increases dural blood volume, leading to the characteristic smooth, diffuse pachymeningeal gadolinium enhancement. • Congenital dural anomalies are unrelated to acquired low-pressure states, postoperative lumbar puncture changes are typically focal and resolve quickly, leptomeningeal carcinomatosis enhances the pia–arachnoid rather than the dura, and hypertrophic pachymeningitis causes nodular or irregular dural thickening, not the smooth enhancement pattern of hypotension. 40. In thyroid eye disease, which extra-ocular muscle is most frequently affected leading to restricted eye movement and diplopia? A. Inferior rectus B. Lateral rectus C. Medial rectus D. Superior rectus E. Superior oblique Explanation: (by Perplexity AI, generated with the OpenAI O3 model) • The inferior rectus is most commonly involved in thyroid eye disease because its tight compartmental space within the orbit and rich glycosaminoglycan-producing fibroblasts make it particularly susceptible to autoimmune-mediated inflammation and fibrosis. • This results in limited up-gaze and vertical diplopia. The medial rectus is the second most frequently affected but less common than the inferior rectus. • The superior rectus, lateral rectus and superior oblique are involved far less often, so choosing them would fail to reflect the typical pattern of extra-ocular muscle involvement seen on CT or MRI. 41. A 47-year-old man presents with chronic burning low-back pain and bilateral leg paraesthesia 18 months after a lumbar discectomy; MRI shows clumped cauda equina roots within an “empty” thecal sac. What is the single most common precipitating cause of spinal arachnoiditis in current clinical practice? A. Intrathecal oil-based contrast (myelography) B. Tuberculous meningitis C. Lumbar spine surgery D. Epidural steroid injection E. Subarachnoid haemorrhage Explanation: (by Perplexity AI, generated with the OpenAI O3 model) • The leading trigger for adhesive arachnoiditis today is mechanical irritation from previous lumbar spine operations; scar tissue and inflammatory exudate develop after surgical manipulation, producing the characteristic root clumping and chronic neuropathic pain seen on imaging.
  • 392.
    EBDR Exam MCQs& Concepts May 2021 Dr. Kareem Alnakeeb 386 • Historical causes such as oil-based myelographic contrast (A) have become rare since water-soluble agents replaced them. • Infective meningitis (B) and subarachnoid haemorrhage (E) are recognised aetiologies but collectively account for fewer cases than postoperative change. • Chemical irritation from epidural steroids (D) is a documented but less frequent iatrogenic cause. 42. A 40-year-old non-smoker presents with recurrent haemoptysis. CT thorax shows a 2.5 cm vividly enhancing endobronchial mass in the right main bronchus, situated 2.5 cm distal to the carina, containing punctate calcification. What is the most likely diagnosis? A. Inflammatory polyp B. Endobronchial lipoma C. Endobronchial sarcoid granuloma D. Typical bronchial carcinoid E. Squamous cell carcinoma Explanation: (by Perplexity AI, generated with the OpenAI O3 model) • Typical bronchial carcinoids arise centrally, often within a main bronchus, show intense homogeneous contrast enhancement due to vascularity, and frequently contain speckled or “popcorn” calcification—features that match this case. • Inflammatory polyps are usually small, minimally enhancing soft-tissue nodules without marked vascularity. • Endobronchial lipomas demonstrate fat attenuation, not vivid enhancement. • Sarcoid granulomas are rare endobronchial lesions and typically non-enhancing with associated perilymphatic nodules elsewhere. • Squamous cell carcinomas may cavitate or invade bronchial wall but enhance less avidly and calcify only if post-therapeutic or dystrophic. 43. A 55-year-old woman with a 20-year history of insulin-dependent diabetes and hypertension presents with multiple firm, painless masses in both upper outer breast quadrants. Mammography shows symmetrically dense glandular tissue without discrete spiculated lesions. Targeted ultrasound demonstrates bilateral, irregular, markedly hypoechoic masses with pronounced posterior acoustic shadowing. MRI reveals slow, minimal contrast enhancement of these lesions. What is the most likely diagnosis? A. Lobular carcinoma B. Invasive ductal carcinoma C. Stromal fibrosis D. Diabetic mastopathy E. Fat necrosis Explanation: (by Perplexity AI, generated with the OpenAI O3 model) • Diabetic mastopathy classically affects long-standing insulin-dependent diabetics, producing multiple bilateral, ill-defined, hypoechoic breast masses with a striking posterior shadow and little or no enhancement on MRI—features that match this case. • Malignancies such as lobular (A) or invasive ductal carcinoma (B) usually show unilateral spiculated or irregular masses with early, avid enhancement. • Stromal fibrosis (C) can mimic malignancy but usually presents as a single lesion and is not strongly linked to diabetes.
  • 393.
    EBDR Exam MCQs& Concepts May 2021 Dr. Kareem Alnakeeb 387 • Fat necrosis (E) typically follows trauma or surgery and often shows oil cysts or calcifications, which are absent here. 44. In MRI assessment of a suspected middle-ear cholesteatoma, which imaging pattern most reliably confirms the diagnosis? A. Central enhancement with restricted diffusion B. Central enhancement with no restricted diffusion C. No central enhancement with no restricted diffusion D. No central enhancement with restricted diffusion E. Peripheral enhancement with T1 hyperintensity Explanation: (by Perplexity AI, generated with the OpenAI O3 model) • Cholesteatoma consists of keratin debris within a sac; on MRI it is typically non-vascular, so it shows no post-gadolinium enhancement. • The keratin content causes markedly restricted diffusion, giving high signal on DWI and low ADC values, making “no central enhancement with restricted diffusion” the hallmark appearance. • An enhancing lesion (Options A & B) suggests granulation tissue or tumour. • Lack of both enhancement and diffusion restriction (Option C) fits simple fluid or serous otitis. • Peripheral enhancement with T1 hyperintensity (Option E) is more consistent with cholesterol granuloma or haemorrhagic fluid. 45. A 39-year-old man with a 10-year history of heavy smoking presents with ischaemic changes in his left hand; catheter angiography demonstrates long-segment occlusion of the infra-popliteal tibial arteries on the right. What is the most likely underlying diagnosis? A. Diabetes mellitus B. Buerger disease (thromboangiitis obliterans) C. Ehlers-Danlos syndrome D. Systemic sclerosis (scleroderma) E. Takayasu arteritis Explanation: (by Perplexity AI, generated with the OpenAI O3 model) • Buerger disease classically affects male smokers younger than 45 years and causes segmental, non- atherosclerotic occlusion of medium- and small-calibre arteries in the extremities, leading to distal limb ischaemia visible on angiography as long, smooth tibial occlusions. • Diabetes mellitus more often produces tibial occlusive disease but typically in older patients and is usually accompanied by microvascular changes elsewhere. • Ehlers-Danlos syndrome causes arterial fragility and aneurysms rather than segmental occlusion. • Systemic sclerosis results in digital ischemia from vasospasm and intimal proliferation but does not usually create long tibial artery occlusions. • Takayasu arteritis targets the aorta and its major branches in young women, not the distal tibial vessels.
  • 394.
    EBDR Exam MCQs& Concepts May 2021 Dr. Kareem Alnakeeb 388 46. A 60-year-old heavy smoker presents with digital clubbing, hypertrophic osteoarthropathy, hypercalcaemia and cerebellar ataxia; CT shows a central hilar lung mass. What is the most likely underlying lung cancer? A. Adenocarcinoma B. Small cell carcinoma C. Large cell carcinoma D. Squamous cell carcinoma E. Bronchioloalveolar carcinoma Explanation: (by Perplexity AI, generated with the OpenAI O3 model) • Squamous cell carcinoma is typically central/hilar, strongly linked to long-term smoking and is the lung cancer subtype most often associated with paraneoplastic hypercalcaemia (via ectopic PTH-rP) and skeletal manifestations such as hypertrophic osteoarthropathy and digital clubbing. • Small cell carcinoma is